12.01.2015 Views

Trigonometry Solutions Manual

Trigonometry Solutions Manual

Trigonometry Solutions Manual

SHOW MORE
SHOW LESS

You also want an ePaper? Increase the reach of your titles

YUMPU automatically turns print PDFs into web optimized ePapers that Google loves.

INSTRUCTOR'S SOLUTIONS<br />

MANUAL<br />

to accompany<br />

<strong>Trigonometry</strong><br />

Seventh Edition<br />

Margaret L. Lial<br />

American River College<br />

John Homsby<br />

University ofNew Orleans<br />

David I. Schneider<br />

University ofMaryland<br />

EXCELSIOR EDUCATION CENTER<br />

7151 SVL BOX<br />

12217 SPRING VALLEY PARKWAY<br />

VICTORVILLE, CA 92392<br />

...<br />

A<br />

----<br />

Addison<br />

Wesley<br />

Boston San Francisc:o New York<br />

London Toronto Sydney Tokyo Singapore Madrid<br />

Mexico City Munich Paris Cape Town Hong Kong Montreal


Reproduced by Addison Wesley Longman from camera-ready copy supplied by the authors.<br />

Copyright © 2001 Addison Wesley Longman.<br />

All rights reserved. No part of this publication may be reproduced, stored in a retrieval system, or transmitted, in<br />

any form or by any means, electronic, mechanical, photocopying, recording, or otherwise, without the prior written<br />

permission of the publisher. Printed in the United States of America.<br />

ISBN 0-321-05761-9<br />

6 7 8 9 10 VO 03 02 01


CONTENTS<br />

1 THE TRIGONOMETRIC FUNCTIONS<br />

1.1 Basic Concepts 1<br />

1.2 Angles 9<br />

1.3 Angle Relationships and Similar Triangles 15<br />

1.4 Definitions of the Trigonometric Functions 21<br />

1.5 Using the Definitions of the Trigonometric Functions 27<br />

Chapter 1 Review Exercises<br />

33<br />

Chapter 1 Test Exercises<br />

41<br />

2 ACUTE ANGLES AND RIGHT TRIANGLES<br />

2.1 Trigonometric Functions of Acute Angles 43<br />

2.2 Trigonometric Functions of Non-Acute Angles 49<br />

2.3 Finding Trigonometric Function Values Using a Calculator 56<br />

2.4 Solving Right Angles 62<br />

2.5 Further Applications of Right Triangles 69<br />

Chapter 2 Review Exercises<br />

74<br />

Chapter 2 Test Exercises<br />

81<br />

3 RADIAN MEASURE AND THE CIRCULAR FUNCTIONS<br />

3.1 Radian Measure 84<br />

3.2 Applications of Radian Measure 88<br />

3.3 Circular Functions of Real Numbers 95<br />

3.4 Linear and Angular Velocity 1..03<br />

Chapter 3 Review Exercises<br />

107<br />

Chapter 3 Test Exercises<br />

112<br />

4 GRAPHS OF THE CIRCULAR FUNCTIONS<br />

- "<br />

4.1 Graphs of the Sine and Cosine Functions 114<br />

4.2 Translations of the Graphs of the Sine and Cosine Functions 121<br />

4.3· Graphs of the Other Circular Functions 129<br />

Chapter 4 Review Exercises<br />

137<br />

Chapter 4 Test Exercises<br />

142


5 TRIGONOMETRIC IDENTITIES<br />

5.1 Fundamental Identities 144<br />

5.2 Verifying Trigonometric Identities 153<br />

5.3 Sum and Difference Identities for Cosine 162<br />

5.4 Sum and Difference Identities for Sine and Tangent 170<br />

5.5 Double-Angle Identities 184<br />

5.6 Half-Angle Identities 195<br />

Chapter 5 Review Exercises<br />

205<br />

Chapter 5 Test Exercises<br />

218<br />

6 INVERSE TRIGONOMETRIC FUNCTIONS AND<br />

TRIGONOMETRIC EQUATIONS<br />

6.1 Inverse Trigonometric Functions 221<br />

6.2 Trigonometric Equations I 234<br />

6.3 Trigonometric Equations n 243<br />

6.4 Equations Involving Inverse Trigonometric Functions 249<br />

Chapter 6 Review Exercises<br />

257<br />

Chapter 6 Test Exercises<br />

266<br />

7 APPLICATIONS OF TRIGONOMETRY AND VECTORS<br />

7.1 Oblique Triangles and the Law of Sines 269<br />

7.2 The Ambiguous Case of the Law of Sines 277<br />

7.3 The Law of Cosines 282<br />

7.4 Vectors 291<br />

7.5 Applications of Vectors 300<br />

Chapter 7 Review Exercises<br />

307<br />

Chapter 7 Test<br />

315<br />

8 COMPLEX NUMBERS, POLAR EQUATIONS, AND<br />

PARAMETRIC EQUATIONS<br />

8.1 Complex Numbers 318<br />

8.2 Trigonometric (Polar) Form of Complex Numbers 323<br />

8.3 The Product and QuotientTheorems 328<br />

8.4 Powers and Roots of Complex Numbers 333<br />

8.5 Polar Equations 343<br />

8.6 Parametric Equations 356<br />

Chapter 8 Review Exercises<br />

364<br />

Chapter 8 Test Exercises<br />

372


9 EXPONENTIAL AND LOGARITHMIC FUNCTIONS<br />

9.1 Exponential Functions 376<br />

9.2 Logarithmic Functions 385<br />

9.3 Evaluating Logarithms and the Change-of-Base Theorem 392<br />

9.4 Exponential and Logarithmic Equations 399<br />

Chapter 9 Review Exercises<br />

405<br />

Chapter 9 Test Exercises<br />

411


<strong>Trigonometry</strong> <strong>Solutions</strong> <strong>Manual</strong><br />

1.1 Basic Concepts<br />

1<br />

CHAPTER 1 THE TRIGONOMETRIC<br />

FUNCTIONS<br />

Section 1.1<br />

Connections (page 5)<br />

2. To graph (-7, 6), go seven units from the origin to<br />

the left along the x-axis, and then go six units up<br />

parallel to the y-axis. The point (-7, 6) is in<br />

quadrant II since the x-coordinate is negative and<br />

the y-coordinate is positive.<br />

y<br />

8+(-9) -1 1<br />

1. x= =-=-­<br />

2 2 2<br />

-4+6 2<br />

y=--=-=I<br />

2 2<br />

Midpoint: (_..!., I)<br />

2 x<br />

2. endpoint: (2, -8)<br />

midpoint: (-1,-3)<br />

other endpoint: (x2'Y2)<br />

Exercises<br />

-I =2+x2<br />

2<br />

-2 = 2+x 2<br />

3. To graph (-7, -4), go seven units from the origin<br />

-4=x2<br />

to the left along the x-axis , and then go four units<br />

-3= -8+ Y2 down parallel to the y-axis. The point (-7 , -4) is<br />

2 in quadrant III since the .r- and y-coordinates are<br />

-6=-8+ Y2 both negative.<br />

2= Y2<br />

y<br />

(xi' Y2) =(-4,2)<br />

1. To graph (3, 2), go three units from the origin to<br />

the right along the x-axis, and then go two units<br />

up parallel to the y-axis. The point (3, 2) is in<br />

quadrant I since the x- and y-coordinates are both<br />

positive.<br />

y<br />

If i ' rrr: i<br />

- rn'-r'-'---"'-U<br />

. ;t +-1--Hi<br />

I-I :- , :: :; , i ! i I i !<br />

r-,_+ ' !; , : ' I<br />

rt"p-h -+t · ··r- + t- +<br />

, , ,! ~ ; - t-"r l--+- i" '<br />

I i I ! I i i : : -l + !--+-H-!-LU...­<br />

I ! T r j i<br />

J : ;-iJ ;; ~ ! -t-rtq. i - ' . 2).TH-i<br />

, j I 1<br />

U+l· l-- q -q -~ -<br />

! I ! I I ! • ! ; I -f--r r r f-r-jt--t"!<br />

till'H-t :t Q TTT i T n I r!<br />

~ _ ' i i ' ~ 1 , I • I I<br />

j I I I : I<br />

! ;<br />

I I ; ;<br />

, I I<br />

,<br />

! . !<br />

I , ! I I H=1<br />

: I Ii i I .<br />

1<br />

f-H : I I<br />

I . i i I<br />

4<br />

H-+ t·t-+ t--t-t rr: : r+H:i j<br />

I<br />

i t ! , I ! !<br />

l<br />

I<br />

l i l<br />

I I i I<br />

·<br />

fIlIi i i I<br />

l i : j<br />

· : :<br />

IV<br />

, ; i I · I u.LYJ<br />

x<br />

x<br />

1


<strong>Trigonometry</strong> <strong>Solutions</strong> <strong>Manual</strong><br />

2 Chapter 1 The Trigonometric Functions<br />

4. To graph (8, -5), go eight units from the origin to<br />

the right along the x-axis, and then go five units<br />

down parallel to the y-axis. The point (8, - 5) is in<br />

quadrant IV since the x-coordinate is positive and<br />

the y-coordinate is negative .<br />

y<br />

6. To graph (-8,0), go eight units from the origin to<br />

the left along the x-axis, Do not move up or down<br />

parallel to the y-axis since the y-coordinate is O.<br />

The point (-8, 0) is not in any quadrant; it lies on<br />

the x-axis.<br />

y<br />

x<br />

x<br />

5. To graph (0, 5), do not move along the x-axis at<br />

all since the x-coordinate is O. Move five units up<br />

along the y-axis. The point (0, 5) is not in any<br />

quadrant since it is on the y-axis.<br />

y<br />

7. To graph (4.5, 7), go four and one half units from<br />

the origin to the right along the x-axis, then go<br />

seven units up parallel to the y-axis. The point<br />

(4.5,7) is in quadrant I since the x- and<br />

y-coordinates are both positive.<br />

y<br />

x<br />

2


<strong>Trigonometry</strong> <strong>Solutions</strong> <strong>Manual</strong><br />

1.1 Basic Concepts<br />

3<br />

8. To graph (-7.5,8), go seven and one-half units<br />

from the origin to the left along the x-axis , and<br />

then go eight units up paralIel to the y-axis. The<br />

point (-7.5,8) is in quadrant II since the<br />

x-coordinate is negative and the y-coordinate is<br />

positive.<br />

y<br />

~ '~- il-~ --'-. - r<br />

IJ~ :(-7 .U) ! ;:<br />

t+: ~t- t-rH+' #~f~~[ i~J~<br />

_c-+-+_+_<br />

~+H+ ~<br />

: ;+-! ! : :<br />

=tJ-+-, '<br />

=I=f _1 1 1 ' _<br />

1<br />

I . 1-+ -1- '-+ -i- '"<br />

1 ' I l l ! ! r~)-f 1-' ! i i-I<br />

Iii ' iii ,<br />

! ; l i i x<br />

' ~<br />

..<br />

I<br />

filiI ! :: : i<br />

~- +f­<br />

- ·-r , imffi<br />

-' ! , i<br />

! J ~<br />

,<br />

i I I<br />

;<br />

!· L .l_ L __ ....!.-J<br />

.._.rrrrrtr _<br />

L .L.<br />

-W--+IVJ<br />

9. (-5, n) lies in quadrant II since the x-coordinate is<br />

negative ad the y-coordinate (n= 3.14) is positive.<br />

10. (rc, -3) lies in quadrant IV since the x-coordinate<br />

(n= 3.14) is positive and the y-coordinate is<br />

negative.<br />

11. (-J2, - 2.fi) lies in quadrant III since the x- and<br />

y-coordinates are both negative.<br />

12. ( 1+../3, ~) lies in quadrant I since the x - and<br />

y-coordinates are both positive.<br />

15. Since xy =1, and 1 is a positive number attained<br />

by the product of two variables x and y, then<br />

either x and yare both positive (quadrant I), or .<br />

both negative (quadrant III). The graph of xy =1<br />

will lie in quadrants I and III.<br />

16. (a) Since (a, b) lies in quadrant II, a < 0, b > O.<br />

Therefore -a > 0 and (-a, b) would lie in<br />

quadrant I.<br />

(b) Since (a, b) lies in quadrant II, a < 0,<br />

b > O. Therefore-a > 0 and -b < 0, and<br />

(-a, -b) would lie in quadrant IV.<br />

(c) Since (a, b) lies in quadrant Il, a < 0, b » O.<br />

Therefore -b < 0, and (a, -b) would lie in<br />

quadrant III.<br />

In Exercises 17-24, the formula<br />

d =~(X2 - Xl)2 +(Y2 - Yl )2 is used to find the distance<br />

between (xl'Yl) and (x2, Y2)'<br />

17. (xl'YI) =(2, -1) and (X2'Y2) = (-3, -4)<br />

d= ~(-3 - 2)2 + [-4 _(_1)]2<br />

=~(_5)2+(_3)2<br />

=·J25+9<br />

=..[34<br />

18. (xl' Yl) =(-5,2) and (x2' Y2) = (3, -7)<br />

~[3 - (_5)]2 + (-7 - 2)2<br />

= ~82 +(_9)2<br />

= ..)64+ 81<br />

= ..)145<br />

19. (Xj, Yj) = (-1,0) and (x2' Y2) = (-4, - 5)<br />

d =~[-4-(-l)f +(-5-0)2<br />

= ~(_3)2 + (_5)2<br />

= ..)9+25<br />

=..[34<br />

20. (xl, Yl) = (-2, -3) and (x2' Y2) = (-6, 4)<br />

d= ~[-6 _(_2)]2 + [4 - (_3)]2<br />

=~(-4)2+72<br />

= ..)16+49<br />

=../65<br />

21. (xl' Yl) = (.fi, -.J5) and (x2, Y2) = (3.fi, 4..)5)<br />

d= ~(3.fi _.fi)2 +[4..)5 -(-.J5)t<br />

= ~(2.fi)2 + (5..)Sf<br />

=..)8+125<br />

= ..)133<br />

22. (Xl' Yl) = (5.,fi, -.J3) and<br />

(x2' Y2) = (-.,fi, 8.J3)<br />

d= ~(-J7- 5.,fif + [8.J3 - (-.J3)t<br />

=~(-6.,fi)2 + (9.J3)2<br />

= ..)252+243<br />

= ..)495<br />

=.J9 .../55<br />

= 3../55<br />

3


<strong>Trigonometry</strong> <strong>Solutions</strong> <strong>Manual</strong><br />

4 Chapter 1 The Trigonometric Functions<br />

23. Let (xI' YI) = (5, - 6). Let (x2' Y2) = (5, 0) since 31. (a, b, e) = (5, 10, 15)<br />

we want the point on the x-axis with x-value 5.<br />

a 2 + b 2 = 52 + 10 2<br />

d = ~(5 _5)2 +[0 _(-6)]2<br />

=25+100<br />

= ~02 +6 2<br />

=125<br />

:;t 15<br />

=J36<br />

2<br />

2<br />

=6<br />

24. Let (xI' YI)= (5, -6). Let (x2' Y2) = (0, -6) since<br />

we want the point on the y-axis with y-value-6.<br />

32. (a, b, e)= (7, 24, 25)<br />

d =~(O - 5)2 + [-6 - (-6)f<br />

a 2 + b 2 =7 2 + 24 2<br />

=~(_5)2 +0<br />

= 49+576<br />

=..fi5<br />

=625<br />

=5<br />

25. In the distance formula, we find the square root of<br />

a 2 + b 2 =c 2 .<br />

(x2 _ XI)2 +(Y2 - Yl)2. From the display,<br />

x2 = -2, xI = 3, Y2 = 1. and Yl = -n. Therefore,<br />

the two points (xI' YI) and (x2' Y2) are (3, -n)<br />

and (-2, 1).<br />

26. Since the point (a, b) lies in quadrant I, that is, has<br />

positive x- and y-coordinates, the point (-a, -b)<br />

must have negative x- and y-coordinates and will<br />

lie in quadrant III. Since (a, b) is located 6 units<br />

from the origin, (-a. -b) will also be located<br />

6 units from the origin (but in the opposite<br />

direction).<br />

a = ~[1-(-2)]2 +(5 _5)2<br />

29. (a, b, e) =(9, 12, 15)<br />

a 2 + b 2 =9 2 + 12 2<br />

= ~32 +0 2<br />

=81+144<br />

=../9<br />

=225<br />

=3<br />

=15 2<br />

b = ~r-(l-_-1)-'-2-+-(9---5)-2<br />

2<br />

=e<br />

=~02 +4 2<br />

This triple is a Pythagorean triple since<br />

a 2 +b 2 = e 2.<br />

=.J16<br />

=4<br />

30. (a, b, e) = (6, 8, 10)<br />

e =~r-[1-_-(_-2-)]::-2+-(-9---5)-=-2<br />

a 2 +b 2 =6 2 +8 2<br />

=36+64<br />

=~32+4 2<br />

=100<br />

=10 2 =e 2<br />

=.J9+16<br />

This triple is a Pythagorean triple since<br />

=..fi5<br />

a 2 +b 2 = e 2.<br />

=e<br />

This triple is not a Pythagorean triple since<br />

a 2 +b 2 :;te 2.<br />

=25 2 = c 2<br />

This triple is a Pythagorean triple since<br />

33. If, for positive integers a, b, and c, a 2 + b 2 = e 2,<br />

then it is not necessarily true that a + b = e. For<br />

example, let a = 3, b = 4, and e = 5.<br />

Then a 2 = 9. b 2 = 16, and e 2 = 25.<br />

a 2 + b 2 =9+16=25=e 2<br />

Note that a + b = 3 + 4 = 7 :;t 5 = e .<br />

35. Are (-2, 5), (1, 5), and (1, 9) the vertices of a right<br />

triangle Use the distance formula to find the<br />

lengths of the sides connecting the points.<br />

=5<br />

a 2+b2 =3 2 +4 2 =9+16=25=e 2<br />

Yes, the given points are vertices of a right<br />

triangle.<br />

4


<strong>Trigonometry</strong> <strong>Solutions</strong> <strong>Manual</strong><br />

1.1 Basic Concepts<br />

5<br />

36. Are (-9. -2). (-I, -2). and (-9. 11) the vertices of<br />

38. (4-.../3.-2.J3}(2-.J3.-./3), (3-.J3.-2.../3)<br />

a right triangle<br />

a 2 =[-9-(-1)]2 +[-2-(-2)]2<br />

a<br />

2<br />

= [(4-.../3)-(2-.../3)t + [-2.J3 -(-./3)t<br />

= (_8)2 + 0 2 = 64<br />

b 2 =[-9-(-)] 9 2 +(-2-11) 2<br />

=2 2+(-.../3)2 =7<br />

= 0 2 + (-13)1 = 169 b 2 = [(4-.J3)-(3-.../3)Y +[-2.../3 -(-2../3)y<br />

c 2 =[-1-(-9)]2+(-2-11)2<br />

= 1 2 +0 2 =I<br />

= 8 2 + (_13)2 = 233<br />

2 c = [(2-.J3)-(3-.../3)Y +[-./3 -(-2.../3)t<br />

64 + 169 = 233, so a 2 + b 2 2<br />

= c .<br />

Yes. the given points are vertices of a right = (_1)2 + (../3)2 = 4<br />

triangle.<br />

Since no two of a 2. b 2 or c 2 add up to the third.<br />

37. (.../3.2.../3+3). (.../3+4. -.../3+3} the given points are not vertices of a right<br />

triangle.<br />

(2.../3. 2.../3+ 4)<br />

39. Find x such that the distance between (x, 7) and<br />

a= ~(.../3 +4 _.../3)2 +[-.../3 +3-(2.../3 +3)t (2. 3) is 5. Use the distance formula. and solve for<br />

x.<br />

=~42+(_M)2<br />

5 = ~(2-x)2 +(3-7)2<br />

=../16+27<br />

= ~(2 - x)2 + (-4)2<br />

=~ =~(2-x)2+16<br />

Square both sides .<br />

b = ~(2.../3 _.../3)2 +[2.../3 + 4 -(2.../3 +3)Y 25 =(2-x)2 +16<br />

= ~(.../3)2 + 1 2 9=(2-x)2<br />

Take the square root of both sides.<br />

=../3+1 3=2-x or -3=2-x<br />

x=-I x=5<br />

=-14<br />

=2 40. Find ysuch that the distance between (5.y) and<br />

c = ~r-[ 2.../3-3---( .../3-3+-4---=)t-+-[2.../3-3-+-4--(-./3-3-+-3)---=-t (8. -I) is 5.<br />

d=~(5-8)2+[y_(-l)f =5<br />

= ~(.../3 - 4)2 + (M+ If ~9+(y+I)2 =5<br />

= ~19 - s.J3 + 28 + 6.../3<br />

Square both sides.<br />

9+(y+I)2 =25<br />

=~47-2.../3<br />

(y+I)2=16<br />

y+1 =±4<br />

If y + I =4. then y =3.<br />

=43+4 If y + I = -4. then y = -5.<br />

2 Therefore. y = 3 or y = -5.<br />

a2 +b2 =(~)2 +22<br />

*,c<br />

or 47-2.../3<br />

No. the triangle is not a right triangle.<br />

5


<strong>Trigonometry</strong> <strong>Solutions</strong> <strong>Manual</strong><br />

6 Chapter 1 The Trigonometric Functions<br />

41. Find y such that the distance between (3, y) and<br />

(-2,9) is 12.<br />

12 = ~'(--2---3--::)2-+-(-9-_-y)"7""2<br />

= ~(_5)2 + (9 _ y)2<br />

= ~25+(9 _ y)2<br />

~<br />

U<br />

6<br />

y<br />

144 = 25+(9- y)2<br />

119=(9- y)2<br />

..jIT9 = 9 - y<br />

y=9-M<br />

or -M = 9 - y<br />

y=9+M<br />

42. Find x such that the distance between (x, 11) and<br />

(5, -4) is 17.<br />

d = ~(x - 5)2 + [11- (-4)]2 = 17<br />

x= 130rx =-3<br />

~(x-5)2+152 =17<br />

(x - 5)2 + 225 = 289<br />

(x _5)2 = 64<br />

x-5=±8<br />

43. Let P =(x, y) be a point 5 units from (0, 0). Then,<br />

by the distance formula.<br />

5 =~(x- 0)2 +(y- 0)2<br />

=~x2+i<br />

2<br />

25 = x 2 + y2 or x + i = 25<br />

This is the required equation. The graph is a circle<br />

with center (0, 0) and radius 5.<br />

y<br />

...y++-H:++++-H-::~ x<br />

-5 5<br />

-5<br />

44. Let P = (x, y) be a point 3 units from (-5, 6).<br />

Then, by the distance formula,<br />

d=~[x-(-5)]2 +(y_6)2 =3<br />

~(x+ 5)2 + (y _6)2 = 3<br />

Square both sides.<br />

(x + 5)2 + (y - 6)2 = 9<br />

This is the equation for all points 3 units from<br />

(-5,6).<br />

The graph is a circle with center at (-5, 6) and<br />

radius 3.<br />

- 5<br />

r= 3<br />

45. By the Pythagorean theorem,<br />

2<br />

x =(¥f +(55.4)2<br />

x = ~(l7 .25)2 + (55.4)2<br />

= ../297.5625 + 3069.16<br />

=58.02 ft<br />

46. Let d = the diagonal by the Pythagorean Theorem<br />

d 2 = 52 + 12 2<br />

d = ../25+ 144<br />

= ../169<br />

=13ft<br />

47. AC 2 + BC 2 =AB 2<br />

(1000)2 + BC 2 = (1000.5)2<br />

1,000,000+ BC 2 = 1,001,000.25<br />

2 .<br />

BC = 1,001,000.25 - 1,000,000<br />

= 1000.25<br />

BC = ../1000.25 := 31.6 ft<br />

48. To find the length AB, apply the Pythagorean<br />

theorem to triangle ABX.<br />

(AB)2 =(AX)2 +(BX)2<br />

=8 2 +8 2<br />

=128<br />

AB = ../128:= 11.3137<br />

The distance across the top of the fabric is<br />

20 . AB (since there are 20 strips) plus 42 in. for<br />

the side of the unused triangle at the end. (Since<br />

this is a 45°- 45° right triangle, it is isosceles and<br />

its two equal sides are each 42 in.) Thus, the<br />

distance across the top is<br />

20(11.3137) + 42 =226.274 + 42<br />

=268 .274 in.<br />

Since the price of the material is $10 per linear<br />

yard, convert from inches to yards to find the<br />

number of linear yards.<br />

268.274:= 7.452 d<br />

36 Y<br />

The cost of the material is 7.452(10) = $74.52 .<br />

6


<strong>Trigonometry</strong> <strong>Solutions</strong> <strong>Manual</strong><br />

1.1 Basic Concepts<br />

7<br />

49. Let b = the leg of the right triangle with leg hand<br />

hypotenuse a.<br />

b 2 +h 2 = a 2<br />

b 2 = a 2 _h 2<br />

=(184.7)2 -(144)2<br />

=13,378<br />

b =115.66<br />

The length I of a side is 2b, or<br />

2( 115.66) =231.3 m.<br />

50. Let x = the length of the leg of the right triangle<br />

with leg a and hypotenuse s. Notice that<br />

x =!... = 231.3 = 115.65.<br />

2 2<br />

a 2 +x 2 = s2<br />

(184.7)2 + (115.65)2 = s2<br />

34,114 .1+ 13,374.92 = s2<br />

47,489.02 = s2<br />

217.9 =s<br />

The length of the edge of the pyramid labeled s is<br />

about 217.9 m.<br />

51. By the Pythagorean theorem.<br />

BC 2 = AC 2 + AB 2<br />

AB=~BC2 -AC 2<br />

=<br />

r-( r<br />

( 42 42<br />

262 260<br />

=~(213l)2 -(2130)2<br />

= .J4,541,161- 4.536.900<br />

'" 65.28 feet<br />

52. {xix> 6}<br />

This is an open interval with interval notation<br />

(6,00).<br />

53. {pjp ~ 1O}<br />

This is a half-open interval with interval notation<br />

[10,00).<br />

54. {xl-3 < x < 7}<br />

This is an open interval with interval notation<br />

(-3,7).<br />

55. {y/8SyS13}<br />

This is a closed interval with interval notation<br />

[8, 13].<br />

56. The graph shows an open interval, starting at 2.<br />

and includes all real numbers greater than 2.<br />

The interval notation is (2, 00).<br />

57. The graph shows an open interval. The interval<br />

does not include 2, but consists of all real<br />

numbers less than 2, written in interval notation<br />

as (-00,2).<br />

58. The graph shows a half-open interval between<br />

1 and 4. It includes 4, but does not include 1.<br />

The interval notation is (1, 4].<br />

In Exercises 61-66, f(x) = -2x 2 + 4x + 6 .<br />

61. f(O) = -2(0)2 + 4(0) + 6<br />

=0+0+6<br />

=6<br />

62. f(-2) = -2(-2)2 + 4(-2) + 6<br />

=-8-8+6<br />

=-10<br />

63. f(-I) = -2(-1)2 + 4(-1) + 6<br />

=-2(1)+4(-1)+6<br />

=-2+(-4)+6<br />

=0<br />

64. f(-m) = -2(_m)2 + 4(-m) + 6<br />

=-2m 2 -4m+6<br />

65. f(1+a)=-2(1+a)2 +4(1+a)+6<br />

= -2(1 +2a + a 2)+ 4(1+a)+6<br />

=-2-4a -2a 2 +4+4a+6<br />

=-2a 2 +8<br />

66. f(2 - p)= -2(2- p)2 +4(2- p)+6<br />

= -2(4-4p+ p2)+8-4p+6<br />

=-8+8p-2p2+8-4p+6<br />

=_2p2 +4p+6<br />

67. (a) We want the value ofJ(or l) when X is 2.<br />

When X = 2 in the table, Y 1 = 1.2, so<br />

/(2) = 1.2.<br />

(b) Now we want the value of X for whichf(or<br />

l) equals -2.4. When f(x) = Y 1 = - 2.4,<br />

X = 5 in the table.<br />

(c) The graph of y = j(x) crosses the y-axis at the<br />

y-intercept (0, 3.6).<br />

(d) The graph of y = j(x) crosses the x-axis at the<br />

x-intercept (3, 0).<br />

7


<strong>Trigonometry</strong> <strong>Solutions</strong> <strong>Manual</strong><br />

8 Chapter 1 The Trigonometric Functions<br />

68. (a) When x = 0 on the graph, y = 11, so<br />

f(O) = 11.<br />

(b) When x = 6 on the graph, y = 9, so f(6) = 9<br />

(c) The value of y =f(a) is 0 when x = -2, so<br />

a=-2.<br />

(d) When x = 2 on the graph, f(x) =y =10.<br />

Also f(7) =10, and f(8) =10. So f(x) =10<br />

for x =2, 7, and 8.<br />

(e)<br />

(8,f(8» =(8, 10) =(xI' YI) and<br />

(10, f(1O» = (10, 1)= (x2' Y2)<br />

d = ~(1O - 8)2 + (1-10)2<br />

=~22 +(_9)2<br />

=.J4+81<br />

=~<br />

69. (a) P is only a function of a when the range is<br />

restricted to (0, 00). If the range is not<br />

restricted, then P = ±,J;;3 and is not a<br />

function.<br />

(b) p2 = (1.524)3<br />

(c)<br />

p 2 =:: 3.5396<br />

p =:: 1.88<br />

Since P is time in years, we only consider the<br />

principal square root in the answer. The<br />

orbital period of Mars is approximately<br />

1.88 years .<br />

3<br />

19.18 2 = a<br />

3<br />

367.8724 = a<br />

7.165 =:: a<br />

70. Y = 4x-3<br />

Domain: (-00,00).<br />

Range: (-00,00).<br />

Y = 4x - 3 is a function because each x-value<br />

gives exactly one y-value.<br />

71. 2x + 5y = 10<br />

Domain: (-00,00)<br />

Range : (-00, 00)<br />

This is a function because each value of x gives<br />

exactly one value ofy .<br />

72. y=x 2 +4<br />

Domain: (-00,00)<br />

Range: The smallest value of x 2 is 0, so the<br />

smallest value of y =x 2 +4 =0 + 4 =4 .<br />

So the range is [4,00).<br />

Y = x 2 + 4 is a function because each x-value<br />

gives exactly one y-value.<br />

73. y=2x 2 - 5<br />

Domain: (-00,00)<br />

2<br />

Range: Since x ~ 0, 2x 2 - 5 ~ -5, and the range<br />

is [-5,00).<br />

This is a function because each value of x gives<br />

exactly one value of y,<br />

74. y =-2(x - 3)2 + 4<br />

Domain: (-00,00)<br />

Range: Since<br />

(x - 3)2 ~ 0<br />

-2(x-3)2 s 0<br />

-2(x - 3)2 +4:S; 4.<br />

The range is (-00, 4].<br />

y =-2(x - 3)2 + 4 is a function because each<br />

x-value gives exactly one y-value.<br />

75. y =3(x+ 1)2_ 5<br />

Domain: (-00,00)<br />

Range: Since (x + 1)2 ~ 0, 2(x + 1)2 - 5 ~ -5.<br />

The range is [-5,00).<br />

This is a function because each value of x gives<br />

exactly one value of y .<br />

76. x = y2<br />

Domain: i ~ 0, so the domain is [0,00).<br />

Range: (-00,00)<br />

X = y2 is not a function because all x-values<br />

(except x = 0) give two y-values. For example, if<br />

x = 4, y = 2 or y = -2.<br />

77. y =.J4+x<br />

Domain: Because we can only take the square<br />

root of a nonnegative number,<br />

4+x~0<br />

x ~ -4<br />

the domain is [-4,00).<br />

Range: For all values of x ~ -4, .J4 + x ~ 0, so<br />

y ~ O. The range is [0, 00).<br />

y =.J4 + x is a function because each x-value<br />

gives exactly one y-value.<br />

8


<strong>Trigonometry</strong> <strong>Solutions</strong> <strong>Manual</strong><br />

1.2 Angles<br />

9<br />

78. y=~x2+1<br />

Domain: (-00,00)<br />

Range: For all x-values, x 2 2: 0, x 2 + 1 2: 1, and<br />

~x2 + 1 ~ 1. The range is [1,00). ·<br />

y =~ is a function because each x-value<br />

gives only one y-value.<br />

79. y=~1-x2<br />

2<br />

Domain: 1- x ~ 0<br />

_x 2 ~-1<br />

x 2 $1<br />

-1 s x $1<br />

The domain is [-1, 1).<br />

Range: [0, 1]<br />

This is a function because each x-value gives only<br />

one y-value.<br />

80. Domain: [-5,4]<br />

Range: [-2,6]<br />

This is a function because any vertical line<br />

intersects the graph in at most one point.<br />

81. Domain: [-5,00)<br />

Range: [0,00)<br />

This is a function because any vertical line<br />

intersects the graph in at most one point.<br />

82. Domain: (-oo,-3]u[3,00)<br />

Range: (-00, 00)<br />

This is not a function because some vertical lines,<br />

for example x = 4, intersect the graph in two<br />

points.<br />

83. Domain: [-4,4]<br />

Range: [-3,3]<br />

This is not a function because some vertical lines<br />

intersect the graph in two points, for example,<br />

x=O.<br />

1<br />

84. y=­ x<br />

The denominator cannot be zero, so x ~ 0<br />

Domain: (-00,0) U (0,00)<br />

-1<br />

86. y=-;===<br />

~x2 +25<br />

The denominator must not be 0 and the<br />

expression under the radical sign must be<br />

nonnegative.<br />

2<br />

Since x ~ 0 for all real numbers x,<br />

x 2 +25 ~25<br />

~x2 +25 ~5>0.<br />

Thus, the denominator is not 0 for any real<br />

number x and the expression x 2 + 25 is<br />

nonnegative for any x.<br />

The domain is (-00,00).<br />

87. From the figure, the length of a side of the large<br />

square is a + b, so its area is (a + b)2 or<br />

a 2 +2ab+b 2 .<br />

88. The area of each right triangle is (~fb, so the<br />

sum of the areas of the four right triangles is<br />

4(~Jab or 2ab. The smaller square has side of<br />

length c, so its area is C • cor c 2 .<br />

89. The sum of the areas of the four right triangles<br />

and the smaller square is 2ab + c 2 •<br />

90. Setting the expressions from exercises 87 and 89<br />

equal to each other we obtain<br />

a 2 + 2ab+b 2 =2ab +c 2.<br />

91. Simplifying the expression in exercise 90 results<br />

in the Pythagorean theorem' a 2 + b 2 =c 2 .<br />

Section 1.2<br />

Exercises<br />

2. 1° represents _1_ of a rotation.<br />

360<br />

Therefore, 45° represents<br />

45 1 fl '<br />

360 =8" 0 a comp ete rotation.<br />

-2<br />

85. y=-­<br />

x+1<br />

The denominator cannot be zero.<br />

x+1 ~O<br />

x~ -1<br />

Domain: (-00, -1) u (-1, 00)<br />

3. Let x =the measure of the angle.<br />

If the angle is its own complement.<br />

x+x=9O<br />

2x=90<br />

x=45<br />

A 45° angle is its own complement.<br />

9


<strong>Trigonometry</strong> <strong>Solutions</strong> <strong>Manual</strong><br />

10 Chapter 1 The Trigonometric Functions<br />

.<br />

o ·<br />

7m+3=73<br />

4. Let x = the measure of the angle.<br />

Ifthe angle is its own supplement,<br />

x+x = 180<br />

2x = 180<br />

x=90.<br />

A 90° angle is its own supplement<br />

25 minutes x degrees<br />

5. = ;x = 150 0<br />

60 minutes 360 degrees<br />

6. The hour hand is ~ of the way between the 1 and<br />

4<br />

2, and therefore is at 8.75 minutes past the 12.<br />

The minute hand is 15 minutes before the 12. The<br />

smaller angle formed by the hands of the clock is<br />

(15+ 8.75) minutes x<br />

--;x = 142.5°<br />

60 minutes 360°<br />

7. 7x+ 11x = 180<br />

18x = 180<br />

x=1O<br />

The measures of the two angles are<br />

(7x)0 = 7(10°) = 70° and (l lx)" = 1l(100) = 110°.<br />

8. The two angles form a right angle.<br />

4y+2y=90<br />

6y=90<br />

y=15<br />

The two angles have measures of 4(15°) = 60°<br />

and 2(15°) = 30°.<br />

9. (5k + 5) + (3k + 5) = 90<br />

8k+1O=9O<br />

8k=80<br />

k =10<br />

The measures of the two angles are<br />

(5k + 5)° = 5(10°) + 5° = 55° and<br />

(3k + 5)° = 3(10°) + 5° = 35°.<br />

10. The sum of the measures of two supplementary<br />

angles is 180°.<br />

1Om+7+ 7m+3 = 180<br />

17m+1O=180<br />

17m =170<br />

m=1O<br />

10m +7 = 107<br />

The angle measures are 107° and 73°.<br />

11. 6x -4+8x -12 = 180<br />

14x-16=180<br />

14x = 196<br />

x =14<br />

The measures of the two angles are<br />

(6x _4)° = 6(14°) _4° = 80° and<br />

(8x - 12)° = 8(14°) - 12° = 100°.<br />

12. The sum of the measures of two complementary<br />

angles is 90°.<br />

9z+6+3z=9O<br />

12z+6=9O<br />

12z = 84<br />

z=7<br />

9z + 6 = 9(7) + 6 = 69<br />

3z = 3(7) = 21<br />

The angle measures are 69° and 21°.<br />

13. Ifan angle measures x degrees, and two angles are<br />

complementary if their sum is 90°, then the<br />

complement of an angle of XO is<br />

(90-x)0.<br />

14. If an angle measures x degrees, and two angles are<br />

supplementary if their sum is 180°, then the<br />

supplement of an angle of XO is<br />

(180-x)0.<br />

15. The first negative angle cotenninal with x<br />

between 0° and 60° is (x - 360)° .<br />

16. The first positive angle coterminal with x between<br />

0° and --600 is (x + 360)°.<br />

17. 62° 18' + 21° 41' =83° 59'<br />

18. 75° 15' + 83° 32' = 158° 47'<br />

19. 71° 18' - 47°29' = 700 78' - 47°29'<br />

=23°49'<br />

20. 47°23' - 73° 48' = -(73° 48' - 47° 23')<br />

= -26°25'<br />

21. 90° -51°28' = 89°60' - 51°28' = 38° 32'<br />

22. 180° -124° 51' = 179°60' -124° 51'<br />

= 55°9'<br />

23. 90° - 72° 58'11" = 89° 59'60"-72°58'11"<br />

= 17°1' 49"<br />

24. 90°-36°18'47" = 89°59'60" -36°18' 47"<br />

= 53° 41' 13"<br />

25.<br />

20054' = 200+ 54°<br />

60<br />

=20°+.900°<br />

=20.900°<br />

26. 38°42' = 380+ 42°<br />

60<br />

=38.700°<br />

10


<strong>Trigonometry</strong> <strong>Solutions</strong> <strong>Manual</strong><br />

1.2 Angles 11<br />

0 0<br />

27. 91 035'54"=910+ 35 + 54<br />

60 3600<br />

= 91.598 0<br />

28.<br />

34051'35"=340+510 + 35°<br />

60 3600<br />

= 34° + .85° + .0097°<br />

= 34.860°<br />

29. 274018'59" = 2740+~+ 59°<br />

60 3600<br />

=274.316°<br />

30. 165051'9" =1650+ 51° +~<br />

60 3600<br />

= 165° + .85° + .0025°<br />

= 165.853°<br />

31. 31.4296° = 31° + (.4296)60'<br />

=31° + 25.776<br />

=31° + 25' + (.776)(60")<br />

= 31°25'47"<br />

32. 59.0854° =59° + .0854°<br />

= 59° + (.0854)(60')<br />

=59° + 5.124'<br />

= 59° 5' + (.124)(60")<br />

=59°5'7"<br />

33. 89.9004° = 89° + (.9004)(60')<br />

= 89° + 54.024'<br />

=89° + 54' +(.024)(60")<br />

=89°54'1"<br />

34. 102.3771°= 102° + (.3771)(60')<br />

= 102° + 22.626'<br />

= 120°22' + (.626)(60")<br />

= 102° 22' 38"<br />

35. 178.5994° = 178° + (.5994)(60')<br />

=178°+35.964'<br />

=178° + 35' + (.964)(60")<br />

= 178°35'58"<br />

36. 122.6853° = 122° + (.6853)(60')<br />

= 122° + 41.118'<br />

= 122° 41' + (.118)(60")<br />

= 122°41'7"<br />

39. -40° is coterminal with -40° + 360° = 320°.<br />

43. 539° is coterminal with 539 0 - 360° = 179°.<br />

44. 699° is coterminal with 699 0 - 360° = 339°.<br />

45. 850° is coterminal with<br />

850° - 2(360°) =850° - 720 0 =130°.<br />

46. 1000° is coterminal with<br />

1000° - 2 . 360 0 = 1000° - 720°<br />

=280°.<br />

47. 30°<br />

A coterminal angle can be obtained by adding an<br />

integer multiple of 360°.<br />

30° + n : 360°<br />

48. 45°<br />

A coterminal angle can be obtained by adding an<br />

integer multiple of 360°.<br />

45° + n . 360°<br />

49. 135°<br />

A coterminal angle can be obtained by adding an<br />

integer multiple of 360°.<br />

135° + n ' 360°<br />

50. 270°<br />

A coterminal angle can be obtained by adding an<br />

integer multiple of 360°.<br />

270° + n . 360 0<br />

51. -90°<br />

A coterminal angle can be obtained by adding an<br />

integer multiple of 360 0 .<br />

-90° + n . 360°<br />

52. -135°<br />

A coterminal angle can be obtained by adding<br />

integer multiple of 360°.<br />

-135 0 + n . 360°<br />

54. (a) 360° + r"is coterminal with r" because you<br />

are adding an integer multiple of 360° to r",<br />

r" + 1 . 360°.<br />

(b) r" - 360° is coterminal with r" because you<br />

are adding an integer multiple of 360° to r",<br />

r" + (-1) . 360° .<br />

(c) 360° - r" is not coterminal with r" because<br />

you are not adding an integer multiple of<br />

360 0 to r,<br />

360° - r" :t: r" + n . 360°<br />

41. -125° is coterminal with -125° + 360 0 = 235°.<br />

42. -203° is coterminal with 360° + (-203°) = 157°.<br />

11


<strong>Trigonometry</strong> <strong>Solutions</strong> <strong>Manual</strong><br />

12 Chapter 1 The Trigonometric Functions<br />

(d) ,0 + 180° is not cotenninal with ,0 because<br />

you are not adding an integer multiple of<br />

360° to ,0. You are adding .!. (360°).<br />

2<br />

Choices (c) and (d) are not cotenninal with ,0.<br />

55. lJ = 3~C:o)-in{3:0))<br />

X=~o<br />

YI = 36{(- 3:~)-in{- 3:~))<br />

=320°<br />

59.<br />

174 0 o<br />

y<br />

534°; -186°;<br />

quadran I II<br />

174° is cotenninal with 174° + 360° = 534° and<br />

174°- 360° = -186°. These angles are in<br />

quadrant II.<br />

56. lJ =31(3:0)- intC:o))<br />

X=-98°<br />

lJ =31(- :6~)-in{- :~))<br />

=26r<br />

60. A positive angle coterminal with 234° is<br />

234° + 360° = 594°.<br />

A negative angle coterminal with 234° is<br />

234° - 360° = -126°.<br />

There angles are in quadrant Ill.<br />

y<br />

For Exercises 57-64 angles other than those given are<br />

possible.<br />

57. A positive angle cotenninal with 75° is<br />

75° + 360° = 435°.<br />

A negative angle cotenninal with 75° is<br />

75° - 360° = -285°.<br />

These angles are in quadrant I.<br />

y<br />

594°; -126°;<br />

quadrant III<br />

61. A positive angle coterminal with 300° is<br />

300° + 360° = 660°.<br />

A negative angle coterminal with 300° is<br />

300° - 360° = -60°.<br />

These angles are in quadrant IV.<br />

y<br />

435°; -285°;<br />

quadrant I<br />

58.<br />

o<br />

89 0<br />

660°; -60°;<br />

quadrant IV<br />

449°; -271°;<br />

quadrant 1<br />

89° is cotenninal with 89° + 360° = 449° and<br />

89° - 360° = -271°.<br />

These angles are in quadrant I.<br />

12


<strong>Trigonometry</strong> <strong>Solutions</strong> <strong>Manual</strong><br />

1.2 Angles 13<br />

62.<br />

y<br />

65.<br />

y<br />

152°;-208°;<br />

quadrantII<br />

A positive angle cotenninal with 512° is<br />

512° - 360° = 152°.<br />

A negative angle cotenninal with 512° is<br />

512° - 2(360') = -208°.<br />

These angles are in quadrant II.<br />

63. -61° is cotenninal with 299° and -421°; these<br />

angles are in quadrant IV.<br />

y<br />

66.<br />

Points: (-3, -3), (0, 0)<br />

r = ~(-3 _0)2 +(-3 _0)2<br />

= .../9+9<br />

=..ff8<br />

=F2<br />

=M<br />

(- 5. 2)<br />

y<br />

64.<br />

299°; -421°;<br />

quadrantIV<br />

y<br />

Points: (-5.2), (0. 0)<br />

r = ~(-5 _0)2 +(2-0)2<br />

=.../25+4<br />

=.fi9<br />

67.<br />

y<br />

-H-t+hI-::+++++-. JC<br />

201°; -519°;<br />

quadrantIII<br />

A positive angle cotenninal with -159° is<br />

-159° + 360° = 201°. .<br />

A negative angle cotenninal with -159° is<br />

-159°-360° = -519°.<br />

These angles are in quadrant III.<br />

Points: (-3. -5). (0. 0)<br />

r = ~(-3- 0)2 + (-5 _0)2<br />

= .../9+25<br />

=../34<br />

13


<strong>Trigonometry</strong> <strong>Solutions</strong> <strong>Manual</strong><br />

14 Chapter 1 The Trigonometric Functions<br />

68.<br />

y<br />

--+--+--::11'+....-. x<br />

o<br />

2<br />

71. 90 revolutions per minute<br />

=-90 revo I unons ' per second<br />

60<br />

= 1.5 revolutions per second<br />

72. 45 revolutions per minute<br />

45 I' d<br />

=60 revo unon per secon<br />

69.<br />

Points: (.,fj, 1), (0, 0)<br />

r = ~(.,fj - 0)2 + (1- 0)2<br />

=..)3+1<br />

=.,[4<br />

=2<br />

y<br />

--+-t-t-+:::l-+-II-+-++ x<br />

-2<br />

Points: (-2, 2.,fj), (0, 0)<br />

r=~(-2-0)2 +(2..J3 _0)2<br />

=..)4+ 12<br />

=.J16<br />

=4<br />

=l revolution per second<br />

4<br />

73. 600 rotations per minute<br />

600. d<br />

= 60 rotations per secon<br />

=10 rotations per second<br />

=5 rotations per .!.. second<br />

2<br />

= 5(360°)<br />

1<br />

= 1800° per - second<br />

2<br />

74. If the propeller rotates 1000 times per minute,<br />

then i 1000 16 2. d<br />

en It rotates --= - times per secon .<br />

60 3<br />

Each rotation is 360 0 , so the total number of<br />

degrees a point rotates in one second is<br />

(36O{16~) = 6000°.<br />

75. 75° per minute = 75°(60) per hour<br />

= 4500° per hour<br />

4500 . h<br />

= 360 rotations per our<br />

=12.5 rotations per hour<br />

70. Y<br />

_H-H::-Id-t++++++ x<br />

Points: (4.,fj,-4), (0, 0)<br />

r = ~(4.,fj - 0)2 + (-4 - 0)2<br />

=..)48+16<br />

=-./64<br />

=8<br />

76. 74.25° = 74° +(.25)(60') = 74° 15'<br />

74°20' -74°15' = 5'<br />

74020' = 740 + 20° = 74.330<br />

60<br />

74 .33° -74.25° = .08°<br />

The difference in measurements is 5' to the _ ____<br />

nearest minute or .08 0 to the nearest hundredth of<br />

a degree.<br />

77. (J = 0.763"<br />

90° - (J = 90° - 0.763"<br />

= 89° 59'60" - 0.763"<br />

= 89° 59'59.237"<br />

78. (J = 0.292"<br />

90° - (J = 90° - 0.292"<br />

= 89° 59' 60" - 0.292"<br />

= 89° 59'59 .708"<br />

14


<strong>Trigonometry</strong> <strong>Solutions</strong> <strong>Manual</strong><br />

1.3 Angle Relationships and Similar Triangles 15<br />

79. The earth rotates 360° in 24 hours. 360° is equal<br />

to 360(60) minutes. or 21.600' .<br />

24 hours x<br />

---=­<br />

21.600'<br />

l'<br />

x= o.ooi .<br />

It should take the motor 0.001 hours or 3.6 =: 4<br />

seconds to rotate the telescope one minute.<br />

SO. Since we have five central angles that comprise a<br />

full circle. we find :<br />

5 · 26 = 360°<br />

106=360<br />

6=36°<br />

The angle of each point of the five-pointed star<br />

measures 36°.<br />

Section 1.3<br />

Exercises<br />

1. In geometry. opposite angles are called vertical<br />

angles.<br />

3. 5x - 129 = 2x - 21 since vertical angles are equal.<br />

3x-129=-21<br />

3x= 108<br />

x=36<br />

5x -129 = 5(36) -129<br />

= 180-129<br />

=51<br />

2x-21 :::;;2(36)-21<br />

=72-21<br />

=51<br />

Both angles are 51 0.<br />

4. l l,r - 37 = Tx + 27 since vertical angles are<br />

equal.<br />

4x-37=27<br />

4x=64<br />

x=16<br />

llx-37:::;; 11(16)-37<br />

:::;; 176-37<br />

:::;; 139<br />

7x+27:::;; 7(16)+ 27<br />

=112+27<br />

:::;; 139<br />

Both angles are 139°.<br />

5. x + (x + 20) + (210 - 3x):::;; 180 since the sum of<br />

the angles of a triangle is 180°.<br />

-x+230:::;;180<br />

-x:::;;-50<br />

x:::;;50<br />

x + 20 = 50 + 20 :::;; 70<br />

210 - 3x:::;; 210 - 3(50)<br />

:::;; 210-150<br />

=60<br />

The three angles are 50°. 70°. and 60°.<br />

6. (lOx-20) + (x + 15) + (x + 5) = 180<br />

since the sum of the angles of a triangle are 180°.<br />

12x = 180<br />

x:::;; 15<br />

lOx - 20:::;; 10(15)- 20<br />

:::;; 150-20<br />

:::;; 130<br />

x + 15 :::;; 15+ 15 = 30<br />

x+5:::;; 15+5:::;;20<br />

The three angles are 130°. 30°. and 20°.<br />

7. (x - 30) + (2x -120) + (~x + 15) = 180 since the<br />

sum of the angles of a triangle is 180°.<br />

L, -135:::;; 180<br />

2<br />

7<br />

-x:::;; 315<br />

2<br />

2 .<br />

x:::;; (315)-=90<br />

7<br />

x - 30 =90 - 30 =60<br />

1 1<br />

-x+ 15:::;; -(90)+ 15<br />

2 2<br />

=45 + 15<br />

:::;;60<br />

2x -120:::;; 2(90) -120<br />

:::;;180-120<br />

=60<br />

All three angles are 60°.<br />

8. x + x + x = 180 since the sum of the angles of a<br />

triangle are 180°.<br />

3x =180<br />

x=60<br />

All three angles are 60°.<br />

15


<strong>Trigonometry</strong> <strong>Solutions</strong> <strong>Manual</strong><br />

16 Chapter 1 The Trjgonometrle Functions<br />

9. (3x + 5) + (5x + 15) = 180 since these two angles<br />

are supplementary.<br />

8x+20= 180<br />

8x= 160<br />

x=20<br />

3x + 5 = 3(20) + 5<br />

=60+5<br />

=65<br />

5x + 15 = 5(20) + 15<br />

= 100+ 15<br />

=115<br />

The two angles are 65° and 115°.<br />

10. (5x - 1) + (2x) = 90 since the angles are<br />

complementary.<br />

7x-l=90<br />

7x=91<br />

x=13<br />

5x -1 = 5(13) -1 = 65 -1 = 64<br />

2x = 2(13) = 26<br />

The two angles are 64° and 26°.<br />

11. 2x - 5 = x + 22 since alternate interior angles are<br />

equal.<br />

x-5 =22<br />

x=27<br />

2x - 5 = 2(27) - 5 = 54 - 5 = 49<br />

x + 22 = 27 + 22 = 49<br />

Both angles are 49°.<br />

12. 2x + 61 = 6x - 51 since alternate exterior angles<br />

are equal.<br />

61=4x-51<br />

112=4x<br />

28=x<br />

2x+61 =2(28) +61 =56+61 =117<br />

6x-51 =6(28)- 51 = 168 -51 = 117<br />

Both angles are 117°.<br />

13. (x + 1) + (4x - 56) = 180 since the sum of interior<br />

angles on the same side of the transversal is 180°.<br />

5x -55 =180<br />

5x=235<br />

x=47<br />

x + 1 = 47 + 1 = 48<br />

4x-56=4(47)-56= 188-56= 132<br />

The angles are 48° and 132°.<br />

14. lOx + 11 = 15x- 54 since alternate exterior<br />

angles are equal.<br />

11 =5x-54<br />

65=5x<br />

13=x<br />

lOx + 11 = 10(13)+ 11<br />

=130+11<br />

= 141<br />

15x - 54 = 15(13)- 54<br />

= 195-54<br />

=141<br />

Both angles are 141°.<br />

IS. Let x =the measure of the third angle.<br />

37° + 52° + x = 180°<br />

89°+x =180°<br />

x=91°<br />

16. Let x = the measure of the third angle.<br />

x + 29° + 104° =180°<br />

x+133°=180°<br />

x=47°<br />

17. Let x = the measure of the third angle.<br />

147°12' + 30°19' + x =180°<br />

177°31'+x =180°<br />

x =179°60' -177°31'<br />

x =2°29'<br />

18. Let x = the measure of the third angle.<br />

x + 136°50' + 41° 38' =180°<br />

x + 178°28' =180°<br />

x =179°60' -178°28'<br />

x=1°32'<br />

19. Let x = the measure of the third angle.<br />

74.2° + 80.4° + x =180°<br />

154.6° + x =180°<br />

x=25.4°<br />

20. Let x = the measure of the third angle.<br />

29.6° + 49.7° + x =180°<br />

79.3° + x =180°<br />

x =100.7°<br />

16


<strong>Trigonometry</strong> <strong>Solutions</strong> <strong>Manual</strong><br />

1.3 Angle Relationships and Similar Triangles 17<br />

23. angle 1 = 55° (vertical angle with 55°)<br />

angle 5 = 180° - 120° = 60° (interior angles on<br />

same side of transversal)<br />

angle 4 = 180° - 60° - 55° = 65°<br />

(supplemental angle with angle 5 and 55°)<br />

angle 2 = 65° (vertical angle with angle 4)<br />

angle 3 = 60° (vertical angle with angle 5)<br />

angle 6 = 120° (vertical angles)<br />

angle 7 = 180° - 120° = 60° (supplemental angle<br />

with angle 6)<br />

angle 8 = 60° (vertical angle with angle 7)<br />

angle 10= 180° - 60° - 65° = 55° (sum of angles<br />

of triangle is 180°) .<br />

angle 9 = 55° (vertical angle with angle 10)<br />

24. Given x + y = 40.<br />

(x + 2y) + l1x = 180 since the angles are<br />

supplementary .<br />

x+ y = 40 (1)<br />

12x + 2y = 180 (2)<br />

Take -2 times equation (1) + equation (2).<br />

-2x-2y=-80<br />

12x +2y = 180<br />

lOx =100<br />

x=1O<br />

Substitute 10 for x in equation (1).<br />

lO+y=40<br />

y=30<br />

x + 2y = 10+ 2(30) = 10 + 60 = 70<br />

llx = 11(10) = 110<br />

The angles are 70° and 110°.<br />

25. The triangle has a right angle, but each side has a<br />

different measure. The triangle is a right triangle<br />

and a scalene triangle.<br />

26. The triangle has one obtuse angle and three<br />

unequal sides, so it is obtuse and scalene.<br />

27. The triangle has three acute angles and three equal<br />

sides, so it is acute and equilateral.<br />

28. The triangle has two equal sides and all angles are<br />

acute, so it is acute and isosceles.<br />

29. The triangle has a right angle and three unequal<br />

sides, so it is right and scalene .<br />

30. The triangle has one obtuse angle and two equal<br />

sides, so it is obtuse and isosceles.<br />

31. The triangle has a right angle and two equal sides,<br />

so it is right and isosceles.<br />

32. The triangle has a right angle with three unequal<br />

sides, so it is a scalene, right triangle.<br />

33. The triangle has one obtuse angle and three<br />

unequal sides, so it is obtuse and scalene.<br />

34. This triangle has three equal sides and all angles<br />

are acute, so it is an acute, equilateral triangle.<br />

35. The triangle has three acute angles and two equal<br />

sides, so it is acute and isosceles.<br />

36. This triangle has a right angle with three unequal<br />

sides, so it is a scalene, right triangle.<br />

40.<br />

Connect the right end of the semi-circle to the<br />

point where the arc crosses the semi-circle. Since<br />

the setting of the compass never changed, the<br />

triangle is equilateral. Therefore, each of its<br />

angles is 60°.<br />

41. Corresponding angles are A and P, Band Q, C<br />

and R; corresponding sides are AB and PQ, CB<br />

and RQ, AC and PRo<br />

42. Corresponding angles are A and P, B and Q, C<br />

and R; Corresponding sides are AC and PR, BC<br />

and QR, AB and PQ.<br />

43. Corresponding angles are A and C, ABE and CBD,<br />

E and D; Corresponding sides are AB and CB, EB<br />

and DB,AE and CD.<br />

44. Corresponding angles are Hand F, K and E, HGK<br />

and FGE; corresponding sides are HK and FE,<br />

GK and GE, HG and FG.<br />

45. Q = A so Q = 42°.<br />

A + B + C= 180° so B = 180° - 90° - 42° = 48°.<br />

B = R so R = 48°.<br />

P = C so P = 90°.<br />

46. A + B + C = 180°<br />

so A = 180° - 46° - 78° = 56°.<br />

M=BsoM=46°.<br />

N = A so N = 56°.<br />

P= CsoP = 78°.<br />

47. K=BsoB= 106°.<br />

A + B + C= 180° so A = 180°.- 30° - 106° = 44°.<br />

A=MsoM=44°.<br />

48. Y = V so Y = 28°.<br />

X + Y + Z = 180° so Z = 180° - 74° - 28° = 78°.<br />

T=XsoT=74°.<br />

W=Zso W=78°.<br />

49. X+ Y+Z= 180° soX= 180°-90°-38°=52°<br />

X = M so M = 52°.<br />

17


<strong>Trigonometry</strong> <strong>Solutions</strong> <strong>Manual</strong><br />

18 Chapter 1 The Trigonometric Functions<br />

50. P + Q + R = 180° so R =180° - 64° - 20° = 96°.<br />

T = P so T = 20°.<br />

U =R so U =96° .<br />

V=QsoV=64°.<br />

In Exercises 51-56, corresponding sides of similar<br />

triangles are proportional.<br />

57. Let T =the height of the tree.<br />

The triangle formed by the tree and its shadow is<br />

similar to the triangle formed by the stick and its<br />

shadow .<br />

L::::dT .6J'<br />

45 3<br />

25 a<br />

51. -=- T 45<br />

10 8<br />

-=­<br />

2 3<br />

200= lOa<br />

T<br />

20=a -=15<br />

25 b<br />

2<br />

-=- T=30<br />

10 6 The tree is 30 m high.<br />

150 = lOb<br />

15=b 58.<br />

a 75 b 75<br />

52. -=- -=­<br />

10 25 20 25<br />

b<br />

~=3 -=3<br />

10 20<br />

a=30 b=60<br />

180<br />

X 9<br />

6 a<br />

53. -=­<br />

-=­<br />

12 12<br />

180 15<br />

72 = 12a<br />

15x =1620<br />

6=a<br />

x=108<br />

b 6<br />

The height of the tower is 108 ft.<br />

-=­<br />

15 12 59. Let x = the middle side of the actual triangle,<br />

b 1<br />

-=­<br />

and y =the longest side of the actual triangle.<br />

15 2 The triangles in the photograph and on the actual<br />

2b=15 piece of land are similar. The shortest side on the<br />

15 1 land corresponds to the shortest side in the<br />

54. -=­<br />

b=-or7­<br />

2 2<br />

photograph.<br />

400 x<br />

-=a<br />

3 4 5<br />

6 9 100 =!.<br />

9a=18 5<br />

a=2 x=500<br />

400 y<br />

-=x<br />

9<br />

55. -=­<br />

4 7<br />

4 6<br />

loo=L<br />

x 3 7<br />

-=­<br />

4 2 y=700<br />

2x=12 The other two sides are 500 m and 700 m long.<br />

x=6<br />

m 21<br />

56. -=­<br />

12 14<br />

14m =252<br />

m=18<br />

18


<strong>Trigonometry</strong> <strong>Solutions</strong> <strong>Manual</strong><br />

60. Let x = the height of the lighthouse. x 100+ 120<br />

63. =---­<br />

50 100<br />

x~ x 220<br />

-=­<br />

28 50 100<br />

x 11<br />

-=­<br />

50 . 5<br />

1.75~ 5x =550<br />

3.5<br />

x=!lO<br />

X 28<br />

--=-<br />

64. L= 40<br />

1.75 3.5<br />

3.5x =49<br />

60 40+160<br />

y 40<br />

x=14 -=­<br />

60 200<br />

The lighthouse is 14 m tall. 200y =2400<br />

61. Let h = the height of the building.<br />

y=12<br />

The triangle formed by the house and its shadow<br />

is similar to the triangle formed by the building<br />

c<br />

65.<br />

10+90<br />

-=--­<br />

and its shadow. 100 90<br />

c 100<br />

15~ -=­<br />

100 90<br />

40 c 10<br />

-=­<br />

100 9<br />

1.3 Angle Relationships and Similar Triangles 19<br />

1000<br />

c=--:::lll.1<br />

9<br />

Map :<br />

T 17 cm Y 75<br />

8c~cm<br />

m=p<br />

3<br />

m=-­<br />

256<br />

300<br />

66. The larger right triangle (with hypotenuse m) and<br />

15 h<br />

-=­<br />

the smaller right triangle (with hypotenuse 80) are<br />

40 300 similar.<br />

4500 = 40h m 75+5<br />

112.5= h -=-­<br />

80 75<br />

The height of the building is 112.5 ft. m 80<br />

-=­<br />

80 75<br />

62. Let a =the distance between Phoenix and Tucson<br />

75m=6400<br />

and b = the distance between Tucson and Yuma.<br />

6400<br />

Actual :<br />

T b Y 67. Let a = the length of the longest side ofthe first<br />

quadrilateral,<br />

x =the length of the shortest side of the second<br />

~<br />

P<br />

quadrilateral,<br />

a 230 b 230 and y =the other unknown length .<br />

-=- -=-­<br />

8 12 17 12<br />

24<br />

I8Q'<br />

12a = 1840 12b = 3910<br />

a « 153.3 b::: 325.8<br />

The distance from Phoenix to Tucson is about<br />

153.3 Ian. The distance from Tucson to Yuma is<br />

about 325.8 km.<br />

«>:<br />

~<br />

19


<strong>Trigonometry</strong> <strong>Solutions</strong> <strong>Manual</strong><br />

20 Chapter 1 The Trigonometric Functions<br />

Corresponding sides are in proportion.<br />

. 32 2<br />

Smce -=-,<br />

48 3<br />

o 2<br />

-=­<br />

60 3<br />

30 =120<br />

0=40<br />

24 2<br />

-=y<br />

3<br />

2y=72<br />

y=36<br />

18 2<br />

-=x<br />

3<br />

2x=54<br />

x=27<br />

The unknown side in the first quadrilateral is<br />

40 em; the unknown sides in the second<br />

quadrilateral are 36 em and 27 em.<br />

68. Let x =length of entire body carved into<br />

mountain.<br />

.1= 61<br />

60 x<br />

-=t<br />

.If<br />

60 x<br />

~x=6OC:)<br />

~x=380<br />

4<br />

x = 1520 = 506~<br />

3 3<br />

Abraham Lincoln's body would be 506~ft tall.<br />

3<br />

69. Corresponding sides are in proportion.<br />

x-5 5<br />

--=­<br />

15 15<br />

15(x-5) = 75<br />

15x-75=75<br />

15x= 150<br />

x=to<br />

x-2y 5<br />

----"'--=­<br />

(x-2y)+(x+y) 15<br />

x-2y 5<br />

--=­<br />

2x - y 15<br />

15(x- 2y) =5(2x - y)<br />

15x-30y= tox-5y<br />

5x= 25y<br />

Substitute to for x.<br />

50=25y<br />

y=2<br />

Therefore, x = to and y =2.<br />

70. The two triangles are similar.<br />

Corresponding angles are equal.<br />

toy+ 8 =58<br />

toy=50<br />

y=5<br />

Corresponding sides are in proportion.<br />

x-y 6 1<br />

--=-=x+<br />

y 18 3<br />

3(x- y) = I(x+ y)<br />

3x-3y= x+ y<br />

2x=4y<br />

x=2y<br />

Substitute 5 for y.<br />

x=2(5) = to<br />

Therefore, x = to and y = 5.<br />

71. (a) Since a thumb covers about 2 arc degrees or<br />

about 120 arc minutes, approximately..!. of a<br />

4<br />

thumb would cover the moon.<br />

(b) 20 + 10 =30 arc degrees<br />

72. (a) Let D s be the Mars-Sun Distance, d s the<br />

diameter of the sun, D m the Mars-Phobes<br />

distance, and d m the diameter of the Phobes.<br />

Then, by similar triangles<br />

Ds =.!!.L<br />

D m d m<br />

D = Dsdm<br />

m<br />

d s<br />

142,000,000(17.4)<br />

=<br />

865,000<br />

=2856 mi<br />

(b) No. Phobes does not come close enough to<br />

the surface of Mars.<br />

73. (a) Let D, be the Jupiter-Sun distance, d, the<br />

diameter of the sun, D m the Jupiter­<br />

Ganymede distance, and d m the diameter of<br />

Ganymede. Then, by similar triangles,<br />

D, d<br />

-=­ s<br />

D m d m<br />

D = Dsdm<br />

m d,<br />

484,000,000(3270)<br />

=<br />

865,000<br />

= 1,830,000 mi<br />

(b) Yes; Ganymede is usually less than half this<br />

distance from Jupiter.<br />

20


<strong>Trigonometry</strong> <strong>Solutions</strong> <strong>Manual</strong><br />

1.4 Definitions of the Trigonometric Functions 21<br />

74. (a) Since CG and DH are parallel and the<br />

triangles b.CAG and b.HAD share the<br />

common angle A, they are similar.<br />

Additionally, BD II EG with the common<br />

angle A, so triangles b.BAD and b.EAG are<br />

similar.<br />

CG AG .<br />

(b) Triangles b.ABD and MEG are similar, so<br />

AG EG<br />

-=-. However, BD = EF,so<br />

AD BD<br />

AG EG<br />

AD = EF'<br />

. EG EG EG<br />

(c) SmceBD= EF= 1 -=-=-.<br />

, EF BD 1<br />

Section 1.4<br />

Exercises<br />

1.<br />

y<br />

8 in Quadrant IV<br />

y<br />

SeCtion 1.4<br />

AG EG<br />

(d) CG feet = - = - = EG paces<br />

AD EF<br />

Connections (page 38)<br />

1. From the figure, we see that b.QOP and MOB<br />

are similar.<br />

PQ=y=l.=l.=sin6 (sincer= 1)<br />

1 r<br />

- = - . HD = 1, so this becomes<br />

HD AD<br />

CG AG<br />

-=­<br />

1 AD<br />

OQ= x = ~= ~ =cos6 (since r= 1)<br />

1 r<br />

AB OA<br />

-=y<br />

x<br />

. AB 1 y<br />

SmceOA= 1, -=-, so AB=-=tan6.<br />

. y x x<br />

2.<br />

3.<br />

y<br />

y<br />

2. y<br />

/<br />

8 in Quadranl III<br />

21


<strong>Trigonometry</strong> <strong>Solutions</strong> <strong>Manual</strong><br />

22 Chapter 1 The Trigonometric Functions<br />

4. y 7. (0,2)<br />

x=O,y =2<br />

r=~02 +2 2<br />

+-+-+-+-+-+-H--+-t-Hi-t;ooIl'I':'t"'!t+x<br />

= ..}0+4<br />

=../4<br />

=2<br />

sinO=~= 1<br />

2<br />

o<br />

cosO=-=0<br />

2<br />

5. (-3,4)<br />

x=-3,y =4<br />

2<br />

r = ~(_3)2 + 4 2 tan0 = ō undefined<br />

=..}9+16 o<br />

cotO =­ = 0<br />

=../25<br />

2<br />

=5<br />

2<br />

sec0 = ­ undefined<br />

sino=z=i<br />

r 5<br />

x 3<br />

cosO=-=-­<br />

r 5<br />

y 4<br />

tanO=-=-­<br />

x 3<br />

x 3<br />

cotO=-=-­<br />

Y 4<br />

r 5<br />

secO=-=-­<br />

x 3<br />

r 5<br />

cscO=-=y<br />

4<br />

o<br />

2<br />

cscO=-= 1<br />

2<br />

8. (-4,0)<br />

r=4<br />

sinO=Q=O<br />

4<br />

-4<br />

cosO=­=-1<br />

4<br />

o<br />

tan=-=O<br />

-4<br />

-4<br />

cot 0 = ­ Undefined<br />

6. (-4, -3) 4<br />

secO=-=-1<br />

x=-4,y=-3 -4<br />

4<br />

r = ~(-4)2 + (_3)2<br />

cscO= ­ Undefined<br />

o<br />

= ..}16+9<br />

=../25<br />

=5<br />

. y -3 3<br />

smO=-=-=-­<br />

r 5 5<br />

x -4 4<br />

cosO=-=-=-­<br />

r 5 5<br />

y -3 3<br />

tanO=-=-=x<br />

-4 4<br />

x -4 4<br />

cotO=-=-=y<br />

-3 3<br />

455<br />

secO=-=-=-­<br />

x -4 4<br />

r 5 5<br />

cscO=-=-=-­<br />

y -3 3<br />

9. (1. .J3)<br />

o<br />

x=l,y=.J3<br />

r = ~12 +(./3)2<br />

=.J1+3<br />

=../4<br />

=2<br />

sinO = .J3<br />

2<br />

1<br />

cosO=­ 2<br />

22


<strong>Trigonometry</strong> <strong>Solutions</strong> <strong>Manual</strong><br />

1.4 Definitions of the Trigonometric Functions 23<br />

12. (-5.1021,7.6132)<br />

tan 8 = ..[3 =..[3<br />

1<br />

r "" .J83.99 "" 9.1647<br />

1 1..[3..[3<br />

cot8=-=-·-=sin8<br />

= 7.6132 "" .83071<br />

..[3 ..[3..[3 3<br />

9.1647<br />

2 cos8 = -5.1021 "" -.55671<br />

sec8=-=2 9.1647<br />

1<br />

2 2..[3 2..[3 tan 8 = 7.6132 '" -1.4922<br />

csc8 = - = _ .- =-- -5.1021<br />

..[3 ..[3..[3 3<br />

cot 8 = -5.1021 '" -.67016<br />

7.6132<br />

10. -2..[3, - 2)<br />

sec 8 = 9.1647 '" -1.7963<br />

r = ~r-(--2..[3-3--::)2-+-(_-2)-2<br />

-5.1021<br />

9.1647<br />

esc 8 = --"" 1.2038<br />

=.J12+4 7.6132<br />

=.J16<br />

=4 15. Since the cot 8 is undefined, and cot 8 = ~,<br />

-2 1 y<br />

sin8=-=-­<br />

4 2 where (x, y) is a point on the terminal side of 8 ,<br />

y = 0 and x can be any nonzero number.<br />

-2..[3 ..[3<br />

cos8=--=-­<br />

4 2 tan 8 = I., which is 0 divided by a nonzero<br />

-2 1 ..[3<br />

x<br />

tan8 = -2..[3 =13 = 3"<br />

number. Therefore, tan 8 = O.<br />

-2..[3 r:; 16. Since the terminal side of angleft is in quadrant<br />

cot8=--=v3<br />

III and (x, y) is a point on that terminal side, both<br />

. -2<br />

4 2 2..[3 x and y must be negative. Since r = ~x2 +i . r<br />

sec 8 = -2..[3 =-13=--3­ is positive .<br />

4<br />

csc8=-=-2<br />

sinf3 = I.<br />

-2 r<br />

Since y is negative and r is positive, sinft is<br />

11. (8.7691, -3.2473) negative.<br />

x = 8.7691, y = -3.2473<br />

x<br />

cosf3 =­ r<br />

r = ~(8.7691)2 + (-3.2473)2<br />

Since x is negative and is positive cosft is<br />

= .J87.442072 negative.<br />

=9.3510466<br />

tanf3 = I.<br />

sin 8 = I. "" -.34727<br />

r<br />

cos8 = ~ "" .93777<br />

r<br />

tan8 = I. "" -.37031<br />

x<br />

cot 8 = ~ '" -2.7004<br />

y<br />

sec 8 =!.. "" 1.0664<br />

x<br />

csc8 =!.. "" -2.8796<br />

y<br />

x<br />

Both y and x are negative, so tanft is positive<br />

cotf3 =- x<br />

Y<br />

Both x and y are negative, so cotft is positive.<br />

r<br />

secf3 =­<br />

x<br />

Since r is positive and x is negative, secft is<br />

negative.<br />

r<br />

cscf3 =­ y<br />

Since r is positive and y is negative, cscft is<br />

negative.<br />

23


<strong>Trigonometry</strong> <strong>Solutions</strong> <strong>Manual</strong><br />

24 Chapter 1 The Trigonometric Functions<br />

In Exercises 17-24, r = ~ x 2 + Y 2 , W hi IC h i IS posinve. . .<br />

. ., y. . ,<br />

17. In quadrant II, y IS positive so - IS posrtive.<br />

r<br />

18. In quadrant II, x is negative so .:::. is negative.<br />

r<br />

26. Since x ~ 0, the graph of the line 3x + 5y = ais<br />

shown to the right of the y-axis , A point on this<br />

graph is (5, -3) since 3(5) + 5(-3) = O.<br />

r=~52+(_3)2<br />

y<br />

=.../25+9=.J34<br />

3x + 5y = O. x


<strong>Trigonometry</strong> <strong>Solutions</strong> <strong>Manual</strong><br />

1.4 Definitions of the Trigonometric Functions 25<br />

28. Since x ::; O. the graph of the line -6x - y = 0 is 30. Since x ~ 0, the graph of the line 6x - 5y = 0 is<br />

shown to the left of the y-axis, A point on this<br />

shown to the right of the y-axis , A point on this<br />

graph is (-1, 6) since -6(-1) - 6 = O. line is (5, 6) since 6(5) - 5(6) = O.<br />

r = ~(_1)2 +6 2 =.Jl + 36 =.J37<br />

r=~52+62 =.J25+36=.J6l<br />

y<br />

y<br />

6<br />

5<br />

6x-5y =O.x~ 0<br />

. y 6 6.J37 . y 6 6.J6l<br />

sm6=-=--=-­<br />

r ~ 37 r .J6l 61<br />

x -1 .../37 x 5 5.J6l<br />

cos6=-=--=-­<br />

r .J6l 61<br />

sm6=-=--=--<br />

cos6 = -;:= 'J37 =-37<br />

y 6 y 6<br />

tan6=-=-=-6<br />

tan6=-=x<br />

-1 x 5<br />

x -1 1 x 5<br />

cot6=-=-=--<br />

cot6=-=y<br />

6 6 y 6<br />

r.J6l<br />

sec6 = !. = .../37 =-.../37<br />

sec6=-=-­<br />

s -1 x 5<br />

r .../37 r .J6l<br />

csc6=-=--<br />

cscf}=-=-­<br />

y 6 y 6<br />

29. Since x ::; 0, the graph of the line -5x - 3y = 0 is 33. cos 90° + 3 sin 270° = 0 + 3(-1) =-3<br />

shown to the left of the y-axis. A point on this line<br />

is (-3, 5) since -5(-3) - 3(5) = O.<br />

r=~(-3)2+52 =.J9+25 =../34 35. 3 sec 180° - 5 tan 360° = 3(-1) - 5(0) =-3<br />

(-3.5)'<br />

5<br />

36. 4 esc 270° + 3cos 180°<br />

=4(-1)+3(-1)<br />

=-7<br />

-3 37. tan3600+4sinI800+5cos 2180°<br />

. = 0 + 4(0) + 5(-1)2<br />

sin6 = 5.J34 38. 2 sec 0° + 4 cot 2 90° + cos 360°<br />

34<br />

=2(1) + 4(0)2 + 1<br />

3../34<br />

=5<br />

cos6=--- =3<br />

34<br />

5<br />

tan6=-­<br />

3<br />

3<br />

cot6 =-- 40. sin 2 360° + cos 2 360° =0 2 + 1 2<br />

5 =1<br />

..J34<br />

sec6=--­<br />

3<br />

..J34<br />

csc6=­<br />

5<br />

41. sec21800-3sin23600+2cosI80°<br />

=(_1)2 _3(0)2 +2(-1)<br />

=1-0-2<br />

=-1<br />

25


<strong>Trigonometry</strong> <strong>Solutions</strong> <strong>Manual</strong><br />

26 Chapter 1 The Trigonometric Functions<br />

42. 5sin 2 90° + 2 cos 2 270° -7 tan 2 360°<br />

== 5(1)2+ 2(0)2 - 7(0)2<br />

==5<br />

43. sin[n · 180°]<br />

This angle is a quadrantal angle whose terminal<br />

side lies on either the positive part of the x-axis or<br />

the negati ve part of the x-axis, Any point on these<br />

terminal sides would have the form (k, 0), where k<br />

is any real number k :;t. o.<br />

sin[n · 180°] ==1. == 0 == 0<br />

r ~k2 +0 2<br />

44. cos [(2n + 1) .90°]<br />

This angle is a quadrantal angle whose terminal<br />

side lies on either the positive part of the y-axis or<br />

the negative part of the y-axis. Any point on these<br />

terminal sides would have the form (0, k), where k<br />

is any real number, k:;t. O.<br />

x<br />

cos[(2n + 1).90°] =­<br />

r<br />

o<br />

45. tan[(2n + 1) . 90°]<br />

This angle is a quadrantal angle whose terminal<br />

side lies on either the positive part of the y-axis or<br />

the negati ve part of the y-axis .<br />

Any point on these terminal sides would have the<br />

form (0, k), where k is any real number, k:;t. O.<br />

tan[(2n + 1).90°] =1. =!5..<br />

x 0<br />

tan[(2n + I) . 90°] is undefined.<br />

46. tan [n . 180°]<br />

The angle is a quadrantal angle whose terminal<br />

side lies on either the positive part of the x-axis or<br />

the negative part of the x-axis, Any point on these<br />

terminal sides would have the form (k, 0), where k<br />

is any real number, k:;t. o.<br />

y 0<br />

tann·180 = - = - =0<br />

k k<br />

47. Using a calculator, sin 15° = .258819045 and<br />

cos 75° == .258819045.<br />

We can conjecture that the sines and cosines of<br />

complementary angles are equal. Try another pair<br />

of complementary angles.<br />

sin 30° = cos 60°<br />

.5=.5<br />

Therefore, our conjecture is true.<br />

48. Using a calculator, tan 25° = .466307658 and<br />

cot 65° = .466307658.<br />

We can conjecture that the tangent and cotangent<br />

of complementary angles are equal. Try another<br />

pair of complementary angles.<br />

tan 45° = .cot 45°<br />

1=1<br />

Therefore, our conjecture is true.<br />

49. Using a calculator, sin 10.° = .173648178 and<br />

sin(-100) =- .173648178.<br />

We can conjecture that the sines of an angle and<br />

its negative are opposites of each other. Using a<br />

circle, an angle ()having the point (x, y) on its<br />

terminal side has acorresponding angle -() with a<br />

point (x, -y) on its terminal side. From the<br />

definition of sine, sin(-(}) = _1. and sin () =1..<br />

r<br />

r<br />

The sines are negatives of each other.<br />

50. Using a calculator, cos 20° =.93969262 and<br />

cos (-20°) =.93969262.<br />

We can conjecture that the cosines of an angle<br />

and its negative are equal. Using a circle, and<br />

angle ()having the point (x, y) on its terminal side<br />

has a corresponding angle -() with point (x, -y)<br />

on its terminal side. From the definition of cosine,<br />

x<br />

x<br />

cos(-(}) = - and cos () = -.<br />

r<br />

r<br />

The cosines are equal.<br />

53. When sin ()== cos (), then 1. = ~ . As a result, the<br />

r r<br />

two values of ()must lie along the line y =x,<br />

which occurs when ()= 45° and ()= 225°.<br />

54. When tan () = 1, we have 1.. = 1or y = x. The<br />

x<br />

two values of () must lie along the line y = x, so<br />

() = 45° and () = 225°.<br />

55. Since cos ()== .8 occurs when x = .8, r =1 and<br />

() == 36.87° , cos ()= -.8 occurs when x = -.8 and<br />

r= 1. As a result, ()= (180 + 36.87) = 216.87°<br />

and () == (180 - 36.87) =143.13°.<br />

56. Since sin ()= .8 occurs when y = .8, r = 1 and<br />

() == 53 .13, sin () = -.8 occurs when y = -.8 and<br />

r= 1. As a result () == (180 + 53,13) = 233.13°<br />

and 8 = (360 - 53.13) =306.87°<br />

57. cos 20° is about .940, and sin 20° is about .342.<br />

58. Use the TRACE feature to move around the circle<br />

in quadrant I.<br />

cos 40° == .766 so T == 40° .<br />

26


<strong>Trigonometry</strong> <strong>Solutions</strong> <strong>Manual</strong><br />

1.5 Using the Definitions of the Trigonometric Functions 27<br />

59. Use the TRACE feature to move around the circle<br />

in quadrant I.<br />

sin 35° == .574 so T =35°<br />

60. Use the TRACE feature to move around the circle<br />

in quadrant I.<br />

cos 45° = sin 45° so T= 45°.<br />

61. As T increases from 0° to 90°, the cosine<br />

decreases and the sine increases.<br />

62. As T increases from 90° to 180°, the cosine<br />

decreases and the sine decreases.<br />

63. (a) From the figure in the text and the definition<br />

of tan (J, we can see that tan (J = I .<br />

x<br />

(b) Solve for x.<br />

tan(J=I<br />

x<br />

x tan (J = y<br />

x=_Ytan(J<br />

64. The area of a triangle is .!.b .h. The area of one of<br />

2<br />

the six equilateral triangles is<br />

.!.(x{.J3 x)=(.!.x2X.J3) .Sin(J= ¥x = .J3.<br />

2 2 2 2 x 2<br />

Section 1.5<br />

Exercises<br />

Substituting sin (J for .J3 in the area of the<br />

2<br />

triangle gives .!.x 2 sin (J. The area of the hexagon<br />

2<br />

is ,~x2 sin(J) = 3x 2 sin(J.<br />

1. 1 is its own reciprocal.<br />

sin (J= esc (J= 1<br />

sin 90° = esc 90° = I<br />

Therefore, (J = 90°.<br />

2. -1 is its own reciprocal.<br />

cos (J = sec (J =-1<br />

cos 180° = sec 180° =-1<br />

3. esc (J= 3<br />

sin (J = _1_ = .!.<br />

csc(J 3<br />

1<br />

5. tanf3=-­<br />

5<br />

1<br />

cot f3 =- f3<br />

tan<br />

1<br />

= -t<br />

=-5<br />

. 1 1 ~<br />

6. sma=--=--=-­<br />

csca ~ 15<br />

7. secf3=_I_= ~ =~<br />

cosf3 -../7<br />

8. cot (J = _ .J5<br />

3<br />

1<br />

tan(J=-­<br />

cot(J<br />

1<br />

=-4­<br />

3 .J5<br />

=-~. .J5<br />

3.J5<br />

=--­<br />

5<br />

9. esc (J= 1.42716321<br />

. (J 1<br />

SID =-­<br />

csc(J<br />

1<br />

=---­<br />

1.42716327<br />

=.70069071<br />

10. cosa = _1_ = 1 = .1019% 57<br />

seca 9.80425133<br />

14. Since tan (J =_1_, multiply both sides by cot (J .<br />

cot (J<br />

tan (J cot (J = 1<br />

Now divide both sides by tan (J .<br />

1<br />

cot(J=-­<br />

tan(J<br />

15. cot Y= 2<br />

1 1<br />

tany=--=cot<br />

y 2<br />

1 1<br />

16. tan¢' = --=-­<br />

cot¢' 3<br />

1 1<br />

4. cosa=--=--=-.4<br />

seca -2.5<br />

27


<strong>Trigonometry</strong> <strong>Solutions</strong> <strong>Manual</strong><br />

28 Chapter 1 The Trigonometric Functions<br />

.J3<br />

17. cotm=­<br />

3<br />

1<br />

tanm=-­<br />

cot to<br />

1<br />

=../3<br />

T<br />

3<br />

=.J3<br />

=.J3<br />

18. tan 9 = _1_ = 1 = 12.,[6 = 12.,[6 =2.,[6<br />

cot 9 if. .,[6 ..J6 6<br />

19. cot a = -.01<br />

1<br />

tana=-­<br />

cota<br />

1<br />

=-­<br />

- .01<br />

=-100<br />

1 1<br />

20. tan/3=--=- =2.5<br />

cot /3 .4<br />

1<br />

21. tan(3B- 4°) =---­<br />

cot(5B-8°)<br />

tan(3B - 4°) = tan(5B - 8°)<br />

3B-4° =5B-8°<br />

4°=2B<br />

B=2°<br />

1<br />

22. cos(6A + 5°) =----­<br />

sec (4A + 15°)<br />

= cos(4A + 15°)<br />

6A + 5° = 4A + 15°<br />

2A =10°<br />

A=5°<br />

23. sec(2a + 6°)cos(5a + 3°) =1<br />

cos(5a + 3°) =---­<br />

sec(2a+6°)<br />

cos(5a + 3°) = cos(2a + 6°)<br />

5a+3°=2a+6°<br />

3a=3°<br />

a=lo<br />

24. sin(49+ 2°) esc (39 + 5°) = 1<br />

sin(49+2 0 ) = 1<br />

esc (39 + 5°)<br />

sin (49+ 2°) =sin(39 + 5°)<br />

49+ 2° = 39+ 5°<br />

9=3°<br />

I<br />

27. sin a> 0 implies a as in quadrant I or II. cos a <<br />

oimplies a is in quadrant II or III. a is in<br />

quadrant II.<br />

28. cosjJ> 0 impliesjJ is in quadrant I or IV.<br />

tanjJ> 0 impliesjJ is in quadrant I or III.<br />

jJ is in quadrant I.<br />

29. tan r> 0 implies ris in quadrant I or III. cot r> 0<br />

implies ris in quadrant I or III. ris in quadrant I<br />

or Ill.<br />

30. tan w < 0 implies w is in quadrant II or IV. cot w<br />

< 0 implies w is in quadrant II or IV. w is in<br />

quadrant II or IV.<br />

31. cos /3< 0 implies /3is in quadrant II or III.<br />

32. tan 9 > 0 implies 9 is in quadrant I or III.<br />

In Exercises 3~, the signs + for positive and - for<br />

negative are given in the following order: sine, cosine,<br />

and tangent.<br />

33. 74° is in quadrant I.<br />

+;+;+<br />

34. 129° is in quadrant II.<br />

+;-;­<br />

35. 183° is in quadrant ill.<br />

-;-;+<br />

36. 298°, is in quadrant IV.<br />

-;+;­<br />

37. 406° is in quadrant I.<br />

+;+;+<br />

38. 412° is in quadrant I.<br />

+;+;+<br />

39. -82° is in quadrant IV.<br />

-;+;­<br />

40. -121° is in quadrant ill.<br />

-;-;+<br />

0 sin 30°<br />

41. tan 30 =--­<br />

cos 30°<br />

cos 30° < I, so sin 30° + cos 30° > sin 30° . .<br />

Therefore, tan 30° is greater.<br />

42. sin 21° > sin 20°<br />

sin ()increases as ()increases from 0° to 90°, so<br />

sin 21° is greater.<br />

28


<strong>Trigonometry</strong> <strong>Solutions</strong> <strong>Manual</strong><br />

1.5 Using the Definitions of the Trigonometric Functions 29<br />

43. sec 33° = __1_; since<br />

cos 33°<br />

1<br />

cos33° < 1, ---> cos 33°.<br />

cos33°<br />

sin 33° < cos 33° < sec 33° .<br />

Therefore, sec 33° is greater .<br />

44. cos 5° < 1 so (cos 5°)2 = cos 2 5° < cos5°<br />

cos 5° is greater.<br />

45. cos 26° > cos 27°<br />

cos ()decreases as ()goes from 0° to 90°, so<br />

cos 26° is greater.<br />

46. cot 2° > cos 2°<br />

cot 2<br />

0<br />

=--<br />

cos 2° S'<br />

10cesm<br />

. 20<br />

<br />

1 1 1<br />

.<br />

sin 2° sin 2°<br />

cos2° 20<br />

-->cos<br />

Multiplying by cos 2°, sin2° ,so<br />

cot 2° is greater.<br />

47. -I:!> sin ()$ 1 for all ()and 2 is not in this range.<br />

sin ()= 2 is impossible.<br />

48. cos a = -1.001 is impossible since -1 :!> cos a $ 1.<br />

49. tan /3can take on all values.<br />

tan /3= .92 is possible.<br />

50. cot w = -12.1 is possible since cot w can be any<br />

real number.<br />

51. sec a = 1 is possible since sec a $ -lor sec a ~ 1.<br />

52. tan ()can take on all values. tan ()= 1 is possible.<br />

53. sin /3+ 1 = .6 means sin /3= -.4, which is possible<br />

because -1 $ sin /3:!> 1. sin /3+ 1 =.6 is possible.<br />

54. sec w + 1 = 1.3 gives sec w = .3 which is<br />

impossible since sec w:!> -1 and sec w ~ 1.<br />

57. Find tan a if sec a = 3, with a in quadrant IV.<br />

Start with the identity tan 2 a + 1 = sec<br />

2<br />

a, and<br />

replace sec a with 3.<br />

tan 2 a +1 = 3 2<br />

2<br />

tan a = 8<br />

tan a = ±..J8 = ±2../2<br />

Since a is in quadrant IV, tan a < 0, so<br />

tan a = -2../2.<br />

58. Find sin a if cos a = _.!. , with a in quadrant II.<br />

4<br />

Start with the identity sin 2 a + cos 2 a = 1, and<br />

replace cos a with _.!..<br />

4<br />

2a+(-±r<br />

sin =1<br />

. 2 1 1<br />

sm a+-=<br />

16<br />

. 2 15<br />

S10<br />

a=­ 16<br />

sina = ± (l5 = ±.Jl5<br />

fi"6 4<br />

Since a is in quadrant II, sin a > 0, so<br />

. .Jl5<br />

s1Oa=--.<br />

4<br />

59. Find esc /3if cot /3 = _.!. and /3is in quadrant IV.<br />

2<br />

2 2<br />

1+ cot /3= csc /3<br />

csc 2 /3 = 1+ (_.!.)2 = 1+.!. = ~<br />

244<br />

csc/3=± V4<br />

fI =±4=±-J5 ,,4 2<br />

/3is in quadrant IV so esc /3 = _ -J5 .<br />

2<br />

55. sin a = .!. is possible because -1 $ sin a $ 1.<br />

2<br />

Furthermore, when sin a = .!.,<br />

2<br />

1 1<br />

csca=--=-=2.<br />

sina<br />

t<br />

sin a = .!. and esc a = 2 is possible.<br />

2<br />

56. tan f3 = 2 is possible. but when tan f3 = 2,<br />

1 1<br />

cot /3 =--= -.<br />

tan/3 2<br />

tan f3 = 2 and cot f3 = -2 is impossible.<br />

29


<strong>Trigonometry</strong> <strong>Solutions</strong> <strong>Manual</strong><br />

30 Chapter 1 The Trigonometric Functions<br />

60. Find sec oif tan 0 =..fi and eis in quadrant m.<br />

3<br />

tan 2 0 + 1 = sec 2 0<br />

2 (..fi)2<br />

sec 0= 3 +1<br />

7 9<br />

=-+­<br />

9 9<br />

16<br />

=­<br />

9<br />

(l6<br />

sec 0 = ±V9<br />

=+i<br />

-3<br />

()is in quadrant III so sec 0 = _i.<br />

3<br />

61. Find cos f3, if esc P= -4 with pin quadrant Ill.<br />

Start with the identity sin f3 =.-.!..-p.<br />

csc<br />

sin p = _1_=_.!.<br />

-4 4<br />

Now use the identity sin 2 f3 + cos 2 P=1 and<br />

replace sin f3 with _.!. .<br />

4<br />

(-±Y+COS 2 P = 1<br />

Smce<br />

..!..-+cos 2 f3 =1<br />

16<br />

cos 2 f3 =~<br />

16<br />

cos f3 =± [l5 =± .Jl5<br />

'116 4<br />

· f3"IS 10 quadrant III, cos f3 =--- .Jl5 .<br />

62. Find sin (), if sec ()::: 2, with 0 in quadrant IV.<br />

Since sec ()=2 and cos 0 =_1_, cos 0 :::.!. .<br />

sec 0 2<br />

Since sin 2 ()+ cos 2 ()::: I, and cos 0 ::: .!. .<br />

2<br />

sin 2 o+(±Y ::: 1<br />

sin 2 O+.!.::: 1<br />

4<br />

. 2 Il 3<br />

SIO 17 =­<br />

4<br />

sinO = ± IT =± .,fj<br />

'14 2<br />

4<br />

Since ()is in quadrant IV, and sin ()is negative in<br />

quadrant IV, sin 0 = - .,fj .<br />

2<br />

63. Find cot a if esc a = -3.5891420, with a in<br />

quadrant III.<br />

1+ cot 2 a ::: csc 2 a<br />

1+ cot 2 a::: (-3.5891420)2<br />

l+cot 2 a::: 12.8819403<br />

cot 2 a::: 11.8819403<br />

cota::: ±3.44701905<br />

Since a is in quadrant Ill, cot a> 0, so<br />

cot a= 3.44701905.<br />

64. Find tan f3, if sin f3::: .49268329, with f3 in<br />

quadrant II.<br />

sin 2 f3 + cos 2 P=1<br />

cos 2 p::: I-sin 2 p<br />

cos p::: ±~1- sin 2 p<br />

cos f3 is in quadrant II, and the cosine is negative<br />

in quadrant II,<br />

cos f3 = _~rl-_-s-in-:2:-f3-<br />

=-Jl-(.49268329)2<br />

= -./1-.24273682<br />

=-.J.75726318<br />

= -.87020870<br />

sin 0<br />

tan f3 =-­<br />

cosO<br />

.49268329<br />

= - .87020870<br />

= - .56616682<br />

65. sinX= .8<br />

sin 2 X +cos 2 X = 1<br />

cos 2 X = I-sin 2 X<br />

cos 2 X :::1-(.8)2<br />

cos 2 X = (cos X)2 = .36<br />

66. tan X :::2<br />

tan 2 X + 1::: sec 2 X<br />

tan 2 X + 1 = (__1<br />

cos 2-) X<br />

(2)2 + 1 = (_1_)2<br />

cosX<br />

1<br />

5- (-­<br />

cos X<br />

)2<br />

30


<strong>Trigonometry</strong> <strong>Solutions</strong> <strong>Manual</strong><br />

1.5 Using the Definitions of the Trigonometric Functions 31<br />

67. sin 2 9+cos 2 8 = 1 3<br />

70. cosa=-­<br />

(.8)2+(-.6)2:b 1<br />

5<br />

.64+.36:b 1<br />

x<br />

cosa=- soletx=-3,,=5.<br />

1=1 True r<br />

x2 + y2 =,2<br />

(-3)2+1 =5 2<br />

Yes, there is an angle 9 for which cos 9 = -.6 and<br />

sin 9= .8.<br />

68. sin 2 8+ cos 2 8 = 1<br />

(iY +(~r ~1<br />

9 4<br />

16 9­<br />

9+1 =25<br />

l =25-9<br />

=16<br />

y =±.J16<br />

-+-~1 y=±4<br />

a is in quadrant ill so y = -4.<br />

145 :# 1 . 4<br />

144<br />

sma=-­<br />

5<br />

No, there is no angle for which cos 9 = ~ and 3<br />

3 cosa=-­<br />

5<br />

• Ll 3<br />

sm e =-.<br />

-4 4<br />

4 tana=-=­<br />

-3 3<br />

3<br />

For Exercises 69-76, remember that' is always positive. cota=­ 4<br />

15 5<br />

69. tana=-­<br />

seea=-­<br />

8<br />

3<br />

5<br />

tan a = Z. and a is in quadrant II, so let y = 15<br />

x<br />

andx=-8.<br />

x2+1 =,2 71. tan fJ = .J3<br />

csca=-­<br />

4<br />

(_8)2 + 15 2 =,2 tan fJ = Z. and fJ is in quadrant ill, so let y = -./3<br />

64+225 =,2<br />

x<br />

and x=-l.<br />

289 =,2 x2 + y2 =,2<br />

17= ,<br />

. y 15<br />

(_1)2 + (-./3)2 =,2<br />

sma=-=­<br />

r 17 1+ 3=,2<br />

x -8 8<br />

cosa=-=-=-­ 4 =,2<br />

, 17 17<br />

2=,<br />

y 15 15<br />

tana=-=-=-- . y -./3 .J3<br />

x -8 8 smfJ=-=--=-­<br />

r 2 2<br />

x -8 8<br />

cota=-=-=-- x -1 1<br />

y 15 15<br />

cosf3=-=-=-­<br />

,22<br />

, 17 17<br />

seea=-=-=-­<br />

x -8 8 tan f3 = Z. = -./3 =.J3<br />

x -1<br />

r 17<br />

csca=-=- x -1 I .J3<br />

y 15 cot f3 = Y= -./3 = .J3 = 3<br />

, 2<br />

seefJ = - = - =-2<br />

x -1<br />

r 2 2.J3<br />

esc f3 = Y= -./3 = - -3­<br />

31


<strong>Trigonometry</strong> <strong>Solutions</strong> <strong>Manual</strong><br />

32 Chapter 1 The Trigonometric Functions<br />

2 r<br />

72. esc f) = 2 = - = - , so let y = 1, r = 2.<br />

1 Y<br />

x 2 + y2 =r2<br />

x 2 +1 2 =2 2<br />

x 2 +1 =4<br />

x 2 =4-1 =3<br />

x = ±.J3<br />

(J is in quadrant II, so x = -J3 .<br />

. f) 1<br />

sm =­<br />

2<br />

.J3<br />

cosf) =-­<br />

2<br />

1 .J3<br />

tanf)=~=-3<br />

cotf) = - .J3 =-J3<br />

1<br />

2 2.J3<br />

secf)=--::)3 =--3­<br />

2<br />

cscf)=-=2<br />

1<br />

73 sin R = ../5, with tan R> 0<br />

• fJ 7 fJ<br />

Since sin {j is positive, {j is in quadrant I or II.<br />

Since tan {j> 0, {j is in quadrant I or Ill.<br />

{j is in quadrant I.<br />

Since sin{j = 1., let y =../5 and r= 7.<br />

r<br />

x 2 + y2 = r2<br />

x 2 +(../5)2 = 7 2<br />

x 2 +5 =49<br />

x 2 =44<br />

x = ±.J44 = ±2.JU<br />

Since fJ is in quadrant I, x = 2..,fli.<br />

. y../5<br />

sm/3=-=r<br />

7<br />

x 2.JU<br />

cos{j=-=-­<br />

r 7<br />

y ../5 .J55<br />

tan{j=-=--=x<br />

2.JU 22<br />

cot/3 = ~ = 2.JU = 2.../55<br />

y ../5 5<br />

r 7 7.JU<br />

sec/3=;=2Jii=~<br />

r 7 7../5<br />

csc{j=y= ../5 =-5­<br />

.J3 x<br />

74. cota=-=­<br />

8 y<br />

sin a> 0 means y > 0, so let x =.J3 y = 8.<br />

x 2 + y2 =r2<br />

(-J3Y +8 2 = r 2<br />

2<br />

3+64 =r<br />

r 2 =67<br />

r =.J67<br />

8 8.J67<br />

sin a = .J67 =(jJ<br />

.J3 .J201<br />

cosa=-=-­<br />

.J67 67<br />

8 8.J3<br />

tan a = .J3 = -3­<br />

.J3<br />

cota=­<br />

8<br />

.J67 .J201<br />

seca= .J3 =-3­<br />

.J67<br />

csca=-­<br />

8<br />

75. cot 0 = -1.49586 with 0 in quadrant IV.<br />

Since cot 0 =~ and 0 is in quadrant IV, let<br />

y .<br />

x = 1.49586 and y =-1.<br />

x 2 +i =r 2<br />

(1.49586)2 +(_1)2 = r 2<br />

2<br />

2.23760 + 1= r<br />

2<br />

3.23760 = r<br />

1.79933= r<br />

sinO = 1.= -1 = -.555762<br />

r 1.79933<br />

o= ~ = 1.49586 _ .831343<br />

cos r 1.79933<br />

tan 0 = 1. = -1 = - .668512<br />

x 1.49586<br />

_<br />

cotO--- x _ 1.49586<br />

-. _ -1 49586<br />

y -1<br />

0 = !-. = 1.79933 = 1.20287<br />

sec x 1.49586<br />

cscO =!-. = 1.79933 =-1.79933<br />

y -1<br />

32


<strong>Trigonometry</strong> <strong>Solutions</strong> <strong>Manual</strong><br />

Chapter 1 Review Exercises 33<br />

76. sina = .164215 = .164215<br />

1<br />

. Y<br />

sma=r<br />

so let y = .164215, = 1.<br />

x 2 + y2 =r2<br />

x 2 +(.164215)2 = 1 2<br />

x 2 +.026966 = 1<br />

2<br />

x = 1-.026966<br />

=.973034<br />

x =±..J.973034<br />

=±.986425<br />

a is in quadrant Il, so, x = - .986425.<br />

sina = .164215<br />

cos a =-.986425<br />

tan a =-.166475<br />

cota =<br />

.986425 = -6.00691<br />

.164215<br />

seca = 1 =-1.01376<br />

-.986425<br />

csca = 1 = 6.08958<br />

.164215<br />

79. The statement is false.<br />

sin 30°+ cos30° 1, 1<br />

.5+ .8660 1, I<br />

l.3660¢ 1<br />

Since 1.3660¢ 1, the given statement is false.<br />

SO. The statement is false.<br />

For any angle (J, -1 :s; sin (J:S; 1.<br />

Since sin (J=2, the given statement is false.<br />

81. Let h =the height of the tree.<br />

50 ft<br />

oos7oo=-­<br />

h<br />

hcos70° =50<br />

50 50<br />

h=--=--= 146 feet<br />

cos 70° 0.3420<br />

82. (8) r 2 = 52 + 12 2<br />

r =..J25+ 144<br />

=..J169<br />

=13prism diopters<br />

(b) tan 0 =2­<br />

12<br />

84. 90° < 0 < 180°;180°< 20 < 360°;<br />

so sin 20 is negative.<br />

85. 90° < 0 < 180°'45°


<strong>Trigonometry</strong> <strong>Solutions</strong> <strong>Manual</strong><br />

34 Chapter 1 The Trigonometric Functions<br />

8. Let A = (-2, -2), B = (8, 4) and C = (2, 14).<br />

Distance from A to B is<br />

~[8 - (_2)]2 + [4 - (_2)]2 = .Jl00+ 36<br />

Distance from A to Cis<br />

= .J136<br />

=2.../34.<br />

~(-2-2)2+(-2-14)2=.J16+256<br />

= .J272<br />

=4.Jfi.<br />

Distance from B to Cis<br />

~(8-2)2 +(4-14)2 =.J36+100<br />

= .J136<br />

=2.../34.<br />

Does (2.../34)2 + (2.../34f = (4.Jfi)2<br />

Yes, since 136 + 136 = 272.<br />

Therefore. these points are vertices of a right<br />

triangle.<br />

10. f(x)=-x 2+3x+2<br />

f(-2) = -{_2)2 +3(-2)+2<br />

=-4-6+2<br />

=-8<br />

11. f(x)=-x 2+3x+2<br />

f(x+ 1)= -{x + 1)2 +3(x+l)+2<br />

= -{x 2 +2x+ 1)+3x +3 +2<br />

= _x 2 -2x-l+3x+3+2<br />

=_x 2 +x+4<br />

12. (a) When X =1. Yl = 6. so f(l) =6.<br />

(b) When f(x) =II =-14. X =5<br />

(c) The graph of y = f(x) crosses the y-axis at<br />

the y-intercept (0. 6).<br />

(d) The graph of y = f(x) crosses the x-axis at<br />

the x-intercept (3. 0).<br />

13. y = 9x + 2<br />

Domain: x can take on any value so the domain is<br />

(-00,00).<br />

Range: y can take on any value, so the range is<br />

(-00,00).<br />

Each x-value gives one y-value so y =9x + 2 is a<br />

function.<br />

14. y =/xl<br />

Domain: x can take on any value. so the domain is<br />

(-00.00).<br />

Range: y can be any nonnegative number, so the<br />

range is<br />

(0,00).<br />

Each x-value gives one y-value so this relation is<br />

a function.<br />

15. y =.,J;<br />

Domain: x ~ O. so the domain is [0,00) .<br />

Range: y can take on any nonnegative value, so<br />

the range is [0, 00).<br />

This is a function.<br />

16. x + I =y2<br />

Domain: y2 is always ~ O. so x + 1 ~ 0 and x ~ -1<br />

The domain is [-1, 00).<br />

Range: y can take on any value. so the range is<br />

(-00,00).<br />

Some x-values give more than one y-value. For<br />

example, if x = 3, y = 2 or y = -2.<br />

So x + 1 = y2 is not a function.<br />

17. Domain: Since the values ofx are -5 S x S 5, the<br />

domain is [-5.5]. Range: Since the values ofyare<br />

-3 S y S 3. the range is [-3.3].<br />

Since there are values of x that correspond to<br />

more than 1 y-value, this is not a function. .<br />

18. Domain: Since any 'value of x would have a point<br />

on the graph. the domain is (-00, 00).<br />

Range: Since any point on this graph has a<br />

y-value greater than or equal to 1. the range is<br />

[1. 00).<br />

Each x-value gives one y-value, so this is a<br />

function.<br />

19. -51° is coterminal with -51° + 360° =309°.<br />

20. -174° is coterminal with 360° + (-174°) = 186°.<br />

21. 792° is coterminal with 792° - 2(360°) =72°.<br />

22. Let n represent any integer. Any angle coterminal<br />

with 270° would be 270° + n . 360°.<br />

23. 320 rotations per minute<br />

=320(360°) per minute<br />

=115.200° per minute<br />

115.200°<br />

= per second<br />

60<br />

= 1920° per second<br />

= 1920 0 ( ~) per ~second<br />

2<br />

= 1280° per - second<br />

3<br />

34


<strong>Trigonometry</strong> <strong>Solutions</strong> <strong>Manual</strong><br />

Chapter 1 Review Exercises 35<br />

24. 650 times per minute<br />

=- 650 =10. 83 times ' per secon d<br />

60<br />

In 2.4 seconds, a point rotates<br />

(2.4)(10.83) = 26 times.<br />

The point rotates 26· 360° = 9360°.<br />

25.<br />

47025'11" = 470 + 25° +...!..!.:..­<br />

60 3600<br />

= 47° + .417° + .003°<br />

=47.420°<br />

° 3 0<br />

26. 119°8'3"=119°+!..+-­<br />

60 3600<br />

= 119.134°<br />

27. -61.5034° = -61° - (.5034)(60')<br />

= -61° - 30.204'<br />

= -61°30' -(.204)(60'')<br />

= -61°30'12"<br />

28. 275 .1005°=275°+(.1005)(60')<br />

= 275° + 6.03'<br />

= 275°6' + (.03)(60")<br />

= 275°6'2"<br />

29. The angles (9x + 4)° and (l2x - 14)° are vertical<br />

angles, which have equal measure.<br />

9x+4 =12x-4<br />

4 =3x-14<br />

18=3x<br />

6=x<br />

The angles are 58° and 58°.<br />

30. The sum of the measures of the angles of a<br />

triangle is 180°.<br />

8x+6x+4x = 180<br />

18x = 180<br />

x=lO<br />

8x = 8(10) = 80<br />

6x =6(10) =60<br />

4x =4(10) =40<br />

The angles are 40°, 60°, and 80°.<br />

31. Since a line has 180°, the angle supplementary to<br />

/3 is 180° - /3. The sum of the angles of a triangle<br />

is 180°, so<br />

O+a+(l80° -,8) = 180°<br />

0= 180-a-1800+ /3<br />

O=-a+/3<br />

O=/3-a<br />

32. Assuming PQ and BA are parallel, 6.PCQ is<br />

similar to MCB since LPCQ = LACB. Then<br />

PQ PC<br />

-=­<br />

BA AC<br />

1.25 mm 150 mm<br />

---=--­<br />

BA 30 Ian<br />

BA(l50 mm) = (30 1an)(1.25 mm)<br />

BA = (30 Ian)(1.25)<br />

150<br />

=0.25 Ian<br />

Hint for Exercises 33-38: In similar triangles,<br />

corresponding sides are in proportion and corresponding<br />

angles are equal.<br />

33. V=X=41°<br />

z= T= 32°<br />

The sum of the angles in a triangle is 180° so<br />

Y = U = 180° - (41° + 32°) = 107°.<br />

34. Q =N = 12°<br />

R = P = 82°<br />

M = S = 86°<br />

75 3<br />

35. -=­<br />

50 2<br />

m 3<br />

-=­<br />

30 2<br />

2m=90<br />

m=45<br />

n 3<br />

-=­<br />

40 2<br />

2n = 120<br />

n=60<br />

36. Since the large triangle is equilateral, the small<br />

triangle is also equilateral.<br />

p=q=7<br />

6 7<br />

37. -=-­<br />

r 11+7<br />

6 7<br />

-=r<br />

18<br />

7r = 108<br />

108<br />

r=­<br />

7<br />

k 12+9<br />

38. -=-­<br />

6 9<br />

k 21<br />

-=­<br />

6 9<br />

. .9k = 126<br />

. k = 14<br />

35


<strong>Trigonometry</strong> <strong>Solutions</strong> <strong>Manual</strong><br />

36 Chapter 1 The Trigonometric Functions<br />

39. Iftwo triangles are similar. their corresponding 42. x = 1, y =-J3<br />

sides are proportional and their corresponding<br />

angles are equal.<br />

r=~x2+l<br />

40. Let x =the shadow of the 30-ft tree. = ~12 + (_$)2<br />

30ft<br />

=.Jl+3<br />

=.J4<br />

=2<br />

. y -J3 $<br />

smy=-=--=-­<br />

r 2 · 2<br />

x 1<br />

cosy=-=­<br />

8 ft r 2<br />

20 30<br />

-=- tan y =1..= -.J3 =_$<br />

8 x x 1<br />

20x =240 x 1 $<br />

x=12 coty=-=--=-­<br />

y -.J3 3<br />

The shadow of the 30-ft tree would be 12 ft.<br />

r 2<br />

41. x = -3. y =-3 secy=-=-=2<br />

x 1<br />

r=~x2 +l<br />

r 2 2$<br />

=~(_3)2+(_3)2 cscy =y=-$ =--3­<br />

= ./9+9<br />

=..ff8<br />

43. sin 180° = 0<br />

cos 180° =-1<br />

=3-J2 tan 180° = 0<br />

. y 3 1 .J2 cot 180° is undefined.<br />

smO=-=---=--=-­<br />

r 3-J2 .J2 2 sec 180° =-1<br />

3 1 .J2<br />

esc 180° is undefined.<br />

cosO = xr =--- =--=-­<br />

3.fi .J2 2 44. 360° is coterminal with 0°.<br />

tan 0 =1..= -3 =1<br />

sin 360=0<br />

x -3 cos 360° = 1<br />

x -3 tan 360° = 0<br />

cotO=-=-=1<br />

y -3<br />

cot 360° is undefined.<br />

sec 360° = 1<br />

esc 360° is undefined.<br />

sec0 = !..= 3.fi =-.J2<br />

x -3<br />

esc 0 =!..=3-J2 =-J2<br />

y -3<br />

36


<strong>Trigonometry</strong> <strong>Solutions</strong> <strong>Manual</strong><br />

Chapter I Review Exercises 37<br />

45. (-8, 15), x = -8, y = 15<br />

r = ~(_8)2 + 15 2<br />

=.../64+225<br />

=.../289<br />

=17<br />

sinO=I=~<br />

r 17<br />

x 8<br />

cosO=-=-­<br />

r 17<br />

tanO=I=~=_~<br />

x -8 8<br />

x 8<br />

cotO=-=-­<br />

y 15<br />

r 17 17<br />

seeO=-=-=-­<br />

x -8 8<br />

r 17<br />

cscO=-=y<br />

15<br />

46. (3, -4), x =3, y =-4<br />

r = ~32 + (-4)2<br />

=.../9+16<br />

=~<br />

=5<br />

sinO=I= -4 =_i<br />

r 5 5<br />

x 3<br />

cosO=-=r<br />

5<br />

tanO=I= -4 =_i<br />

x 3 3<br />

x 3 3<br />

cotO=-=-=-­<br />

y -4 4<br />

r 5<br />

seeO=-=x<br />

3<br />

r 5 5<br />

cscO=-=-=-­<br />

y -4 4<br />

47. (1,-5), x = 1, y =-5<br />

r = ~(1)2 +(_5)2<br />

=~<br />

. y -5 5../26<br />

SlnO=-=-=--­<br />

r ~ 26<br />

x 1 ../26<br />

cosO=-=--=-­<br />

r ~ 26<br />

tanO=I= -5 =-5<br />

x 1<br />

xII<br />

cotO=-=-= -­<br />

y -5 5<br />

seeO=!.= ~ =../26<br />

x 1<br />

r ~ ../26<br />

cscO=-=-=--­<br />

y -5 5<br />

48. (9,-2), x =9, y =-2<br />

r =~(9)2 + (_2)2<br />

=.../81+4<br />

=../85<br />

. y -2 2~<br />

SlnO=;= ../85 =-85<br />

x 9 9~<br />

cosO=-=-=-­<br />

r ../85 85<br />

tanO=I= -2 =_~<br />

x 9 9<br />

x 9 9<br />

cot 0 =­ =­ =-­<br />

y -2 2<br />

r ../85<br />

seeO=-=x<br />

9<br />

r../85 ~<br />

cscO=-=-=--­<br />

y -2 2<br />

37


<strong>Trigonometry</strong> <strong>Solutions</strong> <strong>Manual</strong><br />

38 Chapter 1 The Trigonometric Functions<br />

49. (Wi,-6),x=6$,y=-6<br />

r = ~(6$)2 +(-6)2<br />

=..J108+36<br />

51. Since the terminal side of the angle is defined by<br />

5x - 3y = 0, x :2: 0, a point on this terminal side<br />

would be (3, 5) since 5(3) - 3(5) :=0.<br />

r= ~32 +5 2 =..J9 + 25 =-134<br />

. y 5 5-J34<br />

sm9=-=--=-­<br />

=..J144<br />

r -134 34<br />

=12<br />

x 3 3-134<br />

sin 9 = I = - ~ = _.!. cos9 = -;:= -134 =34<br />

r 12 2<br />

x 6$ $ y 5<br />

cos9=-=--=­<br />

r 12 2 x 3<br />

tan9=-=­<br />

x 3<br />

tan9=I= __6_= __I_=_ $<br />

cot9=-=y<br />

5<br />

x Wi ..[3 3<br />

r -134<br />

sec9=-=-­<br />

cot9=~= Wi =-./3 x 3<br />

y -6<br />

r .J34<br />

r 12 2 2..[3 csc9=-=-­<br />

sec9 = -:;= Wi = ..[3=-3- y 5<br />

r 12 52. If the terminal side of a quadrantal angle lies<br />

csc9=-=-=-2<br />

y -6 along the y-axis, a point on the terminal side<br />

would be of the form (0, k), where k is a real<br />

number, k*O.<br />

50. (-2Ji, 2Ji), x =-2Ji,y =2Ji<br />

r = ~(_2Ji)2 + (2Ji)2<br />

sin9:=I:= ~<br />

r r<br />

cos 9 = -:= x -:= ° 0<br />

=..J8+8 r r<br />

:=...[16<br />

tan 9 = I :=~ undefined<br />

=4 x 0'<br />

. y 2Ji -Ii x 0<br />

sm 9 =- :=--=- cot9:=-= - = 0<br />

r 4 ,2 y k<br />

x -2Ji .J2 r r<br />

cos9=-=--=-- sec 9 = - = - undefined<br />

r 4 2 x 0'<br />

tan9=I= 2~ =-1<br />

x -2...,2<br />

x -2Ji<br />

cot9=y= 2Ji =-1<br />

sec9=!.-=_4_= -2 =-..J2<br />

53.<br />

esc 9 :=!.-:=!.­<br />

y k<br />

The tangent and secant are undefined.<br />

(-I, 5)<br />

x -2Ji Ji y=-5x.x~O<br />

r 4 t:<br />

esc 9 = y:= 2Ji :=...,2<br />

y<br />

9<br />

54. Using the point (-1, 5),<br />

r:= ~(_1)2 + (5)2 :=..J1 + 25 =.../26.<br />

sin 9 :=5.../26<br />

26<br />

.../26<br />

cos9=--­<br />

26<br />

38


<strong>Trigonometry</strong> <strong>Solutions</strong> <strong>Manual</strong><br />

Chapter 1 Review Exercises 39<br />

5<br />

55. tan 0= -=-5<br />

-1<br />

56.<br />

4 sec 180 0-2sin2 270 0=4(-1)-2(-1)2<br />

= -4-2(1)<br />

=-4-2<br />

=-6<br />

57. -cot 2 90 0 + 4sin 270 0 - 3 tan 180°<br />

= -(0)2 + 4(-1) - 3(0)<br />

=0-4-0<br />

=-4<br />

59. From the screen, cos X = .1.<br />

sin 2 X +cos 2 X = 1<br />

sin 2 X = 1-(.1)2<br />

sin 2 X = .99<br />

2 2 sin 2 X<br />

tan X=(tanX) =--2­<br />

cos X<br />

(tan<br />

X)2<br />

=-=99<br />

.99<br />

.01<br />

60. sin 0 ;;;; ~ is possible because -1 s sin 6 s 1 for all<br />

4<br />

6. When sin 0 ;;;; ~ then esc = i because<br />

4 3<br />

1<br />

csc(J=--.<br />

sinO<br />

sin (J ;;;; ~ and esc 0 = i is possible.<br />

4 3<br />

61. For any angle 0, sec (J ~ -lor sec 0 ~ l.<br />

Therefore, sec 0 = - ~ is impossible .<br />

3<br />

1<br />

62. Ifcos 6 = .25, sec 0 = - = 4, so cos 6 = .25 and<br />

.25<br />

sec 6 = -4 is impossible.<br />

63. tan (J can take on all values.<br />

tan (J = 1.4 is possible.<br />

500 sin 50°<br />

64. tan =--­<br />

cos50°<br />

Since cos 50° < I, sin 500+ cos 50° > sin 500.<br />

Therefore tan50 0 > sin 500.<br />

65. In quadrant I as (J increases, sin (J increases .<br />

Therefore sin 44° > sin43°.<br />

66. Sin<br />

·0.J3· = -, Sin 0 = ­Y<br />

5 r<br />

x 2 + y2 = r2<br />

x 2+(.J3)2=52<br />

x 2 +3=25<br />

x 2 =25-3<br />

so y = .J3 and r = 5.<br />

x 2 =22<br />

x=±..fii<br />

cos 6 < 0 means x < 0 so x = -J22.<br />

sin 0 = 1.;;;; .J3<br />

r 5<br />

x ..fii<br />

cosO=-=--­<br />

r 5<br />

y .J3 .J66<br />

tanO=-=--=--­<br />

x -fi2 22<br />

x ..fii .J66<br />

cotO=-=---=--­<br />

y .J3 3<br />

r 5 s..fii<br />

secO=-=--=--­<br />

x -J22 22<br />

r 5 5.J3<br />

cscO=-=-=-­<br />

y .J3 3<br />

67. cosy = _., with rin quadrant ill.<br />

8<br />

x<br />

cosr=-, sox=-5andr=8.<br />

r<br />

x 2 +i =r 2<br />

(_5)2 + i = (8)2<br />

25+ y2 =64<br />

i=39<br />

y=±.,[39<br />

Since r is in quadrant III, y = -.,[39.<br />

. y -.,[39 .J39<br />

slnr=-=--=--­<br />

r 8 8<br />

x 5<br />

cosr=-=-­<br />

r 8<br />

y -.,[39 .J39<br />

tanr=-=--=-­<br />

x -5 5<br />

x -5 5.J39<br />

cotr=-=--;;;;-­<br />

y -.,[39 39<br />

r 8 8<br />

secr=-=-=-­<br />

x -5 5<br />

r 8 8.,[39<br />

cscr=-=--=--­<br />

y -.,[39 39<br />

39


<strong>Trigonometry</strong> <strong>Solutions</strong> <strong>Manual</strong><br />

40 Chapter 1 The Trigonometric Functions<br />

68. tan a = 2 = ~, tana = L with a in quadrant III,<br />

1 x<br />

70. sin0 = - ~, sin 0 = I with () in quadrant Ill, so<br />

5 ,<br />

so let x = -1 and y = -2. let y = -2 and, = 5.<br />

x2 + y2 = ,2 , x2 + y2 =,2<br />

(-Ii + (_2)2 = ,2 x 2 +(_2)2 = 52<br />

1+ 4 = ,2, r = .J5 x 2 +4 =25<br />

. y 2 2.J5<br />

sma = -;:= - .J5 = --5­<br />

x 1 .J5<br />

cosa=-=--=-­<br />

r .J5 5<br />

y -2<br />

tana=-=-=2<br />

x -1<br />

x -1 1<br />

cota=-=-=y<br />

-2 2<br />

seca =!.. = .J5 =-15<br />

x -1<br />

r .J5 .J5<br />

csca=-=-=-­<br />

y -2 2<br />

69. sec /3 = -.J5, with /3 in quadrant II.<br />

see/3 = !.., so ,=.J5 andx= -l.<br />

x<br />

x2 + y2 =,2<br />

(_1)2 + i =(.J5)2<br />

x 2 =21<br />

x =±.fij<br />

Since x is negative in quadrant III, x = -J2j.<br />

. y -2 2<br />

sm 0 =- =­ =-­<br />

,5 5<br />

x -J2j .fij<br />

cosO = ­ = -­=--­<br />

r 5 5<br />

y - 2 2.fij<br />

tan0 = ­ = -­=-­<br />

x -J2j 21<br />

x ~ .fij<br />

cot 0 =­ = -­=-­<br />

y -2 2<br />

r 5 5.fij<br />

sec8=-=--=--­<br />

x -J2j 21<br />

r 5 5<br />

csc 8 = ­ = ­ = -­<br />

y -2 2<br />

71. seca =~, with a in quadrant IV. sec a =!.., so<br />

4 x<br />

,=5 andx= 4.<br />

l+i =5 x2 + y2 =,2<br />

y2 =4 4 2 +y2=5 2<br />

y=±2<br />

16+i =25<br />

Since f3 is in quadrant II, y = 2.<br />

. y 2 2.J5<br />

sm /3 = -;:= .J5 = -5­<br />

x -1 .J5<br />

cos/3=-=-=-­<br />

r.J5 5<br />

tan/3 = L = l:.... =-2<br />

x -1<br />

x -1 1<br />

cot/3=-=-=-­<br />

y 2 2<br />

see /3 = !.. = .J5 =-15<br />

x -1<br />

,.J5<br />

csc/3=-=­<br />

,., y 2<br />

i=9<br />

y=±3<br />

Since a is in quadrant IV, y =-3.<br />

. y -3 3<br />

sma=-=-=-­<br />

,55<br />

x 4<br />

cosa=-=­<br />

, 5<br />

y -3 3<br />

tana=-=-=-­<br />

x 4 4<br />

x 4 4<br />

cot a = ­ = ­ = -­<br />

y -3 3<br />

, 5<br />

seca=-=x<br />

4<br />

,55<br />

csca=-=-=-­<br />

y -3 3<br />

40


<strong>Trigonometry</strong> <strong>Solutions</strong> <strong>Manual</strong><br />

Chapter 1 Test Exercises 41<br />

72. The sine is negative in quadrants III and IV. The<br />

cosine is positive in quadrants I and IV. Since<br />

sin e< 0 and cos e> 0, elies in quadrant IV.<br />

sinO .<br />

Since tan 0 =-- and SIn () < 0 and cos ()> 0,<br />

cosO<br />

the sign of tan ()is negative (a negative divided<br />

by a positive is a negative).<br />

(b) x 2 2: 0, so f(x) 2:-2 . The range is [-2,00).<br />

(c) f(-2) = .5(-2)2 -2 = 0<br />

360° 9<br />

74. ---= - degrees I yr<br />

26,000 yr 650<br />

9<br />

= - (3600) sec I yr<br />

650<br />

648<br />

=- sec/yr<br />

13<br />

'" 50 sec I yr<br />

The celestial north pole moves about 50" each<br />

year.<br />

75, Let x bethe depth of the crater Autolycus. Then ,<br />

x 1.3<br />

--=­<br />

11,000 1.5<br />

1.5x = 1.3(11,000)<br />

14,300<br />

x=--­<br />

1.5<br />

x ",9500<br />

The depth of the crater Autolycus is about<br />

9500 ft.<br />

76. Let x be the height of Bradley. Then,<br />

x 1.8<br />

--=­<br />

21,000 2.8<br />

2.8x = 1.8(21,000)<br />

37,800<br />

2.8<br />

x = 13,500<br />

Bradley is 13,500 ft tall.<br />

Chapter 1 Test Exercises<br />

1. d=~(5-2)2+(-2-(-5»2<br />

=~32 +3 2<br />

= ../9+9<br />

=.Jf8<br />

=3../2<br />

2. (6400km + 150krn)2 = d 2 + (64ookrn)2<br />

6550 2 -6400 2 =d 2<br />

1400'" d<br />

The distance is about 1400 krn.<br />

3. (a) x can be any value, so the domain is<br />

(-00,00) .<br />

rev 1min 7 5 rev<br />

6. 450-·--= . ­<br />

min 60sec sec<br />

7.5 rev = 7.5(360°) = 2700°<br />

The point on the edge of the tire moves 2700° in<br />

one second.<br />

7. The sum of the angles of a triangle is 180°.<br />

60°+ y+90 0 = 180°<br />

y = 180°-60°_90°<br />

y=30°<br />

The angle supplementary to 60° is 120° .<br />

x + 120°+30° = 180°<br />

x = 180° -120° - 30°<br />

x=30°<br />

8. Since we have two similar triangles,<br />

20 x<br />

-=<br />

30 roo-.,<br />

20(100- x) = 30x<br />

2000 - 20x =30x<br />

50x=2000<br />

x=40<br />

x=--­The lifeguard will enter the water 40 yards east of<br />

his original position.<br />

9. (2,-5), x = 2,y =-5<br />

r=~x2+i<br />

r=~22 +(_5)2<br />

r=..fi9<br />

y -5..fi9 5..fi9<br />

sinO=;= ..fi9 '129=-29"<br />

x 2 .J29 2..J29<br />

cos 0 =; = .J29'.J29 = 29"<br />

y -5 5<br />

tan 0 =-=-=-­<br />

x 2 2<br />

10. If cos 0 < 0, then 0 is in quadrant II or III. If<br />

cot 0 > 0 , then 0 is in quadrant I or III. Therefore<br />

o terminates in quadrant III.<br />

41


<strong>Trigonometry</strong> <strong>Solutions</strong> <strong>Manual</strong><br />

42 Chapter 1 The Trigonometric Functions<br />

4 x<br />

11. If cos0 = - = - , let x = 4 and r = 5.<br />

5 r<br />

2<br />

x 2 +i =r<br />

4 2 +i =5 2<br />

i =25-16<br />

y = ±../9<br />

y=±3<br />

Since 0 is in quadrant IV, y is negative, so<br />

y=-3.<br />

. y -3 3<br />

smO=-=-=-­<br />

r 5 5<br />

y -3 3<br />

tanO=-=-=-­<br />

x 4 4<br />

x 4 4<br />

cotO=-=-=-­<br />

y -3 3<br />

r 5<br />

secO=-=x<br />

4<br />

r 5 5<br />

cscO=-=-=-­<br />

y -3 3<br />

12. No; when sin 0 and cos 0 are both negative for<br />

sin O . . .<br />

th e same va I ue 0 f 0 , tan 0 =-­ IS posrtrve.<br />

cosO<br />

42


<strong>Trigonometry</strong> <strong>Solutions</strong> <strong>Manual</strong><br />

2.1 Trigonometric Functions of Acute Angles 43<br />

CHAPTER 2 ACUTE ANGLFS AND . side opposite 45<br />

8. smA= :=;­<br />

RIGHT TRIANGLFS hypotenuse 53<br />

Section 2.1<br />

side adjacent 28<br />

cosA= =­<br />

hypotenuse 53<br />

Exercises side opposite 45<br />

tan A = =­<br />

side adjacent 28<br />

For Exercises 1-6, refer to the Special Angles chart.<br />

cot A :=; side adjacent = 28<br />

1.,C"sin 30° = .!. 2 side opposite 45<br />

sec A = hypotenuse = 53<br />

side adjacent 28<br />

hypotenuse 53<br />

csc A= =­<br />

side opposite 45<br />

. side opposite n<br />

1 1 9. smA= =­<br />

4. G" sec 60° = = - = 2<br />

hypotenuse p<br />

, cos 60° 1­<br />

2 side adjacent m<br />

cosA= =­<br />

hypotenuse p<br />

1 1 2 ..[3 2..[3 side opposite n<br />

5. E; esc 60° = sin 600 = ..{j = ..[3 = "J3 = -3- tan A = =­<br />

side adjacent m<br />

2<br />

side adjacent m<br />

cot A = =­<br />

side opposite n<br />

sec A = hypotenuse =.E..<br />

side adjacent ' m<br />

hypotenuse p<br />

esc A = =­<br />

7. sin A = side opposite = E.. side opposite n<br />

hypotenuse 29<br />

cos A = side adjacent = 20<br />

10.<br />

sin A = side opposite = !<br />

hypotenuse 29 hypotenuse z<br />

side opposite 21 side adjacent y<br />

tan A = =­ cos A = =­<br />

side adjacent 20 hypotenuse z<br />

side adjacent 20 side opposite k<br />

cot A = =­ tan A = =­<br />

side opposite 21 side adjacent y<br />

hypotenuse 29 side adjacent y<br />

secA= =- cot A = =­<br />

side adjacent . 20 side opposite k<br />

hypotenuse 29<br />

csc A=<br />

hypotenuse z<br />

=­ secA= =­<br />

side opposite 21 side adjacent y<br />

hypotenuse z<br />

csc A = =­<br />

side opposite k<br />

11. a =5, b =12<br />

c 2 =a 2 +b 2<br />

c 2 =5 2 +12 2<br />

c 2 =25+144<br />

2<br />

c = 169<br />

c =13<br />

43


<strong>Trigonometry</strong> <strong>Solutions</strong> <strong>Manual</strong><br />

44 Chapter 2 Acute Angles and Right Triangles<br />

. side opposite 12 . side opposite .Jl3<br />

Sin B= =- Sin B= =-­<br />

hypotenuse 13 hypolenuse 7<br />

side adjacent 5 side adjacent 6<br />

cosB= =- cosB= =­<br />

hypotenuse 13 hypotenuse 7<br />

side opposite 12<br />

tan B = =- side opposite .Jl3<br />

side adjacent 5<br />

tanB= =-­<br />

side adjacent 6<br />

side adjacent 5<br />

cotB= =- cot B = side adjacent = ~ = 6.Jl3<br />

side opposite 12<br />

side opposite .Jl3 13<br />

hypotenuse 13<br />

seeB= =- sec B<br />

side adjacent 5<br />

= hypolenuse = 7..<br />

side adjacent 6<br />

hypotenuse 13<br />

cscB= =­<br />

side opposite 12<br />

hypotenuse 7 7.Jl3<br />

cscB= =--=-­<br />

side opposite .Jl3 13<br />

12. a =3, b =5<br />

2 14. b e T, c=12<br />

c = a 2 +b 2<br />

2<br />

2 c =a 2 +b 2<br />

+7<br />

c 2 =9+25<br />

c =32 +5 2 12 2 =a 2 2<br />

c 2 =34<br />

144=a 2 + 49<br />

c =..[34 a 2 =95<br />

. B side opposite 5 5..[34<br />

a=.J95<br />

sinB=£'=2..<br />

Sin = hypotenuse = ..[34 = 34"" c 12<br />

B side adjacent 3 3.J34 a .J95<br />

cosB=-=-­<br />

cos = hypotenuse =""J34 = 34"" c 12<br />

tan B = side opposite = ~ b 7 7../95<br />

side adjacent 3<br />

tan B=-;; =.J95 =.gs­<br />

side adjacent 3<br />

cotB= =- a .J95<br />

side opposite 5 cotB=-=-­<br />

b 7<br />

sec B = hypotenuse =.J34<br />

secB=~=~= 12../95<br />

side adjacent 3 a.J95 95<br />

esc B = hypotenuse =.J34<br />

c 12<br />

cscB=-=side<br />

opposite 5 b 7<br />

13. a=6, c=7 15. sin A = cos (90° - A)<br />

2 cos A = sin (90° - A)<br />

c =a 2 +b 2 tan A = cot (90° - A)<br />

7 2 =6 2 +b 2 cot A = tan (90° - A)<br />

49 =36+b 2 sec A = esc (90° - A)<br />

13=b 2 esc A = sec (90° - A)<br />

.Jl3 =b<br />

16. cot 73° = tan (90° -73°)<br />

= tan 17°<br />

17. see 39° = esc (90° - 39°)<br />

= csc51°<br />

18. cos (a + 20°)<br />

= sin [90°- (a + 20°)]<br />

= sin (70° - a)<br />

44


<strong>Trigonometry</strong> <strong>Solutions</strong> <strong>Manual</strong><br />

2.1 Trigonometric Functions of Acute Angles 45<br />

19. cot (/3_10°) 28. cot (59 + 2°) = tan (29 + 4°)<br />

= tan [90° - (/3 _10°)] Since cotangent and tangent are cofunctions,<br />

= tan (100° - /3)<br />

(59 + 2°) + (29 + 4°) = 90°<br />

79+6° = 90°<br />

20. tan 25.4° = cot (90° - 25.4°) 79 = 84°<br />

= cot 64.6° 9 = 12°.<br />

21. sin 38.7° = cos (90° - 38.7°) 29. tan 28° :5 tan 40°<br />

= cos51.3° tan 9 increases as 9 increases from 0° to 90°.<br />

Since<br />

22. A sin(900-A) cosA 40° > 28°<br />

--~-------+-------<br />

30° sin 60° = .8660 cos 30° = .8660<br />

tan 40° > tan 28°.<br />

Therefore, the given statement is true.<br />

45° sin 45° = .7071 cos 45° = .707 1<br />

60° sin 30° =.5 cos 60° =.5 30. sin 50° > sin 40°<br />

In the interval from 0° to 90°, as the angle<br />

-60° sin 150° = .5 cos(-67°) = .5<br />

increases, so does the sine of the angle, so<br />

120° sin(-300) = -.5 cos 120° = - .5 sin 50° > sin 40° is true.<br />

In each case, sin (90° -A) = cosA. 31. sin 46° < cos 46°<br />

sin 9 increases as 9 increases from 0° to 90° .<br />

23. tan a = cot (a + 10°) Since<br />

Since tangent and cotangent are cofunctions, this 46° > 44°<br />

equation is true if the sum of the angles is 90°.<br />

sin 46° > sin 44°.<br />

a+(a+100) =90° But sin 44° = cos 46°,<br />

2a+ 10° =90° so sin 46° > cos 46° .<br />

2a =80°<br />

False<br />

a=40°<br />

32. cos 28° < sin 28°<br />

24. cos 9 = sin 29<br />

cos 45° = sin (90° - 45°)<br />

Since sine and cosine are cofunctions, this<br />

= sin 45°<br />

equation is true if the sum of the angles is 90°, so<br />

In the interval from 0° to045°, the sine increases<br />

9+29 = 90°<br />

from 0 to sin 45° = .J2 , while the cosine<br />

39=90° 2<br />

9 = 30°. decreases from 1 to<br />

25. sin (2r + 10°) = cos (3r - 20°) cos45°=-.<br />

2<br />

Since sine and cosine are cofunctions,<br />

Therefore, cos 28° > sin 28°, and<br />

(2r + 10°) + (3r - 20°) = 90° cos 28° < sin 28° is false .<br />

5r-lO° =90°<br />

5r = 100° 33. tan 41° < cot 41°<br />

r=20°.<br />

tan 9 increases as 9 increases from 0° to 90°.<br />

Since<br />

26. sec (/3+ 10°) = esc (2/3+ 20°)<br />

49° > 41°<br />

tan 49° > tan 41 0.<br />

Since secant and cosecant are cofunctions,<br />

But<br />

(/3 + 10°) + (2/3 + 20°) = 90°<br />

.... tan 49° = cot 41°,<br />

3/3+ 30° = 90° so<br />

3/3=60° cot 41° > tan 41 0.<br />

/3= 20°. True<br />

27. tan (3B + 4°) =cot (5B - 10°)<br />

Since tangent and cotangent are cofunctions,<br />

(3B + 4°) + (5B _10°) = 90°<br />

8B-6°=90°<br />

8B = 96°<br />

B = 12°.<br />

.J2<br />

45


<strong>Trigonometry</strong> <strong>Solutions</strong> <strong>Manual</strong><br />

46 Chapter 2 Acute Angles and Right Triangles<br />

34. cot 30° < tan 40° hypotenuse<br />

tan (90° - 30°) =tan 60° =cot 30° csc 45 0 = side opposite<br />

In the interval from 0° to 90°, the tangent<br />

increases so .J2<br />

=­<br />

tan 60° > tan 40°. Therefore,<br />

1<br />

cot 30° > tan 40°, so<br />

=.J2<br />

cot 30° < tan 40° is false.<br />

40.<br />

35.<br />

I~"<br />

~l<br />

~ ~<br />

..J3 1<br />

0 side opposite sec 450 = hypotenuse<br />

tan 30 = side adjacent<br />

side adjacent<br />

1 .J2<br />

=.,[3<br />

=­<br />

1<br />

.,[3<br />

=-<br />

=.J2<br />

3<br />

41. See figure in Exercise 35.<br />

36. . 600 side opposite<br />

sm<br />

= hypotenuse<br />

.,[3<br />

=­<br />

2<br />

42. See the figure in Exercise 36.<br />

side adjacent<br />

cos 60 0 = hypotenuse<br />

side adjacent<br />

cot 30 0 = 1<br />

side opposite<br />

=­<br />

2<br />

=­<br />

.,[3<br />

1<br />

=.,[3<br />

37. See the figure in Exercise 35.<br />

. 300 side opposite<br />

sm = hypotenuse<br />

1<br />

=­<br />

2<br />

38. See the figure in Exercise 36. 44.<br />

side adjacent<br />

cos 300 = hypotenuse<br />

.,[3<br />

=­<br />

2<br />

43.<br />

39.<br />

LJ5.<br />

.J2 1<br />

45·<br />

1<br />

46


<strong>Trigonometry</strong> <strong>Solutions</strong> <strong>Manual</strong><br />

2.1 Trigonometric Functions of Acute Angles 47<br />

sin45°=! cos45° =~<br />

4 4<br />

Y = 4sin45° x = 4 cos 45°<br />

=4 . ...[2<br />

...[2<br />

x=4'­<br />

(.70710678, .70710678). This is the point<br />

2 2<br />

=2...[2<br />

=2...[2<br />

45. The legs of the right triangle provide the<br />

coordinates of P.<br />

P is (2...[2, 2...[2).<br />

46. Steps 1 & 2:<br />

y<br />

--1f----f........o...--t-- ..<br />

2<br />

51. Graph Y 1 = x and Y 2 = ~1- x in a window such<br />

as -2 s x s 2, -2 s y s 2. Use the intersection<br />

function to see that the point of intersection is<br />

(~,~}<br />

'," ' ,.<br />

'.<br />

'1',,' t ,<br />

-2 :: :"'1~'"<br />

· : r ~~~ ....<br />

.':<br />

H=.70710&71 '=.70710&71 '<br />

-2<br />

These coordinates are the sine and cosine of 45°.<br />

:; ~ ' , -. . . - ..<br />

IlIt4:rS4:ctitrl :' ''''' ~.'< '<br />

52. y<br />

Step 3: sin 60 0=! 2<br />

cos 600=~<br />

2<br />

y= 2 sin 60° x = 2cos 60°<br />

.J3<br />

1<br />

=2·­ =2·­<br />

2<br />

2<br />

=1<br />

=.J3<br />

(I, {3)<br />

-----+lllI:-..r.....&-_....x<br />

Thus the coordinates of P are (1,.J3).<br />

47. sin 0° = 0<br />

sin 45° = .70711<br />

sin 90° = 1<br />

tanOo=O<br />

tan 45° = 1<br />

tan 90° is undefined.<br />

Y t is sin x and Y 2 is tan x.<br />

48. cos 0° = 1 csc O is undefined<br />

cos600=.5 csc600=1.1547<br />

cos 90° = 0 esc 90° = 1<br />

Y 1 is cos x and Y2 is esc x.<br />

The line passes through (0,0) and (1, .J3). The<br />

slope is .J3. The equation of the line is y = .J3x.<br />

53. y<br />

-----~.....;;.;.........I-\"....x<br />

49. sin 60° = .8660 for A between 0° and 90° .<br />

Therefore, a = 60°.<br />

The line passes through (0, 0) and (.J3, 1). The<br />

slope is .J3. The equation of the line is<br />

3<br />

.J3<br />

y=-x.<br />

3<br />

47


<strong>Trigonometry</strong> <strong>Solutions</strong> <strong>Manual</strong><br />

48 Chapter 2 Acute Angles and Right Triangles<br />

54. The slope of a line is the change in y over the<br />

c hanze ange i mx, or tan 6 . S' mce y=-x ../3 ,<br />

3<br />

../3 . ../3<br />

m = - = tan 6. Since tan 30° = -,<br />

3 3<br />

the line y = ../3 x makes a 30° angle with the<br />

3<br />

positive x-axis. (See Exercise 53 .)<br />

55. The slope of a line is the change in y over the<br />

change in x, or tan 6 . Since y = ../3x,<br />

m = ../3 = tan 6 . Since tan 60° =../3, the<br />

line y = ../3x makes a 60° angle with the positive<br />

x-axis, (See Exercise 52.)<br />

1<br />

56. tan 6=-­<br />

cot 6<br />

tan 6 = cot(90° - 6)<br />

Tangent and cotangent are both reciprocals and<br />

cofunctions.<br />

57. A<br />

(a) Each angle of every equilateral triangle has a<br />

measure of 60°.<br />

(b) The perpendicular bisects the opposite side,<br />

so the length of each side opposite each 30°<br />

angle is k.<br />

(c) Let x equal the length of the perpendicular<br />

and apply the Pythagorean theorem.<br />

x 2 + k 2 =(2k)2<br />

2+k2=4k<br />

x<br />

2<br />

x 2 =3k 2<br />

x =k.../3<br />

The length of the perpendicular is k../3.<br />

(d) In a 30° - 60° right triangle, the hypotenuse<br />

is always 2 times as long as the shorter leg,<br />

58. k<br />

and the longer leg has a length that is ../3<br />

times as long as that of the shorter leg. Also,<br />

the shorter leg is opposite the 30° angle, and<br />

the longer leg is opposite the 60° angle.<br />

{Zl<br />

k<br />

(a) The diagonal forms two isosceles right<br />

triangles. Each angle formed by a side of the<br />

square and the diagonal measures 45°.<br />

2<br />

(b) k 2 +k 2 = c<br />

2<br />

2k 2 =c<br />

~2k2 =c<br />

k..fi = c<br />

(c)<br />

The length of the diagonal is k..fi.<br />

In a 45° - 45° right triangle, the hypotenuse<br />

has a length that is..fi times as long as either<br />

leg.<br />

1 .<br />

59. a = -(24), so a = 12.<br />

2<br />

b = a../3, so b = 12../3.<br />

d = b, so d = 12../3.<br />

c = d..fi, so c = (12../3X:J2) = 12.J6.<br />

1 9<br />

60. y=-(9), soy=-.<br />

2 . 2<br />

x = y../3, so x = 2-.J3.<br />

2<br />

y =z../3, so<br />

y t 9../3 3../3<br />

z= ../3= ../3=-6-=2'<br />

w =2z , so W =2( 3~J = 3.../3.<br />

48


<strong>Trigonometry</strong> <strong>Solutions</strong> <strong>Manual</strong><br />

2.2 Trigonometric Functions of Non-Acute Angles 49<br />

7 7../3<br />

61. 7 = m../3, so m = ../3 = -3-'<br />

a = 2m, so a = 2( 7~) = 14~ .<br />

14../3<br />

n=a, so n=--.<br />

3<br />

q= n.,fi, so q= ( 14~).J2 = 14~ .<br />

62. p = 15<br />

r = p.J2, so r = 15.J2.<br />

r 15.J2 f/<br />

r=q../3, so q=13= ../3 =5"\16.<br />

t = 2q, so t = 2(5../6) = lcN6.<br />

63. Let h be the height of the equilateral triangle. By<br />

geometry, h bisects the base, s;and forms two<br />

30°-60° right triangles.<br />

s s<br />

:2 :2<br />

The formula for the area of a triangle is<br />

1<br />

Area = -bh.<br />

2<br />

In this triangle, b = s. The height h of the triangle<br />

is the side opposite the 60° angle in either<br />

30°-60° right triangle. The side opposite the<br />

30° angle is !... The height is..J3 . !... So the area<br />

2 2<br />

of the entire triangle is<br />

2<br />

1.s(../3 .!..) = s ../3 .<br />

2 2 4<br />

64. Area = 1. base' height<br />

2<br />

1 1 2 s2<br />

=_ ·s·s=-s or­<br />

2 2 2<br />

65. Yes, the third angle can be fund by subtracting the<br />

given acute angle from 90°, and the remaining<br />

two sides can be found using a trigonometric<br />

function involving the known angle and side.<br />

68. (a) 55 mlhr = 80 ~ ftlsec and<br />

3<br />

30 miJhr = 44 ftlsec<br />

D = 1.05(\12 - V 2<br />

2 )<br />

64.4(K 1 + K 2 + sin 9)<br />

Let 9 = 3.5°, K 1 =.4, and K 2 = .02.<br />

1.05(80.67 2 _ 44 2 )<br />

= 64.4(.4 + .02 + sin 3.5°)<br />

== 155 ft<br />

(b) D = 1.05(\12 - V2 2 )<br />

64.4(K 1 + K 2 + sin 9)<br />

Now let 9 = _2°.<br />

1.05(80.67 2 _ 44 2 )<br />

= 64.4[.4 + .02 +sin(-2°)]<br />

'" 194 ft<br />

69. Using the values for K 1 and K 2 from Exercise 68,<br />

determine V 2 when D = 200 and<br />

\1 = 90 mph = 132 ftlsec.<br />

Section 2.2<br />

Exercises<br />

D= 1.05(\12 - Vl)<br />

64.4(K 1 +K 2 +sin9)<br />

2 2<br />

200 = 1.05(132 - V2 )<br />

64.4(.4 + .02 + sine-3.5°»<br />

2<br />

200 =18,295.2 -1.05V 2<br />

23.12<br />

2 23.12(200)-18,295.2<br />

V 2 = -1.05<br />

V 2 "" 114 ftlsec<br />

V 2 ==78 mph<br />

1. C; 180 - 98 =82°<br />

(98° is in quadrant II)<br />

2. F; 212 - 180 = 32°<br />

(212° is in quadrant III)<br />

3. A; -135 + 360 =225°<br />

225 - 180 =45°<br />

(225° is in quadrant III)<br />

4. B; -60 + 360 =300°<br />

360 - 300 = 60°<br />

(300° is in quadrant IV)<br />

5. D; 750 - 2(360) =750 - 720 =30°<br />

(30° is in quadrant I)<br />

49


<strong>Trigonometry</strong> <strong>Solutions</strong> <strong>Manual</strong><br />

50 Chapter 2 Acute Angles and Right Triangles<br />

6. B; 480- 360 = 120°<br />

180 - 120 = 60°<br />

(120° is in quadrant II)<br />

11. 30°:<br />

{j<br />

tan 30° =­ 3<br />

cot 30° = {j<br />

12. 45°:<br />

13. 60°:<br />

sin 450 = ..fi<br />

2<br />

..fi<br />

cos 45°=­<br />

2<br />

sec 45° =..fi<br />

esc 45° =..fi<br />

sin 60° = {j<br />

2<br />

{j<br />

cot 60°=_<br />

3<br />

esc 600 = 2{j<br />

3<br />

14. 120°:<br />

1<br />

cos 120° = -cos 60° =-­ 2<br />

{j<br />

cot 120° = -cot 60°=-­<br />

3<br />

sec 120° = -sec 60° =-2<br />

15. 135°:<br />

tan 135° = - tan 45° =-1<br />

cot 135° = -cot 45° =-1<br />

16. 150°:<br />

sin 150° = sin 30° = ~<br />

2<br />

cotl500 = -cot 30° =-J3<br />

2{j<br />

sec 150° = -sec30° = --­<br />

3<br />

17. 210°:<br />

{j<br />

cos 210° = -cos300=-­<br />

2<br />

2{j<br />

sec 210° = -sec30° = --­<br />

3<br />

18. 240°:<br />

tan 240° = tan 60° = {j<br />

cot 240° = cot 60° = {j<br />

3<br />

19. 225°<br />

Use the method illustrated in Example 2.<br />

y<br />

x=-I,y=-I, r=..fi<br />

. 50 -1 ..fi<br />

sm22 =-=-­<br />

..fi 2<br />

-1 ..fi<br />

cos 225° = - =-­<br />

..fi 2<br />

-1<br />

tan 225° = - = 1<br />

-1<br />

-1<br />

cot 225° =- = 1<br />

-1<br />

, . ..fi<br />

sec 225° = - = -J2<br />

-1<br />

esc 225° = ..fi =-..fi<br />

-I .<br />

20. 300°<br />

Use the method illustrated in Example 3.<br />

y<br />

The reference angle is 60°. Since 300° is in<br />

quadrant IV, the sine, tangent, cotangent, and<br />

cosecant are negative.<br />

sin 300° = - sin 60° = - {j<br />

2<br />

1<br />

cos 300° = cos 60° = ­<br />

2<br />

tan 300° = - tan 60° = -J3<br />

-J3<br />

cot 300° = -cot60°=-­<br />

3<br />

sec 300° = sec 60° = 2<br />

2{j<br />

csc 300° = - csc 60° =--­ 3<br />

50


<strong>Trigonometry</strong> <strong>Solutions</strong> <strong>Manual</strong><br />

2.2 Trigonometric Functions of Non-Acute Angles 51<br />

21. 315°<br />

Use the method illustrated in Example 3.<br />

y<br />

23. 420°<br />

Use the method illustrated in Example 3.<br />

y<br />

315°<br />

60°<br />

The reference angle is 45° Since 315° is in<br />

quadrant IV, the sine, tangent, cotangent, and<br />

cosecant are negative.<br />

sin315 0;;:-sin450 ;;: ­<br />

-Ii<br />

2<br />

~<br />

cos315°;;: cos 45° ;;:­<br />

2<br />

tan 315°;;: -tan 45° ;;:-1<br />

cot 315° ;;:- cot 45° ;;: -1<br />

sec315°;;: sec 45° ;;:.J2<br />

csc315°;;: -csc45° =-J2<br />

22. 405°<br />

Use the method illustrated in Example 3.<br />

y<br />

The reference angle is 60° Since 420° is in<br />

quadrant I, all quadrants are positive.<br />

sin 420° = sin 60° = .fj<br />

2<br />

1<br />

cos 420° = cos 60° = ­<br />

2<br />

tan 420° = tan 60° =.fj<br />

.fj<br />

cot 420° = cot 60° = ­<br />

3<br />

sec 420° = sec 60° = 2<br />

2.fj<br />

esc 420° = esc 60° ;;:-­<br />

3<br />

24. 480°<br />

Use the method illustrated in Example 3.<br />

y<br />

480°<br />

405°<br />

The reference angle is 45° Since 405° is in<br />

quadrant I, all functions are positive.<br />

sin 405° ;;:sin 45° =~<br />

2<br />

~<br />

cos 405° = cos 45° =­<br />

2<br />

tan 405° ;;:tan 45° =1<br />

cot 405° =cot 45° =1<br />

sec 405° = sec 45° =.J2<br />

csc405° = csc45° = ~<br />

The reference angle is 60°. Since 480° is in<br />

quadrant II, the cosine, tangent, cotangent, and<br />

secant are negative.<br />

sin 480° = sin 60° = .fj<br />

2<br />

1<br />

cos 480° = -cos 60° =-­<br />

2<br />

tan 480° = - tan 60° = -.J3<br />

.fj<br />

cot 480° =-cot60° =-­<br />

3<br />

sec 480° =-sec60° =-2<br />

2.fj<br />

esc 480° = esc 60° =-­<br />

3<br />

51


<strong>Trigonometry</strong> <strong>Solutions</strong> <strong>Manual</strong><br />

S2<br />

Chapter 2 Acute Angles and Right Triangles<br />

2S. 495°<br />

Use the method illustrated in Example 3.<br />

y<br />

The reference angle is 45°. Since 495° is in<br />

quadrant II, cosine, tangent. cotangent, and secant<br />

are negative.<br />

sin 495° = sin 450 = .J2<br />

2<br />

.J2<br />

cos 495° =-cos45°=-­<br />

2<br />

tan 495° = - tan 45° = -1<br />

cot 495° =-cot 45° =-1<br />

sec 495° = - sec 45° = -,fi<br />

esc 495° =esc 45° = .J2<br />

26. 570°<br />

Use the method illustrated in Example 3.<br />

y<br />

570·<br />

The reference angle is 30°. Since 570° is in<br />

quadrant III. the sine. cosine, secant, and cosecant<br />

are negative.<br />

sin 570° = - sin 30° =_.!. 2<br />

..fj<br />

cos 570° = -cos30° = -- .<br />

2<br />

tan 570° = tan 30° =..fj<br />

3<br />

cot 570° =cot 30° = ..fj<br />

sec 570° = - sec 300 = _ 2..fj<br />

3<br />

esc 570° = -csc30° =-2<br />

27. 750° is cotenninal with<br />

750° - 2(360°) = 750° - 720°<br />

=30°.<br />

sin 750° = sin 30° =.!.<br />

2<br />

..fj<br />

cos 750° = cos 30° = ­<br />

2<br />

..fj<br />

tan 750° = tan 30° =­<br />

3<br />

cot 750° = cot 30° =..fj<br />

2..fj<br />

sec 750° =-sec 30°=-­<br />

3<br />

esc 750° =esc 30° = 2<br />

28. 1305° is cotenninal with<br />

1305° - 3(360°) = 1305° -1080°<br />

=225°.<br />

The reference angle is 45°. Since 1305° is in<br />

quadrant ill, the sine, cosine, secant, and cosecant<br />

are negative.<br />

sin 1305° = - sin 450 = _ .J2<br />

2<br />

.J2<br />

cos 1305° = -cos45° =-­ 2<br />

tan 1305° = tan 45° = 1<br />

cot 1305° = cot 45° =1<br />

sec 1305° = - sec 45° =-,fi<br />

esc 1305°= -csc45C>=-,fi<br />

29. 1500° is cotenninal with<br />

1500° - 4(360°) =1500° -1440°<br />

=60°.<br />

sin 1500° = sin 60° = ..fj<br />

2<br />

1<br />

cos 1500° =cos 60° =­<br />

2<br />

tan 1500° = tan 60° =..fj<br />

..fj<br />

cot 1500° =cot 60° = ­ 3<br />

sec 1500° = sec 60° =2<br />

2..fj<br />

esc 1500° = esc 60° =-­ 3<br />

52


<strong>Trigonometry</strong> <strong>Solutions</strong> <strong>Manual</strong><br />

2.2 Trigonometric Functions of Non-Acute Angles 53<br />

30. 2670° is coterminal with<br />

2670° -7(360°) =150°.<br />

The reference angle is 30°. Since 2670° is in<br />

quadrant II, the cosine, tangent, cotangent, and<br />

secant are negative .<br />

sin 2670° =sin 30° =.!.<br />

2<br />

.J3<br />

cos 2670° =- cos 30° =-­<br />

2<br />

.J3<br />

tan 2670° =- tan 30° =-­<br />

3<br />

cot 2670° =- cot 30° =-fj<br />

2.J3<br />

sec 2670° =-sec 30° =--­<br />

3<br />

esc 2670° =esc 30° =2<br />

31. -390° is coterminal with<br />

":'390° - (-2)(360°) = 330°.<br />

The reference angle is 30°. Since -390° is in<br />

quadrant IV, the sine, tangent, cotangent, and<br />

cosecant are negative.<br />

sin(-3900) =-sin 30° =_.!.<br />

2<br />

.J3<br />

cos(- 390°) = cos 30° = ­<br />

2<br />

.J3<br />

tan(-390°) =- tan 30° = -­<br />

3<br />

cot(-390°) = - cot 30° =-.J3<br />

2.J3<br />

sec(-390°) = sec 30° =-­<br />

3<br />

csc(-3900) =-csc30° =-2<br />

32. _510° is coterminal with<br />

-510° - (-2)(360°) = -510° + 720°<br />

=210°.<br />

The reference angle is 30°. Since -510° is in<br />

quadrant III, the sine, cosine, and secant and<br />

cosecant are negative.<br />

sin(-510°) =- sin 30° =_.!.<br />

2<br />

.J3<br />

cos(-5100) =-cos 30° =-­<br />

2<br />

.J3<br />

tan(-510°) = tan 30° = ­<br />

3<br />

cot(-510°) = cot 30° = .J3<br />

2.J3<br />

sec(-510°) =- sec 30° =--­<br />

3<br />

csc(-510°) = - csc 30° = -2<br />

33. -1020° is coterminal with<br />

-1020° - (-3)(360°) = -1020° + 1080°<br />

=60°.<br />

sin(-1020°) =sin 600 =.J3<br />

2<br />

1<br />

cos(-1020°) =cos 60° = ­<br />

2<br />

tan(-1020°) =tan 60° =.J3<br />

.J3<br />

cot(-1020°) = cot 60° =­<br />

3<br />

sec(-1020°) =sec 60° = 2<br />

2.J3<br />

csc(-1020°) =esc 60° = -­<br />

3<br />

34. -1290° is coterminal with<br />

-1290°-(-4)(360°) = -1290°+ 1440°<br />

=150°.<br />

The reference angle is 30°. Since -1290° is in<br />

quadrant II, the cosine, tangent. cotangent, and<br />

secant are negative.<br />

sin(-1290°) =sin 30° =.!.<br />

2<br />

.J3<br />

cos(- 1290°) = - cos 30° =-­<br />

2<br />

tan(-1290°) = - tan 300 =_ .J3<br />

3<br />

cot(-1290°) = - cot 30° =-fj<br />

sec(-1290°) = - sec 300 = ~ 2.J3<br />

3<br />

csc(-1290°) = esc 30° = 2<br />

35. 225° is in quadrant III, so the reference angle is<br />

225°- 180° = 45°.<br />

cos 45° = ~<br />

sin 45° = L<br />

r<br />

r<br />

x = r cos45° y = r sin 45°<br />

= 10 . ..fi =5..fi =10 . ..[i =5..J2<br />

2 2<br />

Since 225° is in quadrant 1lI. both the x and y<br />

coordinate will be negative . The coordinate of<br />

Pare: (-5..fi, -5..[i).<br />

36. 150° is in quadrant II, so the reference angle is<br />

180 -150 =30°<br />

cos 30° =~ sin 30° =L<br />

r<br />

r<br />

x = r cos 30° y = r sin 30°<br />

=6· ~ =3.J3 = 6 .±=3<br />

Since 150° is in quadrant n. x will be negative<br />

and y will be positive. The coordinates of Pare:<br />

(-3.J3, 3).<br />

53


<strong>Trigonometry</strong> <strong>Solutions</strong> <strong>Manual</strong><br />

54 Chapter 2 Acute Angles and Right Triangles<br />

37. For every angle O. sin 0 =L and cos 0<br />

,<br />

=~.<br />

,<br />

Thus. for every angle 0,<br />

(sin 0)2 + (cos 0)2 =(-;r +(:;f<br />

i+x<br />

= 2<br />

2<br />

r<br />

,2<br />

=2=1.<br />

r<br />

Since(.6)2 + (_.8)2 =.36 + .64 =1.<br />

there is an angle 0 for which cos 0 =.6 and<br />

sin 0= -.8. Since cos 0> 0 and sin 0< O. it is an<br />

angle in quadrant IV.<br />

38. For every angle O. sin 0 =L and cos 0 =~ .<br />

r<br />

r<br />

Thus for every angle O.<br />

(sin0)2 +(cos 0)2 =(-;r+ (-:;J<br />

i+x 2<br />

=~-;;--<br />

,2<br />

,2<br />

=2<br />

r<br />

=1.<br />

Since<br />

(<br />

~) 2 +(3.)2 =~+i= 145 *1<br />

4 3 16 9 144 •<br />

there is no angle 0 for which cos 0 =3.<br />

3<br />

andsinO=~ .<br />

4<br />

39. If 0 is in the interval (90°. 180°).<br />

90° < 0< 180°.<br />

Dividing by 2 gives 45° < ~ < 90°.<br />

2<br />

Thus, ~ is a quadrant I angle and sin ~ is<br />

2 2<br />

positive.<br />

41. If 0 is in the interval (90°, 180°).<br />

90°< 0 < 180°.<br />

Adding 180° gives 270° < 0+ 180° < 360°.<br />

Thus. 0 + 180° is a quadrant IV angle and<br />

cot (0 + 180°) is negative.<br />

42. If 0 is in the interval (90°. 180°),<br />

90° < 0 < 180°.<br />

Adding 180° gives 270° < 0 + 180° < 360°.<br />

Thus. 0 + 180° is a quadrant IV angle and<br />

sec (0 + 180°) is positive.<br />

43. If 0 is in the interval (90°. 180°).<br />

90° < 0 < 180°.<br />

Multiplying by -1 gives -90° > -0 > -180°.<br />

Thus. -0 is a quadrant III angle and cos (-0) is<br />

negative.<br />

44. If 0 is in the interval (90°. 180°).<br />

90° < 0 < 180°.<br />

Multiplying by -1 gives _90° > -0 > -180°.<br />

Thus. -0 is a quadrant III angle and sin (-8) is<br />

negative.<br />

47. The reference angle is 65°. Since 115° is in .<br />

quadrant II. the cosine is negative. Cos 0<br />

decreases on the interval (90°. 180°) from<br />

oto -1. Therefore. cos 115° is closest to -.4.<br />

48. The reference angle is 65°. Since 115° is in<br />

quadrant II the sine is positive. Sin 0 decreases<br />

on the interval (90°, .180°) from 1 to O. Therefore,<br />

sin 115° is closest to .9.<br />

49. When 0 =45°. sin 0 =cos O. Sine and cosine are<br />

opposites in quadrants II and IV. Thus<br />

180° - 0 =180° - 45° =135° in quadrant II.<br />

360° - 0 =360° - 45° =31Y in quadrant IV.<br />

50. When 0 =45°, sin 0 = cos O. Sine and cosine are<br />

both positive in quadrant I and both negative in<br />

quadrant III.<br />

o + 180° = 45° + 180° =225°<br />

Thus 45° is the quadrant I angle. and 225° is the<br />

quadrant III angle.<br />

40. If 0 is in the interval (90°. 180°),<br />

90° < 0 < 180°.<br />

Dividing by 2 gives 45° < ~ < 90°..<br />

2<br />

Thus.~ is a quadrant I angle and cos~ is<br />

2 2<br />

positive.<br />

54


<strong>Trigonometry</strong> <strong>Solutions</strong> <strong>Manual</strong><br />

2.2 Trigonometric Functions of Non-Acute Angles 55<br />

51. sin 30° + sin 60° J, sin (30° + 60°)<br />

Evaluate each side to determine whether this<br />

statement is true or false.<br />

sin 30° + sin 600 = .!.+ .J3<br />

2 2<br />

1+.J3<br />

=-­<br />

2<br />

sin(30° + 60°) = sin 90°<br />

=1<br />

. 1+.J3 1 h . . f 1<br />

Smce-- * • t e given statement IS a se.<br />

2<br />

52. sin (30° + 60°) J, sin 30° . cos 60°<br />

+ sin 60° . cos 30°<br />

Evaluate each side to determine whether this<br />

equation is true or false.<br />

sin (30° + 60°) = sin 90° = 1<br />

sin 30° . cos 60° + sin 60° . cos 30°<br />

=_._+_.­<br />

1 1 .J3.J3<br />

2 2 2 2<br />

1 3<br />

=-+-=1<br />

4 4<br />

Since. 1 = 1. the statement is true.<br />

53. cos 60°J, 2cos 2300-1<br />

Evaluate each side to determine whether this<br />

statement is true or false.<br />

1<br />

cos600=­<br />

2<br />

2COS 2300-1=2(v;f -1<br />

=2(~)-1<br />

=(~)-1<br />

1<br />

=­<br />

2<br />

S' 11 .<br />

mce - = -. the statement IS true.<br />

2 2<br />

54. cos 60°J, 2 cos 30°<br />

Evaluate each side to determine whether this<br />

statement is true or false.<br />

1<br />

cos60o=­<br />

2<br />

2cos30° = 2( V;) =.J3<br />

Since .!. *.J3. the statement is false.<br />

2<br />

55. sin 120°J, sin 150° - sin 30°<br />

Evaluate each side to determine whether this<br />

statement is true or false .<br />

sin 1200= .J3<br />

2<br />

sin 150° - sin 30° =.!. _.!. = 0<br />

2 2<br />

Since .J3 * O. the statement is false.<br />

2<br />

56. sin 210° J,. sin 180° + sin 30°<br />

Evaluate each side to determine whether this<br />

statement is true or false.<br />

sin210"=-.!.<br />

2<br />

sin 180°+ sin 30° = 0 +.!. = .!.<br />

2 2<br />

Since · - - 11 * -. th e statement IS . fa Ise.<br />

2 2<br />

57. sin 120°J, sin 180° .cos 60° - sin 60° .cos 180°<br />

Evaluate each side to determine whether this<br />

statement is true or false .<br />

sin 1200= .J3<br />

2<br />

sin 180°· cos 60° - sin 60° . cos 180°<br />

=O(~)-( V;}-I)<br />

=0-(- V;)= V;<br />

Since · -.J3 =-. .J3 th e statement IS-true. .<br />

2 2<br />

58. cos 300° J,. cos 240° . cos 60°<br />

- sin 240° .sin 60°<br />

Evaluate each side to determine whether this<br />

statement is true or false.<br />

1<br />

cos3000=­<br />

2<br />

cos 240° . cos 60° - sin 240° .sin 60°<br />

=(-~)(~)-(- V;)( V;)<br />

=-±-(-~)<br />

1<br />

=­<br />

2<br />

· 11 .<br />

Since - = - • the statement IS true.<br />

2 2<br />

55


<strong>Trigonometry</strong> <strong>Solutions</strong> <strong>Manual</strong><br />

56 Chapter 2 Acute Angles and Right Triangles<br />

2<br />

L :::; ~[~4 _-..:....( ---,3)-,,-]3_36__<br />

59. (a)<br />

200(1 .9 + 336 tan .9°)<br />

L '" 550ft<br />

(b)<br />

Section 2.3<br />

Exercises<br />

L:::; [4-(-3)]336 2<br />

200(1.9 + 336 tan 1.5°)<br />

"'369ft<br />

1. The caution at the beginning of this section<br />

suggests verifying that a calculator is in degree<br />

mode by finding sin 90°. Ifthe calculator is in<br />

degree mode, the display should be I.<br />

2. A calculator gives approximate values of almost<br />

all trigonometric functions.<br />

3. To find trigonometric values of cotangent, secant<br />

and cosecant with a calculator, it is necessary to<br />

find the reciprocal of the reciprocal function<br />

value.<br />

4. The reciprocal is used before the inverse function<br />

key when finding the angle, but after the function<br />

key when finding the trigonometric function<br />

value.<br />

For the following exercises, be sure your calculator is in<br />

degee mode. If your calculator accepts angles in<br />

degrees, minutes, and seconds, it is not necessary to<br />

change angles to decimal degrees. Keystroke sequences<br />

may vary on the type and/or model of calculator being<br />

used .<br />

5. tan 29° 30'<br />

29°30' :::; (29 + :)0<br />

:::; 29.5°<br />

Enter: 29.5 8<br />

or 8 29.5 ~<br />

Display: .5657728<br />

6. sin 38° 42'<br />

38 042':::;(38+ :~)"<br />

:::; 38.7°<br />

Enter: 38.7 ~<br />

or ~ 38 .7 ~<br />

Display: .6252427<br />

7. cot 41 ° 24'<br />

42°24':::; (41 + ~~y<br />

:::; 41.4°<br />

Enter: 41.4 8 Gill<br />

or CD 8 41.4<br />

Display: 1.1342773<br />

8. sec 13° 15'<br />

13°15':::; (13+ ~~r<br />

CD 0<br />

:::; 13.25°<br />

Enter: 13.25 @!) Gill<br />

or CD §) 13.25 CD<br />

E§)<br />

Display: 1.0273488<br />

9. esc 44° 30'<br />

44°30':::; ( 44+ :r<br />

:::; 44.5°<br />

Enter: 44.5 ~ Gill<br />

or CD ~ 44.5 CD<br />

Display: 1.4267182<br />

10. esc 145° 45'<br />

145 045':::;(145+ :r<br />

:::; 145.75°<br />

Enter: 145.75 ~ Gill<br />

or CD ~ 145.75 CD<br />

E§)<br />

Display: 1.7768146<br />

11. cot 183° 48'<br />

:::; 183.8°<br />

183°48':::; (183+ :r<br />

Enter: 183.8 8 (§]<br />

or CD ~ 183.8 CD<br />

E§)<br />

Display: 15.0557227<br />

12. cos 421 ° 30'<br />

421°30':::; (421+ :)0<br />

:::; 421.5°<br />

Enter: 421 .5 @!)<br />

or @D 421.5 ~<br />

Display: .4771588<br />

0<br />

E§)<br />

0 ~<br />

0<br />

0<br />

56


<strong>Trigonometry</strong> <strong>Solutions</strong> <strong>Manual</strong><br />

2.3 Finding Trigonometric Function Values Using a Calculator 57<br />

13. sec 312° 12'<br />

312°12' = (312 + ~~ )O<br />

19. = sin 514°24'<br />

csc514°24'<br />

514°24' =(514+ ~~ y<br />

= 312.2°<br />

Enter: 312.2 §J § = 514.4°<br />

or CD §J 312.2 CD 0<br />

Enter: 514.4 ~<br />

~<br />

or ~ 514.4 ~<br />

Display: 1.4887142<br />

Display: .4320857<br />

14. tan (- 80° 6')<br />

20. ---= tan 23.4°<br />

-80°6' = _(80 + 6~ ) cot 23.4°<br />

0<br />

Enter: 23.4 8<br />

= -80.1° or 8 23.4 ~<br />

Enter: 80.1 EJ ~ Display: .4327386<br />

or ~ (ill 80.1 ~<br />

Display: -5.7297416<br />

21. --=tan<br />

sin 33° 330<br />

cos 33°<br />

15. sin (-317° 36') Enter: 33 8<br />

or 8 33 E!§)<br />

-317 ~~y 036'=-(317+ Display: .6494076<br />

0<br />

= -317.6 cos 77° 770<br />

Enter: 317.6 EJ ~ 22. = cot<br />

or<br />

sin 77°<br />

~ (ill 317.6 ~<br />

Enter: 77 8 (@)<br />

Display: .6743024<br />

or CIJ ~ 77 CD 0 ~<br />

Display: .2308682<br />

16. cot (-512° 20')<br />

23. cos (90° - 3.69°) = sin 3.69°<br />

-512°20' = -(512+ ~y<br />

Enter: 3.69 ~<br />

:::: -512.3333333°<br />

or ~ 3.69 ~<br />

Enter: 512.3333333 EJ 8<br />

Display: .0643581<br />

(@)<br />

or CD 8 (B) 512.3333333 CD 0<br />

24. cot (90° - 4.72°) = tan 4.72°<br />

~<br />

Display: 1.9074147<br />

Enter: 4.72 8<br />

or 8 4.72 ~<br />

Display: .0825664<br />

17. cos (-15')<br />

_15'=_(~)O 25. sec 2 47.8°-1 = tan 2 47.8°<br />

Enter: 47.8 8 IZl<br />

=-.25° or CD ~ 47.8 CD I£J ~<br />

Enter: .25 EJ (§ Display: 1.2162701<br />

or ~ (ill .25 ~<br />

Display: .9999905<br />

18. ---= cos 14.8°<br />

29. sin ()= .84802194<br />

secl4.8°<br />

INV<br />

Enter: 14.8 ~<br />

or ~ 14.8 ~<br />

or<br />

Display: .9668234 Enter: .84802 194 ARCSIN<br />

or<br />

1·..·'1<br />

or (E) ~ .84802194 ~<br />

Display : 57.997172<br />

(}:::: 57.997172°<br />

57


<strong>Trigonometry</strong> <strong>Solutions</strong> <strong>Manual</strong><br />

58 Chapter 2 Acute Angles and Right Triangles<br />

30. tan (} = 1.4739716<br />

' INV<br />

Enter: 1.4739716<br />

or<br />

ARCTAN<br />

or<br />

[,...-']<br />

or ~ EJ 1.4739716 ~<br />

Display : 55.845496<br />

(} == 55.845496°<br />

31. tan (}= 6.4358841<br />

Enter: 6.4358841{INV AR~~AN<br />

or<br />

[T...-'I<br />

or ~ EJ 6.4358841 ~<br />

Display: =81.168073<br />

(}== 81.168073°<br />

32. sin (} =.27843196<br />

INV<br />

Enter: .27843196<br />

or<br />

ARCSIN<br />

or<br />

[... ,]<br />

or ~ ~ .27843196 ~<br />

Display: 16.166641<br />

(} == 16.166641°<br />

33. sec (}=1.1606249<br />

Enter: 1.1606249 l:!2D<br />

!NV<br />

or<br />

ARCCOS<br />

or<br />

[""".,]<br />

or ~ @!) 1.1606249 ~ ~<br />

Display: 30.502748<br />

(} == 30.502748°<br />

34. cot (}= 1.2575516<br />

Enter 1.2575516 l:!2D<br />

INV<br />

or<br />

ARCTAN<br />

or<br />

1'...-'1<br />

or B IE] 1.2575516 0 ~<br />

Display: 38.491580<br />

(} == 38.491580°<br />

35. esc (}= 1.3861147<br />

Enter: 1.3861147 ~<br />

INV<br />

or ~ ~ 1.3861147 ~<br />

Display: 46.173582<br />

() == 46.173582°<br />

36. sec (}= 2.7496222<br />

2.7496222 l:!2D {INV AR~~OS<br />

or<br />

[""".,]<br />

or ~ @!) 2.7496222 ~<br />

Display : 68.673241<br />

(}== 68.673241°<br />

37. tan A = 1.482560969<br />

or<br />

ARCSIN<br />

or<br />

["' ~I<br />

Enter : 1.482560969{INV AR~~AN<br />

or<br />

[''''-'1<br />

or ~ EJ J .482560969 ~<br />

Display: 56<br />

A == 56°<br />

38. A =sin 22°<br />

Enter: 22 ~<br />

or~22~<br />

Display : .3746065934<br />

A == .3746065934°<br />

In Exercises 39-42, be sure that your calculator is in<br />

degree mode. Keystroke sequences may vary based on<br />

the type and/or model of calculator being used.<br />

39. sin 35° cos 55° + cos 35° sin 55°<br />

Enter: IT) 35 ~ CD 0 IT) 55<br />

§) CO ITJ IT) 35 §) CD 0<br />

CD 55 ~ ITJ @]<br />

or ~ 35 0 @D 55 0 rn @D<br />

35 CO<br />

EJ 55 ITJ ~<br />

Display: 1<br />

sin 35° cos 55° + cos 35° sin 55° = 1<br />

40. cos 100° cos 80° - sin 100° sin 80°<br />

Enter: 100 @D 0 80 @!) G<br />

100 ~ 0 80 EJ @]<br />

or @D 100 CD @D 80 CO G<br />

EJ 100 CO ~ 80 CO ~<br />

Display: -1<br />

cos 100° cos 80° - sin 100° sin 80° =-1<br />

58


<strong>Trigonometry</strong> <strong>Solutions</strong> <strong>Manual</strong><br />

2.3 Finding Trigonometric Function Values Using a CalcuJator 59<br />

41. cos 75° 29' cos 1431' 44. 0) = 39°, 9z = 28°<br />

- sin 75° 29' sin 14° 31' ~= sin9\<br />

Enter: CD 75 rn 29 CD 60 OJ Cz<br />

~<br />

sin9z<br />

o OJ 14 rn 31 CD 60 OJ ~<br />

c) sinOz<br />


<strong>Trigonometry</strong> <strong>Solutions</strong> <strong>Manual</strong><br />

60 Chapter 2 Acute Angles and Right Triangles<br />

48. cl =3 x 10 8 m per sec, 55. ~ sin 40° =sin ~ (40°)<br />

c2 =2.254 x 10 8 m per sec<br />

9 1 =90° - 29.6° =60.4°<br />

2 2<br />

Using a calculator gives<br />

~sin4O° =.32139380 and<br />

2.254 x 10 8 (sin 60.4°) 2<br />

sin9 2 =<br />

3x 10 8 sin~(400) =.34202014.<br />

9 2 = 40.8° 2<br />

Light from the object is refracted at an angle of<br />

False<br />

40.8° from the vertical. Light from the horizon is<br />

refracted at an angle of 48.7° from the vertical. 56. sin 39° 48' + cos 39° 48' =1<br />

Therefore, the fish thinks the object lies at an<br />

Using a calculator gives<br />

angle of 48.7° - 40.8° =7.9° above the horizon. sin 39° 48' +cos 39° 48' =1.40839322.<br />

49. cos 40° J, 2 cos 20°<br />

False<br />

Using a calculator gives<br />

57. F= Wsin9<br />

cos 40° =.766044 and<br />

F = 2400 sin (-2.4°)<br />

2 cos 20° =1.8793852. = -100.5 Ib<br />

False<br />

F is negative because the car is traveling<br />

SO. sin 10°+sin 10°J, sin 20°<br />

downhill.<br />

Using a calculator gives 58. F= Wsin9<br />

sin 10°+sin 10° =.34729636 and<br />

F =2100sin 1.8 0<br />

sin 20° =.34202014.<br />

=65.% Ib<br />

False<br />

59. F=Wsin9<br />

51. cos 70 0 J, 2 cos 2 35 0 -1 150 = 3000 sin 9<br />

Using a calculator gives 150 . 9<br />

cos 70° =.34202014 and<br />

--=sm<br />

3000<br />

2cos 2 35 0 -1 = .34202014.<br />

.05 =sin9<br />

True<br />

9 = 2.87°<br />

52. sin 50° J, 2 sin 25° . cos 25° 60. F= Wsin9<br />

Using a calculator gives<br />

120 =W sin 1.5°<br />

sin50° = .76604444 and<br />

120<br />

=W<br />

2sin25° ·cos25° = .76604444.<br />

sin 1.5°<br />

True<br />

W=45841b<br />

53. 2 cos 38° 22' =cos 76° 44' 61. F=Wsin9<br />

Using a calculator gives<br />

-145 = Wsin(-3°)<br />

2 cos 38°22' = 1.5681094 and -145<br />

cos 76° 44' =.22948353.<br />

=W<br />

sin(-3°)<br />

False<br />

W=2771Ib<br />

54. cos 40° = 1-2sin 2 80 0 62. F= Wsin9<br />

Using a calculator gives<br />

-130 =2600 sin 9<br />

cos 40° = .76604444 and<br />

-130 .<br />

1-2sin 2 80° = -.93969262.<br />

--=sm9<br />

2600<br />

False<br />

-.05 =sin9<br />

9 =-2.87°<br />

60


<strong>Trigonometry</strong> <strong>Solutions</strong> <strong>Manual</strong><br />

2.3 Finding Trigonometric Function Values Using a Calculator 61<br />

63. F= WsinO 65. For parts (a) and (b), a = 3°, g = 32.2, and<br />

F=2200sin2° /=.14.<br />

F = 76.77889275 Ib<br />

F= WsinO (a) Since 45 rni/hr = 66 ft/sec,<br />

F = 2000sin 2.2°<br />

F = 76.77561818 Ib<br />

The 2200-lb car on a 2° uphill grade has the<br />

greater grade resistance.<br />

y2<br />

R=---­<br />

g(f + tan a)<br />

66 2<br />

32.2(.14 + tan 3°)<br />

reO<br />

64. 0 sin 0 tan 0 - '" 703 ft<br />

180<br />

(b) Since 70 mi/hr = 102213 ft/sec,<br />

0° .0000 .0000 .0000<br />

y 2<br />

.5° .0087 .0087 .0087<br />

R=---­<br />

g(f+tana)<br />

1° .0175 .0 175 .0175 102.67 2<br />

"'-----­<br />

1.5° .0262 .0262 .0262 32.2(.14 + tan 3°)<br />

'" 1701 ft<br />

2° .0349 .0349 .0349<br />

2.5° .0436 .0437 .0436<br />

(c) Intuitively, increasing a would make it easier<br />

to negotiate the curve at a higher speed much<br />

3° .0523 .0524 .0524 like is done at a race track. Mathematically, a<br />

3.5° .0610 .0612 .0611<br />

larger value ofa (acute) will lead to a larger<br />

value for tan a. Iftan a increases, then the<br />

4° .0698 .0699 .0698 ratio determining R will decrease. Thus , the<br />

radius can be smaller and the curve sharper if<br />

(a) From the table we see that if 0 is small,<br />

a is increased.<br />

• Ll Ll reO<br />

sm 17 "" tan 17 '''''-.<br />

180<br />

(b)<br />

(c)<br />

(d)<br />

F = Wsin 0 :=: Wtan 0 '" WreO<br />

180<br />

4<br />

tanO=-=.04<br />

100<br />

y2<br />

R=---­<br />

g(/ +tana)<br />

for small O. 66 2<br />

""-----­<br />

32.2(.14 + tan 4°)<br />

'" 644 ft<br />

y2<br />

R=---­<br />

g(/+tana)<br />

F"" WtanO<br />

= 2000(.04) 102.67 2<br />

=801b<br />

32.2(.14+ tan 4°)<br />

WreO<br />

Use F"" -­ from part (b)<br />

180<br />

Let 0= 3.75 and W = 1800. 66. From Exercise 65,<br />

F "" 18001t(3.75)<br />

180<br />

:=: 117.81Ib<br />

"" 1559 ft<br />

As predicted, both values are less.<br />

y2<br />

R=---­<br />

g(f + tan a)<br />

Let R = 1150, g = 32.2,/= .14, anda= 2.1°.<br />

Then<br />

v = "/""-'R-g("""'/:-+-ta-n-a""'-)<br />

= ../1150(32.2)(.14 + tan 2.1°)<br />

"" 80.9 ftlsec<br />

80.9 ftlsee . 3600 seclhr . 1 mi/5280 ft:=: 55 mph<br />

It should have a 55 mph speed limit.<br />

61


<strong>Trigonometry</strong> <strong>Solutions</strong> <strong>Manual</strong><br />

62 Chapter 2 Acute Angles and Right Triangles<br />

v 2 sin (}cos()+ vcos(}~(vsin (})2 + 64h<br />

67. (a) D =-------....!------.;--­<br />

32<br />

Let v =44 ftlsee and h = 7 ft.<br />

For () =40°,<br />

Section 2.4<br />

44 2 sin40°cos400+44cos400~(44sin400)2 +64(7)<br />

D =: ----------........,;.----'---"""""--'­<br />

32<br />

""67.00 ft<br />

For () =42°,<br />

D =: 44 2 sin 42° cos 42° + 44 cos 42°~( 44 sin 42°)2 + 64(7)<br />

32<br />

==67.14ft<br />

For () =: 45°,<br />

D =: 44 2 sin 45° cos 45° + 44 cos 45°~( 44 sin 45 0)2 + 64(7)<br />

32<br />

== 66.84 ft<br />

D increases and then decreases.<br />

(b) Let h =7 ft and () =42°.<br />

For v =43 ft/sec,<br />

D =43 2 sin 42° cos 42° + 43cos42°~(43sin42o)2 +64(7)<br />

32<br />

== 64.40 ft<br />

For v =44 ft/sec,<br />

D= 44 2 sin42°cos42°+44cos42°~(44sin42o)2 +64(7)<br />

32<br />

==67.14ft<br />

For v =45 ftlsee,<br />

D =: 45 2 sin 42°cos42°+ 45cos42°~(45sin42o)2+64(7)<br />

32<br />

== 69 .93 ft<br />

D increases.<br />

(c) v; the shotputter should concentrate on achieving as large a value of v as possible.<br />

Connections (page 87)<br />

Our Steps 1 and 2 correspond to Polya's Steps 1 and 2. Our Step 3 corresponds to Poyla's Steps 3 and 4.<br />

Exercises<br />

1. 16,454.5 to 16,455.5<br />

2. 28,999.5 to 29,000.5<br />

3. 546.5 to 547.5<br />

4. 8958.5 to 8959.5<br />

8. A measurement of 26.0 lb indicates a weight between 25.95 and 26.05 lb.<br />

62


<strong>Trigonometry</strong> <strong>Solutions</strong> <strong>Manual</strong><br />

9. If h is the actual height of a building and the<br />

height is measured as 58.6 ft, then<br />

Ih-58.6Is .05.<br />

10. If w is the actual weight of a car and the weight is<br />

measured as 15.00 x 10 21b, then Iw -15001 S .5.<br />

11. A = 36°20', c = 964 m<br />

A+B=90°<br />

B=900-A<br />

B = 90° - 36°20'<br />

= 89°60' - 36°20'<br />

B= 53°40'<br />

sin A =~<br />

c<br />

a =c sin A<br />

a = 964 sin 36°20'<br />

Use a calculator and round answer to three<br />

significant digits.<br />

a =571 m<br />

b<br />

cosA=­ c<br />

b =ccosA<br />

b = 964 cos 36°20'<br />

Use a calculator and round answer to three<br />

significant digits.<br />

b=777m<br />

12. A = 31°40', a = 35.9 krn<br />

B=900-A<br />

B = 90° - 31°40' = 58°20'<br />

tan A =~<br />

b<br />

tan 31°40' = 35.9<br />

b<br />

35.9<br />

tan31°4Q'<br />

Use a calculator and round answer to three<br />

significant digits.<br />

= 58.2 krn<br />

. B b<br />

SID<br />

b=---­<br />

sin A =~<br />

c<br />

SID<br />

. 31040' =-­ 35.9<br />

c<br />

35.9<br />

sin 31°4Q'<br />

Use a calculator and round answer to three<br />

significant digits.<br />

= 68.4 krn<br />

13. N = 51.2°, m = 124 m<br />

M+N=9O°<br />

M=900-N<br />

M = 90° - 51.2°<br />

M = 38.8°<br />

tanN=~<br />

m<br />

n=mtanN<br />

n = 124 . tan 51.2°<br />

n = 154 m<br />

2.4 Solving Right Angles 63<br />

cos N =!!!.<br />

p<br />

m<br />

p=-­<br />

cosN<br />

124<br />

p=--­<br />

cos51.2°<br />

p = 198 m<br />

14. X = 47.8 °, z = 89.6 cm<br />

Y = 90° - 47.8° = 42.2°<br />

~=sin47 .8°<br />

89.6<br />

x = 89.6sin47.8°<br />

=66.4 cm<br />

_Y-=cos47.8°<br />

89.6<br />

y = 89.6 cos 47.8°<br />

=60.2 em<br />

15. B = 42.0892°, b = 56.851 ern<br />

A+B=9O°<br />

A =90 0-B<br />

A = 90° - 42.0892°<br />

=47.9108°<br />

=­ c<br />

b<br />

c=-­<br />

. sinB<br />

56.851<br />

c=----­<br />

sin 42.0892°<br />

c= 84.816 em<br />

b<br />

tan B=a<br />

b<br />

a=-­<br />

c=---­ tanB<br />

56.851<br />

a=----­<br />

tan 42.0892°<br />

a=62.942 em<br />

63


<strong>Trigonometry</strong> <strong>Solutions</strong> <strong>Manual</strong><br />

64 Chapter 2 Acute Angles and Right Triangles<br />

16. B = 68.5142°, c = 3579.42 m 24. B = 46.00°, c = 29.7 m<br />

A = 90° - 68.5142° = 21.4858°<br />

B<br />

a<br />

---= cos 68.5142°<br />

3579.42<br />

a = 3579.42 cos 68.5142°<br />

= 1311.04 m<br />

90·<br />

b = 3579.42 sin 68.5142°<br />

Abe<br />

=3330.68 m<br />

A = 90° - 46.00° = 44.00°<br />

17. B<br />

_a_ = cos 46.000<br />

29.7<br />

a = 29.7 cos 46.00°<br />

~ • ~ 1.498426374 =20.6 m<br />

22°<br />

A C .s: = sin 46.00°<br />

29.7<br />

18. A<br />

b = 29.7sin46.00°<br />

=21.4 m<br />

.:<br />

25. B = 73.00°, b = 128 in.<br />

B<br />

5 ~a ~<br />

A 128 in . C<br />

C B "" 56 .44269024°<br />

A = 90° -73.00° = 17.00°<br />

0 _ 128<br />

tan 73.00 -­<br />

a<br />

128<br />

a=---­<br />

tan 73.00°<br />

20. If we are given an acute angle and a side in a right a =39.1 in.<br />

triangle, the unknown part of the triangle<br />

sin 73.000 = 128 .<br />

requiring the least work to find is the other acute<br />

c<br />

angle . It may be found by subtracting the given 128<br />

acute angle from 90°.<br />

23. A = 28.00°, c = 17.4 ft<br />

B<br />

a<br />

90· 28 .00<br />

C b A<br />

A+B=90°<br />

B=900-A<br />

B = 90° - 28.00°<br />

B=62.00°<br />

sinA=~<br />

c=---­<br />

sin 73.00°<br />

c=134 in.<br />

26. A = 61°00', b =39.2 em<br />

B<br />

61.00· 90 ·<br />

c A 39 .2m C<br />

a =csinA<br />

B = 90° - 61°00' = 29°00'<br />

a = 17.4sin 28.00°<br />

~= tan 61°00'<br />

a=8.17 ft 39.2<br />

b<br />

a<br />

cosA =­<br />

= 39.2 tan 61°00'<br />

c<br />

=70.7 em<br />

b=ccosA<br />

_c_ = sec 61°00'<br />

b = 17.4 cos 28.00° 39.2<br />

b = 15.4 ft<br />

c = 39.2 sec 61°00'<br />

=80.9 em<br />

a<br />

64


<strong>Trigonometry</strong> <strong>Solutions</strong> <strong>Manual</strong><br />

2.4 Solving Right Angles 65<br />

27. a = 76.4 yd, b = 39.3 yd 29. a = 18.9 em, c = 46.3 em<br />

B<br />

76 .4 yd<br />

~.18.9 em<br />

90·<br />

A b C<br />

b=~c2_a2<br />

b = ~(46 .3)2 -(18.9)2<br />

90·<br />

C 39.3 yd A b =42.3 em<br />

c 2 =a 2 +b 2 . A 18.9<br />

sm =-­<br />

46.3<br />

c=~a2+b2<br />

A =24°10'<br />

c = ~(76.4)2 + (39.3)2 B 18.9<br />

cos =-­<br />

c=85.9 yd 46.3<br />

B= 65°50'<br />

tan A = 76.4<br />

39.3<br />

A = 62°50'<br />

tanB= 39.3<br />

76.4<br />

B = 27°10'<br />

28. a = 958 m, b = 489 m<br />

B<br />

30. b = 219 m, C = 647 m<br />

B<br />

a<br />

958m<br />

a=~6472-2192<br />

a = .J4 I 8,609 - 47,961<br />

a = .J370,648<br />

90·<br />

a=609m<br />

A 489m C<br />

958 219<br />

tanA=­<br />

cosA=­<br />

489<br />

647<br />

A = 63.0° = 63°00'<br />

A = 70.2° = 70° 10'<br />

489 . B 219<br />

tanB=- sm =­<br />

958<br />

647<br />

B = 27.0° = 27°00'<br />

B = 19.8° = 19°50'<br />

Check: A + B = 90° Check that A + B = 90°.<br />

To find c from the given information, use the<br />

Pythagorean theorem.<br />

c 2 =a 2 +b 2<br />

c=~a2 +b 2<br />

c = ~9582 + 489 2<br />

c = .Jl, 156,885<br />

c= 1080 m<br />

Another way to find c is to use the given<br />

information together with the result for one of the<br />

acute angles.<br />

~=csc63.0°<br />

958<br />

c = 958 esc 63.0°<br />

c= 1080 m<br />

65


<strong>Trigonometry</strong> <strong>Solutions</strong> <strong>Manual</strong><br />

66 Chapter 2 Acute Angles and Right Triangles<br />

31. A = 53°24', C = 387.1 ft<br />

B =90° - 53°24'<br />

B = 89°60' - 53°24'<br />

B= 36°36'<br />

sin 53°24' = _a_<br />

387.1<br />

a = 387.1sin53°24'<br />

a = 310.8 ft<br />

cos 53°24' = _b_<br />

387.1<br />

b = 387. 1cos 53°24'<br />

b = 230.8 ft<br />

32. A = 13° 47', C = 1285 m<br />

B<br />

13°47' 90·<br />

Abe<br />

B= 90°-13°47' == 76°13 '<br />

b = 1285 cos 13°47'<br />

= 1248 m<br />

a = 1285sin 13°47'<br />

=306.2 m<br />

33. B=39° 9', c= .6231 m<br />

B<br />

a<br />

90°<br />

c b A<br />

A =90° - 39°9'<br />

A = 50°51'<br />

cos 3909' = _a_<br />

.6231<br />

a = (.6231) cos 39°9'<br />

a = .4832 m<br />

sin 3909' =_b_<br />

.6231<br />

b = (.6231)sin39~'<br />

b=.3934 m<br />

a<br />

34. B = 82° 51', C = 4.825 em<br />

B<br />

~a<br />

Abe<br />

A = 90° - 82°51' = 7°9'<br />

b =4.825sin 82°51'<br />

=4.787 em<br />

a = 4.825 cos 82°51'<br />

=.6006 em<br />

35. The angle of elevation from X to Y is 90°<br />

whenever Y is directly above X.<br />

36. The angle of elevation from X to Y is the acute<br />

angle formed by ray XY and a horizontal ray with<br />

endpoint at X. Therefore, the angle of elevation<br />

cannot be more than 90° .<br />

39. Let h =the distance the ladder goes up the wall.<br />

sin43 050' = _h_<br />

13.5<br />

h = 13.5 sin 43°50'<br />

h = 9.3496000<br />

The ladder goes up the wall 9.35 m.<br />

40. Let ()= the angle the guy wire makes with the<br />

ground. .<br />

. () 71.3<br />

S10 =-­<br />

77.4<br />

() = 67.100475°<br />

() = 67° + (.100475)(60')<br />

() =67°10'<br />

The guy wire makes an angle of 67° 10' with the<br />

ground.<br />

41. Let x =the length of the guy wire.<br />

S10<br />

. 45030' =-­ 63.0<br />

x<br />

xsin 45°30' = 63.0<br />

63.0<br />

x =-<br />

""7<br />

sin 45°30'<br />

x =88.328020<br />

The length of the guy wire is 88.3m.<br />

66


<strong>Trigonometry</strong> <strong>Solutions</strong> <strong>Manual</strong><br />

2.4 Solving Right Angles 67<br />

42. Since angle T= 32° 10' and angle S = 57° 50',<br />

S + T =90°, and triangle RST is a right triangle.<br />

tan 32010' =-­ RS<br />

53.1<br />

RS =53.1 tan 32°10'<br />

RS"" 33.395727<br />

The distance across the lake is 33.4 m.<br />

43. The two right triangles are congruent.<br />

So corresponding sides are congruent. Since the<br />

sides that compose the base of the isosceles<br />

tnang<br />

. I<br />

e are congruent, eac<br />

. h id . 42.36<br />

si e IS --,or<br />

2<br />

21.18 in. Let x =the length of each of the two<br />

equal sides of the isosceles triangle.<br />

45. tan 30.0° =-y­<br />

1000<br />

y =1000 · tan 30.0°<br />

=577<br />

However, the observer's eye-height is 6 feet from<br />

the ground, so the cloud ceiling<br />

is 577 + 6 = 583 feet.<br />

46. Let x = the length of the shadow.<br />

0 _ 5.75<br />

tan 23 . 4 --­<br />

x<br />

x tan 23.4° =5.75<br />

5.75<br />

x=--­<br />

tan 23.4°<br />

x ""13.287466<br />

The length ofthe shadow is 13.3 ft.<br />

cos38.120=21.18<br />

x<br />

xcos38.12° =21.18<br />

21.18<br />

cos38.12°<br />

X"" 26.921918<br />

The length of each of the two equal sides of the<br />

triangle is 26.92 in.<br />

44. The altitude of an isosceles triangle bisects the<br />

base as well as the angle opposite the base. Let<br />

h =the altitude. The base is 184.2 em so the<br />

altitude forms two congruent right triangles each<br />

with a leg of 184.2 =92.1 cm and with an<br />

2<br />

opposite angle of .!.(68°44') =34°22'.<br />

2<br />

A<br />

47. Let h =height of the tower.<br />

~h<br />

42.36--­<br />

A 40 .6m C<br />

In triangle ABC,<br />

h<br />

tan 34.6° =-­<br />

40.6<br />

h =40.6 tan 34.6°<br />

x=---­<br />

h "" 28.0.<br />

The height of the tower is 28.0 m.<br />

48. Let 0 = the angle of elevation of the sun.<br />

J><br />

./<br />

./<br />

63 .1<br />

tan 0 = 48.6<br />

63.1<br />

0"" 37.603681<br />

"" 37°+(.603681)(60')<br />

0",,37°40'<br />

The angle of elevation of the sun is 37° 40'.<br />

D 92.1 em C 92.1 em B<br />

In triangle ABC,<br />

tan 34<br />

022'<br />

--­<br />

_ 92.1<br />

h<br />

h tan 34~2' = 92.1<br />

92.1<br />

h=--­<br />

tan 34°22'<br />

h =134.67667.<br />

The altitude of the triangle is 134.7 em.<br />

67


<strong>Trigonometry</strong> <strong>Solutions</strong> <strong>Manual</strong><br />

68 Chapter 2 Acute Angles and Right Triangles<br />

49. Let d =the distance from the top B of the building<br />

to the point on the ground A.<br />

B<br />

A<br />

In triangle ABC,<br />

C<br />

sm . 32030' =-­ 252<br />

d<br />

252<br />

sin 32°30'<br />

d==469.<br />

The distance from the top of the building to the<br />

point on the ground is 469 m.<br />

tan () =12.02<br />

d=---­5.93<br />

50. Let x =the horizontal distance that the plan must<br />

fly to be directly over the tree.<br />

tan 13°50' = 10,500<br />

x<br />

x tan 13°50' =10,500<br />

10,500<br />

tan 13°50'<br />

x == 42,600<br />

The horizontal distance that the plan must fly to<br />

be directly over the tree is 42,600 ft.<br />

51. Let x = the height of the taller building,<br />

h = the difference in height between the<br />

shorter and taller buildings, and<br />

d =the distance between the buildings along<br />

the ground.<br />

52. Let a = the angle of depression that the lens<br />

makes with the horizontal and<br />

() =the angle that the lens makes with the<br />

vertical.<br />

camera<br />

5.93~<br />

ft~<br />

12.02ft<br />

head<br />

teller<br />

() =63°40'<br />

a=900-()<br />

=90° - 63°40'<br />

a =26°20'<br />

The lens should make an angle of 26° 20' with<br />

the horizontal.<br />

53. Let ()=the angle of depression.<br />

tan () = 39.82<br />

51.74<br />

x=---­() = 37.58° =37°35'<br />

54. Let d = the diameter of the sun.<br />

..!.(l°4') =32' =.533°<br />

2<br />

Ld<br />

2 =tan .533°<br />

92,919,800<br />

d =2(92,919, 8oo)(tan .533°)<br />

== 1,730,000 mi<br />

The diameter of the sun is about 1,730,000 mi.<br />

55. The triangle with 60° as vertex angle and x as a<br />

base is equilateral. d is the perpendicular bisector<br />

of x and forms a 30°-60° right triangle.<br />

i x i x<br />

60· 60·<br />

d<br />

--= cot 14°10'<br />

28.0<br />

d =28.0 cot 14°10'<br />

=111m<br />

h<br />

- =tan 46°40'<br />

d<br />

h = d tan 46°40'<br />

=111tan 46°40'<br />

=118m<br />

x=h+28.0<br />

= 118+28.0<br />

==146m<br />

The height of the taller building is 146 m.<br />

LX<br />

_2_ = tan 30°<br />

d<br />

x = 2dtan30°<br />

=2(2.894)(tan 30°)<br />

==3.342 mm<br />

68


<strong>Trigonometry</strong> <strong>Solutions</strong> <strong>Manual</strong><br />

2.5 Further Applications of Right Triangles 69<br />

56. Let x = the distance from the assigned target.<br />

A<br />

' .')"",",J;<br />

In triangle ABC,<br />

x<br />

tan 0°0'30" =--­<br />

234,000<br />

x = 234,000 tan 0°0'30"<br />

x'" 34.0<br />

The distance from the assigned target is 34.0 mi.<br />

57. (a) The height h of the peak above 14,545 is<br />

sin5.82 0 = h<br />

27.0134(5280)<br />

h ",14,463 ft.<br />

The total height is about<br />

14,545 + 14,463 = 29,008 ft.<br />

(b) The curvature of the earth would make the<br />

peak appear shorter than it actually is.<br />

Initially the surveyors did not think<br />

Mt. Everest was the tallest peak in the<br />

Himalayas. It did not look like the tallest<br />

peak because it was farther away than the<br />

other large peaks.<br />

58. (a) Let d be the distance between P and Q.<br />

Triangle PQC is a right triangle with<br />

() d<br />

tan -=­<br />

2 R<br />

e<br />

d= R tan­<br />

2<br />

37°<br />

=965 tan­<br />

2<br />

= 965 tan 18.5°<br />

'" 323 ft.<br />

(b) From right triangle CPN,<br />

() NC<br />

cos-=­<br />

2 R<br />

()<br />

NC=Rcos- .<br />

2<br />

Now,<br />

MN+NC=R<br />

MN=R-NC<br />

()<br />

= R-Rcos­ 2<br />

='I -COS~}<br />

59. (a)<br />

Section 2.5<br />

Exercises<br />

f3 ,., 57.3S = 57.3(336) ,., 320<br />

R 600<br />

d =R(I-COS ~ )<br />

'" 600(1- cos 16°)<br />

= 23.4 ft<br />

(b) f3'" 57.3S = 57.3(485) = 46 .3°<br />

R 600<br />

d= 'I-COS~J<br />

00(<br />

,., 48.3 ft<br />

46 30)<br />

=6 I-COST<br />

(c) The faster the speed, the more land needs to<br />

be cleared on the inside of the curve.<br />

1. It should be shown as an angle measured from due<br />

north.<br />

2. It should be shown measured from north<br />

(or south) in the east (or west) direction.<br />

3. A sketch is important to show the relationships<br />

among the given data and the unknowns.<br />

4. The angle of elevation (or depression from X to Y<br />

is measured from the horizontal line through X to<br />

the ray XY.<br />

5. N<br />

(-4.0)<br />

W'''-+-.-l~+-t-+-+-i-t-+--r.-E<br />

s<br />

The bearing of the airplane measured in a<br />

clockwise direction from due north is 270°. The<br />

bearing can also be expressed as N 90° W, or<br />

S 90° W .<br />

69


<strong>Trigonometry</strong> <strong>Solutions</strong> <strong>Manual</strong><br />

70 Chapter 2 Acute Angles and Right Triangles<br />

6. N<br />

9. y<br />

-E-"T'T-=""'-t--- x<br />

The reference angle is 30°. A point on the ray is<br />

(-.J3, -1). Using the equation y = mx + b, the<br />

The bearing of the airplane measured in a<br />

clockwise direction from due north is 225°. The<br />

bearing can also be expressed '3S S 45° W.<br />

7. N<br />

equation of the ray is y = .J3 x, x ~ 0 since the<br />

3<br />

ray lies in quadrant 111.<br />

10. y<br />

wr.r-+-+-+-++-I~+-t-t--t-H-;-- E<br />

-----+=n---x<br />

(1,-..[3)<br />

s<br />

The bearing of the airplane measured in a<br />

clockwise direction from due north is 315°. The<br />

bearing can also be expressed as N 45° W.<br />

8. N<br />

1800<br />

The reference angle is 60°. A point on the ray is<br />

(1, -.J3). Using the equation y =mx + b, the<br />

equation of the ray is y =-.J3x, x~ 0<br />

since the ray lies in quadrant IV.<br />

11. Let x =the distance the plane is from its starting<br />

point. In the figure, the measure of angle ACB is<br />

40° + (180° - 130°) or 90°. Therefore, triangle<br />

ACB is a right triangle.<br />

N<br />

N<br />

s<br />

The bearing of the airplane measured in a<br />

clockwise direction from due north is 180°. The<br />

bearing can also be expressed as S 0° E or<br />

s o-w,<br />

A x B<br />

Distance traveled in 1.5 hr:<br />

(1.5 hr)(110 mph) = 165 mi<br />

Distance traveled in 1.3 hr:<br />

(1.3 hr)(110 mph) = 143 mi<br />

70


<strong>Trigonometry</strong> <strong>Solutions</strong> <strong>Manual</strong><br />

2.5 Further Applications of Right Triangles 71<br />

Using the Pythagorean theorem. we have<br />

x 2 :;:: 165 2 + 143 2<br />

x 2 :;::47.674<br />

x =220.<br />

The plane is 220 mi from its starting point.<br />

12. Let x =the distance from the starting point.<br />

In the figure, the measure of angle ACB is<br />

27° + (180° - 117°) or 90° . Therefore, triangle<br />

ACB is a right triangle.<br />

Applying the Pythagorean theorem,<br />

x 2 =50 2 + 140 2<br />

2<br />

x = 22,100<br />

x =..)22,100<br />

x=150.<br />

The distance of the end of the trip from the<br />

starting point is 150 krn.<br />

13. Let x =distance the ships are apart.<br />

(1.5 hr)(l8 knotsj e 27 nautical mi<br />

(1.5 hr)(26 knots) =39 nautical mi<br />

Since 130° - 40° =90°, we have a right triangle.<br />

Applying the Pythagorean theorem,<br />

x 2 =27 2 +39 2<br />

x 2 =2250<br />

x=47.<br />

The ships are 47 nautical mi apart.<br />

14. Let C =the location of the ship<br />

and c = the distance between the lighthouses.<br />

N<br />

129 043'<br />

B<br />

Since 50° 17' + 39° 43' :;:: 90°. C =90°. and we<br />

have a right triangle.<br />

. 39043' 3742<br />

Sin =-­ c<br />

3742<br />

c= sin 39°43'<br />

c = 5856<br />

The distance between the lighthouses is 5856 m.<br />

15. Let x =distance between the two ships.<br />

The angle between the bearings of the ships is<br />

180° - (28° 10' + 61° 50') or 90°. The triangle<br />

formed is a right triangle.<br />

Distance traveled at 24.0 mph:<br />

(4 hr) (24.0 mph) =96 mi<br />

Distance traveled at 28.0 mph:<br />

(4 hr)(28.0 mph) =112 mi<br />

Applying the Pythagorean theorem gives<br />

x 2 = 96 2 + 112 2<br />

2<br />

x = 21,760<br />

x = 148.<br />

The ships are 148 mi apart.<br />

16. Let C =the location of the transmitter<br />

and Q = the distance of the transmitter from B.<br />

angle CBA =90° - 53°40'<br />

=36°20'<br />

angle CAB =90 - 36°20'<br />

= 53°40'<br />

A + B = 90°, so C = 90° ..<br />

_Q- = sin A =sin 53°40'<br />

2.50<br />

Q =2.50 sin 53°40'<br />

=2.01<br />

The distance of the transmitter from B is 2.01 mi.<br />

17. Draw triangle WDG with Wrepresenting<br />

Winston-Salem, D representing Danville, and<br />

G representing Goldsboro. Name any point X on<br />

the line due south from D.<br />

N<br />

D<br />

S<br />

Angle BAC =180° - 129° 43'<br />

=50° 17'<br />

B<br />

.L.. -+ ~G<br />

x<br />

s<br />

Since the bearing from W to Dis 42°,<br />

angle WDX =42°.<br />

Angle XDG =48°<br />

So angle D =42° + 48° =90°, and triangle WDG<br />

is a right triangle. Using d =rt and the<br />

Pythagorean theorem, we have<br />

71


<strong>Trigonometry</strong> <strong>Solutions</strong> <strong>Manual</strong><br />

72 Chapter 2 Acute Angles and Right Triangles<br />

WG = ~(WD)2 +(DG)2<br />

23. Let x = the side adjacent to 49.2° in the smaller<br />

triangle.<br />

= ~[60(1)f + [60(1.8)]2 In the larger right triangle:<br />

h<br />

= ..)15,264 tan 29.5° = --­<br />

392+x<br />

'" 120. h = (392 + x) tan 29.5°.<br />

The distance from Winston-Salem to Goldsboro<br />

is 120 mi.<br />

tan49.2°=~<br />

18. Let x = the distance from Atlanta to Augusta. x<br />

In the smaller right triangle:<br />

Atlanta x Augusta h = x tan 49.2°.<br />

N<br />

Substitute the first expression for h in this<br />

I• equation. and solve for x .<br />

! 63° (392 + x) tan 29.5° = x tan 49.2°<br />

392 tan 29.5° + x tan 29.5° = x tan 49.2°<br />

Macon<br />

392 tan 29.5° = x tan 49.2° - x tan 29.5°<br />

Note that the line from Atlanta to Macon makes 392 tan 29.5 = x(tan 49.2° - tan 29.5°)<br />

an angle of 27° + 63°. or 90°. with the line from 392 tan 29.5°<br />

Macon to Augusta.<br />

-------=x<br />

Distance from Atlanta to Macon:<br />

tan 49 .2° - tan 29.5°<br />

Then substitute this value for x in the equation for<br />

60(1~)=75 mi<br />

the smaller triangle.<br />

h = xtan 49.2°<br />

h = 392 tan 29.5°(tan 49.2°)<br />

Distance from Macon to Augusta:<br />

tan 49.2° - tan 29.5°<br />

h"'433<br />

60(Ii) = 105 rni<br />

The height of the triangle is 433 ft.<br />

Use the Pythagorean theorem to find x.<br />

24. Let x = the side adjacent to 52 .5° in the smaller<br />

x 2 = 75 2 + 105 2<br />

triangle.<br />

= 16,650<br />

In the larger triangle:<br />

x'" 130 mi<br />

The distance from Atlanta to Augusta is 130 mi. tan 41.2° = h<br />

168+x<br />

h = (168+ x) tan 41.2°.<br />

19. Solve the equation ax = b + ex for x in terms of<br />

a, b. and e.<br />

In the smaller triangle:<br />

ax =b+ex<br />

tan52.5°=~<br />

ax-ex=b<br />

x<br />

x(a-e) =b<br />

h = x tan52.5°.<br />

b Substitute for h in this equation and solve for x.<br />

x=-­<br />

a-e<br />

(168 + x) tan 41.2° = x tan 52S<br />

168tan 41.2° + x tan 41.2° = x tan 52.5°<br />

21. Using the equation y = (tan 9)(x - a) where (a, 0) 168 tan 41.2° = x tan52.5°-x tan 41.2°<br />

is a point on the line and (J>is the angle the line 168 tan 41.2° = x(tan52.5° - tan 41.2°)<br />

makes with the x-axis, y = (tan 35°)(x - 25). 168 tan 41.2°<br />

-------=x<br />

22. Using the equation y = (tan 9)(x - a) where (a. 0)<br />

tan 52.5° - tan 41.2°<br />

is a point on the line and<br />

Then substitute for x in the equation for the<br />

(J> is the angle the line<br />

makes with the x-axis,<br />

smaller triangle.<br />

y = (tan 15°)(x - 5).<br />

h = x tan 52.5°<br />

h = 168 tan 41.2°(tan 52.5°)<br />

tan 52.5° - tan 41.2°<br />

h",448<br />

The height of the triangle is 448 m.<br />

72


<strong>Trigonometry</strong> <strong>Solutions</strong> <strong>Manual</strong><br />

2.5 Further Applications of Right Triangles 73<br />

25. Let x =the distance from the closer point on the 27. Let x = the height of the antenna<br />

ground to the base of height h of the pyramid.<br />

and h =the height of the house.<br />

135 ft<br />

In the larger right triangle:<br />

h<br />

tan 21°10' =--­<br />

In the smaller right triangle:<br />

135+x<br />

h =(135 + x) tan 21°10'. h<br />

tan 18°10' =­<br />

In the smaller right triangle:<br />

28<br />

h<br />

h = 28(tanI8°1O').<br />

tan 35°30' =­<br />

x<br />

In the larger right triangle:<br />

h = x tan 35°30'.<br />

tan 27<br />

010 '<br />

--­<br />

_ x+ h<br />

Substitute for h in this equation, and solve for x.<br />

28<br />

(135 + x) tan 21°10' = x tan 35°30' x + h =28(tan 27° 10')<br />

135 tan 21°10' + x tan21 °10' =x tan 35°30' x =28(tan27°IO')<br />

135 tan 21°10' =x tan 35°30' - x tan 21°10' -28(tan 18°10')<br />

135 tan 21°10' =x(tan 35°30' - tan 21°10')<br />

X"" 5.18.<br />

135 tan 21°10'<br />

-----:-----=x<br />

The height of the antenna is 5.18 m.<br />

(tan 35°30' -tan21°1O')<br />

Then substitute for x in the equation for the<br />

smaller triangle.<br />

h =135 tan 21° 10'(tan35°30')<br />

(tan 35°30' - tan 21°10')<br />

~.<br />

• D<br />

h"" 114 ~B<br />

The height of the pyramid is 114 ft.<br />

In triangle ADC,<br />

28. Let x be the height of Mt. Whitney above the level<br />

of the road and y be the distance shown in the<br />

figure below.<br />

A<br />

26. Let x = the distance traveled by the whale as it tan 22°40' = ~<br />

approaches the tower and<br />

y<br />

y = the distance from the tower to the<br />

Ytan 22°40' = x<br />

whale as it turns.<br />

x<br />

15·50' Y = tan 22040' . (1)<br />

In triangle ABC,<br />

x<br />

tan 10° 50' =--­<br />

y<br />

. y+7.00<br />

68.7 =tan 35°40'<br />

(y + 7.(0)tan 10°50' =x<br />

y<br />

y tan 10° 50' +7.00 tan 10°50' =x<br />

68.7 x - 7.00 tan 10°50'<br />

y =tan 35°40' Y = tan 10050' (2)<br />

68.7 =tan 15050'<br />

x+y<br />

Substituting (l)into (2), we have<br />

x<br />

x -7.00tinlOo50'~<br />

----=------­<br />

68.7 tan 22°40' tan 10°50'<br />

x+Y=---­ x tan 10°50' =x tan 22°40'<br />

tan 15°50'<br />

68.7 ­ 7.00 tan 10 050'(tan 22°40')<br />

x = tan 15050' y<br />

7.00tan 10 050'(tan 22°40')<br />

68.7 68.7 =x tan 22°40' - x tan 10°50'<br />

x=---­ tan 15°50' tan 35°40'<br />

7.00 tan 10<br />

050'(tan 22°40')<br />

X"" 147<br />

=x(tan 22°40' -tan 10°50')<br />

The whale traveled 147 m as it approached the<br />

x ::::2.47 .<br />

lighthouse.<br />

The height of the top of Mt. Whitney above road<br />

level is 2.47 km.<br />

73


<strong>Trigonometry</strong> <strong>Solutions</strong> <strong>Manual</strong><br />

74 Chapter 2 Acute Angles and Right Triangles<br />

29. (a) From the figure in the text,<br />

b a b /3<br />

d = -cot - + - cot­<br />

222 2<br />

d = !!'(cot a + cot /3).<br />

2 2 2<br />

(b) d = !!.(cot a + cot /3)<br />

2 2 2<br />

Let a = 37'48", /3 = 42'3", and b = 2.<br />

2 ( 37'48" 42'3")<br />

d=- cot---+cot-­<br />

2 2 2<br />

d == 345.395 I m<br />

30. Let h =the minimum height above the surface of<br />

the earth so a pilot at A can see an object on the<br />

horizon at C.<br />

Using the Pythagorean theorem,<br />

(4.00 x 10 3 +h)2 = (4.00 x 10 3 )2 + 125 2<br />

(4000+h)2 = 16,015,625<br />

4000+ h = 4001.952648<br />

h = 1.95.<br />

The minimum height above the surface of the<br />

earth would be 1.95 mi.<br />

31. Let x =the minimum distance that a plant needing<br />

full sun can be placed from the fence .<br />

tan 23OZ0' = 4.65<br />

x<br />

x tan 23°20' = 4.65<br />

4.65<br />

tan 23°20'<br />

x == 10.8<br />

The minimum distance is 10.8 ft.<br />

x=---­<br />

2.<br />

sin A = side opposite = 40 = 20<br />

hypotenuse 58 29<br />

32. tan A =1.0837<br />

1.4923<br />

A = 35.987° or 35°59'10"<br />

1.4923<br />

tan B =-­<br />

1.0837<br />

B = 54.013° or 54°00'50"<br />

33. Let y =the common hypotenuse of the two right<br />

triangles.<br />

x<br />

90° 52°20'<br />

cos 30050' =-­ 198.4<br />

y<br />

198.4<br />

y=---­<br />

cos 30°50'<br />

y = 231.06<br />

To find x, first find the angle opposite x in the<br />

right triangle by subtracting.<br />

52°20' - 30°50' = 51°80' - 30°50'<br />

=21°30'<br />

sin 21030' = .::. = __x_<br />

y 231.06<br />

x = 231.06(sin 21°30')<br />

x == 84.7 m<br />

34. Since angle C is 90° and the angles of a triangle<br />

must sum to 180°, angles A and B must sum to<br />

90°, or be complementary, in order to ensure that<br />

AB is a straight line (and therefore the two ends<br />

meet).<br />

Chapter 2 Review Exercises<br />

. side opposite 60<br />

1. smA= =­<br />

hypotenuse 61<br />

cos A = side adjacent =.!..!.<br />

hypotenuse 61<br />

tan A = side opposite = 60<br />

side adjacent 11<br />

side adjacent 11<br />

cot A = =­<br />

side opposite 60<br />

sec A = hypotenuse = ~<br />

side adjacent 11<br />

esc A = hypotenuse = ~<br />

side opposite 60<br />

cos A = side adjacent = 42 = ~<br />

hypotenuse 58 29<br />

tan A = side opposite = 40 = 20<br />

side adjacent 42 21<br />

side adjacent 42 21<br />

cot A = =-=side<br />

opposite 40 20<br />

hypotenuse 58 29<br />

sec A = =-=side<br />

adjacent 42 21<br />

esc A = hypotenuse = 58 = 29<br />

side opposite 40 20<br />

3. sin 4/3 = cos 5/3<br />

Since sine and cosine are cofunctions,<br />

4/3+ 5/3 = 90°<br />

9/3 = 90°<br />

/3 =10°.<br />

74


<strong>Trigonometry</strong> <strong>Solutions</strong> <strong>Manual</strong><br />

Chapter 2 Review Exercises 75<br />

4. sec(2 y + 10°) = csc( 4y + 20°)<br />

Since secant and cosecant are cofunctions,<br />

(2y + 10°)+ (4y + 20°) = 90°<br />

6y + 30° = 90°<br />

6y =60°<br />

Y =10°<br />

5. tan (5x + 11°) = cot (6x + 2°)<br />

Since tangent and cotangent are cofunctions,<br />

5x + 11°+ 6x + 2° = 90°<br />

l l,r = 90° - 11° - 2°<br />

l l,r = 77°<br />

x=7°.<br />

6. co{~ +l1o)=sinG~ +40 0)<br />

Since cosine and sine are cofunctions,<br />

(~ +l1°)+G~ +40 0) =90°<br />

Q9+51°=9O°<br />

10<br />

7. sin 46° < sin 58°<br />

sin 9 increases as 9 increases from 0 to 90°.<br />

Since<br />

58° > 46°<br />

sin 58° > sin 46°<br />

True<br />

8. cos 47° < cos 58°<br />

False because cosine decreases from 0° to 90°.<br />

9. sec 48° ~ cos 42°<br />

sec 9 ~ 1 for all 9 and cos 9 S 1 for all 9.<br />

Therefore,<br />

cos9S1Ssec9.<br />

True<br />

10. sin 22° ~ esc 68°<br />

For all angles in quadrant I,<br />

sin 9 < 1.<br />

For all angles in quadrant I,<br />

esc 9 > 1.<br />

Therefore,<br />

esc 68° > sin 22°,<br />

and the given statement is false.<br />

12. 120°<br />

This angle is in quadrant II, so the reference angle<br />

is 180° - 120° = 60.<br />

Since 120° is in quadrant II, the cosine, tangent,<br />

cotangent, and secant are negative.<br />

sin 120° = sin 600 = ...[3<br />

2<br />

1<br />

cos 120° = -cos6O° =-­<br />

2<br />

tan 120° = - tan 60° = -...[3<br />

...[3<br />

cot 120° = -cot60°=-­<br />

3<br />

sec 120° = -sec60° =-2<br />

2...[3<br />

esc 120° = csc60°=-­<br />

3<br />

13. 300°<br />

This angle is in quadrant IV, so the reference<br />

angle is 360° - 300° = 60°.<br />

Since 300° is in quadrant IV, the sine, tangent,<br />

cotangent and cosecant are negative.<br />

. 3000 . 600 ...[3<br />

sIn =-SIO =-­ 2<br />

1<br />

cos 300° =cos 60° = ­<br />

2<br />

tan 300° =- tan 60° = -...[3<br />

...[3<br />

cot 300° =-cot 60° =-­<br />

3<br />

sec 300° =sec 60° = 2<br />

2...[3<br />

csc 300° =- csc 60° =--­<br />

3<br />

14. -225°<br />

-225° is coterminal with -225° + 360° = 135°.<br />

The reference angle is 45°. Since -225° is in<br />

quadrant II, the cosine, tangent, cotangent, and<br />

secant are negative.<br />

sin(-225 0) = sin 45° = .,fi<br />

2<br />

.,fi<br />

cos(-225°) = -cos45°=-­<br />

2<br />

tan(-225°) = -tan 45° =-1<br />

cot(-225°) = -cot 45° =-1<br />

sec(-225°) =-sec 45° =-J2<br />

csc(-225°) =csc45° =.,fi<br />

75


<strong>Trigonometry</strong> <strong>Solutions</strong> <strong>Manual</strong><br />

76 Chapter 2 Acute Angles and Right Triangles<br />

15. -390° is an angle in quad rant IV with a reference<br />

angle £)'= 30°.<br />

sin(-390°) = - sin 30° = _!<br />

2<br />

..[j<br />

cos(- 390°) = cos 30° = ­<br />

2<br />

tan(-390°) = - tan 30° = - ..[j<br />

3<br />

cot(-390°) = - cot 30° = -..[j<br />

2..[j<br />

sec(-390°) = sec 30° = -­<br />

3<br />

csc(-3900) = -csc30° =-2<br />

16. sin£)=_!<br />

2<br />

Since sin £) < 0, £) is in quadrant III or IV. Since<br />

sin £) = _!, the reference angle is 30°.<br />

2<br />

In quadrant III, £) = 180° + 30° = 210°.<br />

In quadrant IV, £) = 360° - 30° = 330°.<br />

1<br />

17. cos£)=-­<br />

2<br />

Since the cosine is negative, £) is in quadrant II or<br />

quadrant Ill, Since cos 60° = !, £)' = 60°.<br />

2<br />

In quadrant II,<br />

e =180° - £)'= 180° - 60° = 120°.<br />

In quadrant III,<br />

e = 180° + £)'= 180° + 60° = 240°.<br />

18. cot £) =1<br />

Since cot £) < 0, £) is in quadrant 11 or IV. Since<br />

cot e = -1, the reference angle is 45°.<br />

In quadrant II, £) = 180° - 45° = 135°.<br />

In quadrant IV, £) = 360° - 45° = 315°.<br />

2..[j<br />

19. sec£)=--­<br />

3<br />

Since the secant is negative, £) is in quadrant II or<br />

quadrant III. Since sec 30° = 2..[j £)' = 30°.<br />

3,<br />

In quadrant II,<br />

£) = 180° - £)' = 180° - 30° = 150°.<br />

In quadrant III,<br />

£) = 180° + £)'= 180° + 30° = 210°.<br />

2<br />

20. cos600+2sin 30°= ~+ 2(~r<br />

=~+2(±)<br />

1 1<br />

=-+­<br />

2 2<br />

=1<br />

2<br />

21. tan 120 0-2cot2400 = (_..[j)2 -2(~)<br />

7<br />

=­<br />

2<br />

=3- 2..[j<br />

3<br />

In Exercises 24-29, be sure that your calculator is in<br />

degree mode. Keystroke sequences may vary based on<br />

the type andlor model of calculator being used.<br />

24. sin 72° 30'<br />

72030' = (72+ ~)0 "= 72.5<br />

Enter: 72.5 ~<br />

or ~ 72.5 ~<br />

Display: .95371695<br />

25. sec 222° 30'<br />

222°30' =(222 + ~~) 0<br />

= 222.50<br />

Enter: 222.5 ~ (jill<br />

or CD ~ 222.5 CD IE:) ~<br />

Display: -1.3563417<br />

26. cot 305.6°<br />

Enter: 305.6 ~ (jill<br />

or CD ~ 305.6 CD IE:) ~<br />

Display: -.71592968<br />

27. esc 78°21'<br />

78° 21'=(78+ ~~y =78.35°2<br />

Enter: 78.35 8 (jill<br />

or CD ~ 78.35 CD IE:) ~<br />

Display: 1.0210339<br />

76


<strong>Trigonometry</strong> <strong>Solutions</strong> <strong>Manual</strong><br />

Chapter 2 Review Exercises 77<br />

28. sec 58.9041° 34. sec 8 = 1.2637891<br />

Enter: 58.9041 @[) @I<br />

or OJ @[) 58.9041 CD IE:) E!§<br />

Display : 1.9362132<br />

29. tan 11.7689°<br />

Enter: 11.7689 (E)<br />

or ~ 11.7689 E!§<br />

Display: .20834446<br />

Enter : 1.2637891 @I {INV AR~~OS<br />

or<br />

ICOII"1<br />

or ~ @[) 1.2637891 IE:) E!§<br />

Display : 37.695528<br />

8 = 37.695528°<br />

30. If 8 = 135°,8 = 45° .<br />

If8=45°, 8=45°.<br />

If 8 = 300°, 8 = 60°.<br />

If 8 = 140°, 8 = 40°.<br />

Of these reference angles 40° is the only one<br />

which is not a special angle, so (d) tan 140° is the<br />

only one which cannot be determined exactly.<br />

In Exercises 31-38, be sure that your calculator is in<br />

degree mode. Keystroke sequences may vary based on<br />

the type and/or model of calculator being used.<br />

31. sin 8 = .8254121<br />

AR~SIN<br />

Enter: .82584121{INV 8<br />

or<br />

1·..·'1<br />

or 0 8 .82584121 ~<br />

Display: 55.673870<br />

8 = 55.673870°<br />

32. cot 8 = 1.1249386<br />

Enter' 1.1249386<br />

or<br />

Gill {INV ARCTAN<br />

or<br />

IT.....J<br />

or IE) (E) 1.1249386 IE:) E!§<br />

Display: 41.635092<br />

8 = 41.635092°<br />

33. cos 8 = .97540415<br />

Enter: '97540415{INV AR~~OS<br />

or<br />

[cos"J<br />

or 0 @[) .97540415 ~<br />

Display: 12.733938°<br />

8 = 12.733938°<br />

35. tan 8 = 1.9633124<br />

Enter: 1.9633124{INV AR~~AN<br />

or<br />

IT.....I<br />

or 0 (E) 1.9633124 ~<br />

Display: 63.008286<br />

8 = 63.008286°<br />

36. esc 8 = 9.5670466<br />

Enter :9.5670466 @I {INV AR~SIN<br />

or<br />

1"'~ 1 .<br />

or ~ 8 9.5670466 IE:) E!§<br />

Display: 5.9998273<br />

8 = 5.9998273 0<br />

37. sin 8 = .73254290<br />

Since sin 8 is positive, there will be one angle in<br />

quadrant I and one angle in quadrant IT. If 8', is<br />

the reference angle, then the two angles are 8'<br />

and 180° - 8'.<br />

Enter: .73254290 INV 8 .<br />

or ~ 8 .73254290 ~<br />

Display: 47.100000<br />

8'=47.1 0<br />

180° - 8' = 180° - 47.1° = 132.9°<br />

38. Since tan 8 = 1.3865342, 8 is in quadrant I or<br />

quadrant III. If 8' is the reference angle, the two<br />

angles will be 8' and 180° + 8'.<br />

Enter: l.3865342{INV AR~~AN ~<br />

or<br />

[T.....I<br />

or IE) (E) 1.3865342 ~<br />

Display: 54.200000<br />

8' = 54.2°<br />

180 0 + 8' = 180 0 + 54.2° = 234.2°<br />

77


<strong>Trigonometry</strong> <strong>Solutions</strong> <strong>Manual</strong><br />

78 Chapter 2 Acute Angles and Right Triangles<br />

39. sin 50° + sin 40° 1. sin 90°<br />

sin 50° + sin 40° = 1.4088321 and<br />

sin 90° = 1<br />

False<br />

40. cos21O° 1. cos 180° · cos 30° -sin 180° ·sin30°<br />

cos 210° = _ .J3<br />

2<br />

cos 180° .cos 30° - sin 180° .sin 30°<br />

=(-1)( ~J-(o{-})<br />

.J3<br />

=-­<br />

2<br />

True<br />

41. sin 240° 1. 2sin 120° cos 120°<br />

sin 240° = - sin 600 = _ .J3<br />

2<br />

2sin 120°· cos 120° = 2(sin600)(-cos600)<br />

=2( ~J(--})<br />

True<br />

.J3<br />

=-­ 2<br />

42. sin 42° + sin 42 1. sin 84<br />

Using a calculator. we have<br />

sin 42° + sin 42° = 1.3382612.<br />

sin 84° = .99452190<br />

False<br />

1<br />

43. No. cot 25° = --­<br />

tan 25°<br />

Enter: 25 ~ om<br />

or OJ ~ 25 CIJ 0 ~<br />

Tan- 125 is calculating the angle whose tangent<br />

is 25.<br />

46. e = 4000°<br />

cos40000= .766044443<br />

sin 4000° = .642787610<br />

Since cosine and sine are both positive. the<br />

angle (} is in quadrant I.<br />

47. A =58° 30'. c= 748<br />

A+B=90°<br />

B=900-A<br />

B = 90° - 58°30'<br />

B= 31°30'<br />

sin A =!:<br />

c<br />

a = csinA<br />

a = 748sin58°30'<br />

a=638<br />

b<br />

cos A =­<br />

c<br />

b=ccosA<br />

b =748 cos 58°30'<br />

b=391<br />

48. a =129.70. b = 368.10<br />

tan A = 129.70<br />

368.10<br />

A = 19.41° = 19°25'<br />

tan B = 368.10<br />

129.70<br />

B = 70.59° = 70°35'<br />

Check that A + B =90°.<br />

c=~a2 +b 2 .<br />

=~129.702 +368.10 2<br />

c=390.28<br />

49. A =39.72°. b = 38.97 m<br />

A<br />

44. (} = 2976°<br />

cos 2976° = - .104528463<br />

sin 2976° = .994521895<br />

Since cosine is negative and sine is positive. the<br />

angle (} is in quadrant II.<br />

45. (} = 1997°<br />

cos 1997° = -.956304756<br />

sin 1997° =-.292371705<br />

Since sine and cosine are both negative. the<br />

angle (} is in quadrant III. .<br />

78


<strong>Trigonometry</strong> <strong>Solutions</strong> <strong>Manual</strong><br />

Chapter 2 Review Exercises 79<br />

A+B=900=C 53. The diagonal d of the rectangle is the hypotenuse<br />

B=900-A<br />

of a right triangle whose legs are the adjacent<br />

B = 90° - 39.72°<br />

sides of the rectan Ie.<br />

= 50.28°<br />

tanA=!:<br />

b<br />

a=btanA<br />

10 .93cm<br />

a = 38.97 tan 39.72° 35.65° 90°<br />

a =32.38 m<br />

15.24cm<br />

b<br />

By the Pythagorean theorem, the length d of the<br />

cosA=c<br />

diagonal is<br />

b<br />

c=-­d = ~r(I-5-.2-4)--:2-+-(-10-.-93-)7"2 "" 18.75 em,<br />

cos A<br />

38.97<br />

An alternative method gives the same answer.<br />

c=---­<br />

cos 39.72° Sin . 35<br />

. 650 =-­ 10.93<br />

c=50.66 m<br />

d<br />

d = 10.93<br />

SO. B = 47° 53', b = 298.6 m sin35.65°<br />

"" 18.75 em<br />

90·<br />

A 298.6m C<br />

C=9O°<br />

A = 90° - 47°53'<br />

= 42°7'<br />

B<br />

54. Let s = the length of one of the equal sides of an<br />

isosceles triangle. Divide the isosceles triangle<br />

into two congruent right triangles.<br />

49.28m<br />

....--..--­<br />

-Q- = tan 42°7'<br />

d<br />

298.6 1<br />

a = 298.6(tan 42°7')<br />

d = -(49.28 m) = 24.64 m<br />

2<br />

a =270.0 m<br />

(J =.!..(58.746°) = 29.373°<br />

_. _c_ = esc 47°53° 2<br />

298.6<br />

• £I d<br />

c = 298.6(csc47°53') Sin 17 =­<br />

S<br />

c=402.5 m<br />

Sin.<br />

. 29 3730 =-­ 24.64<br />

51. Let x = height of the tower. s<br />

24.64<br />

tan 38020' = _x_<br />

93.2<br />

sin 29.373°<br />

s =50.24<br />

x = 93.2 tan 38°20'<br />

Each side is 50.24 m long.<br />

x "" 73.693005<br />

.The height of the tower is 73.7 ft.<br />

52. Let h = height of the tower .<br />

h<br />

tan 29.5° =-­<br />

36.0<br />

h = 36.0 tan 29.5°<br />

h ""20.4<br />

The height of the tower is 20.4 m.<br />

s=---­<br />

79


<strong>Trigonometry</strong> <strong>Solutions</strong> <strong>Manual</strong><br />

80 Chapter 2 Acute Angles and Right Triangles<br />

55. Draw triangle ABC and extend the north-south<br />

lines to a point X south of A and S to a point Y,<br />

north of C.<br />

N<br />

57. Suppose A is the car heading south at 55 mph,<br />

B is the car heading west, and point C is the<br />

intersection from which they start.<br />

After two hours, by d = rt,<br />

AC= 55(2) =110<br />

Since angle ACB is a right angle, triangle ACB is<br />

a right triangle.<br />

Since the bearing of A from B is 324°,<br />

angle CAB = 360° - 324° = 36°.<br />

w B C<br />

S<br />

AngleACB = 344° - 254°:;:; 90°, so ABC is a<br />

right triangle.<br />

Angle BAX = 32° since it is an alternate interior<br />

angle to 32°.<br />

Angle YCA = 360° - 344° = 16°<br />

Angle XAC = 16° since it is an alternate interior<br />

angle to angle YCA.<br />

Angle BAC = 32° + 16° = 48°<br />

In triangle ABC,<br />

AC<br />

cosA=­<br />

AB<br />

AB= AC<br />

cos A<br />

AB=~<br />

cos 48°<br />

= 1200.<br />

The distance from A to B is 1200 m.<br />

56. Draw triangle ABC and extend north-south lines<br />

from points A and B. Angle ABX is 55° (alternate<br />

interior angle 3 of parallel lines cut by a<br />

transversal are congruent) so Angle ABC is<br />

55° + 35° =90° .<br />

N N<br />

A<br />

x<br />

c<br />

6'o~<br />

B<br />

Since angle ABC is a right angle, use the<br />

Pythagorean theorem to find the distance<br />

from A to C.<br />

(AC)2 =80 2 +74 2<br />

(AC)2 =6400 + 5476<br />

AC = ~ll,876<br />

= 110 km<br />

It is 110 km from A to C.<br />

s<br />

A<br />

AC<br />

cosCAB=­<br />

AB<br />

110<br />

AB= AC =__<br />

cos CAB<br />

::: 140 mi<br />

58. Let x =the leg opposite angle A.<br />

x h+x<br />

tan A =- and tan B :;:;-­<br />

k<br />

k<br />

x=ktanAandx=ktanB- h<br />

Therefore,<br />

k tan A =k tan B - h<br />

h = k tan B - k tan A<br />

h =k(tan B - tan A) .<br />

side adjacent AB<br />

61. cot () = =­<br />

side opposite OA<br />

Since OA =r =1, cot() =AlJ<br />

62. Label the point (0, 1) as P.<br />

sec () = hypotenuse =OC<br />

side adjacent OP<br />

Since OP =r =I , sec () =OC<br />

63. esc () = hypotenuse = OB<br />

side opposite OA<br />

Since OA =r = I, esc () = OB.<br />

64. (a) Let R:;:; 3955, T= 25, P = 140.<br />

h = {.os(~) 1)<br />

= 395{ co,(~) -I)<br />

=716 mi<br />

80


<strong>Trigonometry</strong> <strong>Solutions</strong> <strong>Manual</strong><br />

Chapter 2 Test Exercises 81<br />

(b) Now let T~ 30.<br />

h-J 1 -1] -"l cos(~)<br />

~395{ cos (:Vl 1]<br />

'" 1104 mi<br />

In order to increase the communication<br />

time T, the satellite must have a higher orbit.<br />

65. (a) From the figure in the text,<br />

xQ-xp<br />

sin 8 =~'----'-<br />

d<br />

xQ = x p +dsin8.<br />

Similarly,<br />

YQ -YP<br />

cos 8 =-=--­<br />

d<br />

YQ =Yp +dcos8.<br />

(b) Let (x p ' Yp)= (123.62,337.95),<br />

8 = 17° 19' 22", and d = 193.86.<br />

xQ = x p +dsin8<br />

=123.62 + 193.86sin 17°19'22"<br />

=123.62 + 193.86sinI7.3228°<br />

'" 181.34<br />

YQ =Yp +dcos8<br />

=337.95 + 193.86cos 17°19'22"<br />

=337.95 +193.86 cos 17.3228°<br />

'" 523.02<br />

The coordinates of Q are<br />

(181.34,523.02).<br />

Chapter 2 Test Exercises<br />

1. sin A = side opposite =12<br />

hypotenuse 13<br />

cos A =side adjacent =~<br />

hypotenuse 13<br />

tan A = side opposite =.!3.<br />

side adjacent 5<br />

side adjacent 5<br />

cot A = =­<br />

side opposite 12<br />

sec A = hypotenuse =~<br />

side adjacent 5<br />

esc A = hypotenuse =~<br />

side opposite 12<br />

2. To find w:<br />

sin 30° =.i<br />

w<br />

4<br />

w=-­<br />

sin 30°<br />

4<br />

w=­<br />

(!..)<br />

2<br />

w=8<br />

To find x:<br />

4<br />

tan45°=x<br />

4<br />

x=-­<br />

tan 45°<br />

4<br />

x=­ I<br />

x=4<br />

To findy:<br />

cos 30° =1:.<br />

w<br />

cos 30° = 1'.<br />

8<br />

8cos30° =Y<br />

/3<br />

8·-=y<br />

2<br />

4/3 =y<br />

To find z:<br />

sin 45° =.i<br />

z<br />

4<br />

z=-­<br />

sin 45°<br />

2<br />

z=-­<br />

(..[2)<br />

2<br />

z=4...[i<br />

3. sin (B + 15°)=cos (2B + 30°)<br />

Since sine and cosine are cofunctions, the<br />

equation is true when<br />

(B + 15°) + (2B + 30°) =90°<br />

3B+45 =90°<br />

3B=45°<br />

B=15°<br />

This is one solution; others are possible.<br />

4. sin 8 =.27843196<br />

INV 8<br />

or<br />

Enter: .27843196 ARCSIN<br />

81


<strong>Trigonometry</strong> <strong>Solutions</strong> <strong>Manual</strong><br />

82 Chapter 2 Acute Angles and Right Triangles<br />

5.<br />

or lEJ ~ .27843196 ~<br />

Display: 16.16664145<br />

(J=16.16664145°<br />

-.,fi<br />

cos(J=-­<br />

2<br />

Cosine is negative in quadrants II and III. The<br />

reference angle (J' is 45°. In quadrant II (J is<br />

180° - 45° = 135°. In quadrant III (J is<br />

180° + 45° = 225°.<br />

6. tan (J =1.6778490<br />

To find cot (J,<br />

Enter: 1.6778490 (ill)<br />

or 1.6778490 0 ~<br />

Display: .5960011896<br />

cot (J =.5960011896<br />

7. (a) sin24° J: sin 48°<br />

Sin (Jincreases from 0 to 1in the interval<br />

0°::';(J::';90°. Therefore sin 24° is less<br />

than sin 48° .<br />

True<br />

(b) cos24°


<strong>Trigonometry</strong> <strong>Solutions</strong> <strong>Manual</strong><br />

Chapter 2 Test Exercises 83<br />

Angle ABX is 55° (alternate interior angles of<br />

parallel lines cut by a transversal are congruent).<br />

Angle ABC is 55° + 35° = 90°, therefore triangle<br />

ABC is a right triangle.<br />

AC 2 =S02 +74 2<br />

AC=~S02 +74 2<br />

AC=llOkm<br />

k<br />

k.Jh<br />

t =-- or -- seconds<br />

4h<br />

4.Jh<br />

14. v=32tsin9<br />

v =32(k;';)(I)<br />

v = s.Jh ft per sec<br />

The velocity depends only on the height h;<br />

the length k does not affect the velocity .<br />

..<br />

83


<strong>Trigonometry</strong> <strong>Solutions</strong> <strong>Manual</strong><br />

84 Chapter 3 Radian Measure and the Circular Functions<br />

CHAPTER 3 RADIAN MEASURE AND THE<br />

CIRCULAR FUNCTIONS<br />

Section 3.1<br />

Exercises<br />

1. Since 0 is in quadrant I,<br />

n<br />

0


<strong>Trigonometry</strong> <strong>Solutions</strong> <strong>Manual</strong><br />

3.1 Radian Measure 85<br />

34.<br />

lltr = IlJrC800) =660<br />

30 30 tr<br />

35. 39° = 39(...!!..- radian) = .68 radian<br />

180<br />

36. 74° = 74(~ radian) = 1.29 radians<br />

180<br />

37. 42°30' = 42.5(~ radian) = .742 radians<br />

180<br />

38. 53°40' = 53.6667(...!!..- radian) = .937 radians<br />

180<br />

39. 139°10' = 139"!'(...!!..- radian) = 2.43 radians<br />

6 180<br />

40. 174°50' = 174.833\...!!..- radian) = 3.05 radians<br />

180<br />

41. 64.29° = 64.29(.s: radian)<br />

180<br />

= 1.122 radians<br />

42. 85.04° = 85.04(...!!..- radian) = 1.484 radians<br />

180<br />

43. 56025' = 56 25 ( .s: radian)<br />

60 180<br />

= .9847 radian<br />

44. 122°37' = 122.61~...!!..- radian) = 2.140 radians<br />

. 180<br />

45. 47.6925° = 47.6925(.s. radian)<br />

180<br />

= .832391 radian<br />

46. 23.0143° = 23.0143(.s. radian)<br />

180<br />

= .401675 radian<br />

0)<br />

47. 2 radians = 2 C80 K<br />

= 114.5916°<br />

= 114° + (.5916)(60')<br />

= 114°35'<br />

48. 5 radians = 5 C<br />

K 8OO)<br />

= 286.4789°<br />

= 286° + (.4789)(60')<br />

= 286°29'<br />

0)<br />

49. C80<br />

1.74 radians = 1.74 K<br />

=99.6947°<br />

= 99° + (.6947)(60')<br />

=99°42'<br />

0)<br />

50. o{180<br />

3.06 radians = 3. K<br />

= 175.3251°<br />

= 175° + (.3251)(60')<br />

= 175°20'<br />

51. .3417 radians = .34 17 (180°) K<br />

= 19.5780°<br />

= 19° + (.5780)(60')<br />

= 19°35'<br />

52. 9.84763 radians = 9.8476 \180°) K<br />

= 564.228°<br />

= 564° +


<strong>Trigonometry</strong> <strong>Solutions</strong> <strong>Manual</strong><br />

86 Chapter 3 Radian Measure and the Circular Functions<br />

58. cos : = cos(i,180°) Substitute 180° for TC. 67. sin 5: =sin(~'1800)<br />

= cos30° = sin 150°<br />

..[3 1<br />

=­<br />

=­<br />

2<br />

2<br />

59. tan~ = tan( ~ . 1800) 68. tan 5: = tan( ~ . 1800)<br />

= tan45°<br />

= tan 150°<br />

=1 ..[3<br />

=-­<br />

3<br />

60. cot ~ = cot( ~ .1800)<br />

69. cos 3TC = cos(3. 180°)<br />

= cot 60° = cos540°<br />

=-1<br />

..[3<br />

=­<br />

3 70. see TC = sec180°<br />

=-1<br />

61. sec: = sec(i,180°)<br />

71. sin 4 TC ~ sin( i .1800)<br />

= sec30° . 3 3<br />

2..[3 = sin 240°<br />

3 ..[3<br />

=-­<br />

2<br />

62. csc~ = esc( ~ '180°)<br />

= esc 45° 72. cot(- 2;)= co{-~'1800)<br />

=../2 = cot(-120°)<br />

63. sin ~ = sin(± , 180°)<br />

..[3<br />

=­<br />

3<br />

= sin90°<br />

=1 73. Sin(- 7:) = Sin(-i ,180°)<br />

= sin(-2100)<br />

64. esc ~ = esc(± ,180°) 1<br />

=­<br />

2<br />

= esc90°<br />

=1<br />

74. cos(- :)= cos(-i, 1800)<br />

65. tan 2;= tan(~ ,180°) = cos(-300)<br />

= tan120° ..[3<br />

=­<br />

=-..[3 2<br />

66. cot3<br />

2TC<br />

= cot (2 "3' 180° )<br />

= cot120°<br />

..[3<br />

=-­<br />

3<br />

86


<strong>Trigonometry</strong> <strong>Solutions</strong> <strong>Manual</strong><br />

3.1 Radian Measure 87<br />

75. Answers start at 30° and proceed<br />

counterclockwise around circle.<br />

30° =30 (..!!.... radian) =!E. radian<br />

180 6<br />

x . tr (1800)<br />

- radian =- -- =45°<br />

4 4 tr<br />

60° =60 (..!!.... radian) =tr radians<br />

180 3<br />

2tr diai 2tr (1800) 1200<br />

- ra lans = - -- =<br />

3 3 tr<br />

3tr. 3tr (1800)<br />

4 4 tr<br />

- radians =- -- =135°<br />

150° = 150(..!!.... radian) = 5tr radians<br />

180 6<br />

180°=180(..!!.... radian) =zrradians<br />

180<br />

210° =21O(..!!.... radian) =7tr radians<br />

180 6<br />

225° =225 (..!!.... radian) =5tr radians<br />

180 4<br />

4tr. 4tr (1800)<br />

3 3 tr<br />

- radians =- -- =240°<br />

-<br />

5tr.<br />

radians = -<br />

5tr (1800)<br />

-- = 300°<br />

3 3 tr<br />

315° =315(..!!.... radian) =7n radians<br />

180 4<br />

330° = 330(..!!.... radian) = 1m radians<br />

180 6<br />

76. In degree mode, using tan 9 =!...,<br />

x<br />

tan9<br />

6.3018678<br />

=<br />

9<br />

(} =35°.<br />

In radian mode, using tan 9 =!...,<br />

x<br />

tan(} =6.3018678<br />

9<br />

(} = .611 radian.<br />

9 n tr<br />

77. (a) _ ._=­<br />

10 180 200<br />

(b) Degrees: 3.5 x .9° =3.15°<br />

Radians: 3.5 x ..!!.... ::; .055 radian<br />

200<br />

79. In each rotation, the pulley would rotate 2tr<br />

radians.<br />

(a) In 8 rotations, the pulley would tum<br />

8 . 2tr =2tr radians.<br />

(b) In 30 rotations, the pulley would tum<br />

30 . 2tr =60tr radians.<br />

80. (a) The hour hand completes 2 revolutions in<br />

24 hr.<br />

One revolution is 2tr.<br />

(} =2(2tr) = 4tr<br />

(b) In 4 hr it rotates through<br />

(} =C~)2tr) =2; .<br />

12<br />

81. In each rotation around the earth, the space<br />

vehicle would rotate 2tr radians.<br />

(a) In 2.5 orbits, the space vehicle travels<br />

(2.5)' 2tr= 5trradian~.<br />

(b) In ±of an orbit, the space vehicle travels<br />

3<br />

(~) . 2tr =8: radians.<br />

82. (a) S =1367, N = 80<br />

.1S =.034Ssin [2tr(82.5 - N)J<br />

365.25<br />

=.034(1367)sin [2tr(82.5 - 80)J<br />

365.25<br />

::; 1.998 watts per square m<br />

(b) S =1367, N = 1268<br />

.1S=.034S sin [2tr(82.5 - N) J<br />

. 365.25<br />

= .034(1367) sin [2tr(82.5 -1268)J<br />

365.25<br />

::; -46.461watts per square m<br />

78. The values of sin tr and cos tt are 0 and -1,<br />

respectively, when zrrepresents a measurement in<br />

radians, not degrees. Therefore, the calculator was<br />

in radian mode<br />

87


<strong>Trigonometry</strong> <strong>Solutions</strong> <strong>Manual</strong><br />

88 Chapter 3 Radian Measure and the Circular Functions<br />

(c) The maximum value of!!.S would occur 3n<br />

5. 5=6n (}=when<br />

, 4<br />

. [2n(82.5 - N)J 1 5 = r(}<br />

sm = .<br />

365.25 5 6n<br />

In this case !!.S would be equal to<br />

r=(j=Jir<br />

T<br />

.034S =.034(1367) 4<br />

=46.478 watts per square m =6n·­<br />

3n<br />

=8<br />

(d) !!.S= 0, so<br />

N)J 0<br />

sm<br />

. [2n(82.5 -<br />

= .<br />

n<br />

6. 5=3n (}=­<br />

365.25 , 2<br />

This occurs when N = 82.5,265.125,447.75,<br />

5 =r(}<br />

630.375,813,995.625,1178.25, or 5 3n<br />

r=-=­<br />

1360.875. Since N represents a day number,<br />

(} t<br />

which should be a natural number, we might<br />

interpret day 82.5 as noon on the 82nd day.<br />

Section 3.2 =6<br />

Connections (page 112) 7. r=3,5=3<br />

Answers will vary. The longitude at Greenwich is 0°.<br />

Exercises<br />

1. 5 =r(}<br />

1<br />

2<br />

=3n·­<br />

5 =r(}<br />

x<br />

(}=~=~=1<br />

r 3<br />

8.5=6,r=4<br />

5 = r(}<br />

1=-(} 5 6 3<br />

2<br />

(} = - = - = - or 1.5<br />

r 4 2<br />

(}=2<br />

The gas gauge rotates through 2 radians.<br />

·2 (} 2n di<br />

9• r ='1 .3 em, =- 'ra tans<br />

. . 3 ·<br />

2. Since the seesaw is 12 ft long, the radius of the<br />

seesaw is 6 ft.<br />

s =r(}<br />

3 =6(} =8.2n em<br />

1<br />

(}=­<br />

::::25.8 em<br />

2<br />

The board rotates through '2 1 radian.<br />

5 =r(} =12.3(2;) em<br />

10. r =.892 em, (} =lIn radians<br />

10<br />

n<br />

3. r=4 (}=­<br />

, 2<br />

5 =r(} =.892(\1;) cm<br />

=.9812n cm<br />

s = r(} = { ; ) = 2n<br />

:::: 3.08 em<br />

11. r =4.82 m, (} =60°<br />

Convert (} to radians.<br />

4. r =12<br />

,<br />

(} = n<br />

3<br />

n<br />

(} =60°=_<br />

3<br />

s = re = 12( ~ ) = 4n<br />

s =r(} =4.82( ~ )<br />

= 1.61n<br />

::::5.05m<br />

88


<strong>Trigonometry</strong> <strong>Solutions</strong> <strong>Manual</strong><br />

3.2 Applications of Radian Measure 89<br />

12. r= 71.9 em, 0= 135°<br />

Convert 0 to radians.<br />

135° = 135(-.!!....- radian) = 31t' radians<br />

180 4<br />

s =rO =71.ge: )cm =53.9251t'cm<br />

== 169 em<br />

13. r = 2 in., s = 5 in.<br />

s=rO<br />

o= ~ =~ or 2.5 radians<br />

r 2<br />

14. 0 =2 radians, s =3 ft<br />

s=rO<br />

s 3<br />

r =- =- or 1.5 ft<br />

o 2<br />

For Exercises 19-24, note that since 6400 has two<br />

significant digits and the angles are given to the nearest<br />

degree, we can have only two significant digits in<br />

the answers.<br />

19. 9° N, 40° N<br />

o= 40° - 9° = 31°<br />

s=rO<br />

nt1 311t')<br />

=64 \180<br />

== 3500 km<br />

= 31 (-.!!....- radian)<br />

180<br />

=-<br />

311t'<br />

ra<br />

di<br />

Ian<br />

180<br />

20: 36° N, 49°N<br />

o= 49° - 36° = 13°<br />

15. 0 =1t' radian, s =4 in.<br />

5<br />

s=rO<br />

s 4<br />

r=-=o<br />

1­<br />

=4 '~<br />

1t'<br />

20 .<br />

=-In.<br />

1t'<br />

s=rO<br />

s =~AntI 131t')<br />

~180<br />

== 1500 km<br />

= 13 (~ radian)<br />

180<br />

=- 131t' ra diIan<br />

180<br />

16. s = 30 em, r = 5 em<br />

s=rO<br />

o=-=-=<br />

s 30 6<br />

ra<br />

d"'<br />

ians<br />

r 5<br />

17. The formula for arc length is s = rO. Substituting<br />

2r for r we obtain s = aO. The length of the arc is<br />

doubled.<br />

18. Recall that z radians =180°. If 0 is measured in<br />

degrees, then the formula becomes<br />

91r 1t'r0<br />

s=r·-=-.<br />

180 180<br />

22. 45° N, 34° S<br />

34° S =-34° N<br />

0= 45°- (-34°) =79°<br />

s=rO<br />

s = 64nt1 79Jt)<br />

\180<br />

== 8800 krn<br />

=--<br />

79Jt<br />

ra<br />

d'<br />

lans<br />

180<br />

89


<strong>Trigonometry</strong> <strong>Solutions</strong> <strong>Manual</strong><br />

90 Chapter 3 Radian Measure and the Circular Functions<br />

23. r= 6400 km, s =1200 km<br />

s= r8<br />

1200 =64008<br />

8=1.­<br />

16<br />

Convert l.- radian to degrees.<br />

16<br />

0)<br />

8 = 1.-(180 '" 11°<br />

16 1r<br />

The north-south distance between the two cities<br />

is 11°.<br />

Let x =the latitude of Madison.<br />

x-33°=II°<br />

x=44° N<br />

24. r= 6400 km, s =1100 Ian<br />

s =rO<br />

1100 =64000<br />

.!..!. =0<br />

64<br />

Convert .!..!. radian to degrees.<br />

64<br />

e= .!..!..180° '" 100<br />

64 1r<br />

The north-south distance between the two cities<br />

is 10°.<br />

Let x =the latitude of Toronto.<br />

x-33° = 10°<br />

x=43° N<br />

25. (a) The number of inches lifted is the arc length<br />

in a circle with<br />

r = 9.27 in. and 0 = 71° 50'.<br />

71050' = (71 ° + 50<br />

60 180°<br />

s =rO<br />

0<br />

)(...!E-)<br />

= 9.27(71 ° + 50·X...!E-)<br />

60 180°<br />

= 11.6<br />

The weight will rise 11.6 in.<br />

(b) When the weight is raised 6 in.,<br />

s =rO<br />

O=!..<br />

r<br />

=_6(1800)<br />

9.27 1r<br />

= 37.085° = 37° + (.085)(60')<br />

= 37°5'.<br />

The pulley must be rotated through 37° 5'.<br />

26. To find the radius of the pulley, first convert 51.6°<br />

to radians.<br />

o= 51.6° = 51.6 (...!!-) radians<br />

180<br />

Now substitute this value of 0 and s = 11.4 em<br />

into the equation s = rO and solve for r.<br />

s =rO<br />

11.4 = r(51.6)(~)<br />

180 .<br />

11.4<br />

r =-"""7"----,­<br />

51.6(rn,)<br />

r = 12.7<br />

The radius of the pulley is 12.7 em.<br />

27. A rotation of<br />

o= 60.00 = 1r<br />

3<br />

on the smaller wheel moves through an arc length<br />

of<br />

s = rO =5 .2'~) =5.48 em.<br />

Since both wheels move together, the larger<br />

wheel moves 5.48 em, which rotates it through an<br />

angle 0, where<br />

5.48=8.160<br />

0= 5.48 = .671 radian<br />

8.16<br />

= .67 { 1 ~ 0 ) ':" 38.50.<br />

The larger wheel rotates through 38.5°.<br />

28. The arc length s represents the distance traveled<br />

by a point on the rim of a wheel. Since the two<br />

wheels rotate together, s w~1I be the same for both<br />

wheels.<br />

For the smaller wheel,<br />

o= 80° = 8rI...!!-) = 41r radians.<br />

\180 9<br />

s= -o = 11.\~) "" 16.336 em.<br />

For the larger wheel,<br />

o= 50° = 50 (...!!-) =51r radian.<br />

180 18<br />

s =rO<br />

16.336 = {~;)<br />

16.336<br />

r =---s;r-<br />

IB""<br />

r = 18.7<br />

The radius of the larger wheel is 18.7 em.<br />

90


<strong>Trigonometry</strong> <strong>Solutions</strong> <strong>Manual</strong><br />

29. The chain moves a distance equal to the arc length<br />

on the larger gear. So, for the large gear and<br />

pedal,<br />

s = r()<br />

= 4.72(180)...!:.­<br />

180<br />

= 4.72".<br />

Thus, the chain moves 4.72" in. The small gear<br />

rotates through an angle<br />

()=!..<br />

r<br />

4.72"<br />

=-­<br />

1.38<br />

=3.42".<br />

()for the wheel and ()for the small gear are the<br />

same, or 3.42". So, for the wheel,<br />

s =r()<br />

= 13.6(3.42,,)<br />

= 146 in.<br />

The bicycle will move 146 in.<br />

30. (a) In one hour, the truck travels 55 mi. Since<br />

the radius is given in in., convert 55 mi<br />

to in.<br />

s=55 mi<br />

= 55(5280) ft<br />

= 55(5280)(12) in.<br />

= 3,484,800 in.<br />

s =r()<br />

3,484,800 in. = (14 in.)()<br />

248,914 .29 radians = ()<br />

Each rotation is 2" radians. Thus, the number<br />

of rotations is<br />

9 ' 248,914.29<br />

-=---'--­<br />

2" 2"<br />

= 39,616 rotations.<br />

(b) Find s for the 16-in. wheel.<br />

s=r9<br />

= (16 in.)(248,914.29)<br />

= 3,982, 628.64 in.<br />

=62.9 mi<br />

The truck with the 16-in. tires has gone<br />

62.9 mi in one hour, so its speed is 62.9 mph.<br />

Yes, the driver deserves a ticket.<br />

3.2 Applications of Radian Measure 91<br />

31. Let t = the length of the train. t is approximately<br />

the arc length subtended by 3° 20'.<br />

9=3°20'=3,(<br />

3<br />

() =(3~)C:0)<br />

=<br />

54<br />

"<br />

t = r() = 3.5(~)<br />

=.20 km<br />

The tra~n is about .20 km long.<br />

32. Let L = the length of the ship.<br />

L is approximately the arc length subtended<br />

by 1.5°.<br />

L= r()<br />

= (2.3)(1.5)(...!!-)<br />

180<br />

=.06 km<br />

The length of the ship is about.06 km (or 60 m),<br />

33. Let d = the diameter of the moon. d is<br />

approximately the arc length subtended by .!.0.<br />

2<br />

tt<br />

=­<br />

360<br />

d=r9<br />

= 240,000(3:0)<br />

= 2100 mi<br />

The diameter of the moon is about 2100 mi.<br />

34. Let r = the distance of the boat. The height of the<br />

mast, 32 ft, is approximately the arc length<br />

subtended by 2° 10'.<br />

9 = 2° 10' = 2.!. (...!!-)<br />

6 180<br />

s<br />

r=­ 9<br />

32<br />

r=-­<br />

13"<br />

=-­<br />

1080<br />

1311"<br />

1080<br />

= 850 ft<br />

The boat is about 850 ft away.<br />

91


<strong>Trigonometry</strong> <strong>Solutions</strong> <strong>Manual</strong><br />

92 Chapter 3 Radian Measure and the Circular Functions<br />

x<br />

3rr<br />

35. r=6<br />

,<br />

()=-<br />

3<br />

41. r=52 em, ()=­<br />

10<br />

1 2<br />

A=-r()<br />

rr)<br />

A = .!..(52)2e<br />

2 2 10<br />

= 405.6rr<br />

=~(6)2(~) 2<br />

'" 1300 em<br />

= 6rr<br />

x<br />

42. r = 25 mm, () =!!.... radian<br />

36. r=S, ()=- 15<br />

2<br />

A = '!"r 2 A = '!'(25)2(!!"")<br />

() 2 15<br />

2<br />

125rr<br />

=-­<br />

= ~(S)2(~) 6<br />

=16rr<br />

37. A = 3 units 2, r= 2<br />

2<br />

"'65 mm<br />

43. r =:: 12.7 em, () =:: SI°<br />

2<br />

A =.!. r 2 ()<br />

The formula A = .!. r () requires that () be<br />

2 2<br />

measured in radians. Convert SI ° to radians by<br />

3 = '!"(2)2()<br />

2 multiplying by ...!E...-.<br />

3=2() ISO<br />

() = ~ or 1.5 radians () = S{ ...!E...-) =:: 9rr<br />

2 ISO 20<br />

38. A = S units-, r = 4 A = .!.(l2.7)2(9rr)<br />

2 20<br />

1 2<br />

A=-r()<br />

= 36.29rr<br />

2<br />

'" 114 em 2<br />

S=.!..(4)2()<br />

2 44. r = IS.3 m, () =125°·<br />

S = S()<br />

() = 1 radian The formula A = .!.r 2 () requires that () be<br />

5rr<br />

39. r=29.2 m,()=­<br />

2<br />

measured in radians. Convert 125° to radians by<br />

. 6 multiplying by ..!!-.<br />

ISO<br />

A =.!. r 2 ()<br />

2 A = .!..(lS.3)2(l25{...!E...-)<br />

2 180<br />

= ~(29.2)2( 5:) = 116.2Srr<br />

= 355.271t = 365 m 2<br />

'" 1120 m 2<br />

45. A = 16 in. 2, r;;;:: 3.0 in.<br />

40. r;;;:: 59.8 km, () = 2rr radians A =.!..r 2 ()<br />

3 2<br />

A =.!. r 2 ()<br />

16 = '!'(3)2()<br />

2 2<br />

9<br />

16 =-()<br />

;;;::~(59 .8)2(2:)<br />

2<br />

() '" 3.6 radians<br />

;;;:: 1192.01rr<br />

2<br />

'" 3740 lan<br />

92


<strong>Trigonometry</strong> <strong>Solutions</strong> <strong>Manual</strong><br />

3.2 Applications of Radian Measure 93<br />

46. A =36 ft2, () = lC radian<br />

4<br />

A =.!. r 2 ()<br />

2<br />

36= ~r2(:)<br />

36 = lC r2<br />

8<br />

288 2<br />

-=r<br />

lC<br />

r == 9.6 ft<br />

47, A =64 m 2 , () = lC radian<br />

6<br />

A =.!. r 2 ()<br />

2<br />

64=~r2(:)<br />

64 =.!!-r 2<br />

12<br />

768 2<br />

-=r<br />

lC<br />

r==16 m<br />

48. A =25 in. 2 , r= 10 in.<br />

A =.!. r 2 ()<br />

2<br />

25=.!.(l0)2()<br />

2<br />

25 =500<br />

() =.50 radian<br />

2<br />

49. A =.!. r ()<br />

2<br />

Substituting () =2lC,<br />

2(2lC)<br />

=1CT 2<br />

A =.!. r .<br />

2<br />

2<br />

The area of a circle of radius r is tcr .<br />

50. The formula for the area of a sector is<br />

A =.!. r 2(). Substitute 2r for r.<br />

2<br />

A =.!.(2r)20<br />

2<br />

=4(~r20 )<br />

The area of the sector is quadrupled.<br />

52. x 2 +i =4 is the equation of a circle of radius 2.<br />

y = ( "'7} is the equation ofa line with slope<br />

.fj<br />

- . Therefore,<br />

3<br />

lan() =.fj<br />

3<br />

lC<br />

()=-.<br />

6<br />

Substitute r =2 and () = lC in the formula for the<br />

6<br />

area of a sector.<br />

1 2<br />

A=-r()<br />

2<br />

=~(2)2(:)<br />

x<br />

= 3<br />

53. (a) Central angle in degrees:<br />

360° =13.!.°<br />

27 3<br />

Central angle in radians:<br />

2lC<br />

27<br />

(b) Circumference =2tcr;<br />

radius =76 ft, so .<br />

circumference = 2(76)lC<br />

"" 480 ft<br />

2lC<br />

(c) r =76 ft ()=­<br />

, 27<br />

s =r()= 7{~~)<br />

"= 160 "" 17.8 ft<br />

9<br />

(d) Area of sector with r = 76 ft and 0 = 2lC is<br />

27<br />

A =.!. r 2 0<br />

2<br />

='!'(76 2 ) 2lC<br />

2 27<br />

""672 ft2.<br />

51. Recall that zrradians = 180°. If 0 is measured in<br />

degrees, then the formula becomes<br />

1 2 81C tcr 2 ()<br />

A=-r -=-­<br />

2 180 360<br />

93


<strong>Trigonometry</strong> <strong>Solutions</strong> <strong>Manual</strong><br />

94 Chapter 3 Radian Measure and the Circular Functions<br />

54. (a)<br />

55. The area cleaned is the area of the sector "wiped"<br />

by the total area and blade minus the area<br />

"wiped" by the arm only .<br />

10 - 7 =3, so the arm was 3 in. long.<br />

1 2(951r) . 2<br />

Aarm only =2" (3) 180 == 7.5 m,<br />

The triangle formed by the central angle and<br />

the chord is isoceles . Therefore, the bisector<br />

of the central angle is also the perpendicular<br />

bisector of the chord.<br />

sin Zl = 50<br />

r<br />

50<br />

r=-­<br />

sin 21°<br />

= 140 ft<br />

(b) r=~' 0=42°<br />

sin21° '<br />

Convert 0 to radians:<br />

(c)<br />

42(-!!..... radian) = 71r radians<br />

180 30<br />

s =rO<br />

50 71r<br />

=-_.­<br />

sin 21° 30<br />

351r<br />

= 3sin 21°<br />

= 102 ft<br />

1 ( 50 )2(71r)<br />

Asector = 2" sin 21° 30<br />

== 7135 ft 2<br />

Atriangle =2" 1 bh<br />

= .!.(IOO)(~)<br />

2 tan 21°<br />

= 6513 ft 2<br />

The area bounded by the arc and the chord is<br />

7135 - 6513 = 622 ft2.<br />

1 2(951r) . 2<br />

A arm and blade =-(10) -- =S2.9tn.<br />

2 180<br />

The area of the region cleaned was about<br />

82.9 -7.5 = 75.4 in. 2 .<br />

56. Use the Pythagorean Theorem to find the<br />

hypotenuse of the triangle, which is also the<br />

radius of the sector of the circle.<br />

r 2 = 30 2 +40 2<br />

r = ../900+ 1600<br />

= ../2500<br />

=50<br />

The total area of the lot is the sum of the areas of<br />

the triangle and the sector.<br />

Atriangle = 2" 1 bh =2" 1 (30)(40)<br />

=600 yd 2<br />

.tl = .!.r 20 = .!..(50)2(60 .-!!.....) = 12501r yd 2<br />

'"Sector 2 2 ISO 3<br />

12501r 2<br />

Total area = --+600 = 1900 yd<br />

3<br />

57. The angle of inclination of the sun's rays at<br />

Alexandria is equal measure to the central angle<br />

that subtends the arc from Alexandria to Syene<br />

(5000 stades) because these angles are<br />

corresponding angles formed by a transversal that<br />

intercepts the parallel rays of the sun. To find the<br />

radius r of the earth, use<br />

s = rO, with s = 5000, and 0 = l...(21r) =.!!.... .<br />

50 25<br />

5ooo=r(~)<br />

125,000<br />

r =<br />

== 39,78S.736 stades<br />

1r<br />

C = 2Trr = 21r(125~000) = 250,000 stades<br />

d=2r<br />

= 2( 125~000) stades<br />

= 2e25~000}516.7) ft<br />

__ 2e25~OOO)(516.7)<br />

......:..__.!...-_- mi<br />

5280<br />

== 7800 mi<br />

94


<strong>Trigonometry</strong> <strong>Solutions</strong> <strong>Manual</strong><br />

3.3 Circular Functions of Real Numbers 95<br />

58. Use Volume V =Area of base X height. e h e<br />

62. cos- = -, so h = rcos-.<br />

Area of base = .!.r 2 e 2 r 2<br />

2<br />

height =h<br />

63. d=r-h<br />

V=(~r2e )h)<br />

e<br />

=r- rcos- 2<br />

r 2 eh<br />

=--<br />

2<br />

=r(I-COS~)<br />

(e is in radians.)<br />

. L<br />

64. Smcer=-,<br />

59. The area of the sector using the outside radius is (}<br />

1 2<br />

A, =-rl e. d =r(1-COS~)<br />

2<br />

The area of the sector using the inside radius is<br />

1 2<br />

d = ~ (1- cos ~)<br />

A2 =-r2 e.<br />

2<br />

To find the area of the base, take the larger area 65. (a) MQP is similar to 6.RMO, because<br />

minus the smaller area. angle R = angle R and angle Q = angle M.<br />

A = A,-A 2<br />

1 2 1 2 (b) OR RM<br />

=-r, e--r2 e -=-­<br />

2 2 RP RQ<br />

1 (2 2) r £<br />

="2e r, -r2<br />

_=.1..<br />

To find the volume, multiply the area of the base<br />

c b<br />

2<br />

by the height.<br />

c<br />

rb=­<br />

V=A ·h<br />

2<br />

= ~eh2 - r2 2)h.<br />

r=-<br />

c 2<br />

2b<br />

Note that these calculations assume that (} is<br />

measured in radians. 2<br />

(c) By the Pythagorean theorem, + b 2 2<br />

a = c .<br />

60. (a) Substitute 950,000 for A in the formula for<br />

c 2 a 2 + b 2<br />

r=-=<br />

area of a circle. 2b 2b<br />

2<br />

A =rrr<br />

2 (d) Since a = 1.4 and b =.2,<br />

950,000 = rrr<br />

1.4 2 + .2 2 1.96 +.4 2<br />

r =~950;rOOO r= =­<br />

2(.2) .4 .4<br />

",,550m<br />

= 5 in.<br />

(b) 35° = 35(~) =71C radian<br />

180 36<br />

Section 3.3<br />

Exercises<br />

Substitute 950,000 for A and 71C for (} in<br />

36<br />

the formula for area of a sector. 1. Since (} = 1C ,<br />

.... 2<br />

A =.!.r 2 (}<br />

2 sin 1C is a circular function.<br />

950 000 =.!. r 2 2<br />

( 71C )<br />

, 2 36<br />

2. Since e= -150°,<br />

_ ~ 68.400.000<br />

r -<br />

71C<br />

cos(-150)0 is a trigonometric function.<br />

""1800 m<br />

3. Since e= 45°,<br />

tan 45° is a trigonometric function.<br />

61. Since L is the arc length,<br />

L<br />

L=r9 andr=e<br />

95


<strong>Trigonometry</strong> <strong>Solutions</strong> <strong>Manual</strong><br />

96 Chapter 3 Radian Measure and the Circular Functions<br />

3Jr<br />

4<br />

II<br />

4• S·mce U;;- ,<br />

3Jr) . . If '<br />

cot 4 IS a circu ar unction.<br />

(<br />

II<br />

5 S· 5Jr<br />

• mce u;;-3'<br />

sec - 3 5Jr) IS . a circu . I ar fu nction. .<br />

(<br />

6. Since ();; 240°,<br />

esc 240° is a trigonometric function .<br />

7. Since ();; 3,<br />

tan 3 is a circular function.<br />

8. Since ();; -5,<br />

cos (-5) is a circular function.<br />

9. cos.8 "".7<br />

10. cos 4 "" - .65<br />

11. sin.2 "" .2<br />

12. sin 4 "" - .75<br />

13. x;; -.65<br />

when ();; 4 radians<br />

14. y=-.95<br />

when () "" 4.4 radians<br />

15. (a) Since 1.57 < 2 < 3.14, Jr < 2 < Jr,<br />

2<br />

2 radians is in quadrant II. Therefore, cos 2<br />

is negative.<br />

(b) Since -1.57 < -1 < 0, - Jr < -1 < 0,<br />

2<br />

-1 radian is in quadrant IV. Therefore,<br />

tan -1 is negative.<br />

(c) Since 4.71 < 5 < 6.28, 3Jr < 5 < 2Jr,<br />

2<br />

5 is in quadrant IV. Therefore, sin 5 is<br />

negative.<br />

(d) Since 4.71 < 6 < 6.28, 3Jr < 5 < 2Jr,<br />

2<br />

6 is in quadrant IV. Therefore, cos 6 is<br />

positive.<br />

· d . 7Jr<br />

16 • F 10 SIO-.<br />

6<br />

In radians, the reference angle for 7Jr is<br />

6<br />

7Jr Jr<br />

--Jr;;-.<br />

6 6<br />

7Jr .. dIll h he si .<br />

- IS 10 qua rant ,were t e sme IS negative.<br />

6<br />

. 7Jr . Jr 1<br />

SIO-=-Sln-=--.<br />

6 6 2<br />

Or, convert to degrees.<br />

7Jr = 2(1800) = 2100<br />

6 6<br />

SIO-=SIO . 7Jr . 2100 ;;-­1<br />

6 2<br />

F 'nd<br />

5Jr<br />

3<br />

17• I COS- .<br />

In radians, the reference angle for 5Jr is<br />

3<br />

5Jr n<br />

2Jr--=-.<br />

3 3<br />

- 5Jr .. IS 10 qua d rant IV , were h t h e cosine .. IS<br />

3<br />

positive.<br />

5Jr Jr 1<br />

cos- = cos-=­<br />

332<br />

Or, convert to degrees.<br />

.5Jr = ~(1800) = 3000<br />

3 3<br />

5Jr<br />

1<br />

cos - ;; cos 300° ;; ­<br />

3 2<br />

· d 3Jr<br />

18• F10 tan-.<br />

4<br />

In radians, the reference angle for 3Jr is<br />

4<br />

3Jr Jr<br />

Jr--=-.<br />

4 4<br />

3Jr is in quadrant II, where the tangent is<br />

4<br />

negative.<br />

3Jr Jr<br />

tan - ;; - tan - ;; -1<br />

4 4<br />

Or, convert to degrees .<br />

3Jr ;; ~(1800);; 1350<br />

4 4<br />

tan 3Jr ;; tan 135 0 =-1<br />

4<br />

96


<strong>Trigonometry</strong> <strong>Solutions</strong> <strong>Manual</strong><br />

3.3 Circular Functions of Real Numbers 97<br />

· d . 5Jr<br />

19• F10 SIO-,<br />

3<br />

20• F10<br />

In radians, the reference angle for 5Jr is<br />

3<br />

5Jr Jr<br />

2Jr--=-.<br />

3 3<br />

- 5Jr IS . .<br />

10 qua d rant IV , were h t h e sine . IS , negative . .<br />

3<br />

. 5Jr ,Jr .J3<br />

s1o- = -s1o-=-­<br />

3 3 2<br />

Or, convert to degrees.<br />

5Jr = ~(1800) = 3000<br />

3 3<br />

. 5Jr . 3000 .J3<br />

s1O-=s1o =-­<br />

3 2<br />

" d 4Jr<br />

tan-.<br />

3<br />

In radians, the reference angle for 4Jr is<br />

3<br />

4Jr tt<br />

--Jr=-.<br />

3 3<br />

-4Jr IS ..<br />

10 qua d rant ill , were h th e tangent IS .<br />

3<br />

positive.<br />

4Jr Jr t:<br />

tan-=tan-=",3<br />

3 3<br />

Or, convert to degrees.<br />

4Jr = ~(1800) = 2400<br />

3 3<br />

4Jr<br />

h<br />

tan- = tan 240 0 = ",3.<br />

3<br />

21 F· d 2Jr<br />

• 10 sec-.<br />

3<br />

2Jr , 2Jr Jr<br />

The reference angle for - IS Jr - - = -,<br />

. 3 3 3<br />

2Jr is in quadrant II, where the secant is<br />

3<br />

negative.<br />

2Jr x<br />

sec-=-sec-=-2<br />

3 3<br />

Or, convert to degrees.<br />

2Jr = ~(1800) = 120 0<br />

3 3<br />

sec 2Jr = sec 120 0 =-2<br />

3<br />

' d IlJr<br />

22• F10 CSC-.<br />

6<br />

llJr , 11Jr Jr<br />

The reference angle for - IS 2Jr - - = - .<br />

6 6 6<br />

-<br />

11Jr<br />

IS<br />

. . d IV<br />

10 qua rant , where the cosecant is<br />

6<br />

negative.<br />

IlJr Jr<br />

csc- = -csc- =-2<br />

6 6<br />

Or, convert to degrees.<br />

IlJr =!.!.(1800) = 3300<br />

6 6<br />

IlJr<br />

csc- = csc330° =-2<br />

6<br />

23• Fm<br />

' d<br />

cot-.<br />

5Jr<br />

6<br />

5Jr . 5Jr Jr<br />

The reference angle for - IS Jr - - = -.<br />

6 6 6<br />

5Jr is in quadrant II, where the cotangent is<br />

6<br />

negative.<br />

5Jr Jr h<br />

cot - =- cot - =-V3<br />

6 6<br />

Or, convert to degrees.<br />

51t = ~(1800) = 150 0<br />

6 6<br />

cot 51t =cot 150 0 = -J3<br />

6<br />

24. Find cos(- 4;J<br />

4Jr " . I . h 2Jr .<br />

- - IS cotermma WIt -.<br />

3 3<br />

2Jr . 2Jr Jr<br />

The reference angle for - IS Jr - -. - =-.<br />

333<br />

2Jr " d II h h ."<br />

- IS m qua rant , were t e cosine IS<br />

3<br />

negative.<br />

cos(- 4;) = cos 2: =-cos~ =-~<br />

Or, convert to degrees .<br />

2Jr = ~(1800 ) =1200<br />

3 3<br />

cos(- 4Jr) =cos 2Jr =cos 120 0 =_!<br />

332<br />

97


<strong>Trigonometry</strong> <strong>Solutions</strong> <strong>Manual</strong><br />

98 Chapter 3 Radian Measure and the Circular Functions<br />

d 231C<br />

25. FindSin(_ 5:)- 27• Fm ' sec--.<br />

6<br />

51C . . I . h 71C 231C . bOd<br />

- - IS cotermma Wit -. 6 IS not etween an 21C. Subtract 21C.<br />

6 6<br />

71C. 71C 1C 231C _ 21C = 231C _ 121C = ll1C<br />

The reference angle for - IS - - 1t =-.<br />

6 6 6 6 6 6 6<br />

51C . . dr III h th ' .<br />

231C . . I . ll1C<br />

-- IS m qua ant • W ere e sine IS -- IS cotermma With--.<br />

6<br />

6 6<br />

negative. ll1C . 1m 1C<br />

The reference angle for -- IS 21C - - = - .<br />

. (51C) . 71C . 1C 1<br />

6 6 6<br />

sm -6 =sm 6=-sm'6=-2' 2~1C is in quadrant IV. where the secant is<br />

Or. convert to degrees.<br />

positive.<br />

_ 51C = -~(1800) = -1500<br />

6 6 231C ll1C 1C 2-./3<br />

sec--=sec-=sec- =-­<br />

6 6 6 3<br />

sin(_ 5:) = sin(-150 0) =_~<br />

Or. converting to degrees.<br />

1m =.!..!(1800) = 3300<br />

F· d 171C 6 6<br />

26• m tan--.<br />

3 231C ll1C ° 2-./3<br />

sec--=sec-=sec330 =--.<br />

171C is not between 0 and 21C. Subtract 21t twice. 663<br />

3<br />

that is. 41C.<br />

28• Fm · d csc-. 131C<br />

171C -41C= 171C _121C = 51C<br />

3<br />

3 3 3 3 131C _ 41C =131C _ 121C = 1C<br />

171C . . I ith 51C 3 3 3 3<br />

- IS cotermma WI -.<br />

3 3 131C . . I . h 1C<br />

-- IS cotermma Wit -.<br />

51C . 51C 1C 3 3<br />

The reference angle for - IS 21C - - = -.<br />

333 The reference angle for 1C is 1C .<br />

51C is in quadrant IV. where the tangent is<br />

· 3 3<br />

3 ~ is in quadrant I. where the cosecant is positive.<br />

negative. 3<br />

tan 171C = tan 51C = _ tan 1C = -.Ji 131C 1C 2-./3<br />

3 3 3<br />

csc-- = csc-=-­<br />

333<br />

Or. converting to degrees.<br />

Or. converting to degrees.<br />

51C = ~ (1800)=3000<br />

1C =.!. (1800) =600<br />

3 3<br />

3 3<br />

51C<br />

r;:;<br />

tan- = tan 300° = -v3. 1C 2-./3<br />

3<br />

csc- = csc6O° =--.<br />

3 3<br />

98


<strong>Trigonometry</strong> <strong>Solutions</strong> <strong>Manual</strong><br />

3.3 Circular Functions of Real Numbers 99<br />

29• P· m d cos-. 13K<br />

4<br />

13K _ 2K = 13n _ 8K = 5K<br />

4 4 4 4<br />

13K . . I . h 5K<br />

- IS cotermma wit -.<br />

4 4<br />

5K. 5K K<br />

The reference angle for - IS - - K = -.<br />

4 4 4<br />

13K .. dr ill h h . .<br />

- IS m qua ant ,were t e cosine IS<br />

4<br />

negative.<br />

13K 5n K..fi<br />

cos- = cos- = -cos-=-­<br />

4 4 4 2<br />

Converting to degrees,<br />

5K = ~(l800) = 2250<br />

4 4<br />

13K 5n o..fi<br />

cos- = cos- = cos225 =--.<br />

4 4 2<br />

For Exercises 30-53, your calculator must be set in<br />

radian mode. Keystroke sequences may vary based on<br />

the type and/or model of calculator being used.<br />

30. sin .8203<br />

Enter: .8203 ~<br />

or ~ .8203 ~<br />

sin .8203 = .73135046<br />

31. cos .6429<br />

Enter: .6429 ~<br />

or §J .6429 ~<br />

cos .6429 = .80036052<br />

32. tan .9047<br />

Enter .9047 ~<br />

or ~ .9047 ~<br />

tan .9047 =1.2723944<br />

33. sin 1.5097<br />

Enter: 1.5097 ~<br />

or ~ 1.5097 ~<br />

sin 1.5097 = .99813420<br />

34. cot .0465<br />

Enter: .0465 ~ 8<br />

or ITJ ~ .0465 CIJ [2] l§l<br />

cot .0465 = 21.489874<br />

35. esc 1.3875<br />

Enter: 1.3875 ~ . ~<br />

or ITJ E.J 1.3875 CIJ [2]<br />

§3<br />

esc 1.3875 = 1.0170372<br />

36. sec 7.4526<br />

Enter: 7.4526 §J EJ<br />

or ITJ ~ 7.4526 CIJ [2]<br />

~<br />

cot 7.4526 =2.5595706<br />

37. cos 6.6701<br />

Enter: 6.6701 ~<br />

or §] 6.6701 ~<br />

cos 6.6701 = .92607765<br />

38. tan 4.0230<br />

Enter: 4.0230 §)<br />

or §) 4.0230 ~<br />

tan 4.0230 = 1.2131367<br />

39. cos 4.2528<br />

Enter: 4.2528 §J<br />

or ~ 4.2528 ~<br />

cos 4.2528 = -.44357977<br />

40. sin 3.4645<br />

Enter: 3.4645 E.J<br />

or ~ 3.4645 ~<br />

sin 3.4645 = -.31732498<br />

41. sin (-2.2864)<br />

Enter: 2.2864 EJ EJ<br />

or ~ CEJ 2.2864 ~<br />

sin(-2.2864) = -.75469733<br />

42. cot(-2.4871)<br />

Enter: 2.4871 EJ ~ ~<br />

or CD ~ (BJ 2.4871 CD [2]<br />

~<br />

cot(-2.4871) = 1.3032411<br />

43. cos(-3.0602)<br />

Enter: 3.0602 EJ §J<br />

or §J CEJ 3.0602 ~<br />

cos(-3.0602) = -.99668945<br />

44. tan s = .21264138<br />

Enter: .21264138 EJ ~<br />

or 8 ~ .21264138 l§l<br />

Display: .20952066<br />

s = .20952066<br />

45. cos s =.78269876<br />

Enter: .78269876 ~ §J<br />

or El §J .78269876 ~<br />

Display: .67180620<br />

s = .67180620<br />

46. sin s =.99184065<br />

Enter: .99184065 EJ EJ<br />

or El ~ .99184065 ~<br />

Display: 1.4429646<br />

s = 1.4429646<br />

99


<strong>Trigonometry</strong> <strong>Solutions</strong> <strong>Manual</strong><br />

100 Chapter 3 Radian Measure and the Circular Functions<br />

47. cot S =29949853<br />

Enter: .29949853<br />

or 8 EJ .29<br />

~<br />

Display: 1.2797997<br />

S = 1.2797997<br />

~<br />

949853<br />

8 EJ<br />

~<br />

48. cot S = .62084613<br />

Enter: .62084613 ~ 8 EJ<br />

or 8 EJ .62084613 ~<br />

~<br />

Display: 1.0151896<br />

S = 1.0151896<br />

49. tan S = 2.6058440<br />

Enter: 2.6058440 8<br />

EJ<br />

or 8 EJ 2.6058440 ~<br />

Display: 1.2043741<br />

S = 1.2043741<br />

50. cos S = .57834328<br />

Enter: .57834328 8 EJ<br />

or 8 EJ .57834328 ~<br />

Display: .95409991<br />

S =.95409991<br />

51. sin S = .98771924<br />

Enter: .98771924 8 EJ<br />

or 8 ~ .98771924 ~<br />

Display: 1.4139143<br />

S = 1.4139143<br />

52. cot s = .0963704 1<br />

Enter: .09637041 ~ 8 EJ<br />

or 8 EJ .09637041 ~<br />

~<br />

Display: 1.4747226<br />

s = 1.4747226<br />

53. esc s =1.0219553<br />

Enter: 1.0219553 ~ B ~<br />

or EJ ~ 1.0219553 IE]<br />

~<br />

Display: 1.3631380<br />

s =1.3631380<br />

54.<br />

1C ]. 1<br />

[2'1C ; Slns='2<br />

Because sin s = .!... the reference angle for s must<br />

2<br />

be -1C. since Sill- . 1C = - 1 . F or s to be<br />

In<br />

' t he e i mterva I<br />

6 6 2<br />

[~ , 1CJ. we must subtract the reference angle<br />

from 1C.<br />

Therefore,<br />

1C 51C<br />

S=1C--=-.<br />

6 6<br />

55.<br />

56.<br />

57.<br />

58.<br />

[~.1Cl coss =-~<br />

1<br />

Because coss = --, the reference angle for s<br />

2<br />

1C . 1C 1 F be' th<br />

must be - since cos - = -. or s to In e<br />

3 3 2<br />

interval<br />

[~ , 1CJ. we must subtract the reference angle<br />

from 1C.<br />

Therefore,<br />

1C 21C<br />

S=1C--=-.<br />

3 3<br />

[1C' 3;l tans =~<br />

Because tan s =~. the reference angle for s<br />

must be 1C since tan 1C =~. For s to be in the<br />

3 3<br />

interval [1C' 3;J. we must add the reference<br />

angle to 1C. Therefore,<br />

1C 41C<br />

S=1C+-=--.<br />

3 3<br />

31C] . 1<br />

[1C'T ; Slns=-'2<br />

1 .<br />

Because sin s = - -, the reference angle for s<br />

2<br />

1C • .1C IF be in th<br />

must be - since Sin- = -. or s to III e<br />

662<br />

interval [1C' 3;J. we must add the reference angle<br />

to 1C. Therefore.<br />

1C 71C<br />

s=1C+-=-.<br />

6 6<br />

[3;, 21Cl tans =-1<br />

Because tan s = -1, the reference angle for s must<br />

be 1C since tan 1C = 1. For s to be in the interval<br />

4 4<br />

[3; , 21CJ. we must subtract the reference angle<br />

from 21C. Therefore,<br />

1C 71C<br />

S=21C--=-.<br />

4 4<br />

100


<strong>Trigonometry</strong> <strong>Solutions</strong> <strong>Manual</strong><br />

3.3 Circular Functions of Real Numbers 101<br />

59. [3;, 21rl coss = ~<br />

Jj<br />

Because cos s =-, the reference angle for s<br />

2<br />

1r . 1t Jj .<br />

must be - since cos- =-. For s to be In the<br />

662<br />

interval [3; , 27rJ. we must subtract the<br />

reference angle from 21r. Therefore,<br />

. 1r 111r<br />

s=21r--=-.<br />

6 6<br />

sinx . sinx<br />

60. x -- (radians) -- (degrees)<br />

x<br />

x<br />

.1 .998334 17 .0174532837<br />

.01 .99998333 .0174532924<br />

.001 .99999983 .0174532925<br />

61. Since X =cos s,<br />

coss = .55319149<br />

Use radian mode.<br />

Enter.55319149 ~ ~<br />

or ~ ~ .55319149 ~<br />

Display : .984605869<br />

s


<strong>Trigonometry</strong> <strong>Solutions</strong> <strong>Manual</strong><br />

102 Chapter 3 Radian Measure and the Circular Functions<br />

(c) May: x=4 73.<br />

4n<br />

t =60-30cos­<br />

6<br />

2n<br />

t =60 - 30cos­<br />

3<br />

t =60-3~-~) = 60+15<br />

t = 75°<br />

(d) June: x=5<br />

5n<br />

t = 60-30cos­<br />

6<br />

1= 60-3{­ "7) = 60+ 15.J3<br />

1= 86°<br />

(e) August: x = 7<br />

7n<br />

1= 6O-30cos­ 74.<br />

6<br />

1=60-3{- "7)=6O+15.J3<br />

t "" 86°<br />

(f) October: x = 9<br />

9n<br />

1= 60-30cos­<br />

6<br />

3n<br />

1= 6O-30cos­<br />

2<br />

1= 60-30(0) = 60-0<br />

1=60°<br />

72. T(x) = 37sin[ 2n (x -101)]+ 25<br />

365<br />

(a) March 1 (day 60)<br />

T(6O) = 37sin[ 2n (60 -101)]+ 25<br />

365<br />

""1°F<br />

(b) April 1 (day 91)<br />

T(91) = 37sin[2n (91-101)]+25<br />

365<br />

...19°F<br />

(c) Day 150<br />

T(150) se 53°F<br />

(d) June 15 is day 166.<br />

T(166) '" 58°F<br />

(e) September 1 is day 244.<br />

T(244) '" 48°F<br />

(f) October 31 is day 304.<br />

T(304) = 12°F.<br />

75.<br />

76.<br />

sin x =sin (x + 2)<br />

Ifx is in quadrant I, then x + 2 is in quadrant II.<br />

Therefore,<br />

x+2=n-x<br />

2x =n-2<br />

n<br />

x = --1.<br />

2<br />

On the interval 0 :s; x :s; 2n, this would also be true<br />

if we use an angle of 3n- x which is coterminal<br />

to n - x. Therefore,<br />

2+x=3n-x<br />

2x = 3n-2<br />

3n<br />

x=--1.<br />

2<br />

For the statement sin x =sin (x + 2) to be true<br />

n 3n<br />

x = - - l or x = - - 1.<br />

2 2<br />

(a) New Orleans has a latitude of L =30° or<br />

.5236 radian. Since the day and time have<br />

not changed,<br />

D = -.1425 and CO = .7854.<br />

sin (J = cos Dcos Lcosco + sin Dsin L<br />

= cos(-.1425)cos(.5236) .cos(.7854)<br />

+ sin(-.1425) ·sin(.5236)<br />

= .5352<br />

Thus,<br />

(J= .5647 radian or 32.4°.<br />

To solve this problem make sure your calculator<br />

is in radian mode. 23.44° "" .4091 radian and<br />

44.88°= .7833 radian.<br />

Shortest Day:<br />

cos (.1309H) = -tanDtan L<br />

= -tim (-.4091) tan (.7833)<br />

"" .4317<br />

.1309H "" 1.124<br />

H ""8.6 hi'<br />

Longest Day:<br />

cos (.1309H)=-tanDtanL<br />

cos (.1309H)=-tan (.4091) tan (.7833)<br />

"" -.4317<br />

.1309H "" 2.017<br />

H ...15.4 hr<br />

Hartford's latitude Lis 41.93° ....7318 radian and<br />

D= .26724.<br />

cos (.1309H) = -tan Dtan L<br />

= - tan (.26724) tan (.7318)<br />

"" -.2459<br />

.1309H "" 1.819<br />

H "" 13.9 hr<br />

102


<strong>Trigonometry</strong> <strong>Solutions</strong> <strong>Manual</strong><br />

3.4 Linear and Angular Velocity 103<br />

Section 3.4<br />

6• 6 2n di 5n d" .<br />

=""9 ra ran, co = 27 ra Ian per mm<br />

Exercises<br />

6<br />

co=­<br />

1. The circumference of the unit circle is 21t<br />

t<br />

co= 1 radian per sec, 6= 2nradians<br />

5n =If<br />

6<br />

27 t<br />

co=t<br />

2n 27 6 °<br />

t=_·_=- mm<br />

6<br />

t=co<br />

9 5n 5<br />

2n 3n di n dO °<br />

t = - =2nsec 7• 6 = - ra ians.m = - ra Ian per mm<br />

1 8 24<br />

6<br />

co=­<br />

2• eo = -2n ra d" lans per sec, t = 3 sec<br />

t<br />

3 6<br />

t=­<br />

6<br />

eo=­<br />

eo<br />

t<br />

2n 6<br />

t= ~<br />

.1L<br />

-=­ 24<br />

3 3<br />

=_0­<br />

3n 24<br />

6 = 2n radians<br />

8 n<br />

=9min<br />

3· co = 4" n ra d'<br />

°<br />

Ian per mm, t = 5' mm<br />

8. 6=3.871142 radians, t =21.4693 sec<br />

6<br />

eo=­<br />

6<br />

t<br />

eo=­<br />

t<br />

n 6<br />

-=­ 3.871142<br />

4 5 eo=--­<br />

21.4693<br />

= .180311 radian per sec<br />

6=\:)<br />

9. eo = .90674 radian per min, t = 11.876 min<br />

=- 5n ra dilans<br />

4<br />

6<br />

eo=t<br />

4•<br />

.6<br />

= -<br />

3n<br />

ra<br />

d'<br />

lans,<br />

•<br />

t =<br />

8<br />

sec<br />

.90674 =_6_<br />

4. 11.876<br />

6 3zr 6 = 10.768 radians<br />

eo=-=-.i..<br />

t 8 10. The circumference of the unit circle is 21t.<br />

3n di eo = 1 unit per sec, 6 = 2n radians<br />

= 32 ra I3n per sec<br />

6<br />

eo=t<br />

5. 6 = 2n radians, t = 10 sec 6<br />

t=­<br />

5 eo<br />

6 2n<br />

eo=-<br />

=-=2nsec<br />

t<br />

1 -------- -­<br />

llL<br />

eo =..2­<br />

10 13. r = 12 m, co = 2n radians per sec<br />

2n 3<br />

=- v = reo<br />

50<br />

= -n ra di Ian per sec<br />

v = 12(2;) = 8n m per sec<br />

25<br />

103


<strong>Trigonometry</strong> <strong>Solutions</strong> <strong>Manual</strong><br />

104 Chapter 3 Radian Measure and the Circular Functions<br />

8 91r di<br />

121r 3 21r .<br />

5 5 ' 2' 5<br />

v = rOJ<br />

s = rOJt<br />

14. r = em, OJ =- ra tans per see 22. s =-- m r =- m OJ =- radians per sec<br />

v = 8(9;)<br />

721r<br />

=-- em per see<br />

121r = ~(21r}<br />

5 2 5<br />

5 t= 1~1r(~)(;1t)<br />

t=4see<br />

15. v = 9 m per sec, r = 5 m<br />

v = rOJ<br />

31r<br />

9=5OJ 23. s = - km, r =2 km, t = 4 sec<br />

4<br />

OJ = -9 raditans per see<br />

s = rost<br />

5 31r<br />

-=2(OJ)4<br />

16. v = 18 ft per sec, r =3 ft<br />

4<br />

v= rOJ .k<br />

18 = 30J<br />

OJ =....!...<br />

8<br />

18<br />

OJ=- =-<br />

31r<br />

ra<br />

d'<br />

Ian per sec<br />

3 32<br />

= 6 radians per see<br />

81r 4<br />

17. v = 107.692 m per sec , r= 58.7413 m 24. s=- m r=- m t= 12 sec<br />

v=rOJ<br />

9 ' 3 '<br />

107.692 =58.7413OJ<br />

s =riot<br />

OJ == 1.83333 radians per sec<br />

81r=i OJ(12)<br />

9 3<br />

18. r = 24.93215 em, (0= .372914 radian per sec<br />

81r ( 3 )<br />

v=rOJ OJ =9 4(12)<br />

v = (24.93215)(.372914)<br />

== 9.29755 em per see OJ =-1r radiJan per sec<br />

18 .<br />

19. r = 6 em, OJ =1r radians per sec, t = 9 sec 27. The hour hand of a clock moves through an angle<br />

3 of 21rradians in 12 hours, so<br />

s = r(Ot<br />

(J 21r<br />

(0=-=t<br />

12<br />

S=6(~)9<br />

= 181r em =!!.- radian per hr<br />

6<br />

20. r = 9 yd, (0 = 21r radians per sec, t = 12 sec<br />

28. The minute hand makes one revolution per hour.<br />

5 Eaeh revolution is 21rradians, so<br />

s =rtat<br />

(0= 21rradians per hr<br />

s=,2;}12)<br />

_1_ (21r) =.!!.... radian per min,<br />

60 30<br />

= 2161r yd<br />

29. The second hand of a clock moves through an<br />

5<br />

angle of 21rradians in 60 sec, so<br />

(J 21r<br />

(0=-=­<br />

21. s = 61tem, r = 2 em, OJ = 1r radian per sec t 60<br />

4<br />

s =rOJt =-1r<br />

ra d' Jan per see.<br />

30<br />

61r= 2(:}<br />

30. The line makes 300 revolutions per minute. Eaeh<br />

61r<br />

revolution is 21rradians, so<br />

t=-=12 sec<br />

1­<br />

OJ =21r(300)<br />

=6oo1r radians per min.<br />

104


<strong>Trigonometry</strong> <strong>Solutions</strong> <strong>Manual</strong><br />

3.4 Linear and Angular Velocity 105<br />

31. The minute hand of a clock moves through an<br />

angle of 2rr radians in 60 min, and at the tip of the<br />

minute hand, r = 7 em, so<br />

r(} 7(2rr)<br />

bicycle is<br />

v=-=-­<br />

t 60<br />

7rr .<br />

=- cm per 10m.<br />

30<br />

32. The second hand makes one revolution per<br />

minute. Each revolution is 2rr radians, so<br />

ro =2rr radians per min.<br />

v =rro, r =28 mm<br />

v=28(2rr) =56rr mm per min<br />

56rr<br />

= 60 mm per sec<br />

14rr<br />

=15 10m per sec<br />

37. At 200 revolutions per minute, the bicycle tire is<br />

moving 200 (2rr) = 400rr radians. This is the<br />

angular velocity roo The linear velocity of the<br />

v=rro<br />

= 13(400rr)<br />

= 5200rr in. per min<br />

To convert this to miles per hour:<br />

v =(52007t in'X 60 minutesXJ..!!....)( I mi )<br />

minute hour 12 in. 5280 ft<br />

"" 15.5 mph<br />

38. Mars will make one full rotation (of 2rr radians)<br />

during the course of one day.<br />

2rr radians ( I hr ) "" 24.62 hr<br />

0.2552 radians<br />

33. The flywheel making 42 rotations per min turns<br />

through an angle 42(2rr) or 84rrradians in I min<br />

with r=2m.<br />

r(} 2(84rr) .<br />

v=-=---= 168rrm permm<br />

t I<br />

34. The point on the tread of the tire is rotating<br />

35 times per min . Each rotation is 2rr radians.<br />

ro =35(2rr) =70rr radians per min<br />

v =rm, r =18 cm<br />

v = 18(70rr) = I 260rr ern per min<br />

35. At 500 rotations per min, the propeller turns<br />

through an angle of<br />

()= 500(2rr) = lOOOrrradians in I min with<br />

3<br />

r=-= 1.5 m<br />

2<br />

r(} 1.5(1000rr)<br />

v=- =-...:...----'­<br />

t . I<br />

=1500rr m per min<br />

36. The point on the edge of the gyroscope is rotating<br />

680 times per min. Each rotation is 2rrradians.<br />

ro =680(21t) =1360rr radians per min<br />

v = rca. r = 83 cm<br />

v =83(1360rr)<br />

=I 12,880rrcm per min<br />

39. (a) () =_1_ (2rr) = 2rr radian<br />

365 365<br />

(b) ro = -2rr ra di Ian per day<br />

365<br />

=_.- 2rr I ra d' Ian per hr<br />

365 24<br />

=-..!!..- radian per hr<br />

4380<br />

(c)<br />

v =rro<br />

=(93,000,()()()(-..!!..-)<br />

4380<br />

=66,700 mph<br />

40. Since s =56 cm of belt go around in t = 18 sec,<br />

the linear velocity is<br />

s<br />

v=t<br />

56<br />

=­<br />

18<br />

28<br />

="9 em per sec.<br />

v = rro, r = 12.96 cm<br />

28 =(l2.96)ro<br />

9<br />

18..<br />

ro=_9_<br />

12.96<br />

"".24 radian per sec.<br />

105


<strong>Trigonometry</strong> <strong>Solutions</strong> <strong>Manual</strong><br />

106 Chapter 3 Radian Measure and the Circular Functions<br />

41. The larger pulley rotates 25 times in 36 sec or (c) At the equator, r= 6400 km.<br />

v = rW<br />

- 25. times per sec. Th· us, Its angu I ar ve I ocity ·· IS<br />

= 64oo(21l")<br />

36<br />

= 12,8001l"km per day<br />

W = 25 (21l") = 251l" radians per sec. The linear<br />

36 18<br />

12,8oo1l" km r hr<br />

24 pe<br />

velocity of the belt is<br />

v =rto<br />

es 5331l" km per hr<br />

251l" ) 1251l"<br />

= 15 -- =-- cm per sec.<br />

( 18 6<br />

To find the angular velocity ofthe smaller pulley,<br />

1251l"<br />

usev =-- em per sec and r= 8 cm .<br />

6<br />

v = rW<br />

1251l" =8ro<br />

6<br />

W = 12:1l" (i)<br />

1251l" .<br />

=-- radians per sec<br />

48<br />

42. v = 1.46 m per sec and 46 revolutions per min is<br />

46(21l") 231l" .<br />

W =---=-- radians per sec.<br />

60 15<br />

So,<br />

v =rW<br />

1.46={2:;)<br />

r= ~ "".303m.<br />

15<br />

43. W = (l52)(21l") = 3041l" radians per min<br />

3041l" d.<br />

= ~ ra tans per sec<br />

v =rW<br />

59.4 ={7:;)<br />

r z3.73cm<br />

761l" d. '<br />

=15 ra ians per sec<br />

44. (a) The earth completes one revolution per day,<br />

so itturns through (} = 21l" radians in time<br />

t = 1 day =24 hr.<br />

(} 21l"<br />

w=-=t<br />

1<br />

=21l" radians per day<br />

21l"<br />

W=­<br />

24<br />

=!!- radian per hr<br />

12<br />

(b) At the poles, r = 0, so<br />

v =rto =O.<br />

"<br />

(d) Salem rotates about the axis in a circle of<br />

radius r at an angular velocity W = 21l"radians<br />

per day .<br />

South Pole<br />

sin45o=_r­<br />

6400<br />

r = 6400 sin 45°<br />

=64'~)<br />

r =3200-.12 km<br />

v=rW<br />

=3200-.12(21l")<br />

z 90501l"-km per day<br />

= 28,000 Ion per day<br />

=90501l" km per hr<br />

24<br />

= 3771l" km per hr<br />

z 1200 km per hr<br />

45. Let s = the length of the track on the arc.<br />

400 =4rf...!!.-) = 21l"<br />

s<br />

\.180 9<br />

The length of track, s, is the arc length when<br />

21l"<br />

(} = - and r= 1800 ft.<br />

9<br />

s =r(}<br />

s =1800(2;) =4001l" ft<br />

;:.'. .<br />

106


<strong>Trigonometry</strong> <strong>Solutions</strong> <strong>Manual</strong><br />

Chapter 3 Review Exercises 107<br />

Express the velocity v =30 mph in ft per sec.<br />

v=30 mph<br />

30 .<br />

=-- ml per sec<br />

3600<br />

(30)5280 ft<br />

= per sec<br />

3600<br />

=44 ft per sec<br />

s<br />

v=t<br />

s<br />

t=v<br />

400;rr 100;rr<br />

44 11<br />

== 29 sec<br />

180 9<br />

t=--=-­<br />

46. In one minute, the propeller makes 5000<br />

revolutions. Each revolution is 2;rrradians, so<br />

5000(2;rr)= lO,OOO;rr radians per min.<br />

Since there are 60 sec in a minute,<br />

IO,OOO;rr<br />

ro=---'-­<br />

60<br />

500;rr<br />

=-­<br />

. 3<br />

== 523.6 radians per sec.<br />

Chapter 3 Review Exercises<br />

2. (a) ;rr < 3 < n , so the terminal side is in<br />

2<br />

quadrant·ll.<br />

3;rr h . al 'd . .<br />

(b) ;rr < 4


<strong>Trigonometry</strong> <strong>Solutions</strong> <strong>Manual</strong><br />

108 Chapter 3 Radian Measure and the Circular Functions<br />

20. 9= 1O.5(21r) = 211r 27. 2soN, 12° S<br />

5 =r9= 2(211r) =421r in. 12° S = _12° N<br />

9 = 2So- (-12°) = 40°<br />

21. r=15.2 em, 9=4<br />

5 =r9 =15.2e:)<br />

31r =4~1:0)= 2;<br />

s= r9<br />

= 11.41r =6400(2;)<br />

"" 35.S em ""4500 km<br />

22. r = 11.4, 9 = .769 28. Let 5 = distance between cities<br />

5=r9 at 72° E and 35° W.<br />

= (11.4)(.769) em 35° W =-35° E<br />

=S.77 em 9= 72° - (-35°) = 107°<br />

23.<br />

1071r .<br />

r=S.973 em, 107° = -- radians<br />

ISO<br />

9=49.06° s=r9<br />

= 49.06(.!!.-) = 6400( 1071r)<br />

ISO<br />

ISO<br />

5 =r() = 12,OOOkm<br />

= S.973(49·06{1:0) 29. 5 = 1.5, r = 2<br />

"" 7.6S3 em 5=r9<br />

5 1.5 3<br />

71r<br />

9=-=-=­<br />

24. 9=-, r=2S.69 r 2 4<br />

4<br />

1 2<br />

A=-r9<br />

2<br />

A =.!. r 2 ()<br />

2<br />

=~(2)2(~)<br />

= ~ (2S.69)2(7;)<br />

=- 3 or 15 " umts "2"<br />

2<br />

"" 2263 in. 2<br />

25. 4 = 3S.0 m, 9 = 21°40' 30. 5=4, r= S<br />

5= r9<br />

9=21°40'=21,°= 6:C:0)<br />

541<br />

9=-=-=r<br />

S 2<br />

A =.!. r 2 ()<br />

2 A =.!.r 2 9<br />

65X.!!.-)<br />

= .!.(38.0)2(<br />

2 3 ISO =~(S)2(~)<br />

2<br />

""273 m<br />

2<br />

= 16 units 2<br />

26. Because the central angle is very small, the arc<br />

31. (a) The hour hand of a clock moves through an<br />

length is approximately equal to the length of the<br />

angle of 21rradians in 12 hr, so<br />

inscribed chord. (See directions for<br />

Exercises 31-34 in Section 3.2)<br />

co = - 21r = - 1r ra di Ian per h r.<br />

Let h = height of tree, r = 2000, 9 = 1° 10'. 12 6<br />

h"" r9<br />

In 2 hr the angle would be<br />

rvv11 =2 +.!Q.X.!!.-) 2(:) or ~ radians.<br />

vvv\ 60 ISO<br />

",,41 yd<br />

108


<strong>Trigonometry</strong> <strong>Solutions</strong> <strong>Manual</strong><br />

Chapter 3 Review Exercises 109<br />

(b) The distance s the tip of the hour hand travels<br />

during the time period from I o'clock to<br />

3 o'clock. is the arc length when ()= nand<br />

3<br />

r= 6 in.<br />

s =r(J = ~ ~ ) = 2n in.<br />

33. tan n = tan 60° = .J3<br />

3<br />

34. cos 2n = cos 120° = _.!.<br />

3 2<br />

35.' Si{- 5:) = sin(-~ ,1800)<br />

= sin(-150°)<br />

= sin(-150° + 360°)<br />

= sin 210°<br />

I<br />

=-­<br />

2<br />

36. tan(- 7;) = tan(-~ ,180°)<br />

= tan(-420°)<br />

= tan[-420° + 2(360°)]<br />

= tan 300°<br />

=-.J3<br />

37. ~j lIn) (11 0)<br />

c""~-6 =csc -(;'180<br />

= csc(-3300)<br />

= csc(-330°+ 360°)<br />

= esc 30°<br />

=2<br />

38. cot (--3- 17n) =cot (17 -"3,1800)<br />

= cot(-10200)<br />

= cot(-1020° + 3 .360°)<br />

= cot 60°<br />

.J3<br />

=­<br />

3<br />

39. tan I> tan 2. 0 < I


<strong>Trigonometry</strong> <strong>Solutions</strong> <strong>Manual</strong><br />

110 Chapter 3 Radian Measure and the Circular Functions<br />

47. tan 7.3159<br />

Enter: 7.3159 ~ 56.<br />

or ~ 7.3159 ~<br />

tan 7.3159:: 1.6755332<br />

48. sin 4.8386<br />

Enter: 4.8386 ~<br />

or ~ 4.8386 ~<br />

sin 4.8386 :: -.99204596<br />

49. cos s:: .92500448<br />

Enter: .92500448 EJ [§<br />

or ~ ~ .92500448 ~<br />

Display: .38974894<br />

s :: .38974894<br />

50. tan s:: 4.0112357<br />

Enter: 4.0112357 EJ ~<br />

or ~ ~ 4.0112357 ~<br />

Display: 1.3264768<br />

s :: 1.3264768<br />

51. sin s :: .49244294<br />

Enter: .49244294 EJ ~<br />

or ~ ~ .49244294 ~<br />

Display: .51489440<br />

s:: .51489440<br />

52. esc s:: 1.2361343 58.<br />

Enter: 1.2361343 ~ EJ ~<br />

or ~ ~ 1.2361343 ~ ~<br />

Display : .94240395<br />

s :: .94240395<br />

53. cot s:: .50221761<br />

Enter: .50221761 ~ 8 ~<br />

or ~ ~ .5022176 1 ~ §)<br />

Display: 1.1053762<br />

s:: 1.1053762<br />

54. sec s :: 4.5600039<br />

Enter: 4.5600039 ~ EJ ~ 59.<br />

or ~ §] 4.5600039 ~ §)<br />

Display: 1.3497014<br />

s:: 1.3497014<br />

55. [0,;J. cos s:: ~<br />

60.<br />

$<br />

Because cos s > -, the reference angle for s<br />

2<br />

must be -TC smcecos-::-. . TC $F or s to be emi the<br />

4 4 2<br />

interval [0, ~ Js must be the reference angle.<br />

TC<br />

Therefore, s :: -.<br />

4<br />

57.<br />

[;, TCJ. tans::-J3<br />

Because tan s :: -J3, the reference angle for s<br />

must be TC since tan TC ::.J3. For s to be in the<br />

3 3<br />

interval [~, TCJwe must subtract the reference<br />

angle from TC. Therefore,<br />

TC 2TC<br />

S::TC--=-.<br />

3 3<br />

n 3TC] sec s = _ 2.J3<br />

[ , 2 ' 3<br />

2.J3<br />

Because sec s :: - --, the reference angle for<br />

3<br />

s must be -<br />

TC. n 2..[3<br />

6 6 3<br />

smce sec- =--. For s to<br />

be in the interval [TC, 3;Jwe must add the<br />

reference angle to TC. Therefore,<br />

TC 7TC<br />

s=TC+-::-.<br />

6 6<br />

3 TC 2] ' 1<br />

- TC sms=-­<br />

[ 2" 2<br />

. 1<br />

Because sin s :: - -, the reference angle for s<br />

2<br />

TC. . TC 1 F be' th<br />

must be - smce sm - = -. or s to 10 e<br />

6 6 2<br />

interval [3;, 2nJwe must subtract the<br />

reference angle from 2TC. Therefore,<br />

TC lIn<br />

s=2TC--::-.<br />

6 6<br />

2 radians is in quadrant II so cosine must be<br />

negative. In quadrant II cosine decreases from<br />

oto -1. Therefore, cos 2 is closest to -.4.<br />

2 radians is in quadrant II so sine must be<br />

positive. In quadrant II sine decreases from 1 to O.<br />

Therefore, sin 2 is closest to .9.<br />

110


<strong>Trigonometry</strong> <strong>Solutions</strong> <strong>Manual</strong><br />

Chapter 3 Review Exercises 111<br />

12tr 3<br />

61. tan 8= Y 66. s =-- ft, r = - ft t = 15sec<br />

X 25 5'<br />

5.1961524 s 12n 12tr<br />

=<br />

3<br />

v=-=~=--<br />

t 15 375<br />

"" 1.73205 v = rro<br />

Use degree mode.<br />

12tr 3<br />

Enter: 1.73205 ~ §]<br />

--=-ro<br />

or ~ ~ 1.73205 ~ 375 5<br />

Display: 60°<br />

ro=~(12tr)<br />

8=60° 3 375<br />

8 = 6O(.!!...-) =!!. radian ro = 75 ra Ian per sec<br />

180 3<br />

62. Let s = the length of the arc. 67. r = 11.46 cm, ro= 4.283 radians per sec,<br />

y<br />

t = 5.813 sec<br />

tans =-<br />

s = r8 = r(Ot<br />

X<br />

-.5250622<br />

= (11.46)(4.283)(5.813)<br />

=<br />

.85106383<br />

=285.3 em<br />

= -.6170.<br />

Use radian mode.<br />

68. The flywheel is rotating 90 times per sec or<br />

Enter: .6170<br />

90 . 2tr radians per sec,<br />

~ §]<br />

or<br />

r=7m<br />

~ ~ .6170 ~<br />

Display: -.5528258<br />

v = rro<br />

s = .5528<br />

= 7(90 ·2tr)<br />

Note that s is positive since it represents a length.<br />

= 1260tr m per sec<br />

5tr 8tr . 69. (b) csc8 =2<br />

63. 8 = -, (0= - radians per sec<br />

12 9 . 8 1<br />

SID =­<br />

8<br />

(0=tr<br />

2<br />

t 8=­<br />

6<br />

-=­ 8tr "*<br />

d is double h when the sun is 30° above the<br />

9 t<br />

5tr 9<br />

horizon.<br />

t=_·­<br />

12 8tr<br />

tr _ tr 5<br />

45 15 () c csc-=1 and csc- "" 1.1<br />

=-=-sec 2 3<br />

96 32 When the sun is lower in the sky<br />

64. t = 12 sec, ro= 9 radians per sec (8 = ~). sunlight is filtered by more<br />

8<br />

ro=­<br />

atmosphere. There is less ultraviolet light<br />

t<br />

8<br />

reaching the earth's surface, and therefore,<br />

9=­ there is less likelihood of becoming<br />

12<br />

sunburned. In this case, sunlight passes<br />

8 = 108 radians<br />

through 15% more atmosphere.<br />

65. t = 8 sec, 8 = 2tr radians<br />

5<br />

8<br />

ro=t<br />

l2r.<br />

=.2....<br />

8<br />

2tr tr d'<br />

=-=- ra tans per sec<br />

40 20<br />

4tr<br />

di<br />

111


<strong>Trigonometry</strong> <strong>Solutions</strong> <strong>Manual</strong><br />

112 Chapter 3 Radian Measure and the Circular Functions<br />

Chapter 3 Test Exercises<br />

2. 120 0 = 12"/....!!..- radian) = 21r radians<br />

\180 3<br />

0)<br />

3. 91r =91r(180 =1620<br />

10 10 1r<br />

4. r = 150 em, s = 200 em<br />

s =r()<br />

()=!..<br />

r<br />

200<br />

=­<br />

150<br />

4<br />

=­<br />

3<br />

4<br />

r = 150 em, s = 200 em, (J = ­<br />

5. 3<br />

A =.!. r 2 (J<br />

2<br />

=~(150)2(~)<br />

71r .<br />

6. s=6<br />

=15,000 em 2<br />

. 71r 1<br />

SIO-=-­<br />

6 2<br />

71r .,fj<br />

eos-=-­<br />

6 2<br />

71r .,fj<br />

tan-=­<br />

6 3<br />

7. {s Is '¢ ~ +n1r} are the domains of the tangent and secant circular functions.<br />

8. sin s = .82584121<br />

Enter: .82584121 8 ~<br />

or B ~ .82584 12 1 ~<br />

s « .97169234<br />

112


<strong>Trigonometry</strong> <strong>Solutions</strong> <strong>Manual</strong><br />

Chapter 3 Test Exercises 113<br />

9. In 11.64 years, Jupiter travels 2n- radians.<br />

r = 483,600,000<br />

o 2n­<br />

m=-=-­<br />

t 11.64<br />

v=rm<br />

= 483,600, rvvI~)<br />

vv'\I1.64<br />

= 261,043,678.2 miles per year<br />

To convert this to miles per second:<br />

)( 1minute)<br />

year 365 days }.24 hours 60 minutes 60 seconds<br />

= 8.3 miles per second<br />

v = (261,043,678.2 milesX 1 year Y 1day X 1 hour<br />

10. If the person travels n- radians, then he or she is at the top of the Ferris wheel, so he or she is 50 ft off the<br />

ground. The person is !!.. radians short of n- radians if he or she travels 5n- radians.<br />

6 6<br />

distance = 50 ft - ( 25 - 25sin ~ )<br />

:::: 46.65ft<br />

11. t = 30 sec, 0 =5n- radians<br />

6<br />

e<br />

m=­ t<br />

=~<br />

30<br />

n- di = - ra Jan per sec<br />

36<br />

12. (a) sin 0 = l.<br />

r<br />

. 0 y<br />

Sin =-­<br />

2.625<br />

y = 2.625 sin 0<br />

(b) cosO =!!.<br />

r<br />

cosO=_u­<br />

2.625<br />

u = 2.625 cos 0<br />

s<br />

(c) cosa=­<br />

L<br />

s<br />

cosa=-­<br />

10.5<br />

s= 10.5cos a<br />

(d) x=u+s<br />

x = 2.625cos 0 + 10.5cos a<br />

(e) v=-2.625sin11+ ~ cosO }505,168.1'( 1 )<br />

vl 15 + cos 2 0 \12 ·5280<br />

The maximum velocity of 2 L757 mph occurs when 0 = 4.94415 radians.<br />

113


<strong>Trigonometry</strong> <strong>Solutions</strong> <strong>Manual</strong><br />

114 Chapter 4 Graphs of the Circular Functions<br />

CHAPTER 4 GRAPHS OF THE<br />

CIRCULAR FUNCTIONS<br />

Section 4.1<br />

Connections (page 141)<br />

2<br />

1. One example is f(x) =x .<br />

2. One example is f(x) =x 3 .<br />

3. (a) f(x) =1x 1<br />

(b)<br />

(c)<br />

f(-x) =1- xl= I(-I)x 1=1xl<br />

even<br />

g(x) =V;<br />

g(-x)=~ =~(-I)x =~<br />

odd<br />

h(x) =x<br />

2<br />

+x<br />

h(-x) =(_x)2 +(-x) =x<br />

2<br />

- x<br />

neither<br />

Connections (page 148)<br />

1. X = -0.4161468, Y= 0.90929743, X is cos 2 and<br />

Y is sin 2.<br />

2. X = 1.9, Y= 0.94630009; sin 1.9 = 0.9463009<br />

3. X = 1.9,Y= -0.3232896; cos 1.9 = -0.3232896<br />

Exercises<br />

1. y= sin x<br />

The graph is a sinusoidal curve with an amplitude<br />

of 1 and a period of 21r. Since sin 0 =0, the point<br />

(0. 0) is on the graph . This matches with graph G.<br />

2. y= cos x<br />

The graph is a sinusoidal curve with an amplitude<br />

of 1 and a period of 21r. Since cos 0 = 1, the point<br />

(0, I) is on the graph. This matches with graph A.<br />

3. y = -sin x<br />

The graph is a sinusoidal curve with an amplitude<br />

of 1 and a period of 21r. Because a = -1, the graph<br />

is a reflection of y =sin x in the x-axis. This<br />

matches with graph E.<br />

4. y =-cos x<br />

The graph is a sinusoidal curve with an amplitude<br />

of 1 and a period of 2n: Because a = -1, the graph<br />

is a reflection of y = cos x in the x-axis. This<br />

matches with graph D.<br />

5. y= sin 2x<br />

The graph is a sinusoidal curve with a period of<br />

1'C and an amplitude of 1. Since sin 2(0) = 0, the<br />

point (0, 0) is on the graph . This matches with<br />

graph B.<br />

6. y=cos 2x<br />

The graph is a sinusoidal curve with a period of<br />

1rand an amplitude of 1. Since cos 2(0) =1, the<br />

point (0, 1) is on the graph. This matches with<br />

graph H.<br />

7. y = 2 sin x<br />

The graph is a sinusoidal curve with a period of<br />

21rand an amplitude of 2. Since 2 sin 0 = 0, the<br />

point (0, 0) is on the graph. This matches with<br />

graph F.<br />

8. y=2cosx<br />

The graph is a sinusoidal curve with a period of<br />

21rand an amplitude of 2. Since 2 cos 0 = 2, the<br />

point (0, 2) is on the graph. This matches with<br />

graph C.<br />

9. y = 2 cos x<br />

Amplitude: 121 =2<br />

o 1r 21r<br />

2 ~x 1-----------­<br />

2 o -2 o 2<br />

This table gives five values for graphing one<br />

period of the function.<br />

Repeat this cycle for the interval<br />

[-2n; 0] .<br />

y<br />

y=2cosx<br />

10. y =3 sin x<br />

Amplitude: 131 =3<br />

xj_o_<br />

1'C 21r<br />

3 sin x 0 3 0 -3 0<br />

This table gives five values for graphing one<br />

period of y =3 sin x. Repeat this cycle for the<br />

interval [-21r, 0].<br />

114


<strong>Trigonometry</strong> <strong>Solutions</strong> <strong>Manual</strong><br />

4.1 Graphs of the Sine and Cosine Functions 115<br />

y<br />

y<br />

y=3sinx<br />

y=-cosx<br />

115


<strong>Trigonometry</strong> <strong>Solutions</strong> <strong>Manual</strong><br />

116 Chapter4 Graphs of the CircularFunctions<br />

y<br />

y<br />

y=-3 cos x<br />

-+-I-Hk-::+-I--i-HI--x<br />

116


<strong>Trigonometry</strong> <strong>Solutions</strong> <strong>Manual</strong><br />

4.1 Graphs of the Sine and Cosine Functions 117<br />

21. y = sin 3x<br />

P iod 2n en :­<br />

3<br />

Amplitude: 111 = 1<br />

y<br />

y=2sin!x 4<br />

x<br />

X 0<br />

t t<br />

3x 0<br />

f n Jzr.<br />

2<br />

sin 3x 0<br />

y<br />

y=sin3x<br />

1r<br />

"2 If<br />

2n<br />

0 -1 0 24. y = 3 sin 2x<br />

P en iod : -=1r 21r<br />

2<br />

Amplitude: 3<br />

x<br />

3':2X I<br />

y<br />

0<br />

0<br />

i­<br />

3<br />

f<br />

0<br />

-f­<br />

-3<br />

1r<br />

0<br />

22. y=cos2x<br />

P en iod : -=1r 21r<br />

2<br />

Amplitude: 111 = 1<br />

x<br />

x<br />

2x<br />

cos2x<br />

y<br />

0<br />

0<br />

i­<br />

1r<br />

"2<br />

0<br />

f<br />

-1<br />

1r<br />

Jzr.<br />

4<br />

¥<br />

0<br />

1r<br />

21r<br />

25. y=-2 cos 3x<br />

P en iod :­ 2n<br />

3<br />

Amplitude: 2<br />

y=cos 2x x 0 t t f 1f­<br />

3x 0 f<br />

1r<br />

-l<br />

21r<br />

x<br />

-2 cos 3x -2 0<br />

y<br />

y=-2cos 3x<br />

2<br />

0<br />

-2<br />

23. y= 2 sm-x ' 1<br />

4 x<br />

Period: 2; = 8n<br />

Amplitude: 2<br />

x<br />

l.x<br />

4<br />

2sintx<br />

0<br />

0<br />

0<br />

21r<br />

f<br />

2<br />

41r<br />

1r<br />

0<br />

61r<br />

¥<br />

-2<br />

81r<br />

21r<br />

0<br />

26. Y=-5 cos 2x<br />

P<br />

eno<br />

. d<br />

: -=1r<br />

21r<br />

2<br />

Amplitude: 1- 51=5<br />

-5~'2x I<br />

0<br />

-5<br />

i­<br />

0<br />

f<br />

5<br />

-t<br />

0<br />

1r<br />

-5<br />

117


<strong>Trigonometry</strong> <strong>Solutions</strong> <strong>Manual</strong><br />

118 Chapter 4 Graphs of the Circular Functions<br />

-5<br />

y<br />

y=-5 cos 2x<br />

---+-f+-F+,t-HH--- x<br />

1l rr<br />

2<br />

27. y = cos 1C x<br />

Period: 21C =2<br />

1C<br />

Amplitude: 1<br />

30. The graph has an amplitude of .!.. and a period of<br />

4<br />

4. Then y = a sin bx where a =.!.. and b = 1C , so<br />

4 2<br />

1 . 1C<br />

y=-sm-x.<br />

4 2<br />

31. The graph has an amplitude of 4 and period of 41C.<br />

Then y = a cos(bx - ~) where a = 4 and<br />

b=21C=.!.. SOY=4COJ.!..x- 1C) .<br />

41C 2' \2 2<br />

y<br />

o I 1 3 2<br />

2" 2"<br />

1 o -1 o 1<br />

y = COSlrX<br />

32. The graph has an amplitude of .!.. and period of 4.<br />

4<br />

Then y =a cos(bx :... ~) where a =±and<br />

1C).<br />

b= 21C = 1C so y=.!..coJtr x_<br />

4 2' 4 \.2 2<br />

-t-+-t-I"'*"+-t-I-t-~ x<br />

33. The graph repeats each day, so the period is<br />

24 hours.<br />

. I 2.6-0.2 2<br />

34• Approximate y =1.<br />

2<br />

28. y = -sin 1C x<br />

Period: 21C =2<br />

1C<br />

Amplitude: I-~ =1<br />

x o 1­<br />

2<br />

o<br />

1 1.<br />

2<br />

-sin 1rX o -1 o o<br />

y<br />

Y =-SiD 10:<br />

I<br />

In Exercises 29-32, there are other correct answers.<br />

2<br />

21C<br />

29. The graph has an amplitude of 4 and a period of<br />

. 1<br />

41C. Then y = a sin bxwhere a = 4 and b = -, so<br />

2<br />

. 1<br />

y= 4sm-x.<br />

2<br />

35. Approximately 6 P.M., approximately 0.2 feet.<br />

36. The low tide occurred at 6 P.M. + 1:19, or<br />

7:19 P.M., the height was approximately<br />

0.2 - 0.2 =0 feet.<br />

37. Approximately 2 A.M.; approximately 2.6 feet.<br />

38. The high tide occurred at 2 A.M. + 1:18, or<br />

3:18 A.M.; the height was approximately<br />

2.6 - 0.2 =2.4 feet.<br />

39. (a) The amplitude is 120 - 80 = 40 =20.<br />

2 2<br />

(b) Since the period is .8 sec, there are<br />

-<br />

1<br />

=1.25 beats per sec and the pu<br />

I<br />

se rate<br />

.<br />

IS<br />

.8<br />

60(1.25) =75 beats per min.<br />

40. (a) The latest time that the animals begin their<br />

evening activity is 8:00 P.M., the earliest<br />

time is 4:00 P.M.<br />

4:00 s y s 8:00<br />

Since there is a difference of 4 hr in these<br />

times, the amplitude is .!..(4) =2 hr.<br />

2<br />

(b) The length of this period is 1 yr.<br />

118


\ ••+<br />

<strong>Trigonometry</strong> <strong>Solutions</strong> <strong>Manual</strong><br />

4.1 Graphs of the Sine and Cosine Functions 119<br />

41. (a) The highest temperature is 80°; the lowest is (d) E<br />

50°.<br />

5<br />

(b) The amplitude is .!.(800-500) = 15°.<br />

2<br />

.0083 .025<br />

(c) The period is about 35,000 yr. o<br />

.0 167 .033<br />

(d) The trend of the temperature now is<br />

downward.<br />

- 5<br />

42. (a) The amplitude of the graph is .!.; the period<br />

3<br />

. 3 S· 2n 3 k 4n Th .<br />

=3· e equanon<br />

IS 2'. mce 1:=2"<br />

. 1 . 4n t<br />

IS y=-sm-.<br />

3 3<br />

E=5 cos 120m<br />

46. E = 3.8 cos 40n t<br />

(a) Amplitude: 3.8<br />

Period: 2n =_1<br />

40n 20<br />

3<br />

(b) It takes - sec for a complete movement of (b) Frequency =number of cycles per second<br />

2 .<br />

1<br />

the arm. =--=20<br />

period<br />

43. -1:5y:5 1<br />

Amplitude: 1<br />

(c) t= .02<br />

Period: 8 squares = 8(30°)<br />

E = 3.8 cos 401l(.02) "" -3.074<br />

4n<br />

t= .04<br />

=240° or ­ E = 3.8 cos 40n(.04) = 1.174<br />

3<br />

t= .08<br />

44. -1:5y:51<br />

E = 3.8 cos 40n(.08) "" -3.074<br />

Amplitude: 1<br />

1= .12<br />

E = 3.8 cos 40n(.12):;: -3.074<br />

Period: 4 squares = 4(300)<br />

t =.14<br />

21r<br />

=1200 or ­ E = 3.8 cos 40n(.14) "" 1.174<br />

3<br />

(d) E<br />

45. E = 5 cos 120m­<br />

(a) Amplitude: 151 =5<br />

Period: ~ =_1_ sec<br />

120n 60<br />

(b) Number of cycles per second -3 .8<br />

=_1_=_1 =60<br />

period<br />

io<br />

(c) t = 0<br />

E =5 cos 12011(0) = 5<br />

3.8<br />

E = 3.8 cos 40m<br />

47. (a) The graph has a general upward trend along<br />

with small annual oscillations.<br />

1= .03 L (x ) = .022x 2 + .55x + 316<br />

+ 3.5 sin(21Tx)<br />

E = 5 cos 120Jr(.03) "" 1.545<br />

'------"'""<br />

t= .06 365<br />

E =5 cos 120n(.06) "" -4.045<br />

t= .09<br />

E =5 cos 120n(.09) "" -4.045<br />

t= .12<br />

E =5 cos 120Jr(.12) "" 1.545<br />

15 . ". . 35<br />

325<br />

119


<strong>Trigonometry</strong> <strong>Solutions</strong> <strong>Manual</strong><br />

120 Chapter 4 Graphs of the Circular Functions<br />

(b) The seasonal variations are caused by the<br />

term 3.5 sin (2tr x). The maximums will<br />

occur when<br />

159<br />

x=4"' 4"' 4"'<br />

S· mce x IS<br />

.<br />

in<br />

.<br />

years, x = 4"<br />

1<br />

correspon<br />

d<br />

s to<br />

April when the seasonal carbon dioxide<br />

levels are maximum. The minimum levels<br />

will occur when<br />

3 7 11<br />

x=4"' 4"' 4'<br />

This is .!. yr later which corresponds to<br />

2<br />

October.<br />

48. (a) The graph of C has a general upward trend<br />

similar to L (in Exercise 47) except that both<br />

the carbon dioxide levels and the seasonal<br />

oscillations are lar er for C than L.<br />

C(x) = .04x 2 + .6x + 330<br />

+ 7.5 sin(21Tx)<br />

380<br />

(c) To solve this problem, horizontally translate<br />

the graph of C a distance of 1970 units to the<br />

right. The new C function can be written<br />

C(x) = .04(x -1970)2 + .6(x -1970)<br />

+ 330+ 7.5sin[21t(x -1970)],<br />

where x is the actual year. This function<br />

would now be valid for 1970 Sx S 1995.<br />

53. Since cos (x) S 1 for all x, the calculator will<br />

return 1.<br />

54. Since sin (x) S 1 ~ trfor all x, the calculator will<br />

return 1.<br />

57. (a) The maximum heights will occur when<br />

135<br />

t=4"' 4"' 4' ....<br />

The maximum height above equilibrium is<br />

s(±)= -5COS'±) = -5(-1) =5 in.<br />

1 1<br />

(b) Frequency = --.-= ..1 = 2 cycles per sec<br />

penod ~<br />

Pen<br />

iod =-=- 2Jr 1 sec<br />

4Jr 2<br />

(c) The weight first reaches its maximum height<br />

1<br />

at t = - sec.<br />

4<br />

(d) s(1.3) = -5 cos 41r(1.3) ""4<br />

After t =1.3 sec, the weight is about 4 in.<br />

above the equilibrium position.<br />

58. (a) The maximum heights occur when<br />

Jr 3Jr 5Jr Jr<br />

t=IO'1O'1O=2',····<br />

The maximum height above equilibrium is<br />

s(~) =-4COS1O(~)<br />

=-4cos Jr =-4(-1) = 4 in.<br />

1 1 5<br />

(b) Frequency: -'--=- =- cycles per sec<br />

period f Jr<br />

(c)<br />

Period: 2Jr = Jr sec<br />

10 5<br />

The weight first reaches its maximum height<br />

Jr<br />

when t =-sec.<br />

10<br />

(d) s(1.466) =-4 cos 10(1.466) "" 2<br />

After 1.466 sec, the weight is about 2 in.<br />

above the equilibrium position.<br />

55. sin ~ = 1 and sin 3Jr = -1, so Isin 3Jr I= 1-II= 1.<br />

222<br />

Jr 3Jr<br />

Thus, x=- andx=-.<br />

2 2<br />

56. cos 0 =1 and cos Jr=-1, so Icos JrI= 1-II= 1.<br />

Thus, x = 0 and x =Jr.<br />

120


<strong>Trigonometry</strong> <strong>Solutions</strong> <strong>Manual</strong><br />

4.2 Translations of the Graphs of the Sine and Cosine Functions 121<br />

59. (a) Use the model y =a cos bx.<br />

Since the weight starts at 3 in. below the<br />

equilibrium position, a =-3 and the cosine<br />

function can be used without shifting it to the<br />

right or left.<br />

1<br />

Frequency = --.­<br />

penod<br />

6 1<br />

-=--­<br />

1C period<br />

1C 'od<br />

6"= pen<br />

Period = 21C = 1C . b =12<br />

b<br />

y =-3cosI2t<br />

(b) Period = 1C sec<br />

6<br />

6'<br />

60. (a) Use the model y = a cos bx.<br />

Since the weight starts at 2 in. below the<br />

equilibrium position, a =-2 and the cosine<br />

function can be used without shifting it to the<br />

right or left.<br />

Section 4.2<br />

Exercises<br />

Period = 21C =1.. b = 61C<br />

b 3'<br />

y =-2cos 61C t<br />

1 1<br />

(b) Frequency =--.- =- = 3 cycles per sec<br />

penod<br />

1. B; amplitude =3, period = 21C = 1C,<br />

2<br />

phase shift = .± =2<br />

2<br />

2. D; amplitude =2, period = 21C, phase shift = .±<br />

3 3<br />

3. C; amplitude =4, period = 21C, phase shift = 3­<br />

3 3<br />

4. A; amplitude = 2, period = 21C =1C ,<br />

4 2<br />

phase shift = ~<br />

4<br />

t<br />

5. y =sin(x - :)<br />

The graph is a sinusoidal curve y = sin x shifted<br />

1C units to the right. This matches graph D.<br />

4<br />

6. y = sin ( x + : )<br />

The graph is a sinusoidal curve y =sin x shifted<br />

1C units to the left. This matches graph G.<br />

4<br />

7. y=cos(x- =)<br />

The graph is a sinusoidal curve y = cos x shifted<br />

1C units to the right. This matches graph H.<br />

4<br />

8. y=cos(x+:)<br />

The graph is a sinusoidal curve y =cos x shifted<br />

1C units to the left. This matches graph A.<br />

4<br />

9. y =1 + sin x<br />

The graph is a sinusoidal curve y =sin x<br />

translated vertically 1 unit up. This matches<br />

graph B.<br />

10. y=-1 +sinx<br />

The graph is a sinusoidal curve y = sin x<br />

translated vertically 1 unit down. This matches<br />

graphE.<br />

11. y =1 + cos x<br />

The graph is a sinusoidal curve y =cos x<br />

translated vertically 1 unit up. This matches graph<br />

F. .<br />

12. y =-1 + cos x<br />

The graph is a sinusoidal curve y =cos x<br />

translated vertically 1 unit down. This matches<br />

graph C.<br />

13. right<br />

14. left<br />

15. y =2 sin (x - 1C)<br />

The amplitude is 121,which is 2.<br />

The period is 21C , which is 21C.<br />

1<br />

There is no vertical translation.<br />

The phase shift is 1C units to the right.<br />

121


<strong>Trigonometry</strong> <strong>Solutions</strong> <strong>Manual</strong><br />

122 Chapter 4 Graphs of the Circular Functions<br />

16. 2 . ( n)<br />

Y="3 sm x+2" 21. Y = 2 - sin(3x - ~)<br />

17.<br />

The amplitude is I~ I. which is ~.<br />

The period is 2n. which is 2n.<br />

I<br />

There is no vertical translation.<br />

The phase shift is n units to the left.<br />

2<br />

Y=4co{f+ ;)<br />

I<br />

y = 4cos-(x +n)<br />

2<br />

The amplitude is 141. which is 4.<br />

22.<br />

Y=-Isin{x- ~)+2<br />

The amplitude is 1-II. which is 1.<br />

Th epen iod i IS-. 2n<br />

3<br />

The vertical translation is 2 units up. The phase<br />

shift is !!.... units to the right.<br />

15<br />

1<br />

Y =-1+ -cos(2x - 31t}<br />

2<br />

y=-I+2"cos2 1 ( x-T 31t)<br />

The period is 2;. which is 4n. The amplitude is I~I. which is ~.<br />

There is no vertical translation.<br />

The phase shift is n units to the left.<br />

Th e pen iod i IS -2n W hi IC h iIS 1t.<br />

2<br />

The vertical translation is 1 unit down.<br />

18. y=-cos~(x- ~) The phase shift is 3n units to the right.<br />

2<br />

Y=-ICOS~(X- ~)<br />

The amplitude is I-II. which is 1.<br />

The period is 2;. which is 3n.<br />

There is no vertical translation.<br />

The phase shift is n units to the right.<br />

3<br />

23. Y=Co{x- ;)<br />

The amplitude is 1.<br />

The period is 2n.<br />

There is no vertical-translation,<br />

The phase shift is n units to the right.<br />

2<br />

y<br />

19. Y=3COS2(X-:)<br />

The amplitude is 131. which is 3.<br />

Th e pen iod i IS 2n w hi IC h iIS n.<br />

There is no vertical translation.<br />

The phase shift is ~ units to the right.<br />

4<br />

... 20. 1 . (x )<br />

Y=2"sm 2"+n<br />

IO~·X<br />

-I -!! ~ 2!!'<br />

222<br />

Y =.!.sin.!.(x + 2m<br />

2 2<br />

The amplitude is I~ I. which is ~. The period is<br />

2n h' h i 4 -)- W IC IS n:<br />

'2<br />

There is no vertical translation.<br />

The phase shift is 2n units to the left.<br />

122


<strong>Trigonometry</strong> <strong>Solutions</strong> <strong>Manual</strong><br />

4.2 Translations of the Graphs of the Sine and Cosine Functions 123<br />

24. Y =sin(x - :)<br />

The amplitude is 1.<br />

The period is 21r.<br />

There is no vertical translation.<br />

The phase shift is 1r units to the right.<br />

4<br />

y<br />

27. Y=2CO{X- ~)<br />

The amplitude is 2.<br />

The period is 21r.<br />

There is no vertical translation.<br />

The phase shift is 1r units to the right.<br />

3<br />

y<br />

25. Y =sin(x +:)<br />

The amplitude is 1.<br />

The period is 2It<br />

There is no vertical translation.<br />

The phase shift is 1r units to the left.<br />

4<br />

y<br />

28. Y =3sin(x _ 3;)<br />

The amplitude is 3.<br />

The period is 21r.<br />

There is no vertical translation.<br />

The phase shift is 31r units to the right.<br />

2<br />

y<br />

-~-+-Hrr+t-f--J(<br />

-'><br />

4<br />

y = sin (x + ~<br />

y =3 sin (x - ¥)<br />

26. y=co{x- ~)<br />

The amplitude is 1.<br />

The period is 21r.<br />

There is no vertical translation.<br />

The phase shift is 1r units to the right .<br />

3<br />

y<br />

29. Y =~sin2( x+ :)<br />

The amplitude is l.<br />

2<br />

Thepen 'ad<br />

IS<br />

' 2' 21r<br />

W<br />

hiIC h iIS x.<br />

There is no vertical translation.<br />

The phase shift is 1r units to the left.<br />

4<br />

y<br />

2<br />

y=cos(x-f)<br />

-2 y= ~sin 2(x+~)<br />

123


<strong>Trigonometry</strong> <strong>Solutions</strong> <strong>Manual</strong><br />

124 Chapter 4 Graphs of the Circular Functions<br />

30. y = -±COS4(x+ ~)<br />

The amplitude is 1-±I, which is ~ ,<br />

IS<br />

,2TC<br />

-, W<br />

hi<br />

IC<br />

h i<br />

IS-,<br />

TC<br />

4 2<br />

There is no vertical translation,<br />

The phase shift is TC units to the left.<br />

2<br />

y<br />

The period<br />

32. y = 3 cos (4x + 1t)<br />

Y=3COS4(X+~)<br />

The amplitude is 3.<br />

Th iod i 2TC hi h i TC<br />

e pen IS "4' W IC IS "2'<br />

There is no vertical translation,<br />

The phase shift is TC units to the left.<br />

4<br />

y<br />

--+--;---,f-+--\o-+--+-~ x<br />

2"<br />

-I<br />

-+-~H-++----l-_ x<br />

_!J 0 !J<br />

4 4<br />

31.<br />

y = -4 sin (2x - TC)<br />

Y =-4Sin2(x- ~)<br />

The amplitude is 1- 41, which is 4,<br />

Th e pen iod i IS -, 2TC W hi IC h iIS TC.<br />

2<br />

There is no vertical translation,<br />

The phase shift is TC units to the right.<br />

2<br />

4<br />

y<br />

33.<br />

-3<br />

y=3cos(4x+ n)<br />

y =..!.cos(.!.x _ TC)<br />

2 2 4<br />

y =.!.cos.!.(X _ TC)<br />

2 2 :2<br />

The amplitude is .!.,<br />

2<br />

--1-.....-+---+--'-+---i~X<br />

o ' ~ n 3"<br />

'2<br />

The period is 2 1<br />

1t , which is 41r.<br />

'2<br />

This is no vertical translation,<br />

-4<br />

y =-4 sin (2x ­ 11")<br />

The phase shift is TC<br />

2<br />

y<br />

units to rhe right.<br />

5"<br />

"2<br />

y=!cosHx-~)<br />

124


<strong>Trigonometry</strong> <strong>Solutions</strong> <strong>Manual</strong><br />

34. Y=-'41. (3 sm '4<br />

1r) x +g<br />

1 " 3 (<br />

Y=-'4 sm'4 1r)<br />

x+"6<br />

4.2 Translations of the Graphs of the Sine and Cosine Functions 125<br />

2 . 3<br />

37. y= 1 --sm-x<br />

3 4<br />

The amplitude is 1-±I, which is ±. Th iod i<br />

Th iod i 21r hi h i 81r<br />

epen IS T'w IC IS­<br />

4 3<br />

This is no vertical translation.<br />

The phase shift if 1r units to the left.<br />

6<br />

y<br />

The amplitude is 1-%I, which is %.<br />

epen IS T' 21t w hi<br />

IC<br />

h<br />

IS-.<br />

i 8n<br />

'4 3<br />

The vertical translation is 1 unit up.<br />

There is no phase shift.<br />

y<br />

y = I_l sin Jx<br />

3 4<br />

-~ -~<br />

3 3<br />

38. y = -1 - 2 cos 5x<br />

The amplitude is 1- 21. which is 2.<br />

35. y =-3 + 2 sinx<br />

The amplitude is 2.<br />

The period is 2n.<br />

The vertical translation is 3 units down.<br />

There is no phase shift.<br />

y y = -3 + 2 sin x<br />

-211" -II" 0 II" 2Il"<br />

-I<br />

36. y =2 - 3 cos X<br />

The amplitude is 1- 31, which is 3.<br />

The period is 2n.<br />

The vertical translation is 2 units up.<br />

There is no phase shift.<br />

y<br />

y=2-3cosx<br />

-t-Jt--t-~-Jt--t-t\-t- ..<br />

•<br />

39.<br />

Th epen iod IS-. i 2n<br />

5<br />

The vertical translation is 1 unit down.<br />

There is no phase shift.<br />

y<br />

y =- 1- 2 cos 5x<br />

2Jr<br />

5"<br />

--+fIt+iHH---X<br />

I<br />

y= 1-2cos-x<br />

2<br />

The amplitude is 1- 21, which is 2.<br />

The period is 2 11t,<br />

which is 4n.<br />

Z<br />

The vertical translation is 1 unit up.<br />

There is no phase shift.<br />

y<br />

y= 1-2 cos!x 2<br />

125


<strong>Trigonometry</strong> <strong>Solutions</strong> <strong>Manual</strong><br />

126 Chapter 4 Graphs of the Circular Functions<br />

40. y=-3+3 sin.!..x<br />

2<br />

The amplitude is 3.<br />

Th "00' 2rr hi h i 4<br />

e pen IS T' w IC IS n.<br />

The vertical translation is 3 units down.<br />

There is no phase shift.<br />

y<br />

y = -3 + 3 sin ~x<br />

43. Y=-3+2sin(x+ ~)<br />

The amplitude is 2.<br />

The period is 2rr.<br />

The vertical translation is 3 units down.<br />

The phase shift is rr units to the left .<br />

2<br />

y<br />

-+--=+--+~f--+-. x<br />

-411' 411'<br />

41. y=-2+.!..sin3x<br />

2<br />

The amplitude is .!...<br />

2<br />

Th epen<br />

iod i 2rr<br />

IS 3'<br />

The vertical translation is 2 units down .<br />

There is no phase shift.<br />

y<br />

44. y = 4 - 3 cos (x ­ rr)<br />

The amplitude is 1- 31, which is 3.<br />

The period is 2rr.<br />

The vertical translation is 4 units up.<br />

The phase shift is rr units to the right.<br />

y<br />

7<br />

y =4- 3 cos (x-Ir)<br />

4<br />

1<br />

o<br />

IC 21r 3Jr<br />

42.<br />

2 1<br />

y=l+-cos-x<br />

3 2<br />

The amplitude is %.<br />

The period is 2 1<br />

lt , which is 4rr.<br />

"2<br />

The vertical translation is 1 unit up.<br />

There is no phase shift .<br />

y<br />

45. y =~ + sin 2(x +:)<br />

The amplitude is 1.<br />

. Th . e pen iod i IS - 2rr . w hi Ie h iIS rr.<br />

". 2<br />

The vertical translation is .!.. unit up.<br />

2<br />

The phase shift is !!.. units to the left.<br />

4<br />

y<br />

-II'<br />

"<br />

2<br />

Jr<br />

y= I +~COS!X<br />

3 2<br />

126


<strong>Trigonometry</strong> <strong>Solutions</strong> <strong>Manual</strong><br />

46. y =-%+ cos 3( x - : )<br />

4.2 Translations of the Graphs of the Sine and Cosine Functions 127<br />

51. Converting to a new cosine equation, the<br />

amplitude a =3 and period p = 21C =1C remain<br />

The amplitude is I. 2<br />

the same. We must shift the graph an additional<br />

Thepeno . d i 21r<br />

IS-.<br />

3 - 1C11C'th . - = - Units to eng 'ht so<br />

2 2 4 '<br />

The vertical translation is ~ units down .<br />

2 y =3cos 2(x - : - :)=3cos 2(x - ~ ) ,<br />

The phase shift is 1C units to the right.<br />

6<br />

y<br />

If<br />

v-­<br />

52. Converting to a cosine equation, the amplitude<br />

(;<br />

a =1C and period p = 21C = 2 remain the same.<br />

1r<br />

---=+-+-I-++-lH--- x<br />

o<br />

-~<br />

We must shift the graph an additional 1C •..!.. = .!..<br />

2 x 2<br />

units to the right, so,<br />

y =1C cos n(x -t-t) = 1C cos 1C(X-I),<br />

48.<br />

53. (a) Let January correspond to x = 1, February to<br />

x =2, ... , and December of the second year<br />

to x =24. Yes, the data appear to outline the<br />

graph of a translated sine graph.<br />

70<br />

Using the trace function, we see that the<br />

y-maximum occurs at I and y-minimum occurs at<br />

(b) The sine graph is vertically centered around<br />

1-(-1) 2 the line y =50. This line represents the<br />

-I, so the amplitude a = =- =1. average yearly temperature in Vancouver of<br />

2 2<br />

50°F. (This is also the actual .average yearly<br />

Changing the x-scl to ~, we find that the period temperature. )<br />

3<br />

is 51C _ (_!!.) =61r =21C and the phase shift is<br />

333<br />

1C<br />

a=-.<br />

3<br />

In Exercises 49-52, there are other correct answers.<br />

,: :::L. .~<br />

49. The graph has an amplitude of 3, a period of 1C,<br />

and a phase shift of 1r units to the right. In the<br />

4<br />

equation y =a sin btx - cl),a =3, b =2, and<br />

d = : ; therefore, y = 3sin 2(x - :).<br />

. Ḍ..<br />

50. The graph has an amplitude of n, a period of 2,<br />

and a phase shift of .5 unit to the right. In the<br />

equation y = a sin b(x - cl), a =1C, b =n, and<br />

d =,5; therefore, y =zrsin 1l(x - .5).<br />

127


<strong>Trigonometry</strong> <strong>Solutions</strong> <strong>Manual</strong><br />

128 Chapter 4 Graphs of the Circular Functions<br />

(c) The amplitude of the sine graph is<br />

approximately 14 since the average monthly<br />

high is 64, the average monthly low is 36.<br />

and 64 -36:= 14. The period is 12 since the<br />

2<br />

temperature cycles every twelve months. Let<br />

b =21r =1r. One way to determine the<br />

12 6<br />

phase shift is to use the following technique.<br />

The minimum temperature occurs in January.<br />

Thus, when x =I, b(x - d) must equal<br />

( - ~) + 2nn, where n is an integer, since the<br />

sine function is minimum at these values .<br />

Solving for d<br />

~ (l-d):=- ~<br />

d=4.<br />

This can be used as a first approximation.<br />

(d) Letj{x) =a sin b(x -d) + c.<br />

Since the amplitude is 14, let a = 14. The<br />

period is equal to one yr or 12 mo, so<br />

b =1r. The average of the maximum and<br />

6<br />

. . . 64+36 50 Le<br />

mrrumum temperatures IS<br />

:=. t<br />

2<br />

the vertical translation be c = 50.<br />

Since the phase shift is approximately 4, it<br />

can be adjusted slightly to give a better<br />

visual fit. Try 4.2. Since the phase shift is<br />

4.2, let d =4.2. Thus,<br />

f(x) = 14sin[~(X-4 .2)J+50 .<br />

(e) Plotting the data with<br />

f(x) =14sin[~ (X-4.2)J+50<br />

on the same coordinate axes gives a good fit.<br />

70<br />

f(x) = 14 sin [~(x - 4.2)] + 50<br />

(f)<br />

From the sine regression we get<br />

y:= 13.21sin(.52x -2.18)+49.68<br />

:=13.21sin[.52(x - 4.19)] + 49.68.<br />

54. (a) The maximum average monthly temperature<br />

is 90°F, and the minimum average monthly<br />

temperature is 51°F. The average of these<br />

two values is 90 + 51 =70.5°F, which is<br />

2<br />

very close to the actual value of 70°F.<br />

(c) Let the amplitude a be 90-51 =19.5.Since<br />

2<br />

the period is 12, let b =1r. Let<br />

6<br />

c:= 90+51 :=.. 705 Th emmlmum . .<br />

2<br />

temperature occurs in January. Thus, when<br />

x =1, b(x - d) must equal an odd multiple of<br />

1rsince the cosine function is minimum at<br />

these values . Solving for d<br />

!:(1-d):=-1r<br />

6<br />

d:=7<br />

d can be adjusted slightly to give a better<br />

visual fit. Try d:= 7.2. Thus,<br />

f(x):= acosb(x -d)+ c<br />

:= 19 .5CO{~ (x -7.2)J+ 70.5.<br />

1 "-.;"""""'__-....;.._....;....'--'--~.25<br />

30<br />

128


'<br />

<strong>Trigonometry</strong> <strong>Solutions</strong> <strong>Manual</strong><br />

4.3 Graphs of the Other Circular Functions 129<br />

d. Plotting the data with<br />

30<br />

f(x) = 19.5CO{: (x -7.2)]+ 70.5<br />

on the same coordinate axes give a good fit.<br />

I(x) = 19.5 cos [*(x - 7.2)J + 70.5<br />

95<br />

-.1 13<br />

-2<br />

Section 4.3<br />

Exercises<br />

55. a.<br />

e. SinRe9 ":'" .<br />

· . ~=a"'sin ( bx+c ) .+d<br />

:· a= 1 9 ~ 72 '- , '<br />

.'b=· i 2 '.r...:-./ .:.'<br />

c=­ .17 '.." . .<br />

~ · ~.=r ~.~~~ < ~ . ~~~ .: ~, > .<br />

~ . l ; " . ~. ~:: ~ . - '. ."<br />

TI-83 fixed to the<br />

nearest hundredth<br />

From the sine regression we get<br />

y = 19.72sin(.52x -2.17)+ 70.47<br />

= 19.72sin[.52(x- 4.17)]+ 70.47.<br />

TI-83 fixed to the<br />

nearest hundredth<br />

1. True; since cos!£ = 0, tan lC is undefined.<br />

2 2<br />

2. False; the smallest such number is n.<br />

. sinx 1<br />

3. True; since tan x =-- and sec x =--, the<br />

cos x<br />

cosx<br />

tangent and secant functions will be undefined at<br />

the same values.<br />

4. False; secant values are undefined when<br />

lC .<br />

x = - + nlC, while cosecant values are undefined<br />

2<br />

when x = nn:<br />

sin(-x) -sinx<br />

5. False; tan(-x) = =--=-tanx<br />

cos(-x) cos x<br />

(since sin x is odd and cos x is even) for all x in<br />

the domain.<br />

I I<br />

6. True; sec(-x) = =--=sec(x)<br />

cos(-x) costx)<br />

(since cos x is even) for all x in the domain.<br />

7. y =csc x<br />

The graph is the reflection of the graph of<br />

y = esc x about the x-axis. This matches with<br />

graph B.<br />

-.1 13<br />

20<br />

b. SinRe9 .<br />

~=a*s i n ( bx+c )+d<br />

a=11.36<br />

b=.48<br />

c=1.22<br />

d~12.69 .<br />

TI-83 fixed to the<br />

nearest hundredth<br />

8. y=-secx<br />

The graph is the reflection of the graph of<br />

y = sec x about the x-axis, This matches with<br />

graph C.<br />

9. y =-tan x<br />

The graph is the reflection of the graph of<br />

y = tan x about the x-axis, This matches with<br />

graph E.<br />

10. y =-eotx<br />

The graph is the reflection of the graph of<br />

y = cot x about the x-axis. This matches with<br />

graph A.<br />

129


<strong>Trigonometry</strong> <strong>Solutions</strong> <strong>Manual</strong><br />

130 Chapter 4 Graphs of the Circular Functions<br />

11. y =tan(x - :)<br />

The graph is the graph of y = tax x shifted<br />

1r units to the right. This matches with graph D.<br />

4<br />

12. y=co{x- :)<br />

13.<br />

The graph is the graph of y =cot x shifted<br />

1r units to the right. This matches with graph F.<br />

4<br />

y =csc(x ­ =)<br />

We graph this function by first graphing the<br />

corresponding reciprocal function<br />

15. y =sec(x +:)<br />

We graph this function by first graphing the<br />

corresponding reciprocal function<br />

y =cos(x + :). The period is 21r.<br />

0<br />

-I<br />

y<br />

1:<br />

3" ~<br />

4" IV:<br />

n7" 9"<br />

4 4 4" 4"<br />

I I I<br />

I<br />

I<br />

y=sec(x+n<br />

x<br />

14.<br />

y =sin(x ­ : ). The period is 21r.<br />

y<br />

1<br />

UI I<br />

0<br />

-I<br />

"<br />

Y1<br />

4 4 4<br />

I I I<br />

I<br />

I I I<br />

y=csc(x-~)<br />

y=sec(x+ 3:)<br />

We graph this function by first graphing the<br />

corresponding reciprocal function<br />

y = co{x+ 3:). The period is 21r.<br />

y<br />

i"U<br />

-+-!.,+H--hH-+---'x<br />

SIr 7"<br />

4" 4"<br />

I<br />

I<br />

I<br />

y=sec(x+¥)<br />

x<br />

16. y =csc(x+ ~)<br />

We graph this function by first graphing the<br />

corresponding reciprocal function<br />

y =sin(x + ~). The period is 21r.<br />

y<br />

7"<br />

6"<br />

'n<br />

,,2tr<br />

SIC<br />

-1-:o:J+--+-+--+-=-x<br />

-~-I<br />

I<br />

I<br />

I<br />

y=csc(x+~)<br />

130


<strong>Trigonometry</strong> <strong>Solutions</strong> <strong>Manual</strong><br />

4.3 Graphs of the Other Circular Functions 131<br />

17. y=sec(~x+ ~)<br />

1 ( 21l)<br />

y=sec'2 x+"'3<br />

We graph this function by first graphing the<br />

corresponding reciprocal function<br />

y =cos~(x + 2:). The period is 41l.<br />

y<br />

I<br />

I<br />

I UI I<br />

1 I ¥ I I<br />

--,::++++l-++++l-++++Ir- x<br />

10K 13"<br />

-IOl


<strong>Trigonometry</strong> <strong>Solutions</strong> <strong>Manual</strong><br />

132 Chapter 4 Graphs of the Circular Functions<br />

22. y =2 +±sec(~ x - Jr)<br />

1 1<br />

y =2 +-sec-(x-2Jr)<br />

4 2<br />

We graph this function by first graphing the<br />

corresponding reciprocal function<br />

y = 2 + .!.cos.!.(x - 2Jr). The period is 4Jr.<br />

4 2<br />

y<br />

I<br />

U ,<br />

I<br />

I<br />

I<br />

I I I<br />

--t: -t' t-­<br />

-lC 0 lC 2lC 3lC<br />

I<br />

I<br />

I<br />

Y= 2 +i sec (!X-lC)<br />

23. y = 2 tan x<br />

Two adjacent vertical asymptotes are x =_ Jr<br />

2<br />

and x = Jr . The graph is "stretched" because<br />

2<br />

a =2and 121 > 1.<br />

y<br />

1<br />

25. y= -cotx<br />

2<br />

Two adjacent vertical asymptotes are<br />

x =0 and x =Jr. The graph is "compressed"<br />

because a =~ and I~I< 1.<br />

y<br />

--"7I--"'1o..----I---x<br />

y=!cotx<br />

1<br />

26. y=2tan-x<br />

4<br />

Two adjacent vertical asymptotes are<br />

1 Jr 1 Jr<br />

-x=-- and -x=­<br />

4 2 4 2<br />

or x =~2Jr and x =2Jr.<br />

y<br />

-/---+---:ioI'''''-+--+_x<br />

---+.....::,jL.-+--_x<br />

y = 2 laDix<br />

y=2tanx<br />

24. y= 2 cotx<br />

Two adjacent vertical asymptotes are x =0<br />

and x = Jr. The graph is "stretched" because<br />

a = 2and 121 > 1.<br />

y<br />

27. y =cot3x<br />

Two adjacent vertical asymptotes are<br />

3x =0 and 3x =Jr<br />

Jr<br />

or x =0 and x =-.<br />

3<br />

y<br />

2<br />

---+~,.---/--+-_x<br />

y=2cotx<br />

--"7I----\----,I---x<br />

1<<br />

:3<br />

I<br />

-I I<br />

I<br />

I<br />

y= cot3x<br />

132


<strong>Trigonometry</strong> <strong>Solutions</strong> <strong>Manual</strong><br />

4.3 Graphs of the Other Circular Functions 133<br />

1<br />

28. Y =-cot-x<br />

2<br />

Two adjacent vertical asymptotes are<br />

1 1<br />

-x=O and -x=tr<br />

2 2<br />

or x =0 and x =2tr,<br />

The graph is the reflection of the graph of<br />

y =cot'!" x about the x-axis.<br />

2<br />

y<br />

30. y =tan(i + tr)<br />

1<br />

y =tan -(x + 2tr)<br />

2<br />

Two adjacent vertical asymptotes are<br />

1 tr 1 tr<br />

-(x+2tr)=-- and -(x+21r)=­<br />

2 2 2 2<br />

or x + 2tr =-tr and x + 2tr =tr<br />

or x =-3tr and x=-tr.<br />

y<br />

----j.,.--..".,,4


<strong>Trigonometry</strong> <strong>Solutions</strong> <strong>Manual</strong><br />

134 Chapter 4 Graphs of the Circular Functions<br />

35. y = 1 - cot x<br />

32. y =cot(2x _ 3;) This is the graph ofy = cot x reflected about the<br />

x-axis and then translated vertically 1 unit up.<br />

y<br />

y = cot 2(x _ 3:)<br />

Two adjacent vertical asymptotes are<br />

2(X- 3:)=0 and 2(X- 3:)=1r<br />

.1<br />

31r 31r 1r ---:~4--t"""'t-+---x<br />

or x--=O and x--=­<br />

12K<br />

442 I<br />

31r 51r<br />

or x=- and X=-.<br />

I<br />

4 4 y=l-cotx<br />

y<br />

36. y = -2 - cot x<br />

This is the graph of y = cot x reflected about the<br />

x-axis and then translated vertically 2 units down.<br />

y<br />

I<br />

I<br />

o<br />

Ill'<br />

" 4 -+--+-+t=--t-f+-__ x<br />

-1 I<br />

I<br />

I<br />

I<br />

y= cot (2x-¥)<br />

33. y= 1 + tan x<br />

This is the graph ofy = tan x translated vertically<br />

1 unit up.<br />

y<br />

y =-2-cotx<br />

37. y =-1 + 2 tan x<br />

This is the graph of y = 2 tan x translated<br />

vertically 1 unit down.<br />

y<br />

34. y = -2 + tax x<br />

This is the graph of y = tan x translated vertically<br />

2 units down.<br />

-4--=t-I-t-t++-_x<br />

+-­<br />

I<br />

I<br />

I<br />

I I<br />

y=-I +2tanx<br />

1<br />

38. y= 3+-tanx<br />

2<br />

y 1<br />

I I This is the graph of y = - tan x translated<br />

2<br />

"I 13"<br />

-21 IT vertically 3 units up.<br />

I<br />

-I-­<br />

I<br />

I<br />

I<br />

I<br />

x<br />

y<br />

--++~---r+-t--+-_x<br />

I I I<br />

y=3+ ~ tan x<br />

134


<strong>Trigonometry</strong> <strong>Solutions</strong> <strong>Manual</strong><br />

4.3 Graphs of the Other Circular Functions 135<br />

39.<br />

1<br />

y =,...1 +-cot(2x -<br />

2<br />

y = -1+~cot2(x _ 3;)<br />

Th epen<br />

'00' 1r<br />

IS "2'<br />

Two adjacent vertical asymptotes are<br />

2(X- 3;) = 0<br />

31t) 41. y = ~tan(~x -1C)-2<br />

and 2(X- 3;) = 1r<br />

31r 31r 1r<br />

or x--=o and x--=­<br />

222<br />

31r<br />

or x = ­ and x = 21r.<br />

2<br />

This is the graph of y = .!. cot 2x translated<br />

2<br />

vertically 1 unit down.<br />

y<br />

2<br />

I<br />

-+:-+t--++-+--+-~ x<br />

-l---­<br />

-2 I<br />

I<br />

I<br />

I I<br />

y=-I +! cot (2x-3n'><br />

Y=-2+~tan~(x-41C)<br />

3 4 3<br />

Th '00' 1C 41C<br />

epen IS.l =-.<br />

4 3<br />

Two adjacent vertical asymptotes are<br />

~(x- 4;)=_~ and ~(x- 4;)= ~<br />

41C 21C 41C 21C<br />

or x--=-- and x--=­<br />

3 3 3 3<br />

21r<br />

or x = ­ and x = 21C;'<br />

3<br />

This is the graph of y = ~ tan(~ x ­ 1C) translated<br />

vertically 2 units down.<br />

y<br />

I<br />

I<br />

2 ~ ~<br />

o 1r<br />

---l-!<br />

-2 I<br />

I<br />

I<br />

y =~ tan (~x - n) - 2<br />

L_<br />

I<br />

I<br />

I<br />

I<br />

40. y=-2+3tan(4x+1r)<br />

y=-2+3tan{x+ :) 42. y= 1-2cot2(X+ ~)<br />

Th epen '00'<br />

IS "4. 1r<br />

Th epen '00'<br />

Two adjacent vertical asymptotes are<br />

{x+ :)=­ ~ and {x+ :)= ~<br />

1C 1C 1C 1C<br />

or ·x+-=-­ and x+-=­<br />

4 8 4 8<br />

31C<br />

1C<br />

or x=-- and X=-­ .<br />

8 8<br />

or<br />

or<br />

"2.<br />

IS 1C<br />

The adjacent vertical asymptotes are<br />

2(X+~)=O and 2(X-+~)=1C<br />

x+ 1C=O and x+ 1C=1C<br />

2 2 2<br />

1C<br />

x=-­ and x=O.<br />

2<br />

This is the graph of y = 3 tan { x + :) translated This is the graph of y = -2cot2(x + ~)<br />

vertically 2 units down.<br />

_~I<br />

81<br />

I<br />

1<br />

1<br />

---t­<br />

1<br />

1<br />

1<br />

y<br />

1<br />

1 0<br />

I lr Ilr<br />

18 )8<br />

1-1 I<br />

I I<br />

I<br />

oF<br />

I<br />

-t--­<br />

I I<br />

I I<br />

I I<br />

I<br />

Y=-2 +3 tan (4x+ n)<br />

translated vertically 1 unit up.<br />

I<br />

I<br />

I<br />

I<br />

I<br />

--I<br />

y<br />

I<br />

I<br />

I<br />

Ị<br />

--­<br />

I"<br />

1 2<br />

I<br />

y=I-2cot2~+~)<br />

135


<strong>Trigonometry</strong> <strong>Solutions</strong> <strong>Manual</strong><br />

136 Chapter 4 Graphs of the Circular Functions<br />

43. The domain of the tangent function is<br />

{XF¢ ~ +n1r, wherenisanYinteger},<br />

and the range is (--00, 00). For the function<br />

!(x) =-4 tan(2x +1r)<br />

=-4tan2(x+ ~J<br />

the period is 1r . Therefore, the domain is<br />

2<br />

: + ~ n, wherenis any integer} .<br />

{+ ¢<br />

This can also be written as<br />

{xr ¢ (2n + 1) : ' where n is any integer}.<br />

The range remains (-00, 00).<br />

44. The domain of the function y =esc x is<br />

{xix ¢ nn, where n is any integer} ,<br />

and the range is (-00 , - 1]u [1, 00).<br />

For the function<br />

g(x) = - 2 csc(4x + rr)<br />

(d) d =4 tan 21r(1.2)<br />

=4 tan 2.41r<br />

'" 4(3.0774) =12.3 m<br />

(e) t =.25 leads to tan 1r, which is undefined.<br />

2<br />

48. a =4j sec 21rt I<br />

a. t = 0<br />

a = 41 sec 01 =4 m<br />

b. t = .86<br />

a = 41 sec 21r(.86) I= 6.3 m<br />

c. t =1.24<br />

a =41 sec 21r(1.24) 1= 63.7 m<br />

51. 3<br />

~t•.!2:. .; v,~_,<br />

=-2CSC4(X+ :)<br />

the period is 1r. Therefore, the domain is<br />

2<br />

{xr ¢ n;, where nis any integer}.<br />

The range becomes (-00, - 2] u [2, 00) since<br />

a=-2.<br />

45. The function tan x has a period of 1C, so it repeats<br />

four times over the interval (-21r, 21r). Since its<br />

range is (-00, 00), tan x =c has four solutions for<br />

every value of c.<br />

52.<br />

." • -t •<br />

X=.snS917i ' Y=l:3S&02:S~ .<br />

-3<br />

sin(1r) + sin(21r) =.!..+ $ = 1+ $<br />

6 6 2 2 2<br />

46. None; cos x ~ 1 for all x, so<br />

1<br />

--~ 1 and sec x ~ 1.<br />

cos x<br />

Since sec x ~ 1, secx has no values in the domain<br />

-1


<strong>Trigonometry</strong> <strong>Solutions</strong> <strong>Manual</strong><br />

Chapter 4 Review Exercises 137<br />

5n:<br />

55. y=-+nn:<br />

4<br />

56. approximately .3217505544<br />

57. .3217505544 + n « 3.463343208<br />

58. .3217505544 + nn:<br />

Chapter 4 Review Exercises<br />

1. B; the amplitude is 4 and period is 2n: = x<br />

2<br />

2. D; all other statements are true,<br />

3. The range for sin x and cos x is [-I, I]. The range<br />

for tan x and cot x is (-00, 00). Since .!. falls in<br />

2<br />

these intervals, those trigonometric functions can<br />

attain the value .!..<br />

2<br />

4. The range for esc x and sec x is<br />

(-00, -1]u[l, 00). The range for tan x and cot x<br />

is (-00, 00). Since 2 falls in these intervals, those<br />

trigonometric functions can attain the value 2.<br />

5. y = 2 sin x<br />

Amplitude: 2<br />

Period: 2n:<br />

Vertical translation: none<br />

Phase shift: none<br />

6. y= tan 3x<br />

Amplitude: not applicable<br />

Period: n:<br />

3<br />

Vertical translation: none<br />

Phase shift: none<br />

1<br />

7. y = - - cos 3x<br />

2<br />

Amplitude: .!.<br />

2<br />

P iod 2n: en : ­<br />

3<br />

Vertical translation: none<br />

Phase shift: none<br />

8. y= 2 sin 5x<br />

Amplitude: 2<br />

P iod 2n: en :­<br />

5<br />

Vertical translation: none<br />

Phase shift: none<br />

9. y = 1+ 2 sin.!. x<br />

4<br />

Amplitude: 2<br />

Period: 2~ = 8n:<br />

"4<br />

Vertical translation: up 1 unit<br />

Phase shift: none<br />

1 2<br />

10. y=3--cos-x<br />

4 3<br />

Amplitude: 1-±1= ±<br />

Period: 2; = 3n:<br />

Vertical translation: up 3 units<br />

Phase shift: none<br />

11. y=3CO{X+ ~)<br />

Amplitude: 3<br />

Period: 2n:<br />

Vertical translation: none<br />

Phase shift: n: units to the left<br />

2<br />

12. ' 3n:)<br />

y=-sm( x-4'"<br />

Amplitude: I<br />

Period:2n:<br />

Vertical translation: none<br />

Phase shift: 3n: units to the right<br />

4<br />

13. y=t cSc(2X- :)<br />

Amplitude: not applicable<br />

P en iod : -=n: 2n:<br />

2<br />

Vertical translation: none<br />

Phase shift: 1r units to the right<br />

8<br />

14. y = 2 sec (n:x - 2n:)<br />

Amplitude: not applicable<br />

Period: 2n: = 2<br />

n:<br />

Vertical translation: none<br />

Phase shift: 2n: = 2 · the rizh<br />

- urnts to e ng t<br />

n:<br />

137


<strong>Trigonometry</strong> <strong>Solutions</strong> <strong>Manual</strong><br />

138 Chapter 4 Graphs of the Circular Functions<br />

15.<br />

I<br />

y =~ tan(3x - ~ )<br />

26. y=-secx<br />

2<br />

y =~ tan,x - : )<br />

Period: 2n<br />

n 3n<br />

Amplitude: not applicable<br />

2 2<br />

Period : n<br />

3<br />

Vertical translation: none<br />

Phase shift: !!.. units to the right<br />

9<br />

X<br />

(<br />

3Jr)<br />

16. y=cot '2+4"<br />

Amplitude: not applicable<br />

Asymptotes: x =-, x=­<br />

y<br />

-+--+.:--+--+---+---;.- x<br />

Period: t=2n<br />

27. y =-tanx<br />

r<br />

Period:n<br />

Vertical translation: none<br />

Jr Jr<br />

Phase shift: = 3; units to the left 2 2<br />

Asymptotes: x =--, x=­<br />

y<br />

17. The tangent function has a period of nand<br />

x-intercepts at integral multiples of Jr.<br />

18. The sine function has a period of 2n and passes<br />

through the origin.<br />

-+--+-~-+--.-_ x<br />

19. The cosine function has a period of 2Jr and has the<br />

n<br />

value 0 when x =-.<br />

2 28. y = -2 cos x<br />

Amplitude: 2<br />

20. The cosecant function has a period of 2n and is<br />

Period: 2n<br />

not defined at integral multiples of Jr.<br />

y<br />

y= -tan X<br />

21. The cotangent function has a period of nand y=-2cosx<br />

decreases on the interval 0 < x < n.<br />

2<br />

22. The secant function has a period of 2Jr and --=+--+--il-+--t--x<br />

vertical asymptotes at odd multiples of !!.., that is,<br />

2<br />

x =-n + nit, were<br />

at here n is IS an imteger.<br />

2<br />

-2<br />

25. y =3 sin x<br />

Period: 2Jr<br />

Amplitude: 3<br />

y<br />

y= 3 sinx<br />

3<br />

-f;:--t--+--+----i.....-.- x<br />

2Jr<br />

- 3<br />

138


<strong>Trigonometry</strong> <strong>Solutions</strong> <strong>Manual</strong><br />

Chapter 4 Review Exercises 139<br />

29. y= 2+ cotx 32. y = tan 3x<br />

Period: n<br />

Vertical translation: up 2 units<br />

Period: ~<br />

3<br />

Asymptotes: x = 0, x = 1C<br />

y Asymptotes: x =- -, x = ­<br />

6 6<br />

y<br />

. 1C<br />

1C<br />

----1-:+-+---. x<br />

y=2+cotx<br />

30. y=-l+cscx 33. y=3cos2x<br />

Period: 2n Amplitude: 3<br />

Vertical translation: down 1 unit<br />

31.<br />

iod 21C<br />

y 2<br />

y<br />

I<br />

2 . I<br />

Asymptotes: x = 0, x = n:, x = 2n<br />

3K I<br />

"I T 2,. 2<br />

~....30+o"E'-+-+--j-. x<br />

- __.... 4 1<br />

y= sin 2x<br />

nl<br />

I<br />

I I<br />

I I<br />

y=-I +cscx<br />

P en iod : -=1C 2n<br />

2<br />

Amplitude: 1<br />

y<br />

34.<br />

Pen : -=1C<br />

3<br />

--:+-Hr--f-t--. x<br />

y=3cos2x<br />

1<br />

y =-cot3x<br />

2<br />

Period: ~<br />

Asymptotes: x = 0,<br />

y<br />

1C<br />

x =­<br />

3<br />

y=sin 2x<br />

---+--~--+---_ x<br />

-1<br />

y=!cot3x<br />

139


<strong>Trigonometry</strong> <strong>Solutions</strong> <strong>Manual</strong><br />

140 Chapter 4 Graphs of the Circular Functions<br />

35.<br />

Y=CO{X- :) 37. y =sec(2x+ ;)<br />

=See[2(X+ :)J<br />

Amplitude: 1<br />

Period: 21C<br />

Phase shift: 1C units to the right .00 21C<br />

4 Pen : -=1C<br />

y 2<br />

Phase shift: 1C units to the left<br />

6<br />

y<br />

'V'<br />

--f-+---"'k--+--#--+- x<br />

I<br />

I I I<br />

I I I<br />

36. y = tan(x - ~ )<br />

1C 71C J31C<br />

12 12 12<br />

---::++----1f-+--+-.... x<br />

o I .s I ¥<br />

-1 if\3:<br />

I<br />

I<br />

Y =sec (2x+ n<br />

Period: 1C<br />

Asymptotes: x =0, x = 1C 38. Y =Si{3x + ~)<br />

Phase shift: !E. units to the right<br />

2<br />

y<br />

=sin[{x+ :)J<br />

Amplitude: 1<br />

.00 21C<br />

Pen :­<br />

3<br />

Phase shift: 1C units to the left<br />

6<br />

y<br />

y =sin (3x+f)<br />

-+-~"""",....--+-_x<br />

39. Y =1 + 2 cos 3x<br />

Amplitude: 2<br />

.00 21C<br />

Pen :­<br />

3<br />

Vertical translation: 1 unit up<br />

y<br />

3<br />

2<br />

--:-H~H----x<br />

y=I+2cosh<br />

140


<strong>Trigonometry</strong> <strong>Solutions</strong> <strong>Manual</strong><br />

40.<br />

41.<br />

y =-1 - 3 sin 2x<br />

Amplitude: 3<br />

iod : -=1C<br />

21C<br />

Pen<br />

2<br />

Vertical translation : down 1 unit<br />

y<br />

y = - 1- 3 sin 2x<br />

2<br />

---::-t-+-lt-Ht---- x<br />

-4<br />

y= 2 sin 1CX<br />

Amplitude: 2<br />

Period: 2<br />

y<br />

2<br />

y = 2 sin xx<br />

--.--+- .--__ x<br />

o<br />

2<br />

-I<br />

-2<br />

Chapter 4 Review Exercises 141<br />

(b) The maximum average monthly temperature<br />

is 75°F, and the minimum is 25°F. Let the<br />

amplitude a be 75 - 25 =25. Since the<br />

2<br />

period is 12, let b = 1C. The data are<br />

6<br />

centered vertically around the line<br />

75+25<br />

y = =50, so let c =50.<br />

2<br />

The minimum temperature occurs in January.<br />

Thus, when x =I, b(x - d) =- 1C since the<br />

2<br />

sine function is minimum at _ 1C • Solving<br />

2<br />

ford,<br />

1C<br />

1C<br />

-(l-d)=-­<br />

6 2<br />

d=4.<br />

d can be adjusted slightly to give a better<br />

visual fit. Try d =4.2.<br />

Thus, !(x) =a sin b(x - d) + C<br />

(d) Plotting the data with<br />

=25sin[: (x -4.2)]+ 50.<br />

42.<br />

1<br />

Y =--COS(1C x -1C)<br />

2<br />

Amplitude: .!..<br />

2<br />

Period: 21C =2<br />

1C<br />

Phase shift: 1C =1 unit to the right<br />

1C<br />

y<br />

!(x) =25sin[: (x - 4.2)] + 50<br />

on the same coordinate axes ive a good fit.<br />

f(x) = 25 sin [-ifx - 42)] + 50<br />

80<br />

y =-! cos (1« -tr)<br />

(e)<br />

45.<br />

(a) Let January correspond to x = 1, February to<br />

x =2, ... , and December ofthe second year<br />

to x = 24. The data appear to follow the<br />

pattern of a translated sine graph.<br />

10<br />

.. - ~-• . ~:;:. :'.. . ,!,.<br />

.... ..<br />

-- "... • ~ :• .~ .. -;~ . :. a<br />

:. ~<br />

. .­ ..<br />

,.: . ..--<br />

•<br />

1• ~...;.._.;..........;;;.......;.._;.,;,25<br />

"<br />

20<br />

From the sine regression we get<br />

y =25.77sin(.52x-2.19)+ 50.57<br />

=25.77sin[.52(x - 4.21)] + 50.57.<br />

141


<strong>Trigonometry</strong> <strong>Solutions</strong> <strong>Manual</strong><br />

142 Chapter 4 Graphs of the Circular Functions<br />

46. The shorter leg of the right triangle has length<br />

b: -hI '<br />

(b) When h 2 ::::;:; 55 and hI::::;:; 5.<br />

d = (55 -5)cot8 = 50cot8.<br />

The period is n, but the graph wanted is d for<br />

0


<strong>Trigonometry</strong> <strong>Solutions</strong> <strong>Manual</strong><br />

7. y = 3 csc itx<br />

4. y=tan(X- ~) Period: 2<br />

Period:rr<br />

Amplitude : not applicable<br />

Asymptotes : x = -x, x = 0, x = rr<br />

Vertical translation: none<br />

Phase shift: rr units to the right<br />

2 I I<br />

I I<br />

y Y=IaD(x-~) I<br />

I<br />

I<br />

I<br />

I<br />

I<br />

x =- n<br />

5. y =-co{~x)<br />

Chapter 4 Test Exercises 143<br />

Amplitude: not applicable<br />

Asymptotes: x = -2, x = -I, x = 0, X= I, x = 2<br />

Vertical translation: none<br />

Phase shift: none<br />

'V'<br />

n<br />

I<br />

I<br />

I<br />

8. (a)<br />

I f(x) = 175 sin[~(x - 4) ] + 67.5<br />

x=n<br />

90<br />

Period: 2rr<br />

Amplitude: not applicable<br />

Asymptotes: x = -2rr, x = 0, x = 2rr<br />

Vertical translation: none<br />

. Phase shift: none<br />

y _ 1<br />

y--COli (b) Amplitude: 17.5<br />

I iod 2rr 6<br />

I<br />

I<br />

I<br />

y<br />

Pen : -=2rr·- = 12<br />

f rr<br />

Phase shift: 4 units tothe right<br />

Vertical translation: 67.5 units up<br />

(c) Approximately 52°F<br />

x=o<br />

x= 2lr<br />

(d) 50°F January; 85°F in July<br />

(e) Approximately 67.5°; this is the vertical<br />

translation.<br />

x =¥<br />

U<br />

I<br />

(<br />

I<br />

I<br />

I y=-sccx<br />

I<br />

I<br />

10. C; the graph shows this is an odd function, so<br />

either A or C, Since sin x> 0 from 0 < x < !!...<br />

2 '<br />

the function in the graph must bey =-3 sin x,<br />

143


<strong>Trigonometry</strong> <strong>Solutions</strong> <strong>Manual</strong><br />

144 Chapter 5 Trigonometric Identities<br />

CHAPTER 5 TRIGONOMETRIC IDENTITIES<br />

Section 5.1<br />

Exercises<br />

1. By a negative-angle identity, tan (-x) =-tan x.<br />

Since tan x = 2.6, tan (-x) =-tan x = -2.6.<br />

2. By a negative angle identity, cos (-x) = cos x.<br />

Since cos x = -.65, cos (-x) =cos x =-.65.<br />

3• By a reciproca . l I identi entity, cot x =--. 1<br />

tan x<br />

Since tan x = 1.6, cot x = _1_=.625.<br />

1.6<br />

4• By a quotient I identi entity, tan x =--. sinx<br />

cosx<br />

Since cos x =.8 and sinx= .6,<br />

sinx .6<br />

tanx=--=-=.75.<br />

cosx .8<br />

By a negative angle identity,<br />

tan(-x)=-tanx, so tan(-x) =-.75.<br />

3.. dr I<br />

5· cos s = '4 ' S IS 10 qua ant .<br />

sin 2 s+cos 2 s = 1<br />

2<br />

sin s + (~r = 1<br />

sin 2 s =1-i.=.2­<br />

16 16<br />

sins =± .fi<br />

4<br />

S· mce s IS<br />

.<br />

10<br />

. qua drant, I' sms=-. .fi<br />

4<br />

6. cots = _.!., s in quadrant IV<br />

3<br />

1+ cot 2 s = csc 2 s<br />

1+(-~r =csc 2s<br />

10 = csc 2 s<br />

9<br />

Since s is in quadrant IV, esc s < 0, so<br />

cscs=-N=- ~.<br />

7. coss = S' ../5 tans < o :<br />

sin 2 s+cos 2 s = 1<br />

sin' s+( ~r =1<br />

sin 2 s = 1_ 2. = 20<br />

25 25<br />

4<br />

=­ 5<br />

. 2<br />

SIOS=±../5<br />

Since s is in quadrant IV,<br />

. 2 2../5<br />

SIOS=- ../5=--5-.<br />

.fi<br />

8. tans =-- sees> 0<br />

2 '<br />

tan 2 s+l=see 2 s<br />

(- ~r +1=see<br />

Imp I' res s .. IS 10 qua dr ant IV .<br />

2s<br />

2<br />

!!= sec s<br />

4<br />

.J11<br />

--=secs<br />

2<br />

2<br />

--r:-:" =cos s<br />

...,11 .<br />

sin 2 s + cos 2 s = 1<br />

2s+(lY<br />

sin =1<br />

. 2 7<br />

SIO<br />

s=­<br />

11<br />

Since tan s < 0 and sec s > 0, s is in quadrant IV,<br />

so sin s


<strong>Trigonometry</strong> <strong>Solutions</strong> <strong>Manual</strong><br />

5.1 Fundamentalldentities 145<br />

14. This is the graph off(x) = cotx.<br />

9• sees =-, 11 tans < O unpI' ' res S IS " ID quadrant IV.<br />

4<br />

Since f(-x) = cot(-x)<br />

. 1<br />

cos(-x)<br />

SIDee secs=-­<br />

=<br />

coss<br />

sin(-x)<br />

11 1 cosx<br />

-=-- =-­<br />

4 coss -sinx<br />

4 =-cotx<br />

-=coss.<br />

11 =-f(x),<br />

sin 2 s+cos 2 s = 1<br />

f(-x) =-j(x).<br />

2<br />

sin s +(.~r = 1 15. sin 0 = ~. 0 in quadrant II<br />

S·<br />

3<br />

. 2<br />

SID<br />

16 1<br />

s+-= sin 2 0 +cos 2 0 = 1<br />

121<br />

. 105 cos 2 0 = I-sin 2 0<br />

SIDS=­<br />

12<br />

. .;/105<br />

=I-(%r =i<br />

SIDS=±-­<br />

11<br />

cosO=± ..J5<br />

.. ad IV' , SID ..}105 3<br />

S<br />

11<br />

IDee S IS ID qu rant =---.<br />

Since 0 is in quadrant II, cosO = _ "f5 .<br />

3<br />

10. cscs =-- 8 sinO t 2 2..J5<br />

5 tanO=--=--=--=--­<br />

. 1 cosO _:/l ..J5 5<br />

SIDee CSCs=--,<br />

sins 1 1 "f5<br />

8 1 cotO=--=--=-­<br />

--=-- tan 0 2<br />

5 sins<br />

3<br />

-i<br />

-8sins =5 .. 1 1 3 3..J5<br />

secO=--=--=--=--­<br />

. 5<br />

SinS =--.<br />

cosO -4 ...J5 5<br />

8<br />

1 1 3<br />

cscO=--=-=­<br />

12. This is the graph of f(x) = sec x. sinO t 2<br />

Since f(-x) = sec(-x)<br />

1<br />

=--­<br />

16. cosO =.!., 0 in quadrant I<br />

5<br />

cos(-x)<br />

1 sin 2 0 + cos 2 0 = 1<br />

=-­<br />

cos(x) sin 2 0 = l-cos 2 0<br />

= sec(x)<br />

=f(x),<br />

=1-(~r<br />

f(-x) =f(x). 24<br />

13. This is the graph off(x) = esc x.<br />

Since f(-x) =csc(-x)<br />

1<br />

=--­<br />

sin(-x)<br />

1<br />

f(-x) =-j(x).<br />

= -sin(x) /)<br />

=-~ - CSGc '"<br />

=-f(x),<br />

=­<br />

25<br />

145


<strong>Trigonometry</strong> <strong>Solutions</strong> <strong>Manual</strong><br />

146 Chapter 5 Trigonometric Identities<br />

17.<br />

Since (J is in quadrant I and sin (J > 0,<br />

sin (J= f24 = 2../6 "<br />

V"25 5<br />

. (J 2-/6<br />

SID -<br />

t<br />

tan (J = -­= _5_ = 2../6<br />

cos(J<br />

1 1 ../6<br />

cot(J =-­=-­=tan(J<br />

2../6 12<br />

1<br />

sec(J=--=5<br />

cos(J<br />

1 5 5../6<br />

csc(J = sin(J = 2../6 = U<br />

tan (J= _.!., (J in quadrant IV<br />

4<br />

sec 2 (J= 1+ tan 2 (J<br />

=1+(-if<br />

17<br />

= 16<br />

S· (J.. dr IV (J..ffj<br />

mce IS ID qua ant , sec = 4 "<br />

1<br />

cos(J=-­<br />

sec(J<br />

1 4 4~<br />

= .ffi =..ffj =17<br />

4<br />

sin (J = tan (J"cos(J<br />

1 4 1 ~<br />

=-4"~ =­ .,Jfi =-17<br />

csc(J =_1_ =_1_ =_~<br />

sin(J --i7<br />

1 1<br />

cot(J=--=-=-4<br />

tan(J -i<br />

18. csc(J=-~, (J in quadrant m<br />

. 2<br />

sin (J= _1_ = _ 3­<br />

csc(J 5<br />

cos 2 (J= 1-sin 2 (J<br />

=1-(-~r<br />

21<br />

=­<br />

Since (J is in quadrant III, cos(J < 0, so<br />

..fij<br />

cos(J =---.<br />

5<br />

tan (J = sin (J = --=.L = _2_ = 2..fij<br />

cos(J -41 ..fij 21<br />

1 ..fij<br />

cot(J=--=-­<br />

tan(J 2<br />

1 5 5..fij<br />

sec(J = -­=--­=--­<br />

cos (J ..fij 21<br />

19. cot(J = ±, sin(J > 0<br />

3<br />

Since cot(J > 0 and sin (J > 0, (J is in quadrant I<br />

and all the functions are positive.<br />

1 1 3<br />

tan=--=-=cot(J<br />

t 4<br />

(<br />

3)2 25<br />

sec 2 (J= 1+ tan 2 (J= 1+ - = ­<br />

4 16<br />

sec(J =~<br />

4<br />

1 1 4<br />

cos(J = -­= ­ = ­<br />

sec(J i 5<br />

sin2(J=I-cos2(J=I-(~r = ~<br />

. (J 3<br />

SID =­ 5<br />

1 5<br />

csc(J=--=sin(J<br />

3<br />

3<br />

cos(J =-­ 5<br />

25<br />

20. sin(J = _±, cos(J < 0<br />

5 .<br />

Since sin (J< 0 and cos(J < 0, (J is in quadrant ill.<br />

2 " 2 (4)2 9<br />

cos (J=I-sID (J=I- -'5 = 25<br />

sin(J 4<br />

tan(J =--=­<br />

. cos(J 3<br />

1 3<br />

cot(J=--=tan(J<br />

4<br />

1 5<br />

sec(J=--=-­<br />

cos(J 3<br />

1 5<br />

csc(J =-­=-­<br />

sin (J 4<br />

146


<strong>Trigonometry</strong> <strong>Solutions</strong> <strong>Manual</strong><br />

5.1 Fundamental Identities 147<br />

21. sec 0 = i, sinO < 0<br />

3<br />

Since sec0 > 0 and sin 0 < O. 0 is in<br />

quadrant IV. .<br />

1 1 3<br />

cosO=--=-=sec0<br />

t 4<br />

· 2 2 (3)2 7<br />

SID O=I-cos 0=1-"4 =16<br />

S· mce SID . 0 < 0 , SID . 0 =--. ../7<br />

4<br />

sinO · -:if ../7<br />

tanO=--=--=-­<br />

cosO i 3<br />

4<br />

1 1 3../7<br />

cot 0 =--=--=--­<br />

tanO 7<br />

1 1 4../7<br />

-:if<br />

cscO=--=--=--­<br />

sinO<br />

7<br />

22. cosO = _.!.. sinO> 0<br />

4<br />

Since cos0 < 0 and sin 0 > 0, 0 is in quadrant n.<br />

· 2 2 (1)2 15<br />

SID O=I-cos 0=1- -"4 =16<br />

sinO= "ff5<br />

4<br />

sinO ~<br />

tanO=--=-",15<br />

· cosO .<br />

1 .J15<br />

cotO=--=--­<br />

tan 0 15<br />

secO=_I_ =-4<br />

cosO<br />

1 4.J}5<br />

cscO=--.=-­<br />

sinO 15<br />

cos x<br />

23. --=cotx<br />

sin x<br />

Choose expression B.<br />

sinx<br />

24. tanx=-­<br />

cos x<br />

Choose expression D.<br />

25. cos(-x)=cosx<br />

Choose expression E.<br />

26. tan 2 x+l=sec 2 x<br />

Choose expression C.<br />

27. 1=sin 2 x+cos 2 .x<br />

Choose expression A.<br />

sinx . . ( )<br />

28. -tanxcosx =--_·cosx =-SIDX=SID-X<br />

cosx<br />

Choose expression C.<br />

2 2 sin 2 x<br />

29. sec x-I = tan x =--2­<br />

cos x<br />

Choose expression A.<br />

30.<br />

.....L .<br />

sec x cosx sm r<br />

csc x = sJ x = cos x = tan x<br />

Choose expression E.<br />

2<br />

31. l+sin 2 x=csc x-cot 2 x+sin 2 x<br />

Choose expression D.<br />

2 1<br />

32. cos x=--2­<br />

sec x<br />

Choose expression B.<br />

35. Find sin 0 if cos 0 = _x_.<br />

(x+l)<br />

. x<br />

oosO=-­<br />

x+l<br />

Since sin 20+cos20=1 ,<br />

sin 2 0 = 1-cos 2 0<br />

sin2 0 = 1-(_x_)2<br />

x+l<br />

(x+ 1)2_x 2<br />

=-'---'--..,....-­<br />

(x+ 1)2<br />

x 2 +2x+l-x 2<br />

=<br />

(x+ 1)2<br />

2x+l<br />

- (x+ 1)2<br />

. 0 ±..,J2x+l<br />

SID = .<br />

x+l<br />

147


<strong>Trigonometry</strong> <strong>Solutions</strong> <strong>Manual</strong><br />

148 Chapter 5 Trigonometric Identities<br />

43. (1- cos 0)(1+ sec 0)<br />

36. Find tana if sec a = p + 4 .<br />

p<br />

=1+secO-cosO-cosOsecO<br />

tan 2 a + 1 = sec 2 a<br />

= 1+ sec 0 - cos 0 - cos 0(_1_)<br />

cosO<br />

( +4)2<br />

tan 2 a+ 1= P 2<br />

=1+secO-cosO-l<br />

P<br />

=secO-cosO<br />

2 (p+4)2<br />

tana= 2 1 cosO+sinO cosO sinO .0 1<br />

p 44. =--+--=cotu+<br />

sinO sinO sin O<br />

2 p2+ Sp+ 16 p2<br />

tan a = p2 - p2<br />

cos 2 0 -sin 2 0<br />

45.<br />

2 Sp+ 16<br />

tan a=<br />

sinOcosO<br />

p2 20<br />

cos sin 2 0<br />

2 4(2p+4) =<br />

tan a = 2<br />

sinOcosO sinOcosO<br />

P cosO sinO<br />

=----­<br />

±2.J2P+4<br />

sinO cosO<br />

tan a =<br />

= cot 0 -tanO<br />

p<br />

l-sin 2 0 cos 20<br />

.0 • .0 cosO . .0 .0<br />

37. cot uSIO u =--·SlOu = cOSu 46. 1+ cot 2 0 = csc2 0<br />

sinO 20<br />

cos<br />

. 1 cosO sinO<br />

=-.­<br />

38. sec 0 cot 0 SIO 0 =-_.-_._­<br />

sio<br />

cosO 2 8<br />

sinO 1<br />

sinO cosO<br />

=sin 2 Ocos 2 0<br />

= cosOsinO sinO cosO<br />

=1 47. tan 0 + cot 0 =--+-­<br />

.. cosO sinO<br />

1 sin 2 0.+cos 2 0<br />

39. cosOcscO =cosO·_­ =<br />

sinO<br />

cosO sinO<br />

cosO 1<br />

=-­ =---­<br />

sinO<br />

cosOsinO<br />

= cot 0<br />

=-_._­<br />

1 1<br />

cosO sinO ­<br />

20<br />

2 2 cos 2<br />

40. cot 0(1+ tan 0) = --2-(sec 0)<br />

= secOcscO<br />

sin 0<br />

20( 48. (secO+ cscO)(cosO- sin 0)<br />

cos 1 )<br />

= sin 2 0 cos 2 0 = (_1_ + _._1_)(cos 0 - sin 0)<br />

1<br />

cosO SIOO<br />

= sin 2 0 = -1-(cosO) - -1-(sinO)<br />

2 cosO cosO<br />

= csc 0<br />

+ -._I_(cosO) __._I _ (sin 0)<br />

SIOO SIOO<br />

2 2<br />

41. sin 2 0(csc 0-1)= sin 0(+-1) = 1- sin 0 + cos 0 _ 1<br />

SIO 0<br />

cosO sinO<br />

• 2.0<br />

SIO u . 2.0 = -tanO+cotO<br />

=---SIO u<br />

sin 20 :;::; cotO - tan 0<br />

=1-sin 20<br />

20 49. sinO(csc 0 - sinO) = sinOcscO -sin 2 0<br />

=cos<br />

= l-sin 2 0<br />

42. (sec 0 -1)(sec 0 + 1) = sec 2 0 -1 = cos<br />

2 0<br />

= tan 2 0<br />

148


0<br />

<strong>Trigonometry</strong> <strong>Solutions</strong> <strong>Manual</strong><br />

5.1 Fundamental Identities<br />

149<br />

1+ tan 2 0 see 2 0<br />

50. l+cot20 = CSC2 0<br />

51.<br />

1<br />

0 - ­<br />

29<br />

coo<br />

=-1­<br />

sio 2 9<br />

I sin 20<br />

=COS 2 0 ' - 1­<br />

sin 20<br />

= COS 2 0<br />

=~~.J1 0<br />

sin 20+tan20+cos20<br />

= (sin 2 0 + cos 2 0) + tan 2 0<br />

=1+tan 20<br />

=see 2 0<br />

tan(-0)<br />

- tan0<br />

52. =-1­<br />

see 0 oos9<br />

sinO cosO<br />

=--_._­<br />

cosO<br />

=-sinO<br />

53. Express sin 0 in terms of cot 0 and in terms of<br />

see O.<br />

0<br />

I 1<br />

sinO = --= -;=======<br />

cscO rJI + cot 2 0<br />

_rJI+cot 20<br />

- l+cot 20<br />

sinO = cos 0 tan 0<br />

=_I_.±~see2 0-1<br />

see 0<br />

±.J'-sec--:2=-0---1<br />

=<br />

see 0<br />

54. Express cos 0 in terms of sin O. cot O. and<br />

csc O.<br />

sin 2 0+cos 2 0 =I<br />

cosO =±~I-sin2 0<br />

. cosO<br />

Since ootO=--,<br />

sinO<br />

cosO = sin 0 cot 0<br />

I<br />

=--cotO<br />

cscO<br />

I<br />

= cot 0<br />

±.JI + cot 2 0<br />

rl-+-c-o-t<br />

± cot 0.J 2:--0<br />

=<br />

l+cot 20<br />

cosO = ± It - _1 2<br />

­<br />

" csc 9<br />

20-1<br />

=± csc<br />

20<br />

csc<br />

±~csc2 0-1<br />

=<br />

cscO<br />

55. Express tan 0 in terms of sin O.cos O. sec<br />

esc 0,<br />

sinO<br />

tan O=-­<br />

cosO<br />

sinO<br />

=~;===~<br />

±~I-sin2 0<br />

±sinO<br />

- ~1-sin20<br />

±sin 0~""'I---s-in""'2""'0-<br />

=<br />

l-sin 20<br />

sinO<br />

tan O=-­<br />

cosO<br />

±.Jr- -0<br />

I-_-co-s-=-2<br />

=<br />

cosO<br />

tan 0 =±~see2 0-1<br />

I<br />

tanO=-­<br />

cot 0<br />

I<br />

=--;===<br />

±~csc2 0-1<br />

_ ±~csc2 0-1<br />

- csc 20-1<br />

O. and<br />

149


<strong>Trigonometry</strong> <strong>Solutions</strong> <strong>Manual</strong><br />

150 Chapter 5 Trigonometric Identities<br />

56. Express cot 0 in terms of sin O. cos O. and 58. Express esc 0 in terms of cos 0, tan 0, cot O.<br />

esc O. and sec O.<br />

1<br />

cotO=±~cscZ 0 -1<br />

cscO=-­<br />

sinO<br />

=±~~-1<br />

1<br />

=---;==:==<br />

±~I-sinZO<br />

±~I-cosZ 0<br />

=<br />

sinO _±..Jl-cosZO<br />

- l-cosZO<br />

±~I-sinZ 0<br />

cot 0 =---'---­<br />

sinO<br />

±cosO<br />

=--;====<br />

=<br />

esc 0 = ±~1 + cot Z 0<br />

~1-cosZO<br />

=±~1+ l z<br />

tan 0<br />

± cos O~""'I---c-o""sZ:-O- ±~I+tanZO<br />

l-cos Z 0<br />

=<br />

lanO<br />

cotO=±~cscZ 0-1 cscO= ±~1 + cot Z 0<br />

±~I+tanZO<br />

57. Express sec 0 in terms of sin 0, tan O. cot O.<br />

and esc O.<br />

1<br />

secO=-­<br />

cosO<br />

1<br />

=---;==~<br />

esc O = ---'---­<br />

tan 0<br />

±secO<br />

= ~secZO-l<br />

± sec o-J,....-sec----:Z:--O---l<br />

=<br />

sec Z 0-1<br />

±~I-sinZO<br />

±~I-sinZO<br />

= l-sin Z 0 59. Let cos x = ~. Then x is in quadrant I or quadrant<br />

sec 0 = ±..Jtan Z 0+ 1<br />

Since tan 0 = _1_.<br />

cotO<br />

sec 0 = ±~rta-n-=-Z-0-+-1<br />

=± (_l z )+1<br />

cot 0<br />

±~I+cotZ 0<br />

=<br />

cotO<br />

1<br />

secO=-­<br />

cosO<br />

1<br />

=-p=====<br />

.s:»:<br />

1<br />

=---;===<br />

+~1- cs:2 8<br />

~cscZO<br />

=---;=;:====<br />

±~cscZ 0-1<br />

± esc O~""'c-sc-Z-O---l<br />

=<br />

csc Z 0-1<br />

IV. Then.<br />

sin x = ±~""'i---c-o-sZ-x-.<br />

=±FG)<br />

= ± f24 = ±2J6 .<br />

'/"25 5<br />

. + z../6<br />

smx --5­ rz:<br />

tanx=--=-t-=±2",6 .<br />

cosx S<br />

1<br />

secx=--=5<br />

cosx<br />

150


<strong>Trigonometry</strong> <strong>Solutions</strong> <strong>Manual</strong><br />

5.1 Fundamental Identities 151<br />

Quadrant I:<br />

sinx<br />

Quadrant IV:<br />

secx-tanx 5-(-2..,[6)<br />

=<br />

---r<br />

25+10../6<br />

= -2..,[6<br />

-25..,[6 -60<br />

=<br />

12<br />

sin x 2../6<br />

60. Let esc x = -3. Then x is in quadrant ill or<br />

quadrant IV.<br />

Quadrant ill:<br />

. 1 1<br />

SIDX=--=-­<br />

cscx 3<br />

cos x = -~1- sin 2 x<br />

=-FG)<br />

=_~= __2~_2<br />

1 3<br />

secx =cosx =-"M<br />

. l 2J2<br />

smx+cosx -~-3<br />

sec x =-~<br />

=(-1-2../2X_2~)<br />

3<br />

2../2 +8<br />

=<br />

9<br />

Quadrant IV:<br />

. 1<br />

SIDX =-­<br />

3<br />

cos x = ~1-sin2 x<br />

= f! = 2../2<br />

V9 3<br />

3<br />

secx=-­<br />

2../2<br />

sinx+cosx _ -i+¥<br />

secx - ~<br />

61. y =sin(-2x)<br />

y = -sin(2x)<br />

2<br />

=(-1 +3 ../2X2~)<br />

-2../2 +8<br />

=<br />

9<br />

62. It is the negative of sin(2x).<br />

63. y = cos (-4x)<br />

y= cos (4x)<br />

64. It is the same function.<br />

65. (a) y = sin (-4x)<br />

y=-sin (4x)<br />

(b) y = cos(-2x)<br />

y= cos(2x)<br />

(c) y = -5 sin(-3x)<br />

y = -5[-5in(3x)]<br />

y = 5 sin(3x)<br />

1 1<br />

66. y=sec(-x)= =--=secx<br />

cos(-x) cos x<br />

Since sec(-x) = sec x, the graph of y = sec(-x) is<br />

exactly the same as the graph of y = secx.<br />

1 1<br />

67. y=csc(-x)= . =-.-=-cscx<br />

sm(-x) -smx<br />

Since csc(-x) = -esc x, the graph of y = csc(-x) is<br />

a reflection across the x-axis of the graph of<br />

y = cscx.<br />

sin(-x) -sinx<br />

68. y = tan(-x) = =--=-tan x<br />

cos(-x) cosx<br />

Since tan(-x) = -tan x, the graph of y = tan(-x) is<br />

a reflection across the x-axis of the graph of<br />

y = tan x.<br />

151


<strong>Trigonometry</strong> <strong>Solutions</strong> <strong>Manual</strong><br />

152<br />

Chapter 5 Trigonometric Identities<br />

69.<br />

70.<br />

cos(-x) cosx<br />

y=cot(-x)= . =-.-=-cotx<br />

sm(-x) -smx<br />

Since cot(-x) = -cot x, the graph of y = cot(-x) is<br />

a reflection across the x-axis of the graph of<br />

y =cot x,<br />

74. Does cos(x - y) = cos x - cosy<br />

If it does, then the graphs of Y\ = cos(x - y) and<br />

Yz =cos x - cos y for specific values of y will<br />

overlap. We will graph 3 cases: y = 1r, Y = 1r •<br />

4 2<br />

and y= n.<br />

71.<br />

identity<br />

-4<br />

72.<br />

not an identity<br />

not an identity<br />

73.<br />

not an identity<br />

YJ=cos2x<br />

Y 2 = cos 2x -<br />

4<br />

sin 2x<br />

75. Does sin(x + y) = sin x + sin y If it does. then the<br />

graphs of Y J =sin(x+y) and Y2 =sinx+siny<br />

for specific values of y will overlap. We will<br />

1r<br />

graph 3 cases: y = -. y = -. and y =1r .<br />

4 2<br />

,....---.....,....---......<br />

1r<br />

-4<br />

-4<br />

identity<br />

152


<strong>Trigonometry</strong> <strong>Solutions</strong> <strong>Manual</strong><br />

5.2 Verifying Trigonometric Identities 153<br />

4. cos p(sec fJ + esc 13) = cos J_l_ + _._1_)<br />

I-'l cos fJ smfJ<br />

=l+cotp<br />

5. _1_ 2-+_1-2-=sin<br />

20+cos20=1<br />

esc 0 sec 0<br />

-4<br />

6.<br />

1 ___ = (sina+ 1)-(sina -1)<br />

sina-l sina+l (sina-l)(sina+l)<br />

2<br />

- sin 2a-l<br />

= 2_ or -2sec2 a<br />

cos<br />

2a<br />

Section 5.2<br />

-4<br />

not an identity<br />

Connections (pageJ99)<br />

Exercises<br />

~(I_X2)3 =~(l-(cosO)2)3<br />

=~(sin28)3<br />

= ~sin6 0<br />

= sin 3 0<br />

cos 0 is an appropriate choice since<br />

cos 2 0 + sin 2 0 = 1.<br />

1 cosO sinO<br />

1. co t [7+--=--+-­<br />

Ll<br />

cot 0 sin 0 cos0<br />

cos 2 0 + sin 2 0<br />

= sin8cosO<br />

1·<br />

= or esc 0 sec 0<br />

sin OcosO<br />

secx cscx sinx cos x<br />

2. --+--=--+-­<br />

cscx secx cos x sinx<br />

sin 2 x+cos 2 x<br />

= sinxcosx<br />

1<br />

= or cscxsecs<br />

sinxcosx<br />

3. tan s(cots+cscs) =--<br />

sins (coss<br />

-.-+-.­<br />

1)<br />

coss sins sms<br />

1<br />

=1+-­<br />

coss<br />

=1+secs<br />

cos x sinx .<br />

7. --+--=cosx(cosx)+smx(sinx)<br />

sec x cscx<br />

= cos 2 x +sin 2 x<br />

=1<br />

cosr +<br />

2 . 2<br />

sinr = cosr + cos r+sm r<br />

8. sin r 1+ cos r (sin r)(1 + cos r)<br />

cosr+1<br />

=----''---­<br />

(sin rXI + cos r)<br />

1<br />

=-.- or csc y<br />

sm j­<br />

9. (l+sint)2 +cos 2 t = 1+2sint+sin 2 t+cos 2 t<br />

=2+2sint<br />

10. (1+ tans)2 -2tans = 1+2 tans+ tan 2 s-2 tans<br />

11.<br />

l+cosx<br />

1<br />

= l+tan 2 s<br />

2<br />

=sec s<br />

12. (sina -cosa)2 = sin 2 a-2sinacosa+cos 2 a<br />

= 1-2sinacosa<br />

13. sin 2 r -1 = (sin r -1)(sin r + 1)<br />

14. sec 2 0 -1 = (sec 0 -1)(sec 0 + 1)<br />

153


<strong>Trigonometry</strong> <strong>Solutions</strong> <strong>Manual</strong><br />

154 Chapter 5 Trigonometric Identities<br />

15. (sin x + 1)2 - (sin x _1)2<br />

Let a = sin x + 1 and b = sin x-I.<br />

(sin x + 1)2 - (sin x _1)2<br />

=a 2 _b 2<br />

=(a-b)(a+b)<br />

= [(sin x + I) - (sinx -I)][(sinx + 1)+(sinx-I)]<br />

= 2(2 sin x)<br />

= 4sinx<br />

16. (tanx+cotx)2 -(tanx-cotx)2<br />

Let a =tan x + cot x and b =tan x - cot x.<br />

(tanx+cotx)2 -(tanx-cotx)2<br />

=a 2 _b 2<br />

=(a+b)(a-b)<br />

= [(tan x +cotx) + (tan x - cot x)]·<br />

[(tan x + cot x) - (tan x - cot x)]<br />

=(2 tan x)(2cot x)<br />

=4tanxcotx<br />

=4<br />

17. 2sin 2 x+3sinx+1<br />

Let a = sin x.<br />

2sin 2 x+ 3sinx + 1 = 2a 2 + 3a+ 1<br />

= (2a+ l)(a+l)<br />

=(2 sin x + 'l)(sin x + I)<br />

18. 4 tan 2 fj + tan p- 3<br />

Let a =tanfJ.<br />

4 tan 2 p+tan fj - 3 = 4a 2 +a-3<br />

=(4a - 3)(a + 1)<br />

= (4 tan p- 3)(tan fj + 1)<br />

4x+2cos2x+l<br />

19. cos<br />

2<br />

Let cos x =a.<br />

cos 4 x+2cos 2 x+l =a 2 +2a+l<br />

=(a + 1)2<br />

=(cos 2 x + 1)2<br />

20. cot 4 x+3cot 2 x+2 = (cot 2 x+2)(cot 2 x+l)<br />

21. sin 3 x - cos 3 x<br />

Let sinx = a and cos x = b.<br />

sin 3 x-cos 3 x<br />

=a 3 _b 3<br />

= (a -b)(a 2 +ab +b 2 )<br />

= csc 2 x(cot 2 x + 2)<br />

= (sin x - cosx)(sin 2 x + sin xcosx + cos 2 x)<br />

= (sin x - cos x)(1+ sin x cos x)<br />

22. sin 3 a+cos 3 a<br />

= (sin a + cos a)(sin 2 a - sin a cos a + cos 2 a)<br />

= (sin a + cos a)(I- sin a cos a)<br />

sinO .<br />

23. tan 0 cos 0 = --cosO = smO<br />

cosO<br />

. cosa.<br />

24. cot a sma =-_.sma = cos a<br />

sin a<br />

1<br />

25. secrcosr=--·cosr=1<br />

cosr<br />

cost sint<br />

26. cotr tanr =-_._-= 1<br />

sint cost<br />

27. sinptanfj = tanfjtanp= tan 2 p<br />

cosp<br />

28.<br />

srh-. cok<br />

cot 0 ~<br />

=-_.__._ 1 sinO ­<br />

sinO cosO cosO<br />

1<br />

= cos 2 0<br />

=sec<br />

2 0<br />

esc 0 sec0<br />

2 1<br />

29. sec x-I=---d<br />

cos<br />

2<br />

x<br />

l-cos 2 x<br />

- cos 2 x<br />

2<br />

sin x 2<br />

=--=tan 2 x<br />

cos x<br />

2 2<br />

30. csc t -1 = cot t<br />

sin 2 x . 2 . 1<br />

31. --2-+smxcscx = tan X+SIDX-.­<br />

cmx ~x<br />

=tan 2 x+l<br />

2<br />

=sec x<br />

32. _1_ 2<br />

- + cot atana =cot 2 a + 1=csc 2 a<br />

tan a<br />

. cotO<br />

33. Venfy --=cmO.<br />

cscO<br />

ros9<br />

cotO -;--9<br />

--= .1!!L- = cos 0<br />

esc 0 si~9<br />

154


<strong>Trigonometry</strong> <strong>Solutions</strong> <strong>Manual</strong><br />

5.2 Verifying Trigonometric Identities 155<br />

ify tana ,<br />

34. Ven --=sma.<br />

see a<br />

tana ~ sina .<br />

--=:L =-_·cosa=sma<br />

see a cosa cosa<br />

I-sin z f3 f3<br />

35. Verify = cos .<br />

cosf3<br />

Z Z<br />

1-sin f3 = cos f3 = cos f3<br />

cosf3 cosf3<br />

ify<br />

36• Ven<br />

tan r+1<br />

=seer.<br />

seer<br />

Z<br />

tan Z r + I see Z r<br />

-----''--- =--= see r<br />

seer seer<br />

Z<br />

37. Verify cos Z O(tan 0 + I) = 1.<br />

Z Z<br />

z{sinZO)<br />

cos O(tan O+I)=cos --z-+I<br />

cos 0<br />

Z Z<br />

Z<br />

= cos<br />

{sin O+cos 0)<br />

2<br />

cos 0<br />

=1<br />

38. Verify sin Z ,8(1 + cot Z fJ) = 1.<br />

sin Z f3(l + cot Z fJ) = sin Z f3 csc Z f3<br />

. Z f3 I<br />

=sm . _ ­<br />

sin Z f3<br />

=1<br />

39. Verify cot s + tan s = see s esc s.<br />

coss sins<br />

cot s + tan s =--+-­<br />

sins coss<br />

cos Z s + sin Z s<br />

=<br />

sinscoss<br />

I<br />

=---­<br />

sinscoss<br />

= seescscs<br />

sin z r<br />

42. Verify --=seer-cosr.<br />

cosy<br />

sin z r l-cos z r<br />

--=<br />

cosr cosy<br />

cos<br />

z<br />

r<br />

=-----­<br />

cosy cosy<br />

=secr-cosr<br />

43. Verify sin 4 0 - cos 4 0 = 2 sin Z 0-1.<br />

sin 4 0-cos 4 0<br />

= (sin Z 0 - cos Z O)(sin Z 0 + cos Z 0)<br />

44.<br />

= sin Z 0 -cos Z 0<br />

= sin Z 0 -(l-sin Z 0)<br />

= 2sin Z 0-1<br />

Verify cosO = 1.<br />

sin ocot 0<br />

cosO = cosO . sinO = sinOcosO = I<br />

sinOcotO sinO cosO sinOcosO<br />

45. Verify<br />

(l-cos Z a)(l+cos Z a) = 2sin z a-sin 4 a .<br />

(1-cos z a)(1 + cos z a) = sin z a(l + cos z a)<br />

= sin z a(2 - sin z a)<br />

= 2sin z a -sin 4 a<br />

2<br />

46. Verify tan Z r sin z r = tan r + cos z r-1.<br />

Work with the left side.<br />

tan Z rsinz r = tan Z r(l- cos z r)<br />

Z Z z<br />

= tan r - tan r cos r<br />

Z<br />

= tan r-sinZr<br />

Now work with the right side.<br />

tan Z r+cosZ r -I = tan Z r-(I-cos Z r)<br />

2-sinz<br />

= tan r<br />

40. Verify sin Z a+ tan Z a+cos Z a = sec 2 a .<br />

sin Z a+ tan Z a+cos Z a = 1+ tan Z a = see Z a<br />

. cosa sina Z Z<br />

41. Venfy --+--=sec a-tan a.<br />

seea csca<br />

Work with the left side.<br />

cosa sina cosa sina<br />

--+--= ---l...- +-1­<br />

sec a esc a cos a 5iiia<br />

= cos<br />

za + sin Z a<br />

=1<br />

Now work with the right side.<br />

2<br />

sec a - tan<br />

Za = I<br />

155


<strong>Trigonometry</strong> <strong>Solutions</strong> <strong>Manual</strong><br />

156 Chapter 5 Trigonometric Identities<br />

ify cosO+l cosO<br />

47. Ven =--­<br />

tan 2 0 sec 0 -1.<br />

Work with the left side.<br />

cosO+ 1 cosO+1<br />

tan 20 - sec20-I<br />

eosO+1<br />

=--...,,..--­<br />

-1--1<br />

2<br />

cos o<br />

eosO+I<br />

- l-cos 20<br />

cos 2 o<br />

cos 2 O(eosO + 1)<br />

= (1- cosO)(I+cosO)<br />

20<br />

cos<br />

= I-cosO<br />

Now work with the right side.<br />

20<br />

cosO cosO cosO cos<br />

secO-I = -1--1 = I-cosO = I-cosO<br />

cosO cosO<br />

_<br />

48. Verify<br />

....:. (s_ec_ 0_- _ta_n_0....:,. )_2 _+_1_ = 2 tan O.<br />

sec 0 esc 0 - tan 0 esc 0<br />

(secO-tanO)2 +1<br />

secOcsc0- tanOcsc 0<br />

sec 2 0-2secOtanO+ tan 2 0+ 1<br />

=<br />

esc 6(sec 0 - tan 0)<br />

2 sec 2 0-2secOtanO<br />

= esc O(sec 0 - tan 0)<br />

2 sec O(sec0 - tan 0)<br />

= esc 6(sec 0 - tan 0)<br />

2secO<br />

=--­<br />

cscO<br />

=2 sinO<br />

cosO<br />

=2tanO<br />

49. Verify 1 + 1 2 sec 2 O.<br />

I-sinO l+sinO<br />

1 1 (l+sinO)+(I-sinO)<br />

---+---=<br />

I-sinO 1+ sinO (1- sin 0)(1 + sinO)<br />

2<br />

- I-sin 2 0<br />

2<br />

= cos 20<br />

= 2sec 2 0<br />

50. Verify =seca+tana.<br />

seca-tana<br />

1 seca + tan a<br />

-----=----­<br />

seca-tana<br />

sec a - tan a seca+ tan a<br />

sec a + tan a<br />

- sec 2 a - tan 2 a<br />

seca+tana<br />

=----..,......­<br />

±_ sin 2a<br />

cos a cos 2 a<br />

sec a + tan a<br />

= 2<br />

C:2~<br />

= seca+tana<br />

tans sins<br />

51. Verify + = cots+ secscscs.<br />

I+coss l-coss<br />

---+ tans sins =_--:.__---:._-;:-.....:....__....:­<br />

tans(l-coss)+sins(l+coss)<br />

l+coss l-eoss I-eos 2 s<br />

tans -sins + sins +sinscoss<br />

=<br />

sin 2 s<br />

tans coss<br />

=--+-­<br />

sin 2 s sins<br />

sins 1<br />

=--'--+cots<br />

coss sin 2 s<br />

1 1<br />

=--'--+cots<br />

coss sins<br />

= secscscs+ cots<br />

. l-cosx 2<br />

52. Venfy = (cot x -csex) .<br />

I+cosx<br />

Work with the left side.<br />

l-cosx (1- cosx)(1-cosx)<br />

---=-=---_....:....:._---'­<br />

l+cosx (l+cosx)(I-cosx)<br />

1-2cosx+cos 2 x<br />

=<br />

l-cos 2 x<br />

2<br />

1- 2 cos x + cos x .<br />

=<br />

sin 2 x<br />

Work with the right side.<br />

2 (cosx 1)2<br />

(cotx-cscx) = -.---.­<br />

smx<br />

smx<br />

= (co~x _1)2<br />

sm,r<br />

cos 2 x-2cosx+l<br />

=<br />

sin 2 x<br />

'f eota+ 1 1+ tan<br />

53.<br />

a<br />

Ven y =--­<br />

cota-l I-tana<br />

cot a + 1 tarhi + 1 It~'aa 1+ tan a<br />

cot a -1 =tarhi-1 = I~~':.ra = 1- tan a<br />

156


<strong>Trigonometry</strong> <strong>Solutions</strong> <strong>Manual</strong><br />

5.2 Verifying Trigonometric Identities 157<br />

54. Verify + =-2tana. 58.<br />

tana-seca tana+seca<br />

1 1<br />

-----+----­<br />

tan a -seca tan a +seca<br />

tana+seca+ tan a -seca<br />

= 2 2<br />

tan a - sec a<br />

2tana<br />

= . 2<br />

~~2~ - c~2a<br />

2tana<br />

= -(1-sin 2 a)<br />

cos 2a<br />

2tana<br />

= 2<br />

-:2~<br />

= -2 tana<br />

55. Verify sin 2 a sec 2 a + sin 2 acsc 2 a = sec 2 a. 59.<br />

" 2<br />

. 2 2 . 2 2 sm a 1<br />

sm asec a+sm acsc a=--2-+<br />

cos a<br />

= tan 2 a+I<br />

2<br />

= sec a<br />

. cscO+cotO<br />

56. Venfy = cotOcscO.<br />

tanO+sinO<br />

cscO+cotO srne+~<br />

tanO+sinO = ~+sinO<br />

- ~<br />

- sin6(1+cosB)<br />

cose<br />

(1 + cosO) cosO<br />

= sin 2 0(1+ cosO)<br />

cosO<br />

= sin 20<br />

=-_._-<br />

1 cosO<br />

sinO sinO<br />

=cscOcotO<br />

57. Verify sec 4 x-sec 2 x=tan 4 x + tan 2 x ,<br />

Simplify left side.<br />

4<br />

sec x-sec 2 x= sec 2 x(sec 2 x-I)<br />

60.<br />

2 2 61.<br />

= sec x tan x<br />

Simplify right side.<br />

4 2<br />

tan x+tan x=tan 2 x(tan 2 x+I)<br />

2 2<br />

= tan x sec x<br />

. I-sinO 2 2<br />

Venfy =sec 0-2secOtanO+tan O.<br />

1+sinO<br />

Work with the right side.<br />

sec 2 0 - 2secOtan 0 + tan 2 0<br />

1 2sinO sin 20<br />

=------+--­<br />

20 20 20<br />

cos cos cos<br />

1-2sinO+sin 2 0<br />

= 20<br />

cos<br />

(1 - sin 0)(1-sin 0)<br />

- I-sin 20<br />

_ (1 - sin 0)(1- sin 0)<br />

- (1 + sinOXI-sinO)<br />

I-sinO<br />

= 1+ sinO<br />

.. sinO cosO<br />

Venfy smO+cosO = ~~(J + -.Sin1! •<br />

I-~ I-~<br />

Work with the right side.<br />

sin 0 cosO sin 20 cos 20<br />

6+ " 6 = +<br />

I-~ 1- ~e sinO-cosO cosO-sinO<br />

sin 20 cos<br />

= 20<br />

+<br />

sinO- cosO -{sinO ~ cosO)<br />

sin 2 0-cos 2 0<br />

= sinO-cosO<br />

(sin 0 + cosOXsin0 - cos 0)<br />

=-'--------:...;:.-_--~<br />

(sin0 - cos 0)<br />

= sinO+cosO<br />

sinO sinOcosO 0( 2)<br />

Verify<br />

I-cosO l+cosO<br />

esc I + cos 0 .<br />

sinO sinOcosO<br />

I-cosO I+cosO<br />

sin 0 + sinOcosO - sin OcosO+ sin0 cos 2 0<br />

=<br />

sin 20<br />

20<br />

cos<br />

=cscO+--­<br />

sinO<br />

=cscO(I + cos 2 0)<br />

4<br />

sec s - tan 4 s<br />

Verify<br />

2s-tan2s<br />

2 2 =sec .<br />

sec s+ tan s<br />

2 2<br />

sec 4 s-tan 4 s _ (sec s-tan 2 sX sec2 s+tan s)<br />

sec 2 s+tan 2 s - sec 2 s+tan 2 s<br />

2 2<br />

= sec s - tan s<br />

157


<strong>Trigonometry</strong> <strong>Solutions</strong> <strong>Manual</strong><br />

158 Chapter 5 Trigonometric Identities<br />

62. Veriy f<br />

cot2 t -I I 2 ' 2 t<br />

2=-sm.<br />

1+ cot t<br />

2<br />

cot 2 t-I ~-I<br />

l+cot 2 t = I+~<br />

SID t<br />

COS 2 t-sin 2 t<br />

_ sin 2 t<br />

- sio 2 t+cos 2 t<br />

sin 2 t<br />

cos 2 t - sin 2 t<br />

=<br />

sin 2 t<br />

2t-sin2t<br />

=cos<br />

=(l-sin 2 t) -sin 2 t<br />

=1-2sin 2 t<br />

tan 2 t -I tant - cott<br />

63. Verify sec2 t = tan t + cott<br />

Work with the right side.<br />

tant -cott tant-~<br />

= -ltant<br />

+ cott<br />

tan t + taot<br />

.1a1U:J.<br />

=--¥L-<br />

JilId.±l.<br />

tant<br />

tan 2 t-I<br />

- tan 2 t+1<br />

2<br />

tan t -I<br />

= sec 2 t<br />

64. Verify (l+sinx+cosx)2 =2(I+sinx)(l+cosx).<br />

Work with the right side.<br />

2(1 + sin x)(1 + cosx)<br />

= 2(1 +sinx +cosx+sinxcosx)<br />

= 2 + 2sin x + 2 cos x + 2sin x cos x<br />

Work with the left side.<br />

(l+sinx+cosx)2<br />

. . 2 •<br />

= I+sinx+cosx+smx+sm x+smxcosx<br />

+cosx+sinxcosx+cos 2 x<br />

= 2 + 2sinx + 2cosx + 2sin xcosx<br />

I+cosx I-cosx<br />

65. Verify - =4cotxcscx.<br />

I-cosx I+cosx<br />

I+cosx I-cosx<br />

I-cosx I+cosx<br />

l+cos 2 x+2cosx-I-cos 2 x+2cosx<br />

=<br />

l-cos 2 x<br />

4cosx<br />

= sin 2 x<br />

=4cotxcscx<br />

2 I-sina<br />

66. Verify (seca-tana) = . .<br />

I+sma<br />

(seca-tana)2 = sec 2 a- 2secatana+tan 2 a<br />

I 2sin a sin 2 a<br />

--------+--­<br />

2 2 2 a<br />

- cos a cos a cos<br />

1-2sina+sin 2 a<br />

2a<br />

cos<br />

(l-sina)2<br />

= l-sin 2 a<br />

(l-sina)2<br />

=-~----,..---"----:-<br />

(I-sin a)(1+sin a)<br />

I-sina<br />

= I+sina<br />

67. Verify (sec a + csca)(cosa -sina) =cow -tana.<br />

(seca + esc a)(cosa - sin a)<br />

= (_1_ + -._I_)(cos a - sin a)<br />

cosa smrz<br />

= I +cota - tana-I<br />

=cota-tana<br />

. 4 4<br />

sin a-cos a =I .<br />

68. Verify 2 2<br />

sin a-cos a<br />

sin 4 a - cos 4 a<br />

sin 2 a - cos 2 a<br />

71.<br />

(sin 2 a - cos 2 aXsin 2 a + cos 2 a) _<br />

=<br />

sin 2 a-cos<br />

2<br />

a<br />

-I<br />

The graph of y = (secx + tanx)(l-sinx)<br />

appears to be the same as the graph of y = cos x.<br />

(sec 0 + tan 0)(1- sin0) =(_1_ + _si_n_0)1-sin 0)<br />

cosO cosO<br />

=(I + sin 0)(1- sin 0)<br />

cosO<br />

l-sin 20<br />

=<br />

cosO<br />

cos<br />

20<br />

=--­<br />

cosO<br />

=cosO<br />

158


<strong>Trigonometry</strong> <strong>Solutions</strong> <strong>Manual</strong><br />

72. 74.<br />

+ cotx)(secx - 1) y 1 = tan x sin x + cos x<br />

5.2 Verifying Trigonometric Identities 159<br />

73.<br />

The graph of y = (cscx+cotxXsecx-l)<br />

appears to be the same as the graph of y = tan x .<br />

(cscO+ cotOXsecO -1)<br />

=(_1+ cos 0X-1-1)<br />

sinO sinO cosO<br />

=(1+.COSOXI-COSO)<br />

smO cosO<br />

l-cos<br />

= 20<br />

sinOcosO<br />

sin<br />

= 20<br />

sin 0 cos 0<br />

sinO<br />

=-­<br />

cosO<br />

=tan 0<br />

The graph of y = tan x sin x + cos x appears to be<br />

the same as the graph of y = sec x .<br />

tanOsinO+cosO= sinO (sinO)+cosO<br />

cosO<br />

sin 20 cos 20<br />

=--+--­<br />

cosO cosO<br />

sin 2 0 + cos 2 0<br />

=<br />

cosO<br />

1<br />

=-­<br />

cosO<br />

=secO<br />

2+5coss 2 5 identi<br />

75• Is . = cscs+ cots ani entity<br />

sms<br />

Graph each side of the equation.<br />

y =2+Scos.>:<br />

1 sin r<br />

Y2 = 2cscx + 5cotx<br />

4<br />

-4<br />

cosx+ 1<br />

The graph of y = . appears to be the<br />

smx+tanx<br />

same as the graph of y =cot x.<br />

cosO+ 1 1+ cosO<br />

= .mul.<br />

sinO+tanO sinO+ cosO<br />

l+cosO<br />

- sin 0(1 + JsO)<br />

_<br />

(l+cosO)cosO<br />

- sin 0(1 + cobi)cosO<br />

_ (I + cos 0) cos 0<br />

- sin O(cosO+ I)<br />

cosO<br />

=-­<br />

sinO<br />

= cot 6<br />

-4<br />

2+5cosx<br />

The graphs of y = . and<br />

smx<br />

y = 2 esc x +5 cot x appear to be the same . The<br />

given equation may be an identity. Verify it.<br />

2+5coss 2 5coss<br />

----=--+-­<br />

sins sins sins<br />

= 2 esc s + 5 cot s<br />

The given statement is an identity.<br />

159


<strong>Trigonometry</strong> <strong>Solutions</strong> <strong>Manual</strong><br />

160 Chapter 5 Trigonometric Identities<br />

2<br />

sec s<br />

76. Is I + cot 2 s = 2 an identity<br />

sec s-I<br />

Gra h each side of the equation.<br />

I<br />

y = 1 + 1<br />

Y I = 1 + cotZx I 1 + sin x "':"l-----"s.,...in-x- II<br />

Y _ scc Z x<br />

z-sccZx-l<br />

sec 2 x<br />

The graphs of y = I + cot 2 x and y = --:::2"""'-­<br />

sec x-I<br />

appear to be the same. The given equation may be<br />

an identity . Verify it.<br />

Work with the right side.<br />

sec 2 s sec 2 s<br />

sec 2 s -I =tan 2 s<br />

1 cos 2 s<br />

= cos 2 s . sin 2 s<br />

I 2<br />

=--=csc<br />

sin 2 s<br />

s<br />

= l+cot 2 s<br />

The given statement is an identity.<br />

I I<br />

The graphs of y = . + . and<br />

I+slnx I-smx<br />

y =sec 2 x are not the same. The given statement<br />

is not an identity.<br />

79. Is l-tan2 s<br />

2:= cos 2 s ­<br />

1+ tan s<br />

.. .<br />

Sin S an identity<br />

Graph each side of the equation.<br />

1 ­ tanZ x<br />

YI := -=-l-+---"tan==-Z..:.:.x<br />

77.<br />

l-tan 2 x<br />

The graphs of y = 2<br />

I+tan x and<br />

y = cos 2 x ­ sin x are not the same. The given<br />

statement is not an identity.­<br />

-4<br />

tan x ­ cot x<br />

. 2<br />

The graphs of y = and y = 2sm x<br />

tanx+cotx<br />

are not the same. The given statement is not an<br />

identity.<br />

80. Is sin3 s-cos 3 s . 2 . 2<br />

= sm s + 2 Sin S cos S + cos s<br />

sins-coss<br />

an identity Graph each side of the equation.<br />

YZ = sinZ x + 2sin x cos x + cosZ x<br />

-4<br />

sin 3 x ­ cos 3 x<br />

The graphs of y = .<br />

and<br />

smx-cosx<br />

y=sin 2 x+2sinxcosx+cos 2 x are not the<br />

same. The given statement is not an identity.<br />

160


<strong>Trigonometry</strong> <strong>Solutions</strong> <strong>Manual</strong><br />

5.2 Verifying Trigonometric Identities 161<br />

81. cot 0 84• IS = sec 0 - tan O'd an I entity. '<br />

cscO+ 1<br />

Evaluate each side of the equation using a table of<br />

cot 0<br />

values. Let Y 1 = and Y 2 =secO-tanO.<br />

cscO+ 1<br />

82.<br />

The table values are not the same. The given<br />

statement is not an identity.<br />

The table values are the same for all values for<br />

which Y 1 and Y2 are both defined. If your table<br />

contained only points for which they are both<br />

defined, the equation would appear to be an<br />

identity. Since there are values for which one is<br />

defined and the other is not defined, the given<br />

statement is not an identity.<br />

85. Show that sin (esc s) = 1 is not an identity.<br />

Lets= 2.<br />

sin (esc 2) =:: .89 10940 1<br />

Thus, sin(esc s) ¢. 1 for all teal numbers s, so this<br />

statement is not an identity .<br />

83.<br />

The table values are not the same. The given<br />

statement is not an identity. .<br />

The table values appear to be the same. The given<br />

equation may be an identity. Verify it.<br />

2 . 2 sin 2 x sin 2 xcos 2 x<br />

tan x-sm x=-----....,.....-­<br />

cos 2 x cos 2 x<br />

_ sin 2 x(l- cos 2 x)<br />

- cos 2 x<br />

sin 2 x ( 2)<br />

=--2- l e-cos x<br />

cos x<br />

= tan 2 xsin 2 x<br />

= (tanxsinx)2<br />

The given statement is an identity.<br />

86. Show that ~cos2 s =coss is not an identity.<br />

Let s = 1.<br />

~cos21 =.54030231<br />

cos 1 =.54030231<br />

2n<br />

But let s=-.<br />

3<br />

1<br />

coss=-­ 2<br />

~cos2 s =~(-~r =JT =±<br />

Thus, ~cos2 s ¢. coss for all real numbers s, so<br />

this statement is not an identity.<br />

87. Show that csct =~1 + cot 2 t is not an identity.<br />

Lett =1.<br />

esc =:: 1.1883951<br />

~r-1+-c-o--=t2­ 1<br />

=:: 1.1883951<br />

But let t= 4.<br />

esc 4 =:: -1.3213487<br />

~1 + cot 21 =:: 1.3213487<br />

Recall that .Ja denotes the positive square root of<br />

a.<br />

Thus, esc t ¢. ~1 + cos 2 t for all real numbers t, so<br />

this statement is not an identity .<br />

161


<strong>Trigonometry</strong> <strong>Solutions</strong> <strong>Manual</strong><br />

162 Chapter 5 Trigonometric Identities<br />

88. Show that cos t = ~ I - sin 2 t is not an identity.<br />

6. cos(-105°)<br />

= co~-45°+(-600)]<br />

31r<br />

Let t=-.<br />

2<br />

= cos(-45°)cos(-60°) - sin(-45°)sin(-600)<br />

Then cos t = O. = cos 45° cos 60° - (-sin 45°X- sin 60°)<br />

~1-sin2t =~I-(-I) =.Ji<br />

Thus, cos t *-~1-sin 2 t for all real numbers t, so<br />

this statement is not an identity. =--­<br />

4 4<br />

89. Let tan(J = t and show that sin (Jcos(J =-f-. =<br />

Section 5.3<br />

Exercises<br />

Work with the right side.<br />

t tan(J<br />

=.J2 ..!.- (_ .J2x- .J3]<br />

2 2 2 2<br />

.Ji.J6<br />

.J2-.J6<br />

t +1 4<br />

7. co{~~)=cos(~ + :)<br />

t 2 + I = tan 2 (J+ I<br />

tan(J 1r 1r . 1r . 1r<br />

= cos - cos - - sm-sm­<br />

= sec2 (J 3 4 3 4<br />

=tan (Jcos 2 (J I .Ji .J3.Ji<br />

=-._-_.­<br />

sin (J 2 (J<br />

=--cos<br />

2 2 2 2<br />

cos(J<br />

.Ji.J6<br />

= sin (Jcos (J<br />

=--­<br />

4 4<br />

.Ji-.J6<br />

=<br />

4<br />

3. cos 75°<br />

= cos(30° + 45°)<br />

1r x .1r .1r<br />

= cos 30°cos 45° - sin 30° sin 45° = cos -cos-+sm-sm­<br />

6 4 6 4<br />

4<br />

=_._--.- .J3.JiI.Ji<br />

=_._+_.­ .J3..fiI.J2<br />

2 2 2 2<br />

2 2 2 2<br />

=<br />

.J6-.Ji<br />

.J6.Ji<br />

=-+­<br />

4 4<br />

4. cos(-15°) .J6+.Ji<br />

=<br />

= cos(30° - 45°) 4<br />

=cos 30° cos 45° + sin 30° sin 45°<br />

.J3.JiI.Ji<br />

9. cos 40° cos 50° - sin 40° sin 50°= cos(40° + 50°) .<br />

=_._+_.- = cos 90°<br />

2 2 2 2 =0<br />

.J6+.Ji<br />

= 10. cos(_10°) cos 35° + sin(_10°) sin 35°<br />

4<br />

= cos(-IO° - 35°)<br />

5. cos(105°)<br />

=cos(-45°)<br />

=cos(60° + 45°)<br />

= cos(600)cos(45°) - sin(60°) sin(45°) .J2<br />

=- 2<br />

=!. .J2 -(.J3X.Ji)<br />

2 2 2 2 21r 1r . 21r . 1r J21r 1r)<br />

11. cosScos 10 -smSsm io = CO'S+ 10<br />

=--­<br />

.J2.J6<br />

4 4 51r<br />

=cos­<br />

.J2-.J6 10<br />

= 1r<br />

4 = cos­<br />

2<br />

=0<br />

162


<strong>Trigonometry</strong> <strong>Solutions</strong> <strong>Manual</strong><br />

5.3 Sum and Difference Identities for Cosine 163<br />

7rr 2rr . 7rr . 2rr J7rr 2rr)<br />

12. =co'\9+9<br />

cos<br />

9cos9-sJn 9sJn9<br />

= cosrr<br />

=-1<br />

13. The answer to Exercise 9 is O. Using a calculator<br />

to evaluate cos 40° cos 50° - sin 40° sin 50° also<br />

gives the value of O.<br />

14. The answer to Exercise 10 is ..fi. Using a<br />

2<br />

calculator to approximate ..fi gives the value<br />

2<br />

.70710678. Using a calculator to evaluate<br />

cos(-10°) cos 35° + sin(-IO O )sin 35° also gives<br />

the value .70710678.<br />

17. cos ~ = sin( ~ - ~) = sin ~;<br />

19. csc(-14°24') = sec[900-(-14°24')]<br />

= sec 104°24'<br />

22. cot 9rr = tan( rr _ 9rr) = tan(- 2rr)<br />

10 2 10 5<br />

25. cot 176.9814° = tan(900-176.9814°)<br />

= tan(-86.9814°)<br />

26. sin98.0142° = cos(9O° - 98.0142°)<br />

= cos(-8.0142°)<br />

28. sin _2;_ = ( _ :)<br />

. rr rr 2rr<br />

Since --=--­<br />

6 2 3'<br />

sin 2; = cos(; _ 2;) ::;; cos(- : )<br />

Answer: cos<br />

29. 33° ::;; sin 57°<br />

sin 57° = cos(9O°- 57°)<br />

= cos 33°<br />

Answer: cos<br />

30. 72° ::;; cot 18°<br />

Since 90° - 18°::;; 72°,<br />

cot 18° = tan (90° - 18°)::;; tan 72°.<br />

Answer: tan<br />

1<br />

31. cos 70° = --­<br />

20°<br />

cos 70° = sin(9O°- 700)<br />

= sin 20°<br />

1<br />

=--­<br />

csc20°<br />

Answer: esc<br />

1<br />

32. tan 24° =---­<br />

66°<br />

Since 90° - 24° ::;; 66°,<br />

tan 24° = cot(90° - 24°)<br />

= cot 66°<br />

1<br />

= tan 66° .<br />

Answer: tan<br />

33. tan 8 = cot(45° + 28)<br />

Since tan 8 = cot(90° - 8),<br />

90° - 8 = 45°+28<br />

38 =45°<br />

8 = 15°.<br />

34. sin 8 =cos(28 _10°)<br />

Since sin 8 =cos(90° - 8),<br />

90° - 8 =28 - 10°<br />

100° = 38<br />

100°<br />

8=-.<br />

3<br />

rr rr<br />

27. cot-= ­<br />

3 --6<br />

. rrrrrr rr rr<br />

SInce -::;; - - - cot - = tan - .<br />

6 2 3' 3 6<br />

Answer: tan<br />

163


<strong>Trigonometry</strong> <strong>Solutions</strong> <strong>Manual</strong><br />

164 Chapter 5 Trigonometric Identities<br />

35. sec 8 = csc(~ + 20°)<br />

Since sec 8 = csc(90° - 8).<br />

8<br />

90°-8 = -+20°.<br />

2<br />

~+200=900-8<br />

2<br />

8+~+200=90°<br />

2<br />

28 + 8 + 40° = 180°<br />

30= 140°<br />

140°<br />

8=­<br />

3<br />

36. cos 8 = sin(30+ 10°)<br />

Since cos8 = sin(90° - 8).<br />

90° - 8 = 30 + 10°<br />

80°=40<br />

8 = 20°.<br />

37. sin(30-15°)=(eosO+25°)<br />

Since sin 0 = eos(90° - 0),<br />

sin(30 -15°) = eos(90° - (30 -15°)] .<br />

90° - (30 -15°) = 8 + 25°<br />

105° - 30 = 0 + 25°<br />

80°=40<br />

20°=0<br />

38. eot(0 -10°) = tan(20 + 20°)<br />

Since cot 8 = tan(90° - 0) ,<br />

oot(O _10°) = tan[90° - (0-10 0 )l<br />

90°_ (0-10°) = 20+20°<br />

100°-0 =20+20°<br />

80°=30<br />

80°<br />

-=0<br />

3<br />

39. cos(O°- 0) = cos 0° cos 0 + sinOsin 0°<br />

= 1eosO + Osin 8<br />

=eosO<br />

40. cos(90° - 0) = cos 90° cosO + sin900sin8<br />

... = OcosO+ lsinO<br />

=sinO<br />

43. cos(O°+8) = cosO°cos8 -sinOosin8<br />

= 1cos8-0sin8<br />

= cos8<br />

44. cos(90° + 8) = cos 90° cos 8 - sin 90° sin 8<br />

= 0 cos 8 - 1sin 8<br />

= -sin8<br />

45. cos(180° + 8) = cos 180° cos 8 - sin 180° sin 8<br />

= (-1)cos8 - Osin 8<br />

=-cos8<br />

46. cos(270° + 0) = cos 270° cosO - sin 270°sin 0<br />

= OcosO - (-1)sin 0<br />

=sinO<br />

1 d si 3<br />

47. coss=-- an sml=-.<br />

5 5<br />

sand 1 are in quadrant II.<br />

Solve sin 2 s + cos 2 s = 1 for sin s. Since s is in<br />

quadrant II. sin s > O.<br />

sins=~l-(-~r = ~<br />

Solve sin 2 1 + cos 2 1 = 1 for cos I. Since 1 is in<br />

quadrant II, cos 1 < O.<br />

cOSI=-~l-(~r =-~<br />

cos(s + I) = cos s cosr - sm ssinr<br />

=(-~X -~)-( ~X~)<br />

4 'Wi4<br />

=---­<br />

25 25<br />

4-&./6<br />

=<br />

25<br />

cos(s -I) = eosseOSI + sin ssinr<br />

=(-~X -~)+( ~X~)<br />

4 3..fi4<br />

=-+-­<br />

25 25<br />

4+&./6<br />

=<br />

25<br />

41. cos(180° - 0) = cos 180° cos 0 + sin 180° sin 0<br />

= (-1)cos 0 + Osin 0<br />

=-cosO<br />

42. cos(270° - 0) = cos 270° cos 0 + sin 270° sin 0<br />

= OeosO+ (-l)sin 0<br />

=-sinO<br />

164


<strong>Trigonometry</strong> <strong>Solutions</strong> <strong>Manual</strong><br />

5.3 Sum and Difference Identities for Cosine 165<br />

48• sm · s =-2 an d sm si 1 =- -. 1<br />

3 3<br />

s is in quadrant II and 1 is in quadrant IV.<br />

First, find cos s and cos I .<br />

sin 2 s+cos 2 s = 1<br />

cos 2 s = 1-sin 2 s<br />

=1-(%r<br />

5<br />

=­<br />

9<br />

Since s is in quadrant Il, cos s < 0, so<br />

coss=-~ =- ~.<br />

cos 2 1 = 1-sin 2 1<br />

=1-(-~r<br />

8<br />

=­<br />

9<br />

Since 1 is in quadrant IV, cos I> 0, so<br />

cos 1 = f! = 2.,fi .<br />

~9 3<br />

cos(s + I) = cos s cosr -sinssinl<br />

=(- ~X2~)_(%X-~)<br />

-2.,JW +2<br />

=<br />

9<br />

cos(s -I) = cos s cosr +sinssinl<br />

=(- ~X2~)+(~X-~)<br />

-2.,JW -2<br />

=<br />

9<br />

' 3 d si 12<br />

49• SIOS=- an SIOI=--.<br />

5 13<br />

s is in quadrant I and 1 is in quadrant III<br />

cos s > 0 and cos 1 < O.<br />

COSS=~I-(%r =~<br />

53<br />

cos 1 = -~1-(- ~~r = -1<br />

cos(s + 1)= cos s cos 1- sin s sinr<br />

=(~X-1~)-(~X-~~)<br />

16<br />

=­<br />

65<br />

cos (s - 1)= cosscosl + sin ssinr<br />

5<br />

=(~X-13) + ( %X - ~~)<br />

56<br />

=-­<br />

65<br />

8 3<br />

so. coss =-- and COsI = --.<br />

17 5<br />

sand 1are in quadrant ill.<br />

First, find sin s and sin I.<br />

2<br />

. 2 2 8 225<br />

sm s =1-cos s = 1- ( - 17 ) = 289<br />

Since s is in quadrant ill, sin s < 0, so<br />

sins = -~~~~ = - :~ '.<br />

2<br />

. 2 2 3 16<br />

sm 1=I-cos 1=1- ( -'5 ) = 25<br />

Since 1 is in quadrant ill, sin 1 < 0, so<br />

sinl=-~ =-~.<br />

COS(s+l) =cos s cosr - sins sinr<br />

=(-1~ X-~)-(-:~X-~)<br />

24 60<br />

=--­<br />

85 85<br />

36<br />

=-­<br />

85<br />

cos(s -I) = cosscos 1+sinssinl<br />

24 60<br />

=-+­<br />

85 85<br />

84<br />

=­<br />

85<br />

165


<strong>Trigonometry</strong> <strong>Solutions</strong> <strong>Manual</strong><br />

166 Chapter 5 Trigonometric Identities<br />

51.<br />

. .J5 d. ../6<br />

S1OS=- an S1OI=-.<br />

7 8<br />

sand 1 are in quadrant I.<br />

cos s > 0 and cos 1 > o.<br />

COS'=~I-[ ~)' ~~<br />

~ (../6)Z. $8<br />

COSI= 1-""""8" =-8<br />

cos(s + 1)= cosSCOSI -sinssinl<br />

cos(s ­<br />

=(~X~)-(~)(~)<br />

2.J638 -.J30<br />

=<br />

53.<br />

Since 1 is in quadrant IV, cos I> 0, so<br />

.s:<br />

cosl=--.<br />

6<br />

cos(s + 1)= cos s cosr ­ sin ssinr<br />

=(V;)('T)-(-~X­v;)<br />

...[6i-~<br />

=<br />

24<br />

cos(s ­ I) = coss cosr + sin s sinr<br />

...[6i+~<br />

=<br />

24<br />

cos 42° = cos(30° + 12°)<br />

Since 42° = 30° + 12°, the statement is true.<br />

56<br />

I) =cos scosr + sins sinr 54. cos (_24°) = cos 16° - cos 40°<br />

cos (A ­ B) = cosAcosB+ sinAsinB<br />

=(~X~)+(~X~)<br />

cos (-24°)= cos(16° - 40°)<br />

= cos 16° cos 40° + sin 16°sin 40°<br />

2.J638 +.J30<br />

¢. cos 16° - cos 40°<br />

=<br />

The statement is false.<br />

56<br />

52.<br />

-J2 . .J5<br />

coss=- and sml=--.<br />

4 6<br />

sand 1 in quadrant IV.<br />

First. find sin s and cos I.<br />

sin z s = l-cos Z s<br />

55. cos 74° = cos 60° cos 14° + sin 60° sin 14°<br />

cos(A + B) = cos A cos B-sin Asin B<br />

cos 74° = cos(6O°+ 14°)<br />

= cos 60° cos 14° - sin 60° sin 14°<br />

The statement is false.<br />

56. cos 140° = cos 60° cos 80° - sin 60° sin 80°<br />

=1-(V;)'<br />

cos (A + B) = cos A cos B ­ sin A sin B<br />

1<br />

=1-­ cos 140° = cos(600 + 80°)<br />

8<br />

7<br />

=­ 8<br />

Since s is in quadrant IV, sin s < 0, so<br />

sins=-~ =-i=-21 =­~.<br />

57.<br />

= cos 60° cos 80° - sin 60° sin 80°<br />

The statement is true .<br />

x 1r 1r .1r.1r<br />

cos­ = cos-cos­- SIO -S1O­<br />

3 12 4 12 4<br />

cos(A + B) = cosAcos B-sinAsin B<br />

cos"3=co{41r)<br />

co,, t=l-,in' t= 1-[- ~)' = :~<br />

=co{.!!-+ 31r)<br />

12 12<br />

=co{~ + ~)<br />

12<br />

1r 1r .1r.1r<br />

= cos -cos­ - SIO -S1O­<br />

12 4 12 4<br />

The statement is true.<br />

166


<strong>Trigonometry</strong> <strong>Solutions</strong> <strong>Manual</strong><br />

5.3 Sum and Difference Identities for Cosine 167<br />

2n lIn n . lIn . n<br />

58. cos- =cos--cos-+sm--sm­<br />

3 12 4 12 4<br />

cos (A - B) = cos A cos B + sin A sin B<br />

cos ""3 2n = cos (8n) 12<br />

= cos( lIn _ 3n)<br />

12 12<br />

= cos( lIn _ n)<br />

12 4<br />

lIn n . l br . n<br />

= cos--cos- + sm --sm­<br />

12 4 12 4<br />

The statement is true.<br />

59. cos 70° cos 20° - sin 70° sin 20° = 0<br />

cos(A + B) = cos A cos B - sin A sin B<br />

cos 70° cos 20° - sin 70° sin 20° = cos(70° + 20°)<br />

= cos 90°<br />

=0<br />

The statement is true.<br />

60. cos 85° cos 40° + sin 85° sin 40° = ..fi<br />

2<br />

cos A cos B + sin A sin B = cos (A - B)<br />

cos 85° cos 40° + sin 85° sin 40° = cos(85° - 40°)<br />

= cos 45°<br />

..fi<br />

=­<br />

2<br />

The statement is true.<br />

61. ta{O- ~)=cotO<br />

Simplify the left side.<br />

tan(o _ n) = sin( 0 - t)<br />

2 cos(O-f)<br />

-sin(f-O)<br />

- co~f-O)<br />

-cosO<br />

=--­<br />

sinO<br />

=-cotO<br />

The statement is false.<br />

62. sin(0 - ~) = cos 0<br />

sin(o- ~)=-sin(~-0)=-COSO<br />

The statement is false.<br />

63. Verify cos( ~ + x) = -sinx.<br />

n ) n . n .<br />

co -+x =cos-cosx-sm-smx<br />

{ 2 2 2<br />

= 0 cos x-Isin x<br />

=-sinx<br />

64. Verify sec (n-x) =-secx.<br />

1<br />

sec(n - x) =---­<br />

cosor - x)<br />

1<br />

=-------­<br />

cos n cos x + sin n sin x<br />

1<br />

=------­<br />

(-1) cos x + (0) sin x<br />

1<br />

=-­<br />

cosx<br />

=-secx<br />

2<br />

65. Verify cos2x = cos x - sin 2 x.<br />

cos2x =cos(x+ x)<br />

= cos x cos x - sin x sin x<br />

= cos 2 x -sin 2 x<br />

66. Verify I+cos2x-cos 2 x=cos x.<br />

2<br />

From Exercise 65, cos2x = cos x -sin 2 x .<br />

I+cos2x-cos 2 x=I+cos 2 x-sin 2 x-cos 2 x<br />

= I-sin 2 x<br />

2<br />

=cos x<br />

67. Verify cos(n+ s-t) = -sinssint -cosscost.<br />

cos(n+ s - t) = co~n+ (s-t)]<br />

= cosncos(s--t)-sin nsin(s - t)<br />

= (-I)cos(s - t) -Osin(s - t)<br />

= (-IXcosscost + sin ssin t)<br />

=-cosscost -sinssint<br />

68. Verify co{~ +S-t)=sin(t-s).<br />

co{~ +s-t) =co{~ +(s-t)]<br />

2<br />

= cos n cos(s_ t)- sin n sin(s - t)<br />

2 2<br />

= 0 -Isin(s -t)<br />

= sin(t -s)<br />

167


<strong>Trigonometry</strong> <strong>Solutions</strong> <strong>Manual</strong><br />

168 Chapter 5 Trigonometric Identities<br />

69. Verify cos(a + /3)cos(a - /3)= l-sin 2 a - sin 2 fJ.<br />

Work with the left side.<br />

cos(a + fJ)cos(a"- fJ) = (cos a cos fJ -sin asinfJXcosacos fJ + sin asinfJ)<br />

= cos 2 acos 2 fJ-sin 2 asin 2 fJ +cosacosfJsinasinfJ -sinasinfJcosacosfJ<br />

= (l-sin 2 fJ)cos 2 a -(I-oos 2 a)sin 2 fJ<br />

= cos 2 a - sin 2 fJ cos 2 a - sin 2 fJ + cos 2 asin 2 fJ<br />

= cos 2 a-sin 2 fJ<br />

= (l-sin 2 a)-sin 2 fJ<br />

70. Verify cos 4x cos Tx - sin 4x sin Tx = cos l Ix, 76. (a) cos 255° =cos(180°+ 75°)<br />

cos 4x cos 7x- sin 4x sin 7x =cos (4x+ 7x) == cos 180°cos 75° - sin 180°sin 75°<br />

= cos l l,r<br />

= (-1)cos 75° -(O)sin75°<br />

72. cos 20° =sin (90° - 20°) = sin 70° =-cos75°<br />

Some other values that would make<br />

= - cos(45° + 30°)<br />

cos20° =sin 9 true would be any angle =-(cos45° cos30° - sin 45°sin 30°)<br />

cotenninal with 70°, such as 430°, 790°, 1150 0 ,<br />

-290 0 , -650 0 • =J~.~_~.!)<br />

Furthermore, since sin 70 0 = sin 110°, 9 can be 1.2 2 22<br />

110°or any angle coterminal with 110° (for<br />

example, 470°, 830°, 1190°, -250°, -610°). =­<br />

.J6-~<br />

4<br />

73. cos 195 0 = cos(180°+ 15°) ~-.J6<br />

=<br />

= cos 180°cosl5° - sin180°sin15° 4<br />

= (-1)cos15° - (0)sin 15°<br />

=-cosI5°<br />

(b) cos \1; = cofr - ~)<br />

74. -


<strong>Trigonometry</strong> <strong>Solutions</strong> <strong>Manual</strong><br />

5.3 Sum and Difference Identities for Cosine 169<br />

78. Verify tan A =cot (90° - A).<br />

sinA<br />

tan A =-­<br />

cos A<br />

_ cos(90° - A)<br />

- sin(90° - A)<br />

= cot(90° - A)<br />

79. Verify<br />

- .::....;f(:.-x+_h~)-......:f~(x--"-) =cosx (COSh -1) . (Sinh)<br />

-SIDX -­ .<br />

h h h<br />

Letf (x) = cos x.<br />

f(x + h)=cos(x+ h)<br />

=cosxcosh -sinxsinh<br />

f(x+h)- f(x) =cosxcosh-sinxsinh-cosx<br />

h<br />

h<br />

cosxcosh - cosx - sinxsinh<br />

=<br />

h<br />

cos x cos h - cosx sin xsinh<br />

=<br />

h h<br />

=cosx<br />

COSh-1)<br />

h<br />

. (Sin h)<br />

( -SIDX -h­<br />

80. Verify cos 140° + cos 100° + cos 20° = O.<br />

cos 140° + cos 100° + cos 20° = cos(I20° + 20°) + cos(I20° - 20°) + cos 20°<br />

= cos 120°cos20° - sin 120°sin 20° + cos 120°cos 20° + sin 120°sin 20° +cos 20°<br />

= -.!.cos20° - .J3 sin 20° + ( _.!.) cos 20° + .J3 sin 20° + cos 20°<br />

2 2 2 2<br />

= - cos 20"+cos 20°<br />

=0<br />

81. Since there are 60 cycles per sec, the number of 83. (a) Graph<br />

cycles in .05 sec is given by<br />

(.05 sec)(60 cycles per sec) = 3 cycles . p=;cos[2: -C1J<br />

82. Since V = 163sinca and the maximum value of = ~ co{2H(IO) -10261)<br />

sin ca is 1, the maximum voltage is 163. 10 4.9<br />

Since V = 163sinca and the minimum value of<br />

sin ca is -1, the minimum voltage is -163.<br />

Therefore, the voltage is not always equal to 115.<br />

= .04cos(20H -10261).<br />

4.9<br />

For x = r,<br />

4 [201r ]<br />

P(t) = to cos 4:9 - 1026t<br />

.057/<br />

£-,: r':\('l#,,:;t:;ri; 3~<br />

; \: .r. ';i .~ ,~' ',~ I<br />

o .05<br />

.<br />

.i , ~~ ! t ~, - :< .~ , i> ~~ ::<br />

- .05<br />

The pressure P is oscillating.<br />

169


<strong>Trigonometry</strong> <strong>Solutions</strong> <strong>Manual</strong><br />

170 Chapter 5 Trigonometric Identities<br />

(b) Graph<br />

p=;co{2~ -ctJ<br />

= ~cos[21r(r) -1026(10)]<br />

r 4.9<br />

=~ coJ 2m- -10.260).<br />

r \'4.9<br />

For x = r,<br />

3 [2Trr 1<br />

per) = r cos 4:9 - 10,260<br />

Section 5.4<br />

Exercises<br />

3. sin 15° = sin(45° - 30°)<br />

=sin 45° cos 30° - cos 45° sin 30°<br />

..fi ~<br />

..fi I<br />

=-._-_.­<br />

2 2 2 2<br />

.J6..fi<br />

=--­<br />

4 4<br />

.J6-..fi<br />

=<br />

4<br />

The answer is C.<br />

(c)<br />

-2<br />

The pressure oscillates. and amplitude<br />

decreases as r increases.<br />

p=;co{2~ -ctJ<br />

Let r=nll. .<br />

=.!!....co{2mzll. -ctJ<br />

nil. Il.<br />

a<br />

=- cos(2mz - ct]<br />

nil.<br />

= n~ [cos(2mz)cos(ct) + sin(2mz)sin(ct)]<br />

= n~ [(I)cos(ct) + (O)sin(ct)]<br />

a<br />

=-cos(ct)<br />

nil.<br />

4. sin 105° =sin(60° + 45°)<br />

= sin 60° cos 45° + cos 60° sin 45°<br />

~..fil..fi =_._+_.­<br />

2 2 2 2<br />

.J6+..fi<br />

=<br />

4<br />

The answer is A.<br />

5. tan 15° =tan(6QO -45°)<br />

tan 60° - tan 45°<br />

= I + tan 60° tan 45°<br />

~-I ~-I I-~<br />

= I+~(I) = I+~ ·I-~<br />

~ -3-1+ ·~ -4+2~<br />

= =-_..:..:...<br />

1-3 -2<br />

-2(2-~)<br />

= =2-~<br />

-2<br />

The answer is E.<br />

84. y' =r cos(0 + R)<br />

= r[cosOcos R-sinOsin R]<br />

= (rcosO)cosR-(rsinO)sin R<br />

= ycosR- zsin R<br />

z' =rsin(O+ R)<br />

=rco{~ -(O+R»)<br />

=rco{(~ -O)-R)<br />

= r[cos(~ -0)CosR+sin(~-0)sin R]<br />

=r[sinOcos R+ cosOsin R]<br />

= (rsinO)cos R+(rcosO)sin R<br />

= zcos R+ ysin R<br />

6. tan 105° =tan(60° + 45°)<br />

tan 60° + tan 45°<br />

= 1-tan 60° tan 45°<br />

.J3+1 .J3+1<br />

= 1-(.J3XI) =1-.J3<br />

.J3+ I 1+.J3 4 + 2.J3<br />

=1-.J3 ·I+~ = -2<br />

=-2-.J3<br />

The answer is F.<br />

7. sin(-105°)=sin(45° - 150°)<br />

=sin 45° cos 150° - cos 45° sin 150°<br />

=( ~X-~)-( ~X~)<br />

=<br />

The answer is B.<br />

-J6 -..fi<br />

4<br />

170


<strong>Trigonometry</strong> <strong>Solutions</strong> <strong>Manual</strong><br />

5.4 Sum and Difference Identities for Sine and Tangent 171<br />

8. tan(-105°) = -tan105°<br />

= - tan(60° + 45°)<br />

tan 60° + tan 45°<br />

= 1- tan60° tan 45°<br />

../3 +1 1+../3<br />

=~·1+../3<br />

1+2../3+3<br />

=<br />

-2<br />

4+2../3<br />

=<br />

-2<br />

= 2+../3<br />

The answer is D.<br />

9. sin ~~ =Sin(: + :)<br />

.re re re.re<br />

= sm -cos- + cos-s1o­<br />

4 6 ·4 6<br />

=_._+_.­<br />

-J2../3 -J2 1<br />

2 2 2 2<br />

..f6-J2<br />

=-+­<br />

4 4<br />

..f6+-J2<br />

=<br />

4<br />

10. tan~~ =tan(: +:)<br />

_ tant+tant<br />

-1-tan JI..tan JI..<br />

6 4<br />

*+1 1;[<br />

=1-*=1i 1<br />

1+.J3 ../3+1<br />

= ../3-1 ' .J3+1<br />

1+2.J3+3<br />

=<br />

2<br />

4+2.J3<br />

=<br />

2<br />

=2+../3<br />

11. tan 1~ = tan(: - :)<br />

tant-tant<br />

. l+tanftanf<br />

_ 1-4 _1-4<br />

- 1+(1)4- - 1+4­<br />

3-.J3<br />

=3+.J3<br />

3-.J3 3-.J3<br />

= 3+../3' 3-.J3<br />

=<br />

12-6.J3<br />

6<br />

=2-../3<br />

12. sin ~ =sin(: - :)<br />

.re re re.re<br />

=S1O-COS--cos-s1o­<br />

4 6 4 6<br />

=_._-_.- -J2.J3 ..fi 1<br />

2 2 2 2<br />

../6 ..fi<br />

=--­<br />

4 4<br />

../6-..fi<br />

=<br />

4<br />

13. sin(- ~~) = Sin(- : - ~ )<br />

. (re) re {re). re<br />

=S1O -"4 cos 3--eo -"4 S10 3<br />

.re re re.re<br />

= -S1O- cos - - cos -SID­<br />

4 3 4 3<br />

..fi 1 -J2.J3 ..fi..f6<br />

=-_.--_._=---­<br />

2 2 2 .2 4 4<br />

-J2-../6<br />

=<br />

4<br />

171


<strong>Trigonometry</strong> <strong>Solutions</strong> <strong>Manual</strong><br />

172 Chapter 5 Trigonometric Identities<br />

14. tan(- ;;) = tan[- =+ ( - ~)] 22. tan "* + tan t (5rr rr)<br />

I-tan1ftan t = tan 12+"4<br />

15.<br />

tan(-t) + tan(-i)<br />

= 1-tan(-t)tan(-t)<br />

_ -1+(-J3)<br />

- l-(-lX-J3)<br />

_-I-..[j<br />

- 1-..J3<br />

-1-..J3 1+../3<br />

= 1-13 '1+..J3<br />

-1-..J3 -..J3-3<br />

=<br />

-2<br />

-4-2..J3<br />

=<br />

-2<br />

=2+..J3<br />

8rr<br />

= tan­<br />

12<br />

2rr<br />

= tan­ 3<br />

=-..J3<br />

23. cos(300 + 9) = cos 30° cos 9 - sin30°sin 9<br />

= ..J3cos9-.!.sin9<br />

2 2<br />

=: ±(..J3 cos9-sin9)<br />

..J3cos9 - sin 9<br />

=<br />

2<br />

24. cos(45° - 9) = cos 45°cos 9 + sin45° sin 9<br />

-Ii<br />

-Ii '<br />

= -cos9+-sm9<br />

2 2<br />

sin 76° cos 31° - cos 76° sin 31°<br />

= sin(76° - 31°)<br />

..fi(cos9 + sin 9)<br />

= sin 45°<br />

2<br />

..fi<br />

=­ 2<br />

25. cos(600+9) = cos60°cos9-sin60osin9<br />

= .!.cos9 - ..J3sin9<br />

16. sin 40° cos 50° + cos 40° sin 50°<br />

2 2<br />

= sin(40° + 50°)<br />

= ±(cos9-..J3sin 9)<br />

sin 90°<br />

= 1 cos9-..J3sin9<br />

=<br />

2<br />

17. tan 80° + tan 55° = tan(800+ 550)<br />

1- tan 80° tan 55° 26. cos(9 - 30°) = cos 9 cos 30° +sin9sin 30°<br />

= tan 135°<br />

=-1<br />

..J3 9<br />

1'9<br />

=:-cos +-sm<br />

2 2<br />

18. tan 80° - tan(-55°) = tan[800-(-550)]<br />

..[j cos9+sin9<br />

=<br />

1+ tan 80° tan(-55°)<br />

2<br />

= tan 135°<br />

=-1<br />

31r ) 3rr , 3rr ,<br />

27. co<br />

{--x =cos-cosx+sm-smx<br />

4 4 4<br />

19. tan 100°+ tan 80° = tan(I000+ 800)<br />

-Ii ..fi .<br />

=--cosx+-smx<br />

1- tan 100°tan 800 2 2<br />

= tan 180°<br />

=0<br />

=:- ..fi(-cosx+smx<br />

inx)<br />

2<br />

20. sin 100° cos 10° - cos 100° sin 10°<br />

=: sin(l000-100)<br />

=: sin 90°<br />

= 1<br />

21. sin rr cos 3rr + cos rr sin 3rr = sin(!E.+ 3rr)<br />

510 510 510<br />

, rr<br />

=sm­ 2<br />

=1<br />

=: ..fi(sinx-cosx)<br />

2<br />

172


<strong>Trigonometry</strong> <strong>Solutions</strong> <strong>Manual</strong><br />

5.4 Sum and Difference Identities for Sine and Tangent 173<br />

28. sin(45° + 0) = sin 45° cos 0 + sin 0 cos 45°<br />

= .J2 sinO+ .J2 cosO<br />

2 2<br />

= .J2 (sin 0 + cos 0)<br />

2<br />

.J2(sin 0 + cosO)<br />

=<br />

2<br />

tan 0 + tan 30°<br />

29. tan(0 + 30°) = 1 Ll 300<br />

-tan u tan<br />

_ tanO+i<br />

- l-itan O<br />

.J3 tanO+ 1<br />

= .J3 -tanO<br />

tan -+x 1r ) =_......:t. tan "i + tan x<br />

30. _<br />

( 4 1-tan "itan x<br />

l+tanx<br />

= l-tanx<br />

.{ 1r ) . 1r n .<br />

31. Sl -+x =sm-cosx+cos-smx<br />

4 4 4<br />

.J2 .J2 .<br />

=-cosx+-smx<br />

2 2<br />

..J2(cos x + sinx)<br />

=<br />

2<br />

32. sin(1800-0) = sin 180°cosO-cosI800sinO<br />

= O(cosO)-(-I)(sinO)<br />

=sinO<br />

33. sin(2700 - 0) = sin 270° cos 0 - cos 270° sin 0<br />

34.<br />

35.<br />

= -1(cosO)-O(sinO)<br />

=-cosO<br />

tan(1800+ 0) = tan 180° + tan 0<br />

1- tan 180° tan 0<br />

O+tanO<br />

= 1-0<br />

= tan 0<br />

tan(3600_ 0) = tan 360° - tan 0<br />

1+ tan 360° tan 0<br />

O-tanO<br />

= 1+ O(tanO)<br />

=-tanO<br />

36. sin(n + 0) = sin ncosO+ cos xsin d<br />

37.<br />

= O·cosO+(-l)sinO<br />

= -sinO<br />

o tan n - tan 0 0 - tan 0<br />

tan(n- ) = l+tanntanO = 1+ O(tan0)<br />

=-tanO<br />

40. If A, B, and C are angles of a triangle, then<br />

A + B + C = 180° . Therefore,<br />

sin(A + B+C)= sin 180° = O.<br />

3. 5 d d .<br />

41. cosS=-, smt=-, an san tare m<br />

5 13<br />

quadrant I. First, find sin s, tan s, cos t, and tan<br />

t. Since sand t are in quadrant I, all are<br />

positive.<br />

sins=~I-(~r =~<br />

. 4 4<br />

sms 5'<br />

tans=--=-=coss<br />

! 3<br />

cos t = ~1_ ( 5 )2 = g<br />

13 13<br />

2.. 5<br />

tant = 11. =­<br />

11. 12<br />

13<br />

. 4 12 5 3 63<br />

sm(s + t) = _. - +_.- =­<br />

513 13565<br />

. 4 12 5 3 33<br />

sm(s - t) = - .- - - .- = ­<br />

513 13565<br />

tan s + tan t .<br />

tan(s+t)=---­<br />

1-tan stant<br />

_ l'<br />

4+2<br />

IT<br />

- 1-(tXi!)<br />

63<br />

=­<br />

16<br />

A_...ltan(s<br />

- t) = 3(AX I2S )<br />

1+ 3 IT<br />

33<br />

=­<br />

56<br />

To find the quadrant of s + t, notice that<br />

sin(s + t) > 0, which implies s + t is in<br />

quadrant I or II. tan(s + t) > 0, which implies<br />

s + t is in quadrant I or III. Therefore, s + t is in<br />

quadrant I. Then, sin(s - t) > 0 and<br />

tan (s - t) > O. From the preceding, s - t must<br />

also be in quadrant I.<br />

173


<strong>Trigonometry</strong> <strong>Solutions</strong> <strong>Manual</strong><br />

174 Chapter 5 Trigonometric Identities<br />

42. coss = _.!., sint =~, sand t are in quadrant II.<br />

5 5<br />

First, find sin s, tan s, cos t, and tan r. Since s<br />

is in quadrant II, sin s > O.<br />

sins=~I-cos2 s =~I-(-~r =~ = 2:<br />

. 2,/6<br />

tan s = _sl_n_s = _-r_ = -2../6<br />

coss<br />

-!<br />

Since t is in quadrant II, cos t < O.<br />

cost=-~I-sin2t=-~I-(~r =-~=-~<br />

sint ! 3<br />

tant=--=-=-­<br />

cost --! 4<br />

sin(s + t) = sin s cost + coss sin t<br />

=(2:X-~)+(-~X~)<br />

-8../6 -3<br />

=<br />

25<br />

sin(s - t) = sinscost - cosssint<br />

=(2:X-~)_(-~X~)<br />

-8../6 +3<br />

=<br />

25<br />

tans+ tant<br />

tan(s+t)=---­<br />

I-tanstant<br />

_ -2../6+(-t)<br />

- 1-(-2../6X-t)<br />

-~<br />

- 4-6../6<br />

4<br />

_ -8../6-3<br />

- 4-6../6 .<br />

tan s - tant<br />

tan(s-t)=---­<br />

1+ tanstant<br />

_ -2../6-(-t)<br />

- 1+ (-2../6X-t)<br />

_ -8../6 +3<br />

- 4+6../6<br />

To find the quadrant of s + t, notice from the<br />

preceding that sin(s + t) =<br />

-s.J6 -3<br />

25<br />

< 0 and<br />

-8../6-3 0 Th .. .<br />

tan(s+t) =<br />

L 17 >. e sme IS negative<br />

4-0"'/6<br />

in quadrants III and IV, the tangent is positive<br />

in quadrants I and III. Therefore, s + t is in<br />

quadrant III. Then, sin(s - t) < 0 and<br />

tan(s - t) < O. The tangent is negative in<br />

quadrants II and IV. From the preceding, s - t<br />

must be in quadrant IV.<br />

. 2. I. . d II d<br />

43. SIOS=-, smt=--,s IS 10 qua rant , an t<br />

3 3<br />

is in quadrant IV. Since s is in quadrant II,<br />

cos s < O.<br />

coss=-~I-(~r =-~<br />

Since t is in quadrant IV, cos t > O.<br />

cost=~I-( -~r = ~ = 2~<br />

i 2 2../5<br />

tans=--=--=--­<br />

-4 ../5 ·5 '<br />

--! 1 -Ii<br />

tant=-- =---=-­<br />

~ 2.fi 4<br />

Sin(s+t)=~(-2~-2)~(- ~X-~)<br />

4.fi ..f5<br />

=--+­<br />

9 9<br />

4.fi +../5<br />

=<br />

9<br />

Sin(s-t)=~e~)-(- '7X-~)<br />

=---­<br />

4:J2 ../5<br />

9 9<br />

4.fi-../5<br />

=<br />

9<br />

Different forms of tan (s + t) and tan (s - t)<br />

will be obtained depending on whether tan s<br />

and tan t are written with rationalized<br />

denominators.<br />

174


<strong>Trigonometry</strong> <strong>Solutions</strong> <strong>Manual</strong><br />

5.4 Sum and Difference Identities for Sine and Tangent 175<br />

-~+(-4)<br />

tan(s+l) =1-(-~)-4)<br />

or<br />

-8,/5 -5../2<br />

= 20-2.J!O<br />

-i+(-~)<br />

tan(s+l) = ( ,f )<br />

1- -iJ\-~<br />

_-4../2-,/5<br />

- 2.J!O-2<br />

4../2 +,/5<br />

= 2-2..[W<br />

-¥-(-~)<br />

tan(s-I) = 1+(_2f)-4)<br />

or<br />

-g,f5 +5../2<br />

= 20 +2..[W<br />

-i-(-~)<br />

tan(s - I)= ( )( )<br />

1+ -i --rlr<br />

-4../2+,/5<br />

= 2&+2<br />

To find the quadrant of s + I, notice that<br />

sin(s + I) > 0, which implies s + I is in<br />

quadrant I or II. tan(s + I) < 0, which implies<br />

s + I is in quadrant II or IV. Therefore, s + I is<br />

in quadrant II. Also notice that sin(s - I) > 0,<br />

which implies s - I is in quadrant I or Il, and<br />

tan(s - I) < 0, which implies s - I is in<br />

quadrant II or IV. Therefore, s - I is in quadrant<br />

II.<br />

' 3. 12. . dId<br />

44. sms=-, sml=--,slsmqua rant , an I<br />

5 13<br />

is in quadrant III. Since s is in quadrant I,<br />

cos s > o.<br />

COSS=~l-(~r =~<br />

3<br />

tans=­<br />

4<br />

Since I is in quadrant III, cos I < O.<br />

COSI=_~1_(_12)2<br />

12<br />

tanl=­<br />

5<br />

=_2­<br />

13 13<br />

sin(s + I) =(~)(- 2-) + (iX-g) =_63<br />

5 13 5 13 65<br />

sin(s - I) = (~)(_2-) - (iX-g) = 33<br />

5 13 5 13 65<br />

1. 12. .il 63<br />

4 + 5 20<br />

tan(s + I) = (")('''1) = --r6 =-­<br />

1- t ~ -"2lj 16<br />

t_ U 33<br />

tan(s-I)= (1.)(12.)=-­<br />

1+ 4 5 56<br />

To find the quadrant of s + I, notice from the<br />

preceding that sin(s + I)= - 63 < 0 and<br />

65<br />

tan(s+1)=--O and tan(s-I)=--


<strong>Trigonometry</strong> <strong>Solutions</strong> <strong>Manual</strong><br />

176 Chapter 5 Trigonometric Identities<br />

To find the quadrant of s + t, notice that<br />

sin(s + t) > 0, which implies s + I is in<br />

5 13<br />

quadrant I or II, and tan(s + I) < 0, which<br />

I is in quadrant IV.<br />

implies s + I is in quadrant II or IV. Therefore, Since s is in quadrant Ill, cos s < o.<br />

s + I is in quadrant II. Also notice that<br />

sin(s - I) > 0, which implies s - lis in quadrant<br />

I or II, and tan(s - I) > 0, which implies s - t is<br />

coss =-~I-(-~r =-%<br />

in quadrant I or III. Therefore, s - I is in<br />

quadrant I.<br />

15 4<br />

46. cos s = - 17' sin I = '5' s is in quadrant II, and I<br />

is in quadrant I.<br />

sin s = ~1-( - : ~ I~ r=<br />

8<br />

tans=-­<br />

15<br />

COSI=~I-(~r =%<br />

4<br />

tanl=­<br />

3<br />

(.!.X~) + (-~X~) = ­<br />

sin(s + I) = · 36<br />

17 5 17 5 85<br />

sin(s - I) = (.!.X~)-(-~X~) = 84<br />

17 5 17 5 85<br />

(-fs)+t 36<br />

tan(s + I) = 1-(-i\Xt) = n<br />

(-fs)-t 84<br />

tan(s - t) = 1+ (--AXt) =-13<br />

47. sin s = - ~, cos I = Q, s is in quadrant III, and<br />

Since t is in quadrant IV, sin I < O.<br />

sinl=-~I-C~r =-1 53<br />

-t 4<br />

tans = _j ='3<br />

-f.,-<br />

*<br />

5<br />

tant=--=-­<br />

12<br />

sin(s+l) = (_~XI2)+(_2.X-~) = _ 33<br />

5 13 13 5 65<br />

sin(s-I)=(_iXI2)_(_2.X-~)=- 63<br />

513 13 565<br />

t--& 48-15 33<br />

tan(s+l) = 1-(tX-n) = 36+20 = 56<br />

t+-& 48+15 63<br />

tan(s-I) = I + (tX-n) = 36-20 =16<br />

To find the quadrants of s + I and s - I, notice<br />

that sin(s + I) < 0 and sin(s - I) < 0, which<br />

implies that s + I and s - I are in quadrant III or<br />

IV. Also notice that tan(s + I) > 0 and<br />

tan(s - I) > 0, which implies that both s + I and<br />

s - I are in quadrant I or III. Therefore, both<br />

s + I and s - I are in quadrant III.<br />

To find the quadrant of s + I, notice from the<br />

preceding that sin(s + I) = - 36 < 0 and<br />

85<br />

tan(s + I ) = - 36 > O. The e si sme IS negative .. m<br />

n<br />

quadrants ill and IV, the tangent is positive in<br />

quadrants I and III. Therefore, s + I is in<br />

quadrant ill. Also, sin(s - I) = :: > 0 and<br />

84 .. ...<br />

tan ()=-- s -I


<strong>Trigonometry</strong> <strong>Solutions</strong> <strong>Manual</strong><br />

n+(-t)<br />

16<br />

tan(s + t)= 1-(n-X-t) =- 63<br />

fi-(-t) 56<br />

tan(s - t) = ( 5)( 3) = ­<br />

1+ IT -'4 33<br />

Notice from the preceding that<br />

. 16 16<br />

S1O(S+t) =--< 0 and tan(s+t);;:;; --< O.<br />

65 63<br />

The sine is negative in quadrants III and IV.<br />

The tangent is negative in quadrants II and IV.<br />

Therefore, s + t is in quadrant IV. Also,<br />

. 56 56<br />

S1O(S-t)=->O and tan(s-t)=->O. The<br />

65 33<br />

sine is positive in quadrants I and II, the<br />

tangent is positive in quadrants I and III.<br />

Therefore, s - t is in quadrant I.<br />

.J7. -J3 d d .<br />

49• cos s = - -, S10 t =-, an s an tare 10<br />

4 5<br />

quadrant II.<br />

Since s is in quadrant II, sin s > O.<br />

gnS=~l-(- -:)' =~<br />

Since t is in quadrant II, cos t < O.<br />

cost=-~l-( ~)' =-~<br />

t 3 3..fi<br />

tans=--=--=-­<br />

-d}- .J7 7<br />

:Ii -J3 ~<br />

tant= -42 =- ..[fi=---:n:<br />

Sin(s+t)=(~X-~)+(~X-'7)<br />

=-3..ffi -.J2f =-(3..ffi +.J2f)<br />

20 20<br />

sin(s-t)=(~X- ~)-( ~X-~)<br />

-3..ffi +.J2f<br />

=<br />

20<br />

5.4 Sum and Difference Identities for Sine and Tangent 177<br />

(_1:fi)+ (_::&i)<br />

tan(s+t) = 7 22<br />

1-(-~)(-~)<br />

_ -(66.J7 + 7$6)<br />

- 154- 3.J462<br />

(_1:fi)_(-1i-)<br />

tan(s-t) = 7 2<br />

1+(-¥X-~)<br />

_-66.J1 + 7$6<br />

- 154+ 3.J462<br />

To find the quadrants of s + t and s - t, notice<br />

that sin(s + t) < 0 and sin(s - t) < 0, which<br />

mean s + t and s - t are in quadrant III or IV.<br />

Also notice that tan(s + t) < 0 and<br />

tan(s - t) < 0, which imply that s + t and s - t<br />

are in quadrant II or IV. Therefore, both<br />

s + t and s - t are in quadrant IV.<br />

.JIT.fi .<br />

50. coss = -, cos t ;;:;;-, and sand tare 10<br />

5 6<br />

quadrant IV.<br />

gnS=-~l-( ~)' =-~<br />

.J14 ../154<br />

tans=---=--­<br />

.JIT 11<br />

sint=-~[-[ "1)' =-.;;:<br />

_:iiI .J34<br />

tant = 4- =- .fi =-ffi.<br />

sin(s+t) =(- ~X'7)+( ~X- .;;:)<br />

-..fi8 --J374<br />

=<br />

30<br />

sin(s-t) =(- ~X'7)-(~X-.;;:)<br />

-..fi8 + -J374<br />

=<br />

30<br />

177


<strong>Trigonometry</strong> <strong>Solutions</strong> <strong>Manual</strong><br />

178 Chapter 5 Trigonometric Identities<br />

(-*)+(-~)<br />

tan(s + t) = ""::""'"7"""---:::==--­<br />

1-(- 4r)(-~)<br />

-../154 -11~<br />

=---'----;===-­<br />

11-../2618<br />

(-4F)-(-ffi)<br />

tan(S - t) = ....::.........,.----==="-­<br />

1+(-4F)-ffi)<br />

-/154+11~<br />

=---'-----;===-­<br />

11+../2618<br />

Notice from the preceding that<br />

. -fiB-../374<br />

Sln(s+t) =<br />

< 0 and<br />

30<br />

-../154 -11~ ..<br />

tan(s+t) = ~ >0. The sme IS<br />

11-"\12618<br />

negative in quadrants III and IV, the tangent is<br />

positive in quadrants I and III. Therefore s + t is<br />

in quadrant III. Also,<br />

. -.J28 + ../374<br />

sm(s-t) =<br />

30<br />

>0 and<br />

tan(s-t) = -/154+11~ >0.<br />

11+../2618<br />

The sine is positive in quadrants I and II, and<br />

the tangent is positive in quadrants I and III.<br />

Therefore, s - t is in quadrant I.<br />

52.<br />

53.<br />

The graph of y =Si{3; + x) appears to be the<br />

same as the graph of y = - cos x. Verify<br />

_(31C ) Sin T+x = -cosx.<br />

. (31C ) _ 31C 31C .<br />

sm -+x =sm-cosx+COS-SlnX<br />

2 2 2<br />

= (-I)cosx+Osinx<br />

= -cosx<br />

51.<br />

The graph of y = sin( ~ + x) appears to be the<br />

same as the graph of y =cos x. Verify<br />

sin( ~ + x) = cos x.<br />

. (1C ) . 1C . 1C<br />

Sin -+x =sm-cosx+smxcos­<br />

222<br />

= 1-cosx +sinx ·O<br />

=cosx<br />

The graph of y = tan( ~ + x) appears to be the<br />

same as the graph of y = -cot x . Verify<br />

tan(~ +x) = -cotx.<br />

tan(1C + x) = sin(f+ x)<br />

2 co~f+x)<br />

_ sinfcosx+cosfsinx<br />

- cosfcosx-sinfsinx<br />

_ (lXcosx)+(OXsinx)<br />

- (OXcosx)-(IXsinx)<br />

cos x<br />

=-­<br />

-sinx<br />

=-cotx<br />

178


<strong>Trigonometry</strong> <strong>Solutions</strong> <strong>Manual</strong><br />

5.4 Sum and Difference Identities for Sine and Tangent 179<br />

54. 1 + tan x<br />

Yt=l-tanx<br />

I+tanx (tr )<br />

The graph of y = --- appears to be the same as the graph of y = tan -4 + x . Verify<br />

1-tanx<br />

1+ tan x= tan( tr + x).<br />

I-tanx 4<br />

Work with the right side.<br />

tr<br />

tr ) tan-+tanx<br />

tan - + x = _---'4'-__<br />

( 4 1_ tan tr tan x<br />

4<br />

1+ tan x<br />

= I-tanx<br />

55. Verify sin2x = 2sinxcosx.<br />

sin2x = sin(x+ x)<br />

= sin xcosx + sinxcosx<br />

=2sinxcosx<br />

56. Verify sin (x + y) + sin (x - y) = 2 sin x cos y.<br />

sin (x + y) + sin (x - y) =(sin x cos y + cos x sin y) + (sin x cos y - cos x sin y)<br />

= 2 sinx cos y<br />

57. Verify tan(x _ y) _ tan(y _ x) = 2(tan x - tan y) .<br />

I+tanxtany<br />

tan(x - y) - tan(y - x)<br />

tan x- tany tany -tanx<br />

=------''­<br />

1+ tan x tan y 1+ tan y tan x<br />

tan x - tan y - tan y + tan x<br />

=------''---....;...--­<br />

I + tan x tan y<br />

=2(tanx-tany)<br />

I+tanxtany<br />

58. Verify sin(210° + x)- cos(120° + x) =O.<br />

sin(210° +x)-cos(l20° + x) = (sin2100cosx + cos2100sin x) - (cos120°cosx -sin 120°sin x)<br />

·1 ..[3. (1 ..[3.)<br />

=-"2cosx-Tsmx- -"2cosx-Tsmx<br />

=0<br />

179


<strong>Trigonometry</strong> <strong>Solutions</strong> <strong>Manual</strong><br />

180 Chapter 5 Trigonometric Identities<br />

cos(a - 13)<br />

59. Verify . = tana + cot 13.<br />

cosasmf3<br />

cos(a - f3) _ cosa cos 13 + sin a sin 13<br />

cosasinf3 - cosasinf3<br />

cosf3 sina<br />

=--+-­<br />

sinf3 cosa<br />

= cot 13 + tan a<br />

sin(s+t)<br />

60. Verify = tans + tant.<br />

cosscost<br />

sin(s+t) =sinscost+cosssint<br />

coss cost cosscost<br />

sins sint<br />

=--+-­<br />

coss cost<br />

= tans+ tant<br />

61.<br />

62.<br />

Verify sin(x- y) = tanx-tany<br />

sin(x+y) tanx+tany<br />

tanx-tany = cosx<br />

tan x + tany<br />

~_ siny<br />

cosy<br />

.sin..I.. + sin y<br />

cos x cosy<br />

= sinxcosy -cosxsiny<br />

sinxcos y + cosxsin y<br />

_ sin(x- y)<br />

- sin(x+ y)<br />

Verify sin(x+ y) cotx+coty<br />

cos(x - y) 1+ cotx cot y<br />

Work with the right side.<br />

~+ cosy<br />

cot x + cot y = SID X SiiiY<br />

l+cotxcoty 1+ cosxcosy<br />

sin X SID Y<br />

sinycosx+sinxcosy<br />

sinxsID y<br />

- sinxsiny+cosxcosy<br />

smxsmy<br />

_ sin(x + y)<br />

- cos(x - y)<br />

63.<br />

sint<br />

sins cos 2 t - sintcost coss<br />

=<br />

sint cost<br />

sintcostcoss+sin 2 tsins<br />

+-----:---_.:....:..:=..:...<br />

sintcost<br />

sinscos 2 t+sinssin 2 t<br />

=<br />

sintcost<br />

= sins(cos 2 t+sin 2 t)<br />

sintcost<br />

sins<br />

= sintcost<br />

64. Verify tan(a + 13) - tan 13<br />

1+ tan(a+ f3)lanf3 lana.<br />

lan(a + 13) - tan 13<br />

1+ tan(a + 13) tan13 = tan(a+ 13)- 13] = lana<br />

65. sin 165°= sin(180° -15°)<br />

= sin180°cosI5° - cos180°sin 15°<br />

=sinI5°<br />

=sin(45° - 30°)<br />

=sin45°cos30°- cos 45° sin30°<br />

.J2 ~ .J2 1 =_._-_.­<br />

2 2 2 2<br />

.J6-.J2<br />

=<br />

4<br />

180


<strong>Trigonometry</strong> <strong>Solutions</strong> <strong>Manual</strong><br />

5.4 Sum and Difference Identities for Sine and Tangent 181<br />

66. tan 165° = tan(180° -15°)<br />

tan 180° - tan 15°<br />

= 1+ tan 180°tan 15°<br />

0-tanI5°<br />

= 1+(0)tanI5°<br />

=-tanI5°<br />

= - tan(45° - 30°)<br />

tan 45° - tan 30°<br />

=­ 1+ tan 45° tan 30°<br />

I-if<br />

=-1+(1(4)<br />

1-:f­<br />

=-1+:f­<br />

3-../3<br />

=- 3+../3<br />

../3 -3<br />

= 3+../3<br />

. ../3-3 3-../3<br />

= 3+Jj ' 3-:&<br />

6-J3 -12<br />

=<br />

6<br />

=../3 -2<br />

67. tan 255° = tan(1800+ 75°)<br />

tan 1800+ tan 75°<br />

= 1-tan 180°tan 75°<br />

= tan 75°<br />

= tan(45° + 30°)<br />

tan 45° + tan 30°<br />

= I - tan 45° tan 300<br />

I+if<br />

= 1-4­<br />

3+..[3 3+..[3<br />

= 3-..[3 ' 3+..[3<br />

12+6..[3<br />

=<br />

6<br />

=2+..[3<br />

68. sin 255° = sin(270° - 15°)<br />

= sin 270° cos 15° - cos 270° sin 15°<br />

= -cos 15°<br />

= - cos(45° - 30°)<br />

= -(cos 45° cos 30° + sin 45° sin 30°)<br />

=J..fi ...[3 + ..fi .~)<br />

1.2 2 22<br />

={~+ '7)<br />

-J6 -..fi<br />

=<br />

4<br />

69. sin 285° = sin(360° - 75°)<br />

= sin 360° cos 75° - cos 360° sin 75°<br />

=-sin75°<br />

= - sin(30° + 45°)<br />

= -(sin 30° cos 45° + cos 30° sin 45°)<br />

=JL..fi + ..[3 . ..fi)<br />

1.2 2 2 2<br />

-..fi-~<br />

=<br />

4<br />

70. tan 285° = tan(36O° - 75°)<br />

tan 360° - tan 75°<br />

= I + tan 360° tan 75°<br />

=-tan75°<br />

= -tan(300+45°)<br />

tan 30° + tan 45°<br />

=<br />

1- tan 30° tan 45°<br />

../3+3 ../3+3<br />

=- 3-..[3' 3+..[3<br />

12+6-J3<br />

=­<br />

6<br />

=-2-../3<br />

181


<strong>Trigonometry</strong> <strong>Solutions</strong> <strong>Manual</strong><br />

182 Chapter 5 Trigonometric Identities<br />

_ l br . ( if)<br />

71. smU=sm if- 12<br />

73. tan(- 1:;) = - tan(if+ 1~)<br />

· if . if if<br />

= smifcos--cosifsm- tan x-e tan-i-­<br />

12 12 12<br />

if<br />

· if<br />

=sm-<br />

1-taniftan­<br />

12 12<br />

if<br />

· (if if) =-tan­<br />

=sm 4"-'6 12<br />

. if if if. if<br />

=sm-cos--cos-sm- =-tan(: - :)<br />

4 6 4 6<br />

~../3 ~ 1 = tanf-tant<br />

=_._-_.­<br />

2 2 2 2 l+tanftanf<br />

~-~<br />

=<br />

4<br />

= 1-:1i<br />

3_<br />

1+4<br />

72. tan Ill; = tan(if - 1~) 3-../3 3-../3<br />

=- 3+..J3 · 3-


<strong>Trigonometry</strong> <strong>Solutions</strong> <strong>Manual</strong><br />

5.4 Sum and Difference Identities for Sine and Tangent 183<br />

76.<br />

tan(A _ B) = sin(A - B)<br />

cos(A-B)<br />

sin AcosB- cosAsinB<br />

= cos A cos B+ sin A sin B<br />

sjnAcosB-cosAsin B<br />

_ cosAcosB<br />

- cos Acos B±sioAsin B<br />

cosAcoslf<br />

sjn Acos B cosAsjn B<br />

_ cos Acos B cos Acos B<br />

- cos AcosB + sinAsinff<br />

cos Acos B cos Acos11<br />

. A MK<br />

_ ~-COs1f<br />

- .IDL1...sin..l!.<br />

1+ cosA ' cosB<br />

tanA-tan B<br />

= l+tanAtanB<br />

77. Letf(x) = sin x, Show that<br />

.::....:..-.....:.........:...."'--"- f(x+h)- f(x) = SIn . {COSh-I) + cos {Sinh) --.<br />

h . h h<br />

f(x+h)- f(x) sin(x+h)-sinx<br />

h<br />

h<br />

sinxcosh + sinh cosx - sin x<br />

h<br />

= SIn . {COSh-I) +cosx (Sinh) -­<br />

h<br />

h<br />

78. Since angle /3 and angle ABC are<br />

supplementary, the measure of angle ABC is<br />

180 0 - /3.<br />

79. a +(180 0 - /3)+ 9 = 180 0<br />

a-/3+9=0<br />

9=/3-a<br />

80. tan 9 = tan(/3 - a)<br />

_ tan /3- tan a<br />

- I + tan /3tan a<br />

82. x + y = 9, 2x + Y = -I<br />

Change the equations to slope-intercept form to<br />

find their slopes.<br />

x+y=9<br />

y=-x+9<br />

m2 =-1<br />

2x+ y =-1<br />

y=-2x-1<br />

ml =-2<br />

tan9= m2 -ml<br />

l+m2ml<br />

_ -1-(-2)<br />

- 1+ (-IX-2)<br />

I<br />

=-­<br />

1+2<br />

I<br />

=­<br />

3<br />

9=18.4 0<br />

83. 5x - 2y + 4 = O. 3x + 5y = 6<br />

Change the equations to slope-intercept form to<br />

find their slopes.<br />

5x-2y+4=0<br />

-2y=-5x-4 ·<br />

5<br />

y=-x+2<br />

2<br />

5<br />

m\=­ 2<br />

3x+5y =6<br />

5y=-3x+6<br />

3 6<br />

y=--x+­<br />

5 5<br />

3<br />

m2=-­ 5<br />

. tan /3- tan a<br />

81. From Exercise 80. tan 9 =---'---­<br />

1+ tan /3tan a<br />

Let tan a = m\ and tan /3 = m2 . Then<br />

tan 9 = m2 - ml .<br />

l+mlm2<br />

183


<strong>Trigonometry</strong> <strong>Solutions</strong> <strong>Manual</strong><br />

184 Chapter 5 Trigonometric Identities<br />

84. e=20sin(:t - ~)<br />

=2{sin(:t)cos( ~ ) - cos(:t)si{~)]<br />

= 2{sin( :t }O) _ cos( :t)(1)]<br />

(b)<br />

F =.6Wsin(0+90)0<br />

sin 12°<br />

= .6W(sin 0°cos 90° + sin 90° cos 0°)<br />

sin 12°<br />

=--Wcos .6 0<br />

sin 12°<br />

=2.9WcosO<br />

=-20CO{:t)<br />

85. (a) Graph y = 30 sin 120m + 40 cos 120m<br />

over the interval 0 :5 t :5 .05.<br />

For x = t,<br />

V = V 1 + V 2 = 30 sin 1201r1 + 40 cos 1201T1<br />

(c) F will be maximum when cosO = lor<br />

o= 0°. (0 = 0° corresponds to the back<br />

being horizontal which gives a maximum<br />

force on the back muscles. This agrees<br />

with intuition since stress on the back<br />

increases as one bends farther until the<br />

back is parallel with the ground.)<br />

Section 5.5<br />

Connections (page 221)<br />

-60<br />

(b) Using the TRACE feature on your<br />

calculator, the amplitude is 50 so let<br />

a = 50. Estimate the phase shift by<br />

approximating the first r-intercept where<br />

the graph of V is increasing. This is located<br />

at t = .0142.<br />

sin(120m + q,) = 0<br />

sin[1201r(.0142) + q,] = 0<br />

[1201r(.0142) + q,] = 0<br />

q, = -1201r(.0142)<br />

= -5.353<br />

Thus V = 50 sin (120m - 5.353).<br />

(c) 50 sin (l201r t - 5.353)<br />

= 50[sin 1201r t cos 5.353 - cos 1201rtsin5.353]<br />

= 50[sin 1201r t(.5977) - cos 1201rt(-.8017)]<br />

= 29.89sin 1201r t + 40.09 cos 1201rt<br />

= 30 sin 1201r t + 40 cos 1201r t<br />

I. Let B =A in the identity for sin A cos B.<br />

sin A cos A = .!.[sin(A + A)+ sin(A - A)]<br />

2<br />

sin Acos A = .!.[sin2A +sinO]<br />

2<br />

2sinAcosA = sin2A<br />

2. Let B =A in the identity for cos A cos B.<br />

1<br />

cos A cos A =-[cos(A + A) + cos(A - A)]<br />

2 _<br />

Exercises<br />

cos 2 A =.!.[cos2A + cos 0]<br />

2<br />

2cos 2 A = cos2A + 1<br />

2cos 2 A-I =cos2A<br />

F= .6W sin(0 + 90)0<br />

86. (a)<br />

sin 12°<br />

= .6(170)sin(30+90)0<br />

sin 12°<br />

=425 Ib<br />

(This is a good reason why people<br />

frequently have back problems.)<br />

184


<strong>Trigonometry</strong> <strong>Solutions</strong> <strong>Manual</strong><br />

5.5 Double-Angle Identities 185<br />

3 ..<br />

. 2tan-t 21C 8. cos 2a = 4"' a IS m quadrant III.<br />

6. D, 2 = tan- =-..J3<br />

I-tan -t 3<br />

cos2a = 2cos 2 a-I<br />

3 2<br />

-=2cos a-I<br />

7. cos 2()= ~ , () is in quadrant I. 4<br />

cos 2()= 2cos 2 ()-I 2cos 2 a .i<br />

4<br />

~ =2cos 2 ()-I 2 7<br />

cos a=­<br />

5 8<br />

2cos 2 () = ~ + 1= ! Since a is in quadrant III, cos a < 0<br />

5 5<br />

cos 2 () = .!- = i<br />

10 5<br />

Since () is in quadrant I, cos () > o.<br />

2 2../5<br />

cose= ../5 = -5-<br />

Since () is in quadrant I, sin () > o.<br />

cosa=-~ =- 2~ =-~<br />

Since a is in quadrant III, sin a < 0 .<br />

sina=-~I-cos2a<br />

=-~I-i<br />

sin() = ~1-cos2 e<br />

=-~<br />

1<br />

=~ =-2Ji<br />

=H<br />

1<br />

../2<br />

=-­<br />

4<br />

=.J5<br />

tana=-=!­<br />

.J5<br />

=-<br />

5<br />

../2<br />

=-:Ji4<br />

tan() _ sin() _ :fl _1<br />

-4<br />

- cos() - 2# -'2 =7]<br />

1 1 ..f7<br />

cot()=--= - =2 =­<br />

tan() t 7<br />

1 1<br />

1 1 5 ../5 cot a =--= r;:; = .fi<br />

sec()=--=-=--=- tana ",7<br />

cos() ¥ 2../5 2<br />

1 2../2 2$4<br />

1 1 sec a = cosa = ---::J7'"= --7­<br />

csc()=-= rc =../5<br />

sin () :t1.<br />

5 1 2../2<br />

csc a =--=---= -2../2<br />

sina 1<br />

1<br />

185


<strong>Trigonometry</strong> <strong>Solutions</strong> <strong>Manual</strong><br />

186 Chapter 5 Trigonometric Identities<br />

5 1C 2<br />

9. cos2x=-- -


<strong>Trigonometry</strong> <strong>Solutions</strong> <strong>Manual</strong><br />

­<br />

1 3<br />

cos2x =--=-­<br />

sec2x 5<br />

tan 2a = sin 2a = - ~ = _ 4../55<br />

1 3<br />

cos2a 19 39<br />

cot2x=--=-­<br />

tan2x 4<br />

1 39../55<br />

cot2a=--=--­<br />

sin2x = tan2xcos2x = (-~X-~) = ~<br />

tan2a 220<br />

1 49<br />

1 5<br />

sec2a=--=cos2a<br />

39<br />

csc2x=--=sin2x<br />

4 1 49../55<br />

csc2a=--=--­<br />

sin2a 220<br />

5.5 Double-Angle Identities 187<br />

12. cos /3= _E.., sin /3 > 0<br />

14. tan x = -,<br />

5. 0<br />

SIOX <<br />

13<br />

3<br />

sin /3 = ~1- cos 2 2<br />

/3<br />

2 2 5 34<br />

sec x = 1+ tan x = 1+ ( "3 ) = "9<br />

Since sin x < 0, and tan x > 0, x is in quadrant<br />

=~I-(-~~r<br />

ITI, so cos x and sec x are negative.<br />

5<br />

=­<br />

~<br />

secx=--­<br />

13<br />

3<br />

sin 2/3= 2 sin /3 cos /3<br />

3 ~<br />

12)<br />

cosx =---=--­<br />

=2(2.X_<br />

~ 34<br />

13 13<br />

120<br />

sinx=cosxtanx=(- ~X~)=-~<br />

=-­<br />

34 3 34<br />

169<br />

cos2/3 = 2cos 2 /3-1<br />

sI02x=2smxcosx=2 · . (5~X3~) -- -­ =- 15<br />

34 34 17<br />

=2(- ~~r -1<br />

cos 2x = 2cos<br />

2<br />

x -1 = 2 (9) 34 -1 = - 17 8<br />

119<br />

=­<br />

169<br />

15<br />

tan2x =-­<br />

tan 2/3= sin2/3 = _ 120<br />

8<br />

cos2{3 119<br />

cot2x=-­<br />

119<br />

15 8<br />

cot2{3=-­<br />

17<br />

120<br />

sec2x=-­<br />

169<br />

8<br />

sec2/3=­<br />

17<br />

119<br />

csc2x=­<br />

169<br />

csc2/3=-­<br />

120<br />

15. slOa=-7' · J5 cosa> 0 .<br />

13. tan x = 2, cos x > 0<br />

2tanx<br />

tan2x = 2<br />

2 . 2<br />

1- tan x<br />

cos a=l-sm a=l-(J5)2 -7 = 44 49<br />

2(2) 4<br />

=1-(2)2 =-3"<br />

Since cos a > 0,<br />

Since both tan x and cos x are positive, x must<br />

cos a = (44 = .J44 = 2..JIT .<br />

be in quadrant I. Thus, 2x must be in<br />

""49 7 7<br />

quadrant ll.<br />

2 2 16 25<br />

sec 2x = 1+ tan 2x = 1+ - =<br />

9 9<br />

5<br />

sec2x=-­<br />

3<br />

cos2a = 1-2sin 2 a = 1-2(J5)2 = 39<br />

7 49<br />

sI02a=2smacosa=2 · . (J5X2..JIT)=---<br />

-­ 4../55<br />

7 7 49<br />

187


<strong>Trigonometry</strong> <strong>Solutions</strong> <strong>Manual</strong><br />

188 Chapter 5 Trigonometric Identities<br />

Jj. 0<br />

16. cosa=-, smrz ><br />

5<br />

s;na=~1-cos2a =~1-( "1)' = ~<br />

sin2a = 2sinacosa = 2( ~)( ~) = 2:<br />

cos2a = 2cos 2 a-I = 2(Jj)2 -1 =_.!2.<br />

5 25<br />

25• 2 cos 267Io_I=cos267{_s\'n267'!'o<br />

­<br />

2 2 2<br />

26.<br />

=COS2(67{)<br />

= cos 135°<br />

..fi<br />

=-­ 2<br />

tan 2a = sin 2a = _ 2../66<br />

cos2a 19<br />

19 19../66<br />

cot2a=---=--­<br />

2../66 132<br />

sec2a = _ 25<br />

19<br />

25 25../66<br />

csc2a=--=-­<br />

2../66 132<br />

17. sin2X=2sinXcosX<br />

sin Xcos X = .!.sin 2X<br />

2<br />

If sin 2X = .4. then sin XcosX = .!.(.4) =.2.<br />

2<br />

Therefore, the screen will display .2 for the<br />

final expression.<br />

18. cos(2X) = 1-2 sin 2 X<br />

2sin 2 X=I-cos2X<br />

If cos (2X) = .3, then 2sin 2 X = 1-.3 = .7.<br />

Therefore. the screen will display .7 for the<br />

final expression.<br />

tan51°<br />

27.<br />

I-tan 251°<br />

. 2tanA<br />

Since 2 = tan 2A. then<br />

I-tan A<br />

28.<br />

.!.( 2 tan A ) =.!. tan 2A<br />

2 I-tan 2 A 2<br />

tan A 1<br />

--~2- = - tan 2A.<br />

I-tan A 2<br />

tan51° =~tan2(51°)<br />

I-tan 251° 2<br />

1<br />

= - tan(102°)<br />

2<br />

22.<br />

2 tan 15° tan 15°+ tan15°<br />

1-tan 2 15° - 1-tan 2 15°<br />

= tan(15°+15°)<br />

= tan 30°<br />

1<br />

=Jj<br />

Jj<br />

=­ 3<br />

29. .!.-.!.sin 2 47.1° = .!.(I-2sin 2 47.1°)<br />

424<br />

Since cos2A = 1-2sin 2 A,<br />

~(I-2sin 2 47.1°) = ~cos 2(47.1°)<br />

1<br />

= -cos94.2°.<br />

4<br />

. 2 1° (1°) ..fi<br />

24. 1-2sm 22- = cos2 22- = cos 45° =­<br />

222<br />

188


<strong>Trigonometry</strong> <strong>Solutions</strong> <strong>Manual</strong><br />

29.5° cos 29.5°<br />

2 21r<br />

5-cos 5=<br />

cos 2 A - sin<br />

2; _ sin2<br />

2a-sin 2 2a<br />

cos 5x<br />

sin A cos<br />

cos 5x =<br />

-1 = cos<br />

= sin 90°<br />

= 2 sin<br />

=2('7.<br />

=1<br />

= cos 90°<br />

= 2 cos<br />

={'7r<br />

=0<br />

cos2(600)=cos1200=-­<br />

= cos 2<br />

=(~r<br />

1<br />

=-­<br />

2<br />

5.5 Double-Angle Identities<br />

= tan 120° =-.J3<br />

= 2 tan 60°<br />

1- tan 2 60°<br />

2.J3<br />

= 1_(.J3)2<br />

2.J3<br />

=-­<br />

-2<br />

=-./3<br />

= cos 1~1r<br />

41r<br />

=cos­<br />

3<br />

1r<br />

=-cos­<br />

3<br />

1<br />

::;;;;:-­<br />

2<br />

= cos 2 ( 5;)~sin2(5;)<br />

= cos 2( ~)-(-sin ~r<br />

=(~r -(- ~r<br />

1 3<br />

=--­<br />

4 4<br />

2<br />

=-­<br />

4<br />

1<br />

=-­<br />

2<br />

sin 21r = .J3<br />

3 2<br />

= 2' sm-cos­ 1r 1r<br />

3 3<br />

={~X~)<br />

.J3<br />

=­<br />

2<br />

189<br />

30. .!. sin = J.... (2 sin 29.5° cos 29.5°) 38. tan 2(60°)<br />

8 16<br />

tan 2(600)<br />

= J....sin2(29 .5°)<br />

16<br />

= J....sin59°<br />

16<br />

. 2 21r -( 2 21r . 2 21r)<br />

31. sm cos 5-sm 5<br />

Since 2 A = cos 2A •<br />

-(cos 2 2;) = -cos(2 . 2;)<br />

39. cos 2(5;)<br />

41r<br />

=-COS-.<br />

5<br />

32. cos 2 = cos2(2a)= cos4a<br />

33. 2 sin 5x<br />

Since 2 A = sin 2A.<br />

2 sin 5x sin 2(5x) = sin lOx.<br />

34. 2 cos 2 6a 2(6a)= cos 12a<br />

COS2( 5;)<br />

35. sin 2(45°) = 1<br />

sin 2(45°) 45° cos 45°<br />

'7)<br />

36. cos 2(45°) = 0<br />

cos 2(45°) 2 45° -1<br />

-1<br />

40. sin 2(1r) =<br />

3<br />

sm . 2(1r) -<br />

1<br />

3<br />

37.<br />

2<br />

cos 2(60°) 60° -sin 2 60°<br />

-( ~r<br />

189


<strong>Trigonometry</strong> <strong>Solutions</strong> <strong>Manual</strong><br />

190 Chapter 5 Trigonometric Identities<br />

41. tan2(-~)=tan(_2;)=~ 45. sin2(- 1~1r) == sin(-ll1r) = 0<br />

tan2(- ~ ) = - tan2( ~ ) sin2(_ I:) = 2 Sin(_ 1~1r ) co{_ 1~1r )<br />

2tan1<br />

= l-tan<br />

2<br />

t<br />

=0<br />

=<br />

2~<br />

1_(~)2<br />

= 2(1)(0)<br />

46. Sin2( _1~1r)=Sin(-171r)=O<br />

2~ Sin2(_ 1~1r) = 2Sin(_ 1~1r )co{_ 1~1r)<br />

=--­<br />

1-3 = 2(-1)(0)<br />

=~ =0<br />

42. cos2(- 9:) = cos( _ 9;) = 0<br />

47.<br />

cosz(- 9:) = 2COS 2( _ 9:)_1<br />

=2('7r -I<br />

=0<br />

43. tan2(- ~) = tan(_ 8;) = ~ The graph of y = cos 4 x - sin 4 x appears tobe<br />

tan2(- ~) = - tan 2( 4;)<br />

the same as the graph of y = cos 2x.<br />

cos 4 x-sin 4 x = {cos 2 x+sin 2 xXcos2 x-sin 2 x)<br />

2 tan-¥­<br />

= l·cos2x<br />

= =cos2x<br />

l-tan 2 -¥<br />

2~ 48. 4tan x cos2 x - 2tan x<br />

= 1 - tao 2 x<br />

I-(~f<br />

2~<br />

=--­<br />

1-3<br />

=~<br />

4tanxcos2 x-2tanx<br />

The grap h 0 f y = 2 appears<br />

1- tan x<br />

to be the same as the graph of y = sin 2x.<br />

2<br />

4 tan xcos x-2tanx 2tanx (2 2 I)<br />

-------:;---- = 2 cos x-.<br />

1-tan 2 x 1-tan x<br />

= tan 2x cos 2x<br />

sin2x<br />

=---cos2x<br />

cos2x<br />

=sin2x<br />

190


<strong>Trigonometry</strong> <strong>Solutions</strong> <strong>Manual</strong><br />

5.5 Double-Angle Identities 191<br />

49.<br />

cot 2 x-I<br />

The graph of y = appears to be the<br />

2cotx<br />

same as the graph of y = cot 2x.<br />

cot 2 x-I ~-1<br />

= ---.L<br />

2cotx tanx<br />

2<br />

(± - 1)tan x<br />

_<br />

lan x<br />

-<br />

(!iih ) tan 2 x<br />

1- tan<br />

2<br />

x<br />

=<br />

2tanx<br />

1<br />

-l::/X<br />

1<br />

=-­<br />

tan2x<br />

=cot2x<br />

50. 2tanx<br />

2 - sec2x<br />

2tanx<br />

The graph of y = 2 appears to be the<br />

2-sec x<br />

same as the graph of y = tan 2x.<br />

2tan x 2tanx<br />

2 - sec 2 x := 1-(sec 2 x -1)<br />

2tanx<br />

-1-tan 2x<br />

= tan2x<br />

51. Verify (siny+cosyf =sin2y+1.<br />

(siny+cosy)2 = sin 2 y + cos 2 Y+ 2sinycosy<br />

=1+sin2y<br />

2 4<br />

52. Verify en secx= 2 sec x+sec x<br />

4<br />

"<br />

2+sec 2 x-sec x<br />

Work with the right side.<br />

2 4 (-.l..-+ 1)cos 4 x<br />

sec x+sec x cos 2 x ~<br />

2 4 = (<br />

2 + sec x - sec x 2 + ---l..- - -.l..-) cos" X<br />

cos' x<br />

cos'' x<br />

cos 2 x+l<br />

=--.,....----::--­<br />

2cos 4 x+cos 2 x-I<br />

cos 2 x+l<br />

2<br />

- (2cos 2 x-IXcos x + I)<br />

1<br />

=----:::--­<br />

2cos 2 x-I<br />

I<br />

=-­<br />

cos2x<br />

= sec2x<br />

53. Verify tan 8k - tan 8k tan 2 4k = 2 tan 4k .<br />

tan 8k - tan 8k tan 2 4k = tan 8k(l- tan 2 4k)<br />

54. Ven<br />

ify "2 2tany<br />

SID y = 2"<br />

l+tan y<br />

Work with the right side.<br />

2tany 2tany<br />

1+tan 2y = l+sin~Y<br />

cosy<br />

2 sinr<br />

_ cosr<br />

- cos 2 r+sin2r<br />

cos2r<br />

=2sinycosy<br />

=sin2y<br />

2-sec 2 y<br />

55. Verify cos2y = 2<br />

sec y<br />

Work with the right side.<br />

(2--1-)COS 2<br />

2 - sec 2 y cos! y Y<br />

2<br />

sec Y<br />

= (_I'_)COS 2 Y<br />

cos- y<br />

:=2cos 2 y-l<br />

=cos2y<br />

= 2 tan 4k (1_ tan24k)<br />

l-tan 24k<br />

=2tan4k<br />

191


<strong>Trigonometry</strong> <strong>Solutions</strong> <strong>Manual</strong><br />

192 Chapter 5 Trigonometric Identities<br />

. 2tanO<br />

56. Venfy -tan20= 2 .<br />

sec 0-2<br />

Work with the right side.<br />

2tanO 2tanO<br />

sec 2 0-2 - (1+ tan 2 0)-2<br />

2tanO<br />

- tan 2 0-1<br />

=-tan20<br />

57. Verify sin4a = 4sinacosacos2a.<br />

Work with the right side.<br />

4sinacosacos2a = 2(2 sin acosa)cos2a<br />

= 2sin2acos2a<br />

= sin2(2a)<br />

= sin4a<br />

ify l+cos2x<br />

58. Ven =cot r ,<br />

sin2x<br />

2x-I)<br />

l+cos2x = 1+(2cos<br />

sin2x<br />

2sinxcosx<br />

2cos 2 x<br />

= 2sinxcosx<br />

cosx<br />

=-­<br />

sinx<br />

=cotx<br />

59. Verify tan(O - 45°) + tan(O+ 45°) = 2 tan 20 .<br />

tan(0 - 45°) + tan(0 + 45°)<br />

tan 0 - tan 45° tan0 + tan45°<br />

------+-----­<br />

1+ tan0 tan 45° 1- tan0 tan45°<br />

tan 0 - I tan 0 + I<br />

= +--­<br />

I + tan8 I - tan 0<br />

tan 0 -I tan 0 + I<br />

= tanO+1 tanO-1<br />

tan 2 0-2tanO+1-tan 2 0-2 tanO-1<br />

=<br />

(tanO+IXtanO-I)<br />

-4 tan0<br />

=-..."...-­<br />

tan<br />

20-1<br />

4tanO<br />

= l-tan 2 0<br />

2(2 tan0)<br />

=1-tan 20<br />

= 2tan20<br />

. l-tan 220<br />

60. Venfy cot40=---­<br />

2tan20<br />

I<br />

cot40=-­<br />

tan40<br />

I<br />

2 ran 26<br />

l-tan 226<br />

2cos2a<br />

61. Verify =cota-tana.<br />

sin2a<br />

Work with the right side.<br />

cosa sina<br />

cot a - tana =----­<br />

sina<br />

cosa<br />

cos 2 a - sin 2 a<br />

=<br />

sinacosa<br />

= 2(cos 2 a-sin 2 a)<br />

2sinacosa<br />

2 cos2a<br />

= sin2a<br />

62. Verify sin4r = 4sinrcosr - 8sin 3 rcosr .<br />

sin4r = 2sin2rcos2r<br />

= 2(2 sin rcos r)(I- 2sin 2 r)<br />

= 4sin rcosr - 8sin 3 rcosr<br />

63. Verify sin 2acos2a = sin 2a - 4 sin 3 a cos a .<br />

sin2acos2a = (2sinacosa~I-2sin2 a)<br />

. l-tan 2 x<br />

64. Venfy cos2x = 2'<br />

I+tan x<br />

Work with the right side.<br />

. 2<br />

I 2 I-~<br />

- tan x cos x<br />

1+ tan 2 x = 1+"*<br />

cos x<br />

cos 2 x-sjn 2 x<br />

_ cos2x<br />

- cos 2 x+sin 2 x<br />

2<br />

cos x<br />

=cos 2 x-sin 2 x<br />

=cos2x<br />

= 2sinacos~ -4sin 3 acosa<br />

= sin2a -4sin 3 acosa<br />

192


<strong>Trigonometry</strong> <strong>Solutions</strong> <strong>Manual</strong><br />

5.5 Double-Angle Identities 193<br />

65. Verify tan s + cot s = 2 esc 2s . 68. Verify<br />

sins coss<br />

coss sins<br />

sin 2 s + cos 2 S<br />

tanO+tann<br />

=<br />

tan(0+ n) =----­<br />

cosssins<br />

l-tanOtann<br />

tanO+O<br />

I<br />

=<br />

=<br />

l-tanO ·O<br />

cosssins<br />

=tanO<br />

2<br />

= sin(n-O) = sin ncosO - cosnsinO<br />

2cosssins = O· cosO+ I ·sinO<br />

2<br />

=-­<br />

=sinO<br />

sin2s<br />

= 2csc2s 222<br />

= 0 .cos 0 + I .sin 0<br />

cota-tana 2<br />

66. Verify = cos a.<br />

cota+tana<br />

=sinO<br />

tan s + cot s =--+-­ cotOtan(O+n)-sin(n - O)co{~ -0) = cos ()<br />

cos(n - 0) = cos n cos0+ sin!!..sin0<br />

cota - tan a ~- sina cotOtan(o+n)-sin(n-O)co{~-0)<br />

sma cosa<br />

cota+tana ~~~g + ~~~ = cot 0 tan 0 - sin OsinO<br />

cos 2 a-sin 2 a = l-sin 2 0<br />

= cos 2 a+sin 2 a = cos 2 0<br />

= cos 2 a -sin 2 a<br />

=cos2a<br />

In Exercises 69-72, other forms may be possible.<br />

67. Verify I + tan x tan 2x = sec2x.<br />

1+tanxtan2x = 1+ tanx( 2 tan: )<br />

I-tan x<br />

69. cos3x<br />

=cos(2x+x)<br />

= cos2xcosx -sin 2xsinx<br />

2 tan 2 x = (1- 2sin 2 x)cosx - (2sin xcosx)sinx<br />

I +--....,.....­<br />

- l-tan 2 x =cosx-2sin 2 xcosx-2sin 2 xcosx<br />

l-tan 2 x+2tan 2 x<br />

= = cosx-4sin 2 xcosx<br />

l-tan 2 x<br />

l+tan 2 x<br />

=cosx(l-4sin 2 x)<br />

-1-tan 2x<br />

=cosx[l-4(I-cos 2 x)]<br />

005 2 X:,in2 x<br />

_ cos x =cosx[-3+4cos 2 x]<br />

- cos 2 x-sin 2 x<br />

cosFx<br />

I<br />

=-­<br />

cos2x<br />

=sec2x<br />

=-3cosx+4cos 3 X<br />

70. sin 4x = 2sin 2x cos 2x<br />

=2(2sinxcosx)(cos 2 x-sin 2 x)<br />

=4sinxcos 3 x-4sin 3 xcosx<br />

71. tan 3x = tan(2x + x)<br />

tan 2x+ tan x<br />

= I - tan 2x tan x<br />

~+tanx<br />

_ I-tan x<br />

-I-~tanx<br />

I-tan x<br />

2 tan x + tan x - tan 3 x .<br />

= I - tan 2 x - 2 tan 2 x<br />

3tanx-tan 3 x<br />

=----..,.,-­<br />

1-3tan 2 x<br />

2<br />

193


<strong>Trigonometry</strong> <strong>Solutions</strong> <strong>Manual</strong><br />

194 Chapter 5 Trigonometric Identities<br />

72. cos4x = 2 cos 2 2x-l<br />

=2(2coS 2 X-It -1<br />

=2(4COS 4 x-4cos 2 x+l)-1<br />

=8cos 4 x-8cos 2 x+l<br />

73. ¢ = 47° 12', h = 387.0 ft<br />

Use a calculator and substitute these values in<br />

g =978.0524(1 + .005297sin 2 ¢<br />

- .0000059 sin 2 2¢) - .()()()()94h.<br />

g =978.0524[1 +.005297sin 2 47°12'<br />

- .OOOOO59sin 2(2 .47°12')]<br />

- .000094(387.0)<br />

=980.799 em per sec<br />

2<br />

74. ¢ = 68° 47' = 68.78333°, h = 1145 ft<br />

Use a calculator and substitute these values in<br />

g =978.0542(1 + .005297sin 2 ¢<br />

-.OOOOO59sin 2 2¢) - .000094h.<br />

g = 978.052~1 + .005297 sin 2 68.78333°<br />

- .OOOOO59sin 2(2 .68.78333 0 ) ]<br />

- .000094(1145)<br />

=982 .444 em per sec 2<br />

75. cos 45° sin 25°<br />

=.!.[sin(45° + 25°) - sin(45° - 25°)]<br />

2<br />

= .!.(sin70° -sin20°)<br />

2<br />

76. 2sin74°cosH4°<br />

= 2·.!.[sin(74° + 114°)+ sin(74° -114°)]<br />

2<br />

=sin 188°+ sin(-400)<br />

= sin 188°+ (-sin 40°)<br />

= sin 188° - sin 40°<br />

77. 3 cos 5x cos 3x<br />

=3· .!.[cos(5x +3x)+ cos(5x - 3x)]<br />

2<br />

3<br />

=-(cos8x+ cos2x)<br />

2<br />

78. 2 sin 2x sin 4x<br />

=2 ..!.[cos(2x - 4x) - cos(2x +4x)]<br />

2<br />

=cos(-2x)-cos6x<br />

=cos 2x - cos 6x<br />

79. sin(-8)sin(-38)<br />

=~[CO~-8 - (-38)] - co~-8 + (-38)])<br />

=.!.[cos(28) - cos( --48)]<br />

2<br />

1<br />

=-(cos28 - cos 48)<br />

2<br />

80. 4 cos 8a sin(--4a )<br />

= 4 ·±[sin[8a + (--4a)]-sin[8a - (--4a)]J<br />

=2(sin 4a - sin 12a)<br />

= 2sin4a - 2sin12a<br />

81. -8 cos 4y cos 5y<br />

=-8· '2 1 [cos(4y + 5y)+ cos(4y-5y)]<br />

= --4[cos9y+ cos(-y)]<br />

=--4(cos9y+ cosy)<br />

82. 2 sin 3k sin 14k<br />

1<br />

=2 ·-[cos(3k -14k) -cos(3k + 14k)]<br />

2<br />

=cos(-llk)- cos(17k)<br />

83. (a)<br />

= cosHk -cos 17k<br />

=<br />

32<br />

2v 2 sin8cos8<br />

=<br />

32<br />

=v 2 sin(28)<br />

32<br />

Which is dependent on both the velocity<br />

and angle at which the object is thrown.<br />

(b) D = (36)2 sin(2· 30)<br />

32<br />

= (1296)(0.866)<br />

32<br />

"" 35 ft<br />

194


<strong>Trigonometry</strong> <strong>Solutions</strong> <strong>Manual</strong><br />

5.6 Half-Angle Identities 195<br />

84. From Example 6,<br />

(163sin 1201l' 1)2<br />

W =-'------'-­<br />

15<br />

W "" 1771.3(sin 1201l'1)2.<br />

Thus,<br />

1771.3(sin 1201l'1)2<br />

=1771.3sin 1201l' 1.sin 1201l'1<br />

=(1771.3)(~)[cos(1201l'1-1201l'1)<br />

-cos(1201l'1 + 1201l'1)]<br />

=885.6(cos 0 - cos 2401l' 1)<br />

=885.6(1- cos 2401l' 1)<br />

=-885.6 cos 2401l' 1+ 885.6.<br />

If we compare this to W =a cos(lOt) +c,<br />

then a = -885.6, m =2401l', and c = 885.6. See<br />

the answer graph in the back of the textbook.<br />

85. (a) Graph W =VI =[163sin(1201l'1)]<br />

[1.23sin(1201l'1)] over the interval<br />

0::; 1 ~ .05.<br />

For x = t, W = VI =<br />

(163 sin 120711)(1.23 sin 120171)<br />

3OOr1/~~70 77:'<br />

-so<br />

(b) The minimum wattage is 0 and the<br />

maximum wattage occurs whenever<br />

sin(1201l'1)=1. This would be<br />

163(1.23) = 200.49 watts.<br />

(c) [I63sin(1201l'1)I 1.23sin(1201l'1)]<br />

=200.49sin 2(1201l' 1)<br />

=2oo.49[~(I- cos 2401l' 1)]<br />

=-100.245 cos 2401l' 1+ 100.245<br />

Then a =-100.245, m =2401l', and<br />

c =100.245.<br />

(d) The graphs of<br />

W =[I63sin(1201l'1)11.23sin(1201l'1)] and<br />

W =-100.245 cos 2401l' 1+ 100.245 are the<br />

same.<br />

(e) The graph of W is vertically centered<br />

about the line y = 100.245. An estimate for<br />

the average wattage consumed is<br />

100.245 watts. (For sinusoidal current, the<br />

average wattage consumed by an<br />

electrical device will be equal to half of<br />

the peak wattage.)<br />

86. (a) The period is equal to<br />

21l' 21l' I<br />

-=--=b<br />

21l'm m<br />

Section 5.6<br />

Exercises<br />

1. Since 195° is in quadrant III and since the sine<br />

is negative in quadrant Ill, use the negative<br />

square root.<br />

2. Since 58° is in quadrant I and since the cosine<br />

is positive in quadrant I, use the positive square<br />

root.<br />

3. Since 225° is in quadrant III and since the<br />

tangent is positive in quadrant III, use the<br />

positive square root.<br />

4. Since -10° is in quadrant IV and since the sine<br />

is negative in quadrant IV, use the negative<br />

square root.<br />

s, c;<br />

. 150 ~1-COS300<br />

SIO =<br />

2<br />

=~1-24<br />

=~2-~<br />

2 ·2<br />

=-=---=,;.-­<br />

~2-~<br />

.J4<br />

=-=-_.:...­ ~2-~<br />

2<br />

0 _1_-_c_os_3_0_0<br />

6. A; tan 15 = sin 30°<br />

I_di<br />

=-t<br />

2-~ 2<br />

=-_.­<br />

2 I<br />

=2-~<br />

195


<strong>Trigonometry</strong> <strong>Solutions</strong> <strong>Manual</strong><br />

196 Chapter 5 Trigonometric Identities<br />

7. D;<br />

Jt+COS"<br />

8 2<br />

=~1+24<br />

x<br />

cos-= 4<br />

=~2+.J2".!.<br />

2 2<br />

~<br />

- -J4.<br />

=~<br />

2<br />

10.<br />

B; cos 67.5° = 1+ cos 135°<br />

2<br />

=Jl+(~f)<br />

=~2-.J2 ..!.<br />

2 2<br />

-~<br />

- -J4<br />

=~<br />

2<br />

8. E;<br />

9. F;<br />

tan(- 1C) _ 1-cos(-~)<br />

8 _ ( 1C)<br />

SIn -'4<br />

_ l-cos(lf)<br />

- sin(lf)<br />

_1-4<br />

r:»:<br />

2<br />

_2-.J2 2<br />

--2---.J2<br />

_ -(2 - .J2) .J2<br />

- .J2 "12<br />

-2.J2 +2<br />

=<br />

2<br />

=1-.J2<br />

tan 67.5° = I-cos 135°<br />

sin 135°<br />

_ 1-(-4)<br />

-::li<br />

2<br />

_2+.J2 2<br />

--2-'.J2<br />

2+.J2 .J2<br />

= .J2 '.J2<br />

2.J2 +2<br />

=<br />

2<br />

=1+.J2<br />

11. sin 67.5° =sin(13:0)<br />

Since 67.5° is in quadrant I, sin 67.5° > O.<br />

sin 67.5° = 1- cos 135°<br />

2<br />

l+cos45°<br />

2<br />

=~1+2"f<br />

=~2+4.J2<br />

=~<br />

2 .<br />

12. sin 195° =sine~OO)<br />

Since 195° is in quadrant III. sin 195° < 0,<br />

sin 195° =_~I-CO;3900<br />

=_~I-C~S300<br />

=_~1-2~<br />

=_~2~~<br />

-s:»<br />

2<br />

196


<strong>Trigonometry</strong> <strong>Solutions</strong> <strong>Manual</strong><br />

5.6 Half-Angle Identities 197<br />

13.<br />

14.<br />

15.<br />

3900)<br />

cos195° = cos -2­ (<br />

Since 195° is in quadrant III, cos 195° < O.<br />

1+cos3900<br />

cos 195° = -./----­<br />

2<br />

1+ cos 30°<br />

=­<br />

2<br />

=_~1+2,Jf<br />

=_~2+4.[3<br />

-~2+.[3 =----:._-­<br />

2<br />

3900)<br />

tan 195° = tan -2­ (<br />

sin 390°<br />

= 1+ cos 390°<br />

sin 30°<br />

= 1+ cos 30°<br />

- !<br />

-~<br />

T<br />

1<br />

=2+.[3<br />

1 2-.[3<br />

= 2+.[3 ' 2-.[3<br />

2-.[3<br />

=--­<br />

1<br />

=2-.[3<br />

3300)<br />

cos 165° :::; co -2­ {<br />

Since 165° is in quadrant II, cos 165° < O.<br />

1+cos330°<br />

cos165° =-./----­<br />

2<br />

1+ cos 30°<br />

=<br />

16. sin 165° =sine~~O)<br />

Since 165° is in quadrant II, sin 165° > O.<br />

sin 1650=~1-CO;3300<br />

=l-C~S300<br />

=~1-2,Jf =~2-4..J3<br />

~2-.[3<br />

=-=---~<br />

2<br />

17. tan -<br />

x )<br />

=--­<br />

1-cosx<br />

( 2 sinx<br />

If cos x = .9682458366 and sin x = .25. then<br />

tan(~) =1-.9682458366<br />

2 .25<br />

=.1270166536<br />

Therefore, the screen will display .1270166536<br />

for the final expression if no round off errors are<br />

introduced.<br />

Most trigonometric function values are<br />

transcendental numbers. that is. they are<br />

nonrepeating, nonterrninating decimals. When<br />

we use the value given on the calculator screen<br />

for these numbers, we are not using their exact<br />

value; instead. we are.using a rounded form of<br />

the number. Since the calculator actually stores<br />

more significant digits than is displayed. round<br />

off errors can be introduced when we use only<br />

the numbers shown on the display.<br />

18. tan(~) =_l_-_c_os_x_<br />

2 sin x<br />

If cos x = -.75 and sin x = .6614378278, then<br />

x) 1-(-.75)<br />

(<br />

tan "2 = .6614378278<br />

=2.645751311.<br />

Therefore. the screen will display 2.645751311<br />

for the final expression if no round off errors are<br />

introduced.<br />

Most trigonometric function values are<br />

transcendental numbers. that is. they are<br />

nonrepeating, nonterminating decimals. When<br />

we use the value given on the calculator screen<br />

for these numbers. we are not using their exact<br />

value; instead. we are using a rounded form of<br />

the number. Since the calculator actually stores<br />

more significant digits than is displayed. round<br />

off errors can be introduced when we use only<br />

the numbers shown on the display.<br />

197


<strong>Trigonometry</strong> <strong>Solutions</strong> <strong>Manual</strong><br />

198 Chapter 5 Trigonometric Identities<br />

20. Show that .fi -1 =~3 - 2.fi ' Notice that<br />

.J2 -1>0 and ~3-2.fi >0.<br />

(.fi-If =2- z-Ii +1=3- 2.J2<br />

(~3-2-J2Y =3-2-J2<br />

If a> 0 and b > 0 and a 2 =b 2 • the a =b. Since both<br />

are positive and their squares are equal, by the above<br />

theorem, -J2 -1 =~3 - 2.J2 '<br />

21. Find cos ~, given cos 0 = .!., with 0 < 0 < 1r ,<br />

242<br />

0


<strong>Trigonometry</strong> <strong>Solutions</strong> <strong>Manual</strong><br />

5.6 Half-Angle Identities 199<br />

25. Find sin a , given tan a = 2, with 0 < a O.<br />

sec 2 a = tan 2 a + 1<br />

Thus, cos a is positive.<br />

2<br />

cos-= a ~I +cosa<br />

2 2<br />

=(2)2+ 1<br />

=5 ~~I-:<br />

seca={5<br />

1<br />

cos a = see a<br />

1.J5<br />

= {5 =5<br />

1f a 1f<br />

O


<strong>Trigonometry</strong> <strong>Solutions</strong> <strong>Manual</strong><br />

200 Chapter 5 Trigonometric Identities<br />

28. Find cot fJ if tan fJ = - ..[5, with<br />

2 2<br />

90° < fJ < 180°.<br />

Use identities to find sin fJ and cos fJ .<br />

sec' p= I+tao' p= 1+(-~r =~<br />

Since 90° < fJ < 180°. sec fJ < O.<br />

. 3<br />

secfJ=-­ 2<br />

2<br />

cosfJ=-­ 3<br />

sin fJ = cos fJ tan fJ = ( - ~X-V;J= ~<br />

fJ<br />

cot =~<br />

2 tanT<br />

= 1+cosfJ<br />

sinfJ<br />

1+(-t)<br />

= ~<br />

1<br />

=..[5<br />

..[5<br />

=­ 5<br />

29. Find sin8, given cos28=~, 8 is in quadrant I.<br />

5<br />

Since 8 is in quadrant I. sin 8 > O.<br />

sin 8 = sin 1.(28)<br />

2<br />

=l-~S28<br />

=~1~~<br />

=..[5<br />

5<br />

30. Find cos 8. given cos 28 = .!., 8 is in<br />

2<br />

quadrant II. Since 8 is in quadrant II, cos8 is<br />

negative.<br />

31• FIn<br />

cos8 = _~1 + C;S28<br />

=_~I:t<br />

=-~<br />

.J3<br />

=-­ 2<br />

· d cos x, given . cos 2x =--, 5 -


<strong>Trigonometry</strong> <strong>Solutions</strong> <strong>Manual</strong><br />

5.6 Half-Angle Identities 201<br />

38. sin 158.2° = tan 158.2° = tan 79.10<br />

1+cosI58.2° 2<br />

46.<br />

1+ cos18x 18x 9<br />

39. ± =cos-=cos x<br />

2 2<br />

1+ cos 20a 2Da 10<br />

40. ± = cos-- = cos a<br />

2 2<br />

l-cosSO 80<br />

41. ± = tan- = tan40 1-cosx<br />

l+cos80 2 The graph of y = . appears to be the<br />

smx<br />

l-cos5A 5A<br />

x<br />

same as the graph of y =tan-.<br />

42. ± = tan­<br />

1+cos5A 2<br />

2<br />

1- cos x _ 1- cos 2(t )<br />

l+cos(t) t x sin x - sin 2(t)<br />

43. ± = cos- = cos­<br />

2 2 8 = 1-[1-2sin2t]<br />

2sinfcost<br />

~ 1- cos 1Jl. .3fl. 30<br />

44. ± 2 5 = sin t = sin 10 2sin 2 f<br />

=------'=­<br />

2sintcosf<br />

45.<br />

sin x<br />

=--"£.<br />

cost<br />

=tant<br />

47. tan£-cot.:!.<br />

2 · 2<br />

cot':!' - tan .:!.<br />

2 2<br />

sinx<br />

The graph of y = appears to be the<br />

l+cosx<br />

x<br />

same as the graph of y = tan-.<br />

2<br />

sinx _ sin2(f)<br />

l+cosx - l+cos2(t)<br />

2sin(t)cos(t)<br />

= ~[2 cos 2 (t )-1]<br />

=<br />

2sin(f)cos(f)<br />

2cos 2(f)<br />

sint<br />

=-­<br />

cosf<br />

x<br />

= tan­<br />

2<br />

tan..!".+cot.A<br />

The graph of y = 2 2 appears to be the<br />

cot'f - tan t<br />

same as the graph of y =sec x.<br />

201


<strong>Trigonometry</strong> <strong>Solutions</strong> <strong>Manual</strong><br />

202 Chapter 5 Trigonometric Identities<br />

48.<br />

sin.L cos.L<br />

.:=.l...+~ Verif 2 x (I +cosx)2<br />

tan1" + cot t _ cost sint . sinfcost 50. en y cot -= 2<br />

2 sin x<br />

cotf-tanf - sint _ cost sintcost<br />

cost sint<br />

cot 2 ~ =[_I)2<br />

_ sin 2 A 2 + cos 2 ..\: 2 2 tant<br />

-( 2 x .2..I.) I<br />

- cos 2-sm 2<br />

I<br />

=C:i:~J2<br />

- -cos2(t)<br />

I<br />

_ (I + cosx)2<br />

=--­<br />

cosx - sin 2 x<br />

=-secx<br />

of . 2 x tanx-sinx<br />

51. Ven y sm -= .<br />

2 2tanx<br />

Work with the left side.<br />

°<br />

sm 2 -= x (±PP-cosx)2<br />

2 2<br />

I-cosx<br />

=<br />

2<br />

Work with the right side.<br />

. .sin.L'<br />

tanx-smx = cosx -smx<br />

2 tan x 2 ~~~<br />

The graph of y = 1-8sin 2 ~cos2 ~ appears to<br />

2 2<br />

sinx -cosxsinx<br />

=<br />

be the same as the graph of<br />

2sinx<br />

y =cos h. I-cosx<br />

=<br />

2<br />

1-8sin 2 ~cos2 ~ = 1-2(4sin 2 ~cos2~)<br />

~r<br />

2 2 2 2<br />

. sin 2x 2 x . 2 x<br />

52. Venfy --=cos --sm -.<br />

= 1-2(2 sin ~ cos 2sinx 2 2<br />

Work with the left side.<br />

sin2x 2sinxcosx<br />

= 1-2[sin2(~)r --= =cosx<br />

2sinx 2sinx<br />

Work with the right side.<br />

= 1-2sin 2 x<br />

2 x . 2 x I + cos x 1-cos x<br />

=cos2x cos --sm -=--­<br />

2 2 2 . 2<br />

2<br />

2cosx<br />

49. Verify sec 2 ~ =--­ =--­<br />

2 I+cosx<br />

2<br />

= cos x<br />

2 x I<br />

sec -=--­<br />

2 cos 2 t 2<br />

53. Verify --­<br />

I<br />

I+cosx<br />

2<br />

=<br />

(±~I±CfsX )<br />

1+:'. -tan2~~ [+:0"<br />

(± ::::J<br />

I<br />

= I+cfsx 2 I-cosx<br />

=<br />

2<br />

I +cosx I +cosx<br />

=--- 2-I+cosx l+cosx<br />

I+cosx = =--­<br />

I+cosx I+cosx<br />

=1<br />

202


<strong>Trigonometry</strong> <strong>Solutions</strong> <strong>Manual</strong><br />

54. Verify tanr = esc r<br />

2<br />

r sinr<br />

tan 2" = 1+ cos r<br />

- cot r ' 57.<br />

(I-COSr)<br />

1- cos r<br />

_ sinr-sinrcosr<br />

- I-cos 2 r<br />

_ sinr-sinrcosr<br />

- sin 2 r<br />

= cosr<br />

sinr sin v<br />

=cscr-cotr<br />

55. V<br />

ify 1 2 9<br />

en -tan -=--­<br />

2cos9<br />

2 I+cos9<br />

I-tan2~=1-( sin9 )2<br />

2 1+cos9<br />

_ (1+ cos 9)2 - sin 9<br />

- (1+ cos 9)2<br />

1+ 2cos9 + cos 2 9 - sin 2 9<br />

=<br />

(1+ cos 9)2<br />

_ 1+ 2cos9+ cos 2 9 -(I-cos 2 9)<br />

58.<br />

A sin A<br />

tan-=--­<br />

2 I+cosA<br />

sin A I-cosA<br />

= 1+ cos A 1- cos A<br />

_ sinA(I-cosA)<br />

- I-cos 2 A<br />

_ sinA(l- cosA)<br />

- sin 2 A<br />

I-cosA<br />

=<br />

sin A<br />

· a 1<br />

sm-=­<br />

2 m'<br />

5.6 Half-Angle Identities 203<br />

3<br />

m=­ 2<br />

, a 1 2<br />

sm-=-=­<br />

2 t 3<br />

· 2 a I-cosa<br />

sm -=<br />

2 2<br />

so<br />

56.<br />

- (I+cos9f 1<br />

1+ 2cos 2 9-1 +2cos9<br />

=<br />

(1+ cos 9)2<br />

_<br />

-<br />

2 cos 9(1+ cos 9)<br />

(1+ cos 9)2<br />

59.<br />

5<br />

m=­ 4<br />

cosa=­ 9<br />

a =84°.<br />

2cos9<br />

=<br />

sin a =..!..= ±= .8<br />

1+ cos 9 2 m 5<br />

!!. = 53°<br />

I- tan2 t<br />

Verify cosx = 2 '<br />

1+ tan t<br />

I-tan2t _I-~<br />

2 - .1.=.lm.I.<br />

1+ tan t 1+ licOsx<br />

_ (I+cosx)-(I-cosx)<br />

- (I+cosx)+(I-cosx)<br />

2cosx<br />

=--=cosx<br />

2<br />

60.<br />

2<br />

a « 106°<br />

· a 1 2<br />

sm"2=;;;' m =<br />

, a 1<br />

sm-=­<br />

2 2<br />

· 2 a I-cosa<br />

SID -=<br />

2 2<br />

(±r = I-~osa<br />

2(~ ) =I-cosa<br />

1<br />

--I=-cosa<br />

2<br />

1<br />

--=-cosa<br />

2<br />

1<br />

-=cosa<br />

2<br />

a=60°<br />

203


<strong>Trigonometry</strong> <strong>Solutions</strong> <strong>Manual</strong><br />

204 Chapter 5 Trigonometric Identities<br />

5<br />

61. m:=­<br />

2<br />

sin a = ..!.. :=~ :=.4<br />

2 m 5<br />

a"" 23.60<br />

2<br />

a ""47°<br />

62. a =30°<br />

. 30° 1<br />

sm-=­<br />

2 m<br />

sin 15°:=..!..<br />

m<br />

1<br />

sin 15°<br />

""3.9<br />

63. a=60°<br />

m:=-­<br />

. a 1<br />

sm-:=­<br />

2 m<br />

..!.. =sin 30° :=.!.<br />

m 2<br />

m=2<br />

64. They are both radii of the circle.<br />

65. It is the supplement of a 30° angle.<br />

66. Their sum is 180° - 150° =30°, and they are<br />

equal.<br />

67. Since triangle ACB is a 30°-60°-90° right<br />

triangle with AB =2 and CB:= ../3,<br />

DC= BD+BC=2+../3<br />

68. In right triangle DAC,<br />

(AD =(AC +(DC)2<br />

=12 +(2+../3)2<br />

=1+4+4../3+3<br />

=8+4../3<br />

=6+4../3+2<br />

=6+2.JU+2<br />

=(.J6 + ..fi)2<br />

AD=.J6+5<br />

69. In right triangle EAD,<br />

AD<br />

cosD=­<br />

DE<br />

cos 150 =.J6 +..fi .<br />

4<br />

70. In right triangle EAD,<br />

(DE)2 =(AE)2 + (AD)2<br />

4 2 =(AE)2 +(.J6 +..fi)2<br />

16 = (AE)2 + 8 + 4../3<br />

(AE)2 =8 - 4../3<br />

=6-2.JU,+2<br />

=(.J6 _..fi)2<br />

AE =.J6- -Ii.<br />

Thus,<br />

. D AE<br />

sm =­<br />

DE<br />

sin 150= .J6 -..fi .<br />

4<br />

71. Since angle ADC is 15°,<br />

AC<br />

tanI5°=­<br />

CD<br />

1<br />

=../3 +2<br />

1 (../3-2)<br />

= (../3 +2) . (Jj-2)<br />

../3 -2<br />

=-­<br />

3-4<br />

=2-../3.<br />

(J s-»<br />

72. (a) cos-=-­<br />

2 R<br />

(J 1-cos l!.<br />

(b) tan- = 2<br />

4 sin!<br />

(c)<br />

_<br />

1- R-b<br />

R<br />

- so<br />

Ii<br />

R-(R-b)<br />

=<br />

50<br />

b<br />

=­<br />

50<br />

(J 12<br />

tan-=­<br />

4 50<br />

Using ~ ~ or ITAN"1<br />

find that<br />

!!. "" 13.5°<br />

4<br />

(J "" 54°<br />

on a calculator, we<br />

204


<strong>Trigonometry</strong> <strong>Solutions</strong> <strong>Manual</strong><br />

Chapter 5 Review Exercises<br />

1<br />

1. secx=-­<br />

cos x<br />

The answer is B.<br />

1<br />

2. cscx=-­<br />

sinx<br />

The answer is A.<br />

sinx<br />

3. tanx=-­<br />

cos x<br />

The answer is C.<br />

cos x<br />

4. cotx=-­<br />

sinx<br />

The answer is F.<br />

5. sin 2 x = l-cos 2 x<br />

The answer is H.<br />

6. tan 2 x+l=sec 2 x =_1_ 2<br />

­<br />

cos x<br />

The answer is E.<br />

7. tan 2 x =_1_­<br />

2<br />

cot x<br />

The answer is D.<br />

8. sec 2 0_ tan 2 0 = _1_- - sin: 0<br />

2<br />

cos 0 cos 0<br />

I-sin 20<br />

- cos 20<br />

20<br />

= cos = 1<br />

cos 2 0<br />

2<br />

9. cot 0 =~ = cos 0<br />

sec0 COSlT sinO<br />

10. tan +cot 17 =-- +--­<br />

cos 20 sin 20<br />

20 2<br />

= sin (sin 0+cos 0)<br />

2<br />

cos 2 0 sin 2 0<br />

20<br />

sin ( 1 )<br />

= cos 2 0 sin 20<br />

1<br />

= cos 2 0<br />

2 ~1 2 D) sin 2 0(1 cos 2 0)<br />

Chapter 5 Review Exercises 205<br />

12. csc 2 0+sec 2 0 =_1_ ­<br />

2-+_1_2<br />

sin 0 cos 0<br />

cos 2 0 + sin 2 0<br />

- sin 2 0 cos 2 0<br />

1<br />

sin 0 1 1<br />

13. tan 0 - sec 0 csc 0 =-----._­<br />

cosO cosO sin 0<br />

sin 2 0 -1<br />

= sinOcosO<br />

20<br />

-cos cosO<br />

= =--­<br />

sin 0 cos 0 sin 0<br />

14. sin(-x) = -sinx<br />

tan(-x) = -tan x<br />

cot(-x) =-cotx<br />

csc(-x) =-cscx<br />

The functions sine, tangent, cotangent, and<br />

cosecant satisfy the condition f(-x) = - f(x) .<br />

15. cos(-x) = cosx<br />

sec(-x) = sec x<br />

The functions cosine and secant satisfy the<br />

condition f(-x) = - f(x) .<br />

16. cosx =~. x is in quadrant IV.<br />

5<br />

2<br />

. 2 2 3 16<br />

510 x = 1-cos x = 1- ( '5 ) = 25<br />

Since x is in quadrant IV, sin x < O.<br />

sinx=-~=-~<br />

sinx --! 4<br />

tanx=--=-=-­<br />

cosx ~ 3<br />

1 1 5<br />

secx=--=-=cos<br />

x t 3<br />

1 1 5<br />

cscx =--=- =-­<br />

sinx -! 4<br />

1 1 3<br />

cotx=--=-=-­<br />

tan x -1 4<br />

1 cos0 1+ cos 0<br />

11. cscO+cotO=--+--=--­<br />

sin 0 sin 0 sin 0<br />

205


<strong>Trigonometry</strong> <strong>Solutions</strong> <strong>Manual</strong><br />

206 Chapter 5<br />

Trigonometric Identities<br />

5<br />

17. tan x = - ­<br />

4'<br />

1r<br />

- < x < 1r<br />

2<br />

( 5)2<br />

2 41<br />

sec x=tan 2 x+l= -- +1=­<br />

4 16<br />

Since x is in quadrant II, sec x < 0, so<br />

.,!4f<br />

secx=---.<br />

4<br />

1 4 4.,!4f<br />

cosx =--=---=--­<br />

sec x .,!4f 41<br />

sin x = cosx tan x = _ 4.,!4f (_ ~) = s.,I4f<br />

41 4 41<br />

1 4<br />

cotx=--=-­<br />

tan x 5<br />

1 41 .,!4f<br />

cscx = sinx = 5J4i = -5­<br />

18. (8) . 1r . (1r 1r)<br />

sm 12=sm 4"-6"<br />

.1r 1r 1r .1r<br />

=sm-cos--cos-sm­<br />

4 6 4 6<br />

..fi..[j ..fi 1 =_._-_.­<br />

2 2 2 2<br />

J6..fi<br />

=--­<br />

4 4<br />

J6-..fi<br />

=<br />

4<br />

cos ~ = cos(: - :)<br />

1r x .1r.1r<br />

= cos-cos-+ sm-sm­<br />

4 6 4 6<br />

..fi..[j<br />

=_._+_<br />

2 2 2 2<br />

J6..fi<br />

=-+­<br />

4 4<br />

J6+.J2<br />

=<br />

4<br />

tan ~ = tan(: - :)<br />

_ tanf-tanf<br />

..fi .- 1<br />

- l+tanftanf<br />

1-4<br />

= 1+(I)..[j .<br />

3<br />

3-..[j 3-..[j<br />

= 3+..[j ·~<br />

12-6.J3<br />

=<br />

6<br />

= 2-..[j<br />

(b) In this exercise .!!.... is in quadrant I, where<br />

12<br />

all circular functions are positive.<br />

sin ~ = sin[; )<br />

= ~1-COS1t 6<br />

2<br />

=~1-2~<br />

=~2-4..[j<br />

=~<br />

2<br />

cos~ =co{;)<br />

=~I+~ost<br />

=~1+./3<br />

.:..:.-..L..<br />

2<br />

=~2+4..[j<br />

~<br />

=<br />

2<br />

tan ~ = tan[; )<br />

= sint<br />

l+cost<br />

_ 1. 2 _ 1<br />

- 1+./3-UJ3<br />

2<br />

1 2-..[j<br />

= 2+.J3 · 2-..[j<br />

=2-..[j<br />

206


<strong>Trigonometry</strong> <strong>Solutions</strong> <strong>Manual</strong><br />

Chapter 5 Review Exercises 207<br />

19. Use the fact that 165° = 180° - 15°,<br />

sin 165° =sin 180° cos 15° - cos 180° sin 15°<br />

=sin 15°<br />

=sin(45° - 30°)<br />

=sin 45° cos 30° - cos 45° sin 30°<br />

~..fj -fi 1<br />

=2'2-2'2<br />

-I6-..fi<br />

=<br />

4<br />

cosl65° = cos 180° cos 15° + sin 180° sin 15°<br />

=-cos 15°<br />

=-cos(45° - 30°)<br />

=-(cos 45° cos 30° + sin 45° sin 30°)<br />

=J..fi ...fj + ..fi.!)<br />

1.2 2 22<br />

-J6 --fi<br />

=<br />

4<br />

tan 180° - tan 15°<br />

tan 165° = ...:.;,;;~-__-":"":':'<br />

I + tan 180° tan 15°<br />

=-tanI5°<br />

tan 45° - tan 30°<br />

= 1+ tan 45° tan 30°<br />

I-lj­<br />

=-~<br />

5 0<br />

sec16<br />

3-..fj 3-..fj<br />

=-3+$ '3-Jj<br />

12-6..J3<br />

=­<br />

6<br />

=-2+..fj<br />

I<br />

cos 1 650<br />

I<br />

=-2+$<br />

1 -2-..fj<br />

= -2+..fj -2-..fj<br />

-2-..fj<br />

= 4-3<br />

=-2-..fj<br />

20. cos 2 10° = cos(1500+600)<br />

= cos 150° cos 600 - sin 150° sin 60°<br />

The answer is E.<br />

21. sin 35° =cos(90° - 35°)<br />

= cos55°<br />

The answer is B.<br />

22. tan(-35°) =cot[900 -(-35°)]<br />

= cot 125°<br />

The answer is J.<br />

23. - sin 35° =sin(-35°)<br />

The answer is A.<br />

24. cos35° = cos(-35°)<br />

The answer is I.<br />

150°<br />

25. cos 75° = cos-­<br />

2<br />

I +cosI50°<br />

=<br />

2<br />

The answer is C.<br />

26. sin 75° =sin(15° + 60°)<br />

= sin 15°cos6O° + cos 15°sin 60°<br />

The answer is H.<br />

27. sin 300° = sin 2(150°)<br />

= 2 sin 150° cos 150°<br />

The answer is D,<br />

207


<strong>Trigonometry</strong> <strong>Solutions</strong> <strong>Manual</strong><br />

208 Chapter 5 Trigonometric Identities<br />

28. cos 300° = cos 2(150°)<br />

= cos 2 150° -sin 2 150°<br />

The answer is G.<br />

29. Since sin x = 0.8, sin 2 x = 0.64. Then<br />

cos 2 x = 1-sin 2 x = 1-0.64 = 0.36, and<br />

cos x = ..J0.36 = 0.6 (take the positive root<br />

. 0 Tr<br />

since


<strong>Trigonometry</strong> <strong>Solutions</strong> <strong>Manual</strong><br />

Chapter 5 Review Exercises 209<br />

. 2 I . . d II'<br />

32. smy = --. cosx = -- X IS m qua rant , y IS<br />

3 5<br />

in quadrant III. First, find sin x, cos y, tan x, and<br />

tan y. Since x is in quadrant II, sin x is positive.<br />

sinx=~I-cos2 x<br />

=~<br />

2../6<br />

=-­<br />

5<br />

Since y is in quadrant III, cos y is negative.<br />

cosy = -~I-sin2 y<br />

=-FH)<br />

=-~<br />

~<br />

=-­<br />

. 3 ¥<br />

tan x = smx =--=-2../6<br />

cos x -t<br />

siny -t 2 2~<br />

tany = cosy = _{- = ~ =-5­<br />

sin(x + y) =sin xcosy+ cosxsin y<br />

=(2~X- ~)+(-~X-~)<br />

-2-J3Q +2<br />

=<br />

15<br />

cos(x - y) =cosxcosy+sinxsiny<br />

=(-~X-~)+(2~X-~)<br />

~ -4../6<br />

=<br />

15<br />

_ ta_n_x_+_ta_ n--,Y:....<br />

tan(x+y) = l-tanxtany<br />

-2../6+M<br />

= 5<br />

1-(-2../6X2f5)<br />

-10../6 +2~<br />

=----'-----==-­<br />

5+4-J3Q<br />

To find the quadrant of x + y. notice that<br />

sin(x + y) < 0, which indicates that x + y is in<br />

quadrant III or IV. Also, tan(x + y) < 0, which<br />

indicates that x + y is in quadrant II or IV.<br />

Therefore, x + y is in quadrant IV.<br />

33. Find sin (x + y), cos (x - y), and tan (x + y),<br />

I<br />

•<br />

. . 1 4. . d<br />

given Sin x = -, cos y = -, X IS m qua rant<br />

10 5<br />

y is in quadrant IV.<br />

Since x is in quadrant I, cos x > O.<br />

cosx=~I-sin2 x<br />

=~1-1~<br />

=~:~<br />

3M<br />

=-­<br />

10<br />

Since y is in quadrant IV, sin y < O.<br />

siny = -~I-cos2 x<br />

=-~I-~~<br />

=_~25;16<br />

3<br />

=-­<br />

5<br />

sin(x+ y) = sinxcosy+ cosxsiny<br />

=(-.!.-X~) + 3.J11 (-~)<br />

10 5 10 5<br />

4 9-../li<br />

=---­<br />

50 50<br />

4-9.J!f<br />

=<br />

50<br />

cos(x- y) =cosxcosy+sinxsiny<br />

=( ~X~)+C~X -~)<br />

12M 3<br />

=---­<br />

50 50<br />

12M-3<br />

=<br />

50<br />

209


<strong>Trigonometry</strong> <strong>Solutions</strong> <strong>Manual</strong><br />

210 Chapter 5 Trigonometric Identities<br />

sin(x + y)<br />

tan(x + y) = _':'-""""--7­<br />

cos(x+ y)<br />

_ 4-9P 5<br />

- 120Ji1+3<br />

50<br />

4 - 9.Ji1 12.Ji1 - 3<br />

= 12.Ji1 +3 . 12.Ji1-3<br />

4S.Ji1 -12 -l1(IOS) + 27.Ji1<br />

=_..:...-_--:---:~..:--.:.....-<br />

144(11)-9<br />

75.Ji1-1200<br />

=<br />

1575<br />

_ 75(.Ji1) -75(16)<br />

- 75(21)<br />

.Ji1-16<br />

=<br />

21<br />

To find the quadrant of x + y, notice that<br />

sin(x + y) < 0, which implies x + y is in<br />

quadrant III or IV. Also tan(x + y) < 0, which<br />

implies that x + y is in quadrant II or IV.<br />

Therefore, x + y is in quadrant IV.<br />

2. I. . d IV ..<br />

34. cosx=-, smy=--, IS In qua rant ,y IS In<br />

9 2<br />

quadrant III. First, find sin x, cos y, tan x, tan y.<br />

Since x is in quadrant IV, sin x is negative.<br />

sinx = -~I-cos2 x<br />

=-R%)<br />

=-~<br />

..ff7<br />

=--­ 9<br />

Since y is in quadrant III, cos y is negative.<br />

cosy = -,It -sin 2 y<br />

=-FG)<br />

=-l<br />

.J3<br />

=-­ 2<br />

-lll­<br />

sinx<br />

..ff7<br />

tanx=--=--=--­<br />

cos x ~ 2<br />

siny --!- 1 .J3<br />

tany=--=--=-=cosy<br />

_li .J3 3<br />

2<br />

sin(x+ y) = sinxcosy+cosxsiny<br />

~231-2<br />

=....;"....-­<br />

IS<br />

cos(x- y) = cosxcosy+ sinxsiny<br />

..ff7 -2.J3<br />

=<br />

IS<br />

_ tan ____ta x + _n -,y~<br />

tanx+y ( ) = l-tanxtany<br />

_.ffi +..[3<br />

_ 2 3<br />

- 1-(-~)(~)<br />

_ 2.J3 - 3..ff7<br />

- 6+~231<br />

or<br />

sin(x+y) ~231-2<br />

tan(x + y ) = = ----,=---=<br />

cos(x + y) -..ff7- 2.J3<br />

To find the quadrant of x + y, notice that<br />

sin(x + y) > 0, which indicates that x + y is in<br />

quadrant I or II. Also, tan(x + y) < 0, which<br />

indicates that x + y is in quadrant II or IV.<br />

Therefore, x + y is in quadrant II.<br />

35. Find sin 9 and cos 9, given cos 29 = - ~,<br />

4<br />

90 0 < 29 < IS0 0 •<br />

Since 29 is in quadrant II, 9 is in quadrant I.<br />

cos 29 = 1-2 sin 2 9<br />

-~=1-2sin29<br />

4<br />

7 2· 2<br />

--=- sm 9<br />

4<br />

2. =sin 2 9<br />

S<br />

Since 9 is in quadrant I, sin 9 > 0.<br />

Jt =sin9<br />

sin9 = .J14<br />

4<br />

210


<strong>Trigonometry</strong> <strong>Solutions</strong> <strong>Manual</strong><br />

Chapter 5 Review Exercises 211<br />

36. cos 2B = .!., B is in quadrant IV. Since B is in 39. Find cos !!.., given cosO = _.!.., 90 0 < 0 < 180 0 •<br />

8<br />

2 2<br />

quadrant IV, sin B is negative and cos B is<br />

positive.<br />

Since 0 is in quadrant Il, !!.. is in quadrant I, so<br />

2<br />

sinB=_~I-C;S2B =_~I;t =_~ =_ ~<br />

COSB=P+COS2B =~I+i = f9 =~<br />

2 2 "16 4<br />

37. Find sin 2x and cos 2x, given tan x = 3,<br />

40. Find sin~, cosA=-~,900 0 and sin x < 0, then x is in<br />

A<br />

quadrant ill and 2x is in quadrant I or n.<br />

Since 90° < A < 180°, 45° < - < 90°,<br />

2<br />

2tanx 2(3) 6 3<br />

tan2x= =--=--=-­<br />

l-tan 2 x 1-3 2 . A, , .<br />

Thus, sm- IS positive.<br />

8 4<br />

2<br />

Since tan 2x < 0, 2x is in quadrant n. Thus, sec<br />

2x < 0 and sin 2x > O.<br />

sm-=<br />

, A ~1-COSA<br />

2 2<br />

sec2x = -~1 + tan 2 2x<br />

=-FH)<br />

=~<br />

6<br />

=,fI<br />

=-­<br />

8<br />

..fi../2<br />

5<br />

=-­ =2../2 4<br />

'"J2<br />

1 4<br />

$4<br />

cos2x =--=-­<br />

=-­<br />

sec2x . 5<br />

4<br />

. r-p4)2 3<br />

sm2x = v 1 -l-s) ='5<br />

41. Find tan x, given tan 2x = 2, rr < x < 3rr .<br />

2<br />

tan2x = 2<br />

5, 0<br />

2tanx =2<br />

38• secy = -'3' smy><br />

l-tan 2 x<br />

1 3<br />

cosy=--=-­<br />

2 tan x = 2 - 2 tan 2 x<br />

secy 5<br />

tan 2 x+tanx-l=O<br />

-1 ±.Jl+ 4 -1 ±../5<br />

SinY=~I-COS2Y=~I-(-~r =~<br />

tan x = =-~-<br />

2 2<br />

sin2y = 2sinycosy<br />

Since x is in quadrant III, tan > o.<br />

-1+../5<br />

tanx=--­<br />

=2(~X-~)=-~:<br />

2<br />

cos2y = cos 2 y-sin 2 y<br />

=(-~r -(~r<br />

9 16<br />

=--­<br />

25 25<br />

7<br />

=-­<br />

25<br />

211


­<br />

<strong>Trigonometry</strong> <strong>Solutions</strong> <strong>Manual</strong><br />

212 Chapter 5 Trigonometric Identities<br />

43. 45. s ~<br />

Y I =Y-::~ ~J<br />

4<br />

44.<br />

sin2x+sinx be sinx<br />

The graph of y = appears to The graph of y = appears to be the<br />

cos2x - cos x<br />

1-cosx<br />

x<br />

x<br />

the same as the graph of y = cot -. same as the graph of y = cot - .<br />

2 2<br />

sinx 1<br />

sin2x+sinx 2sinxcosx +sinx 1-cos x - 1-9<br />

= OSX<br />

SID X<br />

cos2x -cosx 2cos 2 x -1-cosx 1<br />

sin x(2 cosx + 1)<br />

=-­<br />

= tan!<br />

(2cosx + 1Xcosx -1)<br />

x<br />

sinx<br />

=cot­<br />

= 2<br />

cosx-1<br />

sinx 46. cos X sin 2x<br />

= 1-cosx 1 +cos2x<br />

1<br />

= I-wsx<br />

Sin X<br />

1<br />

=-­<br />

tan!<br />

x<br />

=cot­<br />

2<br />

-4<br />

cosx sin2x b h<br />

The grap h 0 f y = appears to e t e<br />

1+cos2x<br />

same as the graph of y =sin x.<br />

cosxsin2x cos x ·2sinxcosx<br />

-----=-----.,;:-----.,<br />

1+cos2x 1+2cos 2 x-I<br />

2sinxcos 2 x<br />

= 2cos 2 x<br />

=sinx<br />

1-cos2x 47. 2(sin x - sin 3 x)<br />

The graph of y = . appears to be the YI = --'---cos-x---'-- I<br />

sm2x<br />

same as the graph of y = tan x.<br />

4<br />

----=tan 1-cos2x (2X)<br />

sin2x 2<br />

= tan x<br />

-4<br />

212


<strong>Trigonometry</strong> <strong>Solutions</strong> <strong>Manual</strong><br />

Chapter 5 Review Exercises 213<br />

48.<br />

2(sinx-sin 3 x)<br />

The graph of y =<br />

appears to be<br />

cosx<br />

the same as the graph of y = sin 2x.<br />

2(sinx-sin 3 x) 2sinx(l-sin 2 x) cosx<br />

---:._------"- =<br />

._­<br />

cosx cos x cosx<br />

_ 2sinxcosx(l-sin 2 x)<br />

- cos 2 x<br />

_ sin2x(l-sin 2 x)<br />

- l-sin 2 x<br />

=sin2x<br />

The graph of y = esc x - cot x appears to be the<br />

x<br />

same as the graph of y = tan- .<br />

2<br />

1 cosx<br />

cscx-cotx =----­<br />

sinx sinx<br />

l-cosx<br />

= sinx<br />

x<br />

=13n­<br />

2<br />

2 Y<br />

2<br />

49. Verify sin 2 x - sin 2 y = cos - cos x.<br />

sin 2 x-sin 2 y= (1- cos 2 x) - (l-cos 2 y)<br />

=1-cos 2 x-l+cos 2 Y<br />

2 2<br />

=cos y-cos X<br />

2x-sin2x<br />

. 3 cos<br />

50. Venfy 2cos x-cosx=-----­<br />

secx<br />

Work with the right side.<br />

cos 2 x-sin 2 x cos 2 x-sin 2 x<br />

-----=-----:---­<br />

secx ~<br />

cosx<br />

= cos3x -sin 2 xcosx<br />

3<br />

= cos x -(1-cos 2 x)cosx<br />

= 2 cos 3 x - cos x<br />

sin 2 x 2 x<br />

51. Verify = cos - .<br />

2-2cosx 2<br />

Work with the left side.<br />

sin 2 x 1-cos 2 x<br />

----=---­<br />

2-2cosx 2{I-cosx)<br />

_ (1-cosx){1 + cosx)<br />

- 2{I-cosx)<br />

l+cosx<br />

=<br />

2<br />

Work with the right side.<br />

2 x l+cosx<br />

cos -=--­<br />

2 2<br />

52. Verify si~ 2x = _2_.<br />

smx secx<br />

sin 2x 2sin x cosx<br />

--=<br />

sinx sinx<br />

=2cosx<br />

2<br />

= I<br />

cos x<br />

= 2<br />

sec x<br />

53. Verify 2cosA -secA = cosA _ tan A .<br />

cscA<br />

Work with the right side.<br />

tan A ~1<br />

cosA---=cosA--­<br />

cscA<br />

sr.hsin<br />

2 A<br />

=cosA---­<br />

cosA<br />

2<br />

= cos A-(I-cos 2 A)<br />

cos A<br />

2cos 2 A-I<br />

=<br />

cos A<br />

1<br />

=2cosA--­<br />

cos A<br />

= 2cosA -secA<br />

ify 2 tan B 2<br />

54. Ven ---=sec B.<br />

sin2B<br />

2 tan B 2 . ..s.in..8. cos B<br />

---=--=~sin2B<br />

2sinBcosB<br />

1<br />

= cos 2 B<br />

=sec 2 B<br />

213


<strong>Trigonometry</strong> <strong>Solutions</strong> <strong>Manual</strong><br />

214<br />

Chapter 5 Trigonometric Identities<br />

55.<br />

56.<br />

57.<br />

Verify 1+ tan 2 a = 2 tanacsc2a .<br />

2tana<br />

2tanacsc2a =--­<br />

sin2a<br />

2~~<br />

=-----"="--­<br />

2sinacosa<br />

2sina<br />

=------=-­<br />

2sinacos 2 a<br />

1 2<br />

=--2- ;: sec a<br />

cos a<br />

=1+tan 2a<br />

Verify tan A + tan B+tanC = (tan AXtanBXtanC)<br />

Work with the left side.<br />

tan A + tan B+ tan C;: tan A + tanB+ tan(I80 -(A + B))<br />

= tan A + tanB- tan(A + B)<br />

tan A + tan B<br />

= tan A + tan B - ----­<br />

I-tanA tan B<br />

tan A(I- tan A tan B)+ tan 8(1- tan A tan B) - tan A - tan B<br />

l-tanA tan B<br />

tan A -tan 2 A tan B+ tan B-tanA tan 2 B-tanA -tan B<br />

=<br />

I-tanA tanB<br />

-tan 2 AtanB-tanAtan 2 B<br />

;:<br />

l-tanA tan B<br />

tan A tan B(tan A + tan B)<br />

I-tanA tan B<br />

;: _ tan A tan B. (tan A + tan B)<br />

I-tanA tan B<br />

= -tan A tan Btan(A + B)<br />

= tan Atan B(-tan(A + B))<br />

= (tan A)(tan B)(tanC)<br />

Verify [tan(45° + A)][tan(45° - A)];: 1.<br />

Work with left side.<br />

[tan(45° + A)!tan(45° - A)]<br />

0+ 0<br />

;: [sin(45 A)Isin(45 - A)]<br />

cos(45° + A)<br />

cos(45°-A)<br />

_ (sin 45° cos A + cos 45° sin AXsin 45° cos A -cos45°sinA)<br />

- (cos 45° cos A -sin 45° sin A)(cos45°cos A +sin45°sin A)<br />

sin 2 45°cos 2 A -sin45°cos45°sinAcosA + sin 45° cos 45° sin AcosA -cos 2 45°sin 2 A<br />

- cos 2 45°cos 2 A +sin45°cos45°sin AcosA - sin 45° cos 45° sin AcosA -sin 2 45°sin 2 A<br />

sin 2 45° cos 2 A - cos 2 45°sin 2 A<br />

- cos 2 45° cos 2 A - sin 2 45°sin 2 A<br />

_(~t cos 2 A-(~t sin 2 A<br />

-(~t cos 2 A-(4f sin 2 A<br />

=1<br />

214


<strong>Trigonometry</strong> <strong>Solutions</strong> <strong>Manual</strong><br />

Cbapter 5 Review Exercises 215<br />

. 2cotx 2<br />

58. Venfy --=csc x-2.<br />

tan2x<br />

2cotx<br />

2<br />

tan 2x = tan x(~)<br />

I-tan x<br />

l-tan 2 x<br />

= tan 2 x<br />

I-~<br />

~<br />

= . 2<br />

=2;<br />

cos 2 x-sin 2 x<br />

=<br />

sin 2 x<br />

1-2sin 2x<br />

= sin<br />

2<br />

x<br />

=csc 2 x-2<br />

59. Verify tanOsin20 = 2 -2cos 2 0<br />

tan Osin20 = tan 0(2 sin 0 cosO)<br />

sinO .<br />

=--(2smOcosO)<br />

cosO<br />

= 2sin 20<br />

= 2(I-cos 2 0)<br />

=2-2cos 20<br />

60. Verify esc A sin 2A - sec A = cos 2A sec A .<br />

esc Asin 2A - sec A = _1_(2sin AcosA) __1_<br />

sin A<br />

cos A<br />

2cos 2 A-I<br />

=<br />

cos A<br />

cos2A<br />

=--­<br />

cos A<br />

= cos2A sec A<br />

61. Verify 2 tan x esc 2x - tan 2 x = 1.<br />

2tanxcsc2x-tan 2 x<br />

I 2<br />

=2tanx---tan x<br />

sin2x<br />

I 2<br />

= 2 tan x---­tan x<br />

2sinxcosx<br />

sinx sin 2 x<br />

= sinxcos 2 x - cos 2 x<br />

sin 2 x<br />

= cos 2 x - cos 2 x<br />

l-sin 2 x<br />

= cos 2 x<br />

2<br />

cos x<br />

= cos 2 x<br />

=1<br />

. 2 1-tan 2 0<br />

62. Venfy 2cos 0-1 = 2 '<br />

I+tan 0<br />

Work with the right side.<br />

2 I-~<br />

I - tan 0<br />

cos 2 (}<br />

l+tan 20 = I+~<br />

cos e<br />

cos 2 0 - sin 2 0<br />

= cos 2 0+sin 2 0<br />

= cos 2 0-sin 2 0<br />

= cos20<br />

= 2cos 2 0-1<br />

2 ll<br />

2tanOcos 20-13nO<br />

II<br />

63. Verify tan u cos u = 2<br />

I-tan 0<br />

Work with the right side.<br />

2 tan Ocos 2 0 - tan 0 tan0(2cos 20-1)<br />

l-tan 20 l-tan 20<br />

tan 0(2cos 2 0-1)<br />

= cos 2 (I-sin 2 (I<br />

cos 2 (}<br />

_ cos 2 otan0(2 cos 2 0-1)<br />

- 2cos 20-1<br />

20tanO<br />

=cos<br />

= tanOcos 2 0<br />

64.<br />

Verify<br />

en -cot-=<br />

x _s_in_2_x_+_s_in_x_<br />

2 cos 2x - cos x<br />

Work with the right side.<br />

sin2x + sin x 2sinxcosx +sinx<br />

cos2x-cosx - (2cos 2 X-I)--cosx<br />

2 sin x cos x + sin x<br />

= (2cos 2 x-cosx-I)<br />

sin x(2 cos x + I)<br />

=..,....-~-----'-....,...<br />

(2 cos x + IXcosx -I)<br />

sinx<br />

= cosx-I<br />

sinx<br />

= I-cosx<br />

I<br />

=--­ x<br />

tan­<br />

2<br />

x<br />

=-cot­<br />

2<br />

215


<strong>Trigonometry</strong> <strong>Solutions</strong> <strong>Manual</strong><br />

216 Chapter 5 Trigonometric Identities<br />

3 COS 2 x-sin 2 x<br />

65. Verify 2cos x-cosx=----­<br />

sec x<br />

Work with the right side.<br />

cos 2 x-sin 2 x 2cos 2 x-I<br />

secx<br />

= -..l.­<br />

cosx<br />

= (2cos 2 x-l)cosx<br />

= 2cos 3 x-cosx<br />

66. Verify sin 36 = sin 6 - cos 2 6sin 6.<br />

Work with the right side.<br />

sin6 - cos 2 6sin 6 = sin6 - (1- sin 2 6)sin 6<br />

= sin 6 - sin 6 + sin 3 6<br />

= sin 3 6<br />

67. Verify cos 2 x+sin 2 x(I-2sin 2 y)=cos 2 y+sin 2 y(l-2sin 2 x).<br />

Show that the left side minus the right side equals equals O.<br />

2 2<br />

cos x +sin 2 x(l- 2sin 2 y) - cos y -sin 2 y(1-2sin 2 x)<br />

=cos 2 x+sin 2 x-2sin 2 x,sin 2 y-cos 2 y-sin 2 y+2sin 2 xsin 2 y<br />

= (cos 2 x+sin 2 x)-(cos 2 y+sin 2 y)<br />

=1 -1<br />

=0<br />

68.<br />

. 2 sec2a-l<br />

69. Venfy sec a-I = .<br />

sec2a+l<br />

Work with the right side.<br />

sec2a-l roba- 1<br />

sec2a+ 1 coJ2a + 1<br />

J -1<br />

_ cos 2 a-sin 2 a<br />

- J + 1<br />

cos 2 a-sin 2 a<br />

l-cos 2 a+sin 2 a<br />

- l+cos 2 a-sin 2 a<br />

2sin 2 a 2<br />

= 2 = tan a<br />

2cos a<br />

=sec 2 a-I<br />

70. Verify sin 31 + sin 21 = tan ~ .<br />

sin 31 - sin 21 tan t<br />

Use the identities in Section 5.5 Connections<br />

. 31 + 21 31 - 21<br />

with A=-- and B=--. Then<br />

2 2<br />

A+B=31 and A-B=21.<br />

2sin(1!.¥L) cos(1ft)<br />

sin 31 + sin 21<br />

sin31-sin21 - 2cose';2')sine';2,)<br />

sin~cosL 51 I<br />

= 2 2 = tan-cotcos~sinL<br />

2 2<br />

2 2<br />

tan~<br />

=__ 2_<br />

tant<br />

. 2 tan26<br />

71. Venfy tan 46 = 2'<br />

2 -sec 26<br />

tan46 = 2 tan 26<br />

1-(sec 2 26 -1)<br />

2 tan 26<br />

- 1- sec 2 26 + 1<br />

2tan26<br />

- 2 -sec 2 26<br />

216


<strong>Trigonometry</strong> <strong>Solutions</strong> <strong>Manual</strong><br />

Chapter 5 Test Exercises 217<br />

72.<br />

73.<br />

Verify 2cos 2 '::'tanx = tanx+sinx.<br />

2<br />

Work with the right side.<br />

. sinx .<br />

tanx-e sm r =--+SIOX<br />

COSX<br />

= sin X(I + _1_)<br />

cos x<br />

= sinx(l+ ~ )<br />

2cos "!-I<br />

. (_2C_os_ 2,*"t_-_ I+_I)<br />

=SlOX<br />

2cos 2 f - 1<br />

2sinxcos 2 t<br />

= 2cos 2 f - 1<br />

2 x sinx<br />

=2cos -'-­<br />

2 cosx<br />

= 2 tanxcos 2'::'<br />

2<br />

Verify tan(1+ :) = sec x + tan x .<br />

Work with the left side.<br />

tan('::'+~) = tant+tant<br />

2 4 1-tanttant<br />

=tant+ 1<br />

I-tant<br />

Work with the right side.<br />

I sin x<br />

secx+tanx=--+-­<br />

cos x COSX<br />

_ cos 2 t+ sin2 t+2sintcost<br />

- cos 2 t-sin2 t<br />

_<br />

(cost+sint)2<br />

- (cosf-sintXcost+sint)<br />

= cost+sint<br />

cost-sint<br />

cost + sin!<br />

_ cost cost<br />

- cost _ sinf<br />

cosf cost<br />

=I+tant<br />

I-tant<br />

74.<br />

75.<br />

76. a.<br />

. 1 x 1 x<br />

Venfy -cot---tan-=cotx.<br />

2 2 2 2<br />

'!'cot'::' _.!. tan'::' =.!.( 1+ cosx) _.!. (1-cosx)<br />

2 2 2 2 2 sin x 2 sin x<br />

=(l+cosx)-(l-cosx)<br />

2sinx<br />

2cosx<br />

=--­<br />

2sinx<br />

=cotx<br />

From Exercise 56,<br />

tanA+ tanB+ tanC = (tan AXtanBXtanC).<br />

Let C be the largest angle of D.ABC.<br />

Then 90° < C < 180° (since the angles of a<br />

triangle sum to 180°) and tan C < O. (since<br />

tan x is negative for 90° < x < 180°) Since<br />

C > 90°, then A + B < 90°, so 0 < A < 90° and<br />

o< B < 90° (since a triangle cannot have a<br />

negative angle). Then tan A > 0 and<br />

tan B > 0 (since tan x is positive for<br />

0° < x < 90°), which means that<br />

(tan AXtan BXtan C) is the product of two<br />

positive values and one negative value, and<br />

hence is a negative number (which is < 0).<br />

Since (tan AXtanBXtanC) < 0,<br />

tan A + tanB+tanC < O.<br />

Although the table indicates that the<br />

functions may be equal, the graph clearly<br />

demonstrates that cos x = cos 4x is not an<br />

identity.<br />

217


<strong>Trigonometry</strong> <strong>Solutions</strong> <strong>Manual</strong><br />

218 Chapter 5 Trigonometric Identities<br />

b. x · Vi ·:· Y2 Chapter 5 Test Exercises<br />

-0<br />

c.: ~<br />

0 "<br />

3.1lt1& 0 , .0<br />

-, .'<br />

Un2: 0 . . ~: .~ . 3n<br />

'.'12:ltl "2["n 0 1. Since - < x < 21f, sinx < 0 and cos x > O.<br />

1,".5&& 0 · ' "1E"U .. 2<br />

1!.01 2[~1) 0<br />

.11.15: -"E·l~ 1E~13 Then<br />

~.., ' . .,J<br />

K=i!t<br />

Although the table indicates that the<br />

functions may be equal, the graph clearly<br />

demonstrates that sin (x + n) =.5 sin x is<br />

sec2x=tan2x+l=(-~r +1<br />

25<br />

=-+1<br />

36<br />

61<br />

=­<br />

36<br />

~ kith . .<br />

S0, sec x = 6' ta ng e positive square<br />

6 6~<br />

root. Thus, cos x = 177 =--.<br />

....61 61<br />

. 5 6~ 5.J6l<br />

smx = tan x cosx = -_ .--=-­<br />

6 61 61<br />

2. tan2x-sec2 x=(sinx)2 _(_1_)2<br />

not an identity. cosx cos x<br />

c. Y I = 4sin x cos x<br />

Y 2 = 2sin 2x<br />

sin 2 x-I<br />

- cos 2 x<br />

2<br />

cos X<br />

= - cos2 x<br />

=-1<br />

3. Since x is in quadrant ill, cos x < O. Then<br />

sin 2 x + cos 2 x = 1<br />

cos 2 x =I-sin 2 x<br />

=1-(-~r<br />

1<br />

=1-­<br />

9<br />

8<br />

=­<br />

9<br />

cos x = _ f! = -2.,fi<br />

The table indicates that the functions may<br />

'/'9 3<br />

be equal, the graph confirms that<br />

4sin x cos x = 2sin 2x is an identity.<br />

218


<strong>Trigonometry</strong> <strong>Solutions</strong> <strong>Manual</strong><br />

Chapter 5 Test Exercises 219<br />

Since y is in quadrant II, sin y > o. Then<br />

sin 2y+cos2y=1<br />

. 2 I 2<br />

S10 y= -cos Y<br />

=1-(-~r<br />

=1-~<br />

4. sin(-22.50) = ±~ 1-cOi-45°)<br />

=+~1_.J2 __2_<br />

- 2<br />

= ±~2-4..fi<br />

25<br />

21<br />

=+~<br />

=­<br />

- 2<br />

25 Since -22.5° is in quadrant IV, sin(-22.5°) is<br />

sin y = (2l =..fii<br />

V"25 5<br />

sin(x + y)= sinxcosy + cosxsiny<br />

-~2-.J2<br />

negative. sin(-22.5°) = ----'--­<br />

2<br />

5. Y I = sec x - sin x tan x<br />

2 2...[42<br />

=---­<br />

IS IS<br />

2-2...[42<br />

=<br />

IS<br />

-4<br />

cos(x - y)= cos x cosy+ sin xsiny<br />

. _: ..<br />

~. ~-,<br />

-.~<br />

The graph of sec x - sin x tan x appears to be the<br />

same as the graph of cosx.<br />

secx-s1Oxtanx=---s1OX<br />

. I. (sinx) --<br />

4.J2 .J2! cosx cosx<br />

=----­<br />

15 15 1-siri 2 x<br />

=<br />

4..fi -..fii<br />

cosx<br />

=<br />

15 cos 2 x<br />

=-­<br />

cosx<br />

tan(x + y)= _ta_n_x_+_ta_n.....:y~<br />

=cosx<br />

1- tan x tan y<br />

_ ~+(-4I)<br />

- 1-(1)(- ~)<br />

2.J2 -4.J2!<br />

=---.==-­<br />

8+...[42<br />

6. Y I = rol X<br />

Z<br />

- calx<br />

x<br />

The graph of cot - - cot x appears to be the<br />

2<br />

same as the graph of esc x .<br />

x 1+ cosx cosx<br />

cot - - cot x =--­<br />

2 sinx sinx<br />

I+cosx-cosx<br />

=<br />

sinx<br />

=-­<br />

sinx<br />

=cscx<br />

219


<strong>Trigonometry</strong> <strong>Solutions</strong> <strong>Manual</strong><br />

220 Chapter 5 Trigonometric Identities<br />

ify 2 B 1<br />

7. Ven sec = 2'<br />

I-sin B<br />

Work with the right side.<br />

1 1 2<br />

--....."...- =---=sec B<br />

l-sin 2 B cos 2 B<br />

8.encos Verify 2A = cot A - tan A ,<br />

esc AsecA<br />

Work with the right side.<br />

cos A<br />

sinA<br />

cot A - tan A _ Siii"A - CiiS'A<br />

esc A sec A - (si~A)(C~A)<br />

cos 2 A-sin 2 A<br />

sinAcosA<br />

= J<br />

sin AcosA<br />

cos 2 A -sin 2 A sin A cos A<br />

= sin AcosA<br />

=cos 2 A -sin 2 A<br />

22A<br />

=cos<br />

9. The cofunction identities in Chapter 2 applied<br />

only to acute angles . The cofunction identities<br />

in this chapter apply to any angles and to any<br />

real numbers.<br />

10. (a) V =163sin on, Since sin x=co{~ - x).<br />

V=163CO{~ -Wt).<br />

(b) If V =163CO{ ~ -1201r t} the maximum<br />

voltage occurs when cos (~ -1201r t) = 1.<br />

Thus. the maximum voltagae is<br />

V =163 volts.<br />

cos (~ -1201r t) =1 when<br />

!!.- 1201rt =2k1r. where k is any integer.<br />

2<br />

The first maximum occurs when<br />

1r -1201r t =0<br />

2<br />

1r<br />

- =1201rt<br />

2<br />

1 1r<br />

-_·_=t<br />

1201r 2<br />

1<br />

--=t<br />

240<br />

The maximum voltage will first occur at<br />

1<br />

--sec.<br />

240<br />

220


<strong>Trigonometry</strong> <strong>Solutions</strong> <strong>Manual</strong><br />

CHAPTER 6: INVERSE TRIGONOMETRIC<br />

FUNCTIONS AND<br />

TRIGONOMETRIC EQUATIONS<br />

Section 6.1<br />

Connections (page 240)<br />

j-I(x) = x-5; HAPPINESS IS STAYING OUT OF<br />

2<br />

HOSPITALS AND COURT ROOMS<br />

Exercises<br />

1. (a) [-I, 1]<br />

(b) [_1r ~J<br />

2'2<br />

(c) increasing<br />

(d) -2 is not in the domain.<br />

6.<br />

7.<br />

8.<br />

6.1 Inverse Trigonometric Functions 221<br />

y = arcco{'7)<br />

-13<br />

cos y = -,OS; y S; 1r<br />

2<br />

y is in quadrant I.<br />

1r<br />

y=­ 6<br />

y=tan- Il 1r 1r<br />

tan y = 1 - - < Y < ­<br />

'2 2<br />

y is in quadrant I.<br />

n<br />

y=­ 4<br />

y = sin-I 0<br />

. 1r 1r<br />

sm y = 0 - - S; y S; ­<br />

'2 2<br />

y=O<br />

2. (a) [-1, 1]<br />

(b) [O,1r]<br />

(c) decreasing<br />

41r . . th<br />

(d) -­ IS not In e range.<br />

3<br />

3. (a) (~,oo)<br />

9. y = cos-I (-1)<br />

cosy=-l, OS;yS;1r<br />

y=1r<br />

10. y = arctan(-1)<br />

1r 1r<br />

tany=-l, --


<strong>Trigonometry</strong> <strong>Solutions</strong> <strong>Manual</strong><br />

222 Chapter 6 Inverse Trigonometric Functions and Trigonometric Equations<br />

14. y=arcsm . (-T<br />

~)<br />

21. y = coC1(-I)<br />

coty=-I,O


<strong>Trigonometry</strong> <strong>Solutions</strong> <strong>Manual</strong><br />

6.1 Inverse Trigonometric Functions 223<br />

...<br />

28. y = arcsec 2<br />

Jr<br />

sec y = 2,0:S y:S x, y:;t: 2"<br />

S· Jr 2 x<br />

mce sec 3" = , y = 3"'<br />

29. O=arctan(-I)<br />

tan 0 = -1, - 90° < 0 < 90°<br />

ois in quadrant IV. The reference angle is 45°.<br />

Thus, 0= -45°.<br />

30. 0 = arccos(- ~)<br />

1<br />

cosO =--,Oo:s O:S 180°<br />

2<br />

ois in quadrant II. The reference angle is 60°.<br />

Thus, 0 = 180° - 60° = 120°.<br />

31. 0 = arcsin(- ~)<br />

sin 0 = - .J3 ,-90° :s 0 :s 90°<br />

2<br />

ois in quadrant IV. The reference angle is 60°.<br />

0=-60°.<br />

32. 0 = arcsin(- ~)<br />

sin 0 = - .fi,- 90° :s 0 :s 90°<br />

2<br />

ois in quadrant IV. The reference angle is 45° .<br />

0=-45°<br />

l(<br />

33. 9 =COC - ~)<br />

cot 0=_.J3 0°


<strong>Trigonometry</strong> <strong>Solutions</strong> <strong>Manual</strong><br />

224 Chapter 6 Inverse Trigonometric Functions and Trigonometric Equations<br />

For Exercises 43-48, be sure that your calculator is in<br />

radian mode. Keystroke sequences may vary based on<br />

the type and/or model of calculator being used.<br />

SO.<br />

43. y = arctan 1.1111111<br />

Enter 1.1111111 El ~<br />

orB ~ 1.1111111 ~<br />

Display: .83798122<br />

arctan 1.1111111 = .83798122<br />

44. y = arcsin .81926439<br />

Enter: .81926439 G ~<br />

orB ~.81926439 ~<br />

Display: .96012698<br />

arcsin .81926439 = .96012698<br />

Domain=(~, -~]u [~, DO)<br />

Range=[-~, 0)U(0, ~]<br />

45.<br />

y=coC 1(-.92170128)<br />

Enter:<br />

.92170128 B §) B ~ m [nl<br />

or<br />

B ~ GJ .92170128 0 m Inl ~<br />

Display: 2.3154725<br />

coC I (-.91270128) = 2.3154725<br />

46. y=sec-I(-1.2871684)<br />

Enter: -1.2871684 §) G §)<br />

orB §J GJ 1.2871684 0<br />

Display: 2.4605221<br />

sec-I (-1.2871684) =2.4605221<br />

47. y =arcsin .92837781<br />

Enter: .92837781 B 8<br />

or B ~ .92837781 ~<br />

Display: 1.1900238<br />

arcsin .92837781 = 1.1900238<br />

48. y =arccos .44624593<br />

Enter: .44624593 ~ §)<br />

or B §) .44624593 ~<br />

Display: 1.1082303<br />

arccos .44624593 = 1.1082303<br />

49.<br />

51.<br />

52.<br />

53.<br />

Domain: (-DO, - 2] U [2, DO)<br />

Range=[0, ~) U (~ , 1r]<br />

flf-I(x)] = ~ x; 4)<br />

=\X;2)_2<br />

=x+2-2<br />

=x<br />

f-I[f(x)] =f- I[3x - 2]<br />

(3x-2)+2<br />

=<br />

3<br />

3x<br />

=­<br />

3<br />

=x<br />

In each case the result is x. The graph is a straight<br />

line bisecting the first and third quadrants<br />

(y =x).<br />

Domain: (- DO, DO)<br />

Range: (0, 2n)<br />

It is the graph of y = x.<br />

224


<strong>Trigonometry</strong> <strong>Solutions</strong> <strong>Manual</strong><br />

6.1 Inverse Trigonometric Functions 225<br />

54.<br />

tan(arccos ~) =tan m=~<br />

55.<br />

It does not agree because the range of the inverse<br />

f . . ( 1r 1r)<br />

tangent unction IS -"2'"2 ,not (-.co, 00),as was<br />

the case in Exercise 53.<br />

58. sin ( arccos±)<br />

1 1<br />

Let 0 = arccos-, so that cosO =-.<br />

4 4<br />

Since arccos is defined only in quadrants I and II,<br />

and ± is positive, 0 is in quadrant I. Sketch 0 and<br />

label a triangle with the side opposite 0 equal to<br />

~42 _1 2 =..m.<br />

y<br />

---!...;.....L---_ x<br />

.( 1). 0 ..m<br />

S10 arccos'4 =S1O =7<br />

56.<br />

59. cos(tan-I(-2))<br />

Let co =tan- I (-2), so that tan m =-2. Since<br />

tan -I is defined only in quadrants I and IV, and<br />

-2 is negative, co is in quadrant IV. Sketch CO and<br />

label a triangle with the hypotenuse equal to<br />

~(_2)2 + 1 =$.<br />

y<br />

57. ta{arccos ~)<br />

3 3<br />

Let m=arccos '4' so that cos m='4' Since<br />

arccos is defined only in quadrants I and II, and<br />

~ is positive, cois in quadrant I. Sketch co and<br />

label a triangle with the side opposite CO equal to<br />

~42 _3 2 =.,fi.<br />

y<br />

__-!'-..I.-_--"L...-_ x<br />

3<br />

60.<br />

---r-r-.---- x<br />

- 2<br />

cos(tan-1(_2)) =cos co = $<br />

5<br />

Le 0 . -l( 1) that si 1<br />

t = sm - S ' so at S10 0 =- S ·<br />

Since arcsin is defined only in quadrants I and IV,<br />

1<br />

and -Sis negative, 0 is in quadrant IV. Sketch (J<br />

and label a triangle with the side adjacent to 0<br />

equal to ~52 _(_1)2 =..{i4 = 2-/6.<br />

225


<strong>Trigonometry</strong> <strong>Solutions</strong> <strong>Manual</strong><br />

226 Chapter 6 Inverse Trigonometric Functions and Trigonometric Equations<br />

y<br />

--;-=::-,--...,...-'X<br />

-I<br />

sec( Sin<br />

.-I( -'5 I)J Ll 5 5../6<br />

=secl7=2J6=12<br />

Let m= arcsin ( - ~). so that sin m = - ~. Since<br />

arcsin is defined only in quadrants I and IV, and<br />

-~ is negative, mis in quadrant IV. Sketch m<br />

3<br />

and label a triangle with the side adjacent to m<br />

equal to ~32 - (_2)2 = $.<br />

y<br />

co{arcsin(- %)J= cot m= -;:<br />

62. cos (arctan j)<br />

Let 6 = arctan ~, so that tan 6 = ~. Since arctan<br />

3 3<br />

is defined only in quadrants I and IV, and ~ is<br />

3<br />

positive, 6 is in quadrant I. Sketch 6 and label a<br />

triangle with the hypotenuse equat to<br />

~82 +3 2 =m.<br />

y<br />

---f-"--..J..---__ x<br />

8<br />

co{arctan~) = cos 6 = ~ = 3m<br />

3 v73 73<br />

63. sec(sec- 12)<br />

Since secant and inverse secant are inverse<br />

functions, sec(sec- 12) = 2.<br />

64. esc (esc-I .J2) = esc ==.J2<br />

65. arccos (cos 1r) = arccos .J2 = 1r<br />

4 2 4<br />

66. arctan(tan(- =)) = arctan(-I) = - =<br />

67. arcsin(sin 1r) = arcsin .,fj = 1r<br />

323<br />

68. arccos(cos 0) = arccos 1 = 0<br />

69. sin(2 tan-I 1:)<br />

Let m= tan-I .!3., so that tan t» = .!3..<br />

5 5<br />

Since tan-I is defined only in quadrants I and IV,<br />

an d -<br />

12 . . . .. d I<br />

IS positive, m IS In qua rant .<br />

5<br />

Sketch m and label a right triangle with the<br />

hypotenuse equal to ~122 + 52 = 13.<br />

y<br />

12<br />

---+~--I---. X<br />

. 12<br />

slnm=­<br />

13<br />

5<br />

cos m=­ 13<br />

sin (2 tan-I 1:) = sin(2m)<br />

=2sin m cos m<br />

=2 (.!3.X2.-)<br />

13 13<br />

120<br />

=­<br />

169<br />

226


<strong>Trigonometry</strong> <strong>Solutions</strong> <strong>Manual</strong><br />

6.1 Inverse Trigonometric Functions 227<br />

70. Cos(2sin­ 1 ±)<br />

Let 0 =sin- 1 !-, so that sinO =!-.<br />

4 4<br />

Since sin- 1 is defined only in quadrants I and IV,<br />

and !- is positive, 0 is in quadrant I. Sketch 0<br />

4<br />

and label a triangle with the side adjacent to 0<br />

equal to ~42 _1 2 =.Jl5.<br />

y<br />

y<br />

--I'-.l----L---.x<br />

3<br />

cosro =­<br />

5<br />

. 4<br />

Slnro =­<br />

5<br />

4<br />

co{2 arctan ~) =cos(2ro)<br />

_--!"oo::;,,=.o-=_-..L__ x<br />

cO{2sin-<br />

1±) =cos 20<br />

71. co{2 arctan ~)<br />

=1-2sin 2 0<br />

=1-2(±Y<br />

7<br />

=­<br />

8<br />

Let ro=arctan i, so that tan ro =i. Since<br />

3 3<br />

arctan is defined only in quadrants I and IV, and<br />

i is positive, ro is in quadrant I. Sketch ro and<br />

3<br />

label a triangle with the hypotenuse equal to<br />

~42 +3 2 =5.<br />

72. tan(2cos­ 1 ±)<br />

=cos 2 to - sin 2 ro<br />

=(~r -(~r<br />

9 16<br />

=--­<br />

25 25<br />

7<br />

=-­<br />

25<br />

1<br />

Let 0 =cos- 1 !-, so that cos 0 =!-.Since cos­<br />

4 4<br />

is defined only in quadrants I and II, and !- is<br />

4<br />

positive, 0 is in quadrant I. Sketch 0 and label a<br />

riangle with the side opposite 0 equal to<br />

~42 _1 2 =.Jl5.<br />

y<br />

e<br />

1<br />

$5<br />

tan 0 =-=.Jl5<br />

1<br />

227


<strong>Trigonometry</strong> <strong>Solutions</strong> <strong>Manual</strong><br />

228 Chapter 6 Inverse Trigonometric Functions and Trigonometric Equations<br />

74. cos (2 arctan (-2»<br />

tan(2 cos -I i) = tan 29 Let 9 = arctan (-2), so that tan 9 = -2.<br />

2 tan ()<br />

- l-tan 29<br />

2.J15<br />

= 2<br />

1-(.J15)<br />

2.Jl5<br />

=-­<br />

-14<br />

J15<br />

=--­<br />

and -2 is negative, 9 is in quadrant IV.<br />

Sketch 9 and label a triangle with the hypotenuse<br />

equal to ~(_2)2 +1 2 =$.<br />

Since arctan is defined only in quadrants I and IV,<br />

y<br />

7<br />

73. sin(2 cos-I ~)<br />

Let CO = cos-I .!., so that cos co = .!.. Since<br />

5 5<br />

cos-I is defined only in quadrants I and II, and<br />

t is positive, (0 is in quadrant I. Sketch co and<br />

label a triangle with the side opposite CO equal to<br />

~52 _1 2 =.J24 = 2../6.<br />

y<br />

1 $<br />

cos9="J5=s<br />

cos (2 arctantzj) =cos 29<br />

= 2cos 2 9-1<br />

=2(JsY -I<br />

=~-1<br />

5<br />

3<br />

=-­<br />

5<br />

75. tan(2 arcsi{­ ~)) _<br />

---~---_JC<br />

. 2../6<br />

SIn (0=-­<br />

5<br />

1<br />

cos co=­<br />

5<br />

sin(2 cos-I ~) = sin2co<br />

= 2sin co cos co<br />

={2~X~)<br />

4../6<br />

=-­<br />

25<br />

Let co =arcsin(- ~). so that sin co = ­ ~ .<br />

Since<br />

arcsin is defined only in quadrants I and IV, and<br />

-~ is negative, Cl> is in quadrant IV. Sketch Cl><br />

and label a triangle with the side adjacent to Q)<br />

equal to ~52 _(_3)2 =4.<br />

y<br />

--~r----~"'''<br />

3<br />

tan (0 =-­<br />

4<br />

228


<strong>Trigonometry</strong> <strong>Solutions</strong> <strong>Manual</strong><br />

6.1 Inverse Trigonometric Functions 229<br />

tan(2 arcsin(­ ~) ) = tan(2m )<br />

76. sin(2 arccos %-)<br />

2tanm<br />

-1-tan 2m<br />

2(-t)<br />

= 2<br />

I-(-t)<br />

-i 16<br />

= 1--& ·16<br />

-24<br />

=-­<br />

16-9<br />

24<br />

=-­ 7<br />

2 2<br />

Let 0 = arccos ­ , so that cos 0 = ­ .<br />

9 9<br />

Since arccos is defined only in quadrants I and II,<br />

and ~ is positive, 0 is in quadrant I.<br />

9<br />

Sketch 0 and label a triangle with the side<br />

opposite 0 equal to ~92 - 2 2 =.,fi7.<br />

y<br />

77. sin(sin-I~+tan-I(-3»)<br />

Le<br />

. -I 1 1r < e: 1r<br />

t ml =sm 2' -'2 - ml ~'2.<br />

. 1 . . dr I<br />

sm ml = 2; CO) IS 10 qua ant .<br />

y<br />

.J3.<br />

cosml =-, smml<br />

2<br />

1<br />

=­ 2<br />

Let m2 = tan­ I(-3), - ~ < m2


<strong>Trigonometry</strong> <strong>Solutions</strong> <strong>Manual</strong><br />

230 Chapter 6 Inverse Trigonometric Functions and Trigonometric Equations<br />

78. cos(tan-I 2.-cot-I i)<br />

12 3<br />

-I 5 f3 -1 4<br />

Let a = tan -, = cot ­<br />

0<br />

12 3<br />

a and f3 are both in quadrant 1.<br />

y<br />

y<br />

y<br />

12<br />

--4--1,,-L--.. x<br />

12<br />

o 12<br />

SIn COl =­<br />

13<br />

3 5<br />

__-1";......1;;_...1--_.. X COSC02 =­<br />

4<br />

13<br />

3 5)<br />

0<br />

cos arCSIn - + arccos­<br />

(<br />

o 5 12<br />

SIna=- cosa=-<br />

5 13<br />

13' 13 = cos(COl + co2)<br />

SIn 0f33 =- cos f34 =­<br />

= cos COl cos CO2 - sin COl sin co2<br />

5' 5<br />

4 ) =(~XI53)-(~X~~)<br />

- I 5 -1<br />

{<br />

12 3 20 36 16<br />

co tan --cot ­<br />

=---=-­<br />

=cos(a- {3) 65 65 65<br />

= COS a cos f3 + sin asin {3<br />

=C~X~)+C53X~)<br />

80.<br />

63<br />

=­<br />

tan(arccos ~ - arcSin(-~))<br />

65 Leta = arccos ~, f3=arcSin(-~}<br />

79. coJ arcsin ~ + arccos2.)<br />

'\ 5 13<br />

­<br />

0 3 TC TC<br />

Let col =arcSIn-, - - S COl S<br />

5 2 2 ° 2<br />

o 3 0 0 dr I<br />

SInCOI =-, col IS In qua ant<br />

5<br />

y<br />

0<br />

y<br />

y<br />

--k-~-=--.,....-'x<br />

4<br />

1 3<br />

tan a = - tan{3=-­<br />

.[3 ' 4<br />

tan(arccos ~ - arcSin(-~))<br />

4<br />

COSCOI =­<br />

5<br />

o 3<br />

smcol =­<br />

5<br />

5<br />

Let co2 =arccos-, 0 S CO S TCo<br />

13<br />

5 0 0<br />

COS co2 =-, co2 IS d I<br />

In qua rant 0<br />

13<br />

230


<strong>Trigonometry</strong> <strong>Solutions</strong> <strong>Manual</strong><br />

6.1 Inverse Trigonometric Functions 231<br />

=tan(a- /3)<br />

tana- tan/3 .<br />

= I+tana tan/3<br />

_ i-(-t)<br />

- I+(JJ)(-t)<br />

4+J.J3<br />

= 4.[j<br />

W-3<br />

W<br />

4+3$<br />

= 413-3<br />

4+3.J3 -3-4$<br />

=-3+4J3 ' -3-4$<br />

-12 - 25$- 36<br />

=<br />

9-48<br />

-48-25$<br />

=<br />

-39<br />

48+25$<br />

=<br />

39<br />

For Exercises 81-84, be sure that your calculator is in<br />

degree mode. Keystroke sequences may vary based on<br />

the type and/or model of calculator being used.<br />

81. cos(tan-1.5)<br />

Enter:5~ ~ ~<br />

or§] 8 ~.5~<br />

cosuan" .5) =.894427191<br />

82. sin(cos-1.25) .<br />

Enter: .25 ~ ~ ~<br />

orE! 8 ~ .251§l<br />

sin(cos- 1 .25) =.9682458366<br />

83. tan (arcsin .12251014)<br />

Enter: .122510148 El ~<br />

or~ 8 ~ .12251014<br />

§]<br />

tan (arcsin .12251014) = .1234399811<br />

84. cot(arccos.58236841)<br />

Enter: .58236841 ~ §J ~ \<br />

or(~ ~ §] .58236841)<br />

~ ~<br />

cot(arccos .58236841)=.716386406<br />

85. sin (arccos u)<br />

Let OJ = arccos u, o.s; OJ .s; n.<br />

u<br />

cosOJ=u=­ 1<br />

Ifu > 0, then OJ is in quadrant I.<br />

If u < 0, then OJ is in quadrant II.<br />

In either quadrant, sin OJ > o.<br />

y<br />

u<br />

y=~<br />

Since y > 0, from the Pythagorean theorem,<br />

y =~I-u2.<br />

2<br />

~1- u r:--:;- 2<br />

sin (0 = =" 1-u<br />

1<br />

2<br />

Then sin (arccos u)= sin OJ= ~1 - u •<br />

86. tan (arccos u)<br />

n<br />

Let s =arccos u, 0 .s; s .s; 1C, s '# -.<br />

2<br />

Then coss=u=!!...<br />

1<br />

If u > 0, then s is in quadrant I.<br />

If u < 0, then s is in quadrant II.<br />

y<br />

y=~ y =~<br />

< )<br />

u<br />

uo<br />

In either quadrant, y > O.<br />

2<br />

By the Pythagorean theorem, y =~1- u .<br />

~1_u2<br />

tans =...:....:.--.:.:.­<br />

u<br />

~1-u2<br />

tan(arccosu) = tans = ...:....:.--.:.:.­<br />

u<br />

231


<strong>Trigonometry</strong> <strong>Solutions</strong> <strong>Manual</strong><br />

232 Chapter 6 Inverse Trigonometric Functions and Trigonometric Equations<br />

87. cot (arcsin u)<br />

Let CO = arcsin u, ­ 2E. S; CO S; 2E.. CO ;t: O.<br />

2 2<br />

Then sin CO = u =!!...<br />

1<br />

Ifu > 0, then co is in quadrant I and<br />

cot CO > O.<br />

Ifu < 0, then co is in quadrant IV and<br />

cot co < O.<br />

89. sin(sec-I~)<br />

Let CO = sec-I!!.., 0 S; co S; TC, co;t: TC •<br />

2 2<br />

Since co is in quadrant I or II, sin CO > O.<br />

y<br />

u<br />

y=~<br />

y ="';';-4<br />

u<br />

. ~<br />

Sin co = lui<br />

88.<br />

~1_U2<br />

cot CO = -'--­<br />

u<br />

~1_u2<br />

Then cot( arcsin u) = cot CO =<br />

u<br />

cos (arcsin u)<br />

Let s = arcsinu, _!E. S; u S; TC •<br />

2 2<br />

Then sin s = u =!!...<br />

1<br />

Ifu > 0, then s is in quadrant I.<br />

If u < 0, then s is in quadrant IV.<br />

y<br />

.<br />

90.<br />

Th . ( -IU). ~<br />

en Sin sec 2' =SID CO = Iu I .<br />

co{tan-I~)<br />

-I 3 TC TC<br />

Let s =tan - - - < S < -.<br />

u' 2 2<br />

Since s is in quadrants I or IV, cos s > O.<br />

y<br />

U,U>O<br />

----tE-+---:-.......--x<br />

---f-;+-~-+-_x<br />

U,U O.<br />

coss= = 2<br />

~u2 +9 u +9<br />

x =~I_u2<br />

~1_u2 ~<br />

co { tan -13) -= ~~ 2<br />

coss= ="'II-u­<br />

u u +9<br />

1<br />

cos(arcsinu) =coss = ~I- u 2<br />

232


<strong>Trigonometry</strong> <strong>Solutions</strong> <strong>Manual</strong><br />

6.1 Inverse Trigonometric Functions 233<br />

Thus. 9 = arcsin .fi = 45°.<br />

91. tan(arcsin ~J 2<br />

. u 2 +2 The equation of the asymptote is 9 = 45°.<br />

. U Tr Tr<br />

Let a> =arcsm I •- - ~ a> ~ - .<br />

'Vu 2 +2 2 2 1<br />

Then sin a> = ~.<br />

2+2<br />

u<br />

Ifu > O. then a> is in quadrant I and tan a> > O.<br />

If u < O. then a> is in quadrant IV and a> < O.<br />

y<br />

..[.Tt2<br />

u<br />

(b) x=2. 9= tanu<br />

u<br />

u.Ji<br />

tan a> = .fi =-2­<br />

96- 9 tan- (_x)<br />

x 2 +2<br />

(a) x= 1. 9= tan­<br />

1 (+-)<br />

1 +2<br />

= tan-l(~)<br />

= 18°<br />

1 (+-)<br />

2 +2<br />

=tan-l(~)<br />

= 18°<br />

(c) x = 3. 9 = tan-1(-;-)<br />

. u J u u.fi<br />

3 +2<br />

tan arcsm = tan CO = """"E =-­<br />

( ~u2 +2 ",2 2 1 31)<br />

= tan­ C<br />

= 15°<br />

92.<br />

1+ 1 2<br />

(d) tan(9+a)=-=x<br />

x<br />

1<br />

tana=x<br />

tan 9 + tan a<br />

93. All values in the interval [0. Tr] tan(9 + a)=----­<br />

I-tan9tana<br />

94. All values in the interval [-1. 1] 2 tan9+f<br />

;= I-tan9(f) ..<br />

42<br />

95. 2<br />

(a) 9 = arcsin /--."......--- 2 xtan9+1<br />

2(42 2) + 64(0) -=---­<br />

x x-tan9<br />

. 1 2(x - tan 9) =x(x tan 9+ l)<br />

= arcsm .Ji<br />

. .fi<br />

=arcsm­<br />

2<br />

2x - 2 tan 9 =x 2 tan 9 + x<br />

2x - x =x 2 tan 9 + 2 tan 9<br />

x =tan9(x 2 +2)<br />

x<br />

tan9=-2­<br />

y2 x +2<br />

(b) 9 = arcsin ./-...,.--­<br />

=arcsin IT V~<br />

As y approaches 00.<br />

2y2 +64(6) 9 = tan- I (+-)<br />

x +2<br />

v 2 (l.fi<br />

2v 2 + 384 ="';2 =2"'<br />

233


<strong>Trigonometry</strong> <strong>Solutions</strong> <strong>Manual</strong><br />

234 Chapter 6 Inverse Trigonometric Functions and Trigonometric Equations<br />

(e)<br />

If we graph Yl =tan- I (+ )using a<br />

x +2<br />

graphing calculator, the maximum value of<br />

the function occurs when x is 1.4142 m.<br />

y = tan- 1 (x2 : 2)<br />

o 10<br />

99. Since the diameter of the earth is 7927 miles at<br />

the equator, the radius of the earth is<br />

3963.5 miles . Then<br />

cos 6 = 3963.5 = 3963.5 and<br />

20, ()()() + 3963.5 23,963.5<br />

6 =arcco{ 3963.5 ) "" 80.480<br />

23,963.5<br />

The percent of the equator that can be seen by the<br />

sate II ' . 26 00 2(80.48)<br />

=.<br />

44 701.<br />

ite rs -·1 =<br />

-10<br />

360 360<br />

· 1<br />

97. a= 2arcsmm<br />

Dgl:J:ll"~lS1 ' Y:;3390&Jl .<br />

-.5<br />

Radian mode<br />

(a) m= 1.2<br />

a = 2 arcsin(_I_) =113°<br />

1.2<br />

(b) m =1.5<br />

a = 2arcsin(_I_) =84°<br />

1.5<br />

(c) m = 2<br />

a = 2arcsin(i) = 60°<br />

(d) m =2.5<br />

a = 2 arcsin(_I_) =47°<br />

2.5<br />

98. Let a be the triangle to the "right" of 6 and let p<br />

be the angle to the "left" of 6. Then<br />

p+6+a = tr (since the angles sum to z radians)<br />

and 6= tr- P- a.<br />

150 (150)<br />

tan a =--:;-' so a =arctan --;­<br />

tanp =~, so p=arctan( 75 )<br />

l00-x<br />

l00-x<br />

As a result,<br />

6=tr-p-a<br />

=tr - arctan( 75 ) _ arctan( 150).<br />

l00-x<br />

x<br />

100. The amount of oil in the tank is 20 times the area<br />

shaded in the cross-sectional representation. The<br />

shaded area is the area of the sector of the circle<br />

defined by 26 (let 26 be the measure of the angle<br />

depicted in the figure) minus the area of the<br />

triangles.<br />

Section 6.2<br />

Exercises<br />

I. D<br />

2. C<br />

3. A<br />

4. B<br />

Since tan 6 = ../8 =../8, 6 =arctan../8<br />

1<br />

Using the Pythagorean theorem, we see that<br />

b 2 + 1 2 =3 2<br />

b 2 +1 =9<br />

b 2 =9-1 =8<br />

b =../8, where b is the base ofone triangle.<br />

The area of both triangles is<br />

A~ =2 ·ibh=2·i(../8~1)=../8.<br />

Then the sector area is<br />

As = .!-(26)r 2 = (arct~m ../8)(3)2 (with 6 in<br />

2<br />

radians). Thus, the volume of oil in the tank is<br />

V oil =20(A s - A~)<br />

= 20 (9 arctan(../8) -../8)<br />

= 20(11.08 - 2.83)<br />

"" 165 cubic ft<br />

234


<strong>Trigonometry</strong> <strong>Solutions</strong> <strong>Manual</strong><br />

6.2 Trigonometric Equations I 235<br />

5. From Example I .<br />

I<br />

cosO = --<br />

2<br />

or cosO = 1<br />

0= 120°. 240° 0 = 0°<br />

Thus, all solutions can be represented as<br />

0= 360°·n. 120°+ 360° · n. 2400+360°·n<br />

where n is an integer.<br />

6. From Example 4.<br />

tan x = 1 or tan x = -2<br />

rr 5rr<br />

x="4'""4 x=2.0.5.2<br />

Thus. all solutions can be represented as<br />

rr 51r<br />

x = - + 2nrr. - + 2nrr. 2.0 + 2nrr•5.2 + 2nrr<br />

4 4<br />

where n is an integer.<br />

7. 2 cot x + 1= -1<br />

2cotx =-2<br />

cot x =-1<br />

3rr 7rr<br />

x=­- 4 • 4<br />

8. sinx+2 = 3<br />

sinx = 1<br />

n<br />

x=­ 2<br />

9. 2sinx+3 =4<br />

2sinx = 1<br />

. 1<br />

smx=­<br />

2<br />

n 5rr<br />

x=- - 6' 6<br />

1O.2secx+l=secx+3<br />

sec x = 2<br />

rr 5rr<br />

x=- ­<br />

3' 3<br />

11. tan 2 x+3=0<br />

2<br />

tan x =-3<br />

tanx=±H<br />

The square root of a negative number is not a real<br />

number.<br />

No solution.<br />

12. sec 2 x+2 =-1<br />

2<br />

sec x =-3<br />

secx=±H<br />

The square root of a negative number is not a real<br />

number.<br />

No solution<br />

13. (cot x -1)(..J3cot x + I) = 0<br />

cot x-I = 0 or ..J3cot x + I = 0<br />

cotx = I ..J3cotx =-1<br />

1r 51r I<br />

cot x = - ..J3<br />

x = '4' ""4<br />

..J3<br />

cotx=-­<br />

3<br />

21r 51r<br />

x=- ­<br />

3 • 3<br />

1r 21r 51r 51r<br />

x=- - -­<br />

4' 3 • 4 • 3<br />

14. (cscx+2)(cscx-.J2)=0<br />

esc x + 2 = 0 or esc x -.J2 = 0<br />

cscx =-2 cscx =.J2<br />

71r 1m 1r 31r<br />

x=- - x=-­<br />

6' 6 4 ' 4<br />

1r 31r 71r 111r<br />

x=--- ­<br />

4' 4 • 6 ' 6<br />

15. cos 2 x+2cosx+l=O<br />

x= 1r<br />

(cos x + 1)2 = 0<br />

cosx+l=O<br />

cosx =-1<br />

16. 2cos 2 x -.J3cos x = O·<br />

cosx(2cosx-..J3) = 0<br />

cos x = 0 or 2cosx -..J3 = 0<br />

x=-<br />

1r<br />

-<br />

31r<br />

2cosx=<br />

..J3<br />

3<br />

2' 2<br />

..J3<br />

cosx=­<br />

2<br />

x 111r<br />

x=- ­<br />

6' 6<br />

111r<br />

6<br />

17. -2sin 2 x = 3sinx + I<br />

2sin 2 x +3sinx + 1= 0<br />

(2sin x + I)(sinx + 1)= 0<br />

2 sin x + I = 0 or sin x + I = 0<br />

sin x =_.!. sinx=-I<br />

2<br />

71r l11r<br />

3rr<br />

x=- - x = ­<br />

6 ' 6 2<br />

71r 31r l11r<br />

x=- ­<br />

6' 2' 6<br />

235


<strong>Trigonometry</strong> <strong>Solutions</strong> <strong>Manual</strong><br />

236 Chapter 6 Inverse Trigonometric Functions and Trigonometric Equations<br />

18. tan 3 x = 3tanx 23. 2 sin 0 - I = esc 0<br />

tan 3 x -3tanx = 0<br />

tanx(tan 2 x ­ 3) = 0<br />

tanx = 0 or tan 2 x -3 = 0<br />

2sinO-1 = _1_<br />

sinO<br />

2sin 20-sinO=1<br />

x=O, n tan 2 2sin 2 O-sinO-1 = 0<br />

x=3<br />

(2sin 0 + I)(sin0 -I) = 0<br />

tanx = ±..f3 2sin0 + I = 0 or sin 0 -I = 0<br />

rr 2rr 4rr 5rr<br />

sinO = _! sinO = I<br />

x=3'3'3'3<br />

2<br />

0= 210°,330° 0 = 90°<br />

n 2rr 4rr 5rr<br />

0=90°, 210°, 330°<br />

x=O, 3' 3' n, 3' 3<br />

2 24. tan0 + I = ..f3+..f3 cot 0<br />

19. 2cos 4 x = cos x<br />

2cos 4 x-cos 2 x=O<br />

tan 0 + I = ..f3+ ..f3<br />

cos 2 x(2 cos2 x -I) = 0 or 2cos 2 x-I=O<br />

tan 0<br />

tan 2 0+ tan 0 =..f3 tanO+..f3<br />

cos 2 x =0<br />

cos 2 x=!<br />

2 tan 2 O+(I-..f3)tanO-.J3 = 0<br />

.J2<br />

cosx=O cosx=±­<br />

2<br />

(tan0 - .J3}


<strong>Trigonometry</strong> <strong>Solutions</strong> <strong>Manual</strong><br />

6.2 Trigonometric Equations I 237<br />

28. sin 2 OcosO = cosO<br />

29.<br />

sin 2 0 cos 0 - cos 0 = 0<br />

cosO(sin 2 0 -1) = 0<br />

cosO=O or sin 20-1=0<br />

0=90°, 270° sin 20 = 1<br />

2tan 20sinO-tan20=0<br />

sinO= ±1<br />

IfsinO = I, 0 = 90°.<br />

IfsinO=-l, 0=90°, 270°<br />

tan 2 0(2sinO -1) = 0<br />

tan<br />

20=0<br />

or 2sinO-1=0<br />

tan 0 = 0 sin 0 = 1­<br />

2<br />

0=0°, 180° 0 = 30°, 150°<br />

0=0°,30°,150°,180°<br />

30. sin 2 Ocos 2 0 = 0<br />

sin<br />

20=0<br />

or cos20=0<br />

sinO =0<br />

cosO = 0<br />

0=00,180° 0 = 90°,270°<br />

0=00,90°, 180°, 270°<br />

31. sec 2 otan 0 = 2 tan 0<br />

sec 2 0 tan 0 - 2 tan 0 = 0<br />

tan 8(sec 2 0 - 2) = 0<br />

tan 0 = 0 or sec 2 0 = 2<br />

secO=±.J2<br />

0=0°, 180° 0 =45°, 135°, 225°, 315°<br />

0= 0°, 45°,135°,180°,225°,315°<br />

20-sin20=0<br />

32. cos<br />

cos 2 0 - (1- cos 2 0) = 0<br />

2cos 2 0-1 = 0<br />

cos 2 0 = 1­<br />

2<br />

cosO=±{f<br />

.J2<br />

cosO=±­<br />

2<br />

.J2<br />

If cosO = -,0= 45°,315°<br />

2<br />

.J2<br />

If cosO = --,0= 135°,225°<br />

2<br />

0=45°, 135°, 225°, 315°<br />

33. sinO+cosO = 1<br />

(sinO + cosO)2 = 1 2<br />

sin 2 0+2sinOcosO+ cos 2 0 = 1<br />

1+ 2 sin 0 cos 0 = I<br />

2sinOcosO =0<br />

sinOcosO =0<br />

sin 0 = 0 or cos 0 = 0<br />

0=0°, 180 0 0 =90°, 270°<br />

Since the solution was found by squaring an<br />

equation, we must check that each proposed<br />

solution is a solution of the original equation .<br />

Since sin 180° + cos 180° = -1 and<br />

sin 270° + cos 270° = -1, 180° and 270° are not<br />

solutions.<br />

0=0°,90°<br />

34. sec 0 - tan 0 =1<br />

_1__ sinO = 1<br />

cosO cosO<br />

I-sinO =1<br />

cosO<br />

1-sin 0 = cos 0<br />

1-sin 0 = ±~""I---si-n-2-0<br />

(1-sinO)2 =(±~I-sin2 or<br />

1-2sinO+sin 2 0 = I-sin 2 0<br />

2 sin 2 0 - 2 sin 0 = 0<br />

2sin O(sin0 -1) =0 .<br />

sin 0 =0 or sin 0 =1<br />

0=0° 0=90°<br />

Since the solution was found by squaring an<br />

equation, we must check that each proposed<br />

solution is a solution of the original equation.<br />

sec 90° - tan 90° is undefined. Thus, 90° is not a<br />

solution.<br />

0=0°.<br />

3~ 3~n20-~nO=2<br />

3sin 2 0 - sin 0 - 2 = 0<br />

(3sinO+ 2)(sinO -1) = 0<br />

3sin 0 + 2 =0 or sin 0 -1 = 0<br />

sin 0 = - ~ sin 0 = 1<br />

3<br />

0= 221.8°, 318.2° 0 = 90°<br />

0=90°,221.8°,318.2°<br />

237


<strong>Trigonometry</strong> <strong>Solutions</strong> <strong>Manual</strong><br />

238 Chapter 6 Inverse Trigonometric Functions and Trigonometric Equations<br />

....<br />

36.<br />

37.<br />

38.<br />

39.<br />

40.<br />

2 tan 9 = 1<br />

3-tan 29<br />

2 tan 9 = 3 - tan 2 9<br />

tan 2 9+2 tan9- 3 = 0<br />

(tan 9 -1)(tan 9 + 3) = 0<br />

tan 9 -1 = 0 or tan 9 + 3 = 0<br />

tan 9 = 1 tan 9 =-3<br />

9 = 45°,225° 9 = -71.6°<br />

Since the solution -71.6° is not in the interval<br />

[0°,360°), we must use it as a reference angle to<br />

find solutions in the interval.<br />

9 = -71.6° + 180° = 108.4°<br />

9 = -71.6° + 360° = 288.4°<br />

9 = 45°, 108.4°, 225°, 288.4°<br />

sec 2 9 = 2tan9+4<br />

tan 2 9+ 1= 2tan9+4<br />

tan 2 9 -2tan9-3 = 0<br />

(tan 9 - 3)(tan9 + 1)= 0<br />

tan9-3=0<br />

or tan9+1=0<br />

tan 9 = 3 tan 9 = -1<br />

9 = 71.6°, 251.6° 9 = 135°, 315°<br />

9= 71.6°,135°,251.6°,315°<br />

5sec 2 0 = 6secO<br />

5sec 2 0 - 6secO = 0<br />

secO(5secO-6) = 0<br />

sec0 = 0 or sec9-6 = 0<br />

No solution sec0 = ~<br />

5<br />

o= 33.6°, 326.4°<br />

9 = 33.6°, 326.4°<br />

3cot 2 0 = cot 9<br />

3cot 2 9 - cot 9 = 0<br />

cot 0(3cot 9-1) = 0<br />

cot 9 = 0 or 3cot 0 - 1= 0<br />

0=90°,270° 3cotO =1<br />

1<br />

cotO=­ 3<br />

9 = 71.6°, 251.6°<br />

0= 71.6°, 90°,251.6°,270°<br />

8cos9 = cot9<br />

cos 9<br />

8cosO=-­<br />

sinO<br />

8cosOsin 9 = cosO<br />

8cosOsinO- cos 9 = 0<br />

cos 0(8 sin 0 -1) = 0<br />

cosO= 0<br />

or 8sinO-l=0<br />

0=90°,270° 8sin9 = 1<br />

. 9 1<br />

sm =­<br />

8<br />

0= 7.2°, 172.8°<br />

0= 7.2°,90°,172.8°,270°<br />

41. 9 sin 2 0 - 6 sin 9 = 1<br />

9sin 2 0-6sin9-1 = 0<br />

We use the quadratic formula with a =9, b =~ ,<br />

and c =-1.<br />

. 6 ±.J36- [4· 9(-1)]<br />

sm 0 =---'-----­<br />

2·9<br />

6±m<br />

=<br />

18<br />

1±../2<br />

=-­ 3<br />

. 1+../2<br />

If sm 0 =--= .80473787, 9 = 53.6°, 126.4°.<br />

3<br />

If sinO= 1-../2 =-.13807119, 0=-7.9°. Since<br />

3<br />

this solution is not in the interval [0°, 360°), we<br />

must use it as a reference angle to find angles in·<br />

the interval.<br />

0= 180°+1-7.9°1 = 187.9° or<br />

0= (-7.9°)+360° = 352.1°<br />

9 = 53.6°, 126.4°, 187.9°, 352.1°<br />

42. 4cos 2 0+4cosO = 1<br />

4cos 2 9 +4cosO-l = 0<br />

We use the quadratic formula with a = 4, b = 4,<br />

and c =-1.<br />

-4±~42 -4(4)(-lf<br />

cosO = --~----<br />

2(4)<br />

-4±.J32<br />

=<br />

8<br />

-4+4../2<br />

=<br />

8<br />

-1±../2<br />

=<br />

2<br />

-1 -../2 . I th 1 hi h i . ibl<br />

2 IS ess an - w lC IS an tmpossi e<br />

value for the cosine function .<br />

-1+../2<br />

So we use cosO = cosO= .20710678.<br />

2<br />

0"" 78.0°,282.0°<br />

238


<strong>Trigonometry</strong> <strong>Solutions</strong> <strong>Manual</strong><br />

6.2 Trigonometric Equations I 239<br />

43. tan 2 8 + 4 tan8 + 2 = 0<br />

We use the quadratic formula with a = I, b = 4,<br />

and c= 2.<br />

-4±"16-4·1·2<br />

tan 8 = ----:'---­<br />

2 ·1<br />

-4±.J8<br />

=<br />

2<br />

-4±2..fi<br />

=<br />

2<br />

=-2±..fi<br />

If tan 8 = -2 +..fi = -.5857864,<br />

8 = 149.6°,329.6°.<br />

If tan 8 = -2 -..fi = -3.4142136,<br />

8 = 106.3°,286.3°.<br />

8 = 106.3°, 149.6°, 286.3°, 329.6°<br />

44. 3cot 2 8 - 3cot8 -1 = 0<br />

We use the quadratic formula with a = 3, b = -3,<br />

and c =-1.<br />

-(-3)±~(-3)2-4(3)(-1)<br />

cot 8 = ---'---:...---!....:..-~-....:....:...:-.....:-<br />

2(3)<br />

3±.fil<br />

=<br />

6<br />

If cot = 3+.fil = 1.2637626, 8 = 38.4°,218.4°.<br />

6<br />

3-.fil<br />

If cot 8 = = -.26376262,<br />

6<br />

8 = 104.8°,284.8°.<br />

8 = 38.4°, 104.8°,218.4°,284.8°<br />

45. sin 2 8-2sin8+3 =0<br />

We use the quadratic formula with a = I, b = -2,<br />

and c= 3.<br />

sin 8 = 2 ± ..J4 - (4 ·1 .3)<br />

2 ·1<br />

2±.J=8<br />

=<br />

2<br />

Since .J=8 is not a real number, the equation has<br />

no real number solution.<br />

46. 2cos 2 8+ 2cos8-1 = 0<br />

We use the quadratic formula with a = 2, b = 2,<br />

and c =-1.<br />

_2±~22 -4(2)(-1)<br />

cos 8 = --"------'-..:....;.,.-..:....<br />

2(2)<br />

- 2 ±.J1i<br />

=<br />

4<br />

-2±2.J3<br />

=<br />

4<br />

-1±.J3<br />

=<br />

2<br />

-1+.J3 ° °<br />

If cos 8 = = .36602540,8 = 68.5 ,291.5 .<br />

2<br />

cos 8 = -1-.J3 = -1.3660254 is an impossible<br />

2<br />

value for the cosine function.<br />

8 = 68.5°, 291.5°<br />

47. cot 8 + 2 esc 8 = 3<br />

cos 8 +_2_=3<br />

sin8 sin8<br />

cos8+2 = 3sin8<br />

(cos8+2)2 = (3sin8)2<br />

cos 2 8+ 4cos8+4 = 9sin 2 8<br />

cos 2 8+4cos8+4 = 9(I-oos 2 8)<br />

cos 2 8+ 4cos8+4 = 9 -9cos 2 8<br />

lOcos 2 8+ 4cos8-5 = 0<br />

We use the quadratic formula with a = 10, b = 4,<br />

and c=-5.<br />

n -4± '4 2 -4(10)(-5)<br />

cos o = __V"--_---'-~'-:..<br />

2(10)<br />

= -4±M<br />

20<br />

-4±6-J6<br />

=<br />

20<br />

-2 ±3..J6<br />

=<br />

10<br />

-2+3../6<br />

If cos8 = = .53484692,<br />

10<br />

8 = 57.7°,302.3°.<br />

-2-3..J6<br />

If cos 8 = - .93484692,<br />

10<br />

8 = 159.2°. 200.8°.<br />

Since the solution was found by squaring an<br />

equation. we must check that each proposed<br />

solution is a solution of the original equation.<br />

302.3° and 200.8° do not check.<br />

8= 57.7°,159.2°<br />

239


<strong>Trigonometry</strong> <strong>Solutions</strong> <strong>Manual</strong><br />

240 Chapter 6 Inverse Trigonometric Functions and Trigonometric Equations<br />

48. 2 sin 6 = 1 - 2 cos 6<br />

(2sin6)2 = (1- 2 cos 6)2<br />

4sin 2 6 = 1-4cos6+4cos 2 6<br />

4(I-cos 2 6) = 1-4cos6+4cos 2 6<br />

4-4cos 2 6 = 1-4cos6+4cos 2 6<br />

0= -3-4cos6+8cos 26<br />

We use the quadratic formula with a =8, b =-4,<br />

and c=-3.<br />

cos6 =b -(-4)±~(-4)2 -4(8)(-3)<br />

2(8)<br />

4±M<br />

=<br />

16<br />

4±.w7<br />

=<br />

16<br />

1±..[7<br />

=-­ 4<br />

If cos6 = 1+..[7 =.91143783,6=24.3°,335.7°.<br />

4<br />

1-..[7<br />

If cos6 = --= -.41143783,<br />

4 .<br />

6= 114.3°,245.7°.<br />

Sincethe solution was found by squaringan<br />

equation, we must check that each proposed<br />

solutionis a solution ofthe original equation.<br />

24.3°and 245.7° do not check.<br />

6= 114.3°, 335.7°<br />

49. 2sin 2 x-sinx-I = 0<br />

(2sinx+ I)(sinx -I) = 0<br />

2sinx + 1= 0 or sin x -I = 0<br />

sin x = _!<br />

2<br />

7rr lIrr<br />

x=6'6<br />

rr<br />

x="2<br />

sin x = 1<br />

x = -<br />

rr<br />

+ 2nrr. ­<br />

7rr<br />

+ 2nrr. --+<br />

llrr 2<br />

nrr.w<br />

h<br />

ere n<br />

.<br />

IS<br />

2 6 6<br />

an integer.<br />

51. 4cos 2 x-I=O<br />

cos 2 x=!<br />

4<br />

1<br />

cosx=±­<br />

2<br />

rr 2rr 4rr<br />

x=-+2nrr, -+2nrr. -+2nrr.<br />

3 3 3<br />

- 5rr + 2 nn, w h ere n . .<br />

IS an integer.<br />

3<br />

52. 2cos 2 x+5cosx+2 =0<br />

(2cosx+ l)(cosx+2) = 0<br />

2cosx+I=0 or cosx+2=0<br />

1<br />

cosx=-­ x =-2<br />

2<br />

2rr 4rr<br />

x=­<br />

3 •<br />

- No solution<br />

3<br />

2rr<br />

x=-+2n1C. "'3+<br />

4rr 2<br />

n1C,<br />

h . .<br />

w ere n IS an integer.<br />

3<br />

53. cos 2 x+cosx-6=0<br />

(cosx+3)(cosx-2) =0<br />

cosx+3=0 or cosx-2=0<br />

cos x = -3<br />

No solution.<br />

cos x = 2<br />

54. sin 2 x - sin x =0<br />

sinx(sinx -I) = 0 .<br />

sinx =0 or sinx-I = 0<br />

x=O sinx=1<br />

1C<br />

x=­ 2<br />

1C ..<br />

x =n1C. "2 + 2n1C •where n is an integer.<br />

50. 2cos 2 x+cosx=1<br />

2cos 2 x+cosx-I=O<br />

(2cosx -I)(cosx + I) = 0<br />

2cosx-I=0 or cosx+I=O<br />

1<br />

cosx=- cosx=-I.<br />

2<br />

rr 5rr<br />

x=- - x=rr<br />

3' 3<br />

rr<br />

x =-+2nrr. rr+ 2nn; -+ 5rr 2 nx; were h n .<br />

IS an<br />

3 3<br />

integer.<br />

240


<strong>Trigonometry</strong> <strong>Solutions</strong> <strong>Manual</strong><br />

55. 59. P =A sin(21¢ + (D)<br />

6.2 Trigonometric Equations I 241<br />

rr<br />

(a) Letf= 261.63, A = .004, and (6=-.<br />

7<br />

Graph P =.004 sin[2rr(261.63)t + ~] on the<br />

interval 0, .005 .<br />

Forx=r, [ 7T]<br />

P(r) = .004 sin 27T(261.63)t + "7<br />

- .005<br />

(b) 0 =.004 sin[2rr(261.63)t + ~]<br />

56. The intersection method is shown on the window 0= sin(l643.87t + .45)<br />

Y2 = i 1 X<br />

1643.87t + .45 =ns; where n is an integer.<br />

nx -.45<br />

t=--­<br />

1643.87<br />

Ifn =0, then t =.000274.<br />

Ifn = I, then t = .00164.<br />

Ifn =2, then t =.00355.<br />

Ifn =3, then t =.00546.<br />

The only solutions for t in the interval<br />

[0, .005] are .00164 and .00355 .<br />

(e)<br />

We must solve the trigonometric equation<br />

P =0 to determine when P $; O. From the<br />

graph we can estimate that P$;O on the<br />

interval [.00164, .00355]:<br />

(d) P$;O implies that there is a decrease in<br />

pressure so an eardrum would be vibrating<br />

outward.<br />

60. Since fmeasures cycles per second and T<br />

measures seconds per cycle,<br />

57.2cosx-l=0<br />

2cosx =1<br />

f =..!.. or T =..!... For example, iff=5 cycles per<br />

1<br />

T f<br />

cos x =­<br />

2 sec, then T =-!. sec per cycle. Thus, T is the time<br />

rr 51r<br />

5<br />

x=3"' ""3<br />

for one complete cycle, which is the period.<br />

58. 2 sin x -1 = 0<br />

2sinx = 1<br />

. 1<br />

SlOX=­<br />

2<br />

n 5rr<br />

x="6' 6"<br />

241


<strong>Trigonometry</strong> <strong>Solutions</strong> <strong>Manual</strong><br />

242 Chapter 6 Inverse Trigonometric Functions and Trigonometric Equations<br />

16D 2<br />

61. .342DcosO + hcos 2 0 =-­ 63. e = 20sin( :t-~)<br />

Vo2<br />

Vo = 60. D = 80. h = 2<br />

(a) e =0<br />

2 16(80)2<br />

.342(80)cosO+2cos 0= 2<br />

60<br />

o= 20sin(:t -~)<br />

25_6<br />

2cos 20+27.36cosO-_ =0 Since arcsin 0 = 0, solve the following<br />

9 equation.<br />

cos2 0 + 13.68cosO __12_8 = 0 ret _ re =0<br />

9 4 2<br />

a = 1, b = 13.68. c = -128/9 ret re<br />

-=­<br />

-13.68± 1(13.68)2 -4(~) 4 2<br />

cos 0 = --'V'--- -'---'­ t =2 sec<br />

2<br />

-13.68±15.6215<br />

= (b) e =1M<br />

2<br />

cosO = .97075 or cosO =-14.65075 1M = 20 sin( :t-~)<br />

-14.65075 does not lead to a solution because it is<br />

not in the range of cos O.<br />

If cosO = .97075.0 = 14°.<br />

.J3 = sin(!!.!.. _ re)<br />

2 4 2<br />

1 S· ...[j re I h J:<br />

mce arcsm - =-. so ve t e I .<br />

10 lowing<br />

62. V = cos 21l' t, 0 ~ t ~­ . 2 3<br />

2<br />

equation.<br />

(a) V=0.cos21l't=0 ret 1l' re<br />

---=­<br />

21l't = cos- t 0<br />

4 2 3<br />

ret lOre<br />

21l't =1l'<br />

-=­<br />

4 12<br />

2<br />

10 1<br />

t=-sec=3- sec<br />

t=1:.. 3 3<br />

21l'<br />

1<br />

=-sec<br />

64. s(t) = sin t + 2 cos t<br />

4<br />

2+..[3 2 ..[j<br />

(b) V =.5, cos 21l't= .5<br />

(a) --=-+­<br />

2 2 2<br />

21l't =cos- 1 (.5)<br />

1l'<br />

=2 .'!'+ ..[j<br />

21l't =- 2 2<br />

3<br />

s: = 2 co{ ~ ) + sin( ~ )<br />

t =-2­<br />

2re<br />

1<br />

=- sec<br />

6<br />

(c) V = .25. cos 21l't = .25<br />

One such value is 1l'.<br />

3<br />

2ret = cos-1( .25)<br />

2ret = 1.3181161<br />

1.3181161<br />

t=---­<br />

21l'<br />

=.21 sec<br />

One such value is !!...<br />

4<br />

242


<strong>Trigonometry</strong> <strong>Solutions</strong> <strong>Manual</strong><br />

6.3 Trigonometric Equations II 243<br />

Section 6.3<br />

Exercises<br />

2tr 2tr Str<br />

1. x=6' 2' 6<br />

tr 4tr<br />

x = -<br />

3'<br />

tr<br />

,<br />

­<br />

3<br />

2tr 10tr 10tr<br />

2. x=16' 12' -Str<br />

5tr 5tr<br />

x=g' 6'4<br />

IS0° 630° ' 720° 930°<br />

3. (}=-- -- -- -­<br />

3' 3 ' 3 ' 3<br />

e= 60°, 210°, 240°, 310°<br />

03,<br />

4. ()=45°·3, 60° 75° ·3, 90°·3<br />

e= 135°, IS0°, 225°, 270°<br />

.J3<br />

5. cos2x=­<br />

2<br />

Since 0 ~ x < 2tr, 0 ~ 2x < 4tr.<br />

2x = tr Iltr 13tr 23tr<br />

6' 6 ' 6' 6<br />

tr lItr 13tr 23tr<br />

x= 12 ' 12' 12' 12<br />

1<br />

6. cos2x=-­<br />

2<br />

Since 0 ~ x < 2tr, 0 ~ 2x < 4n:<br />

2x = 2tr 4tr Str IOtr<br />

3' 3' 3 ' 3<br />

tr 2tr 4tr 5tr<br />

x="3' 3' 3' 3<br />

7. sin 3x =-1,<br />

Since 0 ~ x < 21C, 0 ~ 3x < 6tr.<br />

3x = 3tr 71C 1m<br />

2' 2 ' 2<br />

tr 7tr 1I1C<br />

x=2" 6' li<br />

8. sin 3x = 0<br />

Since 0 ~x < 21[, 0 ~ 3x < 6n:<br />

3x =0, n; 2tr, 3tr, 4tr, 5tr<br />

x =0 tr 2tr tr 4tr 5tr<br />

, 3' 3 ' , 3 ' 3<br />

9. 3tan3x =.J3, 0 s 3x < 6tr<br />

.J3<br />

tan3x=­<br />

3<br />

tr 7tr 13tr 19tr 25tr 31tr<br />

h=6'6'li'(5'(5'li<br />

tr 7tr 13tr 19tr 25tr 31tr<br />

x=18' 18' 18' 18' 18' 18<br />

10. cot 3x = .J3, 0 < 3x < 6tr<br />

3x = tr 7tr 13tr 19tr 25tr 31tr<br />

6' 6' 6' 6 ' (5' li<br />

tr 7tr 13tr 19tr 25tr 31tr<br />

x = IS' 18' 18' 18' 18' 18<br />

11. .,fi cos 2x = -1, 0 s 2x < 4tr<br />

-1 .,fi<br />

cos2x=-=-­<br />

.,fi 2<br />

2x = 3tr 5tr lItr 13tr<br />

4' 4' 4' 4<br />

3tr 5tr lItr 13tr<br />

x=g' g' -S-, "8<br />

12. 2.J3 sin 2x =.J3, 0 s 2x < 4tr<br />

. 2 1<br />

sm x=­ 2<br />

2x = tr Str 13tr 17tr<br />

6 ' 6' 6' 6<br />

tr 5tr 13tr 17tr<br />

x=12' 12' 12' 12<br />

0 x (;:;2 0 x 0 ",. !... < '"<br />

13. sm 2=-V ~ - sm 2' .::. 2 '"<br />

oX oX (;:;2<br />

sm- +sm - =-V ~<br />

2 2<br />

2 sin!... = .J2<br />

2 .<br />

o x .,fi<br />

sm-=­<br />

2 2<br />

x tr 3tr<br />

2=4"' 4<br />

tr 3tr<br />

x=2" 2<br />

243


<strong>Trigonometry</strong> <strong>Solutions</strong> <strong>Manual</strong><br />

244 Chapter 6 Inverse Trigonometric Functions and Trigonometric Equations<br />

14.<br />

15.<br />

16.<br />

17.<br />

18.<br />

­<br />

sin x =sin 2x<br />

sin x = 2sin xcosx<br />

sin x - 2sin xcosx =0<br />

sinx(l-2 cos x) =0<br />

sin x = 0 or 1-2cos x = 0<br />

1<br />

x =0, n cos x =<br />

2<br />

n 5n<br />

x=­ 3 ' 3<br />

n 5n<br />

3 , 3<br />

x=O, -, n ­<br />

tan 4x =0, 0 ~ 4x < 8n<br />

4x =0, n, 2n, 3n, 4n, 5n, 6n, 7n<br />

tt n 3n 5n 3n 7n<br />

x=o, 4' 2' 4' n, 4'2'4<br />

cos2x -cosx = 0<br />

(2cos 2 x-l)-cosx=O<br />

2cos 2 x-cosx-l=O<br />

(2cosx+ 1)(cosx -1) = 0<br />

2cosx+l=0 or cosx-l=O<br />

1<br />

cos x = -- cosx = 1<br />

2<br />

2n 4n<br />

x=- - x=O<br />

3' 3<br />

x=O 2n 4n<br />

'3 3<br />

2<br />

8sec .::.=4<br />

2<br />

2 x 1<br />

sec -=­<br />

2 2<br />

x ..[i<br />

sec-=±­<br />

2 2<br />

No solution.<br />

. 2<br />

sm --<br />

x 2<br />

=<br />

0<br />

2<br />

sin 2'::'= 2<br />

2<br />

sin'::'=±..[i<br />

2<br />

No solution.<br />

19.<br />

20.<br />

. x x<br />

Stn'2 = cos'2'<br />

. 2 x 2 x<br />

Sill -=cos ­<br />

2 2<br />

sin 2 '::' = l-sin 2 '::'<br />

2 2<br />

2sin 2'::'=1<br />

2<br />

, 2 x 1<br />

Sill -=­<br />

2 2<br />

sinI=±~-±~<br />

. x ..[i x n 3n n 3n<br />

If sm-=- -=- - and x=- -.<br />

22'24' 4' 2'2<br />

If sm, -x = --,<br />

..[i th ere are no so lutions unons in m the<br />

2 2<br />

interval [0, n).<br />

n 3n<br />

x=- ­<br />

2' 2<br />

Since the solution was found by squaring an<br />

equation, the proposed solutions must be checked.<br />

n<br />

x=-:<br />

2<br />

x<br />

n.J2<br />

. x . n ..[i<br />

sm-=sm-=­<br />

2 4 2<br />

cos-=cos-=­<br />

2 4 2<br />

3n<br />

x=-:<br />

2<br />

. x . 3n ..[i<br />

sm-=sm-=­<br />

2 4 2<br />

x 3n ..[i<br />

cos-=cos-=-­<br />

2 4 2<br />

The only solution is !E...<br />

2<br />

x x<br />

sec-=cos­<br />

2 2'<br />

1 x<br />

--=cos-<br />

COS! 2<br />

2<br />

cos '::' =1<br />

2<br />

cos'::' =±1<br />

2<br />

~=O<br />

2<br />

x=O<br />

244


<strong>Trigonometry</strong> <strong>Solutions</strong> <strong>Manual</strong><br />

6.3 Trigonometric Equations II 245<br />

21.<br />

22.<br />

23.<br />

24.<br />

25.<br />

26.<br />

cos2x + cos x = 0, 0 ~ x < 21r<br />

2cos 2 x-l+cosx=O<br />

2cos 2 x+cosx -1 = 0<br />

(2cosx -1)(cosx+ 1)= 0<br />

2cosx-l=0 or cosx+l=O<br />

1<br />

cosx = - cosx =-1<br />

2<br />

1r 51r<br />

x=- - X=1r<br />

3' 3<br />

1r 51r<br />

x="3' x, 3<br />

. 1<br />

smxcosx=­<br />

4<br />

. 1<br />

2 smxcosx=­<br />

2<br />

sin2x=..!., 0~2x


<strong>Trigonometry</strong> <strong>Solutions</strong> <strong>Manual</strong><br />

246 Chapter 6 Inverse Trigonometric Functions and Trigonometric Equations<br />

32. sin26 = 2cos 2 6<br />

37. 0°::;26


<strong>Trigonometry</strong> <strong>Solutions</strong> <strong>Manual</strong><br />

6.3 Trigonometric Equations II 247<br />

vx-2<br />

-3<br />

41.<br />

42.<br />

43.<br />

In both cases, the value is approximately .7621.<br />

(a)<br />

For x = r,<br />

P( r) = .003 sin 2201rr +<br />

.003 sin 660lTr +<br />

3<br />

.003 sin 1100111 +<br />

5<br />

.003 sin 1540111<br />

'-­__--.7~<br />

.005<br />

--J<br />

44. (a) 3 beats r sec<br />

Forx = r,<br />

P(r) = .005 sin 4401rr<br />

.005sin 446'lTr<br />

'-------.<br />

+<br />

-.01<br />

(b) 4 beats<br />

r sec<br />

(b)<br />

The graph is periodic, and the wave has<br />

"jagged square" tops and bottoms.<br />

+<br />

(c)<br />

The eardrum is moving outward when P < O.<br />

This occurs for the time intervals<br />

.0045 < t < .0091;<br />

.0136 < t < .0182;<br />

.0227 < t < .0273.<br />

.15<br />

- .01<br />

(c) The number of beats is equal to the absolute<br />

value of the difference in the frequencies of<br />

the two tones .<br />

247


<strong>Trigonometry</strong> <strong>Solutions</strong> <strong>Manual</strong><br />

248 Chapter 6 Inverse Trigonometric Functions and Trigonometric Equations<br />

45. (a) For x = I ,<br />

46.<br />

P(I) = -tsin [211'(220)1] +<br />

-lsin(211'(330)1] +<br />

3<br />

-lsin (211'(440)1]<br />

4<br />

(b) h assumes its least value when<br />

sm-­ . 2rr x tak. es on Its . 1 east vue, al w hi IC h IS . - 1.<br />

365<br />

. 2rr x 1<br />

sm--=­<br />

365<br />

2rr x 3rr +2 . .<br />

365 = T - rr n, n IS an integer .<br />

x=e; ±2rrnX~~)<br />

= 3(365) ± 365n<br />

4<br />

= 273.75 ± 365<br />

-1 x = 273.75 means about 273.8 days after<br />

March 21, on December 19.<br />

(b) .0007576, .009847, .01894, .02803<br />

(c) Let h = 10.<br />

(c) 110 Hz 35 7. znx<br />

IO=-+-sm-­<br />

3 3 365<br />

(d) For x = I,<br />

P(I) = sin(211'(110)1] +<br />

30 = 35 + 7 sin 2rr x<br />

-tsin(21T(220)1] +<br />

365<br />

-tsin (21T(330)I] +<br />

- 5 = 7 sm-­ · zxx<br />

365<br />

1- sin[211'(440)1]<br />

4<br />

5 . 2rr x<br />

--=sm-­<br />

7 365<br />

- .71428571 = sin(2rr x)<br />

365<br />

In quadrant III and IV, sine is negative.<br />

In quadrant III,<br />

2rrx .<br />

-­= rr + .79560295<br />

365<br />

=3.9371956<br />

35 7. 2rrx<br />

x = 365 (3.9371956)<br />

h=-+-sm-­<br />

2rr .<br />

3 3 365 =228.7.<br />

x = 228.7 means 228.7 days after March 21,<br />

(a) Find x such that h = 14.<br />

on November 4.<br />

14= 35 +2 sin 2rrx In quadrant IV,<br />

3 3 365<br />

21tt = 2rr - .79560295<br />

14- 35 =2 sin 2rrx 365<br />

3 3 365 = 5.4875823<br />

7 7. 2rr x<br />

-=-sm-­<br />

x = 365 (5.4875823)<br />

3 3 365<br />

2rr<br />

. 2rr x 1<br />

= 318.8<br />

SIO--=<br />

365<br />

x = 318.8 means about 318.8 days after<br />

2rr x rr + 2 . .<br />

March 21, on February 2.<br />

-­=­ _ rr n, n IS an integer.<br />

365 2<br />

x= (~ ±2rrnX~~)<br />

= 365 ±365n<br />

4<br />

= 91.25 ±365n<br />

x =91.25 means about 91.3 days after<br />

March 21, on June 20.<br />

47.<br />

i = l max sin21ift<br />

Let i = 40, lmax =100, f =60.<br />

40 =l00sin2rr(6O)(t)<br />

40 =l00sinl20rrt<br />

.4 =sin 120rrt<br />

248


<strong>Trigonometry</strong> <strong>Solutions</strong> <strong>Manual</strong><br />

6.4 Equatiom Involving Inverse Trigonometric Functions 249<br />

From a calculator,<br />

S2. The error in the solution occurs in going from<br />

1201rt =.4115168 tan 20 2<br />

--=- to tanO=!.<br />

.4115168<br />

t=--­2 2<br />

1201r tan 20<br />

--¢ tan 0 because tan 20 ¢ 2 tan O.<br />

t = .0010916 2<br />

t:: .00 1sec<br />

Sedion6.4<br />

48, i =Imax sin 21!ft<br />

Let i = 50, lmax =100, f = 120.<br />

Exercises<br />

50 = 100 sin 2401rt<br />

1. C; arcsin 0 = 0<br />

sin 2401rt = 1.<br />

2 1r<br />

2. A; arctan 1 = ­<br />

2401rt =!E.<br />

4<br />

6<br />

1<br />

t=-­<br />

1440<br />

3. C', arcco{-..fi) = 31r<br />

2 4<br />

t =.0007 sec<br />

49. i = lmaxsin 21!ft<br />

Let i = lmax' f = 60.<br />

lmax = lmaxsin 21r(6O)t<br />

1= sin 1201rt<br />

S. y=5cosx<br />

I=cosx<br />

1201rt =!E. 5<br />

2<br />

4. C· arcsin(-1.) = -1r<br />

, 2 6<br />

x=arccos I<br />

1201 =1. 5<br />

2<br />

1<br />

t=-<br />

6. 4y=sinx<br />

240 x=arcsin 4y<br />

t =.004 sec<br />

7. 2y =cot3x<br />

SO. i =lmaxsin 21!ft<br />

3x = arccot 2y<br />

Let i =1.lmax' f= 60.<br />

1<br />

2<br />

x = -arccot2y<br />

3<br />

~ lmax = lmax sin 21r(6O)t<br />

1<br />

8. 6y=-secx<br />

1. = sin 1201rt<br />

2<br />

2 12y =sec x<br />

1201rt =!E.<br />

x = arcsec 12y<br />

6<br />

1 9. y=3tan2x<br />

t=­<br />

720<br />

I=tan2x<br />

t = .0014 sec 3<br />

2x=arctan I<br />

. 1 1 .<br />

SIIf • sm-x=--, we must wnte<br />

3<br />

22 . 1 y<br />

x =-arCtan­<br />

1 1<br />

-x =210 2 3<br />

0+n·36O" or -x = 330 0+n '360 0 ,<br />

2 2<br />

where n is any integer. Then multiplying by 2, we<br />

have x = 420 0 + n . 720 0 or x = 660 0 + n ·7 20° .<br />

Since n E {... , - 3, - 2. -1, 0, 1, 2, 3, ...},<br />

there is no value of n which will give values ofx<br />

in the interval [0 0,3600 ) .<br />

249


<strong>Trigonometry</strong> <strong>Solutions</strong> <strong>Manual</strong><br />

250 Chapter 6 Inverse Trigonometric Functions and Trigonometric Equations<br />

10.<br />

11.<br />

12.<br />

13.<br />

14.<br />

15.<br />

x<br />

3 . y = sm<br />

2<br />

1:'.=sin~<br />

3 2<br />

z, arcsinl.<br />

2 3<br />

x = 2 arcsin 1:'.<br />

3<br />

x<br />

y =6cos­<br />

4<br />

z= cos~<br />

6 4<br />

z, arccosl.<br />

4 6<br />

x = 4 arccosl.<br />

6<br />

. X<br />

y=-sm­<br />

3<br />

. x<br />

sm-=-y<br />

3<br />

~ =arcsin(-y)<br />

3<br />

x = 3arcsin(-y)<br />

y= -2cos5x<br />

-1:'.=cos5x<br />

2<br />

5x = arccos(- ~)<br />

x = ~arccos(-~)<br />

y =3cot5x<br />

cot5x =l.<br />

3<br />

5x = arccotl.<br />

3<br />

1 y<br />

x =-arccot­<br />

5 3<br />

y = cos(x + 3)<br />

x + 3 = arccosy<br />

x = -3+ arccosy<br />

17. y=sinx-2<br />

y+2 = sinx<br />

x = arcsin(y + 2)<br />

18. y = cotx+ 1<br />

cotx = y-l<br />

x = arccot(y -1)<br />

19. y=2sinx -4<br />

y+4 = 2sin x<br />

y+4 .<br />

--=smx<br />

2<br />

20.<br />

x = arcsin( y ; 4 )<br />

y=4+3cosx<br />

y-4 =3cosx<br />

y-4<br />

--=cosx<br />

3<br />

x=arcco{y~4)<br />

4 -I Y<br />

23. -cos -=1r<br />

3 4<br />

-IY 31r<br />

cos -=­<br />

4 4<br />

Y 31r<br />

-=cos­<br />

4 4<br />

.J2<br />

y<br />

-=-­<br />

4 2<br />

y=-2.J2<br />

24. 41r+ 4tan- 1 y = 1r<br />

4 tan-I y = -31r<br />

-I 31r<br />

tan y=-­<br />

4<br />

The range of tan-I y is _!E. < y < 1r .<br />

2 2<br />

Since - 31r is not in this interval, the equation<br />

4<br />

has no solution.<br />

25. 2 arcco{ y ~ 1r) = 21r<br />

16.<br />

y = tan(2x -1)<br />

2x - 1= arctan y<br />

2x = 1+ arctany<br />

1<br />

x = -(1 + arctany)<br />

2<br />

arccos( y ~ 1r) = 1r<br />

y-1r<br />

--=COS1r<br />

3<br />

y-1r<br />

--=-1<br />

3<br />

y-1r =-3<br />

y=1r-3<br />

250


<strong>Trigonometry</strong> <strong>Solutions</strong> <strong>Manual</strong><br />

6.4 Equations Involving Inverse Trigonometric Functions 251<br />

­<br />

26. arcco{y- ~) =:<br />

_I . -1 3<br />

29. cos x =sm<br />

5<br />

1r 1r<br />

y--=cos-<br />

. -1 3 . 3<br />

Let sm - = u , so SinU = -.<br />

3 6 5 5<br />

.J3<br />

u is in quadrant I.<br />

1r<br />

y=-+-'<br />

y<br />

2 3<br />

3.J3 +21r<br />

y=<br />

6<br />

3<br />

. 3<br />

27. arcsin x = arctan ­ ---F-......---...... x<br />

4 4<br />

Let arctan-<br />

3<br />

= u, so tanu = -,<br />

3 .. dr I<br />

U IS In qua ant.<br />

4 4<br />

Sketch a triangle and label it. The hypotenuse is 4<br />

cosu =­<br />

~32+42 5<br />

=5.<br />

The equation becomes<br />

y<br />

3<br />

cos-I x = U. or x = cosu.<br />

4<br />

x=­ 5<br />

_I _1 4<br />

30. cot x = tan ­<br />

--I'-.....__.......--x<br />

3<br />

4<br />

_14 4<br />

Let tan - = u, so tan u = - .<br />

3 3<br />

. 3 3 y<br />

smu=-=r<br />

5<br />

This equation becomes arcsin x = u, or x = sin u.<br />

3<br />

x=­<br />

5<br />

4<br />

---I'-"-..............---x<br />

5 3<br />

28. arctan x = arccos­<br />

13<br />

5 5<br />

Let arccos - = u, so cos u =-. 3<br />

13 13<br />

cotu =­<br />

4<br />

Sketch a triangle and label it. The side opposite<br />

angle u is ~132 - 52 = 12. . -I -11 1r<br />

31. sm x-tan =-­<br />

y 4<br />

-----f~::_I---- x<br />

. -I -11 1r<br />

sm x=tan -­<br />

4<br />

. _I 1r 1r<br />

SIn<br />

12 4 4<br />

sin-I x = 0<br />

12<br />

tanu=­<br />

5<br />

The equation becomes arctan x = u, or x =tan u.<br />

12<br />

x=­ 5<br />

x=--­<br />

sinO = x<br />

x=O<br />

251


<strong>Trigonometry</strong> <strong>Solutions</strong> <strong>Manual</strong><br />

252 Chapter 6 Inverse Trigonometric Functions and Trigonometric Equations<br />

, -1 -1 h3 21r<br />

32. Sin x+tan v-' =­<br />

3<br />

, -1 1r 21r<br />

Sin x+-=­<br />

3 3<br />

, -1 1r<br />

Sin x=­ 3<br />

. 1r<br />

x<br />

X=SIn­<br />

3<br />

sinu =~I_x2<br />

.J3<br />

. 1r 1r.<br />

x=­2x = SIn--COSU-COS-SInU<br />

2<br />

6 6<br />

,-./3<br />

33. arccos x + 2 arcsin - = 1r<br />

2<br />

. -./3<br />

arccos x = 1r- 2 arCSIn­<br />

2<br />

arccosx =1r- z( ~)<br />

21r<br />

arccosx = 1r-­<br />

3<br />

9x 2 =3-3x 2<br />

1r<br />

12x 2 =3<br />

arccosx=­<br />

3 2 3 1<br />

x =-=­<br />

1r<br />

12 4<br />

x=cos­ 3<br />

1<br />

x=­<br />

2<br />

, -./3 1r<br />

34. arccos x + 2 arCSin - = ­<br />

2 3<br />

arccosx+z( ~) =~<br />

1r<br />

arccosx=-­<br />

3<br />

- ~ is not in the range of arccos x, Therefore, the<br />

equation has no solution.<br />

1 -./3 ~<br />

2x=-x--(vl-x-)<br />

2 2<br />

4x = x--./3~I-x2<br />

3x =-J3~I- x 2<br />

(3x)2 =(-J3~I- x 2 )2<br />

2)<br />

9x 2 =3(1- x<br />

1<br />

x=±­<br />

2<br />

Check these proposed solutions since they were<br />

found by squaring an equation.<br />

1<br />

x=-:<br />

2<br />

il)<br />

.( I) 1r 1r 1r<br />

arcsin 2'2 +arcco'\2 ='2+3'*6'<br />

1<br />

x=--:<br />

2<br />

arcsi .{ 2 (-'2 I)) +arccos (-2-I ) =-'2+'3=6'<br />

tt 21r 1r<br />

The only solution is _.!..<br />

2<br />

35• arcsin ' 2 x + arccos x = ­1r<br />

6<br />

arcsin -2 x = -1r<br />

- arccos x<br />

6<br />

2x = Sin(: - arccos x )<br />

Use the identity<br />

sin(A - B) sin Acos B-cosAsin B.<br />

2x = sin 1rcos(arccosx)- cos 1rsin(arccosx)<br />

6 6<br />

Let U =arccos x.<br />

x<br />

cosu=x=­<br />

1<br />

36• arcsm<br />

. 2x<br />

+ arcsm<br />

.<br />

x =­<br />

1r<br />

2<br />

arcsin<br />

- 2<br />

x = -<br />

1r<br />

- arcsin<br />

.<br />

x<br />

2<br />

2x = sin( ~ - arcsin x)<br />

Use the identity<br />

sin (A - B) = sin A cos B - cosA sin B.<br />

. 1r ( in r) 1r . ( - )<br />

2x = Sin- cos arcsm x - cos - Sin arcsin x<br />

2 2<br />

= 1cos(arcsin x) - 0<br />

= cos(arcsin x)<br />

252


<strong>Trigonometry</strong> <strong>Solutions</strong> <strong>Manual</strong><br />

6.4 Equations Involving Inverse Trigonometric Functions 253<br />

. . x<br />

Let arcsm x = U, so Sin U = X = -.<br />

1<br />

y<br />

Let U = tan-I x so tan U = x.<br />

x<br />

COSU =~1_x2<br />

2<br />

Substitute ~1- x<br />

2x=~I-x2<br />

4x 2 = l-x 2<br />

5x 2 = 1<br />

for cos(arcsin x).<br />

2 1<br />

x =­<br />

5<br />

..[5<br />

x=±­<br />

5<br />

Check these proposed solutions since they were<br />

found by squaring an equation.<br />

..[5<br />

x=-:<br />

5<br />

arcsin . (2 ..[5) . (.J5) x<br />

..[5<br />

x=--:<br />

5<br />

's +arCSIn "'"5 =2'<br />

arcsin(2· ~) + arCSi{- '7) = - ~<br />

· . .J5<br />

TheonIy so Iution IS -.<br />

5<br />

_I -I x<br />

37. cos x+tan x=­<br />

2<br />

-I 1r -I<br />

cos x == - - tan x<br />

2<br />

x=co{~ -tan-I x)<br />

Use the identity<br />

cos(A - B) = cos A cos B + sin A sin B.<br />

x =cos~cos (tan-I x)+sin 1r sin (tan-I x)<br />

2 2<br />

COSU=<br />

.<br />

SInU=<br />

r-1<br />

vl+x 2<br />

x<br />

r-1<br />

vl+x 2 1r . 1r •<br />

x =cos-,cosU+SIn- 'SInU<br />

2 2<br />

X={b)+lQ<br />

x<br />

X=-.===<br />

~1+x2<br />

x­x ==0<br />

~1+x2<br />

JI- 1 ]=0<br />

l ~1+x2<br />

1<br />

x=Oor 1­<br />

~1+x2<br />

1<br />

~1+x2<br />

~1+x2<br />

=0<br />

-1<br />

-<br />

=I<br />

l+x 2 =1<br />

38. sin-I x+tan- I x=O<br />

x 2 =0<br />

x=O<br />

sin-I x==-tan- I x<br />

sin(sin-I x) = sin(- tan-I x)<br />

x=-sin(tan- I x)<br />

x<br />

x=---;===<br />

~x2 +1<br />

x2=~<br />

x 2 +1<br />

2(x 2<br />

x + 1) == x<br />

2<br />

x 2(x2 +1)-x 2 =0<br />

x 2[(x2 +1)-1]=0<br />

2(x2<br />

x ) == 0<br />

x 4 =0<br />

x==O<br />

253


<strong>Trigonometry</strong> <strong>Solutions</strong> <strong>Manual</strong><br />

254 Chapter 6 Inverse Trigonometric Functions and Trigonometric Equations<br />

39. 41. Y I = (arctan x)3 - x + 2<br />

-3<br />

x =4.4622037<br />

40.<br />

42.<br />

x =2.2824135<br />

43. A =~(AI COSiPI + AZcostPz)2 + (AI siniPI + AZ sin(j)z)2<br />

an d '"<br />

(_A.!..Is_i_n..:,.tPI!...+_A.:. 2_si_n...:.tPz~)<br />

'I' =arctan<br />

Al cos 41 + Az cos tPz<br />

Make sure your calculator is in radian mode.<br />

(a) Let Al =.0012, iPI =.052, Az =.004, and tPz =.61.<br />

A =~(.OO12cos.052+ .OO4cos.61)2 +(.0012sin.052+ .004-sin .61)2<br />

=.00506<br />

'" ( .0012sin .052 + .004sin .61)<br />

'I' =arctan ---------­<br />

.0012cos.052+ .OO4cos.61<br />

=.484<br />

P =A sin(21¢ + 41), let! =220.<br />

P =.00506sin(440m + .484)<br />

(b)<br />

Forx=t,<br />

P( t) = .00506 sine4401rt + .484)<br />

P1(t) + P 2 (t ) = .OOI2sin(440m + .052) +<br />

.004 sin(440m + .61)<br />

~------.<br />

The two graphs are the same.<br />

254


<strong>Trigonometry</strong> <strong>Solutions</strong> <strong>Manual</strong><br />

6.4 Equations Involving Inverse Trigonometric Functions 255<br />

44. A = ~(AI cost/>} + A2COS¢2)2 + (AI sin if>! + A2 sin ¢2)2<br />

A.<br />

d<br />

(AI sin ¢I + A2 sin ¢2 )<br />

Al cos¢+ A2 cos¢2<br />

an 'I' = arctan ~-.........--=------:..=­<br />

Make sure your calculator is in radian mode.<br />

1r<br />

(a) Let AI = .0025, if>! = 7' A2 = .001, ¢2 = 6' and!= 300.<br />

A = ~(AI cos¢! + A2 COS¢2)2 +(AI sin¢1 + A2 sin¢2)2<br />

= [.0025CO{ ~)+.OOlCO{:)r+ [.0025sin( ~)+.OOlsin(:)r<br />

"'.0035<br />

.0025sin(~)+.001Sin(t)]<br />

¢ = arctan ---+'-i-----7'--';­<br />

[ .0025 cos(.,) + .001co~f)<br />

'" .470<br />

P = Asin(21¢ + ¢)<br />

P = .0035 sin(6001rt + .47).<br />

1r<br />

(b)<br />

Forx = t,<br />

per) = .0035 sin(6OOm + .47)<br />

PI(r) + P2(t) = .0025 Sin(6OO1I1 + f) +<br />

.001 sin(600111 + f)<br />

45. (a)<br />

(b)<br />

x<br />

(e) (x+ y)tana = x tan {3<br />

tana=z<br />

x tan {3<br />

tana=-­<br />

x+Y<br />

tan{3=-­<br />

x+y<br />

z<br />

a = arctan(Xlan{3)<br />

x . a x+y<br />

x+y<br />

tana=­ tanJJ=-­<br />

z<br />

z<br />

z tan a =x ztan{3=x+y<br />

(d) xtan{3=(x+y)tana<br />

z =­ x ­z=-­ x+y<br />

tan{3= (x+ y)tana<br />

tan a tan{3 x<br />

x x+y<br />

--=-­ {3 = arctan[(x + y~tana]<br />

tana<br />

tan{3<br />

255


<strong>Trigonometry</strong> <strong>Solutions</strong> <strong>Manual</strong><br />

256 Chapter 6 Inverse Trigonometric Functions and Trigonometric Equations<br />

46. (a) u = arcsinx Divide by ·Ji<br />

. 1t 1t<br />

x =smu - - :s; u :s; -<br />

x 2 -3~+4=O<br />

, 2 2 ,-----­<br />

3../3 ±~(_3../3)2 -4(1)(4)<br />

(b) Y x=<br />

2<br />

3../3±.JIT<br />

x=<br />

2<br />

x = 4.26 ft or .94 ft<br />

(ii) Let ()= rr .<br />

8<br />

rr -I 4 -I 1<br />

-=tan --tan ­<br />

x x~l-x2 8 x X<br />

(c) tanu= = 2<br />

"Jl-x 2 I-x tan-=tan tan --tan ­<br />

8 x x<br />

From part (a),<br />

[ I-x 8 x 2+4<br />

rr (-I 4 -I 1)<br />

(d) x~ ] rr 3x<br />

u =arctan 2 tan-=--.<br />

.414=~<br />

47. (a) e = Emax sin21ift x 2+4<br />

_e_ = sin21ift<br />

E max<br />

.414x 2 + 1.656= 3x<br />

.414x<br />

2 -3x+1.656=O<br />

21fft = arcsin _e_<br />

E max<br />

3± "J9 - 4(.414)(1.656)<br />

x=<br />

1 . e<br />

2(.414)<br />

t=-arcsm-­<br />

21r/ E max 3±2.502<br />

x'"<br />

.828<br />

(b) Let E max = 12, e = 5, and!= 100.<br />

x '" 6.64 ft or .60 ft<br />

1 . 5<br />

t= arcsm­<br />

2rr(I00) 12<br />

n -1 4 _I 1<br />

() c 17 = tan --tan ­<br />

x x<br />

t = _I-arcsin .4166667 = .00068 sec<br />

200rr (i) Let x =4.<br />

n -1 4 -I<br />

17=tan --tan ­<br />

1<br />

6<br />

-II -I 1<br />

7t _I 4 -I 1 6 = tan -tan­<br />

- = tan - - tan - 4<br />

6 x x<br />

1t<br />

6 =--.245<br />

tan-<br />

7t<br />

= tan (-14 tan --tan ~ - 1)<br />

4<br />

6 x x 6= .54<br />

48~ (b)(i) Let 6 =!:.. 4 4<br />

../3 ~_.l<br />

x<br />

_= x<br />

(ii) Let x =3.<br />

3 1+A.·.l<br />

x x -1 4 -I 1<br />

6<br />

../3<br />

=tan --tan ­<br />

_=_x_<br />

1. 3 3<br />

3 1+4­<br />

6 = .927- .322<br />

x<br />

6=.60<br />

../3 ~<br />

3"= ijA<br />

x<br />

../3 3x<br />

""3= x 2+4<br />

../3x 2 + 4../3 = 9x<br />

../3x 2 -9x+4../3 =0<br />

256


<strong>Trigonometry</strong> <strong>Solutions</strong> <strong>Manual</strong><br />

Chapter 6 Review Exercises 257<br />

1 . 4m<br />

49• y = - sm ­<br />

3 3<br />

50.<br />

() a<br />

. 4m<br />

3y=sm­<br />

3<br />

4m . 3<br />

3=arcsm y<br />

4m = 3arcsin 3y<br />

3 .<br />

t=-arcsm 3 y<br />

41r<br />

(b) If y = .3 radian,<br />

t=-arcsm<br />

3 .<br />

. 9<br />

41r<br />

t= .27sec.<br />

6. y = arcco{-~)<br />

1<br />

cosy =-2"' 0:5: y:5: 1r<br />

2n<br />

y=3"<br />

7. y =tan- 1 (-J3)<br />

M x 1r<br />

tany =-v3 --


<strong>Trigonometry</strong> <strong>Solutions</strong> <strong>Manual</strong><br />

258 Chapter 6 Inverse Trigonometric Functions and T rigonometric Equations<br />

1<br />

14. 6 =arccos­<br />

2<br />

1<br />

cos 6 = -, 0° ~ 6 ~ 180°<br />

2<br />

6=60°<br />

15. 6 =arCSin(- ~)<br />

sin 6 = - .J3 , - 90° ~ 6 ~ 90°<br />

2<br />

8=~0<br />

16. 6 =tan- I 0<br />

tan 6 =0, _90° < 6 < 90°<br />

8=0°<br />

For Exercises 17-22, be sure that your calculator is in<br />

degree mode. Keystroke sequences may vary based on<br />

the type and/or model of calculator being used.<br />

17. 6 = arctan 1.7804675<br />

Enter 1.7804675 ~ ~<br />

or~ ~ 1.7804675~<br />

Display: 60. 67924514<br />

6 = 60.67924514°<br />

18. 6 = sin- I (-.66045320)<br />

Enter: -.66045320 8 ~<br />

or~ ~ c=J .66045320<br />

~<br />

Display: -41.33444556<br />

6 == -41.33444556°<br />

.-'<br />

19. 6 = cos- I .80396577<br />

Enter: .80396577 ~ §)<br />

or ~ §) .80396577 ~<br />

Display: 36.4895081<br />

6 = 36.4895081 °<br />

20. 6 = coe l 4.5046388<br />

Enter: 4.5046388 §)~ ~<br />

or 8 §) 4.5045388 ~ ~<br />

Display: 12.51631252<br />

6 = 12.51631252°<br />

21. 6 = arc see 3.4723155<br />

Enter: 3.4723155 §) ~ §)<br />

or ~ §J 3.4723155 ~ ~<br />

Display: 73.26220613<br />

6 = 73.26220613°<br />

22. 6 = csc- 1 7.4890096<br />

Enter: 7.4890096 §)8 ~<br />

or ~ ~ 7.4890096 ~ ~<br />

Display: 7.673567973<br />

6 = 7.673567973°<br />

25. The domain of the arccot function is (-eo, 00)since<br />

the range of the cotangent function is (-eo, 00).<br />

26. The range of the function y =arc sec x as defined<br />

in this text is [0, ~) u(~ ,n] or [0, n], y'# ~ .<br />

. ( . _I 1)<br />

27. sm sm 2<br />

Le . -I 1<br />

t (O=sm 2'<br />

Then si 1 n n<br />

en sm(O=-, --~(O~-.<br />

2 2 2<br />

. ( .<br />

sm sm 21). -I =sm (0 =21<br />

29. cos (arccos (-1»<br />

Let (0 = arccos (-1)<br />

Then, cos (0 = 1, 0 ~ x ~ n ,<br />

costarccost-J) = CO!! (0 =-1<br />

30. sin[arcsin(- ~J) =- ~<br />

31. arcco{cos 3:)=x<br />

3n<br />

cos x =cOS-, 0 ~ x ~ n,<br />

4<br />

3n<br />

so x=-.<br />

( 4 3n) 3n<br />

arccos cos4 =4<br />

32. arcsec(sec n)<br />

seen =-1<br />

arcsec(sec n) =arcsec(-1) =n<br />

258


<strong>Trigonometry</strong> <strong>Solutions</strong> <strong>Manual</strong><br />

1C<br />

sox=-.<br />

4<br />

tan-I(tan =) ==<br />

36. cos (arctan 3)<br />

Chapter 6 Review Exercises 259<br />

Let ()= arctan 3. Then tan () = 3 = ~; 0 is in<br />

1C 1C 1C<br />

I<br />

tan x =tan- --< x


<strong>Trigonometry</strong> <strong>Solutions</strong> <strong>Manual</strong><br />

260 Chapter 6 Inverse Trigonometric Functions arid Trigonometric Equations<br />

Let 0 =sin-'(-~}<br />

Then sin 0 = _.!.; 0 is in quadrant IV.<br />

3<br />

The side adjacent to 0 =~32 - (_1)2<br />

=.J8 =2.fi<br />

y<br />

----loo:=--"*""--r- x<br />

-,<br />

2.fi<br />

cosO =-­ 3<br />

sec( 2sin-'(-~)) =sec 20<br />

. 3 TC TC<br />

Let (0\ =arcsm -, - - ~ co, ~ -.<br />

5 2 2<br />

sinco, =~. The side adjacent to COt is<br />

39. tan(arcsin ~ + arccos %)<br />

~52 _3 2 =4.<br />

y<br />

1<br />

=-­<br />

cos20<br />

y<br />

1<br />

= 2<br />

2ef) -1<br />

---l'-...:...__L....~ x<br />

1<br />

=-'6­<br />

9-1<br />

1<br />

='7<br />

2../6<br />

"9<br />

9<br />

=­ 7<br />

tan (0, ='4 3 5<br />

Let (02 =arccos -, 0 s co2 s TC.<br />

7<br />

5<br />

cos(02 =-.<br />

7<br />

The side opposite (02 is ~72 - 52 =.,fi4 =2../6 .<br />

tan co2 =-­ 5<br />

tan (0, + tanco2<br />

Use tan(co, + (02) = .<br />

1- tan co, tan co2<br />

3 5) i+ 2 ./6<br />

tan(arcsin - + arccos - = -71£)<br />

5 7 1-(iJ\.¥<br />

15+8./6<br />

_ -----zo<br />

- 20-6'J6<br />

-W­<br />

15+ 8../6 20 + 6.J6<br />

= 20- 6.J6 ' 20+ 6../6<br />

588+250../6<br />

=<br />

184<br />

294+125../6<br />

=<br />

92<br />

260


<strong>Trigonometry</strong> <strong>Solutions</strong> <strong>Manual</strong><br />

Chapter 6 Review Exercises 261<br />

40. sinuan"! u)<br />

-I tr tr<br />

Let s =tan u, - - < S 0, then s is in quadrant I.<br />

Ifu < 0, then s is in quadrant IV.<br />

hypotenuse =~u 2 + 1<br />

y<br />

~U2 +1<br />

Then sees = .<br />

u<br />

If u > 0, s is in quadrant I.<br />

If u < 0, s is in quadrant II.·<br />

The side opposite s, which is positive in both<br />

quadrants, is ~(u2 + I) - u 2 =I.<br />

y<br />

U, u>o<br />

--++~--_ x<br />

u, u


<strong>Trigonometry</strong> <strong>Solutions</strong> <strong>Manual</strong><br />

262 Chapter 6 Inverse Trigonometric Functions and Trigonometric Equations<br />

­<br />

45. y = arccot x 51. tan 2 2x -1 = 0, 0 ~2x < 41C<br />

Domain: (-00,00)<br />

Range: (0, 1C)<br />

tan 2 2x = 1<br />

y tan2x = ±1<br />

1C 51C<br />

Iftan 2x = 1, 2x =-,<br />

91C 131C<br />

y =arccot x<br />

4 4' 4' 4<br />

1C 51C 91C 131C<br />

x=- ­<br />

8 ' 8' 8' 8<br />

31C 71C lI1C 151C<br />

---+---:-1---+--_ x If tan 2x = -1, 2x =-, -- ,<br />

-1 0 4 4' 4 4<br />

31C 71C lI1C 151C<br />

x=- -- ,<br />

8 ' 8 ' 8 8<br />

46. sin 2 x = 1, 0 ~ x < 21C 1C 31C 51C 71C 91C 1I1C 131C 151C<br />

x=-, - , - , ­<br />

sin x =±1 8 8' 8' 8' 8' 8 8 8<br />

1C 31C<br />

x=­ x x<br />

2' 2 52. sec-= cos­<br />

2 2'<br />

47. 2tanx-l = 0 1 x<br />

--=cos­<br />

2tanx=1 cost 2<br />

1<br />

tanx=­ cos 2 ::'= 1<br />

2 2<br />

x = .463647609or x = 3.605240263<br />

cos::' = ±1<br />

2<br />

48. 3sin 2 x -5sinx +2 = 0 x<br />

(3sinx -2)(sinx -1) = 0 If cos- = 1,<br />

2<br />

. 2 . 1<br />

::'=0<br />

3 2<br />

1C<br />

x = .7297276562, 2.411864997 or x =­<br />

x=O.<br />

2<br />

Sin X =- or Sin x =<br />

If cos::' =-1<br />

2 '<br />

49. tan x = cot x, 0 ~ x < 21C<br />

::. = 1C which is not in the domain.<br />

Use the identity cot x = _1_.<br />

2 '<br />

tan x<br />

The only solution is O.<br />

1<br />

tanx=--, tan e O<br />

tan x<br />

53. cos2x +cosx =0, 0 ~ x < 21C<br />

tan 2 x = 1<br />

2cos 2 x-l+cosx=O<br />

tan x =±1<br />

2cos 2 x+cosx-l=O<br />

1C 51C<br />

If tanx=l, X=-,<br />

(2cosx -1)(cosx +1)=0<br />

4 4 2cosx-l =0 or cosx+l =0<br />

31C 71C 1<br />

If tan x =-1, x=- - . cosx =- cosx =-1<br />

. 4' 4 2<br />

1C 31C 51C 71C 1C 51C<br />

x=- - x=1C<br />

x="4' 4' 4' 4 3' 3<br />

50. sec 4 2x =4, 0 ~ 2x < 41C<br />

22x=2<br />

sec<br />

sec2x=±.,fi<br />

2x =1C 31C 51C 71C 91C lI1C 131C 151C<br />

4' 4' 4' 4' 4' 4' 4'4<br />

1C 31C 51C 71C 91C lI1C 131C 151C<br />

x=s' 8' 8' 8' 8' 8' -8-"-8­<br />

1C 51C<br />

x=- 3'<br />

1C ,<br />

­<br />

3<br />

262


<strong>Trigonometry</strong> <strong>Solutions</strong> <strong>Manual</strong><br />

Chapter 6 Review Exercises 263<br />

-/.. '<br />

54. 4sinxcosx =.J3<br />

2 . .J3<br />

smxcosx =­ 2<br />

sin2x =.J3 0::; 2x < 41r<br />

2 '<br />

A value for 6 will be a solution if<br />

sin 26 - cos26 = 1.<br />

6=45°,26=90°<br />

sin 90° - cos90° = 1-0 = I<br />

6 =90°, 26 = 180°<br />

sin 180°- cos180° = 0 - (-I) = I<br />

6 = 135°, 26 = 270°<br />

sin2700-cos270° = -1-0* I<br />

2x = 1r 21r 71r 81r<br />

3' 3' 3' 3<br />

1r 1r 71r 41r<br />

6 = 225°, 26 = 450°<br />

x=­ - - - 6' 3' 6' 3 sin4500-cos450° = 1-0 = I<br />

6 = 270°, 26 = 540°<br />

55. sin 26+3sin6+2=0,00::;6


<strong>Trigonometry</strong> <strong>Solutions</strong> <strong>Manual</strong><br />

264 Chapter 6 Inverse Trigonometric Functions and Trigonometric Equations<br />

62. cos8-cos28 = 2cos8<br />

cos8 - (2cos 2 8 -1) = 2cos8<br />

2cos 28+cos8-1 =0<br />

(2cos8 -1)(cos8 + 1)= 0<br />

2cos8 -1 = 0 or cos 8 + 1= 0<br />

cos8 =..!.. cos8 =-1<br />

2<br />

8 = 60°, 300° 8 = 180°<br />

8 = 60°, 180°, 300°<br />

63. 4y =2sinx<br />

2y=sinx<br />

x =arcsin2y<br />

. 2<br />

68. arccosx = arcsm-«<br />

7<br />

. 2<br />

Let u =arcsm-.<br />

7<br />

Then sinu = ~, u is in quadrant I.<br />

7<br />

The side adjacent to u is<br />

~72 _2 2 =.[45 = J$.<br />

y<br />

x<br />

64. y=3cos­ 2<br />

z.=cos~<br />

3 2<br />

z, arccosZ.<br />

2 3<br />

x = 2arccosZ.<br />

3<br />

65. 2y =tan(3x + 2)<br />

3x + 2 = arctan2y<br />

3x = arctan2y - 2<br />

1<br />

x =-(arctan2y-2)<br />

3<br />

x =(~arctan2Y)-~<br />

66. 5y = 4sinx-3<br />

5y+3 .<br />

--=SlnX<br />

4<br />

. 5y+3<br />

x=arcsln-­<br />

4<br />

4 x<br />

67. -arctan- = TC<br />

3 2<br />

X 3TC<br />

arctan-=­<br />

2 4<br />

But, by definition, the range of arctan is<br />

(- ~, ~) . So, this equation has no solution.<br />

3..[5<br />

cosu=-­<br />

7<br />

This equationbecomes arccos x = u, or x = cos u,<br />

J./5<br />

x=-­<br />

7<br />

lITC<br />

69. arccosx + arctan1= ­<br />

12<br />

lITC<br />

arccosx = - - arctan 1<br />

12<br />

11K TC<br />

arccos x = - -­<br />

. 12 4<br />

lITC 3TC 8TC<br />

arccosx=---=­<br />

12 12 12<br />

2TC<br />

arccosx=­<br />

3<br />

2TC<br />

cos-=x<br />

3<br />

1<br />

x=-­ 2<br />

.( --<br />

.J2) 3TC<br />

70. arccotx = arcsin +­<br />

. 2 4<br />

TC 3TC<br />

arccotx = - - + ­<br />

4 4<br />

TC<br />

arccotx=­<br />

2<br />

TC<br />

x=cot-=O<br />

2<br />

264


<strong>Trigonometry</strong> <strong>Solutions</strong> <strong>Manual</strong><br />

Chapter 6 Review Exercises 265<br />

Tr<br />

71. d = 550+450cos-t<br />

50<br />

Tr<br />

450cos-t = d -550<br />

50<br />

Tr d -550<br />

cos-t=<br />

50 450<br />

-t= x arccos (d-550)<br />

50 450<br />

t=-arcco 50 {d-550)<br />

Tr 450<br />

CI sin 0<br />

74. -=-­ 1<br />

c2 sinO 2<br />

.752 = sin 01<br />

sinO 2<br />

If O 2 =90°,<br />

.752 = sin01<br />

sin90°<br />

sin0<br />

=-­ 1<br />

1<br />

= sin0 1<br />

0 1 = 48.8°.<br />

75. If 0 1 > 48.8°, then O 2 > 90° and the light beam is<br />

completely underwater.<br />

76. L = 6077- 31cos20<br />

(a) Solve the equation for O.<br />

L - 6077 = -31cos20<br />

L-6077<br />

cos 20 =--­<br />

-31<br />

20 = cos-I(L-6077)<br />

-31<br />

O=-cos 1 _1(L-6077)<br />

2 -31<br />

O=-cos<br />

1 _1(6074-6077)<br />

2 -31<br />

0:= 42.2°<br />

73. a. Let a be the angle to the left of O.<br />

b.<br />

5+10<br />

Then tan(a + 0) =-­<br />

x<br />

a+O = arctan(~)<br />

o= arctan(~)-a<br />

e= arctan( ~) - arctan(;)<br />

(b) Substitute L = 6108 in the equation from<br />

part (a).<br />

1 _1(6108-6077)<br />

O=-cos<br />

2 -31<br />

0=90°<br />

(c) Substitute L = 6080.2 in the equation from<br />

part (a).<br />

O=-cos<br />

1 _1(6080.2-6077)<br />

2 -31<br />

0:= 48.0°<br />

77. To see the graph, enter the function<br />

YI = «x > 0) - (x < 0» -(~ - tan-I ~x 2 -1).<br />

The maximum occurs at x = 8.6602567.<br />

-"2 7T<br />

Radian mode<br />

265


<strong>Trigonometry</strong> <strong>Solutions</strong> <strong>Manual</strong><br />

266 Chapter 6 Inverse Trigonometric Functions and Trigonometric Equations<br />

78. (a)<br />

(e)<br />

y=csc-1(2../3) -3­<br />

2../3 rr rr<br />

cscy=-- --:5y:5- y;tO<br />

3 ' 2 2'<br />

n<br />

y=3"<br />

(b) In both cases, sin- I (.4) =:: .41151685<br />

Chapter 6 Test Exercises<br />

1.<br />

y<br />

(I. f)<br />

+--+----'-tf---t----t- x<br />

(I) y =cot- I (../3)<br />

t: rr rr<br />

cot y ='" 3, - - :5Y :5-, y;t 0<br />

2 2<br />

rr<br />

y=­ 6<br />

. 2 . 2<br />

3. (a) Le t arcsm- = u, so smu =-.<br />

3 3<br />

Then, co{arcsin ~) =cos u =~<br />

y<br />

Domain: [-I, 1]<br />

Range: [-;, ;]<br />

2. (a)<br />

(b)<br />

y =arcco{-~)<br />

y=cos -I(-'2<br />

I)<br />

1<br />

cosy=--,O:5y:5rr<br />

2<br />

2rr<br />

y=­ 3<br />

y=sin-t-v;)<br />

. ../3 rr rr<br />

smY=-T' -"2:5 y:5"2<br />

x<br />

y=-­ 3<br />

(b) Let arccosej) = U,so cosu =-j.<br />

Then Si{2 cos-I -j) =sin 2u<br />

y<br />

=2sinucosu<br />

_ ' -(2~XI) -2- ­<br />

3 3<br />

4~<br />

=-­ 9<br />

"<br />

(c) y = tan-I (0)<br />

rr rr<br />

tany=O, --


<strong>Trigonometry</strong> <strong>Solutions</strong> <strong>Manual</strong><br />

4• Le t x=arcsmu,<br />

. th<br />

en smx=­<br />

. u<br />

1<br />

tan(arcsinu) = tan x<br />

y<br />

u ~<br />

=~1-u2 . ~1-u2<br />

-s><br />

-----i'--==.......... x<br />

5. sin 2 0 = cos 2 0 + 1<br />

7. cosx =cos2x<br />

- l-u 2 1<br />

cosx =-­<br />

2<br />

2cosx =-1<br />

21C 41C<br />

x=­ -<br />

3' 3<br />

8.<br />

Chapter 6 Test Exercises 267<br />

cosx=2cos 2 x-I<br />

2 cos 2 x ­ cos x-I = 0<br />

(2cosx + 1)(cosx -1) =0<br />

2 cos x + 1=0 or cos x -1 = 0<br />

x =0 21C 41C<br />

, 3' 3<br />

2~ sin(~) = 3<br />

.(0) 3<br />

sm"2 = 2~<br />

x=O<br />

e . (<br />

sin 20-cos20=1 3 )<br />

"2=arcsm 2~<br />

-cos20= 1<br />

cos20=-1<br />

!!- = 60° 120°<br />

2 '<br />

20 = 180° + n36O°<br />

e = 120°, 240°<br />

0= 90° + 180 0n<br />

0=90°, 270°<br />

9. (a) y =cos(3x)<br />

(since 0 must be in the interval [0,360°»<br />

3x = arccosy<br />

6. csc 2 x = arccosy<br />

0 - 2 cot 0 = 4<br />

3<br />

1+ cot 2 0 - 2 cot 0 = 4<br />

cot 2 0 - 2 cot- 3 = 0<br />

(cot 0 -3)(ootO+ 1)= 0<br />

cotO-3=0 or cotO+ 1=0<br />

cot 0 =3<br />

O=tan-I(i)<br />

0=18.4°, 198.4°<br />

0=18.4°, 135°, 198.4°,<br />

cot 0 =-1<br />

0= tan-I(-l)<br />

0= 135°, 315°<br />

315°<br />

(b)<br />

y<br />

cosx = 1<br />

Let (tJ = arctan(~). Then<br />

arcsin x = arctan(~)<br />

arcsin x = (()<br />

. 4<br />

x =sm{() = ­ 5<br />

tan (() = ~ .<br />

___-+::::....--'-L--_ x<br />

4<br />

267


<strong>Trigonometry</strong> <strong>Solutions</strong> <strong>Manual</strong><br />

268 Chapter 6 Inverse Trigonometric Functions and Trigonometric Equations<br />

n-(t- ~) =arccos(O)<br />

1r ( t -<br />

'31) ='21r + nn, were here n iIS an integer<br />

1 1<br />

t--=-+n<br />

3 2<br />

1 1<br />

t=-+-+n<br />

2 3<br />

5<br />

t=-+n<br />

6<br />

In the interval [0, 1r), n =O. I, and 2 provide valid<br />

values for t.<br />

5 5 5<br />

t=- -+1 -+2<br />

6' 6 • 6<br />

5 11 17<br />

t=-sec -sec -sec<br />

6 '6 '6<br />

268


<strong>Trigonometry</strong> <strong>Solutions</strong> <strong>Manual</strong><br />

7.1 Oblique Triangles and the Law of Sines 269<br />

CHAPTER 7 APPLICATIONS OF<br />

5. A = 37°, B = 48°, C = 18 m<br />

TRIGONOMETRY AND VECTORS<br />

C=1800-A-~<br />

Section 7.1<br />

C = 180°- 37°-48°<br />

C=95°<br />

Connections (page 288)<br />

b C<br />

--=-­<br />

sinB sinC<br />

1. House:<br />

b= 18sin48°<br />

X = (7400-150)(.9) "" 1131.8 ft<br />

sin95°<br />

H 6sec4.1<br />

b=13 m<br />

-3.5sin4.1 0 a C<br />

y = (7400 - 150)(3.5)cos 4.1° z 4390.2 ft --=-­<br />

0-3.5sin4.10<br />

H 6sec4.1<br />

sinA sinC<br />

Fire:<br />

18sin37°<br />

a=--­<br />

X = (7400-690)(2.1) "" 2277.5 ft<br />

sin95°<br />

F<br />

a=l1 m<br />

6see 4.1° + 2.4sin 4.1°<br />

Y = (7400-690)(-2.4)eos4.1° z-2596.2 ft 6. B=52°, C=29°, a=43em<br />

F 6sec4.1 0+2.4sin4.10<br />

A=1800-B-C<br />

A = 180°-52° -29° =99°<br />

2. d=~(XF-XH)2+(YF-YH)2 a b<br />

--=-­<br />

sinA sinB<br />

= ~(2277.5 -1131.8)2 + (-2596.2 - 4390.2)2<br />

b = 43sin52°<br />

z 7079.7 ft<br />

sin 99°<br />

b=34 em<br />

Exercises<br />

a C<br />

--=-­<br />

1. C<br />

sinA sin C<br />

43 c<br />

--=-­<br />

2. C,D sin 99° sin 29°<br />

43sin29°<br />

3. The measure of angle C is c =-__.--:-"<br />

sin 99° _<br />

180°- (60° + 75°) = 45°.<br />

C = 21 em<br />

a c<br />

--=-­<br />

sinA sinC<br />

7. A = 27.2°, C = 115.5°, c = 76.0 ft<br />

a .fi<br />

B=1800-A-C<br />

--=--­ B = 180°-27.2°-115.5°<br />

sin60° sin 45°<br />

B= 37.3°<br />

.fisin 60°<br />

a="";""'--- a c<br />

sin 45° --=-­<br />

sinA sinC<br />

.fi(~) 76.0 sin 27.2°<br />

a=----­<br />

a = .J2<br />

sin 115.5°<br />

T<br />

a =38.5 ft<br />

a =..[3 b c<br />

--=-­<br />

sin B sinC<br />

4. The measure of angle B is b =76.0sin37.3°<br />

180°- (45° + 105°) = 30°. sin 115.5°<br />

a b<br />

--=-­<br />

b =51.0 ft<br />

sinA sinb<br />

a 10<br />

--=-­<br />

sin45° sin 30°<br />

lOsin45°<br />

a=--­<br />

sin 30°<br />

1O(~)<br />

a=-r<br />

a =10.J2<br />

269


<strong>Trigonometry</strong> <strong>Solutions</strong> <strong>Manual</strong><br />

270 Chapter 7 Applications of <strong>Trigonometry</strong> and Vectors<br />

8. C=124.1°, B=18.7°,b=94.6m<br />

0-B-C<br />

A=180<br />

A = 180 0 -18.7 0 -124.1 0 = 37.2 0<br />

a b<br />

--=-­<br />

sinA sin B<br />

94.6 sin 37.2 0<br />

B = 18.5 0<br />

a b<br />

--=-­<br />

a=----­sinA<br />

sinB<br />

sinI8.7°<br />

a =178 m<br />

c b<br />

--=-­<br />

a=239 yd<br />

sinC sinB<br />

b c<br />

94.6 sin 124.1°<br />

--=-­<br />

c=<br />

sinB<br />

sinI8.7°<br />

c=244 m<br />

9. A = 68.41 0 , B = 54.23 0 , a = 12.75ft<br />

0-A-B<br />

C=180<br />

C= 180 0-68.410-54.230<br />

C= 57.36 0<br />

a b<br />

--=-­<br />

sinA sin B<br />

b = 12.75sin54.23°<br />

sin68.41°<br />

b =11.13 ft<br />

a c<br />

--=-­<br />

sin A sinC<br />

12.75sin57.36°<br />

sin68.41°<br />

c = 11.55 ft<br />

10. C = 74.08 0 , B = 69.38 0 , c = 45.38 m<br />

A =180 0-B-C<br />

A = 180 0 - 69.38 0 -74.08 0 = 36.54 0<br />

a c<br />

--=-­<br />

sinA sinC<br />

45.38sin 36.54 0<br />

11. A = 87.2 0 , b = 75.9 yd, C = 74.3 0<br />

0-A-C<br />

B= 180<br />

B = 180 0 - 87.2 0 -74.3 0<br />

12. B = 38 040',<br />

a = 19.7 em, C =91 040'<br />

A =180 0-B-C<br />

= 180 0-38040' -91 040'<br />

=49 040'<br />

b a<br />

--=-­<br />

sin B sinA<br />

b = 19.7sin 38 0 40'<br />

sin 49 040'<br />

b=16.1em<br />

c=----­<br />

c a<br />

--=-­<br />

sinC<br />

A = 180 0 - 20 0 50' -103 0 10'<br />

a=-----­A =56 sin 74.08 0 00'<br />

0<br />

a=28.lOm<br />

AC AB<br />

--=-­<br />

b c<br />

sinB sinC<br />

--=-­<br />

sin B sinC<br />

AB = 132sin 103 0 10'<br />

b = 45.38sin 69.38 0<br />

sin 20 050'<br />

sin 74.08<br />

AB= 361 ft<br />

0<br />

b =44.17 m<br />

BC AC<br />

--=-­<br />

sinA sinB<br />

BC = 132sin 56 000'<br />

75.9sin87.2°<br />

a=----­<br />

sin 18.5<br />

0<br />

sinC<br />

75.9sin 74.3 0<br />

c=----­<br />

sin 18.5<br />

0<br />

c=230 yd<br />

sinA<br />

19.7sin 91 040'<br />

c=----­<br />

sin 49<br />

040'<br />

c=25.8 em<br />

13. B = 20 0 SO', AC = 132 ft. C = 103 010'<br />

A =180 0-B-C<br />

sin 20 050'<br />

BC= 308 ft<br />

270


<strong>Trigonometry</strong> <strong>Solutions</strong> <strong>Manual</strong><br />

7.1 Oblique Triangles and the Law of Sines 271<br />

14. A =35.3°, B =52.8°, AC =675 ft<br />

C=1800-A-B<br />

=180°- 35.3° -52.8° =91.9°<br />

BC AC<br />

--=-­<br />

sinA sinB<br />

BC= 675 sin 35.3°<br />

sin 52.8°<br />

BC = 490 ft<br />

AB AC<br />

--=-­<br />

sinC sinB<br />

AB= 675 sin 91.9°<br />

sin 52.8°<br />

AB=847 ft<br />

15. A =39.70 0,C =30.35°, b =39.74 m<br />

B= 180 0-A-C<br />

B =180°- 39.70° - 30.35°<br />

B = 110.0° (rounded)<br />

a b<br />

--=-­<br />

sinA sinB<br />

39.74 sin 39.70°<br />

sin 109.95°<br />

a =27.01 m<br />

b c<br />

--=-­<br />

sinB sinC<br />

39.74 sin 30.35°<br />

c=<br />

sin 109.95°<br />

c=21.36 m<br />

16. C =71.83°, B =42.57°, a =2.614 em<br />

A =180 0-B-C<br />

A =180°- 42.57° - 71.83° =65.60"<br />

b a<br />

C=97°34'<br />

--=-­<br />

sinB sinA<br />

a b<br />

b = 2.614sin42.57°<br />

--=-­<br />

sinA sinB<br />

sin 65.6QO<br />

b = 268.7 sin 42° 32'<br />

b=1.942 em<br />

sin 39°54'<br />

c a<br />

--=-­<br />

b=283.2 m<br />

sinC sinA<br />

a c<br />

2.614sin 71.83°<br />

--=-­<br />

c=-----­sinA<br />

sinC<br />

sin 65.60°<br />

c=2.727 em<br />

17. B =42.88°, C =102.40°, b =3974 ft<br />

A = 180°- B-C<br />

A = 180° - 42.88° -102.40°<br />

A = 34.72°<br />

a b<br />

--=-­<br />

sinA sinB<br />

3974 sin 34.72°<br />

a=----­<br />

sin 42.88°<br />

a =3326 ft<br />

b c<br />

--=-­<br />

sin B sinC<br />

3974 sin 102.40°<br />

c=-----­<br />

sin42.88°<br />

c =5704 ft<br />

18. A = 18.75°, B =51.53°, c = 2798 yd<br />

C=1800-A-B<br />

C =180°-18.75°-51.53° = 109.72°<br />

a c<br />

--=-­<br />

sinA sinC<br />

2798 sin 18.75°<br />

a=----­<br />

a=----­<br />

sin 109.72°<br />

a = 955.4 yd<br />

b c<br />

--=-­<br />

sinB sinC<br />

b = 2798sin51.53°<br />

sin 109.72°<br />

b=2327 yd<br />

19. A =39° 54', a =268.7 m, B = 42° 32'<br />

C=1800-A-B<br />

C = 180°-39°54' -42°32'<br />

268.7sin97°34'<br />

c=-----­<br />

sin 39° 54'<br />

c= 415.2 m<br />

271


<strong>Trigonometry</strong> <strong>Solutions</strong> <strong>Manual</strong><br />

272 Chapter 7 Applications of <strong>Trigonometry</strong> and Vectors<br />

20, C=79°18',c=39.81mm, 26, T<br />

A =32°57'<br />

B=1800-A-C<br />

B =180° - 32°57' -79°18'<br />

=67°45'<br />

a c<br />

--=-­<br />

sinA sinC<br />

39.81 sin 32°57'<br />

a=<br />

sin 79° 18'<br />

s<br />

a=22.04mm<br />

b c<br />

S = 180° - 32°50' -102°20'<br />

--=-- = 44°50'<br />

sinB sinC<br />

b = 39.81sin67°45'<br />

RS 582<br />

=<br />

sin 79°18'<br />

sin 32°50' sin 44°50'<br />

b=37.50 mm<br />

RS=448 yd<br />

22, In Example 1, the Pythagorean theorem does not 27. Let C =the transmitter.<br />

apply because triangle ABC is not a right triangle.<br />

N N<br />

Angle A is 32.0°, B =81.8°, therefore C = 66.2°, t t<br />

One of the angles has to be 90° for ABC to be a I c I<br />

I<br />

I<br />

right triangle.<br />

I<br />

I<br />

147.7'<br />

1 302S<br />

24, No. If a is twice as long as b, then sin A =2 sin B, I<br />

I<br />

but A is not necessarily twice as large as B.<br />

A 3.46mi B<br />

25. c<br />

Since side AB is on an east-west line. the angle<br />

between it and any north-south line is 90°.<br />

A =90° - 47.7° = 42.3°<br />

B =302.5° - 270° = 32.5°<br />

C=1800-A-B<br />

=180° - 42.3° - 32.5°<br />

C=105.2°<br />

AC 3.46<br />

---=---­<br />

sin 32.5° sin 105.2°<br />

AC =3.46sin32.5°<br />

sin 105.2°<br />

AC= 1.93 mi<br />

A =180 0-B-C<br />

A = 180° -112°10'-15°20'<br />

A =52°30'<br />

BC AB<br />

--=-­<br />

sinA sinC<br />

AB =354 sin 15°20'<br />

sin 52°30'<br />

AB= 118 m<br />

Led d =the distance the ship traveled between<br />

the two observations;<br />

L =the location of the lighthouse.<br />

N<br />

44.2'<br />

d<br />

L<br />

12.5<br />

38.8'<br />

L = 180° - 38.8° - 44.2°<br />

L=97°<br />

d 12.5<br />

--=<br />

sin 97° sin 44.2°<br />

d=17.8km<br />

272


<strong>Trigonometry</strong> <strong>Solutions</strong> <strong>Manual</strong><br />

7.1 Oblique Triangles and the Law of Sines 273<br />

29.<br />

x 12.0<br />

32. Label the centers of the atoms A, B, and C .<br />

C 2+3<br />

sin 54.8° sin 70.4°<br />

12.0sin54.8°<br />

x=------:­<br />

sin 70.4°<br />

x =10.4 in.<br />

30. Let M = Mark's location;<br />

L =Lisa's location;<br />

T =the tree's location;<br />

R =a point across the river from Mark so that<br />

MR is the distance across the river.<br />

N<br />

115.45'<br />

428.3rn<br />

45.47'<br />

M - R<br />

8 a T<br />

L<br />

(J = 180° -115.45° = 64.55°<br />

a =180° -45.47°-64.55° = 69.98°<br />

f3 =(J =64 .55° Alternate interior angles<br />

In triangle MTL,<br />

MT 428.3<br />

=<br />

sin 45.47° sin 69.98°<br />

MT = 324.9645.<br />

In right triangle MTR,<br />

MR . R.<br />

-=SlnfJ<br />

MT<br />

MR =sin 64.550<br />

324.9645<br />

MR =293.4 m.<br />

31. Label a in the triangle as shown.<br />

1.6<br />

.." 1.6 + 3.6 =--­<br />

sin38° sina<br />

. (,Ic;6_+-:;2:.... .7"-)s:..:;i:..:;n.:..38=-°<br />

sma=­<br />

1.6+3.6<br />

a=31°<br />

(J= 180 0-38°-a<br />

= 180°-38°-31°<br />

(J=11l0<br />

A<br />

a = 2.0+ 3.0 = 5.0<br />

c = 3.0+4.5 = 7.5<br />

sinC sinA<br />

--=-­<br />

c a<br />

. C 7.5sin 18°<br />

sm =---­<br />

5<br />

C=28°<br />

B = 180° -18° - 28° = 134°<br />

b a<br />

--=-­<br />

sinB sinA<br />

b 5.0sin134° 1<br />

----= 2<br />

sin 18°<br />

The distance between the centers of atoms A and<br />

Cis 12.<br />

33. Let x = the distance to the lighthouse at bearing<br />

N 37° E.<br />

y = the distance to the lighthouse at bearing<br />

N25°E.<br />

N<br />

Lighthouse<br />

2.5 mi<br />

(J =180° - 37° =143°<br />

a = 180 0-(J-25°<br />

= 180° -143° - 25° =12°<br />

2.5 x<br />

--=--­<br />

sina sin 25°<br />

2.5sin 25°<br />

x=---­<br />

sinl2°<br />

=5.1 mi<br />

2.5 y<br />

--=-­<br />

sina<br />

sin(J<br />

2.5sin 143°<br />

y=<br />

sin 12°<br />

=7.2mi<br />

273


<strong>Trigonometry</strong> <strong>Solutions</strong> <strong>Manual</strong><br />

274 Chapter 7 Applications of <strong>Trigonometry</strong> and Vectors<br />

34. Let A = the location of the balloon;<br />

B = the location of the farther town;<br />

C = the location of the closer town.<br />

A<br />

~<br />

B 1.5 C<br />

Angle ABC = 31° and angle ACB = 35° because<br />

the angles of depression are alternate interior<br />

angles with the angles of the triangle.<br />

AngleBAC= 180°-31°-35°= 114°<br />

sin BAC sin ACB<br />

---=--­<br />

1.5 AB<br />

AB = 1.5sin 35°<br />

sinl14°<br />

AB=.94<br />

sin ABC = AD<br />

AB<br />

AD = .94(sin 31°)<br />

AD=.49<br />

The balloon is .49 mi above the ground .<br />

35. c<br />

A<br />

Angle C is equal to the difference between the<br />

angles ofelevation.<br />

C = B - A = 52.7430° - 52.6997° = .0433°<br />

The distance BC to the moon can be determined<br />

using the law of sines.<br />

BC AB<br />

--=-­<br />

sinA sinC<br />

BC= ABsinA<br />

sinC<br />

398 sin 52.6997°<br />

=<br />

sin.0433°<br />

== 418,930 km<br />

Ifone finds distance AC, then<br />

AC= ABsinB<br />

sinC<br />

398sin(180° - 52.7430°)<br />

=<br />

sin .0433°<br />

==419,171 km.<br />

In either case the distance is approximately<br />

419,000 km compared to the actual value of<br />

406,oookm.<br />

36.<br />

We must find the length of CD.<br />

Angle BAC is equal to 35° - 30° = 5°.<br />

Side AB = 5000 ft. Since triangle ABC is a right<br />

triangle,<br />

5000<br />

cos5°=-­<br />

AC<br />

AC= 5000<br />

cos5°<br />

== 5019 ft.<br />

The angular coverage of the lens is 60°, so angle<br />

CAD = 60°. From geometry, angle ACB = 85°,<br />

angle ACD = 95°, and angle ADC =25°. We now<br />

know three angles and one side in triangle ACD<br />

and can use the law of sines to solve for the<br />

length of CD.<br />

CD AC<br />

---=--­<br />

sin 60° sin 25°<br />

CD =5019sin60°<br />

sin 25°<br />

CD == 10,285 ft<br />

The photograph would cover a horizontal distance<br />

. '85 ft 10,285 1 95 .<br />

of approximately 10,2 or ---==. rm,<br />

5280<br />

37. A<br />

Determine the length of CB which is the sum of<br />

CD and DB. Triangle ACE is isosceles so angles<br />

ACE and AEC are both equal to 47°. It follows<br />

that angle ACD is equal to 47° - 5° =42°. Angles<br />

CAD and DAB both equal 43° since the<br />

photograph is taken with no tilt and AD is the<br />

bisector of angle CAB. The length of AD is<br />

3500 ft. Since the measures of the angles in<br />

triangle ABC total 180°, angle ABC equals<br />

180° - (86° + 42°) = 52° .<br />

D<br />

274


<strong>Trigonometry</strong> <strong>Solutions</strong> <strong>Manual</strong><br />

7.1 Oblique Triangles and the Law of Sines 275<br />

Using the law of sines,<br />

CD= 3500 sin 43° "" 3567.3 ft<br />

sin 42°<br />

and<br />

DB = 3500sin 43° "" 3029.1 ft.<br />

sin52°<br />

The ground distance in the resulting photograph<br />

will equal the length of segment CB.<br />

CB=CD+DB<br />

=3567.3+3029.1<br />

CB = 6596.4 ft<br />

38. A<br />

c<br />

In the figure, angle CAB is 72°.<br />

Triangle ACE is isosceles so angles ACE and<br />

AEC are both equal to 54°. It follows that angle<br />

ACB is equal to 54° - 5° = 49° . Angles CAD and<br />

DAB both equal 36° since the photograph is taken<br />

with no tilt and AD is the bisector of angle CAB<br />

with length 3500 ft. Since the measures of the<br />

angles in triangle ABC total 180°, angle ABC<br />

equals 180° - (72° + 49°) = 59°.<br />

Using the law of sines,<br />

CD = 3500 sin 36° ""2725 .9 ft<br />

sin 49°<br />

and<br />

DB= 3500 sin 36° "" 2400.1 ft.<br />

sin 59°<br />

The ground distance in the resulting photograph<br />

will equal the length of segment CB.<br />

CB=CD+DB<br />

= 2725.9 + 2400.1<br />

CB = 5126 ft<br />

The coverage is less with a longer focal length.<br />

Cameras that have shorter focal lengths have a<br />

greater ground coverage in aerial photographs.<br />

39. Using K =!bh,<br />

2<br />

K =!(l)(.J3)<br />

2<br />

.J3<br />

=­<br />

2<br />

E<br />

Using K = !absin C,<br />

2<br />

K = t(.J3)(l) sin 90°<br />

=!(.J3)(l)(I)<br />

2<br />

.J3<br />

=­<br />

2<br />

40. Using K=!bh,<br />

2<br />

K=!(2)(.J3)<br />

2<br />

=.J3.<br />

Using K = !absin C,<br />

2<br />

K = ! (2)(2)sin 60°<br />

2<br />

. = t(2)(2{ ~)<br />

=.J3.<br />

41. Using K =!bh,<br />

2<br />

K = !(l)(.J2)<br />

2<br />

.J2<br />

=­<br />

2<br />

Using K =! ab sin C,<br />

2<br />

K = !(2)(I)sin45°<br />

2<br />

=~(2)(l)( v;)<br />

.J2<br />

=­<br />

2<br />

42. Using K=!bh,<br />

2<br />

K =!(2)(1)<br />

2<br />

=1.<br />

Using K = !absin C,<br />

2<br />

K = !(2)(.J2)(sin45°)<br />

2<br />

= ~(2)(.J2{ v;)<br />

=1.<br />

275


<strong>Trigonometry</strong> <strong>Solutions</strong> <strong>Manual</strong><br />

276 Chapter 7 Appl i ~~ations of <strong>Trigonometry</strong> and Vectors<br />

43. A = 42.5°, b = 13.6 rn, e =10.1 m<br />

Angle A is included between sides band e.<br />

50. Area =.!. (52.1)(21.3) sin 42.2°<br />

2<br />

2<br />

Area =.!. be sin A<br />

= 373 m<br />

2<br />

=.!.(13.6)(10.1) sin 42.5°<br />

51. The function y = sin x is increasing from y = 0 to<br />

2<br />

2<br />

Y =1 on the interval 0 s x s ~ .<br />

=46.4 m<br />

2<br />

44. C = 72.2°, b = 43.8 ft, a = 35.1 ft<br />

a b<br />

Angle C is included between sides a and b.<br />

53. --=-­<br />

sin A sinB<br />

Area =.!. ab sin C<br />

Solve for b.<br />

2<br />

asinB =b<br />

= .!.(35.1)(43.8) sin 72.2°<br />

sin A<br />

2<br />

b=asinB<br />

= 732 ft2<br />

sin A<br />

45. B = 124.5°, a = 30.4 ern, e = 28.4 em<br />

. asinB<br />

Angle B is included between sides a and e.<br />

54. From Exercise 51, b =--.<br />

sin A<br />

Area =.!. ac sin B<br />

sinB<br />

sinB<br />

2<br />

If--< 1, then b =a .--< a ·1 or a.<br />

sin A<br />

sin A<br />

= .!.(30.4)(28.4) sin 124.5°<br />

So, b


<strong>Trigonometry</strong> <strong>Solutions</strong> <strong>Manual</strong><br />

7.2 The Ambiguous Case of the Law of Sines 277<br />

58. (b) If A =18° and B =36° Another possible value for B is<br />

Section 7.2<br />

Exercises<br />

1. A<br />

2. D<br />

A. =[5 sin 18°sin 36°]R2<br />

sin(18 + 36)°<br />

2<br />

'" 1.12257R<br />

(c) i. 11.4 in. x (10 + 13) in. =8.77 in. 2<br />

ii. A =5J5 sin 18°sin 36° }.308)2<br />

'l sin(l8 0+360 )<br />

A =5.32 in. 2<br />

iii. red<br />

3. The vertical distance from the point (3, 4) to the<br />

x-axis is 4.<br />

(a) Ifh is more than 4, two triangles can be<br />

drawn. But h must be less than 5 for both<br />

triangles to be on the positive x-axis.<br />

So, 4 < h 5, then only one triangle is possible on<br />

the positive x-axis.<br />

(c) If h < 4, then no triangle is possible, since the<br />

side of length h would not reach the x-axis.<br />

4. (a) Since the side must be drawn to the positive<br />

z-axis, no value ofh would produce two<br />

triangles.<br />

(b) Since the distance from the point (-3, 4) to<br />

the origin is 5, any value of h greater than 5<br />

would produce exactly one triangle.<br />

(c) Likewise, any value of h less than or equal to<br />

5 would produce no triangle.<br />

180° - 31.2 0 =148.8°. This measure, combined<br />

with the angle of 95°, is too large, however.<br />

Therefore, only one triangle is possible.<br />

6. b =60, a = 82, B =100°<br />

b a<br />

--=-­<br />

sinB sinA<br />

60 82<br />

=-­<br />

sin 100° sin A<br />

. A 82 sin 100°<br />

Sin =---­<br />

60<br />

sin A = 1.346<br />

Since this is impossible, no triangle can be drawn<br />

with these parts.<br />

7. a = 31, b = 26, B = 48°<br />

a b<br />

--=-­<br />

sinA sin B<br />

31 26<br />

--=--­<br />

sinA sin 48°<br />

. A 31sin48°<br />

Sin =---­<br />

26<br />

sin A = .886<br />

A =62.4°<br />

Another possible value for A is<br />

180 0 - 62.4° = 117.6°. Therefore, two triangles<br />

are possible.<br />

8. a = 35, b = 30, A = 40°<br />

a b<br />

--=-­<br />

sinA sinB<br />

35 30<br />

---=-­<br />

sin 40° sinB<br />

. B 30sin40°<br />

Sin =---­<br />

35<br />

sinB =.551<br />

B-=33.4°<br />

Another possible value for B is<br />

180° - 33.4° =146.6°, but this is too large to be<br />

in a triangle that also has a 40° angle. Therefore,<br />

only one triangle is possible.<br />

9. a = 50, b = 61, A = 58°<br />

a b<br />

--=-­<br />

S. a =50, b =26, A =95°<br />

sinA sinB<br />

a b<br />

50 61<br />

--=-­<br />

---=-­<br />

sinA sinB<br />

sin 58° sin B<br />

50 26<br />

. B 61sin58°<br />

---=-­<br />

sm =---­<br />

sin 95° sinB<br />

50<br />

. . 26 sin 95°<br />

sin B -= 1.03<br />

smB=---­<br />

50<br />

sinB '" .518<br />

B = 31.2°<br />

sin B > 1 is impossible. Therefore, no triangle is<br />

possible for the given parts.<br />

277


<strong>Trigonometry</strong> <strong>Solutions</strong> <strong>Manual</strong><br />

278 Chapter 7 Applications of <strong>Trigonometry</strong> and Vectors<br />

10. B=54°,c=28,b=23<br />

b c<br />

--=-­<br />

sinB sinC<br />

23 28<br />

--=-­<br />

sin 54° sinC<br />

. C 28sin54°<br />

Sin =---­<br />

23<br />

sinC:::: .985<br />

C::::80°<br />

Another possible value for C is<br />

180° - 80° =100°. Therefore, two triangles are<br />

possible.<br />

b<br />

sin B<br />

a<br />

sinA<br />

11. --=-­<br />

2 -J6<br />

--=-­<br />

sinB sin 60°<br />

. B 2 sin 60°<br />

sm = -J6<br />

2(4)<br />

= -J6<br />

~<br />

=­<br />

2<br />

B=45°<br />

Another possible value for B is 180° - 45 = 135°,<br />

but this is too large since A = 60°. Therefore,<br />

B = 45°.<br />

12. A =45°, a=~, b=3<br />

b a<br />

--=-­<br />

sinB sinA<br />

3 ~<br />

--=-­<br />

sinB sin 45°<br />

. B 3sin45°<br />

Sin = ~<br />

~4)<br />

=~<br />

1<br />

=­<br />

2<br />

B=3O°<br />

Another possible value for B is<br />

180° - 30° =150°, but this is too large since<br />

A = 45°. Therefore, B = 30°.<br />

In Exercises 13-30, the number of possible triangles is<br />

found based on the given conditions. Remember that, for<br />

example, if angle B and sides a and b are given such that<br />

a> b > a sin B, then two triangles are possible.<br />

13. A = 29.7°, b =41.5 ft, a = 27.2 ft<br />

b > a > b sin A tells us that there are two possible<br />

triangles.<br />

sin B sinA<br />

--=-­<br />

b<br />

a<br />

· B 41.5sin 29.7°<br />

Sin =----­<br />

27.2<br />

sin B = .75593878<br />

Bt=49.1°<br />

C) = 180° - A - B)<br />

= 180°-29.7-49.1°<br />

= 101.2°<br />

~ = 180° - Bt = 130.9°<br />

Cz = 19.4°<br />

14. B = 48.2° , a =890 em, b = 697 ern<br />

a > b > a sin B tells us that there are two possible<br />

triangles.<br />

sinA sinB<br />

--=-­<br />

a b<br />

sin A = 890sin 48.2° = .95189905<br />

697<br />

A) =72.2°<br />

C) = 180°- B-A)<br />

= 180° - 48.2° - 72.2°<br />

=59.6°<br />

A 2 = 180°-A) =107.8°<br />

C 2 = 180°- B-Az<br />

= 180° - 48.2° -107.8°<br />

=24.0°<br />

15. C =41°20', b = 25.9 m, C = 38.4 m<br />

c > b tells us that there is only one triangle.<br />

sinB sinC<br />

--=-­<br />

b c<br />

· B 25.9sin 41°20'<br />

Sin =-----­<br />

38.4<br />

sin B =.44545209<br />

B =26°30'<br />

Note that B,* 153°30' because<br />

153°30'+C> 180°.<br />

A = 180°-26°30' -41°20'<br />

A = 112°10'<br />

16. B =48° 50', a = 3850 in., b = 4730 in.<br />

Since b > a, only one triangle exists.<br />

sinA sinB<br />

--=-­<br />

a b<br />

· A 3850sin48°50'<br />

510 =-----­<br />

4730<br />

= .61274255<br />

A =37°50'<br />

278


<strong>Trigonometry</strong> <strong>Solutions</strong> <strong>Manual</strong><br />

7.2 The Ambiguous Case of the Law of Sines 279<br />

A ~ 142° 10' since 142 010' + B> 180 0<br />

C=1800-A-B<br />

= 180° - 37° 50' - 48°50'<br />

=93°20'<br />

17. B=74.3°,a=859m,b=783m<br />

sinA sinB<br />

--=-­<br />

a b<br />

· A 859 sin 74.3°<br />

Sin =----­<br />

783<br />

sin A =1.0561331<br />

sin A > 1 is impossible.<br />

Note that b < a sin B.<br />

. No such triangle exists.<br />

18. C = 82.2°, a = 10.9 km, C = 7.62 km<br />

sinA sinC<br />

--=-­<br />

a C<br />

· A 10.9 sin 82.2°<br />

Sin =----­<br />

7.62<br />

= 1.4172115<br />

sin A > 1 is impossible.<br />

Note that c < a sin C.<br />

No such triangle exists.<br />

19. A = 142.13°, b = 5.432 ft. a = 7.297 ft<br />

a > b tells us that there is one triangle.<br />

sinB sinA<br />

--=-­<br />

b a<br />

· B 5.432 sin 142.13°<br />

Sin =-----­<br />

7.297<br />

sin B = .45697580<br />

B =27.19°<br />

B must be acute since A is obtuse.<br />

C = 180°-142.13° -27.19°<br />

C=1O.68°<br />

20. B = 113.72°. a = 189.6 yd. b = 243.8 yd<br />

Since b > a, only one triangle exists.<br />

sinA sin B<br />

--=-­<br />

a b<br />

189.6sin 113.72°<br />

sin A<br />

243.8<br />

= .71198940<br />

A =45.40°<br />

A is acute since B is obtuse.<br />

C=1800-A-B<br />

= 1800- 45.40° -113.72°<br />

=20.88°<br />

21. A = 42.5°, a = 15.6 ft, b = 8.14 ft<br />

a > b tells us that there is only one triangle.<br />

sinB sinA<br />

--=-­<br />

b a<br />

· B bsinA<br />

Sin =-­ a<br />

8.14 sin 42.5°<br />

=<br />

15.6<br />

sin B =.35251951<br />

B=20.6°<br />

B ¢.159.4° since 159.4°+ A > 180°.<br />

C =180° - 42.5° - 20.6°<br />

=116.9°<br />

c a<br />

--=-­<br />

sinC sinA<br />

asinC<br />

c=-­<br />

sin A<br />

15.6sin 116.9°<br />

=<br />

sin42'so<br />

c= 20.6 ft<br />

22. C = 52 .3°, a = 32.5 yd, c = 59.8 yd<br />

Since c > a, only one triangle exists.<br />

sinA sinC<br />

--=-­<br />

a c<br />

· A 32.5 sin 52.3°<br />

Sin =<br />

59.8<br />

= .43001279<br />

A = 25.5°<br />

A ~ 154.5° since 154.5° of: C > 180°.<br />

B = 180° - 25.5° - 52.3°<br />

=102.2°<br />

b c<br />

--=-­<br />

sin B sinC<br />

b = 59.8sin102.2°<br />

sin 52.3°<br />

= 73.9 yd<br />

23. B = 72.2°, b = 78.3 m, c = 145 m<br />

sinC sinB<br />

--=-­<br />

c b<br />

· C csinB<br />

Sin =-­ b<br />

145 sin 72.2°<br />

=<br />

78.3<br />

sin C = 1.7632026<br />

sin C > 1 is impossible. Note that c sin B > b.<br />

No such triangle exists.<br />

279


<strong>Trigonometry</strong> <strong>Solutions</strong> <strong>Manual</strong><br />

280 Chapter 7 Applications of <strong>Trigonometry</strong> and Vectors<br />

24. C = 68.5°, C = 258 em, b = 386 em<br />

sin B sinC<br />

--=-­<br />

b C<br />

. B 386 sin 68.5°<br />

sm =----­<br />

258<br />

= 1.39202008<br />

sin B > 1 is impossible. Note that c < b sin C.<br />

No such triangle exists.<br />

25. a = 38°40', a = 9.72 Ian, b = 11.8 Ian<br />

b > a > b sin A tells us that there are two possible<br />

triangles.<br />

sin B sinA<br />

--=-­<br />

b a<br />

. B bsinA<br />

sm =-­ a<br />

11.8sin 38° 40'<br />

=<br />

9.72<br />

sin B = .75848811<br />

~ =49°20'<br />

C l = 180° - 38° 40' - 49° 20'<br />

CI =92°00'<br />

_c_l_=_a_<br />

sinCI<br />

sin A<br />

asinC I<br />

ci = sin A<br />

9.72 sin 92°00'<br />

= sin 38°40'<br />

ci = 15.5 Ian<br />

~ =130°40'<br />

C 2 =180°-38°40'-130°40'<br />

C 2 = 10°40'<br />

~=_a_<br />

sinC 2 sin A<br />

asinC 2<br />

c2 = sinA<br />

9.72 sin 10° 40'<br />

= sin38°4O'<br />

C2 =2.88 km<br />

26. C = 29° 50' , a = 8.61 m, c = 5.21 m<br />

sinA sinC<br />

--=-­<br />

a c<br />

.."<br />

a > c> a sin C tells us that there are two possible<br />

triangles.<br />

sin A = 8.61sin29°50' = .82212894<br />

5.21<br />

Al = 55°20'<br />

~ =180°-55°20'-29°50'<br />

=94°50'<br />

-!L=_c_<br />

sin ~ sinC<br />

~ = 5.21sin 94° 50'<br />

sin29°50'<br />

= 10.4 m<br />

A 2 = 180° - AI<br />

= 124°40'<br />

~ = 180° -124°40' -29°50'<br />

=25°30'<br />

-.!!L=_c_<br />

sin~ sinC<br />

b _ 5.21sin25°30'<br />

2 - sin 29°50'<br />

=4.51m<br />

27. A = 96.80°, b = 3.589 ft, a = 5.818 ft<br />

a > b tells us that there is only one possible<br />

triangle.<br />

sinB sinA<br />

--=-­<br />

b a<br />

. B bsinA<br />

sm =-­ a<br />

3.589 sin 96.80°<br />

=<br />

5.818<br />

sin B = .61253922<br />

B=37.77°<br />

B must be acute since A is obtuse.<br />

C= 180°-96.80°-37.77°<br />

C= 45.43°<br />

c a<br />

--=-­<br />

sinC sinA<br />

asinC<br />

c=-­<br />

sin A<br />

5.818sin45.43°<br />

=<br />

sin 96.80°<br />

c=4.174ft<br />

28. C = 88.70°, b = 56.87 yd, c = 112.4 yd<br />

Since c > b, only one triangle exists.<br />

sinB sinC<br />

--=-­<br />

b c<br />

. B 56.87 sin 88.70°<br />

sm =-----­<br />

112.4<br />

= .50583062<br />

B= 30.39°<br />

B~ 149.61° since 149.61°+C > 180°.<br />

A = 180° - 30.39° - 88.70°<br />

= 60.91°<br />

a c<br />

--=-­<br />

sinA<br />

sin C<br />

112.4sin 60.91 °<br />

a=-----­<br />

sin 88.70°<br />

= 98.25 yd<br />

280


<strong>Trigonometry</strong> <strong>Solutions</strong> <strong>Manual</strong><br />

7.2 The Ambiguous Case ofthe Law of Sines 281<br />

29. B = 39.68°, a = 29.81 m, b = 23.76 m<br />

a > b > a sin B tells us that there are two possible<br />

triangles.<br />

sinA sin B<br />

--=-­<br />

a b<br />

. A osinB<br />

sm =-­<br />

b<br />

29.81sin 39.68°<br />

=<br />

23.76<br />

sin A = .80 108002<br />

Al = 53.23°<br />

C I= 180° - 53.23° - 39.68°<br />

C I =87.09°<br />

b ci<br />

--=-­<br />

sinB sinq<br />

bsinq<br />

CI = sinB<br />

23.76 sin 87.09°<br />

sin 39.68°<br />

ci = 37.16 m<br />

A2 =126.77°<br />

C2 = 180° -126.77° - 39.68°<br />

C2 =13.55°<br />

b c2<br />

--=-­<br />

sinB sinC2<br />

bsinC 2<br />

C2 = sinB<br />

23.76sin 13.55°<br />

= sin 39.68°<br />

c2 =8.719 m<br />

30. A = 51.20°, C = 7986 em, a = 7208 em<br />

c> a > c sin A tells us that there are two triangles.<br />

sinC sinA<br />

--=-­<br />

c a<br />

. C 7986 sin 51.20°<br />

sm =----­<br />

7208<br />

=.86345630<br />

C I =59.71°<br />

Bt =180°-51.20°-59.71°<br />

=69.09°<br />

...l!L=_o_<br />

sin Bt sinA<br />

.. ~ = 7208 sin 69.09°<br />

sin51.20°<br />

=8640 em<br />

C 2 =180 0-q<br />

=120.29°<br />

~ = 180° - 51.20° -120.29°<br />

=8.51°<br />

--.!!L=_a_<br />

sin B 2 sinA<br />

~ = 7208 sin 8.51 °<br />

sin 51.20°<br />

=1369 em<br />

31. a = .$,c = 2...[5, A = 30°<br />

a c<br />

--=-­<br />

sinA sinC<br />

. C 2.$sin 30°<br />

sm = ...[5<br />

sinC= 1<br />

C=90°<br />

This is a right triangle.<br />

35. Let A = 38° 50', a = 21.9, b = 78.3.<br />

b sin A = 78.3 sin 38° 50'<br />

=49.1<br />

Thus, 21.9 < 49.1. That is, a < b sin A.<br />

The piece of property cannot exist with the given<br />

data.<br />

36. Let C = 28° 10', so c = 21.2.<br />

Let a = 26.5. Then B is the obtuse angle in the<br />

figure.<br />

a sin C = 26.5 sin 28° 10'<br />

= 12.509006<br />

Thus, a > c > a sin C.<br />

This fact tells us that there are two possible<br />

triangles with the given datil: one has an acute<br />

angle B and the other has an obtuse angle B as<br />

shown.<br />

sinA sinC<br />

--=-­<br />

a c<br />

. A 26.5 sin 28° 10'<br />

sm =<br />

21.2<br />

=.59004745<br />

Al = 36°10'<br />

Bt = 180° - 28° 10' - 36° 10'<br />

= 115°40'<br />

-.lL=_c_<br />

sinBt sinC<br />

b = 21.2 sin 115° 40'<br />

I sin 28°10'<br />

= 40.5 yd<br />

A 2 = 143°50'<br />

~ = 180° - 28° 10'-143°50'<br />

=8°00'<br />

~ c<br />

--=-­<br />

sin~ sinC<br />

b _ 21.2sin8°00'<br />

2 - sin 28° 10'<br />

=6.25 yd<br />

281


<strong>Trigonometry</strong> <strong>Solutions</strong> <strong>Manual</strong><br />

282 Chapter 7 Applications of <strong>Trigonometry</strong> and Vectors<br />

....<br />

a + b sin A + sin B Connections (page 305)<br />

37. Prove that --= .<br />

b sin B<br />

Start with the law of sines.<br />

1<br />

a b 2<br />

--=-­<br />

sinA sin B = .!.K2 . 3 - 5(-1)+(-1)(0) - 3·4+4·5 - O· 2~<br />

2<br />

(<br />

sin A X-!!-) = (sin A )(-/!-)<br />

b smA b smB = .!.K6+5 -12+20~<br />

a sinA<br />

2<br />

-=-­<br />

b sinB =.!.~9\<br />

2<br />

a<br />

-+<br />

1<br />

=--+<br />

sinA 1<br />

= 9.5 square units<br />

b sinB<br />

a b sinA sinB<br />

-+-=--+-­<br />

1. A =- kX\Y2 - y\x2 + x2Y3 - Y2x3 + x3Y\ - Y3x\ ~<br />

b b sin B sinB 2. a=~(2-(-I»2+(5-3)2<br />

a+b sinA+sinB<br />

--=---- = ~32 +2 2<br />

b<br />

sinB<br />

= ..[13 '" 3.60555<br />

a - b sin A - sin B b = ~(2-4)2 +(5 _0)2<br />

38. Provet h at --= .<br />

a+b sinA+sinB<br />

Start with the law of sines.<br />

= ~(_2)2 +5 2<br />

a b =.J29 '" 5.38516<br />

--=-­<br />

sinA sinB<br />

c = ~(-1-4)2 +(3-0)2<br />

bsinA<br />

a=--.<br />

sin B =~(_5)2 +3 2<br />

Substitute for a in the expression<br />

=.J34 '" 5.83095<br />

(a-b)+(a+b).<br />

1<br />

a-b ~-b<br />

s=-(a+b+c)<br />

2<br />

-=~<br />

a+b sin B +b = .!.(..[13 +.J29 +.J34) '" 7.41083<br />

bsinA-bsinB<br />

_ sIDB<br />

2<br />

- bsjn t\+bsin B<br />

A =..j's("""s---a-)(-s-----,-,b)--,(s:---c....,...)<br />

sIDB<br />

sin A -sinB '" ~(7 .41083)(3.80528)(2.02567)(1.57988)<br />

=<br />

sin A +sinB<br />

'" 9.5 square units<br />

Section 7.3<br />

3. By the law of cosines,<br />

c 2 =a 2 +b 2 -2abcosC<br />

Connections (page 303) 2<br />

2ab cos C =a 2 + b 2 - c<br />

1. In a triangle, the ratio of the tangent of half the a 2 +b 2 _c 2<br />

cosC=----­<br />

difference between two angles to the tangent of<br />

2ab<br />

half the sum of those angles is equal to the ratio<br />

Using the lengths from part 2, we have:<br />

of the difference between the sides opposite those C 13+29-34<br />

angles and the sum of those sides. cos = 2..[13. Ji9<br />

2. In a triangle, the sum of two sides divided by the<br />

third side is equal to the cosine of half the<br />

difference between those two sides and the sine of<br />

half the angle opposite the third side.<br />

'" 0.20601<br />

C '" 78.1 0<br />

A = .!.ab sin C<br />

2<br />

= .!...[13 ..J29 sin 78.1 0<br />

2<br />

'" 9.5 square units<br />

282


<strong>Trigonometry</strong> <strong>Solutions</strong> <strong>Manual</strong><br />

7.3 The Law of Cosines 283<br />

Exercises<br />

1. (a) law of cosines<br />

c 2 = a 2 + b 2 - 2ab cos C<br />

2abcosC=a 2 +b 2 _c 2<br />

a 2 +b 2 _c 2<br />

cosC=----­<br />

2ab<br />

(342)2 + (116)2 - (401)2<br />

=-'-----'---'----'--"'-----'-­<br />

2(342)(116)<br />

"" -.38290<br />

C "" 112.5°<br />

(b) law of cosines<br />

2 2<br />

+ b 2 c = a - 2ab cos C<br />

= (12.2)2 +(13.1)2 -2(l2.2)(13.l)cos20.2°<br />

"" 20.47<br />

c""4.52<br />

(c) law of sines<br />

b a<br />

--=-­<br />

sinB sinA<br />

b<br />

a<br />

--=<br />

sin B sin(180 -(B+ C»<br />

b a<br />

--=<br />

sin B sin(B + C)<br />

b= asinB<br />

sin(B+ C)<br />

(21.13) sin(48° 13')<br />

= sin(48° 13' + 81° 42')<br />

=20.54<br />

(d) Neither is applicable.<br />

2. Subtract the sum of the two given angles from<br />

180° to obtain the third angle.<br />

3. a 2 = 3 2 + 8 2 - 2(3)(8) cos 60°<br />

=9+64-4s(~)<br />

=73-24<br />

=49<br />

a=7<br />

4. a 2 = 1 2 + (4.J2i - 2(1)(4.J2) cos 45°<br />

=1+32- 8 4~)<br />

=33-8<br />

=25<br />

a=5<br />

b 2 +c 2 _a 2<br />

5. cosO=---­<br />

2bc<br />

1 2 + (../3)2 _1 2<br />

= -2""';'(-I)('../3F'"3)­<br />

1+3-1<br />

= 2../3<br />

3<br />

- 2../3<br />

../3<br />

=­ 2<br />

0=30°<br />

b 2 +c 2 _a 2<br />

6. cosO =---­<br />

2bc<br />

3 2 +5 2 _7 2<br />

=<br />

2(3)(5)<br />

9+25-49<br />

=<br />

30<br />

1<br />

=-­<br />

2<br />

0=120°<br />

7. C=28.3°,b=5.71 in.,a=4.21 in.<br />

2 2+b2-2abcosC<br />

c =a<br />

c = ~(4.21)2 +(5.71)2 -2(4.21)(5.71)cos28.3°<br />

c = 2.83 in.<br />

Keep all digits of ...[;2 iii the calculator for use in<br />

the next calculation. If2.83 is used, the answer<br />

will vary slightly due to round-off error. Find<br />

angle A next, since it is the smaller angle and<br />

must be acute.<br />

. A asinC 4.21sin28.3° -<br />

SIn = -c-= ...[;2<br />

sin A = .70581857<br />

A =44.9°<br />

B = 180° - 44.9° - 28.3°<br />

B= 106.8°<br />

8. A = 41.4°, b =2.78 yd, c =3.92 yd<br />

a 2 =b 2 + c 2 - 2bccosA<br />

= (2.78)2 + (3.92)2 - 2(2.78)(3.92)(cos 4 1.4°)<br />

= 6.7459792 •<br />

a = 2.60 yd<br />

Keep all the digits of,J;;2 in your calculator for<br />

use in the next calculation. If2.60 is used, the<br />

answer will vary slightly due to round-off error.<br />

Find angle B next, since it is the smaller angle and<br />

must be acute.<br />

283


<strong>Trigonometry</strong> <strong>Solutions</strong> <strong>Manual</strong><br />

284 Chapter 7 Applications of <strong>Trigonometry</strong> and Vectors<br />

sinB sin41.4°<br />

2.78 = J:i<br />

B=45.1°<br />

C = 180° - 41.4° -45.1°<br />

C=93.5°<br />

9. C =45.6°, b =8.94 rn, a =7.23 m<br />

c 2 =a 2 +b 2 -2abcosC<br />

c = ~(7.23)2 + (8.94)2 - 2(7.23)(8.94) cos 45.6°<br />

c=6.46 m<br />

Find angle A next, since it is the smaller angle and<br />

must be acute.<br />

. asinC 7.23sin45.6°<br />

smA=--= G<br />

c ve 2<br />

sin A =.79946437<br />

A=53.1°<br />

B =180° -53.1°-45.6°<br />

B =81.3°<br />

10. A = 67.3°, b = 37.9 km, e = 40.8 krn<br />

2 = b<br />

2 2<br />

a + c - 2bc cos A<br />

= (37.9)2 + (40.8)2 - 2(37.9)(4O.8)(cos67.3°)<br />

= 1907.581537<br />

a =43.7 km<br />

Find angle B next, since it is the smaller angle and<br />

must be acute .<br />

sin B sin 67.3°<br />

37.9 = ..r;;z<br />

B=53.2°<br />

C= 180°-67.3°-53.2°<br />

C=59.5°<br />

11. A = soo 40' b = 143 cm, c = 89.6 em<br />

a 2 = b 2 + e 2 - 2bc cos A<br />

a = ~1432 + (89.6)2 - 2(143)(89.6) cos 80° 40'<br />

a= 156 em<br />

Find angle C next, since it is the smaller angle<br />

and must be acute.<br />

csinA 89.6sin80040'<br />

sin C = --= r=:;­<br />

a va 2<br />

sin C = .56692713<br />

C=34°30'<br />

B = 1800- 80° 40' - 34°30'<br />

B=64°50'<br />

12. C = 72° 40', a = 327 ft, b = 251 ft<br />

c 2 =a 2 +b 2 -2abcosC<br />

= 327 2 +251 2 -2(327)(251)cos72°40'<br />

c =348 ft<br />

Find angle B next, since it is the smaller angle and<br />

must be acute.<br />

sin B<br />

sin 72° 40'<br />

---<br />

251 .J<br />

B=43°30'<br />

A =180 0-B-C<br />

A = 63°50'<br />

13. B = 74.80°, a = 8.919 in., e = 6.427 in.<br />

b 2 =a 2 +c 2 -2accosB<br />

b = ~(8.919)2 + (6.427)2 - 2(8.919)(6.427) cos 74.80°<br />

b=9.529 in.<br />

Find angle C next, since it is the smaller angle<br />

and must be acute.<br />

Jbi<br />

c sin B 6.427 sin 74.80°<br />

sinC=-b-=<br />

sin C = .65089219<br />

C=4O.61°<br />

A = 180°-74.80° -40.61°<br />

A =64.59°<br />

14. C = 59.70°, a = 3.725 mi, b = 4.698 mi<br />

c 2 =a 2 +b 2 -2abcosC<br />

= (3.725)2 +(4.698)2 - 2(3.725)(4.698) cos 59.70°<br />

c=4.276 mi<br />

Find angle A next, since it is the smaller angle and<br />

must be acute.<br />

sin A sin 59.70°<br />

3.725 = V<br />

A =48.77°<br />

B = 180° -48.77° -59.70°<br />

= 71.53°<br />

IS. A = 112.8°, b = 6.28 m, c = 12.2 m<br />

a 2 =b 2 +c 2 -2bccosA<br />

a = ~(6.28)2 + (12.2)2 - 2(6.28)(12.2)cos 112.8°<br />

a = 15.7 m<br />

Angle A is obtuse, so both Band C are acute. Find<br />

either angle next.<br />

b sin A 6.28 sin 112.8°<br />

sinB=--= r<br />

a va 2<br />

sin B = .36787456<br />

B=21.6°<br />

C = 180° -112.8° - 21.6°<br />

C=45.6°<br />

284


<strong>Trigonometry</strong> <strong>Solutions</strong> <strong>Manual</strong><br />

7.3 The Law of Cosines 285<br />

16. 8 =168.2°, a = 15.1 em, C =19.2 em<br />

b 2 = a 2 + c 2 - 2ac cos 8<br />

= 15.1 2 + 19.2 2 - 2(l5.1)(l9.2)cos 168.2°<br />

b=34.1 em<br />

Find angle A next, since it is the smaller angle and<br />

must be acute.<br />

sin A sin 168.2°<br />

--=<br />

15.1 fbi<br />

A =5.2°<br />

C =180° - 5.2° -168.2°<br />

=6.6°<br />

17. a =3.0 ft, b =5.0 ft, C = 6.0 ft<br />

Angle C is the largest, so find it first.<br />

c 2 =a 2 +b 2 -2abcosC<br />

3.0<br />

222<br />

+5.0 -6.0<br />

cos C=-----­<br />

2(3.0)(5.0)<br />

cos C = -.06666667<br />

C=94°<br />

. 8 bsinC<br />

SIO =-­ c<br />

sin8 = .83147942<br />

8=56°<br />

A = 180° - 56° - 94°<br />

A=300<br />

18. a = 4.0 ft, b = 5.0 ft, c = 8.0 ft<br />

Find the largest angle first, in case it is obtuse.<br />

Since c is the longest side, angle C will be the<br />

largest angle .<br />

2 2 + b<br />

2<br />

c = a - 2ab cos C<br />

a 2+b2_c2<br />

cosC=---­<br />

2ab<br />

= 16+25-64 =-.575<br />

40<br />

C= 125°<br />

sin 8 sin 125°<br />

--=<br />

5.0 8.0<br />

8=31°<br />

A = 180° -31°-125°<br />

A=24°<br />

19. a =9.3 em, b = 5.7 ern, c =8.2 em<br />

Angle A is the largest, so find it first.<br />

a 2 = b 2 + c 2 - 2bc cos A<br />

5.7<br />

222<br />

+8.2 -9.3<br />

cos A =-----­<br />

2(5.7)(8.2)<br />

cosA = .14163457<br />

A =82°<br />

. 8 bsinA<br />

SIO =-­<br />

a<br />

sin B = .60672455<br />

B=37°<br />

C= 180°-82°-37°<br />

C=61°<br />

20. a = 28 ft, b = 47 ft, c =58 ft<br />

Angle C is the largest, so find it first.<br />

28 2 + 47 2 - 58 2<br />

cos C = -----­<br />

2(28)(47)<br />

= -.14095745<br />

C=98°<br />

sinA sinC<br />

--=-­<br />

28 58<br />

A =29°<br />

B = 180° -29° -98°<br />

=53°<br />

21. a = 42.9 m, b = 37.6 m, c = 62.7 m<br />

Angle C is the largest, so find it first.<br />

c 2 = a 2 + b 2 - 2ab cos C<br />

42.9<br />

222<br />

+37.6 -62.7<br />

cos C =<br />

2(42.9)(37.6)<br />

cos C = -.20988940<br />

C= 102.1°<br />

. B bsinC<br />

SID =-­ c<br />

sin 8 = .58632321<br />

B=35.9°<br />

A = 180° - 35.9° -102.1°<br />

A =42.0°<br />

22. a =187 yd, b = 214 yd, c = 325 yd<br />

Angle C is the largest, so find it first.<br />

187 2 +214 2 - 325 2<br />

cosC=<br />

2(187)(214)<br />

= - .3106 1022<br />

C = 108.1°<br />

sinB sinC<br />

--=-­<br />

214 325<br />

B=38.7°<br />

A = 180° - 38.7° -108.1°<br />

= 33.2°<br />

23. A8 = 1240 ft, AC = 876 ft, 8C = 918 ft<br />

AB=c,AC=b,BC=a<br />

Angle C is the largest, so find it first.<br />

c 2 =a 2 +b 2 -2abcosC<br />

a 2 +b 2 _c 2<br />

cosC=---­<br />

2ab<br />

cosC = .04507765<br />

C= 87.4°<br />

285


<strong>Trigonometry</strong> <strong>Solutions</strong> <strong>Manual</strong><br />

286 Chapter 7 Apr. ieatlons of <strong>Trigonometry</strong> and Vectors<br />

. B bsin C<br />

SID =-­<br />

c<br />

sin B = .70573350<br />

B=44.9°<br />

A = 180° - 44.9° - 87.4°<br />

A = 47.7°<br />

24. AB=298 m,AC= 421 m, BC= 324 m<br />

Since AC is the longest side and is opposite<br />

angle B, angle B will be the largest angle. Find<br />

angle B first.<br />

324 2 +298 2 -421 2<br />

cos B= ------­<br />

2(324)(298)<br />

cos B = .08564815<br />

B=85.1°<br />

sinA<br />

sinB<br />

--=-­<br />

324 421<br />

A = 50.1°<br />

C= 180 0-A<br />

- B = 44.8°<br />

27. Findx.<br />

x 2 = 25 2 + 25 2 - 2(25)(25)cos52°<br />

=480<br />

x = 22 ft<br />

28. Let B be the harbor, AB is the course of one ship,<br />

BC is the course of the other ship.<br />

Then b = the distance between the ships.<br />

A<br />

B 402mi C<br />

b 2 = 402 2 + 402 2 - 2(402)(402)cos 135° 40'<br />

b=745 mi<br />

29. Find AC, or b, in the following triangle.<br />

N<br />

t<br />

25. Find AB, or c, in the following triangle.<br />

A<br />

B<br />

c 2 =a 2 +b 2 -2abcosC<br />

c = ~2862 + 350 2 - 2(286)(350) cos 46 .3°<br />

c=257<br />

AB=257m<br />

26. Find the diagonals, BD and AC, of the following<br />

parallelogram.<br />

B<br />

C<br />

Icm/ 122 . /<br />

4D;L<br />

c<br />

A 6.0cm D<br />

BD 2 = AB 2 + AD 2 - 2(AB)(AD)cos A<br />

= 16+36-48(.52991926)<br />

= 26.5638755<br />

BD=5.2cm<br />

AC 2 = AB 2 + BC 2 - 2(AB)(BC) cos B<br />

= 16 + 36 - 48(-.52991926)<br />

= 77.4361245<br />

AC=8.8 em<br />

The lengths of the diagonals are 5.2 em and<br />

8.8 em.<br />

A 450km B<br />

Angle 1 =180°-128°40'=51°20'<br />

Angle 1 = Angle 2<br />

Angle B = 90° - Angle 2 = 38°40'<br />

b 2 =a 2 +c 2 -2ac~osB<br />

b = ~3592 + 450 2 - 2(359)(450)cos38° 40'<br />

b=281 km<br />

30. Let d = the length of the rope from the top of the<br />

tower to the bottom of the hill . The situation is<br />

illustrated in the fOllOrng fig",e.<br />

459.0 ft<br />

1<br />

12.47·<br />

a = 35.98° -12.47°<br />

=23.51°<br />

0= 90° -12.47° = 77.53°<br />

f3 = 180°-77.53°<br />

= 102.47°<br />

d 459<br />

----=--­<br />

sin 102.47° sin 23.51 °<br />

d =1123 ft<br />

286


<strong>Trigonometry</strong> <strong>Solutions</strong> <strong>Manual</strong><br />

7.3 The Law of Cosines 287<br />

31. AB 2 = 10 2 + 10 2 - 2(10)(10)cos 128°<br />

AB 2 =323<br />

AB= 18 ft<br />

32. Let A =the angle between the beam and the 45-ft<br />

cable.<br />

45 2 + 90 2 - 60 2<br />

cos A =-----­<br />

2(45)(90)<br />

A=36°<br />

Let B =the angle between the beam and the<br />

60-ft cable.<br />

90 2 + 60 2 - 45 2<br />

cos B =-----­<br />

2(90)(60)<br />

B=26°<br />

33. Let x = the distance from the tracking station to<br />

the satellite at 12:03 P.M.<br />

Notice that the distance from the center of the<br />

earth to the satellite is 6400 + 1600 = 8000 km.<br />

Let 9 =the angle made by the movement of the<br />

satellite from noon to 12:03 P.M.<br />

21t radians 9 radians<br />

=<br />

2 hours 3 minutes<br />

21t 9<br />

--=­<br />

2(60) 3<br />

9 = 61t = ~ radian<br />

120 20<br />

x 2 = 6400 2 + 8000 2 - 2(64OO)(8000)cos~<br />

20<br />

x=2000<br />

The distance between the satellite and the<br />

tracking station is 2000 km.<br />

34. Using the law of cosines,<br />

c 2 = (14.0)2 + (13.0)2 - 2(l4.0)(l3.0)cos 70°<br />

"" 240.5047<br />

c "" 15.5 ft<br />

The length of the property line is approximately<br />

18.0 + 15.5 + 14.0 = 47.5 feet<br />

35. Let A = the man's location;<br />

B = the factory whistle heard at 3 sec after 5:00;<br />

C = the factory whistle heard at 6 sec after 5:00.<br />

B<br />

c~a<br />

~<br />

Abe<br />

Since sound travels at 344 m per sec, and the man<br />

hears the whistles in 3 sec and 6 sec, the factories<br />

are<br />

c = 3(344) = 1032 m<br />

and b = 6(344) = 2064 m<br />

from the man.<br />

a 2 = 1032 2 + 2064 2 - 2(1032)(2064) cos 42.2°<br />

a = 1470<br />

The factories are 1470 m apart.<br />

36. Let x =the distance from the plane to the<br />

mountain when the second bearing is taken.<br />

course of<br />

plane<br />

7.92km<br />

9 = 180° - 32.7° = 147.3°<br />

Using the law of sines gives<br />

7.92 x<br />

=<br />

sin 147.3° sin 24.1 °<br />

5.99= x.<br />

The plane is 5.99 km from the mountain.<br />

37. Let a be the length of the segment from (0, 0) to<br />

(6,8). Use the distance formula.<br />

a = ~(6 - 0)2 + (8 - 0)2<br />

= ~62 +8 2<br />

= ,.J36+64<br />

=,.J100 = 10<br />

Let b be the length of the segment from (0, 0) to<br />

(4,3).<br />

b = ~r-(4---0-)2-+-(-3_-0-)-2<br />

= ~42 +3 2<br />

=,.J16+9<br />

=.../25 =5<br />

Let c be the length of the segment from (4, 3) to<br />

(6,8). ,------­<br />

c=~(6-4)2 +(8-3)2<br />

= ~22 +5 2<br />

=,.J4+25 =..fi9<br />

a 2 +b 2 _c 2<br />

cos 9 = .:....-....:.....::...----=-­<br />

2ab<br />

10 2 +5 2 _(..fi9)2<br />

cos 9 =..:...-......:....=--~.=.=~<br />

2(10)(5)<br />

100+25 -29<br />

cos 9 = ------=..:....<br />

100<br />

cos9= .96<br />

9"" 16.26°<br />

38. Let a be the length of the segment from (0, 0) to<br />

(8, 6). Use the distance formula.<br />

a = ~(8-0)2 +(6-0)2<br />

= ~82 +6 2<br />

= ,.J64+ 36<br />

= ,.Jloo = 10<br />

287


<strong>Trigonometry</strong> <strong>Solutions</strong> <strong>Manual</strong><br />

288 Chapter 7 Applications of <strong>Trigonometry</strong> and Vectors<br />

....<br />

Let b be the length of the segment from (0, 0) to<br />

(12,5).<br />

b = ~r-(1-2_-0-)-2+-(-5---0)-2<br />

=~122+52<br />

=.J144+25<br />

= .J169 =13<br />

Let c be the length of the segment from (8, 6) to<br />

(12,5).<br />

c = ~'-(l-2_-8-)-=-2+-(-5---6)-=-2<br />

=~42 +(_1)2<br />

=.Jfi<br />

a 2 +b 2 _c 2<br />

cos(}=---­<br />

2ab<br />

10 2 +13 2 _(.Jfi)2<br />

cos() = -----­<br />

2(10)(13)<br />

e 100+169-17<br />

cos =----­<br />

260<br />

cos(} = .9692308<br />

()= 14.25°<br />

39. Using K = .!.bh,<br />

2<br />

K =.!.(16)(3.J3)<br />

2<br />

= 24.J3 = 41.57.<br />

To use Heron's Formula, first find the<br />

semi perimeter.<br />

1<br />

s='2(a+b+c)<br />

1<br />

=-(6+14+16)<br />

2<br />

1<br />

='2(36) =18<br />

Now find the area of the triangle.<br />

K =..js(s ­ a)(s ­ b)(s ­ c)<br />

= ..j18(18­ 6)(18 -14)(18 -16)<br />

= ..j18(12)(4)(2)<br />

= .J1728 '" 41.57<br />

Both formulas give the same area.<br />

40. Using K = .!.bh,<br />

2<br />

K = .!.(10)(M)<br />

2<br />

= 15.J3<br />

=25.98.<br />

To use Heron's Formula, first find the<br />

semiperimeter.<br />

1<br />

s=-(a+b+c)<br />

2<br />

1<br />

=-(10+6+14)<br />

2<br />

=.!. (30)<br />

2<br />

=15<br />

Now find the area of the triangle.<br />

K =..js(s - a)(s - b)(s - c)<br />

= ..j15(15-10)(15 - 6)(15 -14)<br />

= ..j15(5)(9)(1)<br />

= .J675<br />

= 25.98<br />

Both formulas give the same result.<br />

41. a = 12 m, b = 16 m, c = 25 m<br />

1<br />

s = -(12 + 16+ 25)<br />

2<br />

s=26.5<br />

area =..js(s-a)(s-b)(s-c)<br />

=..j(26.5)(14.5)(10.5)(1.5)<br />

=78m 2<br />

42. a = 22 in., b = 45 in., c = 31 in.<br />

1<br />

s=-(a+b+c)<br />

2<br />

1<br />

= -(22 +45+31)<br />

2<br />

=49<br />

The area is<br />

K = .Jr-s(-s---a-)(-s---b)-(s---c-)<br />

= ..j49(49 - 22)(49 - 45)(49 - 31)<br />

= ..j49(27)(4)(18)<br />

= 310 in. 2<br />

43. a = 154 em, b = 179 em, c = 183 em<br />

1<br />

s=-(154+179+183)<br />

2<br />

s=258<br />

area =..js(s-a)(s-b)(s-c)<br />

=..j(258)(104)(79)(75)<br />

=12,600 cm 2<br />

288


<strong>Trigonometry</strong> <strong>Solutions</strong> <strong>Manual</strong><br />

7.3 The Law of Cosines 289<br />

44. a = 25.4 yd, b = 38.2 yd, c = 19.8 yd 50. Find the area of the Bermuda Triangle.<br />

1<br />

s=-(a+b+c) s = .!. (850 + 925 + 1300)<br />

2 2<br />

= 41.7 s = 1537.5<br />

The area is<br />

area = ..jr-15-3-7.-5(-6-87-.5-)-(6-12-.5-)-(23-7-.5-)<br />

K = ..jr-s(-s---a-)(-s--b)-(s---c-) = 392,000<br />

= ~41.7(16 .3)(3.5)(21.9)<br />

The area of the Bermuda Triangle is 392,000 mi 2 .<br />

= 228 yd 2 .<br />

51. Perimeter: 9 + 10 + 17 = 36 feet, so the semi­<br />

45. a = 76.3 ft, b = 109 ft, c = 98.8 ft perimeter is .!..36 = 18 feet.<br />

1<br />

s = -(76.3 + 109 + 98.8)<br />

2<br />

s = 142.05<br />

area =..jr s(":'"""s---a":'""")("-s-_7 b)"-(s---c--:-)<br />

2<br />

Use Heron's Formula to find the area.<br />

Area: =~s(s-a)(s-b)(s-c)<br />

= ~(142 .05)(65 .75)(33.05)(43 .25) = ~18(9)(8)(1)<br />

= ~18(18 - 9)(18 -10)(18-17)<br />

= 3650 ft2 = .J1296 =36 ft<br />

Since the perimeter and area both equal 36 feet.<br />

46. a = 15.89 in., b = 21.74 in., c = 10.92 in. the triangle is a perfect triangle.<br />

1<br />

s=-(a+b+c)<br />

2 1<br />

52. (a) s=-(a+b+c)<br />

=24.275 2<br />

The area is<br />

1<br />

K = ..jr-"s(-s-_-a-)(s-_---,-,.b)-(s-_-c 7)<br />

=-(11+13+20)<br />

2<br />

1<br />

= 83.01 in. 2 =-(44) =22<br />

2<br />

47. AB = 22.47928 mi, AC = 28.14276 mi, Area =.Js(s-a)(s-b)(s-c)<br />

A = 58.56989°<br />

= .J22(22 - 11)(22- 13)(22 - 20)<br />

BC2 = AC2 + AB2 - 2(AC)(AB) cos A<br />

= ~22(11)(9)(2)"<br />

Use a calculator and substitute.<br />

BC 2 = 637.5539346<br />

BC = 25.24983 mi<br />

= .J4356<br />

= 66 which is an integer<br />

48. AB = 22.47928 mi, BC = 25.24983 mi, 1<br />

(b) s=-(a+b+c)<br />

A = 58.56989°<br />

2<br />

sinC sin A 1<br />

= -(13+ 14+ 15)<br />

22.47928 25.24983 2<br />

sin C = .75965065<br />

1<br />

C= 49.4334°<br />

="2(42) =21<br />

B= 180 0 - A - C<br />

~-----<br />

Area =~s(s-a)(s-b)(s-c)<br />

= 71.99670°<br />

= ~21(21-13)(21-14)(21-15)<br />

49. Find the area of the region. = ~21(8)(7)(6)<br />

1<br />

s = -(75+68+ 85)<br />

= .J7056<br />

2<br />

= 84 which is an integer<br />

s=114<br />

~-----<br />

area = ~(114)(39)(46)(29)<br />

1<br />

(c) s=-(a+b+c)<br />

=2435.3571 m 2 2<br />

(area in m 2 )<br />

1<br />

Number of cans needed = "':""""2::---"";" =- (7 + 15 + 20)<br />

(m percan)<br />

2<br />

1<br />

2435.3571 = 32.471428 cans<br />

= -(42) = 21<br />

2<br />

75<br />

She will need to open 33 cans.<br />

289


<strong>Trigonometry</strong> <strong>Solutions</strong> <strong>Manual</strong><br />

290 Chapter 7 Applications of <strong>Trigonometry</strong> and Vectors<br />

Area ="'/s(s-a)(s-b)(s-c)<br />

a 2 =b 2 +c 2 -2bccosA<br />

= "'/21(21-7)(21-15)(21- 20) 2bccosA = b 2 +c 2 _a 2<br />

= .../21(14)(6)(1) b 2 +c 2 _a 2<br />

cosA=---­<br />

=.../1764 2bc<br />

= 42 which is an integer (4)2 +(5)2 _(6)2<br />

=<br />

2(4)(5)<br />

53. Applying the law of cosines when a = 3. b = 4.<br />

and c = 10 gives<br />

c 2 =a 2 +b 2 -2abcosC<br />

10 2 =3 2 +4 2 -2(3)(4)cosC<br />

75 = -24cosC<br />

-3.125 = cos C.<br />

Since -1 ~ cos C ~ 1. a triangle cannot have sides<br />

3,4, and 10.<br />

=<br />

16+25 -36<br />

40<br />

1<br />

=­<br />

8<br />

Does cos A = 2 cos 2 B-1<br />

1 (3)2<br />

'8.1 2 '4 -1<br />

.1 2(~)-~<br />

55. (a) Using the law of sines<br />

16 16<br />

sinA sinC<br />

2 1<br />

--=-­<br />

~-=a<br />

c - 16 8<br />

sin C = E- sin A<br />

a<br />

=~sin60°<br />

13<br />

Since the identity holds, A is twice the size of B.<br />

Recall that cos 2A = 2 cos 2 A-I.<br />

57. Using the law of cosines,<br />

=0.99926<br />

a 2 =b 2 +c 2 -2bccosA<br />

c = 87.8° or 92.2° c 2 - 2bc cos A + (b 2 _a 2 ) = 0<br />

...<br />

(b) By the law of cosines, c 2 - 2(24.9)(cos55.3°) c + (24.9 2 - 22.8 2) = 0<br />

c 2 = a 2 + b 2 - 2ab cos C c 2 - 28.35c + 100.17 = 0<br />

2abcosC= a 2 + b 2 _c 2<br />

Apply the quadratic formula<br />

a 2 +b 2 _c 2<br />

cosC=----­<br />

2ab<br />

28.35 ± ~(-28.35)2 - 4(1)(100.17)<br />

c =---"----'---.;..;...--­<br />

2(1)<br />

(13)2 + (7)2 _ (15)2<br />

28.35 ± .../803.72 - 400.68<br />

=<br />

=<br />

2(13)(7)<br />

2<br />

=-.03846<br />

28.35 ±20.08<br />

=<br />

C=92.2° 2<br />

28.35 + 20.08<br />

(c) With the law of cosines, we are required to ci = 2<br />

find the inverse cosine of a negative number<br />

=24.2 ft<br />

therefore, we know angle C is greater than<br />

28.35 - 20.08<br />

90 0 •<br />

c2 = 2<br />

56. Using the law of cosines, = 4.14 ft<br />

b 2 = a 2 + c 2 - 2ac cos B<br />

2accosB=a 2 +c 2 _ b 2<br />

a 2 +c 2 _b 2<br />

cosB=---­<br />

2ac<br />

(6)2 + (5)2 _ (4)2<br />

=<br />

2(6)(5)<br />

36+25-16<br />

58. Prove that if a and b are equal sides of an<br />

isosceles triangle. then c 2 = 2a 2 (l ­ cos C).<br />

By the law of cosines,<br />

c 2 =a 2 + b 2 - 2abcos C.<br />

Since a = b,<br />

c 2 = a 2 +a 2 -2a 2 cosC<br />

c 2 = 2a 2 - 2a 2 cos C<br />

=<br />

60 c 2 = 2a 2 (1- cos C).<br />

3<br />

=­ 4<br />

290


<strong>Trigonometry</strong> <strong>Solutions</strong> <strong>Manual</strong><br />

7.4 Vectors 291<br />

59. Given that point D is on side AB of triangle ABC,<br />

such that CD bisects angle C; thus,<br />

angle ACD =angle DCB.<br />

AD b<br />

Show that - =-.<br />

DB a<br />

A<br />

63. b 2 + c 2 > b 2 and b 2 + c 2 > c 2 . If a 2 > b 2 + c 2 ,<br />

then a 2 > b 2 from which a > b and a > c<br />

because a, b, and c are all nonnegative.<br />

64. Because A is obtuse it is the largest angle, so the<br />

longest side should be a, not c.<br />

Section 7.4<br />

c<br />

B<br />

Let 9 =the measure of angle ACD;<br />

a = the measure of angle ADC.<br />

Then 9 =the measure of angle DCB and angle<br />

BDC =180 - a.<br />

By the laws of sines,<br />

sin9 sina<br />

--=-­<br />

AD b<br />

. 9 ADsina<br />

SIO =--­<br />

b<br />

and<br />

sin9<br />

--=<br />

DB<br />

sin 9<br />

ADsina<br />

b<br />

=<br />

sin(1800-a)<br />

a<br />

DBsin(l80° - a)<br />

a<br />

DBsin(1800-a)<br />

a<br />

Since sin ex. =sin (1800 _ u), AD = DB .<br />

b a<br />

Mu tip I' b th sid b b AD b<br />

DB DB a<br />

I . ymg 0 SI es y -,we get - =- .<br />

60. Let a = 2, b =2-/3 ,A = 30°, B = 60°.<br />

. tan!(A-B) a-b<br />

Venfy -­<br />

tan!(A+B) a+b<br />

tan!(A-B) _ tan!(300-600)<br />

tan t (A + B) - tan! (30° + 60°)<br />

tan(-15°)<br />

= tan 45°<br />

=-.26794919<br />

a - b = 2 - 2-/3 = -1.46 =- .26794919<br />

a+b 2 +2-/3 5.46<br />

(Note: - .26794919 =-2 + -/3.)<br />

61. Since A is obtuse, 90° < A < 180°. The cosine of a<br />

quadrant II angle is negative.<br />

Exercises<br />

3. Equal vectors have the same magnitude and<br />

direction: m = p, n =r.<br />

4. Opposite vectors have the same magnitude but<br />

opposite direction . Opposite vectors are m and q,<br />

p and q, nand s, rand s.<br />

S. One vector is a positive scalar multiple of another<br />

if the two vectors point in the same direction; they<br />

may have different magnitudes.<br />

m = Ip, m = 2t, n = lr, p = 2t or<br />

1 1<br />

P =1m t =- m, r =In t =- P<br />

'2 ' 2<br />

6. One vector is a negative scalar multiple of another<br />

if the two vectors point in the opposite direction;<br />

they may have different magnitudes.<br />

m = -lq, p =-lq, r= -Is, n = -Ls, q =-2t<br />

7. ­<br />

-b<br />

8.<br />

9.<br />

10.<br />

2<br />

62. In a =b 2 + c 2 - 2bccos A, cos A is negative, so<br />

2<br />

a = b 2 + c 2 plus a positive quantity.<br />

2<br />

Thus, a 2 > b 2 +c .<br />

291


<strong>Trigonometry</strong> <strong>Solutions</strong> <strong>Manual</strong><br />

292 Chapter 7 Applications of <strong>Trigonometry</strong> and Vectors<br />

11. 19.<br />

12. 20.<br />

13. , ,,,,,,<br />

21.<br />

,<br />

14.<br />

22.<br />

15.<br />

23.<br />

16. - - - ; ,<br />

~<br />

" ,<br />

d '0 ,<br />

24.<br />

b<br />

17.<br />

18.<br />

25.<br />

26.<br />

27.<br />

28.<br />

a + (b + c) = (a + b) + c<br />

Yes, vector addition is associative.<br />

c-s d =d+ c<br />

Yes, vector addition is commutative.<br />

lui = 12, Iwi = 20, 8 = 27°<br />

~<br />

2 ~--=- --=a.<br />

20 ----~.-<br />

lui =8, !wI =12, 8 = 20°<br />

-<br />

2~ ~----<br />

-~)<br />

12 ...--­<br />

292


<strong>Trigonometry</strong> <strong>Solutions</strong> <strong>Manual</strong><br />

7.4 Vectors 293<br />

29. lui = 20, IwI = 30, 8 = 30° 34. a =70°, /vi = 150<br />

~<br />

- - -<br />

~-:3)<br />

30 -­<br />

30. lui = 27, Iwl= 50, 8 = 12°<br />

27 #/_ J2" =<br />

50 ~ __ SZ)<br />

y<br />

31. lui = 50, Iwl = 70, 8 = 40° x<br />

50<br />

........ Ix! = 150cos70° = 51<br />

.­ .­<br />

IYl = 150sin 70° =140<br />

.- .­<br />

In Exercises 32-39, x is the horizontal component of v, 35. a =50°, Ivl = 26<br />

and y is the vertical component of v.<br />

32. a = 20°, Iv! = 50<br />

~<br />

e::4CJY<br />

x<br />

cos200=M<br />

cos 500 = IxI<br />

50 26<br />

Ixl = 50 cos 20° = 47 IxI =26 cos 50°<br />

=17<br />

sin200=M<br />

50 sin500=M<br />

~I = 50sin20° = 17 26<br />

IYl = 26sin 50°<br />

33. a = 38°, Ivl = 12 =20<br />

y<br />

x<br />

36. a =35°50', Iv! =47.8<br />

y<br />

x<br />

cos38o=M<br />

12<br />

Ixl =12cos 38°<br />

Ixl =47.8cos 35° 50' =38.8<br />

=9.5<br />

111 =47.8sin35°50' ~ 28.0<br />

sin38°= M<br />

12<br />

~I= 12sin38°<br />

=7.4<br />

x<br />

y<br />

293


<strong>Trigonometry</strong> <strong>Solutions</strong> <strong>Manual</strong><br />

294 Chapter 7 Applications of <strong>Trigonometry</strong> and Vectors<br />

37. a = 27°30', Ivl = 15.4 41. Forces of 250 newtons and 450 newtons, forming<br />

an angle of 85°<br />

250<br />

y<br />

38.<br />

x<br />

cos 27° 30' = ~<br />

15.4<br />

Ixl = 15.4cos27°30'<br />

=13.7<br />

sin 27° 30' = J!L<br />

15.4<br />

IYI = 15.4 sin 27° 30'<br />

=7.11<br />

a = 128.5°, Iv! = 198<br />

250<br />

a = 180°-85° = 95°<br />

1v1 2 = (250)2 +(450i ­<br />

= 284,610<br />

Ivi = 530 newtons<br />

2(250)(450) cos 95°<br />

y<br />

42.<br />

Forces of 19 newtons and 32 newtons, forming an<br />

angle of 118°<br />

180° - 118° = 62°<br />

--------­<br />

62·<br />

\<br />

\<br />

\<br />

\<br />

\<br />

x<br />

8 = 180°-128.5° = 51.5°<br />

cos51.5°=~<br />

198<br />

Ixi = 198cos51.5°<br />

=123<br />

sin 51.5° = J!L<br />

198<br />

IYI = 198sin 51.5°<br />

=155<br />

43.<br />

32newtons<br />

Ivl 2 = 19 2 + 32 2 - 2(19)(32)cos 62°<br />

= 814.1226<br />

Ivi = 29 newtons<br />

Forces of 17.9 Ib and 25.8 Ib, forming an angle of<br />

105.5°<br />

17.9<br />

\<br />

\<br />

\<br />

\<br />

\<br />

\<br />

\<br />

\<br />

\<br />

\<br />

\<br />

39. a = 146.3°,1v1 = 238<br />

y<br />

x<br />

8 = 180°-146.3° = 33.7°<br />

Ixl =238cos33.7° = 198<br />

IYI =238sin 33.7° =132<br />

17.9<br />

a =180° -105.5°<br />

=74.5°<br />

Ivl 2 =(25.8)2 + (17.9)2 - 2(25.8)(17.9)cos 74.5°<br />

= 739.218<br />

IvI= 27.2 Ib<br />

294


<strong>Trigonometry</strong> <strong>Solutions</strong> <strong>Manual</strong><br />

7,4 Vectors 295<br />

44, Forces of 75.6 Ib and 98.2 Ib forming an angle of<br />

82°50'<br />

180°- 82°50' = 97° 10'<br />

75.61b<br />

97'10'<br />

y<br />

----fL-=--~ x<br />

98.21b<br />

Ivf = (75.6)2 + (98.2)2 - 2(75.6)(98.2) cos 97°10'<br />

= 17.210.957<br />

Iv! = 1311b<br />

45, Forces of 116 Ib and 139 lb. forming an angle of<br />

140°50'<br />

, ,,,,<br />

48. Magnitude: ~(-4)2 + (4.J3) 2 =.J64 = 8<br />

Angle: (J = arctan( 4':;)<br />

= arctan(-.J3) (in quadrant II since<br />

x 0)<br />

= 120 0<br />

y<br />

116 , ,,,<br />

140'50' "<br />

_-,"-'---+-...J..-__ X<br />

-4<br />

49. Magnitude: ~(8..J2)2 +(-8..J2) 2 = ~256 = 16<br />

. (-8..J2)<br />

Angle: (J = arctan s..J2<br />

a = 180°-140°50' = 39° 10'<br />

1v!2 = (139)2 + (116)2 - 2(139)(116)cos 39°10'<br />

=7774.74<br />

!vi = 88.2 Ib<br />

= arctan(-l) (in quadrant IV<br />

since x > 0 and y < 0)<br />

= 315°<br />

y<br />

46, Forces of 37.8 lb and 53.7 lb, forming an angle of<br />

68.5°<br />

180°- 68.5° = 111.5°<br />

---------<br />

111.5'<br />

53.71b<br />

1v1 2 = (37.8)2 + (53.7)2 - 2(37.8)(53.7) cos 111.5°<br />

= 5800.422<br />

!vi = 76.2 Ib<br />

47, Magnitude: ~1 2 + 1 2 =..J2<br />

50. Magnitude: ~(.J3)2 + (_1)2 =.J4 = 2<br />

Angle: (J = arctan( ':A)<br />

= arctan(- .J3) (in quadrant IV since<br />

3 x > 0 and y < 0)<br />

=330°<br />

Angle: (J = arc tan(!) (in quadrant I since<br />

1 x > 0 and y > 0)<br />

=45°<br />

295


<strong>Trigonometry</strong> <strong>Solutions</strong> <strong>Manual</strong><br />

296 Chapter 7 Applications of <strong>Trigonometry</strong> and Vectors<br />

y<br />

54. Magnitude: ~02 +(-12)2 =..J144 =12<br />

Angle: e=arctan( - ~2)<br />

(Division by zero makes this undefined so the<br />

--+--k-...l...-'--.,_ .r<br />

-I<br />

only possible values are (} = 90° or (} = 270° .)<br />

Since y < 0, (} = 270°,<br />

y<br />

51. Magnitude: ~(l5)2 + (_8)2 = ·./289 = 17 270'<br />

---+--+---'--_ x<br />

Angle: e = arctan (-8)- (i m qua d rant IV smce '<br />

15 x>Oandy


<strong>Trigonometry</strong> <strong>Solutions</strong> <strong>Manual</strong><br />

7.4 Vectors 297<br />

57. y 60. y<br />

--I--4.-L,,-,-­ x<br />

7890<br />

a = Ivlcos 8 b = Ivlsin 8<br />

=198cos 128°30' = 198sin128°30'<br />

= 198cosI28.5° = 198sin128.5°<br />

== -123.258 == 154.956<br />

v = (-123.258.154.956)<br />

58. y<br />

a = Ivlcos8 b = Ivlsin 8<br />

= 7890cos302°40' = 7890sin 302°40'<br />

== 7890cos 302.67° == 7890sin 302.67°<br />

== 4258.633 == -6641.999<br />

v = (4258.633. -6641.999)<br />

61. u + v = (-2.5)+(4.3)<br />

=(-2+4.5+3)<br />

=(2.8)<br />

62. u - v = (-2.5)-(4,3)<br />

=(-2-4.5-3)<br />

= (-6.2)<br />

59.<br />

a = /vi cos 8 b = /vi sin8<br />

= 238cos 146°10' = 238sin 146°10'<br />

= 238cos 146.167° =238sin146.167°<br />

== -197.697 == 132.513<br />

v = (-197.697.132.513)<br />

y<br />

63.<br />

64.<br />

v -u =(4.3) -(-2.5)<br />

=(4-(-2).3-5)<br />

=(6. -2)<br />

5v =5{4, 3)<br />

= (5 ·4, 5 '3)<br />

=(20,15)<br />

--rl'--T---'--­ x<br />

65. -5v = -5(4. 3}<br />

=(-5 .4. - 5 .3)<br />

=(-20. -15)<br />

. e­<br />

a = /vi cos 8<br />

b = Ivlsin8<br />

= 69.1cos251°20' = 69.1sin251°20'<br />

= 69.1cos 251.33° = 69.1cos251.33°<br />

==-22.116 ==-65.465<br />

v = (-22.116. -65.465)<br />

66.<br />

67.<br />

3u+ 6v = 3(-2.5).+6(4. 3}<br />

=(3·-2,3.5)+(6·4.6·3)<br />

= (-6. 15)+ (24. 18)<br />

= (-6+ 24.15 + 18)<br />

=(18,33)<br />

(-5.8)=-5i+8j<br />

68. (6,-3)=6i-3j<br />

69. (2.0}=2i+Oj=2i<br />

70. (0. -4)= 0i-4j =-4j<br />

297


<strong>Trigonometry</strong> <strong>Solutions</strong> <strong>Manual</strong><br />

298 Chapter 7 Applications of <strong>Trigonometry</strong> and Vectors<br />

....<br />

71. a = 8cos45° b = 8sin45<br />

( (1,7) ·(I,I) J<br />

= 8 . .,fi = 8. .,fi 82. ()= arccos<br />

2 2 ~12 +7 2 ' ~ 1 2 +1 2<br />

=4.,fi =4.,fi<br />

(4.,fi, 4.J2) = 4.,fii + 4.,fij<br />

( 1+7 )<br />

= arccos .J5O ..,fi<br />

72. a = 3cos 210° b = 3sin21O° = arccos( 1~)<br />

-~ 1<br />

=3·- =3·-­<br />

2 2<br />

:= 36.87°<br />

-3~ 3<br />

:;::-­<br />

=<br />

2 2 83. ()= arccos<br />

[ (I, 2) (-6, 3) J<br />

~12+22 ' ~(-6) 2 + 3 2<br />

(_Wi _~)=_3~i_~j<br />

2 ' 2 2 2<br />

= arcco { .J5.J45 -6+6)<br />

73. a = .6cos115° b = .6sin 115° = arccos(O)<br />

:= -.254 :=.544<br />

=90°<br />

(-.254, .544) = -.254i+.544j<br />

74. a = .9cos 208° b = .9sin208° 84. ()= arcco<br />

{ (4, 0}.(2, 2) J<br />

:= -.795 =-.423 ~42 +0 2 .~22 +2 2<br />

(-.795, -.423)=-.795i-.423j<br />

75. (6, -1) ·(2, 5) = 12 - 5 = 7<br />

76. (-3,8)·(7, -5)=-21-40=-61<br />

77. (2, - 3)·(6,5) = 12 -15 =-3<br />

{ 8+0 )<br />

= arcco ..Jl6 ..J8<br />

=arcco{81)<br />

=arcco{*J<br />

78. (1,2)'(3, -1)=3-2=1 =arcco{ ~)<br />

79. (4,0) ·(5, -9)=20+0=20<br />

SO. (2,4) .(0, -1)=0-4=-4 85.<br />

For Exercises 81-86, use the formula ()= arcco {u IuIIvI .v).<br />

=45°<br />

()= arcco<br />

{ (3, 4).(0, I) J<br />

~32 +4 2 . ~0 2 +1 2<br />

{ 0+4 )<br />

= arcco .,fi5 ...[i<br />

=arcco{~)<br />

81. ()= arcco<br />

{ (2, I) (-3, 1) J '" 36.87°<br />

~22 +1 2 '~(_3)2 +1 2 { (-5, 12). (3, 2) J<br />

{ -6+1 ) 86. () = arcco<br />

= arcco .J5..JW ~(_5)2 +12 2 .~32 +2 2<br />

=arcco{5~)<br />

169 13<br />

=arcco{~) =arcco{ ~)<br />

1313<br />

(-15+24)<br />

= arccos ..Jl69.Jl3<br />

=arcco{- ~)<br />

= arcco{ 9.Jl3)<br />

169<br />

= 135° := 78.93°<br />

298


<strong>Trigonometry</strong> <strong>Solutions</strong> <strong>Manual</strong><br />

7.4 Vectors 299<br />

87. (3u)' v = (3(-2, 1)). (3,4) 97. Draw a line parallel to the x-axis and the vector<br />

u + v (shown as a dashed line).<br />

= (--6,3)' (3,4) y<br />

= -18+ 12<br />

=--6<br />

88. u.(v-w)=(-2, 1).(3, 4)-(-5, 12))<br />

=(-2,1)'(3-(-5),4-12)<br />

----r.:::+-:--L-._ x<br />

=(-2,1)'(8, -8)<br />

80' a A<br />

=-16-8 3<br />

=-24 Since 0 1 = 110°, its supplementary angle is 70°.<br />

Further since O 2 = 260° , a = 260° - 180° = 80°.<br />

89. u,v-u'w=(-2, 1)'(3,4)-(-2,1)'(-5, 12)<br />

Then the angle eBA becomes<br />

= (--6+ 4) -(10+ 12)<br />

180 - (80 + 70) = 180 - 150 = 30°. Using the law<br />

=-2-22 of cosines, the magnitude of u + v is<br />

=-24<br />

2+c2-2accosB<br />

In+vl 2 =a<br />

90. u ·(3v)=(-2, 1).(3(3, 4)) = (3)2 + (12)2 - 2(3)(12)cos 30°<br />

=90.65<br />

= (~2, 1).(9, 12)<br />

= -18+ 12 In+ vi = ../90.6461709<br />

=--6 "" 9.52082827.<br />

91. (1, 2) ·(--6, 3) = --6 + 6 Using the law of sines,<br />

=0 sinA sin B<br />

--=-­<br />

The vectors are orthogonal. a b<br />

. A asinB<br />

SIO =-­<br />

92. (3,4)'(6,8)= 18+32 b<br />

=50<br />

The vectors are not orthogonal.<br />

A =arcsl .{asinB) -­<br />

b<br />

.{ 3sin 300)<br />

A =arcs)<br />

93. (1, 0).(..fi,0)=..fi +0 9.52<br />

=..fi<br />

The vectors are not orthogonal.<br />

= 9.0646784°<br />

The direction angle of U + v is<br />

110 + 9.0646784 = 119.0646784°.<br />

94. (1, 1). (1, -1) = 1-1<br />

98. al =lui cos 0 1 ~ = luisin 0 1<br />

=0 =12 cos 110° =12sin 110°<br />

The vectors are orthogonal. = -4.10424172 "" 11.27631145<br />

(-4.10424172, 11.27631145)<br />

95. (...[5,-2),(-5,2...[5) = -5...[5 -4...[5<br />

=-9.,J5 99. a2 =IvicoS 02 ~ =lvl sin 02<br />

The vectors are not orthogonal. =3cos26O° = 3sin260°<br />

96. (-4,3).(8, -6)=-32-18<br />

=-50<br />

The vectors are not orthogonal.<br />

= -5.20944533 = -2.954423259<br />

(-.520944533, - 2.954423259)<br />

100. a =a1 + a2<br />

= -4.10424172 - .520944533<br />

= -4.625186253<br />

b=~+~<br />

= 11.27631145 - 2.954423259<br />

= 8.321888191<br />

(-4.625186253,8.321888191)<br />

299


<strong>Trigonometry</strong> <strong>Solutions</strong> <strong>Manual</strong><br />

300 Chapter 7 Applications of <strong>Trigonometry</strong> and Vectors<br />

101. Magnitude: ~(-4.625186253)2 +(8.321888191)2<br />

= 9.52082827<br />

A 1 8 ( 8.321888191 )<br />

ng e: =arccos -4.625186253<br />

=119.0646784°<br />

(in quadrant II since y > 0 and x < 0)<br />

102. (a) They are the same.<br />

Section 7.5<br />

Exercises<br />

1. Let a = the angle between the forces.<br />

-,<br />

(J ',423<br />

,<br />

-,<br />


<strong>Trigonometry</strong> <strong>Solutions</strong> <strong>Manual</strong><br />

7.5 Applications of Vectors 301<br />

3. Let (J = the angle that the hill makes with the 6. Let r = the vertical component of the person<br />

horizontal.<br />

exerting a 114-lb force and s = the vertical<br />

component of the rson exerting a ISQ-lb force .<br />

25<br />

The 80-lb downward force has a 25-lb component<br />

parallel to the hill. The two right triangles are<br />

similar and have congruent angles.<br />

. (J 25<br />

sm =­<br />

80<br />

(J = 18°<br />

4. Let If1 = the force required to keep the car parked<br />

on hill.<br />

62.4·<br />

The weight of the box is the sum of the<br />

magnitudes of the vertical components of the two<br />

vectors representing the forces exerted by the two<br />

people.<br />

Irl = 114 sin 54.9° = 93.27<br />

~ = ISOsin 62.4° = 132.93<br />

Weight of box = Irl + ~<br />

= 2261b<br />

7. Let Ixl = the weight of the boat.<br />

r<br />

J!L=sinISo<br />

3000<br />

If1 = 3000sin 15° = 780 Ib<br />

5. Find the force needed to pull a 6O-ton monolith<br />

along the causeway.<br />

2.3·<br />

500 =sinI8 0<br />

500lb<br />

Ixl<br />

500<br />

Ixl =sin 180<br />

Ixl= 1600 Ib<br />

The force needed to pull 60 tons is equal to the<br />

magnitude of x, the component parallel to the<br />

causeway.<br />

sin2.3o=M<br />

60<br />

IxI = 60sin 2.3°<br />

=2.4<br />

The force needed is 2.4 tons .<br />

301


<strong>Trigonometry</strong> <strong>Solutions</strong> <strong>Manual</strong><br />

302 Chapter 7 Applications of <strong>Trigonometry</strong> and VectOI'S<br />

The equilibrant has a magnitude of 2640 lb and<br />

makes an angle of 167.2° with the 1480-lb force.<br />

The weight is 2640 lb at an angle of 167.2° with<br />

the 1480-lb force.<br />

9, Find the weight of the crate and the tension on the<br />

horizontal rope.<br />

v<br />

x<br />

-,<br />

'\.<br />

'\.<br />

-,<br />

-,<br />

-,<br />

w<br />

b<br />

v has horizontal component x and vertical<br />

component y. The resultant v + h also has vertical<br />

component y. The resultant balances the weight<br />

of the crate, so its vertical component is the<br />

equilibrant of the crate's weight.<br />

IwI ;;:: ~I ;;:: 89.6 sin 46° 20'<br />

Iwl;;::64.8Ib<br />

Since the crate is not moving side-to-side, h, the<br />

horizontal tension on the rope, is the opposite of<br />

x .<br />

1hI;;:: Ixl;;:: 89.6cos46°20'<br />

IhI;;:: 61.91b<br />

The weight of the crate is 64.8 lb: the tension is<br />

61.9 lb.<br />

10, Draw a vector diagram showing the three forces<br />

acting at a point in equilibrium. Then, arrange the<br />

forces to form a triangle. The angles between the<br />

forces are the supplements of the angles of the<br />

triangle.<br />

760lb<br />

The angle between the 1220-lb and 980-lb forces<br />

is<br />

180° - a= 141.5°.<br />

(3) f3 =180° - a - 0<br />

=53.4°<br />

The angle between the 760-lb and 1220-lb forces<br />

is<br />

180° - f3 = 126.6°.<br />

II, Find the magnitude of the second force and of the<br />

resultant. Use the parallelogram rule; v is the<br />

resultant and x is the second force.<br />

a<br />

I<br />

-71-----v<br />

I<br />

I<br />

I<br />

I<br />

I<br />

78'50'<br />

I<br />

41'10'<br />

176<br />

I<br />

a =180° -78°50' = 101°10'<br />

the angle between the second force and the<br />

resultant is f3 =78° 50' - 41°10' = 37° 40' .<br />

Use the law of sines to find Ivl.<br />

Iv! 176<br />

sin a = sinf3<br />

Ivi = 176 sin 101° 10'<br />

sin37°40' .<br />

Ivi =2831b<br />

Use the law of sines to find Ixl.<br />

IxI = 176<br />

sin41°10 sin 37°40'<br />

Ixl= 176sin41°IO'<br />

sin 37°40'<br />

Ixl;;:: 190 lb<br />

12. Let It1 ;;:: the second force and Irl =the magnitude<br />

of the resultant.<br />

9801~7601b<br />

~<br />

12201b<br />

980 2 +760 2 -1220 2<br />

(1) cosO;;:: 2(980)(760)<br />

0;;:: 88.1°<br />

The angle between the 760-lb and 980-lb forces is<br />

180°-0 =91.9°<br />

sina sinO<br />

(2) 760 =1220<br />

a =38.5°<br />

/<br />

/<br />

r /<br />

/<br />

/<br />

/<br />

/<br />

~2'IO' /<br />

(J/'/<br />

",,-~~<br />

32'40'<br />

0= 180° - 42° 10' =137°50'<br />

f3 =42° 10' - 32° 40';;:: 9° 30'<br />

Irl 28.7<br />

--=--­<br />

sin 0 sin 9° 30'<br />

Irl =117 Ib<br />

302


<strong>Trigonometry</strong> <strong>Solutions</strong> <strong>Manual</strong><br />

7.5 Applications of Vectors 303<br />

Irl<br />

sin 32° 40' sin 137° 50'<br />

IfI = 93.9 Ib<br />

The magnitude of the resultant is 117 lb; the<br />

second force is 93.9 lb.<br />

13. Let v = the ground speed vector of the plane.<br />

Find the actual bearing of the plane.<br />

N<br />

A 25<br />

Angle A = 266.6° -175.3° = 91.3°<br />

Use the law of cosines to find Ivl.<br />

1v1 2 = 25 2 + 650 2 - 2(25)(650) cos 91.3°<br />

=423,862<br />

/vi = 651<br />

Use the law of sines to find the drift angle, B, the<br />

angle that v makes with the airspeed vector of the<br />

plane.<br />

sinB<br />

sinA<br />

25 651<br />

sin B = .038392573<br />

B=2.2°<br />

The bearing is 175.3° - 2.2° = 173.10.<br />

14. Let Ixl = the airspeed and IdI = the ground speed.<br />

N<br />

-+oe:;;;.-.::;.....--~~E<br />

x<br />

(J =90°-74.9° = 15.1°<br />

M=cotI5.1°<br />

.::,<br />

42<br />

Ixl = 42 cot 15.1°<br />

=156mph<br />

1!L=cscI5.1°<br />

42<br />

1


<strong>Trigonometry</strong> <strong>Solutions</strong> <strong>Manual</strong><br />

304 Chapter 7 Applications of <strong>Trigonometry</strong> and Vectors<br />

18. Let c = the ground speed vector.<br />

N<br />

20. Let v = the ground speed vector .<br />

N<br />

B<br />

v 192<br />

By alternate interior angles,<br />

angle A = 57° 40' .<br />

Use the law of sines to find B, the drift angle.<br />

sinA sinB<br />

--=-­<br />

168 27.1<br />

sin B = .13629<br />

B = 7°50'<br />

Bearing = 57° 40' + B = 65° 30'<br />

C= 180°- A- B = 114°30'<br />

~= 27.1<br />

sinC sinB<br />

lei = 181 mph<br />

The bearing is 65° 30'; the ground speed is<br />

181 mph.<br />

19. Let v = the airspeed vector.<br />

The ground speed must be<br />

400<br />

-=160 mph.<br />

2.5<br />

N<br />

II<br />

B<br />

Angle A = 328° - 180° = 148°<br />

Use the law of cosines to find Ivl.<br />

Ivl 2 = 11 2 + 160 2 - 2(l1)(160)cosI48°<br />

Ivl2 = 28,706<br />

Ivl = 170<br />

The airspeed must be 170 mph.<br />

Use the law of sines to find B, the drift angle.<br />

sin B sin 148°<br />

--=<br />

11 170<br />

. B 11sin 148°<br />

Sin =---­<br />

170<br />

B=2.0°<br />

The bearing must be 360° - 2.0° = 358°.<br />

--<br />

A 23 C<br />

Angle C = 180° - 78° = 102°<br />

1v1 2 =23 2 +192 2 -2(23)(192)cosl02°<br />

Ivl= 198 mph<br />

sin B sin 102°<br />

--=<br />

23 198<br />

B=6.5°<br />

Bearing = 180° + B = 186.5°<br />

21. Find the ground speed and resulting bearing.<br />

N<br />

245'<br />

Angle A = 245° -174° = 71°<br />

Use the law of cosines to find Ivl.<br />

Ivl2 = 30 2 + 240 2 - 2(30)(240) cos 71°<br />

1v1 2 "= 53,812<br />

Ivl"=230<br />

The ground speed is 230 Ian per hr.<br />

Use the law of sines to find angle B.<br />

sinB sin71°<br />

--=-­<br />

30 230<br />

sinB=.1233<br />

B=7°<br />

The resulting bearing is 174° _7° = 167°.<br />

22. Let A =the point where the ship changes to a<br />

bearing of 62° and C = the point where it changes<br />

to a bearing of 115°.<br />

N<br />

N<br />

F<br />

D<br />

304


<strong>Trigonometry</strong> <strong>Solutions</strong> <strong>Manual</strong><br />

7.5 Applications of Vectors 305<br />

Angle CAB = 90° - 62° = 28°<br />

Angle FCA =180° - angle DAC<br />

=180°-62° =118°<br />

AngleACB =360°- angle FCB- angle FCA<br />

=360° -115°-118°<br />

=127°<br />

Angle CBA =180°- angleACB- angle CAB<br />

=180° -127° - 28° =25°<br />

Using the law of sines, we have<br />

sin 127° sin 28<br />

=-­<br />

50 CB<br />

CB=29.4<br />

sin 127° sin 25°<br />

=-­<br />

50 AC<br />

AC=26.5.<br />

The ship traveled 26.5 + 29.4 =55.9 mi. To avoid the iceberg, the ship had to travel 55.9 - 50 =5.9 mi.<br />

23. At what time did the pilot turn The plane will fly 2.6 hr before it runs out of fuel. In 2.6 hr, the carrier will travel<br />

(2.6)(32) = 83.2 mi and the plane will travel a total of (2.6)(520) = 1352 mi. Suppose it travels x mi on its initial<br />

bearing; then it travels 1352 - x mi after having turned.<br />

338·<br />

Use the law of cosines to get an equation in x.<br />

(1352 - x)2 =x 2 +(83.2)2 - 2(x)(83 .2)cos 52°<br />

l, 827,904 - 2704x + x 2 =x 2 + 6922.24 - 102.45x<br />

r, 820,982 =2601.55x<br />

7oo=x<br />

To travel 700 mi at 520 mph requires 700 =1.35 hr, or 1 hr and 21 min.<br />

520<br />

The pilot turned at 1 hr 21 min after 2 P.M., or at 3:21 P.M.<br />

24. (a) Relative to the banks. the motorboat will be traveling<br />

2<br />

x +3 2 =7 2<br />

x 2 =49-9<br />

x =.J4O "" 6.32 mph<br />

(b) At 6.32 mph, the motorboat travels<br />

6.32miles 5280 feet 1 hour 1 minute<br />

---_.<br />

._--­<br />

hour 1 mile 60 minutes 60 seconds<br />

=9.276 ft I sec<br />

Crossing a 132-foot wide river would take<br />

1 second<br />

132 feet · '" 14.23 seconds<br />

9.276 feet<br />

305


<strong>Trigonometry</strong> <strong>Solutions</strong> <strong>Manual</strong><br />

306 Chapter 7 Applications of <strong>Trigonometry</strong> and Vectors<br />

() C SID<br />

'() =­ 3<br />

7<br />

() =arcsin(%)<br />

'" 25.38 0<br />

25. (a) First change 10.34" to radians in order to use the length of arc formula.<br />

0<br />

10.34" ._1__ • .....!!..- '"5.013 x 10- 5 radian .<br />

3600" 180 0<br />

In one year Barnard's Star will move in the tangential direction a distance of<br />

s = r(} =(35 x 1OI2X5.013 xJO- 5 )<br />

= 1,754,550,000 mi.<br />

' S 1,754,550,000 . .<br />

Inone second B amard s tar moves '" 56 nu tangentially, Thus, vt '" 56 mi I sec .<br />

60 ·60 ·24 ·365<br />

(b) The magnitude of v is given by<br />

1v!2 =~l +~l<br />

Iv! =~672 + 56 2<br />

'" 87 mi I sec.<br />

26. m =-cot 0<br />

From the figure, we see that L passes through the point (beosO, bsinO).<br />

Using this information and the general form of the line: y - bsinO =-cotO(x -boosO).<br />

(Other answers are possible.)<br />

27. At the end point of v, x =a. Using the formula from Exercise 26:<br />

y- bsinO =-cotO(x - b cos 0)<br />

y - bsinO =-cotO(a -bcosO)<br />

y = -a cot 0+ bcosOcotO+ bsin 0<br />

The coordinates are (a,-a cotO + bcosOcotO + bsin 0).<br />

28. Iv! = ~(XI - 0)2 + (YI - 0)2<br />

=~(a _0)2 + (-a cot 0 + bcosOcotO + bsinO _0)2<br />

=~a2 + a 2 cot 2 ()- 2abcot 2 OcosO - 2abeot Osin 0 + b 2 cos 2 Beot 2 B+ 2b 2 cosOsin BcotO + b 2 sin 2 B<br />

=~a2 + a 2 cot 2 B- 2abcot 2 OcosO- 2abcosO+ b 2 cos 2 Ocot 2 0+ 2b 2 eos 2 0+ b 2 sin 2 B<br />

=<br />

a 2 sin 2 0+a 2 cos 2 0-2abcos 3 B-2abcosBsin 2 0+b 2 cos 4 B+2b 2 cos 2 Bsin 2 0+b 2 sin 4 0<br />

sin 20<br />

~a2(sin2 0 + cos 2 0) - 2abeos 8(cos 2 0 + sin 2 0) + b 2(eos4 0 + eos 2 Osin 2 B+sin 2 0)<br />

sin 0<br />

"<br />

=~a2 .1-2ab cos 0·1 + b 2(cos2 B+ sin 2 B)2<br />

sinO<br />

~a2 _ 2abcosB+ b 2 .1<br />

= 2<br />

sinO<br />

~a2 -2abcosB+b 2 ~a2 +b 2 -2abcosO<br />

= =<br />

sinB<br />

sin B<br />

306


<strong>Trigonometry</strong> <strong>Solutions</strong> <strong>Manual</strong><br />

Chapter 7 Review Exercises 307<br />

29. Applying the law of cosines, we see that the<br />

quantity<br />

a 2 +b 2 -2abcosO<br />

is exactly the segment connecting the endpoint of<br />

a to the endpoint of b.<br />

Chapter 7 Review Exercises<br />

1. Find b, given C = 74.2°, c = 96.3 m, B =39.5°.<br />

Use the law of sines to find b.<br />

b c<br />

--=-­<br />

sinB sinC<br />

b= csinB<br />

sinC<br />

96.3 sin 39.5°<br />

= sin 74.2°<br />

=63.7 m<br />

2. Find B, given A = 129.7°, a = 127 ft,<br />

b= 69.8 ft.<br />

Use the law of sines to find B.<br />

sinB sinA<br />

--=-­<br />

b a<br />

. B 69.8sinI29.7°<br />

Sm =<br />

127<br />

= .42286684<br />

Since angle A is obtuse, angle B is acute.<br />

B=25.0°<br />

3. Find B, given C = 51.3°, c = 68.3 m, b =58.2 m.<br />

Use the law of sines to find B.<br />

sinB sinC<br />

--=-­<br />

b c<br />

. B bsinC<br />

Sm =-­<br />

c<br />

58.2sin51.3°<br />

68.3<br />

sin B = .66502269<br />

B =41.7°<br />

Angle B cannot be obtuse, since b < c , B < C, and<br />

C is acute.<br />

4. Find b, given a =165 m, A = 100.2°, B = 25.0°.<br />

b a<br />

--=-­<br />

sinB sin A<br />

b = 165sin25.0°<br />

sin 100.2°<br />

b=70.9m<br />

S. Find A, given B = 39° 50' , b = 268 m, a =340 m.<br />

a > b > a sin B tells us that two triangles are<br />

possible and A can have two values.<br />

Use the law of sines to find A.<br />

sinA<br />

sinB<br />

--=-­<br />

a b<br />

. A asinB<br />

Sm =-­<br />

b<br />

340 sin 39° 50'<br />

268<br />

sin A = .81264638<br />

A = 54°20' or A = 125° 40'<br />

6. Find A, given C=79°20', c=97.4mm,<br />

a=75.3 mm.<br />

Use the law of sines to find A.<br />

sinA sinC<br />

--=-­<br />

a c<br />

. A 75.3sin 79°20'<br />

Sm =<br />

97.4<br />

=.75974194<br />

Angle A cannot be obtuse, since a < c and Cis<br />

acute.<br />

A = 49°30'<br />

9. a = 10, B = 30°<br />

(a) The value of b that forms a right triangle<br />

would yield exactly one value for A. That is,<br />

b = 10 sin 30° = 5. Also any value of b<br />

greater than or equal to 10 would yield a<br />

unique value for A.<br />

(b) Any value of b between 5 and 10, would<br />

yield two possible values for A.<br />

(c) If b is less than 5, then no value for A is<br />

possible.<br />

10. a = 10, B =150°<br />

(a) any value of b greater than 10 would yield<br />

exactly one possible value for A.<br />

(b) No values of b would yield two values for A,<br />

since A has to be an acute angle .<br />

(c)<br />

Any value of b less than or equal to 10 would<br />

yield no possible values for A.<br />

11. Find A, given a = 86.14 in., b =253.2 in.,<br />

c = 241.9 in.<br />

Use the law of cosines to find A.<br />

a 2 = b 2 +c 2 - 2bc cos A<br />

b2+c2_a2<br />

cos A =<br />

2bc<br />

253.2 2 +241.9 2 -86.14 2<br />

=<br />

2(253.2)(241.9)<br />

cos A = .94046923<br />

A = 19.87° or 19° 52'<br />

307


<strong>Trigonometry</strong> <strong>Solutions</strong> <strong>Manual</strong><br />

308 Chapter 7 Applications of <strong>Trigonometry</strong> and Vectors<br />

12. Find b. given B = 120.7°. a = 127 ft, C = 69.8 ft.<br />

Use the law of cosines to find b.<br />

b 2 =a 2 +c 2 -2accos B<br />

= 127 2 + 69.8 2 - 2(127X69.8)cos 120.7°<br />

b=173ft<br />

13. Find a. given A = 51 ° 20'. C = 68.3 m. b = 58.2 m;<br />

Use the law of cosines to find a.<br />

a 2 =b 2+c2 -2bccosA<br />

a = ~(58.2)2 + (68.3)2 -2(58.2X68.3)cos51° 20'<br />

a=55.5 m<br />

14. Find B. given a = 14.8 rn,b = 19.7 m, c = 31.8 m.<br />

b<br />

Use the law of cosines to find B.<br />

a 2 +c 2 _b 2<br />

cosB=----­<br />

2ac<br />

14.8 2 + 31.8 2 -19.7 2<br />

=<br />

2(14.8X31.8)<br />

B = 265° or 26° 30'<br />

15. Find a, given A = 60°. b = Scm, c = 21 em.<br />

Use the law of cosines to find a.<br />

2 = b<br />

2<br />

a + c 2 - 2bc cos A<br />

a = ~(5}2 + (21)2 - 2(5X21)cos 60°<br />

a =19cm<br />

16. Find A. given a = 13 ft, b = 17 ft, c = 8 ft.<br />

b 2 +c 2 _ a 2<br />

cosA=---­<br />

2bc<br />

17 2 +8 2 _13 2<br />

2(17X8)<br />

A =47°<br />

17. Solve thetriangle, given A = 25.2 0, a = 6.92 yd.<br />

b =4.82 yd.<br />

a > b tells us that only one triangle is possible.<br />

Use thelaw of sines to find B.<br />

sinB smA<br />

--=-­<br />

b a<br />

. B bsinA<br />

sm · =-­<br />

a<br />

. B 4.82 sin 25.2°<br />

sm =----­<br />

6.92<br />

B=I7.3°<br />

(Angle B cannot be obtuse, since b < a, B < A,<br />

and A is acute.)<br />

C =18O"'-A- B= 137.5°<br />

Use the law of sines to find c.<br />

c a<br />

--=-­<br />

sinC sinA<br />

asinC<br />

c=-­<br />

sin A<br />

6.92 sin 137.5°<br />

=<br />

sin25.2°<br />

= 11.0 yd<br />

18. Solve the triangle, given A = 61.7°, a = 78.9 m,<br />

b= 86.4 m.<br />

Since b > a> b sin A, there are two triangles .<br />

sin B sin A<br />

--=-­<br />

a<br />

. B 86.4 sin 61.7°<br />

sm =----­<br />

78 .9<br />

=.96417292<br />

Bt = 74.6°<br />

C 1 =180 0 - A - B = 43.7°<br />

_c_l_=_a_<br />

sinC I sin A<br />

78.9sin43.7°<br />

cl=----­<br />

sin 61.7°<br />

=61.9 m<br />

Bz = 180° - Bt = 105.4°<br />

C2 = 180 0-A-Bz = 12.9°<br />

~=_a_<br />

sinC 2<br />

sinA<br />

78.9sin 12:9°<br />

c2 = sin 61.7°<br />

=20.0m<br />

19. Solve the triangle, given a = 27.6 em. b = 19.8 em,<br />

C=42° 30'.<br />

Using the law of cosines, we have<br />

2 2<br />

+ b 2<br />

c = a - 2ab cos C<br />

c 2 = 27.6 2 + 19.8 2 -2(27.6XI9.8)cos42° 30'<br />

c= 18.7 em.<br />

Note: Keep all values of...[ in your calculator<br />

for use in the next calculation. If 18.7 is used, the<br />

answer will vary slightly due to round-off error.<br />

Now use the law of sines to find B so there is no<br />

ambiguity.<br />

c b<br />

--=-­<br />

sinC sinB<br />

18.65436576 19.8<br />

=-­<br />

sin 42° 30' sin B<br />

. B 19.8sin42° 30'<br />

SID =----­<br />

18.65436576<br />

B=45° 50'.<br />

308


<strong>Trigonometry</strong> <strong>Solutions</strong> <strong>Manual</strong><br />

Chapter 7 Review Exercises 309<br />

Since the sum of the angles of a triangle is 180°, 24. Given a = 43 m, b = 32 m, c = 51 m, find the area.<br />

A = 180°- B-C<br />

Use Heron's formula to find the area.<br />

= 180° - 45° 50' - 42° 30'<br />

1<br />

s=-(a+b+c)<br />

A =91° 40'.<br />

2<br />

1<br />

= -(43+ 32 + 51)<br />

20. Solve the triangle, given a = 94.6 yd, b =123 yd,<br />

2<br />

c = 109 yd.<br />

=63<br />

Using the law of cosines, we have<br />

123 2 + 109 2 -94.6 2<br />

Area = ..j's('-s---a:-)(-s-----,-,b):-(s---c-)<br />

cosA =------­<br />

= ,J63(63 - 43)(63 - 32)(63 - 51)<br />

2(123)(109)<br />

A =47.7°.<br />

= 680m 2<br />

Again, using the law of cosines, we have<br />

94.6 2 + 109 2 -123 2<br />

25. Substitute a = 7, b = 7.J3, c =14, A = 30°,<br />

cos B = --..,.......--..,-..,.......--,--­<br />

2(94.6XI09)<br />

a+b cos.L(A- B)<br />

B = 60° and C = 90° into __ = _...=2:....:-._..:­<br />

B= 73.9°.<br />

, c sintC<br />

Since the sum of the angles of a triangle is 180°,<br />

in order to verify the accuracy of the information.<br />

C=1800-A-B<br />

a+b 7+7.[3<br />

= 180° - 47.7° -73.9°<br />

--=--­<br />

c 14<br />

= 58.4°.<br />

7(1+ .J3) 1+ .J3<br />

= =:-­<br />

21. Given b = 840.6 m, c = 715.9 m, A = 149.3°, find<br />

14 2<br />

the area.<br />

cOS!(A-B) _ cos-t(300-600)<br />

Angle A is included between sides band c.<br />

sin!C - sin-t(900)<br />

area = !bcsin A<br />

2<br />

= cost(-300)<br />

= !(840.6)(715.9)sin 149.3°<br />

sin 45°<br />

2<br />

cos 15°<br />

area = 153,600 m<br />

2<br />

=--­<br />

sin 45°<br />

.,fi +../6 .<br />

22. Given a = 6.90 ft, b = 10.2 ft, C= 35° 10', find the<br />

=-.r- =__<br />

1+.J3<br />

area. Angle C is included between sides a and b.<br />

if 2<br />

Area =!absinC<br />

(Use half-angle formulas to find cos 15°.)<br />

2<br />

. . 1 1+..[3<br />

Each expression IS equa to - --.<br />

= ! (6.90XIO.2) sin 35° 10'<br />

2<br />

2<br />

Area = 20.3 ft2<br />

26. Substitute a = 7, b = 7.J3, c = 14, A = 30°,<br />

23. Given a = .913 km, b = .816 km, c = .582 km, find<br />

a - b sin.L(A - B)<br />

B = 60°, and C = 90° into --= 2.L in<br />

the area.<br />

c<br />

cos~C<br />

1<br />

s=-(a+b+c)<br />

order to verify the accuracy of the information.<br />

2<br />

a-b 7-7-./3<br />

=----'-­<br />

= !(.913+ .816+.582)<br />

c 14<br />

2<br />

= 1.1555<br />

7(1- -./3) 1- -./3<br />

= =-­<br />

area = ~(1.1555)(.2425)(.3395X.5735)<br />

14 2<br />

area = .234 km 2<br />

309


<strong>Trigonometry</strong> <strong>Solutions</strong> <strong>Manual</strong><br />

310 Chapter 7 Applications of <strong>Trigonometry</strong> and Vectors<br />

sint(A-B) sint(300-600)<br />

Use the law of sines to find AC = b.<br />

b c<br />

coste - cos t (90°) --=-­<br />

sinB sinC<br />

= sint(-300)<br />

b = csinB<br />

cos 45°<br />

sinC<br />

-sin 15°<br />

=<br />

7.0sin68°<br />

cos 45° =<br />

sin 30°<br />

=13<br />

The tree is 13 meters tall.<br />

=__4_­__ :fi=:il 1- .J3<br />

..fi - 2<br />

2<br />

(Use half-angle formulas to find -sin 15°.)<br />

.. I 1-.J3<br />

30. Let c = the length of the tunnel.<br />

A<br />

E ac h expression IS equa to --.<br />

2<br />

B<br />

1:1. Since B = 58.4° and C= 27.9°,<br />

A = 180 - (58.4 + 27.9) = 93.7°<br />

Using the law of sines:<br />

C<br />

125 AB<br />

Use the law of cosines to find c.<br />

--=-­<br />

sinA sinC<br />

c 2 = 3800 2 + 2900 2 - 2(3800X2900) cos 110°<br />

AB = (125)sin e c 2 = 30,388,124<br />

sin A c=5500<br />

_ (125Xsin27.9°)<br />

The tunnel is 5500 meters long.<br />

- sin(93.7°)<br />

31. Let h =the height of tree.<br />

=58.6<br />

The canyon 58.6 feet across.<br />

28. The angle opposite the 8.0 ft flagpole is:<br />

180 - (l15 + 22) = 43°<br />

Using the law of sines:<br />

14.3'<br />

8 x<br />

--=<br />

8 = 27.2° -14.3° = 12.9°<br />

sin 43° sin 115° a = 90° - 27.2° = 62.8°<br />

8sin115°<br />

x=--­<br />

h 212<br />

sin43°<br />

--=-­<br />

sin8 sina<br />

=11 h= 212sinI2.9° =53.2<br />

The brace is 11 feet long.<br />

sin 62.8°<br />

The height of the tree is 53.2 ft.<br />

29. Let Ae= the height of the tree.<br />

I C<br />

I<br />

I<br />

I<br />

8~<br />

I<br />

I<br />

.-<br />

A I<br />

B<br />

o<br />

Angle A = 90° - 8.0° = 82°<br />

Angle e = 180° - B ­ A = 30°<br />

310


<strong>Trigonometry</strong> <strong>Solutions</strong> <strong>Manual</strong><br />

Chapter 7 Review Exercises 311<br />

32" Sketch a triangle showing the situation as follows.<br />

N<br />

N<br />

rock<br />

B<br />

35. Let d = the distance between the submarine and<br />

the battleship.<br />

17"30'<br />

24"\0'<br />

45"20'<br />

A l'--------.,~<br />

15.2 C<br />

308"40'<br />

Angle A = 90° - 45° 20' = 44° 40'<br />

Angle C = 308° 40' - 270° = 38° 40'<br />

Angle B = 180° -A - C= 96° 40'<br />

Use the law of sines to find BC = a.<br />

a b<br />

--=-­<br />

sinA<br />

sin B<br />

bsinA<br />

a=-­<br />

sinB<br />

15.2sin44° 40'<br />

=<br />

sin 96° 40'<br />

a = 10.8<br />

The distance between them is 10.8 miles.<br />

33. Let x = the distance between the boats.<br />

In 3 hours, the first boat travels 3(36.2) =108.6 km<br />

and the second travels 3(45.6) = 136.8 km.<br />

\08.6<br />

x<br />

=<br />

sin 6° 40' sin 155° 50'<br />

d = 1450 ft<br />

The distance between them is 1450 ft.<br />

36" To find the angles of the triangle formed by the<br />

ship's positions with the lighthouse, we find the<br />

supplementary angle to 55°:<br />

180° - 55° = 125°<br />

and the third angle in the triangle:<br />

180° - (125° + 30°) = 25°<br />

Lighthouse<br />

Use the law of cosines to find x.<br />

x 2 = (108.6)2 + (136.8)2<br />

- 2(108.6X136.8)cos 54° 10'<br />

=13,113.359<br />

x = 115 km<br />

They are 115 km apart.<br />

34. Sketch the parallelogram. The figure has equal<br />

opposite sides and an les.<br />

12 em 147"<br />

---=--­<br />

sin 25° sin 30°<br />

2sin30°<br />

x= sin 25°<br />

= 2.4 mi<br />

The ship is 2.4 miles from the lighthouse.<br />

15cm<br />

Let d 1 and d2 be the diagonals of the<br />

parallelogram.<br />

d l<br />

2 =12<br />

2<br />

+15 2 - 2(12)(15)cos33°<br />

d l =8.2 em<br />

d 2<br />

2 = 12 2 + 15 2 - 2(12)(15)cos 147°<br />

d 2 =26 cm<br />

The diagonals are 8.2 cm and 26 em.<br />

311


<strong>Trigonometry</strong> <strong>Solutions</strong> <strong>Manual</strong><br />

312 Chapter 7 Applications of <strong>Trigonometry</strong> and Vectors<br />

37. Let A = home plate, B = first base, C = second<br />

base, D =third base, P =the pitcher's rubber.<br />

Draw ACthrough P, draw PB and PD.<br />

C<br />

D £..__.!+---:;7'B<br />

A<br />

In triangle ABC, angle B = 90°,<br />

angle A = angle C = 45°.<br />

AC= ~902 +90 2 =90../2<br />

PC =90../2- 60.5 = 66.8 ft<br />

In triangle APB, angle A = 45°.<br />

PB 2 =Ap 2 +AB 2 -2(AP)(AB)cos45°<br />

= 60.5 2 + 90 2 - 2(60.5)(90)cos 45°<br />

PB= 63.7 ft<br />

Since triangles APB and APD are congruent,<br />

PB = PD = 63.7 ft.<br />

The distance to both first and third base is 63.7 ft<br />

and the distance to second base is 66.8 ft.<br />

38. Let x = the distance between the ends of the two<br />

equal sides.<br />

JC<br />

246.75 ~<br />

ft 246.75 ft<br />

x 2 = (246.75)2 +(246.75)2<br />

- 2(246.75)(246.75) cos 125° 12'<br />

x =438.14<br />

The distance between the ends of the two equal<br />

sides is 438 .14 feet.<br />

39. Use the distance formula to find the distances<br />

between the points.<br />

Distance between (-8, 6) and (0, 0) :<br />

~(-8-0)2 +(6-0)2 = 10<br />

Distance between (-8, 6) and (3, 4):<br />

~(-8 - 3)2 + (6 - 4)2 = ../125 = 11.2<br />

Distance between (3, 4) and (0, 0):<br />

~(3-0)2 +(4-0)2 = 5<br />

1<br />

s = -(10+ 11.2 +5)<br />

2 ..<br />

s =13.1<br />

Area = ../--(1-3.-1)-(3-.1)-(1-.9-)(-8.-1)= 25<br />

The area of the triangle is 25 square units.<br />

40. Divide the quadrilateral into two triangles.<br />

The area of the quadrilateral is the sum of the<br />

areas of the two triangles.<br />

Area = ! (65)(130) sin 80° + ! (120)(120) sin 70°<br />

2 2<br />

=11,000 ft 2<br />

41. a=7, b=7,c=6<br />

1<br />

s =- (7 + 7 + 6) =10<br />

2<br />

area = ~r-10(--;-3-:-:-X3-:-:)(---:-4)<br />

= ../360<br />

= 18.97 m 2<br />

18.97 m 2 ... 7.5 m 2 /can ""2.5 cans<br />

He must buy 3 cans.<br />

42. If C = 90°, then<br />

c 2 =a 2 + b 2 - 2abcos90°<br />

=a 2 + b 2 - 2ab(0)<br />

=a 2 +b 2<br />

This is the Pythagorean Theorem.<br />

43. a+ b<br />

44. a-b<br />

45. a + 3c<br />

/<br />

/ \<br />

/ \<br />

/<br />

/ \<br />

\<br />

\<br />

\<br />

\<br />

\<br />

46. (a) true<br />

(b) false<br />

312


<strong>Trigonometry</strong> <strong>Solutions</strong> <strong>Manual</strong><br />

Chapter 7 Review Exercises 313<br />

47. Force~ol.!.SJ~ and 23 Ib, forming an angle of 87° 52. horizontal:<br />

x = Ivlcos 0 =964 cos IS4° 20'<br />

'" 964 cos IS4.333° =869<br />

vertical:<br />

23<br />

y = Ivlsin 0 =964sin IS4° 20'<br />

'" 964 sin IS4.333° = 418<br />

53. u = (21, -20)<br />

15<br />

0= 180°-87° = 93° lui = ~212 + (-20 2)<br />

Use the law of cosines to find Ivl .<br />

= ..J841 =29<br />

Ivl 2 =IS 2 +23 2 - 2(IS)(23)cos 93° a 21<br />

2<br />

cosO=-=­<br />

Ivl =790 lui 29<br />

Ivi =281b 0 =43.6°<br />

But u is in the 4th quadrant, so<br />

48. Forces of 142 newtons and 21S newtons, forming 0= 360°-43.6° = 316.4°.<br />

an angle of 112°<br />

180° --------<br />

- 112° = 68° 54. u = (-9, 12)<br />

68' \<br />

\ lui =~(_9)2 + 12 2<br />

\<br />

\ = ..J22S =IS<br />

\. 8 9<br />

cosO=-=-­<br />

215 newtons lui IS<br />

1u1 2 = 215 2 + 142 2 - 2(215)(142)cos68°<br />

0=126.9°<br />

lui = 209 newtons<br />

55. (8) u,v=(6, 2}'(3, - 2)<br />

49. Forces of 475 Ib and 586 lb, forming an angle of =18-4<br />

78° 20' =14<br />

9<br />

(b) lui = ~62 + 2 2 =.,f40 = 2.JW<br />

Ivl = ~32 +(_2)2 =.,fl3<br />

u·v<br />

cosO = IuIlvl<br />

586lb<br />

o= 180° - 78° 20' = 101° 40' {u.v)<br />

o= arcco lui Ivl<br />

1u1 2 = 475 2 + 586 2 - 2(475)(586) cos IO!° 40'<br />

lui = 826 Ib -cos-t( 14 )<br />

- 2-JlO .:.rrr<br />

50. horizontal: = 52.13°<br />

50 ·../2 -J2<br />

x = IvlcosO = 50cos45° =--= 25 2 56. (8) u -v =(2,,[3 , 2) .(5,5,,[3)<br />

2<br />

vertical: =10,,[3+ 10,,[3<br />

y = IvIsin 0 =50sin 45° = 50 ·../2 = 25../2<br />

2<br />

51. horizontal: x = Ivlcos0 = 69.2 cos 75° = 17.9<br />

vertical: y = Ivlsin 0 = 69.2sin75° = 66.8<br />

= 20,,[3<br />

313


<strong>Trigonometry</strong> <strong>Solutions</strong> <strong>Manual</strong><br />

314 Chapter 7 Applications of <strong>Trigonometry</strong> and Vectors<br />

2<br />

(b) lui= ~(2.J3)2 + 2 =.J16 = 4<br />

Ivl = ~52 +(5.J3t = ../100 = 10<br />

u·v<br />

cosO =IuJIvI<br />

61. Let 0 = the angle that the hill makes with the<br />

horizontal.<br />

D<br />

o= arcco{ 2~~J .<br />

=cos-t~J<br />

=30°<br />

57. lui =~(-4)2 + 3 2 =..fi5 =5<br />

u=;1 =¥=(-~,~)<br />

58. ~JI = ~52 + 12 2 = ../169 = 13<br />

u=~= (5,12) =(2- g)<br />

lui 13 13' 13<br />

59. Let x = the resultant vector.<br />

N<br />

100<br />

'/<br />

/<br />

/<br />

/<br />

x //200<br />

---~//<br />

100<br />

a = 180° -45° = 135°<br />

1x1 2 = 200 2 + 100 2 - 2(200)(100)cos 135°<br />

Ix! = 280<br />

Using the law of cosines again, we get<br />

200 2 -100 2 - 280 2<br />

cosO=-----.,.....-­<br />

-2(100)(280)<br />

0=30.4°<br />

The force is 280 newtons at an angle of 30.4°<br />

with the first rope.<br />

60. Let Ixl = the resultant force.<br />

~I~).._~<br />

.....................x 10.<br />

18<br />

0= 180° -15° _10° = 155°<br />

1x1 2 = 12 2 +18 2 - 2(12)(18)cos155°<br />

Ixl=29<br />

The magnitude of the resultant force on Jessie and<br />

the sled is 29 lb.<br />

C<br />

B 186<br />

The downward force of 2800 Ib has component<br />

AC perpendicular to the hill and component CB<br />

parallel to the hill. AD represents the force of 186<br />

Ib that keeps the car from rolling down the hill.<br />

Since vectors AD and BC are equal, lRel = 186.<br />

Angle B = angle EAB because they are alternate<br />

interior angles so the two right triangles are<br />

similar. Hence, angle 0 = angle BAC.<br />

SID<br />

. B'AC 186<br />

J'\ =-­<br />

2800<br />

HAC =3° 50' =0<br />

The angle that the hill makes with the horizontal<br />

is 3° 50'<br />

62. Let v = the ground speed vector.<br />

N<br />

310'<br />

a = 212° -180° = 32°<br />

f3 =50°<br />

because they are alternate interior angles.<br />

Angle opposite 520 =a + f3 = 82°<br />

sinO sin 82°<br />

--=-­<br />

37 520<br />

0=4°<br />

Bearing = 360° - 50° - 0 = 306°<br />

Angle opposite v = 180° - 82° _4° = 94°<br />

---.!!L=~<br />

sin 94° sin 82°<br />

Ivl= 524 mph<br />

The pilot should fly on a bearing of 306°. Her<br />

actual speed is 524 mph.<br />

314


<strong>Trigonometry</strong> <strong>Solutions</strong> <strong>Manual</strong><br />

Chapter 7 Test Exercises 315<br />

63. Let v =the resultant vector.<br />

!--...130·<br />

! " 7<br />

I<br />

I "' "'<br />

-,<br />

I<br />

I "' "'<br />

-,<br />

I<br />

I<br />

-,<br />

"'<br />

! A 7 v<br />

Angle A =180° - (130° - 90°) =140°<br />

Use the law of cosines to find the magnitude of<br />

the resultant v.<br />

Ivl 2 =15 2 + 7 2 - 2(15}{7)cos 140°<br />

Ivl =21<br />

The resulting speed is 21 kph.<br />

Use the law of sines to find a.<br />

sina sin 140°<br />

--=<br />

7 21<br />

. 7sin140°<br />

sma=--­<br />

21<br />

a=12°<br />

The bearing is 130° - 12° =118°.<br />

64. Component of C along track is ICI cos 8.<br />

Component of weight along track is !wi sin 8 .<br />

We get<br />

IQcos8 = !wisin 8<br />

or tan 8 = /Cl<br />

!wi<br />

.aut<br />

=_7_<br />

65.<br />

1<br />

=­<br />

2<br />

8 = tan- I .5 = 27°<br />

32m<br />

2<br />

v<br />

=­<br />

32r<br />

40<br />

= 2<br />

32(100)<br />

In each of the triangles ABP and PBC we know<br />

two angles and one side. Solve each triangle using<br />

the law of sines.<br />

AB= 92.13sin63° 4' 25"<br />

sin 2° 22' 47"<br />

"" 1978.28 ft<br />

BC = 92.13sin 74° 19' 49"<br />

sin 5° 13' 11"<br />

"" 975.05 ft<br />

Chapter 7 Test Exercises<br />

1. Using the law of sines:<br />

sin 25.2° sin B<br />

=-­<br />

6.92 4.82<br />

. B (sin 25.2°)(4.82)<br />

sm<br />

=-'-----'-'--~<br />

6.92<br />

. _1( Sin 25.2 0 )(4.82»)<br />

B =sm<br />

6.92<br />

== 17.3°<br />

Using the fact that the angles of a triangle sum to<br />

180°:<br />

C= 180-(A+ B)<br />

=180-(25.2+17.3)<br />

= 137.5°<br />

The angle C is 137.5°.<br />

2. Using the law of cosines:<br />

c 2 =a 2 + b 2 - 2abcosC.<br />

= (75)2 + (130)2 - 2(75)(130) cos 118°<br />

"" 31679<br />

c=..}31679<br />

== 180 km<br />

c is approximately 180 km long.<br />

3. Using the law of cosines:<br />

b 2 =a 2 +c 2 -2accosB<br />

2ac cos B =a 2 + c 2 - b 2<br />

a 2 +c 2 _ b 2<br />

cosB=---­<br />

2ac<br />

2 2<br />

_(a 2 +c _b )<br />

B=cos 1<br />

2ac<br />

_1(17.3)2 + (29.8)2 - (22.6)2)<br />

=cos<br />

2(17.3)(29.8)<br />

== 49.0°<br />

B is approximately 49.0° .<br />

315


<strong>Trigonometry</strong> <strong>Solutions</strong> <strong>Manual</strong><br />

316 Chapter 7 Applications or <strong>Trigonometry</strong> and Vectors<br />

4. A=.!.absinc<br />

2<br />

= .!.(75)(130)sin 118°<br />

2<br />

=4300 km 2<br />

(Note : Since c was found in Exercise 2, Heron's<br />

formula can also be used.)<br />

The area of the triangle is approximately<br />

43ookm 2 .<br />

5. Since B > 90°, b must be the longest side of the<br />

triangle.<br />

(8) b » 10<br />

(b) none<br />

(c) b s 10<br />

6. The semi-perimeter s is: I<br />

s=.!.(a+b+c)<br />

2<br />

= .!.(22 + 26 + 40)<br />

2<br />

=44<br />

Using Heron's formula<br />

A =~s(s-a)(s-b)(s-c)<br />

= ~44(44 - 22)(44 - 26)(44 -40)<br />

=.../69696<br />

= 264 square units<br />

The area of the triangle is 264 square units.<br />

7. In any triangle, the sum of the lengths of any two<br />

sides must be greater than the length of the<br />

remaining side.<br />

8. 1v1=~(-6)2+82 =.../100=10<br />

11. u ·v=(-I, 3)·(2, -6)<br />

=-2-18<br />

=-20<br />

12. Find angle C:<br />

C=180-(47° 20'+24° 50')<br />

= 180-(72° 10')<br />

=107° 50'<br />

Use this information and the law of sines to find<br />

AC:<br />

8.4 AC<br />

----=--=---"":'<br />

sin 107° 50' sin47° 20'<br />

AC= 8.4 sin 47.333°<br />

sin 107.833°<br />

== 6.5 mi<br />

Drop a perpendicular line from C to segment AB:<br />

C<br />

B ~ A<br />

8.4mi<br />

then sin 24.833° = ~<br />

6.5<br />

h = (6.5)(sin 24.833°)<br />

z2.7 mi<br />

The balloon is 2.7 miles off the ground.<br />

13. By the Pythagorean Theorem, we know that the<br />

distance from the home plate to second base is<br />

~602 + 60 2 =roJ2 ft<br />

Since cos 0 = iJ!Ji'<br />

o=cos-l(~)=cos-t ~)<br />

=cos-t~)<br />

tanO=.!<br />

x<br />

O=tan-l(~)<br />

1<br />

= tan- ( ~)<br />

z 126.9°<br />

The magnitude /vi is 10 and 0 = 126.9°.<br />

9. u+v={-I, 3}+{2, -6}<br />

=(-1+2), (3+(-6»))<br />

={1, - 3}<br />

10. -3v = -3{2, - 6)<br />

=(-6, 18)<br />

Using the law of cosines:<br />

x 2 =a 2 +b 2 -2abcosO<br />

=(46)2 +(60)2 -2(46X6O)cos45 0<br />

z 1813<br />

x =.../1813<br />

z 43 ft<br />

The distance from the pitcher'S mound to third<br />

base is 43 feet.<br />

316


<strong>Trigonometry</strong> <strong>Solutions</strong> <strong>Manual</strong><br />

14. horizontal : x =IvlcosO=569 cos 127.5° "" -346<br />

vertical: y = IvjsinO = 569 sin 127.5° "" 451<br />

The vector is (-346, 451).<br />

15. Transmitter<br />

C<br />

west -..L...,:~-------+~...L..+- east<br />

Consider the figure.<br />

Since the bearing is 48° from A, the angle A in<br />

6.ABC must be 90° - 48° = 42° . Since the bearing<br />

is 302° from B, the angle B in 6.ABC must be<br />

302° - 270° =32°. The angles of a triangle sum to<br />

180°, so<br />

C = 180 0-(A + B)<br />

= 180° -(42° + 32°)<br />

=106°<br />

Using the law of sines:<br />

b c<br />

--=-­<br />

sinB sinC<br />

b 3.46<br />

--=<br />

sin 32° sin 106°<br />

b = (3.46)(sin 32°)<br />

sin 106°<br />

=1.91 mi<br />

The distance from A to the transmitter is<br />

1.91 miles.<br />

.... .<br />

Chapter 7 Test Exercises 317<br />

317


<strong>Trigonometry</strong> <strong>Solutions</strong> <strong>Manual</strong><br />

318 Chapter 8 Complex Numbers, Polar Equations, and Parametric Equations<br />

CHAPTER 8 COMPLEX NUMBERS,<br />

POLAR EQUATIONS, AND<br />

PARAMETRIC EQUATIONS<br />

Section 8.1<br />

Connections (page 335)<br />

1. A beginning algebra student might have difficulty<br />

with the concept, because no real number<br />

(positive, negative, or zero) has a square that is<br />

negative. The student might wonder "How can a<br />

square be negative"<br />

2. i 4 = 1 since (i 2 )2 = (_1)2 = 1;<br />

.3 . , .3 .2 - 1 - ­<br />

I = -I, sInce I = I - I = - -I = -I<br />

Exercises<br />

1. B; .J:::s =R .$ =i$<br />

2. C; i.J:::s = i -R .$ = i 2 $ = -$<br />

3. D; -i..J=5 = (-i).J=i .$ = (-i)(i).J5<br />

=_i 2 $ =-(-1)$ =$<br />

4. A; -J=5 =-.J=i ...[5 =-i$<br />

5. (a) Imaginary Numbers<br />

(b) Rational Numbers<br />

2=-16<br />

6. x<br />

x=±..J-16<br />

x=±i.J16<br />

x=±4i<br />

The solutions are 4i and -4i.<br />

7. y2 =-36<br />

y=±..J-36<br />

y=±i../36<br />

y=±6i<br />

The solutions are 6i and -6i.<br />

8. z2 +12 =0<br />

z2 =-12<br />

z=±..J-12<br />

z=±i..Jli<br />

z= ±i(2..[3)<br />

z=±2i..[3<br />

The solutions are 2i..[3 and -2i..[3.<br />

9. w 2 +48=0<br />

w 2 =-48<br />

w =±..J-48<br />

w =±i.,J48<br />

w=±i4..[3<br />

w =±4i..[3<br />

The solutions are 4i..[3 and - 4i..[3.<br />

10. 3x 2 +4x+2 =0<br />

Use the quadratic formula with<br />

a =3, b =4, and c =2.<br />

_4±~42 -4.3-2<br />

x =---=----­<br />

2 ·3<br />

-4±..J16-24<br />

x=<br />

6<br />

-4±H<br />

x=<br />

6<br />

-4±i../8<br />

x=<br />

6<br />

2(-2 ±i...fi)<br />

x=<br />

6<br />

-2±i...fi<br />

x=<br />

3<br />

· 2 ...fi· 2...fi.<br />

The so I unons are --+-1 and ----I.<br />

3 3 3 3<br />

11. 2k 2 + 3k =-2<br />

2k 2 +3k+2 =0<br />

Use the quadratic formula with<br />

a =2, b = 3 , and c =2.<br />

-3±~32 -4·2·2<br />

k = -----:~---<br />

2·2<br />

-3±..J9-16<br />

k =......:...:;:....:...:...-....:..­<br />

4<br />

-3±H<br />

k =......:...----:.­<br />

4<br />

k= -3±i.J7<br />

4<br />

- 3.J7. d 3 ..fi .<br />

Th e so I unons are --+-1 an ----I.<br />

4 4 4 4<br />

318


<strong>Trigonometry</strong> <strong>Solutions</strong> <strong>Manual</strong><br />

8.1 Complex Numbers 319<br />

12. m 2-6m+14=0 15. 9a 2 + 7 =6a<br />

Use the quadratic formula with<br />

9a 2 -6a+7 =0<br />

a = I, b = - 6, and c = 14.<br />

Use the quadratic formula with a =9, b =­ 6, and<br />

-(-6)±~(-6i -4 ·1·14<br />

c=7.<br />

m=<br />

2 ·1 -(-6) ± ~(_6)2 - 4·9·7<br />

6±.J-20<br />

a=<br />

m=-"""';""­<br />

2·9<br />

2 6±..J=2l6<br />

6+ i.,fiO<br />

a=<br />

m=<br />

18<br />

2<br />

6(1 ±i../6)<br />

2(3± i../5)<br />

a=<br />

m =---'-----'­<br />

18<br />

2<br />

m =3±i../5<br />

The solutions are 3 +i../5 and 3 - i../5 .<br />

l±i../6<br />

a=-­<br />

3<br />

Th I · 1../6. d l../6.<br />

e so utions are -+-1 an ---I.<br />

3 3 3 3<br />

13. p2+ 4p+ 11=0<br />

Use the quadratic formula with 16. m 2 +1 =-m<br />

a = I, b =4 , and c = II.<br />

m 2 +m+1 =0<br />

-4±~42-4 ·1 ·11 Use the quadratic formula with a = I , b =I,<br />

p<br />

-4+.J-28<br />

p=--'--­<br />

2<br />

2(-2 ± i.,fi)<br />

p=<br />

2<br />

p=-2±i.,fi<br />

2 ·1 and c= 1.<br />

m=<br />

-l±~12_4.1 .1<br />

2 ·1<br />

-1±.J=3<br />

m=-"""";'­<br />

2<br />

-l±i$<br />

The solutions are -2 + i.,fi and - 2 - i.,fi . m=<br />

2<br />

14. 4z 2 =4z - 7<br />

4z 2 -4z+7= 0<br />

Use the quadratic formula with<br />

a =4, b =- 4, and c = 7.<br />

-(-4)±~(-4)2 -4 ·4·7<br />

z=<br />

. 1../3' ·d<br />

The solutions are --+-1 an<br />

2 2<br />

17. i =2y-2<br />

I $. ----I.<br />

2 2<br />

i-2y+2=0<br />

Use the quadratic formula with a = I, b = -2, and<br />

c=2.<br />

2·4<br />

4±.J=%<br />

z=<br />

-(-2)±~(-2)2-4·1 ·2<br />

8 y= 2 ·1<br />

4±i..j%<br />

z=<br />

8<br />

4(1 ±i../6)<br />

z=<br />

8<br />

I ±i../6<br />

z=-­<br />

2<br />

· 1../6. d<br />

Th e so I unons are -+-1 an<br />

2 2<br />

l ../6 .<br />

---I.<br />

2 2<br />

2±R<br />

y=<br />

2<br />

2(1 ±i)<br />

Y=-2­<br />

y= I±i<br />

The solutions are I + i and I - i.<br />

19. .J=3..J=3 =i../3· i../3<br />

=-3<br />

20. -Pi .-Pi =i..fi .i..fi<br />

=i 2 (..fi)2<br />

= (-1)·2 =-2<br />

319


<strong>Trigonometry</strong> <strong>Solutions</strong> <strong>Manual</strong><br />

320 Chapter 8 Complex Numbers, Polar Equations, and Parametric Equations<br />

320


<strong>Trigonometry</strong> <strong>Solutions</strong> <strong>Manual</strong><br />

8.1 Complex Numbers 321<br />

-~<br />

47. (2 - i)(2 + i) =4 + 2; - 2i - i 2<br />

=4-(-1)<br />

53.<br />

4 +; 4+i 6-2i<br />

--=-_._­<br />

6+2i 6+2; 6-2;<br />

=5 or 5+0; 24-2i-2;2<br />

=<br />

Note that the product of conjugates is the 36-4;2<br />

difference of squares.<br />

24 - 2; - 2(-1)<br />

(2 - i)(2 +i) =2 2 =<br />

_;2<br />

36-4(-1)<br />

=4 -(-I) 26-2i<br />

=-­<br />

=5 40<br />

13-;<br />

48. (5+ 4;)(5 - 4i) = 52 - (4;)2<br />

= 25-(-16)<br />

=41+0ior41<br />

49. 5 5 O-i<br />

-=-_._­<br />

0+; O-i<br />

5 -i<br />

=_.­<br />

; -i<br />

SO.<br />

51.<br />

52.<br />

54.<br />

=-­<br />

20<br />

13 1 .<br />

=---1<br />

20 20<br />

3-2; (3-2i) (5-3i)<br />

-­=~---=- ~---=-<br />

5+3; (5+3;) (5-3i)<br />

15-19;+6;2<br />

=-----:::-­<br />

25 -9;2<br />

-5i 9-19i<br />

=~<br />

=-­<br />

-I 34<br />

-5;<br />

9 19.<br />

=-­<br />

=---1<br />

-(-1) 34 34<br />

-5;<br />

=­<br />

1 57. ;12 = (i 4 )3<br />

=-5i =1 3<br />

orO- 5;<br />

=1<br />

-4 -4 0-;<br />

-=-_.-­<br />

58. ;9 =;8. i<br />

= (i4)2 .;<br />

O+i O-i<br />

(-4)(-i)<br />

=<br />

= 1 2 'i<br />

(i)(-i)<br />

= l·i<br />

4i =;<br />

=~<br />

-I<br />

4;<br />

=<br />

1<br />

= 0+4i or 4i<br />

12-8; 12- 8; 5-i<br />

--=-_.­<br />

5+i 5+i 5-i<br />

60 -52i+ 8i 2<br />

=<br />

25- ;2<br />

52-52;<br />

=<br />

26<br />

,. .<br />

59. ;18 = i16.;2<br />

=(i4)4'(_1)<br />

= 1 4 .(- 1)<br />

=1·(-1)<br />

=-1<br />

60.<br />

·99 .96.3<br />

1 = 1 '1<br />

= (i4)24 .;3<br />

= (1)24 .(-;)<br />

=2-2; = 1(-i)<br />

=-;<br />

-13-141 -13-14i 8-3;<br />

=<br />

·-3<br />

8+3; . 8+3; 8-3i 61.<br />

.-4 ·.1<br />

1 = 1 '1<br />

-104 -73; + 42;2<br />

= (;4)-1 . ;<br />

=<br />

64-9i 2<br />

-146-73i<br />

= 1-1.;<br />

=<br />

73 = 1·;<br />

=-2-i =i<br />

321


<strong>Trigonometry</strong> <strong>Solutions</strong> <strong>Manual</strong><br />

322 Chapter 8 Complex Numbers, Polar Equations, and Parametric Equations<br />

....<br />

62. z<br />

·-5<br />

= z<br />

·-8<br />

·z<br />

·3 69. 1= 7 + 5;, E = 28 + 54;<br />

= (;4)-2 .(_;)<br />

E=IZ<br />

E<br />

= 1- 2 .(-i) Z= Ī<br />

= 1·(-;) 28+54;<br />

= -; =<br />

7+5;<br />

28+54; 7-5;<br />

.-10<br />

63.<br />

·- 12 .2<br />

z =z · z =<br />

7+5; 7-5;<br />

= (;4)-3 .(- 1) 196-140; + 378; - 270;2<br />

=<br />

= r 3 .(- 1) 49-25;2<br />

=1 ·(-1) 466+238;<br />

=<br />

=-1 74<br />

233 119.<br />

;-40 = (;4)-10<br />

=-+-z<br />

64. 37 37<br />

= 1- 10 70. E = 35 + 55;, Z = 6 + 4;<br />

=1<br />

E=IZ<br />

35 + 55; = I( 6 + 4;)<br />

65. 1+; +;2 +;3<br />

35+55;<br />

= 1+; +(-1)+(-;) 1= 6+4;<br />

=0<br />

35+55; 6-4;<br />

=<br />

6+4; 6-4;<br />

66. 1+; +;2 +;3 + ... + ;100<br />

210 -140; + 330; - 220;2<br />

=(1+;+;2 +;3)+(;4 +;5 +;6 +;7) =<br />

36-16;2<br />

+(i8 +;9 +;10 +;11)+ ... 430+ 190;<br />

+(i96 + ;97+ ;98+ ;99) + ;100 =<br />

52<br />

Notice that each set of 4 terms within parentheses 215 95.<br />

=-+-z<br />

can be simplified to 26 26<br />

1+; +;2 +;3<br />

71. From the figure in the text. the light bulbs have<br />

= 1+; +(-1)+(-;).= O.<br />

resistances of 50 and 60 ohms and the motors<br />

Then ;100 = (i4)25 = 1 25 = 1.<br />

have reactances of 15 and 17 ohms, respectively.<br />

Therefore,<br />

Thus, the total impedance is given by<br />

1+ i + ;2 + ;3 + ... + ;100 Z = 50+ 60+ 15; + 17;<br />

= 110+ 32;<br />

= 0+0+0+...+0+ 1<br />

=1. 72. From Exercise 71, a = 110 and b = 32. Then<br />

b<br />

67. 1= 8 + 6;, Z =6 + 3; tan6=a<br />

E=IZ 32<br />

= (8 + 6;)(6 + 3;)<br />

tan6=­<br />

110<br />

= 48+24;+ 36; + 18;2 6"" 16.22°.<br />

= 48 + 60; + 18(-1)<br />

=30+60;<br />

68. 1= 10 + 6;, Z = 8 + 5;<br />

E=IZ<br />

E = (1O+6;)(8+5i)<br />

= 80+50;+48; + 30;2<br />

= 80+98;+30(-1)<br />

=50+98;<br />

322


<strong>Trigonometry</strong> <strong>Solutions</strong> <strong>Manual</strong><br />

8.2 Trigonometric (Polar) Form of Complex Numbers 323<br />

73. (~+ ~.J<br />

74.<br />

=( ~)' +2~~ i+(~ i)'<br />

2 4. 2 .2<br />

=-+-1+-1<br />

4 4 4<br />

1 . 1<br />

=-+1+-(-1)<br />

2 2<br />

1 . 1 .<br />

=-+1--=1<br />

2 2<br />

SİDce<br />

root of i.<br />

(..fi..fi .)2 ..J2 ..fi ..<br />

-+-1 =I,-+-Ilsasquare<br />

2 2 2 2<br />

Section 8.2<br />

Connections (page 344)<br />

1. (6, -2}-(-4, -3)<br />

=(6-(-4), -2-(-3)} =(10, I}<br />

or 10+ i<br />

y<br />

-4-3; -4<br />

2. Answers will vary.<br />

Exercises<br />

lO+i<br />

1. The modulus of a complex number represents the<br />

magnitude (or length) of the vector representing it<br />

in the complex plane.<br />

2. It is the angle formed by the vector and the<br />

positive horizontal axis.<br />

3.<br />

y<br />

-2+3;<br />

3<br />

+-t-+-t-='-+-t-+~ JC<br />

-2<br />

4.<br />

y<br />

5<br />

=i<br />

t+++++n'H-H-H~JC<br />

.J3 1. . be f'<br />

Th ere ti ore - - + -I IS a cu root 0 I.<br />

2 2<br />

75. The step ..[::t...[::t= ~( -1)(-1) is invalid, since<br />

the rule ..r;;:b = ~ ..,Jb does not apply when<br />

botha and b are negative.<br />

77. b = 0<br />

5.<br />

y<br />

323


<strong>Trigonometry</strong> <strong>Solutions</strong> <strong>Manual</strong><br />

324 Chapter 8 Complex Numbers and Polar Equations<br />

6.<br />

y 11.<br />

y<br />

++++++d,rt-++-+4~ x<br />

-8<br />

7.<br />

6-5;<br />

y 12.<br />

y<br />

x 0<br />

2<br />

x<br />

8.<br />

-4<br />

y 13 1-4;<br />

14. -4-;<br />

6<br />

15. 4 -3;, -1+2;<br />

x<br />

(4 - 3;)+ (-1 + 2;) =3


<strong>Trigonometry</strong> <strong>Solutions</strong> <strong>Manual</strong><br />

25. 2(cos 45° + i sin 45°)<br />

=2(~ + i~)=~ +;~<br />

8.2 Trigonometric (Polar) Form of Complex Numbers 325<br />

34. 6 cis 135°<br />

= 6(cos 135° + i sin<br />

={-~+~i)<br />

26. 4(cos 60° +; sin 60°) =-~+(~);<br />

=4(~+i~)<br />

= 2 +2;../3<br />

27. 10(cos 90° + i sin 90°)<br />

= 10(0+ i)<br />

=O+lOi= lOi<br />

28. 8(cos 270° + i sin 270°)<br />

=-3~+3;~<br />

35. -Iicis 180°<br />

135°)<br />

=~(cos 1800+; sin 180°)<br />

= ..{i(-1 +;·0)<br />

= -..{i +0;<br />

=-..{i<br />

= 8(0 -Ii) = -8; 36. .../3 cis 315°<br />

29. 4(cos 240° +; sin 240°) =.../3(cos 315°+i sin 315°)<br />

={-~-i~) =../3[~ +{­ ~)]<br />

= -2-2;.../3<br />

30. 2(cos 330° +; sin 330°)<br />

=2(~ -~;)<br />

37.<br />

.J6 ~ .<br />

=---1<br />

2 2<br />

3-3; =x+ yi<br />

Find rand ()<br />

x=3.y=-3<br />

=.../3-;<br />

r=~x2 +y2<br />

~<br />

31. cis 30°<br />

= (cos 30° + i sin 30°)<br />

32.<br />

33.<br />

../3 1.<br />

=-+-1<br />

2 2<br />

3 cis 150°<br />

= 3(cos 150° + i sin 150°)<br />

={- ~+~;)<br />

3..J3 3 .<br />

=--+-1<br />

2 2<br />

5 cis 300°<br />

= 5(cos 300° +; sin 300°)<br />

={~+~)]<br />

5 5.../3 .<br />

=---1<br />

2 2<br />

= ~32 +(_3)2<br />

=.J18<br />

=~<br />

y -3<br />

tan (}=-=-=-1<br />

x 3<br />

3 - 3; is in quadrant IV, so () = 315°.<br />

3-3; = ~(cos 315°+; sin 315°)<br />

38. -2 +2;.../3<br />

Find rand e.<br />

Since x = -2 and y = 2../3,<br />

r=~x2+i<br />

=~(_2)2 +(2../3)2<br />

= ~4+ 12 =.Jl6<br />

=4.<br />

tan () = r = 2../3 =-.J3<br />

x -2<br />

325


<strong>Trigonometry</strong> <strong>Solutions</strong> <strong>Manual</strong><br />

326<br />

Chapter 8 Complex Numbers, Polar Equations, and Parametric Equations<br />

Since - 2 + 2;.J3 is in quadrant II, 6 = 120°.<br />

Thus,<br />

43. -5-5;<br />

x=-5,y=-5<br />

-2 + 2;.J3 =4(cos 120°+; sin 120°). r = ~(_5)2 +(_5)2<br />

39.<br />

40.<br />

41.<br />

42.<br />

-3-3;.J3 =.J50 =5../2<br />

-5<br />

x=-3, y=-3.J3<br />

tan6=-=1<br />

-5<br />

r= ~(_3)2 + (_3.J3)2 = 6 -5 - 5; is in quadrant III, so 6 = 225°.<br />

-3.J3 .J3 -5 - 5; = 5../2(cos 225°+; sin 225°)<br />

tan 6=--= 3<br />

-3<br />

-3-3;.J3is in quadrant III, so 6 = 240°. 44. -J2 +;../2<br />

-3 - 3;.J3= 6(cos 240° +; sin 240°) x =-J2, y =../2<br />

r=~(-J2)2 +(../2)2 =2<br />

1+;.J3<br />

x=l, y=..J3<br />

r= ~12 + (.J3)2 =.J4 =2<br />

../2<br />

tan6=~=-1<br />

-J2 +;../2 is in quadrant II, so 6 = 135°.<br />

-J2 +;../2 = 2(cos 135°+; sin 135°)<br />

tan 6=1..=.J3 =.J3<br />

x 1 · 45. 2+2;<br />

Since 1+;.J3 is in quadrant I, 6 = 60°.<br />

x=2,y=2<br />

1+;.J3 = 2(cos 60° +; sin 60°) r = ~22 + 2 2 =.J8 = 2../2<br />

.J3 -t<br />

2<br />

tan 6 =-= 1<br />

2<br />

x=.J3, y=-1 2 + 2; is in the quadrant I, so 6 = 45°.<br />

r=~(.J3)2 +(_1)2 =2<br />

2+2i =2../2(cos 45~+; sin 45°)<br />

-1 .J3<br />

tan6=13=-3<br />

.J3 -; is in quadrant IV, so 6 = 330°.<br />

46. -J3+;<br />

x=-J3,y=1<br />

.J3-; = 2(cos 330° +; sin 330°) r=~(-J3)2+12 =2<br />

1 .J3<br />

4.J3 +4; tan6==J3=3<br />

x=4.J3,y=4 -J3+ i is in quadrant II, so6 = 150°.<br />

r=~(4.J3)2+42 =...[64 =8 -J3+; = 2(cos 150°+; sin 150°)<br />

4<br />

tan6=4J3 47. -4=-4+0;<br />

4 3<br />

x=-4,y=0<br />

1 .J3<br />

=.J3=3 r=~(-4)2 +0 2 =4<br />

Since 4.J3 + 4; is in the quadrant I, 6 = 30°.<br />

4.J3 + 4; = 8(cos 30° + i sin 30°)<br />

-4 + 0; is on the negative x-axis, so 6 = 180°.<br />

-4 = 4(cos1800+;sinI800)<br />

48. 5; =0+5;<br />

x=0,y=5<br />

r=~02 +5 2 =5<br />

0+5; is on the positive y-axis, so 6 =90°.<br />

5; = 5(cos 90° + i sin 90°)<br />

326


<strong>Trigonometry</strong> <strong>Solutions</strong> <strong>Manual</strong><br />

49. -2;=0 - 2;<br />

x=O,y=-2<br />

r = ~02 +(_2)2 = 2<br />

0-2; is on the negative y-axis so, 0 = 270°.<br />

-2; = 2(cos 270° +; sin 270°)<br />

50. 7 =7+0;<br />

x=7,y=0<br />

r=~72 +0 2 =7<br />

7 + 0; is on the positive x-axis, so 0 = 0°.<br />

7 = 7(cos 0° +; sin 0°)<br />

51. 2+3;<br />

x=2,y=3<br />

r=~22+32=..Jf3<br />

tan8=~<br />

2<br />

2 + 3; is in quadrant I, so 8 = 56.31°.<br />

2 + 3; = ..Jf3(cos 56.31° +; sin 56.31°)<br />

52. cos 35°+ i sin 35°<br />

= .81915204 + .57357644;<br />

53. 3(cos250° +; sin 250°)<br />

= 3[-.34202014+ (-.93969262);]<br />

= -1.0260604 - 2.8190779;<br />

54. -4+;<br />

x=-4,y =1<br />

r =~(_4)2 + 1 =.Jfi<br />

1 1<br />

tan8=-=-­<br />

-4 4<br />

-4+; is in the quadrant II, so 0 = 165.96°.<br />

-4 +; = .Jfi(cos 165.96° +; sin 165.96°)<br />

55. 12; = 0 + 12;<br />

x=0,y=12<br />

r=~02+122=12<br />

o+ 12; is on the positive y-axis, so 0 = 90°.<br />

12; = 12(cos 90° +; sin 90°)<br />

56. 3 cis 180°<br />

= 3(cos 180°+; sin 180°)<br />

= 3(-1+0;)<br />

= -3+0; or -3<br />

8.2 Trigonometric (polar) Form of Complex Numbers 327<br />

57. 3+5;<br />

x=3,y=5<br />

r = ~32 + 52 =.J34<br />

5<br />

tan 0=­ 3<br />

3+5; is in the quadrant I, so 8= 59.04°.<br />

3+ 5; = .J34(cos 59.04°+; sin 59.04°)<br />

58. cis 110.5°<br />

= cos 110.5° + i sin 110.5°<br />

= -.35020738 + .93667219;<br />

59. Since the modulus represents the magnitude of the<br />

vector in the complex plane, r =1 would<br />

represent a circle of radius one centered at the<br />

origin.<br />

60. When graphing x + y; in the plane as (x,y) , if x<br />

and y are equal, we are graphing points in the<br />

form (x, x). These points make up the line y =x.<br />

61. Since the real part of Z = x + yi is 1, the graph of<br />

1+ yi would be the vertical line x = 1.<br />

62. When graphing x + y; in the plane as (x,y) , since<br />

the imaginary part is I, the points are of the form<br />

(x, 1). These points constitute the horizontal line<br />

y =1.<br />

63. z =-.2;<br />

z2 -1 =(-.2;)2 -1<br />

=-.04-1<br />

=-1.04<br />

The modulus is 1.04.<br />

(z2 _1)2 -1 = (-1.04)2 -1 = .0816<br />

The modulus is .0816.<br />

[(z2 _1)2 _1]2 -1 = (.0816)2 -1<br />

=-.9933<br />

The modulus is about .9933.<br />

The moduli do not exceed 2. Therefore, z is in the<br />

Julia set.<br />

64. (b) Let zl =a+b;andz2 =a-b;.<br />

zr -1 =(a + bi)2 -1<br />

= (a 2 - b 2 -1)+(2ab);<br />

=c+d;<br />

z~ -1 =(a - b;)2 -1<br />

= (a 2 - b 2 -1) - (2ab);<br />

=c-d;<br />

The results are again complex conjugates of<br />

each other. At each iteration, the resulting<br />

values from zl and z2 will always be<br />

complex conjugates.<br />

327


<strong>Trigonometry</strong> <strong>Solutions</strong> <strong>Manual</strong><br />

328 Chapter 8 Complex Numbers, Polar Equations, and Parametric Equations<br />

(c) We must show that if (a, b) is on the graph of<br />

the Julia set, then (a, -b) is also on its graph .<br />

Notice that the results after the first<br />

calculation are complex conjugates of each<br />

other. By part (a), they have the same<br />

modulus. If the computation is then repeated<br />

with c + di and c - di from part (b), they will<br />

again produce complex conjugates with the<br />

same modulus.<br />

Either both numbers will be larger than 2 in<br />

modulus or both will be less than or equal to<br />

2 in modulus. Thus, either both (a, b) and<br />

(a, -b) are on the graph of the Julia set<br />

determined by z2 - 1 or both are not.<br />

(d) By part (c), if (a, b) is on its graph then,<br />

a, -b) is also on its graph. This means that<br />

the graph is symmetric with respect to the<br />

x-axis.<br />

(e) We must show that if (a, b) is in the Julia set,<br />

then (-a, b) is also in the set. Test<br />

zi =a + bi and Z2 =-a+ bi.<br />

Zl2 -1 =(a + bi)2_ 1<br />

= (a 2 - b 2 -1) +(2ab)i<br />

=c+di<br />

zl-1 =(-a + bi)2 -1<br />

= (a 2 - b 2 -1) - (2ab)i<br />

=c-di<br />

Again they are complex conjugates which<br />

have the same modulus. By the same<br />

reasoning as above, the graph of the Julia set<br />

is also symmetric with respect to the y-axis.<br />

67. (a+bi)-(c+di)=e+Oi, so,b-d=Oorb=d.<br />

Therefore, the terminal points of the vectors<br />

corresponding to a + bi and c + di lie on a<br />

horizontal line. (B)<br />

Section 8.3<br />

Exercises<br />

1. multiply, add<br />

2. divide; subtract<br />

3. [3cos 60° + i sin 60°)]· [2(cos 90° + i sin 90°)]<br />

=6(cos 150° + isin 150°)<br />

=6(- ~ +i±)<br />

=-M+3i ·<br />

4. [4(cos 30° + i sin 30°)]<br />

· [5(cos 120° + i sin 120°)]<br />

=20(cos 150° + i sin 150°]<br />

=2{-~ +±i)<br />

=-1M+lOi<br />

5. [2(cos45° + isin45°)]<br />

· 2[(cos225 + isin 225°)]<br />

=4(cos 270° + i sin 270°)<br />

=4(0-i)<br />

= 0-4i or -4i<br />

6. [8(cos3000+isin3000)]<br />

·[5(cos 120° + isin 120°)]<br />

=40(cos420° + i sin 420°)<br />

=40(cos 60° + i sin 60°)<br />

=41J.. + i .,[3)<br />

'\2 . 2<br />

=20 + 20i.,[3<br />

68. The modulus of a complex number is equal to the<br />

length of the vector representing it in the complex<br />

plane. Therefore, the only way for the modulus of<br />

the sum of two complex numbers to be equal to<br />

the sum of their moduli is for the two vectors<br />

representing the numbers to have the same<br />

direction. (C)<br />

7.<br />

[4(cos6O° + i sin 60°)]· [6(cos 330° + i sin 330°)]<br />

=24(cos 390°+ i sin 390°)]<br />

=24(cos 30° + i sin 30°)<br />

=24( ~ +i±)<br />

69. The difference of the vectors is equal to the sum<br />

of one of the vectors plus the opposite of the other<br />

vector. Ifthis sum is represented by two parallel<br />

vectors the sum of their moduli will equal to<br />

modulus of their sum. In order for this sum to be<br />

represented by two parallel vectors their<br />

difference must be represented by two vectors<br />

with opposite directions. (A)<br />

=12.,[3 + 12i<br />

8. [8(cos2100+ isin2100)]<br />

. 2(cos330° + isin 330°)]<br />

=16(cos 540° + i sin 540°)<br />

=16(cos 180° +isinI800)<br />

= 16(-1+0·i)<br />

=-16+0i or -16<br />

328


<strong>Trigonometry</strong> <strong>Solutions</strong> <strong>Manual</strong><br />

8.3 The Product and Quotient Theorems 329<br />

9. [5 cis 90°][3 cis 45°] 16. 24(cos 150° + isin 150°)<br />

= 15 cis 135° 2(cos 30° + i sin 30°)<br />

== 15(cos 135° + i sin 135°) = 12(cos1200 + isin 120°)<br />

=1{-'7 + '7 i)<br />

=1{-~+i~)<br />

15.fi 15.fi. =-6+6i../3<br />

=---+--1<br />

2 2<br />

17. 3cis 305°<br />

10. [6 cis 120°][5 cis(-300)] 9 cis 65°<br />

= 30 cis 90°<br />

=.!. cis 240°<br />

= 30(cos 90° + i sin 90") 3<br />

= 30(0+ Ii)<br />

=.!. (cos 240° + i sin 240°)<br />

=0+30i<br />

Of 30i<br />

3<br />

=~(-~- ~ i)<br />

11. [../3 cis 45°][ .J3 cis 225°]<br />

= 3 cis 270° 1 ../3.<br />

=----1<br />

= 3(cos 270° + i sin 270°) 6 6<br />

=3(O-i)<br />

=0-3i or -3i 12 cis 293°<br />

18.<br />

6cis 23°<br />

12. [.fi cis 3000][.J2 cis 270°] =2 cis 270°<br />

= 2(cos 270° + i sin 270°)<br />

= 2 cis 570°<br />

=2(0-lrl<br />

=2 cis 210° = 0-2i or -2i<br />

= 2(cos 210° + isin2100)<br />

19. numerator: r =~82 + 0 2 = 8<br />

=2(-;: -~i)<br />

tan 0 =~<br />

=-./3-t<br />

13. 4(cos 120° + i sin 120°)<br />

8<br />

0=0°<br />

denominator: r =~(../3)2 + 1 2 =2<br />

2(cos 150° + i sin 150°) tan 0 =13<br />

=2[cos(-300) + i sin(-300)]<br />

0=30°<br />

8 _ 8cisOo _ 4 . ( 31\0)<br />

=2(~ -~i) --- - CIS - v<br />

../3+i 2 cis 30°<br />

=../3-i<br />

=4[cos(-300) + isin(-300)]<br />

IO(cos225° + isin225°)<br />

14. =4[~ +{-~)]<br />

5(cos 45° + i sin 45°)<br />

=2(cosI80° + isin 180°)<br />

=2../3 -2i<br />

=2(-1 +O·i)<br />

=-2+0i<br />

or -2<br />

15. 16(cos3000+ isin 300°)<br />

8(cos 60° + i sin 60°)<br />

=2(cos 240° + i sin 240°)<br />

=2(-~- ~ i)<br />

=-I-i../3<br />

1<br />

329


<strong>Trigonometry</strong> <strong>Solutions</strong> <strong>Manual</strong><br />

330 Chapter 8 Complex Numbers, Polar Equations, and Parametric Equations<br />

20. numerator: r = ~02 + 2 2 = 2<br />

tan 0 = ~ is undefined<br />

o<br />

0=90°<br />

denominator: r = ~r-(_-1-)2-+-(-./3-3-)-2 = 2<br />

2i _<br />

---;= ­<br />

-1-i..[3<br />

tan 0 = --J3<br />

-1<br />

o= 60° + 180°= 240°<br />

2cis90° _ . ( 1500)<br />

- CIS ­<br />

2 cis 240°<br />

= cos(-1500)+ isin(-1500)<br />

..[3 1.<br />

=----1<br />

2 2<br />

21. numerator: r = ~02 + (_1)2 = 1<br />

tan 0 = -1 is undefined<br />

o<br />

0= 90° + 180° = 270°<br />

denominator: r = ~1 2 + 1 2 =.J2<br />

tanO=!<br />

1<br />

0=45°<br />

~ = cis 270° =.J2 cis 2250<br />

l+i .J2cis 45° 2<br />

22. numerator: r = ~12 + 0 2 = 1<br />

=.J2 (cos225° + i sin 225°)<br />

2<br />

=.J2 (_ .J2-i .J2)<br />

222<br />

1 1.<br />

=----1<br />

2 2<br />

tan0 =Q<br />

1<br />

0=0°<br />

denominator: r = ~22 + (_2)2 = 2.J2<br />

-2<br />

tanO=­<br />

2<br />

0=-45°<br />

... 1 cis0° .J2 . 450<br />

--= =-CIS<br />

2 - 2i 2.J2 cis(-45°) 4<br />

=.J2 (cos45° + i sin 45°)<br />

4<br />

=.J2(.J2 +i.fi)=.!.+!.i<br />

4 2 2 4 4<br />

330


<strong>Trigonometry</strong> <strong>Solutions</strong> <strong>Manual</strong><br />

8.3 The Product and Quotient Theorems 331<br />

23. numerator: r = ~(2.,J6)2 +(-2../2)2 = 4../2<br />

-2../2 1<br />

tan9=--=-­<br />

2.,J6 ../3<br />

9 =-30°<br />

denominator: r = ~r-(../2-2-)-2-+-(-./6-6-)-2 = 2../2<br />

-./6<br />

tan 9 = .J2 =-J3<br />

9=-60°<br />

2.,J6 -2i.J2 = 4..[2cis(-300) = 2 cis 30°<br />

.J2 - i.,J6 2../2 cis (-600)<br />

= 2(cos 30° + i sin 30°)<br />

=2( ~ +i~)<br />

=../3+i<br />

24. numerator: r = ~4 2 + 4 2 = 4../2<br />

4<br />

tan9=-=1<br />

4<br />

9=45°<br />

denominator: r = ~'22=-+-(-_-2)-=-2 = 2../2<br />

-2<br />

tan9=-=-1<br />

2<br />

9 = -45°<br />

4 + 4i = 4../2 cis 45° = 2 cis 900<br />

2-2i 2../2 cis (-45°)<br />

= 2(cos 90° + i sin 90°)<br />

= 2(0+i)<br />

=2i<br />

25. [2.5(cos35°+ isin35°)] .[3.0(cos500 + isin500)] = 7.5(cos85° + isin85°)<br />

= .65366807 + 7.4714602i<br />

26. [4.6(cosI2°+ isin12°)] ·[2.O(cos13° + isin 13°)] = 9.2(cos 25° + i sin 25°)<br />

= 8.3380316 + 3.8880880i<br />

27. (12cis 18.5°)(3cis 12.5°) = 36cis 31°<br />

= 36(cos 31°+isin31°)<br />

= 30.858023+ 18.54137li<br />

28. (4 cis 19.25°)(7 cis 41.75°) = 28 cis 61°<br />

= 28(cos61 ° + isin61°)<br />

= 13.574669 + 24.489352i<br />

45(cosl27°+ isinl27°) 2( 840 . , 840)<br />

29• = cos +Ism<br />

22.5(cos43° + i sin 43°)<br />

= .20905693 + 1.9890438i<br />

30 30(cos 130°+ isin 130°) 3( 1090 . , 1090)<br />

• = cos +Ism<br />

IO(cos21° + isin 210)<br />

= -.97670446 + 2.8365557i<br />

331


<strong>Trigonometry</strong> <strong>Solutions</strong> <strong>Manual</strong><br />

332 Chapter 8 Complex Numbers, Polar Equations, and Parametric Equations<br />

36. 2 cis 0° = 2(cos 0° + i sin 0°)<br />

31. [2 cis 5;r= [2 cis 5;12 cis 5;] = 2(1+ ;(0»<br />

=2(1)<br />

. IOlC<br />

=4 CIS- =2<br />

9 The result here is the same as the result in<br />

Exercise 33.<br />

= 4(cos l~lC + isin l~lC)<br />

= -3.7587705 -1.3680806i w -1+i<br />

37. -=-­<br />

z<br />

-1-i<br />

-1+i -1+i<br />

=-_._­<br />

-1-; -1+i<br />

l-i - i+ i 2<br />

=--.....,....­<br />

1-;2<br />

-2i<br />

=­<br />

2<br />

=-i<br />

In Exercises 33-39, w = -1 + i and z = -1 - i.<br />

33. w . z =(-1 + ;)(-1 - i)<br />

=(-1)(-1) + (-I)(-i)<br />

+(i)(-I) +(i)(-i)<br />

=1+;-i-i 2<br />

=1-(-1) =2<br />

39. cis (-90°)<br />

34. w=-I+i =-i<br />

r = ~r--(--1)-=-2+-1--=-2 =.J2<br />

= cos(-90") + i sin(-900)<br />

=O+i(-I)<br />

The result here is the same as the result in<br />

Exercise 37.<br />

1<br />

tan(}=-=-1<br />

-1 43. E = 8(cos 20° + i sin 20°), R = 6,<br />

() =135° X L=3<br />

w =.,fi cis 135°<br />

z=-I-i<br />

r = ~,-(_-1)~2-+-(_-1)~2 =../2<br />

I=E,Z=R+XLi<br />

Z<br />

Write Z = 6 + 3i in trigonometric form.<br />

-1 X= 6,y = 3, so r =~62 +3 2 =.J45.<br />

tan(}=-=1<br />

-1 3 1<br />

tan(} = - = - so () = 26.6°<br />

(}=225°<br />

6 2'<br />

z = .J2 cis 225°<br />

Z = .J45 cis 26 .6°<br />

1= 8 cis 20°<br />

35. w· z = (../2 cis 135°)(../2 cis 225°) J45 cis 26.6°<br />

= (.J2)(../2) cis (135° + 225°) = 1.19 cis (-6.6°)<br />

= 2 cis 360° .... 1.18-.14;<br />

=2cisO°<br />

332


<strong>Trigonometry</strong> <strong>Solutions</strong> <strong>Manual</strong><br />

44. E=12(cos25°+;sin25°), R=3, Section 8.4<br />

XL =4, and Xc == 6<br />

1. [3(cos300+isin300)]3<br />

1= E<br />

R+(X L -Xc);<br />

8.4 Powers and Roots of Complex Numbers 333<br />

= 3 3 [cos(3)(300)+;sin(3)(300)]<br />

12 cis 25° = 27(cos 90° + ;sin 90°)<br />

=<br />

3+(4-6);<br />

=0+27; or 27;<br />

12 cis 25°<br />

= 2. [2(cos 135° + ; sin 135°)]4<br />

3-2;<br />

=2 4[cos(4 .135°)+isin(4 · 135°)]<br />

Write 3 - 2; in trigonometric form. x =3, y =-2. = 16(cos 540° + isin 540°)<br />

== 16(-1+0 ·;)<br />

so r =~32 +(_2)2 =.Jl3.<br />

=-16 + 0; or - 16<br />

2<br />

tan 0 =--, so 8 = 326.31°.<br />

3 3. (cos 45° +; sin 45°)8<br />

Continuing to find the current, we have<br />

= [cos(45°)(8) +; sin(8)(45°)]<br />

1= 12 cis 25°<br />

.Jl3 cis 326.31°<br />

=1+0; or 1<br />

=3.3282 cis (-301.31°)<br />

"'" 1.7+ 2.8;.<br />

4. [2(cos 120° +;sin 120°)]3<br />

= 8(cos 360° + i sin 360°)<br />

45. Since zl =50 + 25; and Z2 =60 + 20;, it follows =8(1+0·i)<br />

that<br />

= 8+0; or 8<br />

1 1 50 - 25i 50 - 25;<br />

-= =<br />

ZI 50+25; 50-25i 3125<br />

5. [3 cis 100°]3<br />

2 1. = 3 3 cis (3 ·100°)<br />

=---1;<br />

125 125<br />

=27 cis 300°<br />

1 1 60 - 20; 60 - 20; = 27(cos 300° + i sin 300°)<br />

-= =<br />

Z2 60 + 20; 60 - 20; 4000<br />

3 1.<br />

=27(~- ~ i)<br />

=---1.<br />

200 200 27 27.J3 .<br />

' - - - - 1<br />

11[2 I'J[3 I'J 2 2<br />

ZI + Z2 = 125 - 125 1 + 200 - 200 1<br />

31 13 . 6. [3 cis 40°]3<br />

=-----1<br />

1000 1000 = 3 3 cis (3.40°)<br />

1<br />

Z=-­ =27 cis 120°<br />

= 27(cosI20° + isinl200)<br />

*+*<br />

1<br />

- 1660 -1&10;<br />

=27(-~+ ~;)<br />

1000 31+13; 27 27.J3.<br />

= =--+--1<br />

31-13; 31+ 13; 2 2<br />

31,000+ 13,oooi<br />

==<br />

1130<br />

3100 1300.<br />

=--+--,<br />

113 113<br />

"'" 27.43 + 11.5;<br />

46. tan 0 = J..!2.<br />

27.43<br />

8 = tan- l ...!..!.2- == 22.75°<br />

27.43<br />

333


<strong>Trigonometry</strong> <strong>Solutions</strong> <strong>Manual</strong><br />

334 Chapter 8 Complex Numbers, Polar Equations, and Parametric Equations<br />

7. (...[3 + i)5<br />

x =...[3, Y = 1<br />

r=~(...[3)2+12 =2<br />

tan 8 = JJ so 8 = 30°.<br />

(...[3 + i)5<br />

= [2(cos30° + isin 30°)]5<br />

= 2 5(cos 150°+ i sin 150~<br />

=32(- ~ +i~)<br />

= -16...[3+ 16i<br />

8. (2..{i - 2i..{i) 6<br />

x= 2..{i, Y = -2..{i<br />

r = ~(2..{i)2 + (-2..{i) 2 = .J8+8 = 4<br />

-2..{i<br />

tan 8 =--zJ2" =-1 so 8=-45°.<br />

2 2<br />

(2..{i -2i..{i)6<br />

= [4(cos(-45°) + i sin(-45o)]6<br />

= 4 6 (cos(-270, + isin(-2700)]<br />

=4 6(0+I·i)<br />

= 0+4 6i<br />

= 0+4096i<br />

or 4096i<br />

9. (2 - 2i...[3)4<br />

r=~22+(-2...[3)2 =4<br />

-2...[3 -fj<br />

tan6=--= 3 so 8=300°.<br />

2<br />

(2 -2i...[3)4<br />

= [4(cos 300° + i sin 300°)]4<br />

= 4 4 (cos1200° + isin 1200°)<br />

{ 1 i...[3)<br />

=25 -2'+2<br />

= -128+ 128;...[3<br />

10.<br />

(-;--;J<br />

..fi<br />

..{i<br />

x=T' Y=T<br />

r=~~+~ = 1<br />

tan 8 = -1 so 8 = -45°<br />

(-;--; I)'<br />

= [1(cos(-45°)+ isin(-45°))]8<br />

= 1 8 (cos(- 360°)+ i sin(-360°))<br />

= 1(1 +0 ·i)<br />

=1+0ior 1<br />

11. (-2-2i)5<br />

r = ~(_2)2 +(_2)2<br />

=../8 = 2..{i<br />

-2<br />

tan8=-=1 so 8=225°.<br />

-2<br />

(-2-2i)5<br />

= (2..{i)5(cos225° + i sin 225°)5<br />

= 32.[32(cos1125°+ i sin 1125°)<br />

= 128..{i(cos45° + i sin 45°)<br />

=1284 ~+ ~i)<br />

= 128+128i<br />

12. (-I+i)7<br />

x=-I, y= 1<br />

r=~(-1)2+12 =...fi<br />

1<br />

tan 8 =-=-1 so 8= 135°.<br />

-1<br />

(_I+i)7<br />

= [..{i(cos135°+ isin 135°)f<br />

= 8...fi(cos 945° + i sin 945°)<br />

= s..J2(cos225° +;sin 225°)<br />

=8...fi<br />

(-2-'2<br />

..{i . ...fi)<br />

=-8-8i<br />

334


<strong>Trigonometry</strong> <strong>Solutions</strong> <strong>Manual</strong><br />

8.4 Powers and Roots of Complex Numbers 335<br />

13. (cos 0° + i sin 0°)<br />

= l(cosO° + isin 0°)<br />

r= 1,0=0°, n=3<br />

r 1 / 3 =1 1 / 3 =1<br />

0+36O° ·k<br />

a=-....:..:..-~<br />

n<br />

y<br />

I<br />

t----:¥=--------1- x<br />

- I<br />

-I<br />

15. 8 cis 60°<br />

r =8, 0 =60°,<br />

r 1l3 = 8 113 = 2<br />

n =3<br />

a = 600+36O·k<br />

3 ; k = 0, I, 2<br />

So the cube roots are<br />

(cos 0° + i sin 0°),<br />

(cos 120° + isin 120°),<br />

(cos 240° + i sin 240°).<br />

y<br />

a = 20°, 140°, 260°<br />

So the cube roots are<br />

2 cis 20°.<br />

2 cis 140°,<br />

2 cis 260°.<br />

y<br />

~-+-¥~-+-_ x<br />

--;f---'+--*,=,..- x - 2 2<br />

-I<br />

-I<br />

- 2<br />

14. Find the cube roots of<br />

16. 27 cis 300°<br />

(cos 90° + i sin 90°).<br />

r=27, 0=300°, n=3<br />

r= 1,0=90°, n=3 r 1 / 3 =27 1 / 3 =3<br />

r 1/3= 1 113 =1<br />

300° + 360° . k<br />

a= 3 ; k =0, 1, -2<br />

0+360° ·k<br />

a=--"":":""":":" a = 100°, 220°, 340°<br />

n<br />

The cube roots are<br />

90 0+36O°·k 3 cis 100°,<br />

= 3 k =0, I, 2<br />

3 cis 220°,<br />

Ifk=O, a= 90°+360° ·0 =30°.<br />

3 cis 340°.<br />

y<br />

3<br />

90°+ 360°·1<br />

If k =1, a = 3 = 150° .<br />

Ifk=2, a= 90°+360° ·2 =270°<br />

3 . --::t-+-+~d--+-+-"'- X<br />

- 3 3<br />

The cube roots are<br />

(cos 30° + i sin 30°),<br />

(cos 150° + i sin 150°), - 3<br />

(cos 270° + i sin 270°) .<br />

335


<strong>Trigonometry</strong> <strong>Solutions</strong> <strong>Manual</strong><br />

336 Chapter 8 Complex Numbers, Polar Equations, and Parametric Equations<br />

17. -8; = 8(cos 270° + i sin 270°)<br />

r = 8, 8 = 270°, n = 3<br />

l /3<br />

=8 1/ 3 r =2<br />

270° + 360° . k<br />

a = 3 ; k = 0, 1, 2<br />

a = 90°, 210°, 330 0<br />

So the cube roots are<br />

2(cos 90° + i sin 90°),<br />

2(cos 210° + i sin 210 0 ) ,<br />

2(cos 330° + i sin 330 0 ) .<br />

y<br />

2<br />

~-+-_:-----l--+:--x<br />

-2 2<br />

-2<br />

18. 27; = 27(cos 90° +; sin 90 0 )<br />

r=27, 8=90 0 , n=3<br />

ll3<br />

=27 1/ 3 r =3<br />

90 0+36O°·k<br />

a= 3 ;k=0,I,2<br />

a = 30 0 , 150 0 , 270°<br />

So the cube roots are<br />

3(cos 30° + i sin 30 0 ) ,<br />

3(cos 150 0 + i sin 150 0 ) ,<br />

3(cos 270 0 + i sin 270 0 ) .<br />

y<br />

3<br />

y<br />

20. 27 =27(cos 0 0 + i sin 0°)<br />

r = 27, 8 = 0 0 , n =3<br />

r l/ 3 =271/3 =3<br />

00+36O°·k<br />

a= k=O, 1, 2<br />

3<br />

a = 0°, 120°, 240°<br />

The cube roots are<br />

3(cos 0 0 + i sin 0 0 ) ,<br />

3(cos 120 0 + i sin 120°),<br />

3(cos 240 0 + i sin 240 0 ) .<br />

y<br />

3<br />

+-+-+--J::o-+....~- x<br />

-3 3<br />

21. I+;..fj<br />

-3<br />

r=~12+(..fj)2 =2<br />

+-+-+~~-+-1_ x<br />

tan 8 = ~ so 8=60 0 •<br />

1<br />

-3<br />

19. -64 = 64(cos 180 0 + i sin 180 0 )<br />

r=64, 8=180°, n=3<br />

ll3<br />

=64 1/ 3 r =4<br />

180° + 360 0 • k<br />

a = 3 ; k =0, 1, 2<br />

a = 60 0 , 180 0 , 300 0<br />

So the cube roots are<br />

4(cos 60 0 + i sin 60°),<br />

4(cos 180 0 + i sin 180°),<br />

4(oos 300 0 + i sin 300°).<br />

1+;..fj = 2(cos 60° + i sin 60 0 )<br />

ll3 1/ 3<br />

r =2 ='Jfi<br />

60 0+360°·k<br />

a= k =0, 1, 2<br />

3<br />

a = 20°, 140 0 , 260 0<br />

The cube roots are<br />

V2(cos200+ isin200)<br />

V2(cos 140° + i sin 140°)<br />

V2


<strong>Trigonometry</strong> <strong>Solutions</strong> <strong>Manual</strong><br />

8.4 Powers and Roots of Complex Numbers 337<br />

2<br />

-2 2<br />

y<br />

The cube roots are<br />

:v4(cos500+ isin500),<br />

:v4(cos 170° + isin 170°),<br />

:v4(cos290° + i sin 290°).<br />

y<br />

2<br />

- 2<br />

22. 2 - 2i../3<br />

r = ~r-22-+-(--2-../3-3)-2 = 4<br />

tan 8 = - 2../3 =--J3<br />

2<br />

8 = 300° (quadrant IV)<br />

ll 3<br />

r<br />

=:v4<br />

300° + 360° . k<br />

a = ; k =0, I, 2<br />

3<br />

a = 100°, 220°, 340°<br />

The cube roots are<br />

:v4(cos 100° + isin 100°),<br />

:v4(cos 220° + i sin 220°),<br />

:v4(cos 340° + i sin 340°).<br />

y<br />

2<br />

- 2 2<br />

-+-~~-4-+-I-+- .r<br />

24. ../3 - i<br />

-2<br />

r = ~r-(../3-3-:")2-+-(_-1)-=-2 = 2<br />

tan 8 = - ..[3 , 8 = 330°<br />

3<br />

..[3 - i = 2(cos 330° + isin 330°)<br />

l / 3<br />

r<br />

=Vi<br />

a = 330° + 360° .k; k = 0, I, 2<br />

3<br />

a = 110°, 230°, 350°<br />

The cube roots are<br />

Vi(cos 110° + isin 110°),<br />

Vi(cos230° + isin 230°),<br />

Vi(350° + i sin 350°).<br />

y<br />

2<br />

-2<br />

23. -2..[3 +2i<br />

r = ~r-(--2..[3--3--)2-+-2-:"'2 = 4<br />

tan 8 = 2 M so 8 = 150° .<br />

-2"\13<br />

-2..[3 + 2i = 4(cos 150° + isin 150°)<br />

ll 3 = 4<br />

1/ 3<br />

r = :v4<br />

150° + 360° . k<br />

a = k =0, I, 2<br />

3<br />

a = 50°, 170°, 290°<br />

-2<br />

25. Find all the second (or square) roots of 1.<br />

1= l(cosOO+ isin 0°)<br />

r=I, 8=0°, n=2<br />

r 1l2 =11/2 = 1<br />

00+36O°·k<br />

a =<br />

k =0, J<br />

2<br />

a =0°, 180°<br />

The second roots of 1 are<br />

(cos 0° + i sin 0°),<br />

(cos 180° + i sin 180°).<br />

337


<strong>Trigonometry</strong> <strong>Solutions</strong> <strong>Manual</strong><br />

338 Chapter 8 Complex Numbers, Polar Equations, and Parametric Equations<br />

y 28. Find all the eighth roots of 1.<br />

I = cos 0° + i sin 0°<br />

r = I, () = 0, n = 8<br />

00+360° ·k<br />

a=----­<br />

8<br />

k=O, 1,2,3,4,5,6,7<br />

a =0°, 45°, 90°, 135°, 180°, 225°<br />

270°, 315°<br />

The eighth roots of I are<br />

26. Find all the fourth roots of I. (cos 0° + i sin 0°),<br />

1 =cos 0° + i sin 0° (cos 45° + i sin 45°),<br />

r=I, (}=Oo, n=4 (cos 90° + i sin 90°),<br />

r l / 4 = 1 114 (cos 135° + i sin 135°),<br />

= 1<br />

(cos 180°+ i sin 180°),<br />

00+360° ·k<br />

a = 4 ; k =0, 1, 2, 3<br />

(cos 225° + i sin 225°),<br />

(cos 270° + i sin 270°),<br />

a =00, 90°, 180°, 270° (cos 315° + i sin 315°).<br />

The fourth roots of 1 are<br />

y<br />

1<br />

(cos 0° + i sin 0°),<br />

(cos 90° + i sin 90°) ,<br />

(cos 180° + i sin 180°),<br />

(cos 270° + i sin 270°).<br />

y<br />

1<br />

*"__ ~ __~~ x<br />

-I o<br />

27. Find alI the sixth roots of 1.<br />

1 =1(cos 0° + i sin 0°)<br />

r = 1, () = 0°, n = 6<br />

-1<br />

r l/6 = 1 1/6 = 1<br />

00+360°·k<br />

a=---­<br />

6<br />

k = 0,1,2,3,4,5<br />

a =0°, 60°, 120°, 180°, 240°, 300°<br />

The sixth roots of 1 are<br />

(cos 0° + i sin 0°),<br />

(cos 60° + i sin 60°),<br />

-1<br />

29. Find all the second (square) roots of i.<br />

i = cos 90° + i sin 90°<br />

r = 1, () = 90°, n = ~<br />

r 1l2 =1 11 2 =1<br />

(cos 120° + i sin 120°),<br />

(cos 180° + i sin 180°),<br />

(cos 240° + i sin 240°),<br />

-I<br />

(cos 300° + i sin 300 °).<br />

y<br />

90 0+360°·k<br />

a=<br />

k =0, 1<br />

2<br />

a= 45°, 225°<br />

The second roots of i are<br />

(cos 45° + i sin 45°),<br />

(cos 225° + i sin 225°).<br />

y<br />

-+--~


<strong>Trigonometry</strong> <strong>Solutions</strong> <strong>Manual</strong><br />

30. Find all the fourth roots of i.<br />

i =cos 90° + i sin 90°<br />

r =1. (J=90°. n =4<br />

90°+360° ·k<br />

a = ; k =0, I, 2, 3<br />

4<br />

a =22.5°, 112.5°, 202.5°, 292 .5°<br />

The fourth roots of i are<br />

(cos 22.5° + i sin 22.5°),<br />

(cos 112.5° + i sin 112.5°),<br />

(cos 202.5° + i sin 202.5°),<br />

(cos 292.5°.+ i sin 292.5°).<br />

, y<br />

1<br />

8.4 Powers aod Roots of Complex Numbers 339<br />

33. x 3 +i=O<br />

x 3 =-i<br />

=O-i<br />

= 1(cos 270° + i sin 270°)<br />

11/3 =1<br />

270° + 360° . k<br />

a= 3 ;k=O,I,2<br />

a = 90°, 210°, 330°<br />

x =(cos 90° .+ i sin 90°) ,<br />

(cos 210° + i sin 210°) ,<br />

(cos 330° + i sin 330°)<br />

34. x 4 +i =0<br />

4 .<br />

X =-1<br />

=O-i<br />

+------:::*::......---t.. x<br />

=1(cos 270° .+i sin 270°)<br />

31. x 3 -1=0<br />

- I<br />

x 3 =1<br />

= 1+0i<br />

=l(cosoo + isinOO)<br />

The modulus of the solutions is<br />

1 1 / 3 = 1.<br />

The arguments of the solutions are<br />

00+360° ·k<br />

a = ' ; k =0, I, 2.<br />

3<br />

Ifk =0, 0° + 360° . 0 = 00.<br />

3<br />

Ifk =I, 0° + 360° . 1 = 1200.<br />

3<br />

1 1 / 4 =1<br />

270° + 360° . k<br />

a = " k =0, 1, 2, 3<br />

·4<br />

x =(cos 67.5° + i sin 67.5°),<br />

(cos 157.5° + i sin 157.5°),<br />

(cos 247 .5° + i sin 247.5°),<br />

(cos 337 .5° + i sin 337 .5°)<br />

35. x 3 -8=0<br />

x 3 =8<br />

=8+0i<br />

=8(cosO° + isinOO)<br />

81/3 =2<br />

00+360°·k<br />

a = 3 ; k =0, I, 2<br />

a = 0°, 120°, 240°<br />

x = 2(cosO° + isinOO),<br />

2(cos 120° + i sin 120°),<br />

2(cos 240° + i sin 240°)<br />

Ifk = 2, 0°+360° ·2 = 2400.<br />

3<br />

x = (cos 0° + isinOO),<br />

(cos 120° + i sin 120°),<br />

36.<br />

x 3 +27 =0<br />

(cos 240° + isin 240°) x 3 =-27<br />

32. x 3 +1 =0<br />

=-27+0i<br />

=27(cos 180° + i sin 180°)<br />

x 3 =.:...1<br />

=-1+0i<br />

27 1 / 3 =3<br />

a = 180 0+360° ·k . k =0 1 2<br />

=l(cos 180° + i sin 180°) 3 ' "<br />

The modulus of the solutions is a = 60°,180°,300°<br />

1 1 / 3 =1. x = 3(cos60° + isin600)<br />

The arguments of the solutions are 3(cos 1800 + i sin 180°)<br />

a =1800+ 36QO · k ; k = 0, I, 2. 3(cos 300° .+i sin 300°)<br />

3<br />

a =60°, 180°, 300°<br />

x =(cos 60° + i sin 60°),<br />

(cos 180° + i sin 180°),<br />

(cos 300° + i sin 300°)<br />

339


<strong>Trigonometry</strong> <strong>Solutions</strong> <strong>Manual</strong><br />

340 Chapter 8 Complex Numbers, Polar Equations, and Parametric Equations<br />

37. x 4 +1 =0<br />

x 4 =-1<br />

=-1+0;<br />

= l(cos 180° + i sin 180°)<br />

4<br />

1 1 / =1<br />

0+360°<br />

a= 180 ·k . k =0 I 3<br />

4 ' " 2,<br />

a = 45°, 135°, 225°, 315°<br />

x = (cos 45° +;sin 45°),<br />

(cos 135° + i sin 135°),<br />

(cos 225° + i sin 225°),<br />

(cos315° + ;sin 315°)<br />

38. x 4 +16 =0<br />

4<br />

x =-16<br />

=-16+0i<br />

= 16(cos 180° + isin 180°)<br />

161/4 =2<br />

180° + 360°· k<br />

a= 4 ;k=0,1,2,3<br />

a = 45°, 135°, 225°, 315°<br />

x = 2(cos 45° + i sin 45°),<br />

2(cos 135° + i sin 135°),<br />

2(cos 225° + i sin 225°),<br />

2(cos 315° + i sin 315°)<br />

39. x 4 _ ; = 0<br />

x 4 =i<br />

=O+i<br />

= I(cos 90° + i sin 90°)<br />

4<br />

1 1 / =1<br />

90°+ 360° · k<br />

a= 4 ;k=0,1,2,3<br />

a = 22.5°, 112.5°, 202.5°, 292.5°<br />

x = (cos 22.5° +;sin 22.5°),<br />

(cos 112.5° + i sin 112.5°),<br />

(cos 202 .5° + i sin 202 .5°),<br />

(cos 292.5° + i sin 292.5°)<br />

40. x 5 -i= 0<br />

x 5 =i<br />

." =O+i<br />

= I(cos 90° + i sin 90°)<br />

1 1 / 5 =I<br />

90 0+36O°·k<br />

a = k =0, I, 2, 3, 4<br />

5<br />

a = 18°, 90°, 162°, 234°, 306°<br />

x = (cosI8° + i sin 18°),<br />

(cos 90° + i sin 90°),<br />

(cos 162° + i sin 162°),<br />

(cos 234° +;sin 234°),<br />

(cos 306° + isin306°)<br />

41. x 3 - (4 + 4i.J3) =0<br />

x 3 =4+ 4;.J3<br />

r = ~(4)2 + (4.J3)2 = 8<br />

4.J3<br />

tan8=-- 8=60°<br />

4 '<br />

x 3 = 8(cos 60° +;sin 60°)<br />

8 1 / 3 = 2<br />

60°+ 360° · k<br />

a = 3 ; k =0, 1, 2<br />

a = 20°, 140°, 260°<br />

x = 2(cos 20° + i sin 20°),<br />

2(cos 140° + isin 140°),<br />

2(cos 260° + i sin 260°)<br />

42. x 4 - (8+8i.J3) =0<br />

4<br />

x =8+8;.J3<br />

r = ~r-:82'---+-(8-.J3-3)-2= ../256 = 16<br />

tan 8 = 8~ = .J3;' 8 = 60°<br />

4<br />

x = 16(cos600+;sin60 0)<br />

16 114 =2<br />

60°+ 360°· k .<br />

a = 4 ; k =0, 1, 2, 3<br />

a = 15°, 105°, 195°, 285°<br />

x = 2(cosI5° + ksin 15°),<br />

2(cos 105° + i sin 105°),<br />

2(cosI95° + isin 195°),<br />

2(cos 285° + i sin 285°)<br />

43. x 3 -1 =0<br />

(x -1)(x 2 + x + I) = 0<br />

x-I =0<br />

x= I<br />

or<br />

x 2 +x+1 =0<br />

Use the quadratic formula with<br />

a =1, b = I, and c = 1.<br />

340


<strong>Trigonometry</strong> <strong>Solutions</strong> <strong>Manual</strong><br />

8.4 Powers and Roots of Complex Numbers 341<br />

-1±~12 -4·1 ·1<br />

x - --"----­<br />

2·1<br />

x=<br />

-1±H<br />

2<br />

-1±i../3<br />

x---­<br />

2<br />

x=_.!.±../3 i<br />

2 2<br />

1 ../3 . 1 ../3 .<br />

x=1 --+-1 ----I<br />

• 2 2' 2 2<br />

We see that the solutions are the same as<br />

Exercise 31.<br />

For Exercise 31. the solutions are<br />

(cos 0° + i sin 0°)<br />

(cos 120 0 + i sin 120°)<br />

(cos 240° + i sin 240°).<br />

In standard form. these are<br />

1, _.!. + ../3; _.!. _../3 ;.<br />

2 2' 2 2<br />

44. x 3 + 27 = 0<br />

(x+3)(x 2 - 3x+9) =0<br />

x+3=0<br />

x=-3<br />

or<br />

x 2 -3x+9 =0<br />

Use the quadratic formula with<br />

a =I, b=-3, and c =9.<br />

x<br />

-(-3)±~(-3)2-4·1·9<br />

2·1<br />

3±..J-27<br />

x=--'-­<br />

2<br />

3±(3../3)i<br />

x=<br />

2<br />

x=~±3../3 i<br />

2 2<br />

x =-3 ~+ 3../3; ~_ 3../3;<br />

'2 2' 2 2<br />

For Exercise 36, the solutions are<br />

3(cos6O°+ ;sin600).<br />

(cos180° + i sin 180°),<br />

(cos300° + isin3000).<br />

In standard form, these are<br />

~ + 3../3 i _ 3 ~ _ 3../3t.<br />

2 2 ' , 2 2<br />

45. (cos6+;sin6)2<br />

=1 2(cos26+ isin 26)<br />

=cos 26 +; sin 26<br />

46. (cos6+;sin6)2<br />

=cos 2 6 + 2isin6cos6+ i 2 sin 2 6<br />

=cos 2 6+ 2isin6cos6-sin 2 6<br />

=(cos 2 6 -sin 2 6) + i(2sin 6 cos 6)<br />

47. Two complex numbers a + bi and c + di are equal<br />

only if a =c and b =d. Therefore,<br />

cos 2 6-sin 2 6 =cos26<br />

and<br />

2sin6cos6 =sin 26.<br />

48. cos 26 + i sin 26<br />

=(cos 2 6 - sin 2 6) + i(2 cos 6sin6)<br />

Since a + bi =c + di only if a =c and b = d.<br />

i sin 26 =i(2 cos 6 sin 6) and<br />

sin 26 =2cos6sin6<br />

49. (a) Ifz = 0 + Oi,we must make the following<br />

calculations:<br />

0 2 + 0 =0; 0 2 + 0 =O.<br />

The calculations repeat as 0, 0, 0, . .., and<br />

will never exceed a modulus of 2. The point<br />

(0, 0) is part of the Mandelbrot set. The pixel<br />

at the origin should be turned on.<br />

(b) Ifz=l-li.<br />

(1- ;)2 +(1- ;) =1-3i.<br />

The modulus of 1 .:.. 3i is .,flO > 2.<br />

Therefore, 1 - 1i is not part of the<br />

Mandelbrot set. and the pixel at (1. -1)<br />

should be left off.<br />

(c) Ifz = -.5i,<br />

(-.5;)2 - .5; =- .25 - .5;;<br />

(-.25)- .5i)2 + (-.25 -.5i)<br />

=-.4375 - .25i;<br />

(-.4375 - .25i)2 + (-.4375 - .25i)<br />

= -.308593- .03125;;<br />

(-.308593 - .03125i)2<br />

+(-.308593 - .03125i)<br />

=-.214339 - .0119629i;<br />

(-.214339 - .0119629i)2<br />

+(-.214339 - .0119629;)<br />

=-.16854 - .00683466i.<br />

This sequence appears to be approaching the<br />

origin. and no number has a modulus greater<br />

than 2. Thus, -.5; is part of the Mandelbrot<br />

set, and the pixel at (0, -.5) should be turned<br />

on.<br />

341


<strong>Trigonometry</strong> <strong>Solutions</strong> <strong>Manual</strong><br />

342 Chapter 8 Complex Numbers, Polar Equations, and Parametric Equations<br />

2z 3 + 1 .<br />

50. (a). Let f(z) =--2- and ZI =I.<br />

3z<br />

Then,<br />

2;3 + 1<br />

z2= f(zl) =3j2<br />

1 2 .<br />

=--+-,.<br />

3 3<br />

Similarly,<br />

3<br />

2z 2 + 1<br />

z3 = f(Z2) = 2<br />

3z 2<br />

'" -.58222 + .92444;,<br />

2z 3 + 1<br />

z4 = f(Z3) = --2­<br />

3z 3<br />

'" -.50879 + .868165;.<br />

The values of z seem to approach<br />

1 .../3.<br />

w2 =--+-1.<br />

2 2<br />

Color the pixel at (0, 1) blue.<br />

2z 3 +1 .<br />

(b) Letf(z) =--2- and Zl = 2 + I.<br />

3z<br />

Then,<br />

z =f(z ) = 2(2 + ;)3 + 1<br />

2 I 3(2 + 02<br />

'" 1.37333 + .61333;.<br />

2z 2<br />

3<br />

+ 1<br />

z3=f(Z2)= 2<br />

3z 2<br />

'" 1.01389 + .29916li<br />

2z 3<br />

3<br />

+ 1<br />

Z4 =f(Z3) = 2<br />

3z 3<br />

'" .926439 +.0375086;<br />

2zl+1<br />

Zs = f(Z4) = 2<br />

3z 4<br />

'" 1.00409 - .00633912;.<br />

The values of z seem to approach WI =1.<br />

Color the pixel at (2, 1) red.<br />

2z 3 + 1 .<br />

(c) Let f(z) =--2- and zl =-I-I.<br />

3z<br />

Then,<br />

2(-1- ;)3 + 1<br />

z - f(z ) ­<br />

2- 1- 3(-1-0 2<br />

2 5.<br />

=----1.<br />

3 6<br />

2z 2<br />

3<br />

+ 1<br />

z3 =f(Z2) = 3 2<br />

(;2<br />

'" .508691 - .841099;<br />

2zl+1<br />

Z4 =f(Z3) = 2<br />

3z 3<br />

'" -.499330- .866269;<br />

The values of z seem to approach<br />

1 .../3 .<br />

W3 =----1.<br />

2 2<br />

Color the pixel at (-I, -1) yellow .<br />

51. Using the trace function, we find that the other<br />

four fifth roots of 1 are: .3090 1699 + .95105652;,<br />

- .809017 + .58778525;,<br />

-809017 - .5877853;, .30901699 - .9510565;<br />

52.<br />

-1.5<br />

-l.SI--+---t--iE-- 1.5<br />

T=O<br />

11=1 y=O<br />

1.5<br />

Using the trace function, we find that three of the<br />

tenth roots of 1 are: I, .8090.1699 + .58778525;,<br />

.30901699 + .95105652i<br />

53. 2 + 2.,[3i is one cube root.<br />

r=~22 +(2.../3)2 =.J4+12 =4<br />

2.../3 .../3<br />

tan8 =--= 3' 8=60°<br />

2 '<br />

This root is<br />

4 cis 60° = 4 cis (60° . 1).<br />

Since the graphs of the other roots must be<br />

equally spaced around a circle and the graphs of<br />

these roots are all on a circle that has center at the<br />

origin and radius 4, the other roots are<br />

4 cis (60° + 120°) = 4 cis 180°<br />

and<br />

4 cis (60° + 2· 120°) = 4 cis 300°.<br />

342


<strong>Trigonometry</strong> <strong>Solutions</strong> <strong>Manual</strong><br />

8.5 Polar Equations 343<br />

4 cis 180° = 4(cos 180° + i sin 180°)<br />

=4(-I+i ·0) =-4<br />

4 cis 300° = 4(cos 300° + i sin 300°)<br />

=2-2i..[3<br />

The roots are 2 - 2i..[3, - 4 and 2 + 2i..[3 .<br />

54. x 3 + 4 - 5; =0<br />

x 3 =-4+5;<br />

r = ~(-4)2 +5 2 =..J41<br />

tan (} = -~, (} is in quadrant II.<br />

4<br />

(} = 128.65981°<br />

x 3 = ..J41(cosI28.65981°<br />

+ i sin 128.65981°)<br />

(..J41)1I3 = 1.8569376<br />

a=(}+36O°.k· k=0 1 2<br />

3 ' "<br />

Irk = 0, a = 128.65981° + O· 36QO<br />

3<br />

=42.886603°.<br />

Irk = I, a = 128.65981° + 1·360°<br />

3<br />

= 162.8866°.<br />

If k = 2, a = 128.65981° + 2.360°<br />

3<br />

= 282.8866°.<br />

x = 1.8569376<br />

·(cos 42.886603° +; sin 42.886603°),<br />

1.8569376<br />

· (cos 162.8866° +; sin 162.8866°),<br />

1.8569376<br />

· (cos 282.8866° +;sin 282.8866°)<br />

In standard form,<br />

x =1.3606 + 1.2637;,<br />

-1.7747 + .5464;,<br />

.4141-1.8102;.<br />

55. x 5 + 2 +3; =0<br />

x 5 =-2 -3;<br />

r= ../4+9 =../13<br />

r 1/ n =(../13)115 = 13 1/ 10 = 1.292<br />

-3<br />

tan(}=-=1.5<br />

-2<br />

(}is in quadrant III.<br />

(} = 236.30993°<br />

236.30993 ° + 360° . k<br />

a=------­<br />

5<br />

k=0,1,2,3,4<br />

a =47.26°, 119.26°, 191.62°,<br />

263.26°, 335.26°<br />

x'" 1.292(cos47.26°+;sin47.26°)<br />

'" .87708 + .94922i<br />

1.292(cosI19.26° + isin 119.26°)<br />

'" -.63173 + 1.1275i<br />

1.292(cos 191.26° + ;sin 191.26°)<br />

'" -1.2675 - .25240;<br />

1.292(cos 263.26° + i sin 263.26°)<br />

'" -.15164 -1.28347i<br />

1.292(cos 335.26° +; sin 335.26°)<br />

'" 1.1738 - .54083i<br />

56. The number 1 has 64 complex 64th roots . Two of<br />

them are real, 1 and -1, and 62 ofthem are not<br />

real.<br />

57. The statement, "Every real number must have two<br />

real square roots," is false. Consider, for example,<br />

the real number -4. Its two square roots are 2;<br />

and -2; which are not real.<br />

58. The statement, "Some real numbers have three<br />

real cube roots,"is false. Every real number has<br />

only one cube root that is also a real number.<br />

Then there are two cube roots, which are<br />

conjugates that are not real.<br />

Section 8.5<br />

Connections (page 363)<br />

1. y=x-3<br />

-x+ y =-3<br />

x-y=3<br />

2. y=~<br />

i =4-x 2<br />

Exercises<br />

x 2 +i =4<br />

1. (a) II (since r » 0 and 90° < (}< 135°)<br />

(b) I (since r » 0 and 0° < (J < 90°)<br />

(c) IV (since r> 0 and _90° < (} < 0°)<br />

(d) III (since r > 0 and 180° < (J < 270°)<br />

2. (a) positive x-axis<br />

(b) negative x-axis<br />

343


<strong>Trigonometry</strong> <strong>Solutions</strong> <strong>Manual</strong><br />

344 Chapter 8 Complex Numbers, Polar Equations, and Parametric Equations<br />

(c) negative y-axis<br />

(d) positive y-axis (since 450 - 360 =90°)<br />

7. Two other pairs of polar coordinates for (5, -600)<br />

are (5, 300°) and (-5 , 120°).<br />

For Exercises 3-12, answers may vary.<br />

3. Two other pairs of polar coordinates for (1, 45°)<br />

are (1, 405°) and (-I, 225°).<br />

8. Two other pairs of polar coordinates for (2, - 45°)<br />

are (2, 315°) and (-2, 135°).<br />

90°<br />

270°<br />

9. Two other pairs of polar coordinates for (-3,<br />

_2 10°) are (-3, 150°) and (3, -30°).<br />

90°<br />

Two other pairs of polar coordinates for<br />

(- 2, 135°) are (-2, 495°) and (2, 315°).<br />

90°<br />

270·<br />

6. Two other pairs of polar coordinates for (-4,27°)<br />

are (-4, 387°) and (4, 207°).<br />

90°<br />

10. Two other pairs of polar coordinates for (-1,<br />

-120°) are (-I, 240°) and (1, 60°).<br />

90° ,<br />

./)(;~r:=lt(\ , •<br />

18W~jW<br />

270°<br />

11. Two other pairs of polar coordinates for (3, 300°)<br />

are (3, 660°) and (-3, 120°).<br />

344


<strong>Trigonometry</strong> <strong>Solutions</strong> <strong>Manual</strong><br />

8.5 Polar Equations 345<br />

12.<br />

For Exercises 13-21, answers may vary.<br />

13. r = ~(_1)2 + 1 2 =.J1+l =..fi<br />

8 = arctan( ~1) = 135°<br />

(since x < 0 and y > 0)<br />

So one possibility is (..fi, 135°)<br />

Alternatively if r = -.{i, 8 = 135+ 180 = 315°<br />

A second possibility is (-.{i, 315°)<br />

Two pairs:<br />

(..fi, 135°), (-.{i, 315°)<br />

y<br />

3<br />

2<br />

.1<br />

-3 -2 -1 0 1 2 3<br />

-1<br />

-2<br />

-3<br />

14. r = ~12 + 1 2 =..fi<br />

8 = arcta{D= 45°<br />

(sincex>O and y > 0)<br />

One possibility is: (..fi, 45°)<br />

Alternatively, if r = -.{i, 8 = 45 + 180 = 225°<br />

A second possibility is: (-.{i, 225°).<br />

Two pairs:<br />

(..fi, 45°), (-Ii, 225°)<br />

y<br />

3<br />

2<br />

•<br />

+-+-+-:+---t--t-~ x<br />

-3 -2 -1 0 1 2 3<br />

-I<br />

-2<br />

-3<br />

15. r = ~02 + 3 2 =..[9 = 3<br />

The point (0. 3) is on the positive y-axis. Thus,<br />

8=90°.<br />

One possibility is (3. 90°).<br />

Alternatively, if r= -3, 8 = 90+ 180 = 270°<br />

A second possibility is: (-3,270°)<br />

Two pairs :<br />

(3,90°), (-3, 270°)<br />

y<br />

3<br />

2<br />

+-+-+-.l--+-+-~ .x<br />

-3 -2 _I 0 I 2 3<br />

-I<br />

-2<br />

-3<br />

16. r=~02+(-3)2 =..[9=3<br />

The point (0, -3) is on the negative y-axis. Thus,<br />

8=270°.<br />

One possibility is: (3, 270°)<br />

Alternatively, if r = -3, 8 = 270 -180 = 90°<br />

Two pairs :<br />

(3,270°), (-3, 90°)<br />

y<br />

3<br />

2<br />

+-+--t---:;d-+-+-~x<br />

-3 -2 -1 0 123<br />

-I<br />

-2<br />

-3<br />

17. r=~(..fi)2 + (..fi)2 =·./2+2 =../4 =2<br />

8 = arctan(~) = arctan (1)= 45°<br />

(since x > 0 and y > 0)<br />

One possibility is: (2, 45°)<br />

Alternatively, if r =-2,<br />

8 = 45°+180= 225°<br />

A second possibility is: (-2,225°)<br />

Two pairs:<br />

(2,45°), (-2, 225°)<br />

y<br />

3<br />

2<br />

•<br />

+-+-+-nt--+--+-,. .x<br />

-3 -2 _I 0 1 2 3<br />

-I<br />

-2<br />

-3<br />

345


<strong>Trigonometry</strong> <strong>Solutions</strong> <strong>Manual</strong><br />

346 Chapter 8 Complex Numbers, Polar Equations, and Parametric Equations<br />

18. r=~(--J2)2 + (-J2)2 =../2+2 =../4 =2<br />

f) = arc 3 11 ( ~J= arctan (-1) = 135°<br />

(since x < 0 and y > 0)<br />

One possibility is: (2, 135°)<br />

Alternatively, if r = -2, f) = 135+ 180= 315°<br />

A second possibility is: (-2, 315°)<br />

Two pairs:<br />

(2, 135°), (-2, 315°)<br />

y<br />

3<br />

2<br />

•<br />

-3 -2 -1 0 1 2 3<br />

-1<br />

-2<br />

-3<br />

19. r= (~r +(%y =~~+~ =~<br />

3<br />

2<br />

-3 -2 -18 0 1 2 3<br />

-I<br />

-2<br />

-3<br />

21. r =~32 + 0 2 = ~ =3<br />

f) =arctan(j)= 0°<br />

y<br />

(since tan f) =0 and x > 0).<br />

One possibility is: (3, 0°)<br />

Alternatively, if r= -3, f) = 0+180= 180°<br />

A second possibility is (-3,180°)<br />

Two pairs:<br />

(3,0°), (-3, 180°)<br />

y<br />

3<br />

2<br />

f) = arcta{%...lJ) =arctan (~) = 60°<br />

(since x > 0 and y > 0)<br />

One possibility is: (.,fj, 60°)<br />

Alternatively, if r = -./3, f) = 60+ 180 = 240°.<br />

Two pairs:<br />

(..[3, 6QO), (-Jj, 240°).<br />

y<br />

3<br />

2<br />

•<br />

+-+-+-~-+-+-....... x<br />

- 3 -2 -1 0 1 2 3<br />

-1<br />

-2<br />

-3<br />

20. r= (~r+(-fY =~~+~=1<br />

f) = arctan( ~1 -( ~)) = arctan( ~) = 210°<br />

(since x < 0 and y < 0)<br />

One possibility is: (1, 210°)<br />

Alternatively, if r = -I, f) = 210 -180 = 30°<br />

A second possibility is: (-1, 30°)<br />

Two pairs:<br />

(1,210°), (-1 ,30°)<br />

0<br />

-3 -2 -1 1 2 3<br />

-1<br />

22. (a) C<br />

(b) D<br />

(c) A<br />

(d) B<br />

-2<br />

-3<br />

x<br />

346


<strong>Trigonometry</strong> <strong>Solutions</strong> <strong>Manual</strong><br />

8.5 Polar Equations 347<br />

23. r = 2 + 2 cos 0 (cardioid)<br />

0 0° 30° 60° 90° 120° 150°<br />

cos 0 1 .9 .5 0 -.5 -.9<br />

r 4 3.8 3 2 1 .2<br />

0 180° 210° 240° 270° 300° 330°<br />

cos 0 -1 - .9 -.5 0 .5 .9<br />

r 0 .3 1 2 3 3.7<br />

270·<br />

r=2+2cos9<br />

24. r =2(4 + 3cos 0) (cardioid)<br />

=8+6cosO<br />

0 0° 30° 60° 90° 120° 150°<br />

cosO 1 .9 .5 0 -.5 - .9<br />

r 14 13.2 11 8 5 2.8<br />

0 180° 210° 240° 270° 300° 330°<br />

cos 0 -1 -.9 -.5 0 .5 .9<br />

r 2 2.8 5 2 11 13.2<br />

90·<br />

ISO· -+-+-++-+++-+tHlb.<br />

270·<br />

r= 2(4+ 3 cos 9)<br />

25. r =3 + cos 0 (limacon)<br />

0 0° 30° 60° 90° 120° 150°<br />

r 4 3.9 3.5 3 2.5 2.1<br />

0 180° 210° 240° 270° 300° 330°<br />

r 2 2.1 2.5 3 3.5 3.9<br />

347


<strong>Trigonometry</strong> <strong>Solutions</strong> <strong>Manual</strong><br />

348 Chapter 8 Complex Numbers, Polar Equations, and Parametric Equations<br />

90°<br />

26. r = 2 - cos 6 (limacon)<br />

--1<br />

0° 30° 60° 90° 135°<br />

1.1 1.5 2 2.7<br />

6 180° 225° 270° 315°<br />

r 3 2.7 2 1.3<br />

R90°<br />

'Y<br />

\.......+->~.<br />

~ ,..--r·y -,<br />

i l / \ i


<strong>Trigonometry</strong> <strong>Solutions</strong> <strong>Manual</strong><br />

8.5 Polar Equations 349<br />

Pattern 3,0, -3, 0, 3 continues for every 18°,<br />

90° 72D<br />

't9~<br />

180· _.;.-. -+-j.- .....•...-+_.,-. O·<br />

,~~)i<br />

2~Oo 288 0<br />

r=3cos59<br />

29. r 2 =4 cos 29 (lemniscate)<br />

r =±2.Jcos29<br />

Graph only exists for [0°, 45°J,<br />

[135°, 225°J, and [315°, 360 0J<br />

because cos 29 must be positive.<br />

r ±2 ±1.4 0 0 ±1.4<br />

9 180<br />

r ±2 ±1.4 0 0 ±1.4<br />

90·<br />

30. r 2 = 4sin29 (lemniscate)<br />

r =±2.Jsin 29<br />

Graph only exists for [0°, 90 0J, [180°, 270 0J because sin 9 must be positive.<br />

r 0 ±1.86 ±2 ±1.86 0 o ±2 0<br />

90·<br />

270·<br />

2 r = 4 sin 29<br />

349


<strong>Trigonometry</strong> <strong>Solutions</strong> <strong>Manual</strong><br />

350 Chapter 8 Complex Numbers, Polar Equations, and Parametric Equations<br />

31. r = 4(1- cos 9) (cardioid)<br />

9 0° 30° 60° 90° 120° 150°<br />

r 0 .5 2 4 6 7.5<br />

f) 180° 210° 240° 270° 300° 330°<br />

r 8 7.5 6 4 2 .5<br />

90 0<br />

32. r = 3(2- cos f)) (cardioid)<br />

=6-3cos9<br />

r 3 3.9 6 8.1 9 6 3<br />

180"<br />

270 0<br />

r = 3(2 - cosO)<br />

33. r = 2 sin f) tan f) (cissoid)<br />

r is undefined at 9 = 90° and B = 270°.<br />

r 0 .6 1.4 3 -3 -1.4 -.6 0<br />

Notice that for [180°, 360°), the graph retraces the path traced for [0°,180°).<br />

90 0<br />

270 0<br />

r = 2 sin 0 tan 0<br />

350


<strong>Trigonometry</strong> <strong>Solutions</strong> <strong>Manual</strong><br />

8.5 Polar Equations 351<br />

cos20 (ci 'd ·th I )<br />

34. r =-- CISSOI WI a oop<br />

cosO<br />

r is undefined at 0 = 90° and 0 = 270°<br />

r = 0 at 45°, 135°,225°, and 315°.<br />

0 0° 45° 60° 70° 80°<br />

r 0 - 1 -2.2 -5.4<br />

0 90° 100° 110° 135° 180°<br />

r 5.4 2.2 0 -1<br />

Notice that for [180°, 360°), the graph retraces the<br />

path traced for<br />

[0°, 180°).<br />

35. (a) (r,-0)<br />

36. -0<br />

(b) (r,1t - 0) or (-r, -0)<br />

(c) (r, 1t+ 0) or (-r, 0)<br />

37. 1t-0<br />

38. -r;-O<br />

39. -r<br />

40. 1t+ 0<br />

41. the polar axis<br />

42. the line 0 = ~<br />

2<br />

43. r = 4 cos 20, 0° s 0 < 360°<br />

Since the largest value of cos 20 is 1 (the range<br />

of the cosine function is [-I, 1]), the largest value<br />

of r is 4 . 1 =4.<br />

4cos20=0<br />

cos20=0<br />

Since 0°::; 0 < 360°,0°::; 20 < 720°.<br />

The cosine is 0 at 90°, 270°, 450°, 630°. Thus,<br />

20 = 90° or 20 = 270°<br />

o= 45° or 0 = 135°<br />

20 = 450° or 20 = 630°<br />

0=225° or 0 = 315°.<br />

44. r = 5 sin 30, 0° ::; 0 < 180°<br />

Since -1 ::; sin 0::; I, the largest value for sin 30<br />

is 1.The largest value r can be is<br />

5 sin 30 = 5· 1 = 5.<br />

0= 5 sin 30<br />

0= sin 30<br />

Since 0° ::; 0 < 180°,0° ::; 30 < 540°.<br />

Therefore,<br />

30 = 0° or 30 = 180° or 30 = 360°<br />

0=0° or 0 = 60° or 0 = 120°.<br />

46. r = 4 sin 0, r = 1 + 2 sin 0 ,<br />

0::; 0 < 21t<br />

4sinO = 1+ 2sinO<br />

2sinO = 1<br />

. n 1<br />

sm(7 =­<br />

2<br />

1t 51t<br />

0=- or­<br />

6 6<br />

The points of intersection are<br />

( 4 sin ~' : ) = (2, ~) and<br />

4 sin 51t 51t) = (2 51t).<br />

( 6' 6 ' 6<br />

47. r = 2 + sin 0, r = 2 + cos 0,<br />

0::; 0 < 21t<br />

2 + sin 0 = 2 + cos 0<br />

sinO=cosO<br />

1t 51t<br />

0=- or­<br />

4 4<br />

. 1t -Ii 4+-li<br />

r =2 + sm - =2 +- =--­<br />

4 2 2<br />

. 51t -Ii 4 - -Ii<br />

r = 2 +sm- = 2 --=--­<br />

4 2 2<br />

The points of intersection are<br />

(4+ :} (4­5;).<br />

2-1i, 2-1i,<br />

351


<strong>Trigonometry</strong> <strong>Solutions</strong> <strong>Manual</strong><br />

352 Chapter 8 Complex Numbers, Polar Equations, and Parametric Equations<br />

48. r = sin 20. r = .ficos o. 0 ~ 0 < 1t 52. r=2cos 0<br />

Multiply both sides by r.<br />

sin 20 = -ficos 0<br />

r 2 =2rcosO<br />

2sin OcosO = -ficosO<br />

Since r 2 = x 2 + y2 and x =rcosO.<br />

2sinOcosO - -ficosO = 0<br />

x<br />

cos 0(2 sin 0 ­ -fi) 2 +i =2x.<br />

= 0<br />

Complete the square on x to get the equation of a<br />

cos 0 = 0 or 2 sin 0 - -fi = 0<br />

circle.<br />

2sinO =-fi x 2 -2x+i =0<br />

sinO = -fi<br />

2<br />

1t<br />

1t<br />

31t<br />

0=- or 0=- or­<br />

244 y<br />

The points of intersection are<br />

( s in 2 . ~ 2 ' 1t)=(0 2 ' ~I 2)<br />

(<br />

sin 2 .! ~) = (1 !)<br />

4' 4 • 4<br />

and (sin 2 . 31t<br />

4'<br />

51. r= 2 sin 0<br />

Multiply both sides by r .<br />

31t) = (-1 31t).<br />

4 • 4<br />

r 2 =2rsinO<br />

Since r 2 = x 2 + y2 and y = rsin O. 53.<br />

x 2+i =2y.<br />

Complete the square on y.<br />

x 2 +i - 2y + 1= 1<br />

x 2 + (y _1)2 =1<br />

The graph is a circle with center at (0. 1) and<br />

radius 1.<br />

y<br />

(x _1)2 +i = 1<br />

The graph is a circle with center at O. 0) and<br />

radius 1.<br />

-+-+--+-....,.f--+--I--+-. x<br />

o<br />

r=2cosO<br />

(x-I)2+ r=1<br />

2<br />

r=--­<br />

I-cosO<br />

Multiply both sides by 1 - cos O.<br />

r-rcosO =2<br />

~x2+i -x=2 .<br />

~x2 +i =2+x<br />

x 2 +i =(2 + x)2<br />

x 2 + i =4+4x+;2<br />

2<br />

i =4(I+x)<br />

The graph is a parabola with vertex at (-I, 0) and<br />

axis y = O.<br />

y<br />

r=2 sinO<br />

x 2 + (y_1)2= I<br />

-Hf-+-f-:+-++++-+-t-l-. x<br />

-1<br />

r-_2_<br />

- I -cos9<br />

y2=4(x+l)<br />

352


<strong>Trigonometry</strong> <strong>Solutions</strong> <strong>Manual</strong><br />

3<br />

3<br />

54. r=--­56. r=----­<br />

I-sinO<br />

4cosO-sinO<br />

r-rsinO =3<br />

4rcosO- rsinO = 3<br />

r= rsinO+3<br />

4x- y=3<br />

~x2+l =y+3<br />

The graph is a line with intercepts (0, -3) and<br />

x 2+l (3/4,0).<br />

=l +6y+9<br />

y<br />

x 2 =6y+9<br />

X~=6(Y+~)<br />

The graph is a parabola with axis x = 0 and vertex<br />

(0, -312).<br />

y<br />

8.5 Polar Equations 353<br />

_ 3<br />

r- 4cos8-sin8<br />

4x-y=3<br />

55.<br />

r+2cosO = -2sinO<br />

x 2 + i<br />

x 2 +i+2y+2x =0<br />

r 2 =-2rsinO-2rcosO<br />

=-2y-2x<br />

(x+l)2 +(Y+ 1)2 =2<br />

The graph is a circle with center (-1. -1) and<br />

radius .fi.<br />

y<br />

+-++-T-+--+-+-" x<br />

r + 2 cos 9 = -2 sin 9<br />

(x+I)2+(y+I)2=2<br />

57. r=2secO<br />

2<br />

r=-­<br />

cosO<br />

rcosO = 2<br />

x=2<br />

The graph is a vertical line.<br />

y<br />

-1--4--++-+-+--i--- x<br />

o<br />

2<br />

1<br />

r=2sec9<br />

x=2<br />

58. r=-5cscO<br />

5<br />

r=--­<br />

sinO<br />

rsinO=-5<br />

y=-5<br />

The graph is a horizontal line.<br />

y<br />

1<br />

o<br />

~-5+---~<br />

r=-5 esc 9<br />

y=-5<br />

353


<strong>Trigonometry</strong> <strong>Solutions</strong> <strong>Manual</strong><br />

354 Chapter 8 Complex Numbers, Polar Equations, and Parametric Equations<br />

59. r(cos 8 + sin 8) = 2<br />

rcos8+ rsin8 = 2<br />

x+y=2<br />

The graph is the line with intercepts (0, 2) and<br />

(2,0).<br />

y<br />

-+-+-+-:+-+""k--t-.. x<br />

65. y= 2<br />

rsin8 = 2 or<br />

2<br />

r=--=2csc8<br />

sin8<br />

66. x=4<br />

rcos8 = 4 or<br />

4<br />

r=--=4sec8<br />

cos8<br />

67. Graph r = 8, a spiral of Archimedes.<br />

, (cos 6 + sin 6) = 2<br />

x+y=2<br />

60. r(2 cos 8 + sin 8) = 2<br />

2rcos8+ rsin8= 2<br />

2x+y=2<br />

The graph is the line with intercepts (0, 2) and<br />

(1,0).<br />

y<br />

8 -6.3 -4.7 -3.1 -1.6<br />

(radians)<br />

r -6.3 -4.7 -3.1 -1.6<br />

o 1.6 3.1 4.7 6.3<br />

(radians)<br />

r o 1.6 3.1 4.7 6.3<br />

1<<br />

2<br />

,(2 cos 9 + sin 9) = 2<br />

2x+ y=2<br />

61. x+y=4<br />

rcos8 + rsin8 = 4<br />

r(cos8 + sin 8) = 4<br />

4<br />

r=---­<br />

cos8+sin8<br />

68.<br />

62. 2x-y=5<br />

2(rcos8) - rsin8 = 5<br />

r(2cos8 - sin 8) = 5<br />

5<br />

r=----­<br />

2cos8-sin8<br />

63. x 2 + y2 =16<br />

r 2 =16<br />

r=4<br />

Degree mode<br />

64. x 2 +i =9<br />

r 2 =9<br />

r=3<br />

354


<strong>Trigonometry</strong> <strong>Solutions</strong> <strong>Manual</strong><br />

8.5 Polar Equations 355<br />

69. In rectangular coordinates, the line passes through<br />

(1,0) and (0, 2). So<br />

2-0 2<br />

m =--=- =-2 and<br />

0-1 -1<br />

(y - 0) = -2(x -1)<br />

2x+y=2<br />

y = -2x+ 2<br />

Converting to polar form r = c. 0<br />

acosO+bsm<br />

2<br />

we have: r =----­<br />

2cosO + sin 0<br />

70. (a) Plot the following polar equations on the<br />

same polar axis:<br />

.39(1-.206 2)<br />

Mercury: r = ;<br />

1+ .206 cos0<br />

.78(1- .007 2 ).<br />

Venus: r<br />

1+ .007 cosO'<br />

2<br />

Earth: r = 1(1- .017 ) ;<br />

1+.017cosO<br />

2)<br />

Mars: r = 1.52(1- .093 .<br />

1+ .093cosO<br />

Use a viewing window of [-2.4,2.4] by<br />

[-1.6, 1.6] with Xscl =.2, Yscl =.2.<br />

1.6<br />

(c) We must determine if the orbit of Pluto is<br />

always outside the orbits of the other planets.<br />

Since Neptune is closest to Pluto, plot the<br />

orbits of Neptune and Pluto on the same<br />

PQlar axes.<br />

30.1(1 - .009 2 )<br />

r = 1 + .009cos 0<br />

60<br />

2<br />

30.1(1-.009 )<br />

Neptune: r = 1+.009 cos 0<br />

..:....<br />

39_.4~ ( 1__ - . 2_ 4_~ 9 2 )<br />

PIuto: r = 1+ .249cosO<br />

The graph shows that their orbits are very<br />

close near the polar axis.<br />

30.1 (1 - .009 2 )<br />

r= 1 + .009 cos 8<br />

(b) Plot the following polar equations on the<br />

same polar axis:<br />

2<br />

Earth: r = 1(1- .017 ) ;<br />

1+.017cosO<br />

2<br />

. 5.2(1-.048 )<br />

Jupiter: r = ;<br />

1+.048cosO<br />

19.2(1-.047 2)<br />

Uranus: r= ;<br />

1+ .047 cos 0<br />

2<br />

39.54(1-.249 )<br />

PIuto: r =--..:..----~<br />

1+ .249 cos 0<br />

Use a viewing window of [~, 60] by [-40,<br />

40] with Xscl =10, Yscl = 10.<br />

From the graph, Earth is closest to the sun of<br />

these four planets.<br />

Use ZOOM to determine that the orbit of<br />

Pluto does indeed pass inside the orbit of<br />

Neptune.<br />

Therefore, there are times when Neptune, not<br />

Pluto, is the farthest planet from the sun.<br />

(However, Pluto's average distance from the<br />

sun is considerably greater than Neptune's<br />

average distance.)<br />

355


<strong>Trigonometry</strong> <strong>Solutions</strong> <strong>Manual</strong><br />

356 Chapter 8 Complex Numbers and Polar Equations<br />

Section 8.6<br />

1. C; at t = 2, x = 3(2) + 6 = 12<br />

y = -2(2)+ 4 =0<br />

2. D; att =:' x=co{:)=~<br />

Y =Sin(:) =~<br />

6.<br />

x=t +2, y=t 2,fortin[-I,I]<br />

t x=t+2 y = t 2<br />

-1 -1+2=1 (-1)2=1<br />

o 0+2 =2 0 2 =0<br />

1 1+2=3 1 2=1<br />

y<br />

3. A; at r = 5, x =5<br />

Y =5 2 =25<br />

4. B; at t =2, x =2 2 + 3 =7<br />

y=2 2-2=2<br />

5. x=2t,y=t+l,fortin[-2,3]<br />

x=2t<br />

y=t+l<br />

-2 2(-2)=-4 -2+1 =-1<br />

-1 2(-1) =-2 -1 + 1=0<br />

o 2(0) =0 0+1=1<br />

1 2(1)= 2 1+1=2<br />

2 2(2) = 4 2+1=3<br />

3 2(3) = 6 3+1=4<br />

x = 2t y<br />

Y= t+ I<br />

for t in [-2, 3]<br />

2 3<br />

x=t+2<br />

y=t 2<br />

for t in [-I. I]<br />

x - 2 = t, therefore<br />

y =(x - 2)2 or y =x 2 - 4x + 4.<br />

Since t is in [-1, 1], x is in [-1 + 2, 1 + 2] or<br />

[1,3].<br />

7. x =..[i, y = 3t - 4, for t in [0,4].<br />

x=..[i<br />

y=3t-4<br />

o .JO = 0 3(0) - 4 = -4<br />

1 .J!=1 3(1)-4=-1<br />

2 ..fi = 1.4 3(2) - 4 = 2<br />

3 ~ =1.7 3(3)-4=5<br />

4 ..[4=2 3(4)-4=8<br />

y<br />

8 (2. 8)<br />

x=-{i<br />

y= 3t-4<br />

for t in [0. 4]<br />

x=2t,y=t+ 1<br />

S . x x<br />

mce - =t, Y =- + 1.<br />

2 2<br />

Since t is in [-2,3],x is in [2(-2),2(3)] or [-4, 6].<br />

(0. -4)<br />

o 2<br />

x=..[i y=3t-4<br />

Since x 2 =t, y=3x 2 -4.<br />

Since t is in [0, 4],x is in hlO, ..[4] or [0, 2].<br />

356


<strong>Trigonometry</strong> <strong>Solutions</strong> <strong>Manual</strong><br />

8.6 Parametric Equations 357<br />

8. x = t 2, Y = .,fi, for t in [0,4] 10. x=2t-l, y=t 2+2 ,fortin(-oo,oo)<br />

t x =t 2 Y =.,fi<br />

x=2t-l y = t 2 +2<br />

0 0 2 =0 .JO =0<br />

-3 2(-3)-1=-7 (-3) 2 +2=11<br />

1 1 2 = 1 ..JI = 1<br />

-2 2(-2)-1 =-5 (_2)2 +2 =6<br />

2 2 2 =4 -fi = 1.414<br />

-1 2(-1)-1=-3 (_1)2 +2 =3<br />

3 3 2 =9 ..fj = 1.732<br />

4 4 2 = 16 ..[4 =2<br />

y<br />

2<br />

0<br />

1<br />

2<br />

3<br />

2(0)-1 =-1<br />

2(1)-1 = 1<br />

2(2)-1 = 3<br />

2(3)-1 = 5<br />

0 2 +2=2<br />

1 2+2=3<br />

2 2+2=6<br />

3 2 +2 =11<br />

y<br />

9.<br />

++-t+t++lf+t++lf+t-H.- x<br />

5 16<br />

x= 1 2<br />

y={t<br />

for 1 in[0, 4]<br />

Since y =.,fi, y2 = t, and x = t 2 = (y2)2 =l.<br />

Since t is in [0, 4], x is in [0 2,42], or [0, 16].<br />

x=t 3+1, y=t 3-1 ,fortin(-oo,00)<br />

t x =t 3 +1<br />

-2 (-2)3+ 1=-7<br />

-1 (_1)3 + 1= 0<br />

o 0 3 + 1=1<br />

1 1 3+1=2<br />

2 2 3 +1 =9<br />

3 3 3 +1 =28<br />

y<br />

(-2)3_ 1=-9<br />

(_1)3-1 =-2<br />

0 3 -1 =-1<br />

1 3 -1 =0<br />

2 3 -1 =7<br />

3 3 -1 =26<br />

+-+-+-+-++d-++-+-+.- o<br />

x<br />

x=21-1<br />

y=~+2<br />

for 1in(--, ...)<br />

x+l=2t<br />

x+l<br />

--=t<br />

2<br />

Since y = t 2 + 2,<br />

X +1)2 1 2<br />

y= -2­ +2='4(x+l) +2.<br />

(<br />

Since t is in (-00, 00) and x = 2t ­ 1, x is in<br />

(-eo, 00).<br />

11. x=2sint, y=2cost, fortin[0,2n]<br />

t x=2sint y=2cost<br />

o 2sinO = 0 2cos6 = 2<br />

f 2sinf = 1 2cosf =.J3<br />

t 2sint =.J2 2cost =.J2<br />

Since x = t 3 + 1, i 2sini =..fj 2cosi = 1<br />

x-I =t 3 .<br />

i<br />

Since y = t 3 -1,<br />

y =(x -1) - 1=x ­ 2.<br />

Since t is in (-eo, 00), x is in (-eo, 00).<br />

2<br />

2sini = 2 2cosi = 0<br />

y<br />

x=2sinl<br />

y=2cosl<br />

for 1 in [0, 21r]<br />

Since x = 2sint and y = 2cost,<br />

~ = sint and L = cost.<br />

2 2<br />

357


<strong>Trigonometry</strong> <strong>Solutions</strong> <strong>Manual</strong><br />

358 Chapter 8 Complex Numbers and Polar Equations<br />

Since sin 2 I + cos 2 I = 1,<br />

y<br />

x 2<br />

i<br />

-+-=1<br />

4 4<br />

x 2 + y2 =4.<br />

Since I is in [0, 2n] , x is in [-2.2] because the<br />

graph is a circle. centered at the origin. with<br />

radius 2.<br />

12. x =../5 sinr, y =../3 COS/. for I in [0. 21t]<br />

x . y<br />

../5 =smr• ../3 = cosr<br />

Since sin 2 I + cos 2 1= 1.<br />

Since<br />

'3......<br />

......<br />

...... ......<br />

x= 3 tao r<br />

y=2secr<br />

for r io (-~, ~)<br />

~ = tan I and 1.= secI<br />

3 2'<br />

x 2<br />

and 1+tan 2 I = (ff = sec 2 I,<br />

i<br />

1+(~r =(ff<br />

5 3<br />

-+-=1.<br />

Since-d s sinr s I, -.J5 :s;../5sin/:S;../5.<br />

Therefore. x is in [-J5, ../5]. The graph is an<br />

ellipse. centered at the origin. with vertices<br />

(../5. 0), (-J5, 0), (0• ../3). (0. - ../3).<br />

13. x = 3tanI,<br />

y<br />

(lS",o)<br />

x='"ISsior<br />

y='Y3cosr<br />

for r io [0, 211'J<br />

y = 2 sec I. for I<br />

in (- ~, ~)<br />

I x = 3tanl y = 2secl<br />

-of 3tan(-t) = -3../3 2sec(-t) = 4<br />

-f 3tan(-t)=-13 2sec(-f)=¥<br />

o 3tanO=0 2secO=2<br />

~<br />

t 3tanf=../3 2sect= 4f<br />

t 3tant=3.J3 2sect=4<br />

x2 y2<br />

1+-=­<br />

9 4<br />

i ={1+ x:)<br />

r7<br />

y=2 V l + 9 ·<br />

Since this graph is the top half of a hyperbola, x is<br />

in (-00, 00).<br />

14. x=cot/. y=csct. for lin (0, It)<br />

Since 1+ cot 2 I = csc 2 I,<br />

l+x2 =y2<br />

~1+x2 =y.<br />

Since I is in (0. n) and the value of the cotangent<br />

of a value close to 0 is very large and the value of<br />

the cotangent of a value close to n is very small. x<br />

would be (--. 00). The graph is the top half of a<br />

hyperbola with vertex (0. 1).<br />

y<br />

7<br />

'/<br />

/<br />

/<br />

/ ',I<br />

x=cotr<br />

y=cscr<br />

for r in (0, 1t)<br />

358


<strong>Trigonometry</strong> <strong>Solutions</strong> <strong>Manual</strong><br />

8.6 Parametric Equations 359<br />

y=! x<br />

for x in (0, I]<br />

n. I)<br />

-t--+-:+-t-ll-t-t-t-- x<br />

o<br />

-H-t--t-ht--I-H-t--I-x<br />

o<br />

16. x = tant,y = cott, for t in (0, ~)<br />

1<br />

15. x=sint, y=csct=-.­18. x = -Ii, y = t 2 -1. for t in [0,00) .<br />

smr<br />

2<br />

Since x = -Ii, x = t, Therefore,<br />

Therefore,<br />

1<br />

y = t<br />

2 -1 = (x<br />

2)2 -1 = x<br />

4<br />

-1.<br />

y = - .<br />

x<br />

Sincetis in [0, 00),x is in [../0, ~)or[O, 00).<br />

Since t is in (0, n), x is in (sin 0, sin 1&) or (0, 1).<br />

y<br />

y<br />

19. Sincex=2+sint andy=I+cost,<br />

1 1<br />

x-2 = sint andy-I = cost.<br />

y=cott=--=tant<br />

x<br />

2<br />

Sincesin 2 t + cos t = I,<br />

Since t is in (0, ;). x is in (0, 00).<br />

y<br />

y:x 4 - t<br />

for x in [0,00)<br />

(x - 2)2 +(y _1)2 =1.<br />

Since this is a circle centered at (2, 1) with radius<br />

I, and t is in [0, 2n]. x is in [1,3].<br />

y<br />

2<br />

y= 1.<br />

x<br />

for x in (0, 00)<br />

17. Sincex=tand y=~t2+2, y=~x2+2.<br />

Since t is in (- 00)and x = t, x is in (-,00).<br />

Y<br />

+t-l+tffiP.lEt+t-t-H-t+. x<br />

Y : ~X2 + 2 for x in (_00, 00)<br />

+-t-H-f3'~I-+-+-- x<br />

o<br />

3<br />

(x _ 2)2 + (y _ t )2 : I<br />

for x in [1,3]<br />

20. x=I+2sint, y=2+3cost,<br />

for t in [0, 21&]<br />

x-I = 2sint<br />

x-I .<br />

--=smt<br />

2<br />

y-2=3cost<br />

y-2<br />

--=cost<br />

3<br />

2<br />

Since sin 2 t + cos t = I,<br />

(X~Ir +(Y;2r =1<br />

(x-I)2 (y_2)2 1<br />

~----:;,-+ ­<br />

4 9 -.<br />

Since - 1$ sin t $ I,<br />

-2 $2sint $2<br />

-1$I+2sint$3.<br />

359


<strong>Trigonometry</strong> <strong>Solutions</strong> <strong>Manual</strong><br />

360 Chapter 8 Complex Numbers and Polar Equations<br />

Therefore, x is in [-1, 3]. The graph is an ellipse<br />

with center (1, 2) and vertices (3,2), (-1, 2),<br />

(1,5), (1, -1).<br />

y<br />

23. x = sin t, y =cos t<br />

This graph is a circle centered at the origin with<br />

radius 1.<br />

y<br />

x =sin /<br />

y e cosr<br />

-t--+---::-I-+--t--- x<br />

21.<br />

(x_1)2 ~-1<br />

4 + 9 -<br />

for x in [-1, 3]<br />

Since x =t + 2 and y =_1_, y =1..<br />

t+2 x<br />

Since t ¢ -2, x 'I:- -2 + 2, x ¢ O.<br />

Therefore, x is in (-00, 0) V (0, 00).<br />

y<br />

24.<br />

~4-4t2<br />

X= t, Y = -'---­<br />

2<br />

This graph is the semicircle formed by the top<br />

half of the graph of #23 .<br />

y<br />

22.<br />

-2<br />

2<br />

2<br />

-2 y = 1 x<br />

for xin<br />

(-00,0) u (0, 00)<br />

2<br />

x=t-3, y=--, t'l:-3<br />

t-3<br />

S . 2 2<br />

mce y=--, y=-.<br />

t-3 x<br />

Since t'l:- 3, x ¢ 3 - 3 =O.<br />

Therefore, x is in (-00, 0) V (0, 00).<br />

y<br />

25.<br />

--.---::-I--+--- x<br />

-I<br />

x = /<br />

Y =-14 -41 2<br />

2<br />

x =t + 2, y = t ­ 4<br />

This graph is a line through (6, 0) and (0, -6).<br />

y<br />

:E::t-++-tAf-+-t-++4" x<br />

y = 1<br />

x<br />

for x in<br />

(--,0) u (0, 00)<br />

26.<br />

x =t 2 +2, Y =t 2 - 4<br />

This graph is a line segment which has (2, -4) as<br />

its endpoint and goes up to the right. It contains<br />

the point (6, 0).<br />

y<br />

6<br />

HH--t-H-+::ofL-i-t--t- o<br />

x<br />

360


<strong>Trigonometry</strong> <strong>Solutions</strong> <strong>Manual</strong><br />

8.6 Parametric Equations 361<br />

27. y<br />

2<br />

x = t -sin t<br />

y = 1 - cos t for t in [0, 4n)<br />

+----1f--~-l---+. x<br />

n 2n 31r 4n<br />

28. y x = 2t - 2 sin t • Y= 2 - 2 cos t<br />

for t in [0 , 81r]<br />

4<br />

For Exercises 29-32, recall that the motion of a projectile (neglecting air resistance) can be modeled by:<br />

x =(vO cos6)t, y =(vOsin6)t -16t 2 for t in [0, k).<br />

29. (a) x =(vo cosO)t y =(vo sin 6)t -16t 2<br />

=(48 cos 60 0 )t =(48sin 60 0 )t -16t 2<br />

..fj 2<br />

=48(~} =48--t-16t<br />

2<br />

=24t<br />

= -16t 2 +2Wt<br />

31. (a) x =(vo cosO)t y =(vosin6)t-16t 2 + 2<br />

=(88 cos 20 0 )t = (88sin200)t -16t 2 + 2<br />

(b) t=-- x ___<br />

88 cos 20°<br />

)-l j )2 +2<br />

Y=88sin20 0 ( x x<br />

88 cos 20° '\88cos 20°<br />

= (tan200)x- 1 2 x 2 +2<br />

484cos 20°<br />

361


<strong>Trigonometry</strong> <strong>Solutions</strong> <strong>Manual</strong><br />

362 Chapter 8 Complex Numbers and Polar Equations<br />

(c) 0=-161 2 +(88sin200)1+2<br />

-88 sin 20° ± ~r-(8-8-si-n-2-00-)2-_-4(---16-)-(2-)<br />

1 =-------:..:.....---....:....-....:....---=-~<br />

(-16)(2)<br />

(using the quadratic form ula)<br />

-30.098 ± .../905.8759 + 128<br />

=<br />

-32<br />

=:: -0.064sec, 1.95sec<br />

Discard 1=-0.064 sec since it is an unacceptable answer.<br />

At 1 = 1.95 sec<br />

x = (88 cos 20°)(1.95)<br />

=:: 161ft<br />

The softball traveled 1.95 sec and 161 feet.<br />

32. (a) x = (vo cos 0)1 y = (vo sin 0)1 -161 2 + 2.5<br />

= (136 cos 29°)1 = (136 sin 29°)1-161 2 + 2.5<br />

= -161 2 + (136sin 29°)1+ 2.5<br />

(b)<br />

x<br />

1 =---­<br />

136 cos 29°<br />

Y=(136sin29 oj x )-li<br />

'\136cos29° \<br />

= 1 2 x 2 + (tan 29°)x + 2.5<br />

1156cos 29°<br />

x )2 +2.5<br />

136cos 29°<br />

(c) 0 =-161 2 + (136sin 29°)1 + 2.5<br />

-136sin29o±~r-(1-3-6-si-n-29-0-)2-_-4-(---16-)(-2-.5-)<br />

1 =-----.!...:...---~-..:....--..:....:~.:....<br />

2(-16)<br />

(using the quadratic formula)<br />

-65.934 ± ../4347.3066 + 160<br />

=<br />

-32<br />

=::-.04, 4.2<br />

(Discard 1 = -.04 since it gives an unacceptable answer).<br />

At 1= 4.2 sec, x =(136 cos 29°)(4.2) =:: 495 ft.<br />

The baseball traveled 4.2 sec and 495 feet.<br />

33. (a)<br />

x = 82.692650631<br />

Y = -161 2 + 30.09m2611<br />

,<br />

"<br />

: .. ; ".<br />

.",1; , 200<br />

(b)<br />

x = 82.692650631 = vo(COSO)1<br />

82.69265063 = vo cosO<br />

Y = -161 2 + 30.097772611 = vo(sinO)1-161 2<br />

30.09777261 = vo sin 0<br />

30.09777261 = vosinO<br />

82.69265063 vo cosO<br />

0.3697 = tan 0<br />

0=20.0°<br />

(c)<br />

30.09777261 = vosin20.0°<br />

vo = 88.0 ftlsec<br />

x =88(cos200)1<br />

Y = -161 2 + 88(sin 20°)1<br />

362


<strong>Trigonometry</strong> <strong>Solutions</strong> <strong>Manual</strong><br />

34. (a)<br />

x = 56.56530965t<br />

y = -16t 2 + 67.41191099t<br />

l<br />

2<br />

x<br />

37. ---=1<br />

2<br />

a b 2<br />

8.6 Parametric Equations 363<br />

To find a parametric representation, let<br />

x=asecO.<br />

Then<br />

(a sec 0)2 i_<br />

a2 b2- 1<br />

(b) x =56.565309651=vo(COSO)1<br />

56.56530965 = vocosO<br />

(e)<br />

y =-161 2 +67.411910991<br />

= -16/ 2 + vo(sinOi<br />

67.41191099 = Vo sinO<br />

67.41191099 = Vo sinO<br />

56.56530965 Vo cos 0<br />

1.1918 = tan 0<br />

0=50.0 0<br />

67.41191099 = vosin 500<br />

Vo = 88.0 ftIsec<br />

x =88(cos500)1<br />

Y = -161 2 + 88(sin 50 0)1<br />

35. The equation of a line with slope m through<br />

(Xl. Yl) is<br />

Y- YI =m(x-xl)'<br />

To find two parametric representations, let x = I.<br />

Then<br />

Y-Yl =m(/-xl)<br />

Y =m(/-xI)+ YI'<br />

2•<br />

Or, let x = 1<br />

Then<br />

2<br />

Y - Yl =m(/ - xl)<br />

2-XI)+Yl'<br />

y=m(/<br />

36. Y=a(x _h)2 +k<br />

To find one parametric representation. let<br />

x e t,<br />

Then<br />

Y =a(/-h)2 +k.<br />

For another representation, let<br />

x=1 + h.<br />

Then<br />

Y =a(/+h _h)2 + k<br />

2<br />

=al +k.<br />

sec 2 0-i = 1<br />

b 2 y<br />

2<br />

20-1<br />

sec =­<br />

b 2<br />

l<br />

2<br />

tan 20=L<br />

b 2<br />

b 2tan20=y2<br />

btanO= y.<br />

2<br />

x<br />

38. -+-=1<br />

2<br />

a b 2<br />

Let x = a sin I.<br />

Then<br />

a 2 sin21 y<br />

2<br />

2<br />

a b 2<br />

----;;--+- =1<br />

2<br />

sin 2 / + L =1<br />

b 2<br />

2 ­<br />

L=I-sin 2 1<br />

b 2<br />

l = b 2 (l - sin 2 I)<br />

i =b 2 COS 2 1<br />

Y =b cost.<br />

41. The second set of equations x = cos I, Y =-sin I, I<br />

in [0, 2n] trace the circle out clockwise. A table of<br />

values confirms this.<br />

I x =cosr y= -sinl<br />

o cosO = 1 -sinO=O<br />

t cost=1/­ -sint=-!<br />

t<br />

cost=1f - sint=-4<br />

t cost=! -sini=-~<br />

t cos!=O -sin! =-1<br />

42. If x = j(I) is replaced by x = c +j(/), the graph will<br />

be translated c units horizontally. IfY = g(/) is<br />

replaced by Y = d + g(/), the graph is translated<br />

vertically d units.<br />

363


<strong>Trigonometry</strong> <strong>Solutions</strong> <strong>Manual</strong><br />

364 Chapter 8 Complex Numbers and Polar Equations<br />

Chapter RReview Exercises 11. (2 + 6i)2 = 4 + 24i + 36i 2<br />

1. H = i.J9 = 3i<br />

2. -./-12 =iM=2i-J3<br />

= 4+24i +36(-1)<br />

=-32+24i<br />

12. (6 - 3i)2 = 36 - 36i + 9i 2<br />

3. x 2 =-81 = 27 -36i<br />

x = ±-./-81 13.<br />

x =±i.J8l<br />

(1-i)3 =(1-i)2(1-i)<br />

x = ±i(9)<br />

x=±9i<br />

=(1-2i+i 2)(I-i)<br />

The solutions are 9i and -9i. = -2i(1- i)<br />

4. x(2x + 3) =-4<br />

=-2i+2i 2<br />

=-2i-2<br />

2x 2 +3x+4 =0<br />

or-2 - 2i<br />

Use the quadratic formula with a = 2, b = 3, and<br />

c=4.<br />

14. (2 + i)3 = (2 + i)2(2 + i)<br />

-3±~32-4·2·4<br />

x= = (4 + 4i + i 2)(2 + i)<br />

2 ·2<br />

=(3 + 4i)(2 + i)<br />

-3±-./9-32<br />

x = -----:.-­<br />

4<br />

= 6 + 3i + 8i + 4i 2<br />

-3±..J=23<br />

=2+ 11i<br />

x=<br />

4<br />

6 + 2i 6 + 2i 3 + i<br />

15. --=-_.­<br />

-3± i..fi3<br />

x=-----'­ 3-i 3-i 3+i<br />

4<br />

18+12i+2i 2<br />

· 3..fi3. d 3 ..fi3 .<br />

=-----,.-­<br />

The soIutions are --+--1 an -----1- 9-i 2<br />

4 4 4 4<br />

18+12i+2(-I)<br />

=<br />

5. (l -I) - (3 + 4i) + 2i 9-(-1)<br />

=(1-3)+(-1-4+2)i<br />

16+ 12i<br />

=-2- 3i =<br />

10<br />

6. (2 - 51)+ (9 - 101) - 3 8+6i<br />

=-­<br />

=(2 + 9 - 3) + (-5 - 1O)i 5<br />

=8 -15i 8 6.<br />

=-+-1<br />

5 5<br />

7. (6-5/)+(2+7i)-(3-21)<br />

=(6 + 2 - 3) + (-5 + 7 + 2)i<br />

2 - 5i (2 - 5i)(I- i)<br />

=5 +4i 16. --=~-;....;.....---'-<br />

1+ i (1+ i)(I- i)<br />

8. (4 - 2i) - (6 + 5i) - (3 - i)<br />

2 - 2i - 5i + 5i 2<br />

=(4-6-3)+(-2-5+ l)i =<br />

= -5 - 6i<br />

1-i 2<br />

-3-7i<br />

9. (3+ 5i)(8 - i)<br />

=-­<br />

2<br />

=24- 3i+ 40i - 5i 2 3 7.<br />

= 24 + 37i - 5(-1)<br />

= 29+ 37i<br />

10. (4 -i)(5+2i) =20+8i -5i -2i 2<br />

=22+3i<br />

=----1<br />

2 2<br />

364


<strong>Trigonometry</strong> <strong>Solutions</strong> <strong>Manual</strong><br />

Chapter 8 Review Exercises 365<br />

2 +; 2 +; 1+5;<br />

17. --=-_.-­<br />

1-5i 1-5i 1+5;<br />

18.<br />

2+ IIi +5i 2<br />

1-25;2<br />

2 + 11i+ 5(-1)<br />

=<br />

1-25(-1)<br />

-3 + IIi<br />

=<br />

26<br />

3 11 .<br />

=--+-1<br />

26 26<br />

3+2i (3+ 2i)(-i)<br />

(i)(-i)<br />

19. i 53 = i 52 . i<br />

-3i-2i 2<br />

=<br />

-I·2<br />

=2-3i<br />

=(;4pi<br />

=1 13;<br />

= I ·i<br />

=i<br />

or 0 +;<br />

·-41 ·-44·3<br />

20. 1 = 1 . 1<br />

= (;4)-1 I(-i)<br />

= I-II(_i)<br />

=-i or 0-;<br />

21. 1.i . i 2 . i 3 = 1. i 6 = i 6 = ;4 .;2<br />

= (1)(-1) =-1<br />

22. 1·;·i 2 ·...· i loo<br />

Notice that i 2 = -1 and i 3 = -i.<br />

Also notice that<br />

i 4 = ;8 = ;12 = I,<br />

;5 = ;9 = i l 3 = i,<br />

.6 .10 .14 1<br />

1=1 =1 =- ,<br />

and; 7 = ill = ;15 = _i.<br />

(1 ..2 .3) ('4 ·5 ·6 .7 )<br />

.1. 1 . 1 . 1 . 1 .1 .1<br />

. (i8 .;9 .;10 . ;11) ... ..;100<br />

= (1. i -(-1)(-;))<br />

.(1. i- (-1) ' (-i))<br />

. (1 . i .(-1) .(-i)) ..... iI00<br />

= (-1) · (-1) ' (-1) ' .... ;100<br />

There will be twenty-five -Is.<br />

=-I -i loo<br />

=(-1)'(1)<br />

=-1<br />

23. [5(cos 90° + i sin 90°)]<br />

{6(cos 180° + ;sin 1800)]<br />

= 5 . 6[oos(90° + 180°)<br />

+; sin(90° + 180°))<br />

= 30(cos2700 + ;sin2700)<br />

= 30(0-i)<br />

= 0 - 30i or - 30i<br />

24. [3cis135°][2cisI05°]<br />

= 3· 2cis(135° + 105°)<br />

= 6cis240°<br />

= 6(cos 240° + isin 240°)<br />

=6(-~- ~ ;)<br />

=-3-M;<br />

25.<br />

2(cos 60° + i sin 60°)<br />

8(cos 300° + sin 300°)<br />

= ...!.[cos(-2400)+ isin(-2400)]<br />

4<br />

1 .J3 .<br />

=--+-,<br />

8 8<br />

365


<strong>Trigonometry</strong> <strong>Solutions</strong> <strong>Manual</strong><br />

366 Chapter 8 Complex Numbers and Polar Equations<br />

26.<br />

4cis 270° 33.<br />

2cis90°<br />

y<br />

4 . -4 +2i<br />

=- cIs(270°- 90°) 2<br />

2<br />

+++++F!!H-H-H-f.- x<br />

= 2cis180° -4<br />

= 2(cos 180° + isin 180°)<br />

= 2(-1 + Oi)<br />

= -2+0i or -2<br />

27. (.J3 + i)3<br />

r = ~r-(.J3"""":3:-:)2"-+-17"2 = 2<br />

34. y<br />

1 .J3<br />

tan 0 = .J3 = 3""' 0 = 30°<br />

3<br />

( .J3 + i)3 = [2(cos 30° + i sin 30°)]3<br />

= 2 3(cos90° + isin 90°)<br />

=0 +8i or 8i<br />

35. (7+3i)+(-2+i)<br />

28. (2-2i)5 =(7-2)+(3+1)i<br />

=5+4i<br />

r =.J8 = 2..[i<br />

y<br />

-2<br />

tanO=-=-1<br />

2<br />

5 +4i<br />

Since 2 - 2i is in quadrant IV,<br />

0=315°.<br />

(2-2i)5<br />

= [2..[i(cos315° + isin 315°)]5<br />

= (2..[i)5[cos(5. 315°) + i sin(5· 315°)]<br />

= 128..[i(cosI575° + isin 1575°)<br />

= 12g..fi(cos 135° + i sin 135°)<br />

++++++-dG--H-H~x<br />

36. -2 + 2i<br />

r =~r-(_-2-)2=--+-27"2<br />

1284-'7 = +i '7) =../8 =2..[i<br />

=-128+128i<br />

2<br />

tanO=-=-1<br />

-2<br />

29. (cosl000+isinl000)6 ois in quadrant ITso 0 = 135°.<br />

= cos 600° + i sin 600° -2+2r<br />

1 .J3. = 2..[i(cos 135° + i sin 135°)<br />

=----1<br />

2 2<br />

37. 3(cos900+isin900)<br />

30. The vector representing a real number will lie on = 3(0+ i)<br />

the x-axis in the complex plane.<br />

= 3i or 0+ 3i<br />

32.<br />

y<br />

38. 2(cos 225° + i sin 225°)<br />

5i<br />

=2(-'7 _i~)<br />

++++++++++++--+-..- x<br />

o<br />

= --Ii -i..[i<br />

366


<strong>Trigonometry</strong> <strong>Solutions</strong> <strong>Manual</strong><br />

Chapter 8 Review Exercises 367<br />

39. -4 +4i.J3<br />

r=..J16+48 =8<br />

tan8 = 4.J3 =-J3<br />

-4<br />

8 is in quadrant II.<br />

8 = 120 0<br />

-4+4i.J3<br />

= 8(cos120 0 + i sin 120 0)<br />

40. 1-i<br />

r = ~r-12-+-(_-1)-2 =.,fi<br />

-1<br />

tan8=-=-1<br />

1<br />

8 is in quadrant IV so 8 = 315 0.<br />

1-i = .,fi(cos315° + isin 315 0)<br />

41. 4 cis 240 0<br />

= 4(cos240° + i sin 240 0)<br />

={-±-i~)<br />

:: -2-2i.J3<br />

42. -4i<br />

r=4<br />

8=2700<br />

-4i = 4(cos270 0 + i sin 270 0)<br />

43. Since the modulus of z is 2, the graph would bea<br />

circle, centered at the origin, with radius 2.<br />

44. z=x+yi<br />

Since the imaginary part ofz is the negativeof the<br />

real part of z, we are saying<br />

y=-x.<br />

This is a straight line.<br />

45. Convert -2 + 2i to polar form.<br />

r=~(_2)2+22 =.J4+4 =../8<br />

8 = arctan(:J = arctan(-I) = 135 0<br />

(since x < 0 andy> 0)<br />

then -2 + 2i = ../8(cos135°+ isin 135 0) and the<br />

5th roots of ../8(cos135°+isin135°) are:<br />

l~(cosa + isina)<br />

8+360 0k<br />

where a = for k = 0, 1, 2, 3, 4<br />

n<br />

k = 0: a = 135±360(0) = 135 = 270<br />

5 5<br />

k = 1: a = 135+ 360(1) = 495 = 990<br />

5 5<br />

k = 2: a = 135+ 360(2) = 855 = 171°<br />

5 5<br />

k = 3: a = 135+ 360(3) = 1215 = 2430<br />

5 5<br />

k = 4: a = 135+360(4) = 1575 = 3150<br />

5 5<br />

So the fifth roots of -2 + 2i are:<br />

l~(cos 27 0 + isin 27 0)<br />

l~(cos 99 0 + i sin99 0)<br />

l!Y8(cos 171 0+ isin 171 0)<br />

l~(cos 243 0 + isin 243 0)<br />

l~(cos 315 0 + isin315°)'<br />

46. Convert 1 - i to polar form<br />

r=~12+(-1)2 =.J1+l =../2<br />

8 = arctan( ~ 1 ) = arctan(-1) = 315 0<br />

(since x > 0 andy < 0)<br />

then 1-i = .,fi(cos315° + isin315°) and the cube<br />

roots of .,fi(cos315°+isin315°) are:<br />

~(cosa+isina) where<br />

8+36O° ·k<br />

a= fork=O, 1,2<br />

n<br />

k=O: a> 315+(360)(0) = 315 =1050<br />

3 3<br />

k = 1: a = 315 + (360)(1) = 675 = 2250<br />

3 3<br />

k = 2: a = 315 + (360)(2) = 1035 = 3450<br />

3 3<br />

So the cube roots of 1 - i are:<br />

~(cos 105 0 + isin 105 0)<br />

~(cos225° + isin 225 0)<br />

~(cos 345 0 + isin 345 0)<br />

367


<strong>Trigonometry</strong> <strong>Solutions</strong> <strong>Manual</strong><br />

368 Chapter 8 Complex Numbers and Polar Equations<br />

47. The real number - 32 has one real fifth root. The<br />

one real fifth root is -2, and all other fifth roots<br />

are not real.<br />

48. The number -64 has no real sixth roots because a<br />

real number raised to the sixth power will never<br />

be negative.<br />

49. x 3 + 125 = 0<br />

3<br />

x = -125<br />

-125 = -125 +Oi<br />

-125 = 125(cosI80° + isin 180°)<br />

1251/3 = 5<br />

180° + 360° . k<br />

a k =0, 1, 2<br />

3<br />

a = 60°, 180°, 300°<br />

x = 5(cos6O° + isin600),<br />

5(cos 180° + isin 180°),<br />

5(cos 300° + i sin 300°)<br />

50. x 4 + 16 = 0<br />

x 4 =-16<br />

=16(cos 180° + isin 180°)<br />

r =16, r ll4 =2<br />

_180 0+360° ·k. k=O 1 2 3<br />

a- 4 ' ' "<br />

a = 45°, 135°, 225°, 315°<br />

x = 2(cos 45° + i sin 45°),<br />

2(cos135° + isin 135°),<br />

2(cos 225° + i sin 225°),<br />

2(cos315° + isin 315°)<br />

51. x 2 +i =0<br />

x 2 =-i<br />

=O-i<br />

=1(cos 270° + i sin 270°)<br />

~=1<br />

a = 270° + 360° . k; k = 0,<br />

2<br />

a = 135°, 315°<br />

x = (cos 135° + i sin 135°),<br />

(cos315° + i sin 315°)<br />

52. r =~(_1)2 + (.J3)2 = -J1+3 =.J4 =2<br />

53. x =rcose y =rsine<br />

= 5cos315° =5sin315°<br />

5../2 ../2<br />

=-- =5-­<br />

2 2<br />

5../2<br />

=--­ 2<br />

the rectangular coordinates are:<br />

5../2 _5../2)<br />

( 2' 2<br />

54. the angle must be quadrantal.<br />

55. Since r is constant and e ~ e~ 2n, it will be a<br />

circle.<br />

56. r=4 cos e<br />

e<br />

r 4 3.5 2.8 2 0<br />

e 120° 135° 150° 180°<br />

r -2 -2.8 -3.5 -4<br />

Graph is retraced in the interval (180°, 360°)<br />

90"<br />

270"<br />

r=4cos8<br />

57. r = -1 + cos e<br />

e<br />

r o -.7 -.3 -.5 0<br />

r -1.5 -1.7 -1.9 -2 -1 -.3<br />

90"<br />

e=arctan(- ~ J=arctan(-J3) = 120°<br />

(sin x < 0 and y > 0)<br />

the polar coordinates are: (2, 120°)<br />

270"<br />

r=-I+cos8<br />

368


<strong>Trigonometry</strong> <strong>Solutions</strong> <strong>Manual</strong><br />

Chapter 8 Review Exercises 369<br />

58. r= 2 sin 4£}<br />

£} 0° 7.5° 15° 22.5° 0° 37.5° 45°<br />

r 0<br />

-13<br />

2<br />

-13<br />

1 0<br />

£} 52.5° 60° 67S 75° 82.5° 90°<br />

r -1<br />

--13<br />

-2<br />

-J3 -1 0<br />

The graph continues to form eight petals for the<br />

interval [0°,360°).<br />

90°<br />

i<br />

64. r =sin £)+ cos£}<br />

r 2 =rsin £} + rcos£}<br />

x<br />

2+l<br />

=x+y<br />

x<br />

2+l-x-y=0<br />

or<br />

65. r=2<br />

~x2+l =2<br />

x 2 +l =4<br />

r=2sin 48<br />

59. Suppose (r, £}) is on a graph. -£} reflects this<br />

point with respect to the x-axis, and -r reflects the<br />

resulting point with respect to the origin. The net<br />

result is that the original point is reflected with<br />

respect to the y-axis (B).<br />

60. If(r, £}) lies on a graph, (-r, £}) would reflect that<br />

point into the quadrant diagonal to the original<br />

quadrant. Therefore, there is symmetry about the<br />

origin (A).<br />

61. If(r, £}) lies on a graph, (r, -£}) would reflect that<br />

point across the x-axis, Therefore, there is<br />

symmetry about the x-axis (C).<br />

62. If(r, £}) lies on a graph, (r, 1t - £}) would reflect<br />

that point into the other quadrant on the same side<br />

of the x-axis as the original quadrant. Therefore,<br />

there is symmetry about the y-axis (B).<br />

3<br />

2<br />

63. r=--­r=-­<br />

J + cos£}<br />

cos£}<br />

r(1 + cose) =3<br />

I<br />

r=2·-­<br />

r+ reas£} =3<br />

cos£}<br />

r =2 sec£}o<br />

~x2+l +x=3<br />

x 2 + l =(3-x)2<br />

70. This is a circle centered at the origin with radius<br />

x 2 + l =9 - 6x + x 2<br />

2. Its equation is<br />

x 2 +l =4.<br />

l =9-6x<br />

l+6x-9=0<br />

or l =-6x+9<br />

66. y=x<br />

rsin£} = rcos£}<br />

sin£}= cos£}<br />

or<br />

tan £} =1<br />

67.<br />

y=x<br />

2<br />

rsin £} =(rcos£})2<br />

sinO<br />

r =--2- =tan £)sec £}<br />

cos £}<br />

tan £}<br />

or r= -­<br />

cos£}<br />

68. This is the line y = 2. Since y = r sin £},<br />

rsin£} =2<br />

1<br />

r=2·-­<br />

sin£}<br />

r = 2 csc£}.<br />

69. The line shown is a vertical line, x == 2. Since<br />

x = 2 and x =r cos£},<br />

rcos£} = 2<br />

Since r = ~r-x2--+y-2,<br />

r 2 =x2 + y2<br />

r 2 =4<br />

r=2.<br />

369


<strong>Trigonometry</strong> <strong>Solutions</strong> <strong>Manual</strong><br />

370 Chapter 8 Complex Numbers and Polar Equations<br />

71. x = t + cost, Y = sint for t in [0,27t]<br />

t x::::: t + cos t y::::: sinr<br />

o O+cosO:::::I sin 0 =0<br />

7t 7t 7t 7t+ 3.J3 · 7t 1<br />

-+cos-=--­ sm-=­<br />

6 666<br />

6 2<br />

7t 7t 7t 27t+ 3<br />

-+cos-=-­ sm-=­<br />

· 7t .J3<br />

3 3 3 6 3 2<br />

7t 7t 7t 7t<br />

-+cos-=- sm-= · 7t 1<br />

2 2 2 2 2<br />

37t 37t 37t 37t- 2...{i · 37t Ii<br />

-+cos-=---- sm-=­<br />

4 4 4 4 4 2<br />

7t 7t+cos7t=7t-I sin 7t= 0<br />

y<br />

I<br />

o<br />

(~ ,-1)<br />

x=t+cost<br />

y= sin t<br />

for t in 10,2n)<br />

72. x::::: 3t + 2, Y =t - 1, for t in [-5,5]<br />

Solve for tin terms of y.<br />

y+I:::::t<br />

Substitute y + 1 for t in the equation for x.<br />

x =3(y+I)+2<br />

x=3y+3+2<br />

x=3y+5<br />

x-3y=5<br />

Since t is in [-5,5], x is in [3(-5) + 2,3(5) + 2] or<br />

[-13, 17].<br />

73. x =.Jt::J" y =.Ji, for t in [1, 00)<br />

Solve for t in terms of x.<br />

x 2 =t-I<br />

x 2 +1 =t<br />

Substitute x 2 + 1 for t in the equation for y.<br />

y :::::~x2 +1<br />

Since t is in [1, 00), x is in [.JI=l, 00) or [0, 00).<br />

74. x =t 2 +5, y =~, for t in (-00, 00)<br />

t +1<br />

Then x - 5 = t 2 . Substitute x - 5 for t 2 in the<br />

equation for y.<br />

1<br />

y=-­<br />

t<br />

2<br />

+1<br />

1<br />

y=--­<br />

x-5+I<br />

1<br />

y=-­<br />

x-4<br />

2 2<br />

Since x = t + 5 and t ~ O. x ~ 0 + 5 =5.<br />

Therefore, x is in [5,00).<br />

75. x = 5 tan t, y ::::: 3 sec t, for t in ( - ~ , ~)<br />

Then<br />

~ = tant and L = sect.<br />

5 3<br />

2 2<br />

Since 1+ tan t = sec t<br />

I+(~r =(fr<br />

x2<br />

y2<br />

1+-=­<br />

25 9<br />

{I+~:)=i<br />

A=y<br />

3~I+x2<br />

25<br />

=y.<br />

S· mce t .. ( tr tr) id 5 .<br />

IS 10 - '2''2 ' an x = tan t IS<br />

undefined at - tr and tr, x is in (-00, 00).<br />

2 2<br />

76. x =cos 2t, Y = sin t for tin (-tr, n)<br />

cos 2t = cos 2 t - sin 2 t (double angle formula)<br />

Pythagorean Theorem:<br />

cos 2 t + sin 2 t =1<br />

. cos 2 t -sin 2 t+2sin 2 t =1<br />

x+2i =1<br />

2/ =I-x<br />

2 1<br />

y =-(I-x)<br />

2<br />

Since t is in e-x. n), and cos 2t is in [-1, 1], x is in<br />

[-1,1].<br />

370


<strong>Trigonometry</strong> <strong>Solutions</strong> <strong>Manual</strong><br />

77. The vertex occurs halfway between the zeros of the given equation. If y = O. then<br />

16 2<br />

0= (tanO)x - 2 2 x<br />

vo cos 0<br />

-16x 2 + v0 2(cos2 O)(tan 0) x = 0<br />

j -16x+ Vo2 cos 2 0 sin 0) = 0<br />

A\ cosO<br />

x=O. -16x+vo 2cosOsinO=0<br />

16x = Vo 2 cosOsinO<br />

vo 2 cosOsinO<br />

x =--"----­<br />

16<br />

v02 sin20)<br />

or x=~---<br />

( 32<br />

the midpoint between these values is:<br />

vQ2sinOcosO -<br />

0 2· ( 2· )<br />

=<br />

16 vo smOcosO or vo sm20<br />

2 32 64<br />

v02 sinOcosO<br />

At x =-"----­<br />

32 •<br />

= (tano{V 02 sinOcosO)_( 16 Xv 2 sinOCoso)2<br />

y 32 vo2 cos 2 0 0 32<br />

_ V0 2 sinOcosOsin 0 16vo 4 sin 2 Ocos 2 0<br />

- 32 cosO (32)2v02 cos 2 0<br />

= v 2 sin 2 0 v 2 sin 2 0 _0"'--__ 0<br />

32 64<br />

= 2vo2 sin 2 0 - Vo 2 sin 2 0<br />

64<br />

v 2 sin 2 0<br />

= 0<br />

64<br />

The vertex is:<br />

2 2 2 2<br />

V 02 sinOcosO V0 sin 0) or (V 02 sin 20 V0 sin 0)<br />

( 32 '64 64' 64<br />

78. Since x = a + rcos t, y =b + r sin t, tin [0. 21l'l. is a circle with center (a. b) and radius r<br />

a circle with center (3. 4) and radius<br />

Chapter 8 Test Exercises 371<br />

r= ~(3-0)2 +(4-0)2 =.,fi5 =5<br />

would be<br />

x =3+Scost<br />

Y =4 +Ssint<br />

tin [0. 21t]. Note that the radius is the distance between (3,4) and (0.0) since (3.4) is the center and the<br />

circle must contain the origin.<br />

80. (a) x = (vo cosO)t y = (vO sin O)t-16t 2 + 3.2<br />

= (118cos27°)t y =(118sin 27°)t -16t 2 + 3.2<br />

371


<strong>Trigonometry</strong> <strong>Solutions</strong> <strong>Manual</strong><br />

372 Chapter 8 Complex Numbers and Polar Equations<br />

(b)<br />

x<br />

t=--­<br />

118cos27°<br />

y=118sin270 ' x -16( x )2 +3.2<br />

118cos27° 118cos27°<br />

4 2<br />

= (tan 27°)x - 2 x + 3.2<br />

3481cos 27°<br />

(c) 0=(118sin27°)t -16t 2 +3 .2<br />

-118sin 27° ± ~r-(l-18-s-in-2-7-0)-2---4-(--1-6)-(3-.2-)<br />

t =------'--------.....;...;----'­<br />

2(-16)<br />

-53.571±.J2869.839 + 204.8<br />

=<br />

-32<br />

== -.059, 3.4 sec.<br />

(Discard t =-0.59 sec since it is an unacceptable answer.)<br />

At t = 3.4 sec,<br />

x = (118cos27°)(3.4)<br />

== 357 ft<br />

The baseball traveled for 3.4 sec and 357 feet.<br />

Chapter 8 Test Exercises (d) ~= 2-4i. 5-i<br />

z 5+i 5-i<br />

1. (8) W+Z =(2-4i)+(5+0<br />

=(2+5)+(-4+1)i<br />

1O-2i -20i+4;2<br />

=7-3i - 25 - 5; + 5i - i 2<br />

y<br />

10+ 4(-1) - 22;<br />

=<br />

25-(-1)<br />

6-22i .<br />

=-­<br />

26<br />

3 11 .<br />

=---1<br />

7 -3;<br />

13 13<br />

2. (a) il5 =;4 .;4 .;4 . i3<br />

(b) w-z=(2-4i)-(5+;) = 1.1.1 .i 3<br />

=(2-5)+(-4-1)i =i 3<br />

=-3-5i<br />

=-i<br />

(c) WZ :::: (2 - 4i)(5 +;)<br />

(b) (l+i)2 =O+;)(I+i)<br />

:::: 10+ 2i - 20; - 4i 2 = l+i+;+i 2<br />

= 10-4(-1)-18;<br />

=2;+1+(-1)<br />

=14-18;<br />

=2i<br />

372


<strong>Trigonometry</strong> <strong>Solutions</strong> <strong>Manual</strong><br />

Chapter 8 Test Exercises 373<br />

3. 2x 2 -x+4 = 0<br />

Using the quadratic formula :<br />

1±~(-1)2 -4(2)(4)<br />

x =--'-----'----­<br />

2(2)<br />

1±.Jl=32<br />

=<br />

4<br />

1±.J=3l<br />

=<br />

4<br />

1±i.J3i<br />

=<br />

4<br />

x ={1+ i.J3i<br />

4 '<br />

4. (a) r = ~02 + 3 2 =.J9 = 3<br />

The point (0, 3) is on the positive y-axis.<br />

Thus, () = 90°.<br />

3i = 3(cos 90° + i sin 90°)<br />

(b) r=~12+22 =-./5<br />

() = arctan(T) = arctan(2) = 63.43°<br />

(since x > 0 and y > 0)<br />

1+ 2i = .J5(cos63.43° + isin63.43°)<br />

(c) r=~(-1)2 + (-J3)2 =.../1+3 =-14 =2<br />

() = arctan( -!!) = arctan(.J3) = 240°<br />

(since (x < 0 and y < 0)<br />

-1- .J3i =2(cos 240° + i sin 240°)<br />

S. (a) a =rcos() b =rsin()<br />

= 3cos30° =3sin30°<br />

=\±)<br />

3<br />

= 2<br />

3(cos30° + isin300) = 3.J3 +~i<br />

2 2<br />

(b) a = rcos()<br />

b = rsin()<br />

= 4 cos 40° =4sin40°<br />

=3.06 = 2.57<br />

4 cis 4° = 3.06 + 2.57i<br />

(c) a = rcos() b = rsin()<br />

= 3cos90° = 3sin90°<br />

=0 =3<br />

3(cos 90° + i sin 90°) = 3i<br />

6. (a) wz= 8 .2(cos(40° + 10°)+ i sin(40° + 10°)]<br />

= 16(cos500+isin500)<br />

w 8<br />

(b) - = -[cos(40° _10°) + isin(40° _10°)]<br />

z 2<br />

= 4(cos 30° + i sin 30°)<br />

a = r cos () b = r sin ()<br />

=4cos30°<br />

=4sin30°<br />

.J3 = 4 . ..!.<br />

=4·- 2<br />

2<br />

=2<br />

=2.J3<br />

a+bi = 2.J3 +2i<br />

(c)<br />

z3 = [2(cos1Oo+ isin100)]3<br />

= 2 3(cos3 · 10° + isin3 · 10°)<br />

= 8(cos30° + isin300)<br />

a = 8cos 30° b = 8sin30°<br />

=8 . .J3 =8 . ..!.<br />

2<br />

2<br />

=4.J3<br />

=4<br />

a + bi = 4.J3 + 4i<br />

7. Put -16i into polar form<br />

r=~02+(-16)2 =../256=16<br />

() =arctan( -~ 6 ) = 270°<br />

(since tan «() is undefined and y < 0)<br />

then -16i =16(cos 270° +i sin 270°).<br />

The fourth roots of -16i have the fonn<br />

q{;(cosa+isina) where<br />

a =() + 360 0k, k =0, I, 2, 3<br />

n<br />

k = 0: a = 270° + 360(0) = 67.5<br />

4<br />

k = 1: a = 270 ± 360(1) = 157.5<br />

4<br />

k = 2: a = 270 + 360(2) = 247.5<br />

4<br />

k = 3: a = 270 + 360(3) = 337.5<br />

4<br />

The fourth roots of-16i are:<br />

2(cos67.5° + sin67.5)<br />

2(cos 157.5° + i sin 157.5°)<br />

2(cos247.5° + isin 247S)<br />

2(cos337.5° + isin 337.5°)<br />

373


<strong>Trigonometry</strong> <strong>Solutions</strong> <strong>Manual</strong><br />

374 Chapter 8 Complex Numbers and Polar Equations<br />

For Exercise 8, answers may vary.<br />

8. (a) r =~02 + 52 =.J25 =5<br />

The point (0, 5) is on the positive y-axis.<br />

Thus, 9 =90".<br />

One possibility is: (5, 90°)<br />

Alternatively, if,=-5,<br />

9 = 90 + 180 =270°<br />

A second possibility is: (-5,270°)<br />

Two pairs :<br />

(5,90°), (-5, 270°).<br />

(b) '=~(_2)2+(_2)2=.,J4+4=..[8=2.,fi<br />

9 = arctan( =~) = arctan(l) = 225°<br />

(since x < 0 and y < 0)<br />

One possibility is: (2.,fi, 225°)<br />

Alternatively, if r = -2.,fi<br />

9 = 225-180 = 45°<br />

A second possibility if: (-2.,fi, 45°)<br />

Two pairs:<br />

(2.,fi, 225°), (-2.,fi, 45°)<br />

9. (a) x ='cosO y = ,sinO<br />

= 3cos315° = 3sin315°<br />

.,fi<br />

=3·­<br />

2 ={- ~)<br />

3.,fi<br />

=-- -3.J2<br />

2 =-­<br />

2<br />

The rectangular coordinates are:<br />

3.,fi - 3.,fi).<br />

( 2' 2<br />

11. r = 3cos39<br />

90·<br />

r=l-cosO<br />

r 3 0 -2.1 -3 0 3<br />

o 135° 150 0 180 0<br />

r 2.1 0 -3<br />

Graph is retraced in the interval (180 0 , 360°).<br />

90·<br />

)/-±--,;<br />

-/.. \~ rA.'I'').,·<br />

I 7'. -< K'\ \<br />

18O·t ._-...- -' 3~ 0"<br />

-\- . Y/.<br />

/<br />

I<br />

270·<br />

r=3OO530<br />

12. This is the line where a = -1, b = 2, and c = 4<br />

The rectangular form is:<br />

-x+2y=4, so<br />

x-2y=-4<br />

y<br />

(b) x = ,cos9<br />

y = ,sinO<br />

= -4 cos 90° = -4sin 90°<br />

=0 =-4<br />

The rectangular coordinates are: (0, -4).<br />

10. ,=I-cosO<br />

r o .1 .3 .5 1.7<br />

r 2 1.7 1 .3 o<br />

374


<strong>Trigonometry</strong> <strong>Solutions</strong> <strong>Manual</strong><br />

Chapter 8 Test Exercises 375<br />

13. x = 4t ­ 3, y = t 2 for tin [-3,4]<br />

x y<br />

-3 -15 9<br />

-1 -7<br />

0 -3 0<br />

1 1<br />

2 5 4<br />

4 13 16<br />

y<br />

(13, 16)<br />

y<br />

~~<br />

x=2cos 21<br />

y =2 sin 21<br />

for 1 in [0, 21f]<br />

15. z2 - 1= (-1 + ;)2 - 1<br />

2<br />

=1-;-;+;2 -1<br />

=-2;-1<br />

= -1-2;<br />

r= ~(_1)2 +(_2)2 =../1 +4 =.J5 > 2<br />

Since r » 2, z is not in the Julia set.<br />

-16 (-3,0)<br />

x<br />

2<br />

x=41-3<br />

Y=1 2<br />

for 1in [-3.4]<br />

14. x = cos 2t, y = sin 2t for tin [0, 21t]<br />

t x Y<br />

0 0<br />

f 4­ 4­<br />

i<br />

0<br />

t -4­ 4­<br />

t<br />

-1 0<br />

-Sf -4­ -4­<br />

¥<br />

0 -1<br />

1t 1 0<br />

¥<br />

0<br />

'*<br />

-1 0<br />

1Jr.. 0 -1<br />

4<br />

21t 1 0<br />

375


<strong>Trigonometry</strong> <strong>Solutions</strong> <strong>Manual</strong><br />

376 Chapter 9 Exponential and Logarithmic Functions<br />

CHAPTER 9 EXPONENTIAL AND<br />

LOGARITHMIC FUNCTIONS 7. g(X)=(~r<br />

Section 9.1 1) -2 2=16<br />

g(-2)= ( '4 =4<br />

Connections (page 397)<br />

1 2 1 3 1 4<br />

1. 1+1+-+--+--­ 8. g(X)=(±r<br />

2 ·1 3·2 ·1 4 ·3 ·2·1<br />

15 1) -3<br />

+ ,z 2.717. g(-3)= ( '4 =4 3 =64<br />

5 ·4 ·3·2 ·1<br />

The calculator gives 2.718.<br />

9. /(x) = 3 x<br />

2. 1+ (-.05) + (-.05)2 + (-.05)3 + (_.05)4 /(1.5) = 31.5 ,z 5.196152423<br />

2 ·1 3 ·2·1 4 ·3 ·2 ·1<br />

5<br />

+ (-.05) ,z .9512.<br />

5 ·4·3 ·2 ·1 10. g(X)=(~r<br />

The calculator gives..9512.<br />

g(1.5) =(±r =i<br />

3. 6·5·4 ·3 ·2 ·1<br />

11. g(x) =(±r<br />

Exercises<br />

1. /(x) =3 x g(2.34) =(±f.34<br />

/(2)= 3 2 =9<br />

,z .0390103297<br />

2. /(x) = 3 x 12. /(x) =3 x<br />

/(3)= 3 3 =27 f(1.68) =31.68<br />

,z 6.332332457<br />

3. /(x) = 3 x 13. Yes;f(x) =aX is a one-to-one function.<br />

/(-2) =r 2 =.!.<br />

9 Therefore, an inverse function exists for f<br />

4. /(x) =3 x 14. Since f(x) =aX has an inverse, the graph of<br />

/(-3)=r 3=-.!..­<br />

5. g(x) =(~r<br />

g(2)=(.!.J2 =-.!..­<br />

4 16<br />

/-I(x) will be the reflection of/across the line<br />

27 y=x.<br />

y<br />

6. g(X)=(±r<br />

g(3)=(.!.)3 =_1<br />

15. Since /(x) =aX has an inverse, we find it as<br />

. 4 64 follows.<br />

y=a<br />

x<br />

x=a Y<br />

376


<strong>Trigonometry</strong> <strong>Solutions</strong> <strong>Manual</strong><br />

9.1 Exponential Functions 377<br />

16. Ifa =10, the equation for I-I(x) will be given by<br />

x = IO",<br />

17. Ifa =e, the equation for I-I(x) will be given by<br />

x=e Y •<br />

18. If the point (P, q) is on the graph of f, then the<br />

point (q, p) is in the graph of 1-1.<br />

19. I(x) = 3 x<br />

Make a table of values.<br />

x -2 -1 0 2<br />

1<br />

y 3 9<br />

9 3<br />

Plot these points and draw a smooth curve<br />

through them. This is an increasing function. The<br />

domain is (- 00,00) and the range is (0,00). The<br />

x-axis is a horizontal asymptote.<br />

20. I(x) = 4 x<br />

y<br />

Make a table of values.<br />

2<br />

x -2 -1 0 1 2<br />

I 1 4 16<br />

Y 16 4<br />

Plot these points and draw a smooth curve<br />

through them. This is an increasing function. The<br />

domain is (- 00,00) and the range is (0, 00). The<br />

x-axis is a horizontal asymptote.<br />

y<br />

x -2 -1 0 2<br />

1 1<br />

y 9 3<br />

3 9<br />

Plot these points and draw a smooth curve<br />

through them. This is a decreasing function . The<br />

domain is (- 00,00) and the range is (0,00). The<br />

x-axis is a horizontal asymptote.<br />

y<br />

_1 0 1<br />

3 !(x) = (Ir<br />

I 3<br />

22. I(x) =(±r<br />

Make a table of values.<br />

x -1 0 1 2<br />

I I<br />

Y 4 1 - ­<br />

4 16<br />

Plot these points and draw a smooth curve<br />

through them. This is a decreasing function. The<br />

domain is (- 00,00) and the range is (0, 00). The<br />

x-axis is a horizontal asymptote.<br />

y<br />

23. I(X)=(%r<br />

4<br />

!(x) =(~r<br />

The domain is (- 00, 00) and the range is (0, 00).<br />

Make a table of values.<br />

x -2 -1 0 2<br />

y<br />

4 2 3 9<br />

9 3 2 4<br />

21. I(X)=Gr<br />

Make a table of values.<br />

377


<strong>Trigonometry</strong> <strong>Solutions</strong> <strong>Manual</strong><br />

378 Chapter 9 Exponential and Logarithmic Functions<br />

y<br />

y<br />

!(X) = (~r<br />

5<br />

o<br />

3<br />

24. f(X)=(%r<br />

The domain is (- 00,00) and the range is (0, 00).<br />

Make a table of values.<br />

x -2 -1 o 2<br />

y<br />

9 3 2 4<br />

1<br />

4 2 3 9<br />

y<br />

27. f(x)=e- x<br />

Use a calculator to find approximate values of y .<br />

x -2 -1 o 1 2<br />

y 7.39 2.72 1 .368 .135<br />

The graph of f(x) =e- x is the reflection of the<br />

graph f(x) =eX about the y-axis.<br />

y<br />

+-+---1--::1--+--+---'1- x<br />

f(x) =e- X<br />

x<br />

25. f(x) =e<br />

The domain is (- 00,00)and the range is (0, 00).<br />

Use a calculator to find approximate values of y.<br />

x 1 0 -1<br />

y 2 .36<br />

y<br />

28. f(x) = lO-x<br />

Make a table of values.<br />

x -1 0<br />

y 10 1<br />

3<br />

1<br />

10<br />

The graph of f(x) =lO-x is the reflection of the<br />

graph of f(x) =lOX about the y-axis.<br />

y<br />

10<br />

26. f(x) =lOx<br />

The domain is (- 00,00) and the range is (0, 00).<br />

Make a table of values.<br />

x 0 -1<br />

y<br />

10<br />

1<br />

10<br />

!(x) = 10.....<br />

1<br />

_2_1 0<br />

I 2<br />

29. f(x) =2 1xl<br />

Make a table of values.<br />

x -3 -2 -1 0 2 3<br />

y 8 4 2 1 2 4 8<br />

x<br />

378


<strong>Trigonometry</strong> <strong>Solutions</strong> <strong>Manual</strong><br />

9.1 Exponential Functions 379<br />

Notice that for x < 0, Ix 1= -x, so the graph is the 32. f(x) =:' 2 x - 4<br />

This graph is obtained by translating the graph of<br />

same as that of f(x) =2- x . For x ~ 0, Ix 1= x, so<br />

f(x) =2 x down 4 units.<br />

the graph is the same as that of f(x) =2 x . Since<br />

y<br />

1- x I=Ix ~ the graph is symmetric with respect<br />

to the y-axis.<br />

y<br />

y=-4<br />

1 33. f(x) =2 x + 1<br />

o<br />

3<br />

This graph is obtained by translating the graph of<br />

f(x)=PI<br />

f(x) =2 x to the left 1 unit.<br />

30. f(x)=2-!xl y<br />

Make a table of values.<br />

x ~ -I 0 1 2<br />

y<br />

1 1 1 1 1<br />

4 2 2 4<br />

The domain is (- 00,00) and the range is (0, 1].<br />

The graph is symmetric with respect to the y-axis.<br />

14~+-I-d--+-+-+-+-+~ x<br />

y o<br />

4<br />

34. f(x) =2 x - 4<br />

This graph is obtained by translating the graph of<br />

f(x) =2 x to the right 4 units.<br />

y<br />

+-+-+--:d--+---+~- x<br />

-2 2<br />

8<br />

31. f(x)=2 x + l<br />

This graph is obtained by translating the graph of<br />

f(x) =2 x up 1 unit. 2<br />

y<br />

o<br />

4<br />

8<br />

f(x) = 2" + 1<br />

y=1<br />

H-+-H-"t-+-+-+-+-+- x<br />

-2 0 2<br />

379


<strong>Trigonometry</strong> <strong>Solutions</strong> <strong>Manual</strong><br />

380 Chapter 9 Exponential and Logarithmic Functions<br />

I)X-4<br />

35. f(X)=(~r -2 38. f(x)= ("3<br />

This graph is obtained by translating the graph of<br />

f(x) =(~r down 2 units.<br />

y<br />

This graph is obtained by translating the graph of<br />

f(x) = (~r 4 units to the right.<br />

y<br />

9<br />

!(x)=(tY-2<br />

+++++-+-H\tR--H--x<br />

4<br />

39. a =2.3<br />

36. f(X)=(~r +4 Since graph A is increasing, a > 1. Since graph A<br />

This graph is obtained by translating the graph of<br />

f(x) =(~r up 4 units.<br />

!(x)=HY+4<br />

y<br />

is the middle of the three increasing graphs, the<br />

value of a must be the middle of the three values<br />

of a greater than I.<br />

40. a = 1.8<br />

Since graph B is increasing, a > 1. Since graph B<br />

increases at the slowest rate of the three<br />

increasing graphs, the value of a must be the<br />

smallest of the three values of a greater than 1.<br />

41. a =.75<br />

y:4 Since graph C is decreasing, 0 < a < 1. Since<br />

graph C decreases at the slowest rate of the three<br />

+t-+++-H~++++-x<br />

o<br />

decreasing graphs, the value of a must be the<br />

3<br />

closest to 1 of the three values of less than 1.<br />

1)X+2 42. a =.4<br />

37. f(x)= ( "3<br />

Since graph D is decreasing, 0 < a < 1. Since<br />

The graph is obtained by translating the graph of<br />

graph D is the middle of the three decreasing<br />

f(x) =(~r 2 units to the left.<br />

graphs, the value of a must be the middle of the<br />

three values of a less than 1.<br />

y<br />

43. a =.31<br />

Since graph E is decreasing, 0 < a < 1. Since<br />

graph E decreases at the fastest rate of the three<br />

decreasing graphs, the value of a must be the<br />

closest to 0 of the three values of a less than 1.<br />

44. a =3.2<br />

+t-+++-HH-ff"l'"'l'-X<br />

Since graph F is increasing, a > 1. Since graph F<br />

2 increases at the fastest rate of the three increasing<br />

graphs, the value of a must be the largest of the<br />

three values of a greater than 1.<br />

380


<strong>Trigonometry</strong> <strong>Solutions</strong> <strong>Manual</strong><br />

9.1 Exponential Functions 381<br />

48. I(x) =x 2 . T X<br />

This function may be graphed in the standard<br />

viewing window . but a better picture of the graph<br />

is obtained by using the window<br />

[-2.8] by [-2. 5].<br />

~.....:.._--­<br />

I(x) = x2 • 2- .1<br />

-10 10<br />

46.<br />

x -x<br />

f(x) =e +e<br />

2<br />

Graph this function in the standard viewing<br />

window.<br />

e.r + e-.r<br />

I(x) =----r­<br />

10<br />

49. 4 x =2<br />

Write both sides as powers of 2.<br />

(2 2 )x =2 1<br />

2 2x =2 1<br />

2x=1 Property (b)<br />

1<br />

x=­ 2<br />

47.<br />

-10<br />

f(x) =x·2 X<br />

Although this function can be graphed in the<br />

standard viewing window. a better picture of the<br />

graph is obtained by using window<br />

[-5.5] by [-2. 5].<br />

I(x) = x· 2.1<br />

5<br />

10<br />

SO.<br />

51.<br />

125' =5<br />

Write both sides as powers of 5.<br />

(53), =51<br />

53' = 51<br />

3r= 1 Property (b)<br />

1<br />

r=­ 3<br />

(~r =4<br />

(2-1)k =2 2 .!. =2- 1<br />

2<br />

2- k =2<br />

-k=2<br />

k=-2<br />

Property (b)<br />

52.<br />

(~r =~<br />

(~r =(%r<br />

(~r =(~r2<br />

x=-2<br />

Property (b)<br />

381


<strong>Trigonometry</strong> <strong>Solutions</strong> <strong>Manual</strong><br />

382 Chapter 9 Exponential and Logarithmic Functions<br />

53. 2 3 - y =8 4 =8 213 4 =(_8)213<br />

2 3 - y =2 3<br />

3-y=3<br />

-y=O<br />

y=O<br />

54. 52p+1 =25<br />

5 2p + 1 = 52<br />

2p+l =2<br />

2p=1<br />

1<br />

p=­ 2<br />

Property (b)<br />

Property (b)<br />

= (WY = (~t=8Y<br />

=2 2 =(_2)2<br />

=4 =4<br />

Both proposed solutions check.<br />

58. z5/2 =32<br />

Raise both sides to the ~ power.<br />

5<br />

2/5<br />

( z5/2<br />

)<br />

=(32)215<br />

z=(32)2/5<br />

= (32 215)2<br />

=2 2<br />

=4<br />

59. 27 4z =9 z +1<br />

(3 3 )4Z =(3 2 )z+1<br />

312z =3 2z + 2<br />

12z =2z+2<br />

lOz=2<br />

1<br />

z=­ 5<br />

56. ..!- -_ k- 4<br />

81<br />

81- 1 =k- 4<br />

([(±3)]4fl =r 4 60. 32 1 =16 1 - 1<br />

(±3r 4 =k- 4<br />

[(±3)r 4-1I4 =(k- 4 ) 1/ 4<br />

±3=k<br />

Write both sides as powers of 2.<br />

(2 5 )1 =(2 4 )]- 1<br />

2 51 =2 4 - 41<br />

5t =4 - 4t<br />

9t=4<br />

4<br />

t=­ 9<br />

Property (b)<br />

....<br />

57. 4 =,213<br />

Raise both sides of the equation to the<br />

3/ 2<br />

since<br />

(<br />

,213<br />

)<br />

=,I =r.<br />

43i2 =,<br />

(213)3/2<br />

(±~)3 =,1<br />

~ power,<br />

2<br />

(±2)3 =,<br />

±8=,<br />

Since raising both sides of an equation to the<br />

same power may result in false "solutions," it is<br />

necessary to check all proposed solutions in the<br />

original equation.<br />

61. (~rX =(~r+1<br />

(w=[mT'<br />

(~rX =(~rX+2<br />

-x=2x+2<br />

-3x=2<br />

2<br />

x=-­ 3<br />

382


<strong>Trigonometry</strong> <strong>Solutions</strong> <strong>Manual</strong><br />

9.1 Exponential Functions 383<br />

62. (~Y-I =G~Y+I<br />

(~r~[mT+I)<br />

(~r =[(~rrl)<br />

k-I=4k-4<br />

5k=-3<br />

k=-~<br />

5<br />

63. Use the compound interest formula to find the<br />

future amount A if the present amount<br />

P = 8906.54. r= .05. m = 2. and t = 9<br />

A=~I+ ~)nn<br />

I )9(2)<br />

= (8906.54,-1+ .~5<br />

= (8906.54)(1.025)18<br />

= (8906.54)(1.5596)<br />

= 13,891.16<br />

The future value is $13.891.16.<br />

23<br />

64. P=56.780. t =-, m =4. r=.053<br />

4<br />

A=~I+ ~)un<br />

053)(2t~4)<br />

=56.78 1+-'­<br />

~ 4<br />

= 56.780(1 .01325)23<br />

=76.855.95<br />

The future value is $76.855.95.<br />

65. Use the compound interest formula to find P if<br />

11<br />

A =25.000. t =-. m = 4. and r= .06.<br />

4<br />

A=~l+ ~)nn<br />

.06)(1114)


<strong>Trigonometry</strong> <strong>Solutions</strong> <strong>Manual</strong><br />

384 Chapter 9 Exponential and Logarithmic Functions<br />

(1.4833)1/20 = (1.4833)"05<br />

'" 1.01991.<br />

1.01991=1+.!:.<br />

4<br />

.01991=.!:.<br />

4<br />

.07964 = r<br />

The interest rate , to the nearest tenth, is 8.0% .<br />

69. (a) 1200<br />

(b)<br />

(c)<br />

-1000 11,000<br />

°<br />

From the graph above, we can see that the<br />

data are not linear but exponentially<br />

decreasing.<br />

71•<br />

(c) T(5) = 1.03(5) =5.15<br />

When R = 5 w/m 2 the global temperature<br />

increase is 5.15 DF .<br />

y = 448kolS6x<br />

(a) 1990 is 10 years after 1980, so use t = 10.<br />

A(lO) = 448kO IS6(10)<br />

= 4481e,IS6<br />

'" 4481(1.168826)<br />

= 5238<br />

The function gives a population of about<br />

5238 million , which differs from the actual<br />

value by 82 million.<br />

(b) For 1995, use t = 1995 - 1980 =15.<br />

A(15) =448kO IS6(IS)<br />

=448le"234<br />

'" 4481(1.2636445)<br />

=5662<br />

The function gives a population of about<br />

5662 million.<br />

(c) For 2005, use t = 25.<br />

A(25) = 448kO IS6(2S)<br />

(d)<br />

-1000<br />

°<br />

P(x) = 1013e-·000134lx<br />

P(l5OO) = 1013e-0001341(ISOO)<br />

=828<br />

P(ll,ooo) = 1013e-·0001341(11000)<br />

=232<br />

When the altitude is 1500 m, the function P<br />

gives a pressure of 828 mb, which is less<br />

than the actual value of 846 mb. When the<br />

altitude is 11,000 m, the function P gives a<br />

pressure of 232 mb, which is more than the<br />

actual value of 227 mb.<br />

70. (a) T is a linear function, not an exponential<br />

function.<br />

(b) The graph of T(R) passes through the points<br />

(0, 0) and (20, 20.6). The slope between<br />

th . . 20.6-0 103<br />

ese points IS m = =..<br />

20-0<br />

Since the slope is 1.03 and the y-intercept is<br />

0, the equation is T(R) =1.03R.<br />

= 4481e.39<br />

'" 4481~1.416991)<br />

=6618<br />

The function gives a population of about<br />

6618 million.<br />

72. (a) T = 50,000{1 + .00)R.<br />

After 4 years, n =4.<br />

(b)<br />

T= 50,000(1+ .06)4<br />

=63,000<br />

Total population after 4 years is about<br />

63,000.<br />

T = 30,000{1 + .12)R<br />

After 3 years, n = 3.<br />

T =30,000(1 + .12)3<br />

=: 42,000<br />

There would be about 42,000 deer after<br />

3 years.<br />

384


<strong>Trigonometry</strong> <strong>Solutions</strong> <strong>Manual</strong><br />

9.2 Logarithmic Functions 385<br />

(c) T = 45,000(1 + .08)n<br />

After 5 years, n = 5.<br />

T = 45,000(1 + .08)5<br />

=66,000<br />

We can expect about<br />

66,000 - 45,000 = 21,000 additional deer<br />

after 5 years.<br />

(d)<br />

73. p(t) = 250 -120(2.8)-·51<br />

(a) P(2) =250 -120(2.8)-·5(2)<br />

=250 -120(2.8)-1<br />

=207<br />

After 2 months, a person will type about<br />

207 symbols per minute,<br />

(b) P(4) = 250-120(2.8)-,5(4)<br />

= 250 -120(2.8)-2<br />

=235<br />

After 4 months , a person will type about<br />

235 symbols per minute.<br />

(c) P(1O) = 250 -120(2.8)-·5(10)<br />

= 250 -120(2.8)-5<br />

=249<br />

After 10 months, a person will type about<br />

249 symbols per minute.<br />

74. C(x) =130,700e·027x<br />

(a) 1990 is 0 years after 1990, so use x =O.<br />

027(0)<br />

C(O) = 130,700e ·<br />

=130,700(1)<br />

= 130,700<br />

The median cost in 1990 was $130,700.<br />

(b) 1993 is 3 years after 1990, so use x =3.<br />

C(3) =130,700e,027(3)<br />

=130,700e·081<br />

= 130,700(1.08437)<br />

= 141,700 .<br />

The median cost in 1993 was $141,700.<br />

(c) 1998 is 8 years after 1990, so use x =8.<br />

C(8) =130,700e·027(8 )<br />

=130,700e·216<br />

= 130,700(1.241102)<br />

= 162,200<br />

The median cost in 1998 was $162,200.<br />

Section 9.2<br />

Connections (page 409)<br />

1. loglO 458.3 = 2.661149857<br />

+ loglO 294.6 = 2.469232743<br />

"" 5.130382600<br />

105.130382600 "" 135,015.18<br />

2. Answers will vary.<br />

Exercises<br />

1. E; log216 = 4 because2 4 = 16.<br />

2. A; logj l =0 because 3 0 =1.<br />

3. G; loglO.1=-1 because In"" =.1.<br />

4. B;log2 .J2 =.!.. because2 112 =.J2.<br />

2<br />

5. C; log1010 5 = 5 because 10 5 = 10 5.<br />

6. H; lOge(e; ) = -2 because e-2 = e12 '<br />

cr3<br />

7. F;logI/28=-3 because"2 =8.<br />

8. D; log5(-I) is not a real number.<br />

9. 3 4 = 81 is equivalent to log3 81 = 4.<br />

10. 2 5 = 32 is equivalent to log2 32 = 5.<br />

11. (~r3 = 2; is equivalentto IOg2/3(2;) = -3.<br />

12. 10- 4 = .0001 is equivalent to loglO .~1 = -4.<br />

385


<strong>Trigonometry</strong> <strong>Solutions</strong> <strong>Manual</strong><br />

386<br />

Chapter 9 Exponential and Logarithmic Functions<br />

13.<br />

14.<br />

15.<br />

16.<br />

18.<br />

19.<br />

20.<br />

log6 36 :;::; 2 is equivalent to 6 2 :;::; 36.<br />

logs 5 :;::; 1 is equivalent to 51 :;::; 5.<br />

10g.J381 :;::; 8 is equivalent to (.J3t :::: 81.<br />

I (1) . . I 4-3 1<br />

og4 64 :::: -3 IS equiva ent to :;::; 64 .<br />

loga 1= 0 for all real numbers a, because aO = 1,<br />

(a *-0) for all real numbers a.<br />

X::::IOgs(_I)<br />

625<br />

5x ::::_1_<br />

625<br />

5x __1<br />

- 54<br />

5 x = 5-4<br />

x=-4<br />

x= IOg3(8\)<br />

24.<br />

25.<br />

26.<br />

x:;::; 8 10ggll<br />

x:;::; 11<br />

log, 25 :;::;-2<br />

x- 2 :;::; 25<br />

x- 2 :::: 52<br />

(x- 2 )- 1I2 :::: (52)-112<br />

x=5- 1<br />

1<br />

x::::­<br />

5<br />

IOgxL~)=-2<br />

-2 1 x :;::;­<br />

16<br />

-2 1<br />

x =­<br />

2 4<br />

x- 2 = 2- 4<br />

(x-2)-1/2 = (r4)- 1I2<br />

x=2 2<br />

x:;::;4<br />

21.<br />

22.<br />

3 x =..!.­<br />

81<br />

3x =_1<br />

3 4<br />

3 x = 3- 4<br />

x=-4<br />

x = 10gJO .001<br />

lOX = .001<br />

lOx:::: 10-3<br />

x=-3<br />

X::::IOg 6(_I )<br />

216<br />

6x =_1_<br />

216<br />

6x =_1<br />

6 3<br />

6 x = 6- 3<br />

x=-3<br />

27.<br />

28.<br />

29.<br />

log4x = 3<br />

4 3 =x<br />

64=x<br />

log2x =-1<br />

2- 1 =x<br />

1<br />

-=x<br />

2<br />

x=log 4 Vl6<br />

4x=~<br />

4 x = (16)113<br />

4 x =(4 2 ) 113<br />

4 x = 4 2 / 3<br />

2<br />

x=­ 3<br />

23.<br />

x =2log2 9<br />

x=9<br />

386


<strong>Trigonometry</strong> <strong>Solutions</strong> <strong>Manual</strong><br />

9.2 Logarithmic Functions 387<br />

30. x =logs V25<br />

5 x =tfi5<br />

5 x =(25)114<br />

5 x =(5 2 )1/4<br />

5 x =5 2/4<br />

=<br />

5 x 5 112<br />

1<br />

x=­<br />

2<br />

32. y (2.2319281) represents the exponent to which 2<br />

must be raised in order to obtain x (5).<br />

35. fex)=llog2eX+3)1<br />

First. translate the graph of fex) =log2 x to the<br />

left 3 units to obtain the graph of log2 (x + 3).<br />

(See Exercise 34.) For the portion of the graph<br />

where f(x) ~ O. that is. where x ~ -2. use the<br />

same graph as in 35. For the portion of the graph<br />

in 35 where fex) < 0, x < -2, reflect the graph<br />

about the x-axis. In this way. each negative value<br />

of f(x) on the graph in 35 is replaced by its<br />

opposite. which is positive. The graph has a<br />

vertical asymptote at x = -3.<br />

y<br />

33. f(x) =(log2 x) +3<br />

This graph is obtained by translating the graph of<br />

f(x) =]og2 x up 3 units.<br />

y<br />

6<br />

4<br />

2<br />

+-+AIt+-+-++-+-++-+.. x<br />

o<br />

2 4 6 8<br />

36. fex)=(logll2x)-2<br />

This is the graph of fex) =10gl12 x translated<br />

down 2 units.<br />

y<br />

34. f(x) = log2(X +3)<br />

This graph is obtained by translating the graph of<br />

f(x) =]og2 x to the left 3 units. The graph has a<br />

vertical asymptote at x =-3.<br />

y<br />

+-+-+M-::+-+-+-+-H" x<br />

!(x) = log2(x+ 3)<br />

+lH\++++-H-+-++- x<br />

-4<br />

f(x) = (log 1/2 x) - 2<br />

37. f(x) =10gll2(x - 2)<br />

This is the graph of fex) =10gl12x translated to<br />

the right 2 units. The graph has a vertical<br />

asymptote at x = 2.<br />

y<br />

!(x) =log 1/2 (x - 2)<br />

2<br />

+ll-=t-t+~--H-+t+- x<br />

o<br />

-2<br />

387


<strong>Trigonometry</strong> <strong>Solutions</strong> <strong>Manual</strong><br />

388 Chapter 9 Exponential and Logarithmic Functions<br />

38. f(x) =Ilog1l2(x - 2) I 41. f(x) =logIl2(1-x)<br />

This is the same graph as 37, with the part below<br />

Make a table of values.<br />

the x-axis reflected about the x-axis.<br />

,<br />

1<br />

x -7 -3 -I 0<br />

3 7<br />

2 4 8<br />

4 I-x 8 4 2<br />

2<br />

1 1 1<br />

248<br />

-Hf:t-H-H+t-Hf-t-- x y -3 -2 -1 0 1 2 3<br />

o I 3 6 9<br />

-2 Ix=2 The graph has a vertical asymptote at x =1.<br />

(x) = /Iog 1/2(x - 2)1<br />

39. f(x) =log3 x<br />

I<br />

Write y =log3 x in exponential form as x =3 Y to<br />

find ordered pairs that satisfy the equation. It is<br />

easier to choose values for y and find the<br />

corresponding values of x. Make a table of values.<br />

y<br />

Ix= I<br />

-Hf-t-H--+-+-hI~H--- x<br />

f(x) =log IflO - x)<br />

x 1 1 1 3 9<br />

42. f(x) = log1l3(3 - x)<br />

9 3<br />

Make a table of values.<br />

y -2 -1 0 1 2<br />

The graph can also be found by reflecting the x -6 0 2 2!<br />

graph of f(x) =3 x about the line y = x. The graph 1 1<br />

3-x 9 3<br />

has the y-axis as a vertical asymptote. 3 9<br />

J<br />

y -2 -1 0 1 2<br />

f(x) = log3 x<br />

x =3 is a vertical asymptote.<br />

y<br />

2<br />

-H-±li~H-++-H-+-~ x<br />

3 9<br />

-3 2 I<br />

-2 I<br />

I<br />

40. f(x) =10gIO x I<br />

I<br />

Write y = 10gIO x in exponential form as x = lOy<br />

f(x) = log 113(3 - x)<br />

to find ordered pairs that satisfy the equation. It is<br />

easier to choose values for y and find the<br />

43. f(x) =log3(x -1)<br />

corresponding values of x. Make a table of values. To graph the function, translate the graph of<br />

f(x) =log3 x (from Exercise 39) 1 unit to the<br />

~ .001 .0 1 .1 1 10 100 1000<br />

right. The vertical asymptote will be x =1.<br />

~ -3 -2 -1 0 1 2 3 y<br />

I<br />

x =0 is a vertical asymptote.<br />

JX=I<br />

y<br />

1<br />

2<br />

2<br />

3p _<br />

- I<br />

0- , 2 S 10 x<br />

-3 1<br />

+-t-;d-,I9--+-1I-++-i-+-+-i_x I f(x) =log3(x- I)<br />

5 10<br />

-1 f(x) = log JO X<br />

44. f(x) =log2(X 2)<br />

-2<br />

Make a table of values.<br />

388


<strong>Trigonometry</strong> <strong>Solutions</strong> <strong>Manual</strong><br />

9.2 Logarithmic Functions 389<br />

1<br />

x .354 - .707 1.414 2 2.828<br />

2<br />

I 1 1<br />

xl- - - - 2 4 8<br />

8 4 2<br />

y -3 -2 -1 0 1 2 3<br />

x =0 is an asymptote. The graph is symmetric<br />

with respect to the y-axis, Sketch the graph.<br />

y<br />

so. f(x) = logz( -x)<br />

The graph of this function is the reflection of the<br />

graph of f(x) = logz x about the y-axis. Note that<br />

the x-intercept is -I since<br />

logz[-(-l)] = logj l = O. The correct graph is A.<br />

51. f(x) =x loglo x<br />

Using a graphing calculator, enter x log x for Y l­<br />

The graph is as shown.<br />

f(x) = x logJOx<br />

+-+--+-;;t--+-t~- x<br />

45. f(x) = logz x<br />

The x-intercept is 1, and the graph increases. The<br />

correct graph is E.<br />

46. f(x) =logz(2x)<br />

The graph will be similar to that of f(x) =logz x<br />

(graph E) but will increase more rapidly. Note<br />

that while the x-intercept for f(x) =logz 2x will<br />

-1<br />

52. f(x) = x Z loglO x<br />

Graph f(x) = x Z log x in the window<br />

[-1,5] by r[-.;:..,,5,'-'-Z..... ).__""'""""\<br />

f(x) = x 21ogJOx<br />

be ±' since logz ~~) = logz 1= O. The correct<br />

graph isD.<br />

47. f(X)=IOgz(~)=IOgZX-l =-Iogzx<br />

The graph will be similar to that of f(x) = logz x,<br />

but will be reflected across the x-axis. The<br />

x-intercept is 1. The correct graph is B.<br />

48. f(x) =10gz(~)<br />

1<br />

=Iogz-x<br />

2<br />

The graph will be similar to that of f(x) = logz x<br />

but will increase less rapidly. The z-intercept is 2<br />

since 10gz(~) =logz 1=O. The correct graph<br />

isC.<br />

49. f(x) =logz(x -1)<br />

The graph will be similar to that of f(x) =logi x,<br />

but will be shifted 1 unit to the right. The<br />

x-intercept is 2 since logz(2 -1) = logzl = O.<br />

The correct graph is F.<br />

-5<br />

53. Ifx and y are positive numbers,<br />

x<br />

log, - = loga x - loga y.<br />

y<br />

54. Since IOgz(1")= logz x - logz 4 by the quotient<br />

rule, the graph of y = IOgz(1") can be obtained<br />

by shifting the graph of y = logz x down<br />

logi 4 =2 units.<br />

55. Graph f(x) = 10gz(~) and g(x) = logz x on the<br />

same axes. The graph offis 2 units below the<br />

graph of g. This supports the answer in<br />

Exercise 54.<br />

389


<strong>Trigonometry</strong> <strong>Solutions</strong> <strong>Manual</strong><br />

390 Chapter 9 Exponential and Logarithmic Functions<br />

y<br />

60. log2 ( -5- 2../3J<br />

=log2 2../3 -log2 5<br />

Logarithm ofa quotient<br />

=log2 2 + log2 ../3 -log2 5<br />

Logarithm of a product<br />

1<br />

=1+-log2 3 -log2 5<br />

56. Ifx =4,Iog 2<br />

2 ~ =0; since log2 x =2 and<br />

4 Logarithm of a power<br />

log2 4 =2, log2 x -log2 4 =O.<br />

By the quotient rule, 61. log4(2x+ 5y)<br />

IOg2(~) =log2 x -log2 4.<br />

Both sides should equal O. Since 2 - 2 = 0, they<br />

do.<br />

Since this is a sum, none of the logarithm<br />

properties apply, so this expression cannot be<br />

simplified.<br />

62. log6(7m + 3q)<br />

Since this is a sum, none of the logarithm<br />

57. IOg2(6: ) properties apply, so this expression cannot be<br />

simplified.<br />

=log2 6x -log2 y<br />

Logarithm of a quotient<br />

=log2 6+log2 x-log2 y<br />

Logarithm of a product<br />

63. 10gmH<br />

, [5r 3 )112<br />

=logm -2­<br />

z<br />

58. IOg3(~) 1 5r 3<br />

=log3 4p-Iog3<br />

='2 logm q<br />

-;s<br />

Logarithm of a quotient<br />

Logarithm of a power<br />

=log-,4 + log3 P - log3 q<br />

=.!..Oogm 5r 3 -logm zS)<br />

Logarithm of a product 2<br />

Logarithm of a quotient<br />

=~ (logm5 + logm r 3 -.1og m zS)<br />

59. lOgs( 5~ J<br />

Logarithm of a product<br />

=logs(5.J7)-log S 3 1<br />

=-(logm 5+3 logm r-510gm z)<br />

Logarithm of a quotient 2<br />

=logs 5 + logs .fi-logs 3<br />

Logarithm of a power<br />

Logarithm of a product<br />

64.<br />

=1+ logs .J7- logs 3 10gbb =1<br />

=1+ logs 71/2 -logs 3 .J7 =7 112<br />

1<br />

= 1+ -logs 7 -logs 3<br />

2<br />

Logarithm of a power<br />

390


<strong>Trigonometry</strong> <strong>Solutions</strong> <strong>Manual</strong><br />

9.2 Logarithmic Functions 391<br />

65. loga x + logo Y - log, m<br />

=log, xy - logo m<br />

Logarithm of a product<br />

= 10ga(:) Logarithm of a quotient<br />

66. (log, k -10gb a) =logj, a<br />

k<br />

=10gb- - 10gb a Logarithm of a quotient<br />

m<br />

=logb(':)<br />

Logarithm of a quotient<br />

2)<br />

67. 210gma - 310gm(b<br />

2<br />

=logm a 2 -log m(b )3<br />

Logarithm of a power<br />

=logm a 2 -Iogmb 6<br />

68.<br />

=10gm(::) Logarithm of a quotient<br />

72. log10 12=loglO(3· 2 2 )<br />

= 10g1O 3+2 . log10 2<br />

=.4771+ 2(.3010)<br />

= 1.0791<br />

9<br />

73. 10g1O - =loglo 9 -loglO 4<br />

4<br />

=10g1O 3<br />

2 -log lO 2 2<br />

= 210g 1O 3-210g 10 2<br />

::: 2(.4771) - 2(.3010)<br />

=.9542 - .6020<br />

=.3522<br />

74. 10gIOG~)<br />

=10g1O 20 -loglO 27<br />

=log10 2 ·10 -log10 3 3<br />

= loglo 2 + log1010 - 3 log10 3<br />

....3010+ 1-3(.4771)<br />

=-.1303<br />

75. (a)<br />

69. 210ga(z -1) + 10ga(3z + 2)<br />

=loga(z _1)2 + loga(3z + 2)<br />

Logarithm of a power<br />

=loga[(z -1)2(3z + 2)]<br />

Logarithhm of a product<br />

1<br />

70. logb(2y + 5) - -logb(Y + 3)<br />

2<br />

2y+5<br />

=10gb 1/2<br />

(y+3)<br />

2 Y + 5 ) =10gb ~<br />

( vy+3 .<br />

71. 10glO 6 =loglO(2 · 3)<br />

=log10 2 + loglo 3<br />

=.3010 + .477 1<br />

=.7781<br />

(b) The interest rates increase with time, but not<br />

at a constant rate. Therefore, a linear function<br />

would not model the data well. The interest<br />

rates gradually level off. This resembles a<br />

shifted logarithmic function. An (increasing)<br />

exponential function does not level off, but<br />

rather continues to increase at an even faster<br />

rate.<br />

76. (a) From the graph.Iogj .3 =-1 .1<br />

(b) From the graph.IogjB =-.2<br />

77. /(3)= 2<br />

/(x) =logax<br />

2 =loga 3<br />

a 2 =3<br />

(a 2 )112 =3 112<br />

a=.,J3<br />

391


<strong>Trigonometry</strong> <strong>Solutions</strong> <strong>Manual</strong><br />

392 Chapter 9 Exponential and Logarithmic Functions<br />

(a) I(x) = log, x 3. I(x) = 5 x<br />

I(x) = log.J3 x<br />

y=5 x<br />

x = 5 Y Exchange x and y to find the inverse.<br />

~i) = IOg.J3(i)<br />

or logsx = y<br />

y = log.J39 I If I(x) = 5 x , then I-I(x) = logs x.<br />

(~r=i<br />

(3 112)y =_1<br />

3 2<br />

3y12 =r 2<br />

L=-2<br />

2 logarithm.<br />

y=-4<br />

(b) I(x) = loga x<br />

Inl2<br />

6. log3 12=-­<br />

In3<br />

I(x) = log.J3 x<br />

1(27) = log.J3 27<br />

y = log.J3 27<br />

(-\J3Y = 27<br />

4. 4l0g4Jl=1l<br />

The exponent to which 4 must be raised to obtain<br />

11 is log4 II.<br />

5. A base e logarithm is called a natural logarithm,<br />

while a base 10 logarithm is called a common<br />

7. logj O is undefined because there is no power of<br />

2 that yields a result of O.<br />

8. x = log212<br />

2 x =12<br />

(3 112)y = 3 3 Since2 3 = 8 and 2 4 = 16. log2 12 must lie<br />

3y/2 = 33 between 3 and 4.<br />

I=3<br />

2<br />

9. log 36 = 1.5563<br />

y=6 10. log 72 = 1.8573<br />

78. (a) lOOlog3 = 102log 3 11. log .042 = -1.3768<br />

= IOlog32<br />

= IO Iog9<br />

12. log .319 = -.4962<br />

=9 13. log(2 x 10 4 ) = 4.3010<br />

(b) loglo .01 3 = loglO(IO- 2)3<br />

14. log(2 x 10- 4) = -3.6990<br />

= loglOIO- 6 15. In 36 = 3.5835<br />

=-6<br />

Section 9.3<br />

Ell:ercises<br />

1. For I(x) = a", where a > 0, the function is<br />

increasing over its entire domain.<br />

2. For g(x) = log, x, where a> I, the function is<br />

increasing over its entire domain .<br />

16. In 72 = 4.2767<br />

17. In .042 =3.1701<br />

18. In .319 =-1.1426<br />

4<br />

19. In(2 x e ) = 4.6931<br />

20. In(2 x e- 4 ) = -3.3069<br />

392


<strong>Trigonometry</strong> <strong>Solutions</strong> <strong>Manual</strong><br />

9.3 Evaluating Logarithms and the Change-of-Base Theorem 393<br />

21. Grapefruit, 6.3 x 10- 4 26. Wine, 3.4<br />

pH = -log[H 30+]<br />

3.4 = -log[H 30+]<br />

= -log(6.3 x 10- 4 ) -3.4 = log[H 3O+]<br />

= -(log6.3 + loglO- 4 )<br />

[H 30+] = 10- 3 .4<br />

= -(.7793-4) [H 30+] "" 4.0 X 10- 4<br />

= -.7993+4<br />

pH=3.2 27. Beer, 4.8<br />

The answer is rounded to the nearest tenth pH = -log[H 30+]<br />

because it is customary to round pH values to the<br />

4.8 = -log[H<br />

nearest tenth. The pH of grapefruit is 3.2.<br />

30+]<br />

-4.8 = log[H 3O+]<br />

22. Crackers, 3.9 x 10-9 [H 30+]<br />

= 10-4·8<br />

pH = -log[H 3 0+]<br />

[H 30+] = 1.6 x 10- 5<br />

pH = ~log(3.9 x 10- 9 )<br />

= -(log 3.9 + log 10- 9 ) 28. Drinking water, 6.5<br />

= -(.59106-9) 6.5 = -log[H30+]<br />

=-(-8.409) -6.5 = log[H 3O+]<br />

pH=8.4<br />

[H 30+]<br />

= 10-6.5<br />

The answer is rounded to the nearest tenth<br />

because it is customary to round pH values to the [H 30+] = 3.2 X 10- 7<br />

nearest tenth. The pH of crackers is 8.4.<br />

29. Wetland, 2.49 x 10- 5<br />

23. Limes, 1.6x 10- 2 pH = -log(2.49 x 10- 5 )<br />

pH = -log[H 30+] = -(log2.49+ loglO- 5 )<br />

= -log(1.6 x 10- 2 ) = -log2.49- (-5)<br />

= -(log1.6+ log10- 2 ) =-log2.49+5<br />

=-(.2041-2) ""4.6<br />

= -(-1.7959) Since the pH is between 4.0 and 6.0, it is a<br />

pH = 1.8<br />

poor fen.<br />

The pH of limes is 1.8.<br />

30. Wetland, 2.49 x 10- 2<br />

24. Sodium hydroxide (lye), 3.2 x 10- 4 pH = -log(2.49 x 10- 2 )<br />

pH = -log[H30+] = -(1og2.49 + log10- 2 )<br />

pH = -log(3.2 x 10- 14 )<br />

= -log2.49-(-2)<br />

= -(log3.2 + log10- 14 )<br />

= -log2.49+2<br />

= -(.5052 -14)<br />

= 1.6<br />

Since the pH is 3.0 or less, it is a bog.<br />

= -(-13.49)<br />

pH=13.5<br />

31. Wetland, 2.49 x 10-<br />

The pH of sodium hydroxide is 13.5<br />

7<br />

pH = -log(2.49 x 10- 7 )<br />

... 25. Soda pop, 2.7 = -(log 2.49 + log 10- 7 )<br />

pH = -log[H 30+] = -log2.49 - (-7)<br />

2.7 = -log[H 30+] = -log 2.49 + 7<br />

-2.7 = log[H =6.6<br />

3O+]<br />

Since the pH is greater than 6.0, it is a rich fen.<br />

[H30+] = 2.0x 10- 3 32. (a) log 398.4 = 2.60031933<br />

(b) log 39.84 "" 1.60031933<br />

(c) log 3.984 "" .6003193298<br />

393


<strong>Trigonometry</strong> <strong>Solutions</strong> <strong>Manual</strong><br />

394 Chapter 9 Exponential and Logarithmic Functions<br />

33.<br />

34.<br />

(d) The whole number parts will vary, but the<br />

decimal parts will be the same.<br />

In5<br />

log25=­<br />

In2<br />

1.6094<br />

"'-­<br />

.6931<br />

'" 2.3219<br />

35. logs .59 = log .59<br />

log8<br />

-.2291<br />

"'-­<br />

.9031<br />

'" - .2537<br />

36. I ogs· 71 --­<br />

- log.71<br />

log8<br />

-.1487<br />

"'-­<br />

.9031<br />

'" -.1647<br />

37.<br />

In12<br />

log.Ji312 = In.Ji3<br />

2.4849<br />

"'-­<br />

1.2825<br />

'" 1.9376<br />

41. 21n3x = In(3x)2<br />

=ln3 2x 2<br />

= In9x 2<br />

It is the same as D.<br />

42. In4x - In2x<br />

4x<br />

=In­<br />

2x<br />

4<br />

=In­ 2<br />

= In2<br />

It is the same as D.<br />

43. In(b4~) = In(b 4aIl 2 )<br />

44.<br />

=lnb 4+lnaIl2<br />

= 4Inb+.!.lna .<br />

2<br />

1<br />

=4v+-u<br />

2<br />

a 3<br />

In2" = Ina 3 -In b 2<br />

b<br />

=3Ina-2Iob<br />

=3u-2v<br />

45. InB=In(::)'"<br />

=I{::;~)<br />

In5 =Ina 3/ 2 _lnb 5/ 2<br />

38. 10g,Ji9 5 = In.J19<br />

1.6094<br />

"'-­<br />

1.4722<br />

'" 1.0932<br />

=~Ina-~Inb<br />

2 2<br />

3 5<br />

=-u--v<br />

2 2<br />

In9<br />

log29=­<br />

In2<br />

2.1972<br />

"'-­<br />

.6931<br />

'" 3.1699<br />

..­<br />

39.<br />

40.<br />

log5<br />

log325=-­<br />

46. In(~. b 4 ) =In(a t / 3 . b 4 )<br />

. log.32<br />

.6990<br />

= Ina t/3 + Inb 4<br />

"'-­<br />

-.4949 = .!.Ina + 41n b<br />

'" -1.4125 3<br />

log8<br />

log918=-­<br />

. log.91<br />

.9031<br />

47. g(x) =eX<br />

1<br />

=-u+4v<br />

3<br />

"'-­<br />

-.0410<br />

",-22.0488 (a) By the theorem on inverses,<br />

g(lo 3) =e\n3<br />

=3.<br />

394


<strong>Trigonometry</strong> <strong>Solutions</strong> <strong>Manual</strong><br />

9.3 Evaluating Logarithms and the Change-or-Base Theorem 395<br />

(b) By the theorem on inverses,<br />

2<br />

g[ln (52)] = e'nS<br />

= 52 = 25<br />

(e) By the theorem on inverses,<br />

48. f(x) =3 X<br />

1n<br />

g[ln(;)] = e (1l e )<br />

1<br />

=-.<br />

e<br />

(a) By the theorem on inverses,<br />

1og37<br />

f(log3 7) = 3<br />

=7.<br />

(b) JIlog 3(1n<br />

3)] = 3 Iog3(ln3)<br />

=ln3<br />

(e) JIlog 3(2ln<br />

3)] = 3 Iog3(2In3)<br />

49. f(x) = Inx<br />

=21n3<br />

or In9<br />

(a) By the theorem on inverses,<br />

feeS) = Ine s<br />

=5.<br />

(b) By the theorem on inverses,<br />

f(e 1n3) = Ine 1n3<br />

=ln3.<br />

(e) By the theorem on inverses,<br />

f(e 2 In3) = Ine 21n3<br />

=21n3<br />

or In9.<br />

50. f(x) = log2 x<br />

(a) By the theorem on inverses,<br />

f(2 3) = log2 2 3<br />

=3.<br />

(b) f(210822) = log2 2'082 2<br />

= log2 2 = I<br />

(e) f(2 210 822) = log2 2210822<br />

=210g22<br />

=2·1=2<br />

51. f(x) =In Ix I<br />

The domain is (-00,0) u (0,00).<br />

The range is (- 00,00).<br />

It is symmetric with respect to the y-axis.<br />

52. f(x) =log31 x I<br />

(a) The domain off is all real numbers except 0:<br />

(-00, O)u(O, 00).<br />

(b) The graph of f(x) is given below.<br />

[ex) = log31xl<br />

4<br />

-4<br />

(e) The graph appears to show a point with<br />

x-value zero, indicating a domain of<br />

(- 00,00), which is incorrect.<br />

53. The table for Y 1 = log3(4 - x) shows "ERROR"<br />

for x ~ 4. This is because the function is<br />

undefined for x ~ 4. The domain ofy = loga x is<br />

x > 0, which means that for Y 1 , the domain is<br />

4 - x > 0, or x < 4.<br />

54. y= log, x<br />

Since 1 = log., 4, then a l = 4 or a = 4<br />

Check: 2 = log, 16, then a 2 = 16<br />

or ..r;;z =.J16 or a = 4<br />

a=4<br />

55. Let r = the decibel rating of a sound .<br />

1<br />

r= 1O ·log lO ­<br />

1 0<br />

100 ·1<br />

(a) r=IO·log 0 10--"'­<br />

1 0<br />

= 10 · log lO 100<br />

= 10·2<br />

=20<br />

1000· 1 0<br />

(b) r = 10 ·log 10 ---"­<br />

1 0<br />

= 1O ·log lO 1000<br />

=10 ·3<br />

=30<br />

395


<strong>Trigonometry</strong> <strong>Solutions</strong> <strong>Manual</strong><br />

396 Chapter 9 Exponential and Logaritbmic Functions<br />

(c) r= 10· log 10 100,000./ 0<br />

1 0<br />

=10·5<br />

=50<br />

(a) r = 10glO 1000·1<br />

1 0<br />

= 10gl01000<br />

=3<br />

(d) r= 1O·10g lO<br />

1,000,000 ./ 0<br />

1 0<br />

=10·6<br />

=60<br />

(b)<br />

r= 10g\0 1,000,000 . I<br />

1 0<br />

= 10glO 1,000,000<br />

=6<br />

(e) 1=2/ 0<br />

21<br />

r = 10· log 10 ­ 0<br />

10<br />

r = 10 . log 10 2<br />

r=3.0103<br />

The described rating is increased by 3.0103.<br />

58.<br />

56. Let d represent the loudness in decibels.<br />

I<br />

d=10 ·1og lO­<br />

1 0<br />

(a) d = 10· log 10 115./ 0<br />

1 0<br />

= 10 log 115<br />

d",21<br />

(c)<br />

100,000,000 ./<br />

r= 10glO<br />

0<br />

1 0<br />

= 10glO 100,000,000<br />

=8<br />

r=IOgIOC:J<br />

6.7 =10glO( I: )<br />

10 6 . 7 .s.<br />

1 0<br />

1 010<br />

6 .<br />

7 = I<br />

1=5,011,872/ 0<br />

. ...<br />

(b) d = 10· log 10 9,500,000./ 0<br />

1 0<br />

= IOlog lO 9,500,000<br />

d",70<br />

(c)<br />

(d)<br />

1,200,000,000./ 0<br />

d = 1O·10g 10<br />

1 0<br />

= 10 10g\0 1,200,000,000<br />

d",,91<br />

d = 1O·log lO<br />

895,000,000,000· 1 0<br />

1 0<br />

= 10 . loglO 895,000,000,000<br />

d= 120<br />

(e) d = 1O·10g 10<br />

109,000,000,000,000 ./ 0<br />

1 0<br />

= 1010g l0 109,000,000,000,000<br />

do:: 140<br />

57. Let r =the Richter scaJe rating of an earthquake.<br />

r= IOglO(~ J<br />

59. r= 10glO(~)<br />

60.<br />

8.1 = 10gl~( ~)<br />

10 8 .1 .:':<br />

1 0<br />

/ 0 10 8.1 = I<br />

1=125,892,5411 0<br />

108.1<br />

---;j'f = 10 8 . 1 - 6 . 7 = 101.4 = 25<br />

10'<br />

It was about 25 times greater.<br />

61. /(x) = -273 + 74 In x<br />

In the year 2000, x = 100.<br />

/(100) = -273 + 74 In 100<br />

=68<br />

Thus, the number of visitors in the year 2000 will<br />

be about 68 million. Beyond 1997, we must<br />

assume that the rate of increase continues to be<br />

logarithmic.<br />

396


<strong>Trigonometry</strong> <strong>Solutions</strong> <strong>Manual</strong><br />

9.3 Evaluating Logarithms and the Change-or-Base Theorem 397<br />

62. (a) /(I)=-608.5+149Inl (c) Substitute a = .88, n = 250.<br />

In the year 1999,1= 99<br />

/(99) = -608.5 + 1491n99 = 76<br />

Thus, the percent of freshmen entering<br />

college in 1999 who performed volunteer<br />

work during their last year of high school<br />

will be about 76%.<br />

63. s; = a In(1+;)<br />

= .36In(I+~)<br />

.36<br />

(8) S(I00) = .36 In( 1+ 100)<br />

.36<br />

= .36 In 278.77<br />

= (.36)(5.6304)<br />

=2<br />

(b) S(200) = .36In(1+ 200)<br />

.36<br />

= .361n(556.555)<br />

= .36(6.322)<br />

::::2<br />

(c) S(l50) = .36 In ( 1+ -150)<br />

.36<br />

= .36In417.666<br />

= .36(6.0347)<br />

::::2<br />

(d) S(IO)= .36 In(I+..!Q..)<br />

.36<br />

=.36 In 28.777<br />

= .36(3.3596)<br />

::::1<br />

S(250) = .88 In(1 + 250)<br />

.88<br />

=4.97<br />

The equation predicts there will be 5 species.<br />

65. The index of diversity H for 2 species is given by<br />

H = -[1\ log21\ + P2 log2 P 2 ]<br />

50<br />

1\ =­ 100<br />

=.5<br />

50<br />

P 2 = ­ 100<br />

= .5<br />

Substituting into the formula gives<br />

H = -[.5log2 .5 + .5 1og 2 .5].<br />

Since log2 .5 = log2 .!. = -1, we have<br />

2<br />

H = -{ .5(-1)+ .5(-1)]<br />

=-{-I)<br />

=1<br />

The index of diversity is 1.<br />

66. H = [1\ log21\ + P 2 log2 P 2<br />

+ P:3 log2 P:3 + P 4 log2 P 4 ]<br />

= -[.5211og 2 .521 + .324Iog 2.324<br />

+ .081log2 .081+ .0741og 2 .074]<br />

=1.59<br />

The index of diversity is 1.59.<br />

67. From Example 5,<br />

T(R) = 1.03R<br />

and R=kln( ~}<br />

By substitution. we have<br />

64. S(n)=aln(I+;)<br />

(8) Substitute a = .88, n = 50.<br />

S(50) = .88 In( 1+ 50)<br />

.88<br />

=3.57<br />

The equation predicts there will be 4 species.<br />

(b) Substitute a = .88, n =100.<br />

S(I00)=.881n(l+ 100)<br />

.88<br />

=4.17<br />

The equation predicts there will be 4 species.<br />

T(k) =1.03k In( ~ )­<br />

Since 10 s k ~ 16 and ( ~) = 2, the range for<br />

T = 1.03k In( ~) will be between<br />

T = 1.03(1O)ln2 = 7.1<br />

and T = 1.03(16) In 2 = 11.4.<br />

The predicted increased global temperature due to<br />

the greenhouse effect from a doubling of the<br />

carbon dioxide in the atmosphere is between<br />

7°F and 11°F.<br />

397


<strong>Trigonometry</strong> <strong>Solutions</strong> <strong>Manual</strong><br />

398 Chapter 9 Exponential and Logarithmic Functions<br />

68. (a) C(x) =353(1.006}~-1990<br />

(b)<br />

T(x) =6.489In(.£)<br />

280<br />

=6.489In[353(1 .006)X-I990]<br />

280<br />

2100<br />

69. (a) The table of natural logarithms takes the<br />

following form. Let x =In D and y = In P for<br />

each planet.<br />

Planet InD InP<br />

Mercury - .94 -1.43<br />

Venus - .33 -.48<br />

Earth 0 0<br />

Mars .42 .64<br />

Jupiter 1.65 2.48<br />

1990 1:.: • •:....:....;.:;........:..~=--,:;,:..;~...J "<br />

2275<br />

300<br />

T(x) = 6.489In(2~ ).Where<br />

C(x) is defined as given earlier<br />

Saturn 2.26 3.38<br />

Uranus 2.95 4.43<br />

Neptune 3.40 5.10<br />

Plot this data in the window [-2,4] by<br />

[-2,6].<br />

C is an exponential function and T is a linear<br />

function over the same time period. While<br />

the carbon dioxide levels in the atmosphere<br />

increase at an exponential rate, the average<br />

global temperature will rise at a linear rate.<br />

(c) The slope of the graph of T can be<br />

approximated using two points. Since<br />

T(2000) == 1.89 and T(2100) == 5.77 , the slope<br />

. _ 5.77 -1.89 _ 0388<br />

IS m- -..<br />

2100-2000<br />

This means that temperature is expected to<br />

rise at an average rate of .04°P per year from<br />

1990 to 2275.<br />

(d) Graph y = T(x) and y = 10 on the same<br />

screen. Their graphs intersect when<br />

.... x == 2208.9. C(2208.9) :: 1308 ppm. which<br />

would be about 4.67 times above<br />

preindustrial carbon dioxide levels.<br />

-2<br />

Yes, the data points appear to be linear.<br />

(b) Choose two points from the table showing<br />

In D and In P and find the equation through<br />

them. If we use (0, 0), representing Earth and<br />

(3.40,5.10) representing Neptune, we obtain<br />

m= 5.10-0 =1.5.<br />

3.40-0<br />

Since the y-intercept is O. the equation is<br />

y =1.5x<br />

or In P =1.5 In D.<br />

Since the points lie approximately but not<br />

exactly on a line, a slightly different equation<br />

will be found if a different pair of points is<br />

used.<br />

6<br />

-21--~o':---"""'---"1 4<br />

-2<br />

398


<strong>Trigonometry</strong> <strong>Solutions</strong> <strong>Manual</strong><br />

9.4 Exponential and Logarithmic Equations 399<br />

Section 9.4<br />

Exercises<br />

(c) For Pluto, D = 39.S, so<br />

InP= l.5lnD<br />

= I.S In 39.5<br />

Then<br />

P = e1.5ln39.5<br />

1039.51.5<br />

=e<br />

=(39.5)1.5<br />

:::: 248.3.<br />

The linear equation predicts that the period<br />

of the planet Pluto is 248.3 years, which is<br />

very close to the true value of 248.5 years.<br />

1. Since x is the exponent to which 7 must be raised<br />

in order to obtain 19, the solution is<br />

log19 In19<br />

Iog7 19 or-- or --.<br />

log7 1n7<br />

2. Since x is the exponent to which 3 must be raised<br />

in order to obtain 10, the solution is<br />

10g1O In10<br />

Iog3 10 or-- or -­<br />

log3 In3<br />

3. Since x is the exponent to which .!.. must be<br />

2<br />

raised in order to obtain 12, the solution is .<br />

log12<br />

In12<br />

logl/2 12 or -(.1) or (.1)<br />

log 2 In I<br />

4. Since x is the exponent to which .!. must be raised<br />

3<br />

in order to obtain 4, the solution is<br />

log 4 In4<br />

4<br />

logl/3 or log(t) or In(t)'<br />

6. 4 x = 12<br />

Take natural logarithms (base e) of both sides<br />

In4 x = In12<br />

x In4 = In 12 Logarithm of a power<br />

In12<br />

x=-­<br />

In4<br />

2.4849<br />

x=--­<br />

1.3863<br />

==: 1.792S<br />

7. 6 1 - 2x =8<br />

Take base 10 (common) logarithms of both sides.<br />

(This exercise can also be done using natural<br />

logarithms .)<br />

log6 1 -<br />

2x = log8<br />

(l-2x)log6 = log8<br />

1-2x = log8<br />

log6<br />

log 8<br />

2x = 1_<br />

log 6<br />

8. 3 2x - 5 =13<br />

x = .!.(1- IOg8)<br />

2 log6<br />

In3 2x - 5 = In13<br />

(2x - S) . In 3 = In13<br />

==: .!.(1- .9031)<br />

2 .7782<br />

==: -.0803<br />

2x-S = In 13<br />

In3<br />

2x =S+ In13<br />

In3<br />

x = .!..[s+ In13]<br />

2 In3<br />

x==: 3.6674<br />

5. 3 x =6<br />

Take base e (natural) logarithms of both sides.<br />

In3 x = In6<br />

x In3 = In6 Logarithm of a power<br />

In6<br />

x=­<br />

In3<br />

1.7918<br />

=-­<br />

1.0986<br />

:::: 1.6309<br />

9.<br />

2 x + 3 =SX<br />

log2 x + 3 = 10gSX<br />

(x+3)log2 = xlogS<br />

xlog2+310g2 =xlogS<br />

xlog2 -xlogS = -310g2<br />

x(log2 -logS) = -310g2<br />

x = _-_3_lo....:g::,.2_<br />

log2-logS<br />

x ==:2.2694<br />

399


<strong>Trigonometry</strong> <strong>Solutions</strong> <strong>Manual</strong><br />

400 Chapter 9 Exponential and Logarithmic Equations<br />

10. 6 x + 3 =4 x<br />

log6x+3 = log4 X<br />

(x+3)log6 = xlog4<br />

x log6 + 310g6 = x log4<br />

xlog6-xlog4 = -3 log 6<br />

x(log6-log4) = -310g6<br />

-310g6<br />

x= log6-log4<br />

x =-13.2571<br />

x-l<br />

11. e =4<br />

Ine x-l = In4<br />

x-l= In4<br />

x = In4+1<br />

= 1.3863+ 1<br />

=2.3863<br />

2 x<br />

12. e - =12<br />

Ine 2 - x =lnl2<br />

(2 -x) ·Ine = In12<br />

2-x = In12<br />

x=2-ln2<br />

x =-.4849<br />

13. 2e Sx + 2 =8<br />

e Sx + 2 =4<br />

Ine Sx + 2 = In 4<br />

5x+2=ln4<br />

5x=ln4-2<br />

1<br />

x = -(ln4-2)<br />

5<br />

... .!. (1.3863 - 2)<br />

5<br />

= -.1227<br />

14. 10e 3x - 7 = 5<br />

In(1O 'e 3x-7 ) = In5<br />

InlO+ Ine 3x-7 = In5<br />

In1O+(3x -7)-Ine = In5<br />

3x - 7 =In5 - In 10<br />

3x = In5 -In 10 + 7<br />

1<br />

x =-[ln5 -In 10+ 7]<br />

3<br />

x =2.1023<br />

15. 2 x = -3 has no solution since 2 raised to any<br />

power is positive.<br />

8x 2x 20<br />

17. e . e = e<br />

lOx 20<br />

e =e<br />

lOx =20<br />

x=2<br />

18. e<br />

6x<br />

. eX =e<br />

2l<br />

7x 21<br />

e =e<br />

7x=21<br />

x=3<br />

19. loo(1.02)x/4 = 200<br />

(1.02)x/4 =2<br />

(~) log 1.02 = log2<br />

x log2<br />

-=--=-­<br />

4 log 1.02<br />

x =4 log2<br />

log 1.02<br />

x'" 140.0112<br />

20. 500(1.05)x/4 =200<br />

(1.05)x/4 =~<br />

5<br />

In(1.05)x/4 = In~<br />

5<br />

x 2<br />

- In(I .05) =In­<br />

4 5<br />

x<br />

In!<br />

21. logx + log(x - 21) =2<br />

log[x(x - 21)]=2<br />

-=-­<br />

4 In1.05<br />

lln.4 )<br />

x =\ln1.05<br />

x'" -75.1209 ­<br />

10 2 =x(x - 21)<br />

loo=x 2-21x<br />

0=x 2 -2lx-loo<br />

0= (x - 25)(x+4)<br />

x - 25 = 0 or x + 4 = 0<br />

x=250rx=-4<br />

Since x =- 4 is not in the domain of log x or<br />

log (x - 21), it cannot be used. So x =25.<br />

16. 3 x =-6<br />

Since 3 raised to a power will never result in a<br />

negative value, this equation has no solution.<br />

400


<strong>Trigonometry</strong> <strong>Solutions</strong> <strong>Manual</strong><br />

22. log x + log(3x - 13 = 1<br />

log[x(3x -13)] = 1<br />

9.4 Exponential and Logarithmic Equations 401<br />

27. SX+2 =2 2x-1<br />

logSx+2 = log 2 2x - 1<br />

WI = x(3x -13)<br />

(x+2)logS =(2x -1)log2<br />

1O=3x 2 -13x<br />

xlogS+ 210gS = 2xlog2-log2<br />

O=3x 2 xlogS-2xlog2 = -2 logS -log2<br />

-13x-1O<br />

x(logS-210g2) = -2 logS -log2<br />

O=(3x+2)(x-S)<br />

x = -210gS-log2<br />

3x + 2 =0 or x - S=0<br />

logS -210g2<br />

2<br />

x=-- or x=S<br />

x ...-17.S314<br />

3<br />

Since x = -~ is not in the domain of<br />

28. 6 x ­<br />

3 =3 4x + 1<br />

3<br />

log6<br />

log x or log (3x - 13). it cannot be used. So x X 3<br />

- = log3<br />

= S.<br />

4x+1<br />

(x - 3)log6 = (4x + 1)log3<br />

23. In(S+ 4x) -10(3 + x) = In3<br />

xlog6-310g6 = 4xlog3+ log 3<br />

xlog6-4xlog3 = 310g6+ log3<br />

In S+4x =ln3<br />

x(log6-410g3) = 310g6+ log 3<br />

3+x<br />

S+4x =3<br />

x = 310g6+ log3<br />

3+x<br />

log6-410g3<br />

S+4x=3(3+x)<br />

x ...-2.4874<br />

S+4x =9+3x<br />

x=4<br />

29. In eX -lne 3 = Ine s<br />

x-3=S<br />

24. In(2x+S)+ Inx = In7<br />

x=8<br />

In[x(2x + S)] = In7<br />

x(2x+S) =7<br />

30. ln e" -2Ine= Ine 4<br />

2x 2 +Sx-7 =0<br />

eX<br />

(2x+ 7)(x -1) =0<br />

In-=lne<br />

2<br />

4<br />

e<br />

7<br />

x=-- orx=1<br />

4<br />

2<br />

:: =e<br />

We reject _2, since it is not in the domain of<br />

eX =e 6<br />

2<br />

either In x or In (2x + S). So x = 1.<br />

x=6<br />

31.<br />

25. log6 4x -log 6 (x - 3) = log6 12<br />

log2(log2 x) = 1<br />

4x<br />

log2(log2 x) = log2 2 since log22 = 1<br />

log6--= log6 12<br />

log2 x=2<br />

x-3<br />

x=4<br />

~=12<br />

x-3<br />

4x= 12(x-3)<br />

32. log x = .Jlog x<br />

4x=12x-36<br />

(logx)2 = (.Jlogx)2<br />

36=8x<br />

36<br />

(logx)2 = log x<br />

-=x<br />

8<br />

(logx)2 -Iogx = 0<br />

4.5=x<br />

logx(logx-l) = 0<br />

log x = 0 or log x -1 = 0<br />

26. log2 3x + log2 3 = log2(2x + IS)<br />

log x = 1<br />

log2[3x(3)] = log2(2x + IS)<br />

3x(3)=2x +<br />

10° =x WI =x<br />

IS<br />

l=x or 10=x<br />

9x=2x+lS<br />

7x= IS<br />

IS<br />

x=­ 7<br />

Since the work involves squaring both sides, both<br />

proposed solutions must be checked in the<br />

original equation. Both answers check.<br />

401


<strong>Trigonometry</strong> <strong>Solutions</strong> <strong>Manual</strong><br />

402 Chapter 9 Exponential and Logarithmic Equations<br />

33. 10gx Z =(logx)Z<br />

210gx =(logx)Z<br />

Let w = logx.<br />

2w=w z<br />

o=w z -2w<br />

0=w(w-2)<br />

w=O or w=2<br />

Replace w with log x to find the values for x.<br />

o= log x or 2 =log x<br />

1=x or 100 =x<br />

34. logz ~2xZ = ~<br />

2<br />

2 3 /2 =~2xZ<br />

(2 3IZ)Z =(~2XZ)Z<br />

2 3 =2x Z<br />

8=2x Z<br />

4=x Z<br />

±2=x<br />

Since the solution involves squaring both sides,<br />

both proposed solutions must be checked in the<br />

original equation. Both answers check.<br />

36. loga(4x -7) + 10ga(x Z + 4) = 0<br />

log, t is defined when t is in the interval (0, 00),<br />

but it is undefined when t is in the interval<br />

(- 00,0]. Sincex z + 4 is positive for all values of<br />

.r, loga(x Z + 4) will be defined for all values of x.<br />

However, loga(4x -7) will be undefined when<br />

4x - 7 is in the interval (- 00,0], which<br />

corresponds to 4x - 7 S 0<br />

4xS7<br />

7<br />

xS-.<br />

4<br />

Thus, the numbers in the interval ( -00, ~J could<br />

not be solutions of the given equation.<br />

37. j(x) =e x + 1 - 4<br />

x +<br />

I<br />

y=e - 4<br />

x =e Y +I - 4 Exchange x and y.<br />

y<br />

x+4 =e +<br />

1<br />

y 1<br />

In(x + 4) =In e +<br />

In(x+ 4) =y+ 1<br />

In(x + 4)-1 =y<br />

j-l(x) =In(x + 4)-1<br />

Domain: (-4,00)<br />

Range: (- 00,00)<br />

38. j(x) = 21n3x<br />

y = 21n3x<br />

x = 21n3y Exchange x and y.<br />

x<br />

-=ln3y<br />

2<br />

e X<br />

Z 1n 3y<br />

/ = e<br />

e X / Z = 3y<br />

e X / Z<br />

--=y<br />

3<br />

x/Z<br />

j-I(x)=_e_<br />

3<br />

Domain: (- 00, 00)<br />

Range: (0, 00)<br />

39. eX +lnx=5<br />

Graph Y 1 = eX + In x and Yz = 5 on the same<br />

screen.<br />

Using the "intersect" option in the CALC menu,<br />

we find that the two graphs intersect at<br />

approximately (1.52, 5). The x-coordinate of this<br />

point is the solution of the equation. So x::: 1.52.<br />

40. e" -In(x + 1) =3<br />

Graph Y 1 = eX -In(x+ 1) and Yz =3 on the<br />

same screen.<br />

Use the "intersect" option in the CALC menu.<br />

The intersection points are at x « -.93 and<br />

x::: 1.35.<br />

41. 2e x +1=3e- x<br />

Graph Y 1 =2e x + 1 and Y z =3e- x on the same<br />

screen. The two curves intersect at the point (0,<br />

3). The x-coordinate of this point is the solution<br />

of the equation. So x =O.<br />

42. eX +6e- x =5<br />

Graph Y 1 =eX+ 6e- x and Yz = 5 on the same<br />

screen.<br />

The intersection points are at x ::: .69 and x::: 1.10.<br />

43. logx = x Z -8x+ 14<br />

Graph Y 1 =logx and Yz =x Z -8x+ 14.<br />

The intersection points are at x = 2.45 and<br />

X= 5.66.<br />

44.lnx=-'J.jx+3<br />

Graph Y 1 = In x and YZ =--l[;+3.<br />

The intersection point is at x ::: .23.<br />

402


<strong>Trigonometry</strong> <strong>Solutions</strong> <strong>Manual</strong><br />

9.4 Exponential and Logarithmic Equations 403<br />

45. Double the 1998 value is 2(97 .3) = 194.6.<br />

48. f(x) =-301ln(~)<br />

f(x) =23 · 1.2 x 207<br />

194.6 =23 ·1.2 x<br />

194.6 =1.2x<br />

23<br />

194.6 1 12x<br />

1ogn= og .<br />

194.6<br />

log-- =x log 1.2<br />

23<br />

10g.l!M& ______2""3_ =x<br />

log 1.2<br />

x == 11.7<br />

1990+ 11.7 = 2001.7<br />

During 2001, the worldwide shipments will be<br />

double their 1998 value.<br />

46. (a) At the finish line t = 9.86.<br />

(b)<br />

f(t) =11.65(1-e- tI 1. 27 )<br />

f(9.86) =11.65(1-e-9·86/1.27)<br />

f(9.86) == 11.6451<br />

He was running 11.6451 meters per second<br />

as he crossed the finish line.<br />

f(t)=11.6~I-e-tI1.27)<br />

10 =11.6* _e- tI 1. 27 )<br />

~ =l_e- tl l. 27<br />

11.65<br />

e- tl 1. 27 =1-~<br />

11.65<br />

Ine- t l 1.27 =In(I-~)<br />

11.65<br />

- 1.~7 =In(1- 1:~5)<br />

I = -1.27In(I-~)<br />

11.65<br />

I"" 2.4823<br />

After 2.4823 seconds he was running at a<br />

rate of 10 meters per second.<br />

(a) The left side is a reflection of the right side<br />

across the vertical axis of the tower; the<br />

graph of f(-x) is exactly the reflection of the<br />

graph of f(x) across the y-axis.<br />

(b) x is half the length<br />

x = 15.7488 =7.8744<br />

2<br />

f(7 .8744) =-301ln 7.8744<br />

207<br />

y==984<br />

The height of the Eiffel Tower is 984 feet.<br />

(c) Let y =500 and solve for x.<br />

500=-301ln~<br />

207<br />

500 x<br />

--=In­<br />

-301 207<br />

eSOO/-301 =eln(xI207)<br />

eSOO/-301 =.z,<br />

207<br />

207eSOO/-301 =x<br />

x=39.<br />

The height is 500 feet, about 39 feet from the<br />

center.<br />

49. (a) In(1- P) = -.0034 ­ .0053T<br />

Change this equation to exponential form<br />

1- P =e- .OO34-.00S3T<br />

(b)<br />

P(T) =1-e- .OO34-.0034T<br />

47. f(x) = 25<br />

1+ 1364.3exI9.316<br />

In 1997, x =97.<br />

f(97) _ 25<br />

- 1+1364.3e-97/9.316<br />

f(97) =24<br />

In 1997, about 24% of U.S. children lived in a<br />

home without a father.<br />

(c)<br />

From the graph one can see that initialIy<br />

there is a rapid reduction of carbon dioxide<br />

emissions. However, after a while there is<br />

little benefit in raising taxes further.<br />

P(T) =1-e-·OO34-.00S3T<br />

P(60) =1-e-·OO340.00S3(60)<br />

== .275 or 27.5%<br />

The reduction in carbon emissions from a tax<br />

of $60 per ton of carbon is 27.5%.<br />

403


<strong>Trigonometry</strong> <strong>Solutions</strong> <strong>Manual</strong><br />

404 Chapter 9 Exponential and Logarithmic Equations<br />

(d) We must determine T when P = .5.<br />

pen = 1-e-.OO34-.0053T =.5<br />

52. A =~1+ ~r"<br />

.5 = 1-e-.OO34-.0053T To solve for t, substitute<br />

.5 = e-.OO34-.0053T A =2063.40, P =1786, r = .116, and m = 12.<br />

In.5 = -.0034 - .0053T rJ 6)(')(12)<br />

2063.40 =178\.1 + .~~<br />

T= In.5+.0034 == 130.14<br />

-.0053<br />

The value T= $130.14 will give a 50%<br />

reduction in carbon emissions.<br />

2063.40 ( .116)12'<br />

---= 1+-­<br />

1786 12<br />

In 2063.40 = 121 In(1 + .116)<br />

1786 12<br />

50. (a) Letting k =6.3 and ( ~ ) = 2 results in In~<br />

----;'.......~~ =1<br />

12In(I+,*)<br />

R = k In( ~ ) = 6.3 In 2 1.25 == 1<br />

To the nearest hundredth, 1 =1.25 yr.<br />

w<br />

==4.4---y.<br />

m 53.<br />

(b)<br />

T(R) =1.03R<br />

T(4.4) = 1.03(4.4) == 4.5°F<br />

This is less than that predicted by Arrhenius<br />

in 1896; however, his values are still<br />

consistent with some current computer<br />

models.<br />

To solve to I, substitute<br />

A =30,000, P =27,000, r =.06, and m =4.<br />

tvvI 06)(')(4)<br />

30,000=27,\1+ 7 The interest rate is about 6.48%.<br />

1.1111,.. (1.015)4'<br />

In1.1111 = In(1.015<br />

4,) 54.<br />

In1.1111 = 41 In 1.015<br />

Inl.l111<br />

t<br />

41n1.015<br />

1.8", t<br />

To the nearest tenth of a year, Tom will be ready<br />

to buy a car in 1.8 yr.<br />

The interest rate is about 4.27%.<br />

404


<strong>Trigonometry</strong> <strong>Solutions</strong> <strong>Manual</strong><br />

55. (a) At the age of 60 your investment of $2000<br />

has earned interest for 35 years .<br />

Your investment has grown to<br />

A=~I+ .r<br />

.08)(1)(35)<br />

A =2 1+­ 000(<br />

1<br />

=29,570.69<br />

The tax is 40% of this, or<br />

(.40)(29,570.69) = 11,828.28<br />

So you have $17,742.41 left.<br />

(b) A=~I+ ~rl andr=.048,P=1200,<br />

A = 1200(1 + .048)(1)(35) = $6191.93<br />

(c) With the IRA you earn<br />

17,742.41 - 6191.93 = $11,550.48 more.<br />

(d) If you pay taxes on the original $2000, then<br />

you have $1200 to invest.<br />

A= ~1+ ~)mt<br />

.08)(1)(35)<br />

A=12 00( 1+­l<br />

1<br />

= 17,742.41<br />

This is the same as with the IRA.<br />

Chapter 9 Review Exercises<br />

1. y= log.3x<br />

The point (1, 0) is on the graph of every function<br />

of the form y =loga x, so the correct choice must<br />

beeither B or C.<br />

Since the base is a = .3 and<br />

o< .3 < I, y =log.3 x is a decreasing function,<br />

and so the correct choice must be B.<br />

Chapter 9 Review Exercises 405<br />

3. y = Inx<br />

or<br />

y = loge x<br />

The point (1, 0) is on the graph of every function<br />

of the form y = log., x, so the correct choice must<br />

beeither B or C.<br />

Since the base is a = e and e > I, y = In x is an<br />

increasing function, and so the correct choice<br />

must be C.<br />

4. y =(.3)x<br />

The point (0, 1) is on the graph since (.3)° =1, so<br />

the correct choice must beeither A or D.<br />

Since the base is .3 and 0 < .3 < I, y =(.3)x is a<br />

decreasing function, and so the correct choice<br />

must be D.<br />

5. 2 5 = 32 is written in logarithmic form as<br />

log2 32 =5.<br />

6. 100 1/2 =10 is written in logarithmic form as<br />

1<br />

logloo 10 =2'<br />

7. (%r =~ is written in logarithmic form as<br />

8.<br />

IOg3/4(~) =-1.<br />

y<br />

+--+-,..+--+-+-+-1- .x<br />

2. y=e x<br />

The point (0, 1) is on the graph sincee'' = I, so<br />

the correct choice must be either A or D.<br />

Since the base is e and e> l.y = eX is an<br />

increasing function and so the correct choice must<br />

be A.<br />

9.<br />

2<br />

y<br />

!(x) = 108) x<br />

+---!~-+-+---1_ x<br />

10 20 30<br />

-1<br />

10. log3 4 is the logarithm to the base 3 of 4.<br />

(log, 3 would be the logarithm to the base<br />

40f3.)<br />

405


<strong>Trigonometry</strong> <strong>Solutions</strong> <strong>Manual</strong><br />

406 Chapter 9 Exponential and Logarithmic Equations<br />

11. The exact value oflog-, 9 is 2 since 3 2 =9.<br />

12. The exact value oflog-, 27 is 3 since 3 3 =27.<br />

13. log316 must lie between 2 and 3. Because the<br />

function defined by y =log3 x is increasing and<br />

9 < 16 < 27, we have logj 9 < log316 < log, 27.<br />

14. By the change-of-base theorem,<br />

_ log16 _ In16<br />

Iog3 16-----­<br />

log3 In3<br />

== 2.523719014.<br />

This value is between 2 and 3, as predicted in<br />

Exercise 21.<br />

15. The exact value of logj (~) is -1 since 5- 1 =~.<br />

The exact value oflog, 1 is 0 since 5° =1.<br />

16. logs .68 must lie between -I and O.Since the<br />

function defined by y =logs x is increasing and<br />

I<br />

- = .2 < .68 < 1, we must have<br />

5<br />

logs .2 < logs .68 < logs 1.<br />

By the change-of-base theorem,<br />

_ log.68 _ In.68<br />

Iogs'68-----­<br />

log5 In5<br />

...-.239625573.<br />

This value is between -1 and 0, as predicted<br />

above.<br />

3<br />

17. log927 =­<br />

2<br />

9 312 =27<br />

18. log3.45 == .5378<br />

IO Ṣ378 == 3.45<br />

19. In 45 == 3.8067<br />

3 8067 e . == 45<br />

20. Letf(x) =log; x be the required function. Then<br />

f(81) =4<br />

loga 81 =4<br />

a 4 =81<br />

a 4 =3 4<br />

a=3.<br />

The base is 3.<br />

21. Letf(x) = aX be the required function. Then<br />

1<br />

f(-4) =16<br />

-4 1<br />

a =­<br />

16<br />

a-4 =2-4<br />

a=2.<br />

The base is 2.<br />

22. IOg3( ~; )<br />

=log3 mn - log3 5r<br />

Logarithm of a quotient<br />

=log-,m+ log3 n -log 3 5 -log 3 r<br />

Logarithm of a product<br />

23. logs(x2y4~m3p)<br />

=logs x 2 y4(m 3 p)I/S<br />

=logs x 2 + logs i + logs(m 3 p)IIS<br />

Logarithm of a product<br />

=2 logs x+ 4 logs y+ 5 1 (1ogs m 3 p)<br />

Logarithm of a power<br />

=210g sx + 4 logs y + 5 1 (1ogs m 3 + logs p)<br />

Logarithm of a product<br />

. I<br />

= 2log sx+4logs y+-(3Iog s m+ logs p)<br />

5<br />

Logarithm of a power<br />

24. log7 (7k +5r2 )<br />

Since this is the logarithm ofa sum, this<br />

expression cannot be simplified.<br />

25. log 45.6 =1.6590<br />

26. log .0411 = -1.3862<br />

27. In 470 =6.1527<br />

28. In 144,000 =11.8776<br />

29. To find log , 769, use the change-of-base theorem.<br />

In 769<br />

Iog3 769 =-­<br />

In3<br />

== 6.0486<br />

406


<strong>Trigonometry</strong> <strong>Solutions</strong> <strong>Manual</strong><br />

Chapter 9 Review Exercises 407<br />

35. ex+1 = 10<br />

30. IOg2l3(%)<br />

X<br />

Ine + 1 = In 10<br />

_Iogi<br />

(x + 1)Ine = In10<br />

-log!<br />

x+l =In 10<br />

-.20412<br />

x = InlO-1<br />

=<br />

-.17609<br />

=2.3026-1<br />

= 1.1S92<br />

= 1.3026<br />

gX =32<br />

1<br />

31. 36. 10g64 x = ­<br />

(2 3)x = 2 5 3<br />

64 1 3<br />

/ = x Change to exponential form<br />

2 3x = 2 5 4=:x<br />

3x=S<br />

S<br />

37. In(6x) -In(x + 1)= In 4<br />

x=­ 3<br />

In(~)=ln4<br />

x+l<br />

-3 8 Logarithm of a quotient<br />

32. x =­<br />

27<br />

~=4<br />

x+l<br />

(;f =(~f<br />

4x+4=6x<br />

1 2<br />

4=2x<br />

-=- 2=x<br />

x 3<br />

3<br />

x=-<br />

2<br />

38. logl6.JX + 1 =.!.<br />

4<br />

16 114 =.Jx+l<br />

33. 10 2x - 3 =17<br />

Take common logarithms of both sides.<br />

Change to exponential form<br />

log 102x-3 = log 17 (16 114)2 = (.Jx + 1)2<br />

(2x- 3) log 10 = log17<br />

Square both sides<br />

2x-3= log17<br />

16 1 / 2 =x+l<br />

2x= logl7+3<br />

4=x+l<br />

x = _lo...::g:...l_7_+..:..3<br />

3=x<br />

2 The proposed solution must be checked in the<br />

x =2.11S2<br />

original equation since both sides were squared.<br />

The proposed solution checks, so x = 3.<br />

34. 4 x + 3 =S2- x<br />

Take common logarithms of both sides .<br />

log4 x + 3 = logS2 -x<br />

(x + 3) log4 = (2 - x) log S<br />

xlog4+ 310g4 = 210gS -x log S<br />

x log4 + x log S = 2 log S - 310g4<br />

x(log4+ log S) = 210gS - 310g4<br />

x = 2 log S- 3 log 4<br />

log4+ log S<br />

x::: -.3138<br />

407


<strong>Trigonometry</strong> <strong>Solutions</strong> <strong>Manual</strong><br />

408 Chapter 9 Exponential and Logarithmic Equations<br />

1<br />

39. Inx+3ln2 = In~ 43. (a) 6.6= 10glOx<br />

1 0<br />

Inx+ In2 3 = In~ Logarithm of a power .!... =10 6 .<br />

x<br />

1 0<br />

In(x. 2 3 ) = In ~ Logarithm of a product 1<br />

x<br />

- ::: 4,000,000<br />

1<br />

2<br />

0<br />

In8x = In­ I::: 4,000,000./ 0<br />

x<br />

The magnitude was about 4,000,000/ 0 ,<br />

Thus,<br />

2<br />

8x=­ 1<br />

x<br />

(b) 6.5 =10glO­<br />

1 0<br />

8x 2 =2<br />

5<br />

2 I L =10 6 .<br />

x =- 1<br />

4<br />

0<br />

1<br />

I - ::: 3,200,000<br />

x=±-. 1<br />

2<br />

0<br />

I::: 3,200,000./ 0<br />

- -I IS . not m . the d ornaṁ 0<br />

fl n x an d I n-, 2 so It . IS .<br />

The magnitude was about 3,200,0001 2 x<br />

0 ,<br />

not a solution. So x = ~.<br />

(c) Consider the ratio of the magnitudes.<br />

4,000,000 10 = 40 = ~ = 1.25<br />

40. log x + log(x - 3) = I 3,200,000 1 0 32 4<br />

log x(x - 3) = 1 Logarithm of a product The earthquake with a measure of 6.6 was<br />

10 1 =x(x-3)<br />

about 1.25 times as great.<br />

lO=x 2 -3x 1<br />

44. (a) 8.3= loglo­<br />

O=x 2 -3x-1O 1 0<br />

0= (x - 5)(x + 2)<br />

L =10 8 .<br />

x-5=O or x+2=O 3<br />

1 0<br />

x =5 or x =-2<br />

1=10 8 . 3 -2 is not in the domain of log x or log (x - 3), so<br />

./ 0<br />

it is not a solution. So x = 5. 1 = 200,000,000/ 0<br />

The magnitude was about 2000,000,000/ 0 ,<br />

41. In[ln(e- x»)= In3<br />

In(-x) = In3 Ine- x =-x<br />

1<br />

(b) 7.1=loglo­<br />

by theorem on inverses<br />

1 0<br />

-x=3 L =10 7 .1<br />

x=-3<br />

1 0<br />

1=10 7 . 1 . / 0<br />

42. 1 =13,000,000/ 0<br />

S = a In( I + ~) for n<br />

The magnitude was about 13,000,000/ 0 ,<br />

;= In(I+~)<br />

(c) 200,000,000./0 ::: 15.38<br />

13,000,000· 1<br />

e Sla -I =!!..<br />

0<br />

a<br />

The 1906 earthquake had a magnitude more<br />

Sla<br />

a(e -I) =n<br />

than 15 times greater than the 1989<br />

earthquake.<br />

n=~eSla_l)<br />

6<br />

408


<strong>Trigonometry</strong> <strong>Solutions</strong> <strong>Manual</strong><br />

Chapter 9 Review Exercises 409<br />

45. For 89 decibels, we have<br />

I<br />

89 = 10 log­<br />

1 0<br />

I<br />

8.9=log-.<br />

1 0<br />

Change this equation to exponential form.<br />

.l: =10 8 . 9<br />

1 0<br />

1=10 8 . 9 / 0<br />

For 86 decibels, we have<br />

I<br />

86= 10log­<br />

1 0<br />

L =10"8.6<br />

1 0<br />

1=10 8 . 6 / 0<br />

,<br />

To compare these intensities, find their ratio .<br />

10 8 . 9 t 10 8 . 9<br />

---;~O~ =__ "" 2<br />

10 8 . 6 / 10 8 . 6<br />

0<br />

From this calculation, we see that 89 decibels is<br />

about twice as loud as 86 decibels, for a 100%<br />

increase.<br />

46. A = 8780, P = 3500, t= 10, m = 1<br />

A =~1+ ~)nn<br />

rvI )10(1)<br />

8780 = 35'\.1+f<br />

2.50857 =(1+ r)1O<br />

(2.50857)1110 =1+ r<br />

(2.50857)1110 -1 = r<br />

.09631 =r<br />

The annual interest rate, to the nearest tenth, is<br />

9.6%.<br />

47. Substitute P =48,000, A =58,344, r= .05, and<br />

m =2 into the formula.<br />

A=~I+ ~)'m<br />

58, 344 =48, ~1+ .~5 f)(2)<br />

58,344 = 48,000(1.025)21<br />

1.2155 =(1.025)21<br />

In1.2155 =In(1.025) 21<br />

In1.2155 = 2t In1.025<br />

In1.2155<br />

---=t<br />

21n 1.025<br />

4.0""t<br />

$48,000 will increase to $58,344 in about 4.0 yr.<br />

48. A =~1+ ~)'m<br />

First, substitute P = 10,000, r= .12, t =12, and<br />

m =1 into the formula.<br />

.12)12(1)<br />

A=10, I­<br />

m{<br />

I<br />

=10,000(1.12)12<br />

""10,000(3.8960)<br />

=38,959.76<br />

After the first 12 yr, there would be about<br />

$38,959.76 in the account. To finish off the<br />

21-year period, substitute P =38,959.76, r= .10,<br />

t =9, and m =2 into the original formula,<br />

A = 38, 959.7{1 + .~0)9(2)<br />

=38,959.76(1.05)18<br />

= 38,959.76(2.4066)<br />

""93,761.31<br />

At the end of the 21-year period, about<br />

$93,761.31 would be in the account.<br />

49. A =~1+ ~)tm<br />

First, substitute P =12,000, r =.05, t =8, and<br />

m =1 into the formula.<br />

A =12,m{I+ .~5)8(1)<br />

=12,000(1.05)8 .<br />

= 12,000(1.4775)<br />

= 17,729.47<br />

After the first 8 yr, there would be $17,729.47 in<br />

the account. To finish off the 14-year period,<br />

substitute P =17,729.47, r= .06, t = 6, and m = 1<br />

into the original formula,<br />

A = 17,729.47(1 + .~)6(I)<br />

= 17,729.47(1.06)6<br />

= 17,729.47(1.4185)<br />

= 25,149.59<br />

At the end of the 14-year period, $25,149.59<br />

would be in the account.<br />

50. A =Pen<br />

To find t, substitute a =2, P = 1, and r =.1.<br />

2 =I·e·11<br />

Take natural logarithms of both sides.<br />

In2 =In e o)1<br />

In2 =.It<br />

In2<br />

-=t<br />

.1<br />

6.93""t<br />

It would take about 6.9 yr.<br />

409


<strong>Trigonometry</strong> <strong>Solutions</strong> <strong>Manual</strong><br />

410 Chapter 9 Exponential and Logarithmic Equations<br />

51. f(x);;: 6.2(1O)-12(1.4)X<br />

Find x when f(x);;: 2000.<br />

2000;;: 6.2(10)-12(1.4)x<br />

322.6 x 10 12 ;;:(1.4)x<br />

log(322.6 x 10 12);;: x log 1.4<br />

99= x<br />

Software exports will double their 1997 value in<br />

1999.<br />

52. Let K be the total number of executives,<br />

managers, and administrators in communications.<br />

The number of black executives, managers, and<br />

administrators is .034K. If this number increases<br />

at a rate of 5% per year, we must determine when<br />

it will reach .12K. That is, we must determine<br />

when .034K(1.05)' ;;:.12K.<br />

Use logarithms to solve this equation for 1.<br />

(1.05)' ;;:-E..<br />

.034<br />

1In1.05 =In(-E..)<br />

.034<br />

1=In(1&) = 25.8<br />

In 1.05<br />

Since 1991+ 25.8;;: 2020.8, this will occur during<br />

the year 2020 .<br />

53. (a) Compute the natural logarithm of P for each<br />

value of P given in the table in the textbook.<br />

x<br />

InP<br />

0 6.921<br />

1000 6.801<br />

2000 6.678<br />

3000 6.553<br />

4000 6.425<br />

5000 6.293<br />

6000 6.157<br />

7000 6.019<br />

8000 5.878<br />

9000 5.730<br />

10,000 5.580<br />

Plot this data on a graphing calculator using<br />

the window [-100, 11,000] by [5, 7] with<br />

Xscl ;;: 1000 and Ysci ;;: 1.<br />

54.<br />

(b) P= Ce kx<br />

Take natural logarithms on both sides.<br />

InP;;: InCe kx<br />

In P= InC+lne kx<br />

InP;;:kx+lnC<br />

Thus, y = In P = ax + b where a ;;: k and<br />

b ;;: In C, which is a linear function.<br />

(a) Plot the year on the x-axis and the number of<br />

processors on the y-axis. Let x =0<br />

correspond to the year 1971.<br />

10,000,000<br />

-1,000,000<br />

(b) The data are clearly not linear and do not<br />

level off like a logarithmic function. The data<br />

are increasing at a faster rate as x increases.<br />

Of the three choices, the exponential function<br />

(iii) will describe this data best .<br />

(c)<br />

Let x;;: 0 correspond to the year 1971. We<br />

can require that the exponential function pass<br />

through the points (0, 2300) and<br />

(26, 9,500,(00).<br />

We want to find a function of the form<br />

f(x);;: ae bx .<br />

First, find the value of a.<br />

f(O) =aeo =2300<br />

a;;: 2300<br />

Now find the value of b.<br />

f(26);;: 23OOe 26b ;;: 9,500,000<br />

26b 9,500,000<br />

e ;;:<br />

2300<br />

26b<br />

e = 4130<br />

26b =In 4130<br />

b == In4130<br />

26<br />

b = .3202<br />

410


<strong>Trigonometry</strong> <strong>Solutions</strong> <strong>Manual</strong><br />

Chapter 9 Test Exercises 411<br />

Thus, we obtain the-exponential function<br />

I(x) =2300e,3202x.<br />

(If a different pair of points from the table is<br />

used, a slightly different function will be<br />

found.)<br />

(d) We can predict the number of transistors on a<br />

chip in the year 2002 by evaluating<br />

1(31) =2300e·3202(31) :: 47,000,000.<br />

55. (a) A(t) =t 2 - t + 350<br />

~-----~-~--...<br />

Fort = x,A(x) = x 2 - x +350<br />

56. Graph Y 1 =x 2 and Y2 =2 x on the same screen.<br />

Since the range of both functions is (0, 00), there<br />

is no need to include Quadrants ill and N in the<br />

viewing window. A good choice for the window<br />

is [-5, 5] by [0, 20].<br />

The graph shows that the two curves intersect in<br />

three points. Two of these points are (2, 4) and<br />

(4, 16), as given in the exercise. To find the<br />

coordinates of the third point, use the "intersect"<br />

option in the CALC menu. The calculator<br />

displays the coordinates (-.7666647, .58777476).<br />

(a) Use the change-of-base theorem with base e<br />

to write the function as<br />

2<br />

I(x) = In(2x - x)<br />

In4<br />

o<br />

(b) A(t) =350 ]og(t +1)<br />

--....;;;..------.,<br />

(b) Graph the function.<br />

In(Zx2 - x)<br />

1B4<br />

(e)<br />

(d)<br />

0ll:========­<br />

Fort = x,A(x) = l00(.95)~<br />

(e) From the graph, the x-intercepts are .<br />

_.!.. and 1.<br />

2<br />

(d) From the graph, the vertical asymptotes are<br />

1<br />

x=Oandx=-.<br />

2<br />

(e) To make a y-lntercept, x = 0 must be in the<br />

domain, which is not the case here.<br />

Chapter 9 Test Exercises<br />

1. (a) I and g are inverse functions.<br />

(b) The graph of g is the reflection of the graph<br />

ofI across the line y = x.<br />

(e) (9.2)<br />

o<br />

o<br />

Function (c) best describes A(t).<br />

411


<strong>Trigonometry</strong> <strong>Solutions</strong> <strong>Manual</strong><br />

412 Chapter 9 Exponential and Logarithmic Equations<br />

2. (a) y = logl/3 x<br />

The point (1, 0) is on the graph of every<br />

function of the form y = loga x, so the<br />

correct choice must be either B or C.<br />

(b)<br />

(c)<br />

(d)<br />

i<br />

Since the base is a = 1. and<br />

3<br />

O l,y = eX is an<br />

increasing function, and so the correct choice<br />

must be A.<br />

Y = In x or y = loge x<br />

The point (1, 0) is on the graph of every<br />

function of the form y = log, x, so the<br />

correct choice must be B or C.<br />

Since the base is a = e and e> l,y = In x is<br />

an increasing function, and the correct choice<br />

must be C.<br />

y=Gr<br />

The point (0, 1) is on the graph since<br />

(~r = 1, so the correct choice must be<br />

either A or D.<br />

Since the base is.!.. and 0 < 1. < 1, y = (1.r<br />

3 3 3<br />

is a decreasing function, and so the correct<br />

choice must be D.<br />

( )2x-3 1<br />

3. = 16 x +<br />

(2-3)2x-3 = (24 ) x+ 1<br />

2-3(2x-3) = 24(x+1)<br />

2- 6x+9 = 2 4x+4<br />

-6x+9=4x+4<br />

-6x-4x=4-9<br />

-lOx =-5<br />

1<br />

x=­ 2<br />

4. (a) 4 3 / 2 = 8 is written in logarithmic form as<br />

3<br />

log4 8 =­<br />

2<br />

(b) logg 4 = ~ is written in exponential form as<br />

3<br />

8 2/3 = 4<br />

S. f(X)=(~r andg(x)=logll2 x<br />

y<br />

A:<br />

g(x) =108 11<br />

2 X<br />

They are inverses of each other.<br />

6. [x2~)<br />

log7~<br />

=log7x 2 +log7 ~ y-Iog 7z<br />

3<br />

1<br />

= 210g7x+-log7 y-310g 7 z<br />

4<br />

7. log 237.4 = 2.3755<br />

8. In.0467 = -3.0640<br />

In13<br />

9. log913 = --= 1.1674<br />

In9 ­<br />

10. log(2.49 x 10- 3) = -2.6038<br />

11. log; 25 =2<br />

x 2 =25<br />

x 2 =5 2<br />

x=5<br />

Solution set: {5}<br />

12. log, 32 =x<br />

4 x =32<br />

2 2x =2 5<br />

2x=5<br />

--­<br />

5<br />

x=­ 2<br />

13. log2 x + log2(x + 2) = 3<br />

log2 x(x+2) = 3<br />

2 3 = x(x+2)<br />

8 = x 2 +2x<br />

0=x 2 +2x-8<br />

0= (x+4)(x-2)<br />

412


<strong>Trigonometry</strong> <strong>Solutions</strong> <strong>Manual</strong><br />

Chapter 9 Test Exercises 413<br />

x+4=0<br />

or x-2=0<br />

x=-4 or x=2<br />

x =- 4 is not in the domain of log2 x or<br />

log2(x + 2), so it is not a solution. So x =2.<br />

14. SX+l =7 x<br />

InS x + l =In 7 x<br />

(x + 1)InS =x In 7<br />

x InS + InS =x In7<br />

InS = xln7 -xlnS<br />

InS =x(ln 7 -InS)<br />

InS<br />

---=x<br />

In7 -InS<br />

4.7833 = x<br />

15. In x - 41n 3 =In(';)<br />

In(; ) =In(';)<br />

x 5<br />

-=­<br />

81 x<br />

x 2 =4OS<br />

x =± 20. 1246<br />

Since the domain of In x is (0, 00),only the<br />

positive square root of 40S can be a solution.<br />

So x = 20.1246.<br />

16. logs 27 is the exponent to which S must be raised<br />

in order to obtain 27.<br />

To approximate log5 27 on your calculator use the<br />

change-of-base formula;<br />

log 27<br />

Iog5 27 =-­<br />

logS<br />

1.43136<br />

= .69897<br />

=2.048.<br />

17. v(1)=176(I-e-.J8t)<br />

Find the time 1 at which v(1)=147.<br />

147 =176(1-e-.J8t)<br />

147 =1_e-.J8r<br />

176<br />

-.J8r 176 147<br />

e =--­<br />

176 176<br />

-.J8t _ 29<br />

e -­<br />

176<br />

Ine-·18t = In 29<br />

176<br />

- .lSt = In.I6477<br />

In.I6477<br />

1=--­<br />

-.18<br />

= -1.8 = 10<br />

-.18<br />

It will take the skydiver about 10 sec to attain the<br />

speed of 147 ft per sec (100 mph).<br />

18. (8) A=~I+ ~)hn<br />

(b)<br />

18,000 =s


<strong>Trigonometry</strong> <strong>Solutions</strong> <strong>Manual</strong><br />

414 Chapter9 Exponential and Logarithmic Equations<br />

20. Let x =0 correspond to the year 2000. The<br />

population ofNew York (in millions) can be<br />

approximated by y =18. 15e·OO21x •<br />

The population ofFIorida (in millions) can be<br />

approximated by y =15.23e·0131x.<br />

Graph the two functions on the same screen and<br />

use the "intersect" option to find the coordinates<br />

of the intersection.<br />

30<br />

X"" 15.9 corresponds to the end ofthe year 2015.<br />

Near the end of2015 the populations will be<br />

equal.<br />

414

Hooray! Your file is uploaded and ready to be published.

Saved successfully!

Ooh no, something went wrong!